Вы находитесь на странице: 1из 1900

1

For something in mind to be, a duty the purity to see shall be

МАТЕМАТИКА С НУЛЯ
(xyz)

Бишкек, 2021
2

Числа

Число – это некоторая сущность, обозначающая количество


каких-нибудь предметов. Примеры: два яблока, пять ложек,
десять книг, сто рублей, семь тюльпанов. Каждый день мы
обращаемся к числам порой даже не замечая этого.

Первое время, когда люди обучались грамоте и расчётам,


число предметов изображали с помощью палочек:

Один предмет изображали как |

Два предмета как | |

Три предмета как | | |

Четыре предмета как | | | |

Пять предметов как | | | | |

Когда люди стали более грамотными, они поняли, что


большое число предметов палочками не изобразить, и
заменили эти палочки цифрами.

Сегодня в математике числа обозначаются с помощью цифр.


Это цифры 0, 1, 2, 3, 4, 5, 6, 7, 8, 9. Цифры отпугивают
большинство школьников и студентов, поскольку
ассоциируются с математикой и «страшными формулами».
На самом деле ничего страшного нет. Цифры это просто
набор символов, которые предназначены для обозначения
чисел. Проще говоря, для обозначения количества
предметов.
3

На этом всё. Вводный урок по числам завершён. В будущем


мы изучим его намного лучше, а пока рассмотренного в
данном уроке будет достаточно.
4

Основные операции

Основные операции, которые используются в математике это


сложение, вычитание, умножение и деление. Помимо этих
операций существуют ещё и операции отношения, такие как
равно (=), больше (>), меньше (<), больше или равно (≥),
меньше или равно (≤), не равно (≠).

Вообще, операции можно разделить на два вида:

1. операции действия;
2. операции отношения.

Операции действия это:

 сложение (+)
 вычитание (-)
 умножение (×)
 деление ( ÷ ).

Операции отношения это:

 равно (=)
 больше (>)
 меньше (<)
 больше или равно (≥)
 меньше или равно (≤)
 не равно (≠).

Содержание урока

 Операции отношения
 Операция сложения
5

 Операция вычитания
 Операция умножения
 Операция деления
 Задания для самостоятельного решения

Операции отношения

Начнем с операций отношения. Слово «отношение» говорит


само за себя. Примеры из жизни: что-то имеет отношение к
чему-то. Папа имеет отношение к маме. Это отношение

называют браком:

Примеров отношений множество. Можно сказать, что наш


красивый мир, который развивается гармонично, тоже
состоит из отношений.

Если пятёрка больше тройки, то мы говорим, что «пятерка


больше по отношению к тройке» и записываем как 5 > 3
(читается: пять больше, чем три). Острый угол знака
отношения должен быть направлен в сторону меньшего
числá. В нашем примере число 3 было меньше, чем число 5,
поэтому острый угол знака отношения был направлен в
сторону числа 3.
6

Ещё пример. Число 11 меньше, чем число 15. Эту фразу


можно записать так:

11 < 15

В математике с помощью отношений можно записывать


законы, формулы, уравнения и функции. Можно записать,
что одно выражение равно другому, либо какое-то действие
недопустимо по отношению к какому-нибудь объекту, числу,
закону.

Например, знаменитая фраза «на ноль делить нельзя»


записывается следующим образом:

Не будем опережать события и забегать вперёд. Просто


скажем, что в этом выражении вместо a и b могут стоять
любые числа. Но потом говорится, что b не должно быть
равным нулю.

Знак равенства = стáвится между величинами и говорит о


том, что эти величины равны между собой.

Например, «пять равно пять» записывается как 5 = 5.


Понятно, что две пятерки равны между собой. Помимо
простых чисел, знаком равенства могут соединяться более
сложные выражения, например: 9 + x + y = 4 + 5 + x + y.

Ещё пример: если один большой арбуз весит 20 кг, а два


маленьких арбуза весят по 10 кг каждый, то между арбузом в
20 кг и двумя арбузами по 10 кг можно поставить знак
равенства. Это отношение можно прочитать так: «один арбуз
7

весом в 20 килограмм равен весу двух арбузов, каждый из


которых весит 10 кг». Ведь 20 кг = 10 кг + 10 кг.

Знак не равно ≠ ставится между величинами тогда, когда они


не равны между собой.

Например, 5 ≠ 7. Ясно, что пятёрка не равна семёрке. Ещё


примеры: отличник не равен двоечнику, собака не равна
кошке, мандарин это не апельсин:

отличник  ≠  двоечник

собака  ≠  кошка

мандарин  ≠  апельсин

Вы можете осмотреться вокруг себя и найти множество


примеров отношений, которые можно истолковать с точки
зрения математики.

Операция сложения

Операция сложения обозначается знаком «плюс» (+) и


используется, когда складывают числа.
8

Числа, которые складывают называются слагаемыми.


Число, которое получается в результате их сложения,
называется суммой.

Например, сложим числа 3 и 2.

Записываем 3 + 2 = 5

В этом примере 3 − это слагаемое, 2 − второе слагаемое, 5 −


сумма.

В будущем придётся складывать довольно большие числа.


Но сложение этих больших чисел в конечном итоге будет
сводиться к тому, чтобы сложить маленькие.

Поэтому нужно научиться складывать маленькие числа в


диапазоне от 0 до 9. Например:

2+2=4

3+4=7

7+2=9

0+7=7

Можете потренироваться, записав в тетради несколько


простых примеров. Поверьте, ничего в этом постыдного нет.

Операция вычитания

Операция вычитания обозначается знаком «минус» (−) и


используется тогда, когда из одного числа вычитают другое.
9

Число, из которого вычитают другое число, называется


уменьшаемым. Число, которое вычитают из уменьшаемого
числа, называется вычитаемым. Число, которое получается
в результате, называется разностью.

Например, вычтем из числа 10 число 2.

10 − 2 = 8

В этом примере число 10 − это уменьшаемое, число 2 −


вычитаемое, а число 8 − разность.

Операция умножения

Обозначается знаком умножения (×) и используется, когда


одно число умножается на другое. Слово умножение говорит
само за себя — какое-то число увеличивается в
определенное количество раз, то есть множится.

Например, запись 4 × 3 означает, что четверка в ходе


операции умножения будет увеличена в три раза.

Число, которое увеличивают, называется множимым. Число,


которое показывает во сколько раз нужно увеличить
множимое, называется множителем. Число, которое
получается в результате называется произведением.

Например, умножим число 4 на 3.

4 × 3 = 12

В этом примере 4 − это множимое, 3 − множитель, 12 −


произведение.
10

Запись 4 × 3 можно понимать как «повторить число 4 три


раза». Например, если у нас имеются четыре конфеты и мы
повторим их три раза, то полýчится двенадцать конфет:

Другими словами, умножение 4 на 3 можно представить как


сумму трёх четвёрок. Схематически это выглядит
следующим образом:

Умножение можно понимать и другим образом, а именно как


взятие чего-то определенное количество раз. Допустим, в
вазе лежат конфеты. Возьмём четыре конфеты один раз:

4 конф. × 1 = 4 конф.

У нас в руках окажется четыре конфеты.

Попробуем взять четыре конфеты 2 раза:

4 конф × 2 = 8 конф.

У нас в руках окажется восемь конфет.

Попробуем взять четыре конфеты ноль раз, то есть ни разу:


11

4 × 0 = 0

У нас на руках не окажется конфет, поскольку мы ни разу их


не взяли. Поэтому умножение любого числа на ноль даёт в
ответе ноль.

В некоторых книгах множимое и множитель называют одним


общим словом — сомножители. Например, в записи 4 × 3
множимым является 4, а множителем 3, но эти два числа
ещё можно назвать сомножителями. Ошибкой это не будет.

В будущем мы будем умножать довольно большие числа. Но


умножение больших чисел свóдится к тому, чтобы умножить
маленькие. Поэтому сначала нужно научиться умножать
маленькие числа. Благо, они уже перемножены и записаны в
специальную таблицу, которую называют таблицей
умножения. Если вы живёте в России или в странах бывшего
СССР, то наверняка знаете эту таблицу наизусть. Если не
знаете, обязательно выучите!

Операция деления

Обозначается знаком деления (÷ или : ) и используется


тогда, когда делят числа.

Число, которое делят называют делимым. Число, которое


указывает на сколько частей делят делимое, называется
делителем. Число, которое получается в результате,
называется частным.

Например, разделим число 10 на 2.

10 : 2 = 5
12

В этом примере число 10 − это делимое, число 2 − делитель,


число 5 − частное.

Если у нас имеются десять конфет и мы разделим их на две


равные части, то в каждой части полýчится по пять конфет:

Так можно понять смысл записи 10 : 2 = 5.


13

Выражения

Выражение — это любое сочетание чисел, букв и знаков


операций. Можно сказать, что вся математика состоит из
выражений.

Выражения бывают двух видов: числовые и буквенные.

Числовые выражения состоят из чисел и знаков


математических операций. Например, следующие
выражения являются числовыми:

Буквенные выражения помимо чисел и знаков операций


содержат ещё и буквы. Например, следующие выражения
являются буквенными:

Буквы, которые содержатся в буквенных выражениях,


называются переменными. Запомните это раз и навсегда!
Спросите любого школьника что такое переменная — этот
вопрос поставит его в ступор, несмотря на то что он будет
решать сложные задачи по математике, не зная что это
такое. А между тем, переменная это фундаментальное
понятие, без понимания которого математику невозможно
изучать.
14

Под словом «изучать» мы подразумеваем самостоятельное


чтение соответствующей литературы  и способность
понимать, что там написано. А то вроде и знаешь математику
на четвёрку, задачи решаешь, но не можешь понять, что
написано в лекциях и книгах. Каждому знакомо такое чувство,
особенно студентам.

Поскольку понятие переменной очень важно, остановимся на


нём подробнее. Посмотрите внимательно на слово
«переменная». Ничего не напоминает? Слово «переменная»
происходит от слов «меняться», «изменить», «изменить своё
значение». Переменная в математике всегда выражена
какой-то буквой. Например, запишем следующее выражение:

a+5

Это буквенное выражение. Здесь одна переменная a.


Поскольку она является переменной, значит может изменить
свое значение в любой момент времени. Изменить значение
может любой: вы, учитель, ваш товарищ, кто угодно.
Например, давайте изменим значение этой переменной.
Присвоим ей значение 5. Для этого запишем саму
переменную, затем поставим знак равенства и запишем 5

a = 5 

Что случится в результате этого? Значение переменной a, то


есть 5 отправится в главное выражение a + 5, и подставится
вместо a.
15

Значение переменной a
подставляется в исходное выражение.

В результате имеем: 5 + 5 = 10

Конечно, мы рассмотрели простейшее выражение. На


практике встречаются более сложные выражения, в которых
присутствуют дроби, степени, корни и скобки. Выглядит это
устрашающе. На самом деле ничего страшного. Главное
понять сам принцип.

В учебниках часто встречаются задания следующего


содержания: найдите значение выражения x + 10, при x = 5.
Такие задания как раз и требуют, чтобы вместо переменной
подставили её значение. Давайте выполним это задание.
Значение переменной x равно 5. Подставляем эту пятёрку в
исходное выражение x + 10 и получаем 5 + 10 = 15.

Значение переменной x
подставляется в выражение x + 10

Переменная это своего рода контейнер, где хранится


значение. Переменные удобны тем, что они позволяют, не
приводя примеров доказывать теоремы, записывать
различные формулы и законы.

Вспомните второй урок «Основные операции». Чтобы понять,


что такое сложение, мы привели пример 5 + 2 = 7, и сказали,
16

что числа 5 и 2 являются слагаемыми, а число 7 — суммой.


Но мы могли бы понять эту тему и без примера, если бы
воспользовались буквенным выражением. Обозначили бы
слагаемые любыми буквами, например a и b, а сумму
обозначили бы как с. Тогда у нас получилось бы выражение с
тремя переменными a + b = c, и мы бы сказали, что a и b —
это слагаемые, c — сумма.

И вот, имея выражение a + b = c, можно пользоваться им,


подставляя вместо переменных a и b любые числа. А
переменная c будет получать своё значение автоматически,
в зависимости от того, какие числа мы подставим вместо a и
b

В качестве практики можете выполнить следующее задание.


Дано выражение a + b = c. Найдите его значение, если
a = 10, b = 6. Переменная c получит своё значение
автоматически. Ответ запишите следующим образом: при
a = 10 и b = 6, переменная c равна такому-то числу.

Решение:

a+b=c

10 + 6 = 16

Ответ: при a = 10 и b = 6, переменная c равна 16.

Значение выражения

Фраза «выполнить действие» означает выполнить одну из


операций действия. В учебниках младших классов часто
можно встретить задания следующего содержания:
17

выполнить действия, и далее перечисляются примеры,


которые нужно решить. Когда перед вами подобное задание,
вы сразу должны понимать, что от вас требуют решить
пример. В народе это звучит как «решить пример«, но если
быть более  грамотным, то надо говорить «найти значение
выражения». Решить пример и найти значение выражения
это фактически одно и то же.

Например, дано выражение 10 + 6, и от нас требуют найти


значение этого выражения. Это означает, что нам нужно
решить данный пример. Поставить знак равенства = и
записать ответ:

10 + 6 = 16

Сумма 16, которая получилась в результате и называется


значением выражения 10 + 6.

Значение выражения — это результат выполнения


действий, содержащихся в выражении.

Рассмотрим еще примеры:

 16 это значение выражения 4 × 4, поскольку 4 × 4 = 16


 20 это значение выражения 10 + 10, поскольку 10 + 10 =
20
 5 это значение выражения 10 ÷ 2, поскольку 10 ÷ 2 = 5
18

Замены в выражениях

Любое число в выражении может быть заменено таким же


числом, но записанным в другой форме. Возьмём для
примера следующее выражение, которое уже вычислено:

15 + 3 = 18

Давайте заменим число 15 на само себя, но запишем его в


другом виде:

(10 + 5) + 3 = 18

Видно, что мы заменили число 15 на выражение в скобках


(10 + 5). Но главное выражение 15 + 3 = 18 не пострадало от
этого, потому что 15 и (10 + 5) это одно и то же. Ведь
10 + 5 = 15.

Давайте заменим число 18 на само себя, но запишем его в


другом виде:

(10 + 5) + 3 = 3 × 6

Теперь заменим последнюю шестёрку на неё же саму, но


опять же запишем её в другом виде:

(10 + 5) + 3 = 3 × 2 × 3

Теперь сравним два выражения: первое, которое у нас было


и новое, которое мы видоизменили:

15 + 3 = 18

(10 + 5) + 3 = 3 × 2 × 3
19

На первый взгляд покажется, что это два разных выражения.


И так подумает любой, кто увидит эти два выражения в
первый раз. Но мы знаем, что это одно и то же выражение.
Вся разница в том, что мы видоизменили некоторые его
параметры.

Изменять внешний вид этого выражения можно хоть до


бесконечности. Главное, чтобы не нарушалось равенство.
Значок равенства (=) должен оправдывать своё положение.
Помните второй урок? Знак равенства ставится между
числами или выражениями только тогда, когда они равны
между собой.

Подобные операции, где одно число или выражение


заменяется на само себя, но записанное в другом виде,
называют преобразованием или представлением.

Содержание урока

 Представление в виде суммы


 Представление в виде разности
 Представление в виде произведения
 Представление в виде частного

Представление в виде суммы

Любое число или выражение можно представить в виде


суммы. Например, число 10 можно представить в виде
суммы 5+5 или 7+3 или 8+2. Как угодно, лишь бы
соблюдалось равенство между числом и представленной
суммой. Выглядеть это может следующим образом:

10 = 5 + 5
20

10 = 7 + 3

10 = 8 + 2

10 = 6 + 4

В книгах можно встретить задания следующего содержания:


представьте в виде суммы и далее приводятся числа или
выражения, которые нужно представить в виде суммы. Это
как раз тот случай, когда надо включать свои творческие
способности и решить какие числа (или выражения)
использовать, чтобы выполнить задание.

Представление в виде разности

С прошлых уроков известно, что разность это результат,


который получается в результате вычитания одного числа из
другого. Но разностью также называется выражение,
которое соединено знаком вычитания (−). Например
следующие выражения являются разностями:

15 – 5

10 – 6

20 – 10

Любое число можно представить в виде разности. Например,


число 50 можно представить в виде разности 90−40 или
80−30 или 60−10. Как угодно, лишь бы соблюдалось
равенство между числом 50 и представленной разностью.
Выглядеть это может следующим образом:

50 = 90 − 40
21

50 = 80 − 30

50 = 60 − 10

Представление в виде произведения

С прошлых уроков известно, что произведение это


результат, который получается в результате умножения
одного числа на другое. Но произведением также называется
выражение, которое соединено знаком умножения (×).
Например следующие выражения являются произведениями:

3×2

15 × 2

12 × 3

Любое число можно представить в виде произведения.


Например, число 30 можно представить в виде произведения
5×6 или 10×3 или 15×2. Как угодно, лишь бы соблюдалось
равенство между числом 30 и представленным
произведением. Выглядеть это может следующим образом:

30 = 5 × 6

30 = 10 × 3

30 = 15 × 2

Представление в виде частного

С прошлых уроков известно, что частное это результат,


который получается в результате деления одного числа на
22

другое. Но частным также называется выражение, которое


соединено знаком деления (÷). Например, следующие
выражения являются частными:

15 ÷ 5

30 ÷ 6

12 ÷ 4

Любое число можно представить в виде частного. Например,


число 5 можно представить в виде частного 15÷3 или 25÷5
или 30÷6. Как угодно, лишь бы соблюдалось равенство
между числом 5 и представленным частным. Выглядеть это
может следующим образом:

5 = 15 ÷ 3

5 = 25 ÷ 5

5 = 30 ÷ 6

На этом данный урок завершён. Для закрепления материала,


попробуйте выполнить следующие задания:

Задание 1. Представьте в виде суммы следующие числа: 20,


30, 45, 50. Можете представить любыми числами. Например,
первое число 20 можно представить как 15 + 5.

Задание 2. Представьте в виде разности следующие числа:


10, 15, 12, 5 Можете представить любыми числами.
Например, первое число можно представить как 15 − 5.

Задание 3. Представьте в виде произведения следующие


числа: 30, 40, 72.
23

Задание 4. Представьте в виде частного следующие числа:


7, 5, 9, 3
24

Разряды для начинающих

Наш первый урок назывался числа. Мы рассмотрели лишь


малую часть этой темы. На самом деле тема чисел
достаточно обширна. В ней много тонкостей и нюансов,
много хитростей и интересных фишек.

Сегодня мы продолжим тему чисел, но опять же не будем


рассматривать её всю, чтобы не затруднять обучение
лишней информацией, которая на первых порах не особо то
и нужна. Мы поговорим о разрядах.

Содержание урока

 Что такое разряд?


 Группировка предметов
 Где применить разряды?
 Переполнение разряда
 Сложение в столбик
 Вычитание в столбик
 Задания для самостоятельного решения

Что такое разряд?

Если говорить простым языком, то разряд это позиция


цифры в числе или место, где располагается цифра.
Возьмём для примера число 635. Это число состоит из трёх
цифр: 6, 3 и 5.

Разряды надо читать справа налево. В числе 635 на первой


позиции располагается цифра 5, на второй позиции – цифра
3, на третьей позиции – цифра 6.
25

Позиция, где располагается цифра 5, называется разрядом


единиц

Позиция, где располагается цифра 3, называется разрядом


десятков

Позиция, где располагается цифра 6, называется разрядом


сотен

Каждый из нас слышал со школы такие вещи как «единицы»,


«десятки», «сотни». Разряды помимо того, что играют роль
позиции цифры в числе, сообщают нам некоторую
информацию о самом числе. В частности, разряды сообщают
нам вес числа. Они сообщают сколько в числе единиц,
сколько десятков и сколько сотен.

Вернёмся к нашему числу 635. В разряде единиц


располагается пятёрка. О чём это говорит? А говорит это о
том, что разряд единиц содержит пять единичек. Выглядит
это так:
26

В разряде десятков располагается тройка. Это говорит о том,


что разряд десятков содержит три десятка. Выглядит это так:

В разряде сотен располагается шестёрка. Это говорит о том,


что в разряде сотен располагаются шесть сотен. Выглядит
это так:

Если сложить число получившихся единиц, число десятков и


число сотен, то получим наше изначальное число 635

Существуют и более старшие разряды такие как разряд


тысяч, разряд десятков тысяч, разряд сотен тысяч,  разряд
миллионов и так далее. Такие большие числа мы будем
рассматривать редко, но тем не менее о них тоже
желательно знать.

Например, в числе 1 645 832 разряд единиц содержит 2


единицы, разряд десятков — 3 десятка, разряд сотен — 8
сотен, разряд тысяч — 5 тысяч, разряд десятков тысяч — 4
десятка тысяч, разряд сотен тысяч — 6 сотен тысяч, разряд
миллионов — 1 миллион.
27

На первых этапах изучения разрядов желательно


разбираться сколько единиц, десятков, сотен содержит то
или иное число. К примеру, число 9 содержит 9 единиц.
Число 12 содержит две единицы и один десяток. Число 123
содержит три единицы, два десятка и одну сотню.

Группировка предметов

После подсчета каких-нибудь предметов, разряды можно


использовать для группировки этих предметов. К примеру,
если мы насчитали во дворе 35 кирпичей, то можно
использовать разряды для группировки этих кирпичей. В
случае группировки предметов, разряды можно читать слева
направо. Так, цифра 3 в числе 35 будет говорить о том, что в
числе 35 содержатся три десятка. А это значит, что 35
кирпичей можно сгруппировать три раза по десять штук.

Итак, сгруппируем кирпичи три раза по десять штук:


28

Получилось тридцать кирпичей. Но осталось еще пять


единиц кирпичей. Их мы назовем как «пять единиц»

Получилось три десятка и пять единиц кирпичей.

А если бы мы не стали группировать кирпичи на десятки и


единицы, то можно было бы сказать, что число 35 содержит
тридцать пять единиц. Такая группировка тоже была бы
допустимой:
29

Аналогично можно рассуждать и про другие числа. К


примеру, о числе 123. Ранее мы сказали, что это число
содержит три единицы, два десятка и одну сотню. Но можно
ещё сказать, что это число  содержит 123 единицы. Более
того, можно сгруппировать это число и другим образом,
сказав что оно содержит 12 десятков и 3 единицы.

Слова единицы, десятки, сотни, заменяют собой множимые


1, 10 и 100. К примеру, в разряде единиц числа 123
располагается цифра 3. С помощью множимого 1 можно
записать, что эта единица содержится в разряде единиц три
раза:

1 × 3 = 3

Далее в разряде десятков числа 123 располагается цифра 2.


С помощью множимого 10 можно записать, что эта десятка
содержится в разряде десятков два раза:

10 × 2 = 20
30

Далее в разряде сотен числа 123 располагается цифра 1. С


помощью множимого 100 можно записать, что эта сотня
содержится в разряде сотен один раз:

100 × 1 = 100

Если сложить полученные результаты 3, 20 и 100, то получим


число 123

3 + 20 + 100 = 123

То же самое будет происходить если мы скажем, что число


123 содержит 12 десятков и 3 единицы. Другими словами,
десятки будут сгруппированы 12 раз:

10 × 12 = 120

А единицы три раза:

1 × 3 = 3

Это можно понять на следующем примере. Если имеется 123


яблока, то можно сгруппировать первые 120 яблок 12 раз по
10 штук:
31

Получилось сто двадцать яблок. Но осталось еще три


яблока. Их мы назовем как «три единицы»
32

Если сложить полученные результаты 120 и 3, снова


получим число 123

120 + 3 = 123

Ещё можно сгруппировать 123 яблока на одну сотню, два


десятка и три единицы.

Сгруппируем сотню:

Сгруппируем два десятка:


33

Сгруппируем три единицы:

Если сложить полученные результаты 100, 20 и 3, снова


получим число 123

100 + 20 + 3 = 123

Ну и наконец, рассмотрим последнюю возможную


группировку, где яблоки не будут распределяться на десятки
и сотни, а будут собраны вместе. В таком случае число 123
будет читаться как «сто двадцать три единицы». Такая
группировка тоже будет допустимой:
34

1 × 123 = 123

Пример 3. Прочитать число 523 всеми возможными


способами.

Число 523 можно прочесть, как 3 единицы, 2 десятка и 5


сотен:
35

1 × 3 = 3 (три единицы)

10 × 2 = 20 (два десятка)

100 × 5 = 500 (пять сотен)

3 + 20 + 500 = 523

Ещё  можно прочесть, как 3 единицы 52 десятка:

1 × 3 = 3 (три единицы)

10 × 52 = 520 (пятьдесят два десятка)

3 + 520 = 523

Ещё число 523 можно прочесть, как 523 единицы:

1 × 523 = 523 (пятьсот двадцать три единицы)

Где применить разряды?

Разряды существенно облегчают некоторые вычисления.


Представьте, что вы у доски и решаете задачу. Вы почти
закончили задачу, осталось только вычислить последнее
выражение и получить ответ. Выражение, которое надо
вычислить, выглядит следующим образом:

Калькулятора под рукой нет, а хочется быстро записать ответ


и удивить всех скоростью своих вычислений. Всё просто,
если отдельно сложить единицы, отдельно десятки и
отдельно сотни. Начинать нужно с разряда единиц. В первую
очередь после знака равно (=) необходимо мысленно
36

поставить три точки. Вместо этих точек будет располагаться


новое число (наш ответ):

Теперь начинаем складывать. В разряде единиц числа 632


располагается цифра 2, а в разряде единиц числа 264 —
цифра 4. Это означает, разряд единиц числа 632 содержит
две единицы, а разряд единиц числа 264 содержит четыре
единицы. Складываем 2 и 4 единицы — получаем 6 единиц.
Записываем цифру 6 в разряде единиц нового числа (нашего
ответа):

Далее складываем десятки. В разряде десятков числа 632


располагается цифра 3, а в разряде десятков числа 264 —
цифра 6. Это означает, что разряд десятков числа 632
содержит три десятка, а разряд десятков числа 264 содержит
шесть десятков. Складываем 3 и 6 десятков — получаем 9
десятков. Записываем цифру 9 в разряде десятков нового
числа (нашего ответа):

Ну и в завершении складываем отдельно сотни. В разряде


сотен числа 632 располагается цифра 6, а в разряде сотен
числа 264 — цифра 2. Это означает, что разряд сотен числа
632 содержит шесть сотен, а разряд сотен числа 264
содержит две сотни. Складываем 6 и 2 сотни, получаем 8
сотен. Записываем цифру 8 в разряде сотен нового числа
(нашего ответа):
37

Таким образом, если к числу 632 прибавить 264, получается


896. Конечно, вы вычислите подобное выражение быстрее и
окружающие начнут удивляться вашим способностям. Они
будут думать, что вы быстро вычисляете большие числа, а
на самом деле вы вычисляли маленькие. Согласитесь, что
маленькие числа вычислять легче, чем большие.

Переполнение разряда

Разряд характеризуется одной цифрой от 0 до 9. Но иногда


при вычислении числового выражения в середине решения
может произойти переполнение разряда.

Например, при сложении чисел 32 и 14 переполнения не


происходит. Сложение единиц этих чисел даст 6 единиц в
новом числе. А сложение десятков этих чисел даст 4 десятка
в новом числе. Получится ответ 46 или шесть единиц и
четыре десятка.

А вот при сложении чисел 29 и 13 произойдёт переполнение.


Сложение единиц этих чисел даёт 12 единиц, а сложение
десятков 3 десятка. Если в новом числе в разряде единиц
записать полученные 12 единиц, а в разряде десятков
записать полученные 3 десятка, то получится ошибка:

Значение выражения 29 + 13 равно 42, а не 312. Как же


следует поступать при переполнении? В нашем случае
переполнение случилось в разряде единиц нового числа.
38

При сложении девяти и трёх единиц у нас получилось 12


единиц. А в разряд единиц можно записывать только цифры
в диапазоне от 0 до 9.

Дело в том, что 12 единиц это не просто «двенадцать


единиц». По другому это число можно прочитать как «две
единицы и один десяток». Разряд единиц предназначен
только для единиц. Десяткам там не место. Здесь и
заключается наша ошибка. Сложив 9 единиц и 3 единицы мы
получили 12 единиц, которые по-другому можно назвать
двумя единицами и одним десятком. Записав две единицы и
один десяток в одном разряде, мы допустили ошибку,
которая в итоге привела к неправильному ответу.

Чтобы исправить ситуацию, две единицы нужно записать в


разряде единиц нового числа, а оставшийся десяток
перенести на следующий разряд десятков. После сложения
десятков в примере 29 + 13, мы прибавим к полученному
результату тот десяток, который остался при сложении
единиц.

Итак, из 12 единиц две единицы запишем в разряде единиц


нового числа, а один десяток перенесем на следующий
разряд
39

Как видно на рисунке, 12 единиц мы представили как 1


десяток и 2 единицы. Две единицы мы записали в разряде
единиц нового числа. А один десяток перенесли к разрядам
десятков. Этот десяток мы прибавим к результату сложения
десятков чисел 29 и 13. Чтобы не забыть о нем, мы
надписали его над десятками числа 29.

Теперь складываем десятки. Два десятка плюс один десяток


будет три десятка, плюс один десяток, который остался от
предыдущего сложения. В результате в разряде десятков
получаем четыре десятка:

Пример 2. Сложить по разрядам числа 862 и 372.

Начинаем с разряда единиц. В разряде единиц числа 862


располагается цифра 2, в разряде единиц числа 372 — также
цифра 2. Это означает, что разряд единиц числа 862
содержит две единицы, и разряд единиц числа 372 также
40

содержит две единицы. Складываем 2 единицы плюс 2


единицы — получаем 4 единицы. Записываем цифру 4 в
разряде единиц нового числа:

Далее складываем десятки. В разряде десятков числа 862


располагается цифра 6, а в разряде десятков числа 372 —
число 7. Это означает, что разряд десятков числа 862
содержит шесть десятков, а разряд десятков числа 372
содержит семь десятков. Складываем 6 десятков и 7
десятков — получаем 13 десятков. Произошло переполнение
разряда. 13 десятков это десятка повторенная 13 раз. А если
повторить десятку 13 раз, то получится число 130

10 × 13 = 130

Число 130 состоит из трех десятков и одной сотни. Три


десятка мы запишем в разряде десятков нового числа, а одну
сотню отправим на следующий разряд:

Как видно на рисунке, 13 десятков (число 130) мы


представили как 1 сотню и 3 десятка. Три десятка мы
записали в разряде десятков нового числа. А одну сотню
перенесли к разрядам сотен. Эту сотню мы прибавим к
41

результату сложения сотен чисел 862 и 372. Чтобы не забыть


о ней, мы надписали её над сотнями числа 862.

Теперь складываем сотни. Восемь сотен плюс три сотни


будет одиннадцать сотен плюс одна сотня, которая осталась
от предыдущего сложения. В результате в разряде сотен
получаем двенадцать сотен:

Здесь также происходит переполнение разряда сотен, но это


не приводит к ошибке, поскольку решение завершено. При
желании с 12 сотнями можно провести те же действия, что
мы провели с 13 десятками.

12 сотен это сотня, повторенная 12 раз. А если повторить


сотню 12 раз, то получится 1200

100 × 12 = 1200

В числе 1200 две сотни и одна тысяча. Две сотни


записываются в разряд сотен нового числа, а одна тысяча
перенеслась к разряду тысяч.

Теперь рассмотрим примеры на вычитание. Для начала


вспомним, что такое вычитание. Это операция, которая
позволяет от одного числа вычесть другое. Вычитание
состоит из трёх параметров: уменьшаемого, вычитаемого и
разности. Вычитать тоже нужно по разрядам.

Пример 3. Вычесть из числа 65 число 12.

Начинаем с разряда единиц. В разряде единиц числа 65


располагается цифра 5, а в разряде единиц числа 12 —
42

цифра 2. Это означает, что разряд единиц числа 65 содержит


пять единиц, а разряд единиц числа 12 содержит две
единицы. Вычтем из пяти единиц две единицы, получим три
единицы. Записываем цифру 3 в разряде единиц нового
числа:

Теперь вычитаем десятки. В разряде десятков числа 65


располагается цифра 6, а в разряде десятков числа 12 —
цифра 1. Это означает, что разряд десятков числа 65
содержит шесть десятков, а разряд десятков числа 12
содержит один десяток. Вычтем из шести десятков один
десяток, получим пять десятков. Записываем цифру 5 в
разряде десятков нового числа:

Пример 4. Вычесть из числа 32 число 15

В разряде единиц числа 32 содержится две единицы, а в


разряде единиц числа 15 — пять единиц. От двух единиц не
вычесть пять единиц, поскольку две единицы меньше, чем
пять единиц.

Сгруппируем 32 яблока так, чтобы в первой группе было три


десятка яблок, а во второй — оставшиеся две единицы
яблок:
43

Итак, нам нужно из этих 32 яблок вычесть 15 яблок, то есть


вычесть пять единиц и один десяток яблок. Причем вычесть
по разрядам.

От двух единиц яблок нельзя вычесть пять единиц яблок.


Чтобы выполнить вычитание, две единицы должны взять
несколько яблок у соседней группы (разряда десятков). Но
нельзя брать сколько хочется, поскольку десятки строго
упорядочены по десять штук. Разряд десятков может дать
двум единицам только один целый десяток.

Итак, берём один десяток из разряда десятков и отдаём его


двум единицам:
44

К двум единицам яблок теперь присоединился один десяток


яблок. Получается 12 единиц яблок. А от двенадцати можно
вычесть пять, получится семь. Записываем цифру 7 в
разряде единиц нового числа:

Теперь вычитаем десятки. Поскольку разряд десятков отдал


единицам один десяток, сейчас он имеет не три, а два
десятка. Поэтому вычитаем из двух десятков один десяток.
Останется один десяток. Записываем цифру 1 в разряде
десятков нового числа:

Чтобы не забывать, что в каком-то разряде был взят один


десяток (либо сотня либо тысяча), над этим разрядом
принято ставить точку.

Пример 5. Вычесть из числа 653 число 286


45

В разряде единиц числа 653 содержится три единицы, а в


разряде единиц числа 286 — шесть единиц. От трёх единиц
не вычесть шесть единиц, поэтому берем один десяток у
разряда десятков. Ставим точку над разрядом десятков,
чтобы помнить о том, что мы взяли оттуда один десяток:

Взятый один десяток и три единицы вместе образуют


тринадцать единиц. От тринадцати единиц можно вычесть
шесть единиц, получится семь единиц. Записываем цифру 7
в разряде единиц нового числа:

Теперь вычитаем десятки. Раньше разряд десятков числа


653 содержал пять десятков, но мы взяли с него один
десяток, и теперь в разряде десятков содержатся четыре
десятка. Из четырех десятков не вычесть восемь десятков,
поэтому берем одну сотню у разряда сотен. Ставим точку
над разрядом сотен, чтобы помнить о том, что мы взяли
оттуда одну сотню:

Взятая одна сотня и четыре десятка вместе образуют


четырнадцать десятков. От четырнадцати десятков можно
вычесть восемь десятков, получится шесть десятков.
Записываем цифру 6 в разряде десятков нового числа:

Теперь вычитаем сотни. Раньше разряд сотен числа 653


содержал шесть сотен, но мы взяли с него одну сотню, и
46

теперь в разряде сотен содержатся пять сотен. Из пяти сотен


можно вычесть две сотни, получается три сотни. Записываем
цифру 3 в разряде сотен нового числа:

Намного сложнее вычитать из чисел вида 100, 200, 300,


1000, 10000. То есть числа, у которых на конце нули. Чтобы
выполнить вычитание, каждому разряду приходится
занимать десятки/сотни/ тысячи у следующего разряда.
Давайте посмотрим, как это происходит.

Пример 6. Вычесть из числа 200 число 84

В разряде единиц числа 200 содержится ноль единиц, а в


разряде единиц числа 84 — четыре единицы. От нуля не
вычесть четыре единицы, поэтому берем один десяток у
разряда десятков. Ставим точку над разрядом десятков,
чтобы помнить о том, что мы взяли оттуда один десяток:

Но в разряде десятков нет десятков, которые мы могли бы


взять, поскольку там тоже ноль. Чтобы разряд десятков смог
дать нам один десяток, мы должны взять для него одну
сотню у разряда сотен. Ставим точку над разрядом сотен,
чтобы помнить о том, что мы взяли оттуда одну сотню для
разряда десятков:

Взятая одна сотня это десять десятков. От этих десяти


десятков мы берём один десяток и отдаём его единицам.
Этот взятый один десяток и прежние ноль единиц вместе
47

образуют десять единиц. От десяти единиц можно вычесть


четыре единицы, получится шесть единиц. Записываем
цифру 6 в разряде единиц нового числа:

Теперь вычитаем десятки. Чтобы вычесть единицы мы


обратились к разряду десятков за одним десятком, но на тот
момент этот разряд был пуст. Чтобы разряд десятков смог
дать нам один десяток, мы взяли одну сотню у разряда
сотен. Эту одну сотню мы назвали «десять десятков». Один
десяток мы отдали единицам. Значит на данный момент в
разряде десятков содержатся не десять, а девять десятков.
От девяти десятков можно вычесть восемь десятков,
получится один десяток. Записываем цифру 1 в разряде
десятков нового числа:

Теперь вычитаем сотни. Для разряда десятков мы брали у


разряда сотен одну сотню. Значит сейчас в разряде сотен
содержатся не две сотни, а одна. Поскольку в вычитаемом
разряд сотен отсутствует, мы переносим эту одну сотню в
разряд сотен нового числа:

Получили окончательный ответ 116.

Естественно, выполнять вычитание таким традиционным


методом довольно сложно, особенно на первых порах. Поняв
сам принцип вычитания, можно воспользоваться
нестандартными способами.
48

Первый способ заключается в том, чтобы уменьшить число, у


которого на конце нули на одну единицу. Далее из
полученного результата вычесть вычитаемое и к полученной
разности прибавить единицу, которую изначально вычли из
уменьшаемого. Давайте решим предыдущий пример этим
способом:

Уменьшаемое здесь это число 200. Уменьшим это число на


единицу. Если от 200 вычесть 1 получится 199. Теперь в
примере 200 − 84 вместо числа 200 записываем число 199 и
решаем пример 199 − 84. А решение этого примера не
составляет особого труда. Единицы вычтем из единиц,
десятки из десятков, а сотню просто перенесем к новому
числу, поскольку в числе 84 нет сотен:

Получили ответ 115. Теперь к этому ответу прибавляем


единицу, которую мы изначально вычли из числа 200

Получили окончательный ответ 116.

Пример 7. Вычесть из числа 100000 число 91899

Вычтем из 100000 единицу, получим 99999

Теперь из 99999 вычитаем 91899


49

К полученному результату 8100 прибавим единицу, которую


мы вычли из 100000

Получили окончательный ответ 8101.

Второй способ вычитания заключается в том, чтобы


рассматривать цифру, находящуюся в разряде, как
самостоятельное число. Решим несколько примеров этим
способом.

Пример 8. Вычесть из числа 75 число 36

Будем считать, что каждая цифра в разряде это


самостоятельное число.

Итак, в разряде единиц числа 75 располагается число 5, а в


разряде единиц числа 36 располагается число 6. Из пяти не
вычесть шести, поэтому берем одну единицу у следующего
числа, находящегося в разряде десятков.

В разряде десятков располагается число 7. Берем от этого


числа одну единицу и мысленно дописываем её слева от
числа 5
50

А поскольку от числа 7 взята одна единица, это число


уменьшится на одну единицу и обратится в число 6

Теперь в разряде единиц числа 75 располагается число 15, а


в разряде единиц числа 36 число 6. Из 15 можно вычесть 6,
получится 9. Записываем число 9 в разряде единиц нового
числа:

Переходим к следующему числу, находящемуся в разряде


десятков. Раньше там располагалось число 7, но мы взяли с
этого числа одну единицу, поэтому сейчас там располагается
число 6. А в разряде десятков числа 36 располагается число
3. Из 6 можно вычесть 3, получится 3. Записываем число 3 в
разряде десятков нового числа:

Пример 9. Вычесть из числа 200 число 84

Будем считать, что каждая цифра в разряде это


самостоятельно число.

Итак, в разряде единиц числа 200 располагается ноль, а в


разряде единиц числа 84 — располагается четыре. От нуля
51

не вычесть четыре, поэтому берем одну единицу у


следующего числа, находящегося в разряде десятков. Но в
разряде десятков тоже ноль. Ноль не сможет дать нам
единицу. В таком случае за следующее принимаем число 20.

Берём одну единицу от числа 20 и мысленно дописываем её


слева от нуля, располагающегося в разряде единиц. А
поскольку от числа 20 взята одна единица, это число
обратится в число 19

Теперь в разряде единиц располагается число 10. Десять


минус четыре равно шесть. Записываем число 6 в разряде
единиц нового числа:

Переходим к следующему числу, находящемуся в разряде


десятков. Раньше там располагался ноль, но этот ноль
вместе со следующей цифрой 2 образовал число 20, от
которого мы брали одну единицу. В результате число 20
обратилось в число 19. Получается, что теперь в разряде
десятков числа 200 располагается число 9, а в разряде
десятков числа 84 располагается число 8. Девять минус
52

восемь равно одному. Записываем число 1 в разряде


десятков нашего ответа:

Переходим к следующему числу, находящемуся к разряду


сотен. Раньше там располагалось число 2, но это число
вместе с цифрой 0 мы приняли за число 20, от которого
взяли одну единицу. В результате число 20 обратилось в
число 19. Получается, что теперь в разряде сотен числа 200
располагается число 1, а в числе 84 разряд сотен пустой,
поэтому мы переносим эту единицу к новому числу:

Этот метод поначалу кажется сложным и лишенным всякого


смысла, но на деле он самый лёгкий. В основном мы будем
им пользоваться при сложении и вычитании чисел в столбик.

Сложение в столбик

Сложение в столбик это школьная операция, которую помнят


многие, но не мешает вспомнить её ещё раз. Сложение в
столбик происходит по разрядам — единицы складываются с
единицами, десятки с десятками, сотни с сотнями, тысячи с
тысячами.

Рассмотрим несколько примеров.

Пример 1. Сложить 61 и 23.

Сначала записываем первое число, а под ним второе число


так, чтобы единицы и десятки второго числа оказались под
53

единицами и десятками первого числа. Всё это соединяем


знаком сложения (+) по вертикали:

Теперь единицы первого числа складываем с единицами


второго числа, а десятки первого числа складываем с
десятками второго числа:

Получили 61 + 23 = 84.

Пример 2. Сложить 108 и 60

Записываем числа в столбик. Единицы под единицами,


десятки под десятками:

Теперь складываем единицы первого числа с единицами


второго числа, десятки первого числа с десятками второго
числа, сотни первого числа с сотнями второго числа. Но
сотня есть только у первого числа 108. В этом случае цифра
1 из разряда сотен добавляется к новому числу (нашему
ответу). Как говорили в школе «сносится»:
54

Видно, что мы снесли цифру 1 к нашему ответу.

Когда речь идёт о сложении, нет разницы в каком порядке


записывать числа. Наш пример вполне можно было записать
и так:

Первая запись, где число 108 было наверху, более удобнее


для вычисления. Человек вправе выбирать любую запись, но
обязательно нужно помнить, что единицы надо записывать
строго под единицами, десятки под десятками, сотни под
сотнями. Другими словами, следующие записи будут
неправильными:

Если вдруг при сложении соответствующих разрядов


получится число, которое не помещается в разряд нового
числа, то необходимо записать одну цифру из младшего
разряда, а оставшуюся перенести на следующий разряд.
55

Речь в данном случае идет о переполнении разряда, о


котором мы говорили ранее. Например, при сложении 26 и 98
получается 124. Давайте посмотрим, как это получилось.

Записываем числа в столбик. Единицы под единицами,


десятки под десятками:

Складываем единицы первого числа с единицами второго


числа: 6+8=14. Получили число 14, которое не вместится в
разряд единиц нашего ответа. В таких случаях мы сначала
вытаскиваем из 14 цифру, находящуюся в разряде единиц и
записываем её в разряде единиц нашего ответа. В разряде
единиц числа 14 располагается цифра 4. Записываем эту
цифру в разряде единиц нашего ответа:

А куда девать цифру 1 из числа 14? Здесь начинается


самое интересное. Эту единицу мы переносим на следующий
разряд. Она будет добавлена к разряду десятков нашего
ответа.

Складываем десятки с десятками. 2 плюс 9 равно 11, плюс


добавляем единицу, которая досталась нам от числа 14.
Добавив к 11 нашу единицу, мы получим число 12, которое и
запишем в разряде десятков нашего ответа. Поскольку это
конец решения, здесь уже не стоит вопрос о том, вместится
ли полученный ответ в разряд десятков. 12 мы записываем
целиком, образуя окончательный ответ.
56

Получили ответ 124.

Говоря традиционным методом сложения, при сложении 6 и


8 единиц получилось 14 единиц. 14 единиц это 4 единицы и 1
десяток. Четыре единицы мы записали в разряде единиц, а
один десяток отправили на следующий разряд (к разрядам
десятков). Затем сложив 2 десятка и 9 десятков, мы
получили 11 десятков, плюс добавили 1 десяток, который
остался при сложении единиц. В результате получили 12
десятков. Эти двенадцать десятков мы записали целиком,
образуя окончательный ответ 124.

Этот простенький пример демонстрирует школьную


ситуацию, в которой говорят «четыре пишем, один в уме».
Если вы будете решать примеры и у вас после сложения
разрядов останется цифра, которую надо держать в уме,
запишите её над тем разрядом, куда она будет потом
добавлена. Это позволит вам не забыть о ней:

Пример 2. Сложить числа 784 и 548

Записываем числа в столбик. Единицы под единицами,


десятки под десятками, сотни под сотнями:
57

Складываем единицы первого числа с единицами второго


числа: 4+8=12. Число 12 не вмещается в разряд единиц
нашего ответа, поэтому мы из 12 вынимаем цифру 2 из
разряда единиц и записываем её в разряд единиц нашего
ответа. А цифру 1 переносим на следующий разряд:

Теперь складываем десятки. Складываем 8 и 4 плюс


единица, которая осталась от предыдущей операции
(единица осталась от 12, на рисунке она выделена синим
цветом). Складываем 8+4+1=13. Число 13 не вместится в
разряд десятков нашего ответа, поэтому мы запишем цифру
3 в разряде десятков, а единицу перенесём на следующий
разряд:

Теперь складываем сотни. Складываем 7 и 5 плюс единица,


которая осталась от предыдущей операции: 7+5+1=13.
Записываем число 13 в разряд сотен:
58

Вычитание в столбик

Пример 1. Вычтем из числа 69 число 53.

Запишем числа в столбик. Единицы под единицами, десятки


под десятками. Затем вычитаем по разрядам. Из единиц
первого числа вычитаем единицы второго числа. Из десятков
первого числа вычитаем десятки второго числа:

Получили ответ 16.

Пример 2. Найти значение выражения 95 − 26

Записываем в столбик данное выражение:

Разряд единиц числа 95 содержит 5 единиц, а разряд единиц


числа 26 содержит 6 единиц. От пяти единиц нельзя вычесть
шесть единиц, поэтому берем один десяток у разряда
десятков. Этот десяток и имеющиеся пять единиц вместе
составляют 15 единиц. Из 15 единиц можно вычесть 6
59

единиц, получится 9 единиц. Записываем цифру 9 в разряде


единиц нашего ответа:

Теперь вычитаем десятки. Разряд десятков числа 95 раньше


содержал 9 десятков, но мы взяли с этого разряда один
десяток, и сейчас он содержит 8 десятков. А разряд десятков
числа 26 содержит 2 десятка. Из восьми десятков можно
вычесть два десятка, получится шесть десятков. Записываем
цифру 6 в разряде десятков нашего ответа:

Воспользуемся нестандартным способом вычитания при


котором каждая цифра, входящая в число, рассматривается
как отдельное число. При вычитании больших чисел в
столбик этот способ очень удобен.

В разряде единиц уменьшаемого располагается число 5. А в


разряде единиц вычитаемого число 6. Из пятёрки не вычесть
шестёрку. Поэтому берем одну единицу у числа 9. Взятая
единица мысленно дописывается слева от пятёрки. А
поскольку у числа 9 мы взяли одну единицу, это число
уменьшится на одну единицу:
60

В результате пятёрка обращается в число 15. Теперь можно


из 15 вычесть 6. Получается 9. Записываем число 9 в
разряде единиц нашего ответа:

Переходим к разряду десятков. Раньше там располагалось


число 9, но поскольку мы взяли у него одну единицу оно
обратилось в число 8. В разряде десятков второго числа
располагается число 2. Восемь минус два будет шесть.
Записываем число 6 в разряде десятков нашего ответа:
61

Пример 3. Найдем значение выражения 2412 − 2317

Записываем в столбик данное выражение:

В разряде единиц числа 2412 располагается число 2, а в


разряде единиц числа 2317 располагается число 7. Из
двойки не вычесть семёрку, поэтому берем единицу у
следующего числа 1. Взятую единицу мысленно дописываем
слева от двойки:
62

В результате двойка обращается в число 12. Теперь можно


из 12 вычесть 7. Получается 5. Записываем цифру 5 в
разряде единиц нашего ответа:

Переходим к десяткам. В разряде десятков числа 2412


раньше располагалось число 1, но поскольку мы взяли у него
одну единицу, оно обратилось в 0. А в разряде десятков
числа 2317 располагается число 1. Из нуля не вычесть
единицу. Поэтому берем одну единицу у следующего числа
4. Взятую единицу мысленно дописываем слева от нуля. А
поскольку у числа 4 мы взяли одну единицу, это число
уменьшится на одну единицу:
63

В результате ноль обращается в число 10. Теперь можно из


10 вычесть 1. Получается 9. Записываем цифру 9 в разряде
десятков нашего ответа:

В разряде сотен числа 2412 раньше располагалось число 4,


но сейчас там располагается число 3. В разряде сотен числа
2317 также располагается число 3. Три минус три равно
нулю. То же самое и с разрядами тысяч в обоих числах. Два
минус два равно нулю. А если разность старших разрядов
равна нулю, то этот ноль не записывают. Поэтому
окончательным ответом будет число 95.

Пример 4. Найти значение выражения 600 − 8

Запишем в столбик данное выражение:

В разряде единиц числа 600 располагается ноль, а в разряде


единиц числа 8 само это число. Из нуля не вычесть
восьмерку, поэтому берем единицу у следующего числа. Но
64

следующее число это тоже ноль. Тогда за следующее число


принимаем число 60. Берем одну единицу у этого числа и
мысленно дописываем её слева от нуля. А поскольку у числа
60 мы взяли одну единицу, это число уменьшится на одну
единицу:

Теперь в разряде единиц располагается число 10. Из 10 


можно вычесть 8, получится 2. Записываем число 2 в
разряде единиц нового числа:

Переходим к следующему числу, находящемуся в разряде


десятков. В разряде десятков раньше располагался ноль, но
сейчас там располагается число 9, а во втором числе разряд
десятков отсутствует. Поэтому число 9 переносится к новому
числу:

Переходим к следующему числу, находящемуся в разряде


сотен. В разряде сотен раньше располагалось число 6, но
65

сейчас там располагается число 5, а во втором числе разряд


сотен отсутствует. Поэтому число 5 переносится к новому
числу:

Пример 5. Найти значение выражения 10000 − 999

Запишем в столбик данное выражение:

В разряде единиц числа 10000 располагается 0, а в разряде


единиц числа 999 располагается число 9. Из нуля не вычесть
девятку, поэтому берем одну единицу у следующего числа,
находящегося в разряде десятков. Но в следующем разряде
тоже ноль. Тогда за следующее число принимаем 1000 и
берем от этого числа единицу:
66

Следующее число в данном случае было 1000. Взяв у него


единицу, мы обратили его в число 999. А взятую единицу
дописали слева от нуля.

Дальнейшее вычисление не составило особого труда. Десять


минус девять равно одному. Вычитание чисел, находящихся
в разряде десятков обоих чисел дало ноль. Вычитание
чисел, находящихся в разряде сотен обоих чисел тоже дало
ноль. А девятка из разряда тысяч была перенесена к новому
числу:

Пример 6. Найти значение выражения 12301 − 9046

Запишем в столбик данное выражение:

В разряде единиц числа 12301 располагается число 1, а в


разряде единиц числа 9046 располагается число 6. Из
единицы не вычесть шесть, поэтому берем одну единицу у
следующего числа, находящегося в разряде десятков. Но в
следующем разряде располагается ноль. Ноль ничего нам
дать не сможет. Тогда за следующее число принимаем 1230
и берем от этого числа единицу:
67

Следующее число в данном случае было 1230. Взяв у него


единицу, мы обратили его в число 1229. А взятую единицу
мысленно дописали слева от единицы, находящейся в
разряде единиц.

Дальнейшее вычисление не составило особого труда.


Одиннадцать минус шесть равно пять. Вычитание чисел,
находящихся в разряде десятков обоих чисел дало число
5. Вычитание чисел, находящихся в разряде сотен обоих
чисел дало число 2. Вычитание чисел, находящихся в
разряде тысяч обоих чисел дало число 3.
68

Умножение

В этом уроке мы изучим умножение чисел. Напомним, что


для умножения маленьких чисел предназначена таблица
умножения. Обязательно выучите её наизусть, поскольку
любое умножение больших чисел в конечном итоге свóдится
к тому, чтобы умножить маленькие.

Содержание урока

 Однозначные и многозначные числа


 Умножение однозначных чисел
 Умножение на 10, 100, 1000
 Умножение чисел, которые оканчиваются нулями
 Умножение многозначного числа на однозначное
 Умножение многозначных чисел на многозначные
 Задания для самостоятельного решения

Однозначные и многозначные числа

Для начала введём два новых понятия: однознáчные и


многознáчные числа.

Однознáчным называется число, которое состоит из одной


цифры. Например, следующие числа являются
однознáчными:

0, 1, 2, 3, 4, 5, 6, 7, 8, 9

Слово «однознáчные» говорит само за себя. Однознáчное —


значит состоит из одного знака (цифру иногда называют
знáком).
69

Многознáчным называется число, которое состоит из двух и


более цифр. Например, следующие цифры являются
многознáчными:

10, 11, 15, 255, 350, 1000, 12500

Многознáчных чисел бесконечно много. Их не сосчитать.


Кроме того, они подразделяются на следующие виды:

 двузнáчные, которые состоят из двух цифр (например,


25);
 трёхзнáчные, которые состоят из трёх цифр (например,
563);
 четырёхзнáчные, которые состоят из четырёх цифр
(например, 1400)

и так далее, в зависимости от того сколько цифр в числе.

Умножение однозначных чисел

Однозначные числа умножаются легко. Достаточно знать


таблицу умножения. Примеры:

5 × 5 = 25

3 × 5 = 15

7 × 6 = 42

5 × 8 = 40

Если по каким-либо причинам не удаётся вспомнить таблицу


умножения, то можно воспользоваться сложением. Ведь
умножение это ни что иное как многократное сложение.
70

Чтобы умножить, например, число 4 на число 3, нужно число


4 сложить три раза:

Умножение на 10, 100, 1000

Чтобы умножить любое число на 10,  100 или 1000,


достаточно дописáть к множимому количество нулей из
множителя.

Например, чтобы умножить 12 на 10, нужно к множимому 12


дописать в конце ноль из множителя 10. В результате
получим ответ 120

Еще примеры:

12 × 100 = 1200 (к 12 дописали два нуля, поскольку в числе


100 два нуля)

12 × 1000 = 12000 (к 12 дописали три нуля, поскольку в числе


1000 три нуля)
71

15 × 100 = 1500 (к 15 дописали два нуля, поскольку в числе


100 два нуля)

320 × 100 = 32000 (к 320 дописали два нуля, поскольку в 100


два нуля)

Если нулём оканчивается не множитель, а множимое, то для


получения ответа нужно дописать ноль после множителя.

Например, чтобы умножить 10 на 12, нужно в ответе


записать множитель 12 и дописать в конце один ноль:

10 × 12 = 120

Умножение чисел, которые оканчиваются нулями

Если оба числа оканчиваются нулями, то нужно перемнóжить


те цифры, которые нулями не являются, затем к
полученному результату дописáть все нули из обоих чисел.

Например, умнóжим 20 на 30.

20 × 30

Видим, что оба числá содержат по нулю. Сначала


перемнóжим те цифры, которые нулями не являются. Это
цифры 2 и 3. Два умножить на три будет шесть:

20 × 30 = 6

Теперь к полученному результату, то есть к числу 6


дописываем все нули из обоих чисел. В числе 20 один ноль,
в числе 30 также один ноль. Итого два нуля. Дописываем два
нуля к числу 6
72

20 × 30 = 600

Пример 2. Умножить 40 на 300

Сначала перемнóжим те цифры, которые нулями не


являются. Это цифры 4 и 3. Четыре умножить на три будет
двенадцать:

40 × 300 = 12

Теперь к полученному результату, то есть к числу 12


дописываем все нули из обоих чисел. В числе 40 один ноль,
в числе 300 — два нуля. Итого три нуля. Дописываем три
нуля к числу 12

40 × 300 = 12000

Пример 3. Умножить 600 на 3000

Сначала перемнóжим те цифры, которые нулями не


являются. Это цифры 6 и 3. Шесть умножить на три будет
восемнадцать:

600 × 3000 = 18

Теперь к полученному результату, то есть к числу 18


дописываем все нули из обоих чисел. В числе 600 два нуля,
в числе 3000 — три нуля. Итого пять нулей. Дописываем пять
нулей к числу 18

600 × 3000 = 1800000


73

Умножение многозначного числа на однозначное

Чтобы умножить многозначное число на однозначное, надо


умножить каждую цифру многозначного числа на это
однозначное число. Например, найдем значение выражения
12 × 3. Записываем данное выражение в столбик, при этом
единицы должны быть под единицами. Всё это соединяется
знаком умножения ( × )

Далее каждая цифра многозначного числа умножается на 3.


Умножать начинаем с разряда единиц, то есть с цифры 2.
Два умножить на три будет шесть. Записываем цифру 6 в
разряде единиц нашего ответа:

Теперь умножаем 1 на 3, получаем 3. Записываем цифру 3 в


разряде десятков нашего ответа:

Получили ответ 36.

В данном примере множимым было число 12, а множителем


число 3. Число 12 это две единицы и один десяток. Наша
задача заключалась в том, чтобы увеличить эти две единицы
74

и один десяток в 3 раза. Тогда решая этот пример, можно


было бы рассуждать следующим образом:

Увеличим две единицы в 3 раза: 2 × 3 = 6. Получили шесть


единиц. Записываем цифру 6 в разряде единиц нового числа

Увеличим один десяток в 3 раза: 1 × 3 = 3. Получили три


десятка. Записываем цифру 3 в разряде десятков нового
числа:

Иногда при умножении одной цифры многозначного числа на


однозначное число получается многозначное число. В этом
случае сначала записывается одна цифра из разряда
единиц, а остальные цифры переносятся на следующий
разряд, к которому они будут добавлены после вычисления.

Например, найдем значение выражения 23 × 6

Умножаем каждую цифру числа 23 на 6. Начинаем с тройки:


3 × 6 = 18. Восемнадцать не вмещается в разряд единиц
нашего ответа, поэтому сначала записывается 8, а 1
переносится на следующий разряд. Эта единица будет
прибавлена к результату умножения 2 на 6
75

Теперь умножаем 2 на 6, получаем 12, плюс единица,


которая досталась от предыдущего умножения. На рисунке
эта единица выделена синим цветом. Вычисляем
(2 × 6) + 1 = 13

Получили ответ 138. В данном примере множимым было


число 23, а множителем число 6. Число 23 это три единицы
и два десятка. Наша задача заключалась в том, чтобы
увеличить эти три единицы и два десятка в 6 раз. Тогда
решая этот пример, можно было бы рассуждать следующим
образом:

Увеличим три единицы в 6 раз: 3 × 6 = 18. Получили


восемнадцать единиц. Произошло переполнение разряда в
разряде единиц. Число 18 это 8 единиц и 1 десяток. 8
единиц записываем в разряде единиц нового числа, а 1
десяток отправляем к разряду десятков. Этот десяток
мы прибавим, когда увеличим два десятка в шесть раз:
76

Увеличим два десятка в 6 раз: 2 × 6 = 12. Получили


двенадцать десятков. Плюс прибавляем один десяток,
который остался от числа 18.

12 десятков плюс 1 десяток будет 13 десятков.


Записываем число 13 в разряде десятков нового числа,
образуя окончательный ответ:

Пример 3. Найти значение выражения 326 × 5

Записываем в столбик данное выражение:

Умножаем каждую цифру числа 326 на 5. Начинаем с


шестёрки: 6 × 5 = 30. Число 30 не вмещается в разряд
единиц нашего ответа, поэтому сначала записываем 0, а
тройку переносим на следующий разряд:
77

Теперь умножаем 2 на 5, получаем 10 плюс тройка, которая


досталась от предыдущей операции: (2 × 5) + 3 = 13.
Получили число 13, которое не вмещается в разряд десятков
нашего ответа. Поэтому записываем сначала 3, а единицу
переносим на следующий разряд:

Теперь умножаем последнюю тройку на 5, плюс прибавляем


единицу, которая досталась от предыдущей операции:
(3 × 5) + 1 = 16. Получили 16. Записываем это число целиком,
образуя окончательный ответ:

Умножение многозначных чисел на многозначные

Умножение многозначных чисел на многозначные


происходит таким же образом, как и умножение
многозначных на однозначные. Каждая цифра многозначного
78

числа умножается на каждую цифру другого многозначного


числа. Единственное отличие заключается в том, что в конце
образуется своего рода лесенка ответов, которые надо
сложить. Рассмотрим несколько примеров, чтобы хорошо
понять это.

Пример 1. Найти значение выражения 12 × 14

Записываем данное выражение в столбик — единицы под

единицами, десятки десятками:

Теперь умножаем каждую цифру числа 12 на каждую цифру


числа 14. Делать это надо по-очереди, начав с четвёрки. В
результате таких действий мы приходим к умножению
многозначного числа на однозначное, которое проходили
ранее:

Умножив 12 на 4, мы получили число 48, которое записали


таким образом, чтобы разряд единиц этого числа оказался
под четверкой, на которую мы умножали число 12.

Теперь умножаем 12 на 1:
79

Умножив 12 на 1 мы получили число 12 и записали его таким


образом, чтобы разряд единиц этого числа оказался под
единицей, на которую мы умножали число 12.

Мы получили лесенку ответов 48 и 12, которую надо сложить.


Складываем и получаем ответ 168

В данном примере множитель 14 это четыре единицы и один


десяток. Тогда умножение 12 на 14 можно понимать как
увеличение числа 12 в четыре раза и в десять раз. Этим и
объясняется появление лесенки в конце решения. Давайте
посмотрим как это выглядит на каждом этапе:

Увеличим число 12 в четыре раза, получим число 48


80

Увеличим число 12 в десять раз, получим число 120.


Записываем 120 так, чтобы можно было сложить единицы
этого числа с единицами числа 48, а десятки числа 120
можно было сложить с десятками числа 48

Теперь сложим получившуюся лесенку ответов. Единицы


сложим с единицами, десятки с десятками, сотни с сотнями.
В результате образуется окончательный ответ:

Но чаще всего множитель не группируется с помощью


разрядов, и умножение выполняют, умножая каждую цифру
множимого на каждую цифру множителя.

Пример 2. Найти значение выражения 25 × 36


81

Записываем данное выражение в столбик

Умножаем каждую цифру числа 25 на каждую цифру числа


36.

Умножим 25 на 6:

Умножаем 25 на 3:

Теперь сложим получившуюся лесенку:


82

Получили ответ 900.

Рассмотрим большой и сложный пример на умножение:


12305 × 5641. Будем придерживаться ранее изученных
правил.

Сначала записываем в столбик данное выражение

Теперь начинаем умножать. Число 12305


надо умножить на каждую цифру числа 5641.
83

Умножив 12305 на 1, мы получили 12305 и записали это


число так, чтобы разряд единиц этого числа оказался под
единицей, на которую мы умножили 12305.

Теперь умножаем 12305 на следующую цифру 4:

Умножив 12305 на 4, мы
получили 49220 и записали это число так, чтобы разряд
единиц этого числа оказался под четверкой, на которую
умножали 12305.

Умножаем 12305 на следующую цифру 6:

Умножив 12305 на 6, мы получили 73830 и записали это


число так, чтобы разряд единиц этого числа оказался под
шестёркой, на которую мы умножали 12305.
84

Теперь умножаем 12305 на последнюю цифру 5:

Умножив 12305 на 5, мы получили 61525 и записали это


число так, чтобы разряд единиц этого числа оказался под
пятёркой, на которую умножали 12305.

В результате мы получили большую лесенку, которую надо


сложить. Складываем:

Получили окончательный ответ 69412505.

Если вы поняли этот пример, то можно сказать, что


умножение больших чисел вы усвоили на отлично.
85

На этом урок по умножению можно завершить. Обязательно


потренируйтесь, решив несколько примеров, которые даны
ниже.

Важно отметить, что все эти стрелки и подробные решения,


как на картинках в «боевых условиях» рисовать не принято.
Нужно уметь сразу записывать ответы, выполняя в уме все
вычисления.

Исключением является то, если человек давно не занимался


математикой или никогда ею не занимался. В таком случае
можно рисовать для себя стрелки и другие вспомогательные
схемы для хорошего усвоения материала.
86

Деление

В данном уроке мы изýчим деление чисел. Деление чисел


довольно непростая операция как в освоении, так и в
использовании. Рекомендуем набраться терпения, чтобы
осилить этот урок до конца.

Содержание урока

 Что такое деление?


 Деление с остатком
 Деление уголком
 Деление многозначного числа на однозначное
 Деление чисел, у которых на конце 0
 Деление многозначного числа на многозначное
 Задания для самостоятельного решения

Что такое деление?

Деление это действие, позволяющее что-либо разделить.

Деление состоит из трёх параметров: делимого, делителя и


частного. Делимое это то, что делят. Делитель это число,
показывающее на сколько частей нужно разделить делимое.
Частное это собственно результат.

Пусть у нас имеются 4 яблока:


87

Разделим их поровну на двоих друзей. Тогда деление


покажет сколько яблок достанется каждому. Нетрудно
увидеть, что каждому достанется по два яблока:

Процесс деления четырех яблок на двоих друзей можно


описáть следующим выражением:
88

В этом примере роль делимого играют яблоки. Роль


делителя играют двое друзей, показывающих на сколько
частей нужно разделить 4 яблока. Роль частного играют два
яблока, показывающие сколько досталось каждому.

Говоря о делении, можно рассуждать и по-другому. Вернёмся


к предыдущему выражению 4 : 2 = 2. Можно посмотреть на
делитель 2 и задать вопрос «сколько двоек в четвёрке?» и
ответить: «две двойки». Действительно, если сложить две
двойки, то получится число 4

В ситуации с четырьмя яблоками можно задать вопрос


«сколько раз два яблока содержатся в четырёх яблоках» и
ответить: «два раза».

Чтобы научиться делить, нужно хорошо знать таблицу


умножения. Почему же умножения? Ведь мы говорим о
делении. Дело в том, что деление это действие, обратное
89

умножению. Данную фразу можно понимать в прямом


смысле. Например, если 2 × 5 = 10, то 10 : 5 = 2.

Видно, что второе выражение записано в обратном порядке.


Если у нас имеются два яблока и мы захотим увеличить их в
пять раз, то запишем 2 × 5 = 10. Получится десять яблок.
Затем, если мы захотим обратно уменьшить эти десять
яблок до двух, то запишем 10 : 5 = 2

Знак деления выглядит в виде двоеточия : но также можно


встретить знак двоеточия и тире ÷ 

На письме разумнее использовать двоеточие, поскольку оно


выглядит аккуратнее.

Деление с остатком

Остаток — это то, что осталось от действия деления


неразделённым.

Например, пять разделить на два будет два и один в остатке:

5 : 2 = 2 (1 в остатке)

Можно проверить это умножением:

(2 × 2) + 1 = 5

Допустим, у нас имеются пять яблок


90

Разделим их поровну на двоих друзей. Но разделить поровну


пять целых яблок не получится. Тогда данное деление
покажет, что каждому достанется два яблока, а одно яблоко
будет в остатке:

Деление уголком

Когда требуется разделить большое число, то прибегают к


такому методу как деление уголком.
91

Прежде чем делить уголком, человек должен понимать:

 обычное деление маленьких чисел;


 деление с остатком;
 умножение в столбик;
 вычитание в столбик.

Рассмотрим деление уголком на простом примере. Пусть


требуется найти значение выражения 9 : 3. Уголком это
выражение записывается  следующим образом:

Это простой пример. Все знают, что девять разделить на три


будет три. Ответ (частное) записывается под правым углом:

Чтобы проверить есть ли остаток от деления, нужно частное


умножить на делитель и полученный ответ записать под
делимым. Частное в данном случае это 3, делитель тоже 3.
Перемножаем эти два числа: 3 × 3 = 9. Получили 9.
Записываем эту девятку под делимым:
92

Теперь от делимого вычитаем девятку, которую мы под ним


написали: 9 − 9 = 0. Остаток равен нулю. Проще говоря,
остатка нет. На этом деление успешно завершено:

Пример 2. Найти значение выражения 8 : 3

Восемь на три просто-так не разделится. Таблица умножения


тоже не поможет. В данном случае будет присутствовать
остаток от деления.

Сначала запишем данное выражение уголком:

Теперь надо задать вопрос: «сколько троек в восьмёрке?» В


восьмёрке содержится две тройки. Это можно увидеть даже
воочию, если мы представим восьмёрку как восемь палочек:
93

В школе частное подбирается методом подбора. Все мы


слышали такие фразы как «берём по одному» , «берём по
два» или «берём по три». У нас сейчас как раз такой случай.
Мы взяли по два, ответив что в восьмёрке две тройки.
Записываем двойку в правом уголке:

Теперь вынимаем остаток. Для этого умножаем частное на


делитель (2 на 3) и записываем полученное число под
делимым:

Далее из 8 вычитаем 6. Полученное число и будет остатком:

8 : 3 = 2 (2 в остатке)

Проверка: (2 × 3) + 2 = 6 + 2 = 8
94

Деление многозначного числа на однозначное

Данная тема с первого раза может показаться непонятной.


Не спешите отчаиваться и забрасывать обучение.
Понимание придёт в любом случае. Если не сразу, то
немного позже. Главное не сдаваться и продолжать упорно
изучать.

В предыдущих примерах мы делили однозначное число на


однозначное, и это не доставляло нам лишних проблем.
Сейчас мы займёмся тем, что будем делить многозначное
число на однозначное.

Если непонятно, что такое однозначные и многозначные


числа, советуем изучить предыдущий урок, который
называется умножение.

Чтобы разделить многозначное число на однозначное, нужно


сначала посмотреть на первую цифру этого многозначного
числа, и проверить больше ли она делителя. Если больше,
то разделить, а если нет, то проверить больше ли делителя
первые две цифры многозначного числа. Если первые две
цифры больше делителя, то разделить, а если нет, то
проверить больше ли первые три цифры многозначного
числа. И так до тех пор, пока не будет выполнено первое
деление.

Сложно? Ни чуть, если мы разберём несколько примеров.

Пример 1. Найти значение выражения 25 : 3

25 это многозначное число, а 3 — однозначное. Применяем


правило. Смóтрим на первую цифру многозначного числа.
95

Первая цифра это 2. Два больше, чем три? Нет. Поэтому


смóтрим первые две цифры многозначного числа. Первые
две цифры образуют число 25. Двадцать пять больше, чем
три? Да, больше. Поэтому выполняем деление числа 25 на 3.
Записываем уголком данное выражение и начинаем делить:

Сколько троек в числе 25? Если с первого раза ответить


сложно, можно заглянуть в таблицу умножения на три. Там
необходимо отыскать произведение, которое меньше 25, но
очень близко к нему или равно ему. Если найдём такое
произведение, то необходимо забрать оттуда множитель,
который дал такое произведение:

Это таблица умножения на три. В ней необходимо найти


произведение, которое меньше 25, но очень близко к нему
или равно ему. Очевидно, что это произведение 24, которое
выделено синим. Из этого выражения необходимо забрать
множитель, который дал такое произведение. Это множитель
8, который закрашен красным.
96

Данная восьмёрка и отвечает на вопрос сколько троек в


числе 25. Записываем её в правом уголке нашего примера:

Теперь вынимаем остаток. Для этого умножаем частное на


делитель (8 на 3) и полученное число записываем под
делимым:

Теперь из делимого вычитаем число 24, получим


1. Это и будет остатком:

25 : 3 = 8 (1 в остатке)

(8 × 3) + 1 = 24 + 1 = 25

Последний остаток всегда меньше делителя. Если


последний остаток больше делителя это означает, что
деление не завершено.

В приведённом примере последним остатком было число 1, а


делителем число 3. Единица меньше, чем три, поэтому
97

деление завершено. Последний остаток, меньший делителя,


говорит о том, что он не содержит чисел, равных делителю.

В нашем примере, если задать вопрос «сколько троек в


единице?», то ответом будет «нисколько», потому что
единица не содержит троек, поскольку она меньше тройки.

Пример 2. Разделить 326 на 4.

Смотрим на первую цифру числа 326. Первая цифра это 3.


Она больше делителя 4? Нет. Тогда проверяем две цифры
делимого. Две цифры делимого образуют число 32. Больше
ли оно делителя 4? Да, больше. Поэтому делим. Записываем
уголком данное выражение:

Теперь задаём вопрос: «сколько четвёрок в числе 32?». В


числе 32 восемь четвёрок. Это можно увидеть в таблице
умножения на четыре:
98

Данная восьмёрка, которая выделена красным отвечает на


вопрос сколько четвёрок в числе 32. Записываем её в правом
уголке нашего примера:

Теперь умножаем 8 на 4, получаем 32


и записываем это число под делимым. Далее вычитаем это
число из 32. Получим 0. Поскольку решение ещё не
завершено, ноль не записываем:

Первое число 32 разделили.


Осталось разделить оставшуюся 6. Для этого сносим эту
шестёрку:

Теперь делим 6 на 4. Для этого задаём вопрос: «сколько


четвёрок в шестёрке?» В шестёрке одна четвёрка, это
можно увидеть воочию, если представить шестёрку как
шесть палочек:
99

Записываем единицу в правом уголке нашего ответа:

Теперь умножаем нашу единицу на


делитель (1 на 4) и записываем полученное число под
шестёркой:

Затем из 6 вычитаем 4, получаем число 2, которое является


остатком:
100

Получили 326 : 4 = 81 (2 в остатке)

Проверка: (81 × 4) + 2 = 324 + 2 = 326

Процедура, в которой мы ищем первое число для деления,


сравнивая больше ли оно делителя или меньше, называется
нахождением первого неполного делимого.

Вернёмся к предыдущему примеру 326 : 4. Первое неполное


делимое в данном выражении было число 32, поскольку
его мы разделили в первую очередь.

А в примере 25 : 3 первое неполное делимое было 25.

Пример 3. Найти значение выражения 384 : 5

Записываем данное выражение в уголком:


101

Сначала находим первое неполное делимое. Первая


цифра меньше делителя, поэтому проверяем две цифры.
Две цифры вместе образуют число 38, которое больше
делителя. Это число будет первым неполным делимым.
Его и будем в первую очередь делить на делитель:

Сколько пятёрок в числе 38? Если сразу ответить сложно, то


можно посмотреть в таблицу умножения на пять и найти
произведение, которое меньше 38, но очень близко к нему
или равно ему. Найдя такое произведение, нужно забрать
оттуда множитель, который будет отвечать на наш вопрос:

Это таблица умножения на пять. Находим произведение,


которое меньше 38, но очень близко к нему или равно ему.
Очевидно, что это произведение 35, которое выделено
синим. Из этого выражения забираем множитель, который
дал такое произведение. Это множитель 7, который выделен
красным.
102

Данная семёрка отвечает на вопрос сколько пятёрок в числе


38. Записываем эту семёрку в правом уголке нашего
примера:

Умножаем 7 на 5, получаем 35 и записываем его


под 38:

Теперь из 38 вычитаем 35, получим 3:

Эта тройка является остатком, которая осталась


неразделённой в результате деления 38 на 5. Но видно, что
ещё надо разделить и 4. Эту 4 мы снесём и разделим вместе
с тройкой:

Видно, что после того, как мы снесли четвёрку, она вместе с


тройкой  образовала число 34. Это число 34 мы будем
делить на 5. Для этого опять задаем вопрос: «сколько
пятёрок в числе 34?». Можно снова глянуть в таблицу
103

умножения на пять и найти произведение, которое меньше


34, но очень близко к нему или равно ему:

Видно, что в таблице умножения на пять число 30 меньше


нашего 34, но близко к нему. Из этого выражения забираем
множитель 6, который отвечает на наш вопрос. Записываем
эту шестёрку в правом уголке нашего примера:

Теперь умножаем 6 на 5, получаем 30 и записываем это


число под 34:

Теперь из 34 вычитаем 30, получаем 4. Эта четвёрка будет


остатком от деления 384 на 5
104

384 : 5 = 76 (и 4 в остатке)

Проверка: (76 × 5) + 4 = 380 + 4 = 384

Пример 4. Найти значение выражения 8642 : 4

Этот пример немного посложнее. Записываем уголком


данное выражение:

Первая цифра 8 больше делителя. Эта восьмёрка будет


первым неполным делимым. Делим 8 на 4, получаем 2

Теперь умножаем 2 на 4, получаем 8.


Записываем эту восьмёрку под первым неполным делимым:
105

Вытаскиваем остаток: 8 − 8 = 0. Остаток от деления 8 на 4


это ноль. Ноль не записываем, поскольку решение примера
не завершено.

Далее сносим цифру 6 и делим её на делитель, получаем 1

Умножаем 1 на 4, получаем 4. Записываем


эту четвёрку под снесённой шестёркой. Затем вынимаем
остаток, отняв от шести четыре:

Получили остаток 2. Это остаток, который остался от


деления 6 на 4.

Теперь сносим следующую цифру из делимого. Это цифра 4.


Эта четвёрка вместе с предыдущим остатком 2 образует
число 24. Его делим на делитель. Получим 6
106

Умножаем 6 на 4, получаем 24. Записываем это число под 24

Вытаскиваем остаток: 24 − 24 = 0. Ноль это остаток от


деления 24 на 4. Ноль, как мы уже договорились, не
записываем. Далее сносим последнюю цифру 2

Здесь начинается самое интересное. Двойка это последняя


цифра, которую мы снесли и которую надо разделить на
делитель 4. Но дело в том, что двойка меньше четвёрки, а
ведь делимое должно быть больше делителя. Если мы
107

зададим вопрос «сколько четвёрок в двойке?«, то ответом


будет ноль, поскольку двойка меньше четвёрки и не может
содержать в себе число, бóльшее себя самогó.

Поэтому два разделить на четыре это ноль:

Умножаем 0 на 4, получаем 0. Пишем этот 0


под двойкой:

Теперь находим остаток: 2 − 0 = 2. Двойка это остаток от


деления 8642 на 4. Таким образом, пример завершён:
108

8642 : 4 = 2160 (2 в остатке)

Проверка: (2160 × 4) + 2 = 8640 + 2 = 8642

Деление чисел, у которых на конце 0

Чтобы разделить число, у которого на конце ноль, нужно


временно отбросить этот ноль, выполнить обычное деление,
и дописать этот ноль в ответе.

Например, разделим 120 : 3

Сколько троек в числе 120? Чтобы ответить на этот вопрос,


временно отбрасываем ноль на конце у 120 и делим 12 на 3,
получаем 4. И дописываем этот ноль в частном. В итоге
получаем 40:
109

Теперь умножаем частное на делитель (40 на 3), получаем


120. Далее находим остаток: 120 − 120 = 0. Остаток равен
нулю. Пример завершён.

120 : 3 = 40

Проверка 40 × 3 = 120.

Такие простые примеры не нуждаются в том, чтобы их


решали уголком. Достаточно знать таблицу умножения.
Далее просто дописывать нули на конце. Например:

12 : 3 = 4 (делимое без нулей на конце)

120 : 3 = 40 (здесь у делимого один ноль)

1200 : 3 = 400 (здесь у делимого два нуля)

12000 : 3 = 4000 (здесь у делимого три нуля)

В этом способе есть небольшой подвох. Если вы заметили,


деля такие числа, мы ссылаемся на таблицу умножения. А
представьте, что надо разделить 400 на 5.

Можно рассуждать по старому — отбросить временно все


нули и разделить обычные числа. А что будет если
110

отбросить все нули в числе 400? Мы обнаружим, что делим 4


на 5, что недопустимо. В этом случае, надо отбрасывать
только один ноль, и делить 40 на 5, а не 4 на 5

Завершаем этот
пример, как обычно умножая частное на делитель, и выводя
остаток:

Этот способ работает только в том случае,


если удаётся гладко применить таблицу умножения. В
остальных случаях, придётся искать обходные пути,
вычисляя уголком или собирая частное подобно детскому
конструктору.

Например, найдём значение выражения 1400 : 5. Здесь


отбрасывание нулей нам ничего не даст. Этот пример надо
решать уголком или собрать ответ, подобно конструктору.
Давайте рассмотрим второй способ.

Что такое 1400? Вспоминаем разряды чисел. 1400 это одна


тысяча и четыре сотни:

1000 + 400 = 1400

Можно по-отдельности разделить 1000 на 5 и 400 на 5:

1000 : 5 = 200
111

400 : 5 = 80

и сложить полученные результаты:

200 + 80 = 280

Итого: 1400 : 5 = 280

Решим этот же пример уголком:

Деление многозначного числа на многозначное

Здесь придётся хорошенько напрячь свой мозговой аппарат


и выжать из него по максимуму, потому что разделить
многозначное число на многозначное не так то просто.

Принцип деления остаётся тем же что и раньше. Здесь так


же надо находить первое неполное делимое. Здесь так же
могут присутствовать остатки от деления.

Для начала введём новое понятие — круглое число.


Круглым будем называть число, которое оканчивается нулём.
Например, следующие числа являются круглыми:

10, 20, 30, 500, 600, 1000, 13000


112

Любое число можно превратить в круглое. Для этого первые


цифры, образующие старший разряд, оставляют без
изменений, а остальные цифры заменяют нулями.

Например, превратим число 19 в круглое число. Первая


цифра этого числа 1 образует старший разряд (разряд
десятков) — эту цифру оставляем как есть, а оставшуюся 9
заменяем на ноль. В итоге получаем 10

Ещё пример. Превратим число 125 в круглое число. Первая


цифра 1 образует старший разряд (разряд сотен) — эту
цифру оставляем без изменений, а оставшиеся цифры 25
заменяем нулями. В итоге получаем 100.

Ещё пример. Превратим число 2431 в круглое число. Первая


цифра 2 образует старший разряд (разряд тысяч) — эту
цифру оставляем без изменений, а остальные цифры 431
заменяем нулями. В итоге получаем 2000.

Ещё пример. Превратим число 13 735 в круглое число.


Первые две цифры 13 образуют старший разряд (разряд
десятков тысяч) — эти две цифры оставляем без изменений,
а остальные цифры 735 заменяем нулями. В итоге получаем
13 000.

Внимание! В дальнейшем понятия круглого числа и


перевод любого числа в круглое будут обобщены.

Возвращаемся к делению многозначных чисел на


многозначные. Сложность деления таких чисел заключается
в том, что частное надо находить методом подбора. Для
этого прибегают к различным техникам, например,
превращают делимое и делитель в круглые числа.
113

Пример 1. Найти значение выражения 88 : 12

Записываем данное выражение уголком:

Задаём вопрос сколько чисел 12 в числе 88? С


первого раза ответить сложно. Придётся рассуждать.

Со школы мы помним, что частное подбиралось методом


угадывания, говоря «берем по два» или «берем по три».

Давайте попробуем угадать частное. К сожалению, его


просто так с неба взять нельзя. Это частное должно быть
таким, чтобы при его умножении на делитель, получалось
число, которое меньше делимого, но очень близко к нему или
равно ему.

Давайте предположим, что частное равно 2. Умножаем это


частное на делитель 12

Что это нам дало? Полученное число меньше делимого, но


близко к нему? Нет. Оно конечно же меньше делимого 88, но
очень далеко от него. Значит двойка как частное не
подходит.

Пробуем следующее число. Допустим частное равно 5


114

Полученное число конечно меньше, но оно не близко к


делимому 88. Значит пятёрка как частное тоже не подходит.

Попробуем сразу взять по 8

На этот раз полученное число превзошло делимое. А оно


должно быть меньше делимого, но очень близким к нему или
равным ему. Значит восьмёрка как частное тоже не подходит
Попробуем тогда взять по 7

Наконец-то нашли подходящее частное! Умножив частное 7


на делитель 12, мы получили 84, которое меньше делимого,
но близко к нему. Теперь находим остаток от деления. Для
этого из 88 вычитаем 84, получаем 4.
115

88 : 12 = 7 (4 в остатке)

Проверка: (12 × 7) + 4 = 84 + 4 = 88

Как видно из примера, на подбор частного уходит


драгоценное время. Если мы будем сидеть на контрольной
или на экзамене, где каждая минута очень дорогá, этот метод
нам явно не поможет.

Чтобы сэкономить время, можно делимое и делитель


превратить в круглые числа, а затем осуществить деление
этих круглых  чисел. Делить круглые числа намного проще и
удобнее.

Например, чтобы разделить 90 на 10, достаточно отбросить


нули у обоих чисел и разделить 9 на 1. В итоге получим 90 :
10 = 9.

Количество отбрасываемых нулей должно быть строго


одинаковым. К примеру, если мы делим 900 на 90, то
отбрасываем по нулю от каждого числа, поскольку у числа
900 два нуля, а у 90 только один. Отбросив по нулю от
каждого числа, мы получим выражение 90 : 9 = 10. В итоге
получаем 900 : 90 = 10.

В делении круглых чисел также нет ничего сложного.


Постарайтесь понять это. Если непонятно, изучите этот
момент несколько раз. Это очень важно.

Ниже приведено несколько примеров, где делятся круглые


числа. Отбрасываемые нули закрашены серым цветом:

800 : 10 = 80 (отбросили по нулю и разделили 80 на 1,


получили 80)
116

800 : 80 = 10 (отбросили по нулю и разделил 80 на 8,


получили 10)

900 : 10 = 90 (отбросили по нулю и разделили 90 на 1,


получили 90)

400 : 50 = 8 (отбросили по нулю и разделили 40 на 5,


получили 8)

320 : 80 = 4 (отбросили по нулю и разделили 32 на 8,


получили 4)

Заметно, что всё в конечном итоге свóдится к таблице


умножения. Именно поэтому в школе требуют знать её
наизусть. Мы тоже этого требуем, хоть и не принуждаем.

Теперь давайте решим предыдущий пример 88 : 12 где мы


бились, находя частное методом угадывания.

Для начала превращаем делимое и делитель в круглые


числа.

Круглым числом для 88 будет число 80.

А круглым числом для 12 будет число 10.

Теперь делим полученные круглые числа:

80 разделить 10 будет 8. Эту


восьмёрку мы пишем в частном:
117

Теперь проверяем, верно ли подобралось частное. Для этого


умножаем частное на делитель (8 на 12). Восьмёрку как
частное мы уже проверяли, когда решали этот пример
методом угадывания. Она нам не подошла, поскольку после
её умножения на делитель, получилось число 96, которое
больше делимого. Зато подошло частное 7, которое меньше
восьмёрки всего-лишь на единицу.

Отсюда можно сделать вывод, что в выражении 88 : 12


частное, полученное путём превращения делимого и
делителя в круглые числа, больше лишь на единицу. Наша с
вами задача уменьшить это частное на единицу.

Так и сделаем — уменьшим 8 на единицу: 8 − 1 = 7. Семёрка


это частное. Записываем её в правом уголке нашего
примера:

Как видно, этим способом мы решили этот пример намного


быстрее.

Пример 2. Найти значение выражения 1296 : 144

Записываем уголком данное выражение. Сразу же находим


первое неполное делимое. Его образуют все четыре цифры
делимого:
118

Это деление многозначного числа на


многозначное. Давайте применим только что изученный
метод. Превратим делимое и делитель в круглые числа, а
затем разделим их.

Для делимого 1296 круглым числом будет 1000. А для


делителя 144 круглым числом будет 100.

Делим 1000 на 100, получим 10. Проверим полученную


десятку, умножив её на делитель 144

Десятка не подходит, поскольку при умножении получается


число, которое больше делимого.

Попробуем взять по 9, уменьшив десятку на единицу.

Проверяем девятку. Для этого умножаем её на делитель:


119

Красота! Полученное число оказалось не только ближе к


делимому, но и равным ему. Это значит, что деление
выполнилось без остатка. Завершаем данный пример,
вычитая из 1296 полученное число 1296

1296 : 144 = 9

Проверка: 144 × 9 = 1296

Пример 3. Попробуем решить большой и сложный пример


227 492 : 331

Записываем уголком данное выражение. Сразу же


определяем первое неполное делимое. Его образуют первые
четыре цифры делимого 2274. Значит сначала будем делить
2274 на 331. Их же превратим в круглые числа.

Для числа 2274 круглым числом будет 2000. А для 331


круглым числом будет 300
120

Получили 6.
Проверим верно ли подобралась эта шестёрка. Для этого,
умножим её на делитель 331:

Шестёрка
подошла, потому что она отвечает на вопрос сколько чисел
331 в числе 2274. Если бы мы взяли по семь, то получилось
бы следующее:

Если бы мы взяли по 7 и проверили эту


семёрку, то получили бы 2317, которое больше делимого, а
это недопустимо.

Продолжаем решать наш пример. Вычитаем из 2274 число


1986, получаем 288:
121

288 это остаток от деления 2274 на 331. Далее, чтобы


продолжить деление, нужно снести девятку:

Теперь надо разделить 2889 на 331. Превращаем их в


круглые числа и делим их. Сразу же проверяем полученное
таким способом частное:

Умножив 6 на
331, мы снова получили 1986. Это число должно быть
меньше делимого 2889, но близким к нему или равным ему.
Но 1986 очень далеко от него. Значит шестёрка, как частное
не подходит. Проверим тогда семёрку. Это первый случай,
когда нам не помог второй способ, который экономил нам
время. Дальнейшее решение придётся проводить методом
угадывания частного:

Проверили семёрку. Снова получили


число, которое далеко от делимого 2889. Значит семёрка
тоже не подходит. Проверим восьмёрку:
122

Восьмёрка подошла. Она отвечает на вопрос сколько чисел


331 в числе 2889. Если бы мы взяли по девять, то при
умножении на делитель, получили бы число 2979, а это уже
больше делимого 2889.

Теперь вынимаем остаток от деления 2889 на 331. Для этого


от 2889 вычитаем 2648 и получаем 241

241 это остаток от деления 2889 на 331. Чтобы продолжить


деление, нужно снести 2 из главного делимого:

Теперь делим 2412 на 331. Возьмём по 7


123

Теперь находим последний остаток. Для этого из 2412


вычитаем 2317, получаем 95. На этом пример завершается:

227 492 : 331 = 687 (95 в остатке)

Проверка: (331 × 687) + 95= 227 397 + 95 = 227 492

На этом данный урок можно завершить. Не расстраивайтесь,


если сразу не научитесь делить числа уголком. Этот навык
нарабатывается со временем в сочетании с интенсивными
тренировками. Ошибки дело не страшное. Самое главное —
понимать.
124

Отметим, что в данном уроке рассмотрено только деление с


остатком. Деление без остатка мы рассмотрим в следующих
уроках. Сделано это с целью не усложнять обучение. Как
говорится, всему своё время.
125

Порядок действий

В уроке выражения мы узнали, что они бывают числовые и


буквенные. Мы рассмотрели несколько числовых и
буквенных выражений. Это были самые простейшие
выражения.

Настало время сдвинуться с мёртвой точки и рассмотреть


более сложные выражения. В данном уроке мы
познакомимся с порядком выполнения действий.

Выражения могут состоять из нескольких чисел. Таковыми к


примеру являются следующие выражения:

10 − 1 + 2 + 3
(3 + 5) + 2 × 3
5 × 2 + (5 − 3) : 2 + 1

Такие выражения нельзя вычислить сразу, то есть поставить


знак равенства и записать значение выражения. Да и
выглядят они не так просто, как 2 + 2 или 9 − 3.

Для подобных выражений принято соблюдать так


называемый порядок действий. Суть в том, что выражение
вычисляется кусочками по определённому порядку.

Когда нам требуется решить подобные примеры, мы сразу


должны мысленно прочитать следующее правило:

Сначала вычислить то, что находится в скобках!

Посмотрим на выражение 10 − 1 + 2 + 3. Видим, что в нём


нет никаких скобок. Тогда переходим к следующему правилу,
которое выглядит так:
126

Читаем выражение слева направо. Если встретится


умножение или деление, то сразу же выполняем эту
операцию!

Читаем наше выражение 10 − 1 + 2 + 3 слева направо.


Видим, что в нём нет никакого умножения или деления. Тогда
переходим к следующему правилу:

Читаем выражение слева направо. Если встретится


сложение или вычитание, то сразу же выполняем эту
операцию!

Читаем наше выражение 10 − 1 + 2 + 3 слева направо.


Встречаем вычитание 10 − 1. Сразу выполняем эту
операцию: 10 − 1 = 9. Полученную девятку запишем в
главном выражении вместо 10 − 1

Затем снова читаем те, правила, которые мы прочитали


выше. Читать их нужно в следующем порядке:

1. Сначала вычислить то, что находится в скобках!


127

2. Читаем выражение слева направо. Если встретится


умножение или деление, то сразу же применяем эту
операцию!

3. Читаем выражение слева направо. Если встретится


сложение или вычитание, то сразу же применяем эту
операцию!

Сейчас у нас имеется выражение 9 + 2 + 3 Читаем его слева


направо и встречаем сложение 9 + 2. Выполняем эту
операцию: 9 + 2 = 11. Запишем число 11 в главном
выражении вместо 9 + 2:

Осталось простейшее выражение 11 + 3, которое


вычисляется легко:

11 + 3 = 14

Таким образом, значение выражения 10 − 1 + 2 + 3 равно 14

10 − 1 + 2 + 3 = 14
128

Иногда удобно расставить порядок действий над самим


выражением. Для этого над операцией, которую необходимо
выполнить, указывают её очередь. К примеру, в выражении
10 − 1 + 2 + 3 все действия выполняются последовательно
слева направо, поэтому для него можно определить
следующий порядок:

И далее можно выполнить действия по отдельности, что


очень удобно:

1)  10 − 1 = 9

2)   9 + 2 = 11

3)  11 + 3 = 14

Также, можно поставить знак равенства и сразу начать


вычислять выражение в порядке приоритета действий.
Например, решение для выражения 10 − 1 + 2 + 3 можно
записать следующим образом:

Но если человек не научился быстро считать в уме, то не


рекомендуется использовать такой способ.
129

Пример 2. Найти значение выражения (3 + 5) + 2 × 3

Применим правила порядка действий. Прочитаем правила в


порядке их приоритета.

Сначала вычислить то, что находится в скобках!

Посмотрим на выражение (3 + 5) + 2 × 3. Видим, что в нём


есть выражение в скобках (3 + 5). Вычислим то, что в этих
скобках: 3 + 5 = 8. Запишем полученную восьмёрку в главном
выражении вместо выражения в скобках:

8+2×3

Снова читаем первое правило:

Сначала вычислить то, что находится в скобках!

Видим, что в выражении 8 + 2 × 3 нет никаких скобок. Тогда


читаем следующее правило:

Читаем выражение слева направо. Если встретится


умножение или деление, то сразу же выполняем эту
операцию!

Посмотрим на наше выражение 8 + 2 × 3. Видим, что в нём


есть умножение 2 × 3. Выполним эту операцию: 2 × 3 = 6.
Запишем полученную шестёрку в главном выражении вместо
2 × 3

8+6

Осталось простейшее выражение 8 + 6, которое вычисляется


легко:
130

8 + 6 = 14

Таким образом, значение выражения (3 + 5) + 2 × 3 равно 14

(3 + 5) + 2 × 3 = 14

Также, этот пример можно решить, расставив порядок


действий над самим выражением. Действие в скобках будет
первым действием, умножение — вторым действием, а
сумма — третьим:

И далее можно выполнить действия по отдельности, что


очень удобно:

1)  3 + 5 = 8

2)   2 × 3 = 6

3)  8 + 6 = 14

Также, можно поставить знак равенства и сразу начать


вычислять выражение в порядке приоритета действий:

Но опять же, используя такой способ, нужно быть очень


внимательным.
131

Пример 3. Найти значение выражения 5 × 2 + (5 − 3) : 2 + 1

Расставим порядок действий над выражением. Действие в


скобках будет первым действием, умножение — вторым
действием, деление — третьим действием,  четвёртое и
пятое действие являются суммами и они будут выполнены в
порядке их следования:

1)  5 − 3 = 2

2)  5 × 2 = 10

3)  2 : 2 = 1

4)  10 + 1 = 11

5)  11 + 1 = 12

Также, можно поставить знак равенства и сразу начать


вычислять выражение в порядке приоритета действий:

Четвёртое и пятое действие заключалось в том, чтобы


вычислить оставшееся простейшее выражение 10 + 1 + 1.
Мы не стали тратить время на выполнение каждого из этих
действий, а поставили знак равенства и записали ответ 12.
132

Пример 4. Найти значение выражения (3250 − 2905) : 5

Расставим порядок действий над выражением. Действие в


скобках будет первым действием, а деление — вторым

1)  3250 − 2905 = 345

2)  345 : 5 = 69

В скобках могут выполняться два и более действия. Бывает


даже так, что в скобках встречаются другие скобки. В таких
случаях нужно применять те же правила, которые мы
изучили ранее.

Пример 5. Найти значение выражения (6 411 × 8 − 40799) × 6


133

Расставим порядок действий над выражением. Действие в


скобках будет первым действием. При этом в скобках
выполняется умножение и вычитание. Согласно порядку
действий, умножение выполняется раньше вычитания.

В данном случае сначала нужно 6 411 умножить на 8, и из


полученного результата вычесть 40 799. Полученный
результат будет значением выражения, содержащегося в
скобках. Этот результат будет умножен на 6.

В результате будем иметь следующий порядок:

1)  6 411 × 8 = 51 288

2)  51 288 − 40 799 = 10 489

3)  10 489 × 6 = 62 934


134

Пример 6. Найти значение выражения:


1 657 974 : 822 × 106 − (50 377 + 20 338)

Расставим порядок действий над выражением. Действие в


скобках будет первым действием, деление будет вторым
действием, умножение — третьим, вычитание — четвёртым.

1) 50 377 + 20 338 = 70 715

2) 1 657 974 : 822 = 2 017
135

3) 2 017 × 106 = 213 802

4) 213 802−70 715 = 143 087

Пример 7. Найти значение выражения:


14 026 − (96 : 4 + 3680)

Расставим порядок действий над выражением. Действие в


скобках будет первым действием. При этом в скобках
136

выполняется деление и сложение. Согласно порядку


действий деление выполняется раньше сложения.

В данном случае сначала нужно 96 разделить на 4, и


полученный результат сложить с 3 680. Полученный
результат будет значением выражения, содержащегося в
скобках. Этот результат нужно вычесть из 14 026. В
результате будем иметь следующий порядок:

1) 96 : 4 = 24

2) 24 + 3 680 = 3 704

3) 14026 − 3 704 = 10 322


137
138

Законы математики

В нашей жизни есть законы, которые надо соблюдать.


Соблюдение законов гарантирует стабильность и
гармоничное развитие. Несоблюдение же законов приводит к
печальным последствиям.

У математики есть свои законы, которые тоже следует


соблюдать. Несоблюдение законов математики приводит в
лучшем случае к тому, что оценка учащегося снижается, а в
худшем случае приводит к тому, что падают самолёты,
зависают компьютеры, улетают крыши домов от сильного
ветра, снижается качество связи и тому подобные нехорошие
явления.

Законы математики состоят из простых свойств. Эти


свойства нам знакомы со школы. Но не мешает вспомнить их
ещё раз, а лучше всего записать или выучить наизусть.

В данном уроке мы рассмотрим лишь малую часть законов


математики. Их нам будет достаточно для дальнейшего
изучения математики.

Содержание урока

 Переместительный закон сложения


 Сочетательный закон сложения
 Переместительный закон умножения
 Сочетательный закон умножения
 Распределительный закон умножения
 Задания для самостоятельного решения
139

Переместительный закон сложения

Переместительный закон сложения говорит о том, что от


перестановки мест слагаемых сумма не изменяется.
Действительно, прибавьте пятерку к двойке — получите
семёрку. И наоборот, прибавьте двойку к пятерке — опять
получите семёрку:

5+2=7

2+5=7

Если положить на одну чашу весов 10 килограмм яблок и на


другую чашу так же положить 10 килограмм яблок, то весы
выровнятся, и не важно, что яблоки в пакетах лежат
вразброс. Если мы возьмём пакет с весов и перемешаем
яблоки находящиеся в нём, словно шары в лотерейном
мешке, пакет всё так же будет весить 10 килограмм. От
перестановки мест слагаемых сумма не изменится.
Слагаемые в данном случае это яблоки, а сумма это
итоговый вес.

Таким образом,  между выражениями 5 + 2 и 2 + 5 можно


поставить знак равенства. Это будет означать, что их сумма
равна:

5+2=2+5

7=7

Полагаем что, вы изучили один из предыдущих уроков,


который назывался выражения, поэтому мы без тени
смущения запишем переместительный закон сложения с
помощью переменных:
140

a+b=b+a

Записанный переместительный закон сложения будет


работать для любых чисел. Например, возьмём любых два
числа. Пусть а = 2, b = 3. Мы присвоили переменным a и b
значения 2 и 3 соответственно. Эти значения отправятся в
главное выражение a + b = b + a и подставятся куда нужно.
Число 2 подставится вместо а, число 3 место b

Сочетательный закон сложения

Сочетательный закон сложения говорит о том, что результат


сложения нескольких слагаемых не зависит от порядка
действий. Этот закон позволяет группировать слагаемые для
удобства их вычислений.
141

Рассмотрим сумму из трёх слагаемых:

2+3+5

Чтобы вычислить данное выражение, можно сначала


сложить числа 2 и 3 и полученный результат сложить с
числом 5. Для удобства сумму чисел 2 и 3 можно заключить в
скобки, указывая тем самым, что эта сумма будет вычислена
в первую очередь:

2 + 3 + 5 = (2 + 3) + 5 = 5 + 5 = 10

Либо можно сложить числа 3 и 5, затем полученный


результат сложить с числом 2

2 + 3 + 5 = 2 + (3 + 5) = 2 + 8 = 10

Видно, что в обоих случаях получается один и тот же


результат.

Таким образом, между выражениями (2 + 3) + 5 и 2 + (3 + 5)


можно поставить знак равенства, поскольку они равны
одному и тому же значению:

(2 + 3) + 5 = 2 + (3 + 5)

10 = 10

Запишем сочетательный закон сложения с помощью


переменных:

(a + b) + c = a + (b + c)
142

Переместительный закон умножения

Переместительный закон умножения говорит о том, что если


множимое и множитель поменять местами, то произведение
не изменится. Давайте проверим так ли это. Умножим
пятерку на двойку, а затем наоборот двойку на пятерку.

5 × 2 = 10

2 × 5 = 10

В обоих случаях получается один и тот же результат,


поэтому между выражениями 5 × 2 и 2 × 5 можно поставить
знак равенства, поскольку они равны одному и тому же
значению:

5×2=2×5

10 = 10

Запишем переместительный закон умножения с помощью


переменных:

a×b=b×a

Для записи законов в качестве переменных необязательно


использовать именно буквы a и b. Можно использовать
любые другие буквы, например c и d или x и y. Тот же
переместительный закон умножения можно записать
следующим образом:

x×y=y×x
143

Сочетательный закон умножения

Сочетательный закон умножения говорит о том, что если


выражение состоит из нескольких сомножителей, то
произведение не будет зависеть от порядка действий.

Рассмотрим следующее выражение:

2 × 3 × 4

Данное выражение можно вычислять в любом порядке.


Сначала можно перемножить числа 2 и 3, и полученный
результат умножить на 4:

Либо сначала можно перемножить числа 3 и 4, и полученный


результат перемножить с числом 2
144

Таким образом, между выражениями (2 × 3) × 4 и 2 × (3 × 4)


можно поставить знак равенства, поскольку они равны
одному и тому же значению:

Запишем сочетательный закон умножения с помощью


переменных:

a × b × с = (a × b) × с = a × (b × с)

Пример 2. Найти значение выражения 1 × 2 × 3 × 4

Данное выражение можно вычислять в любом порядке.


Вычислим его слева направо в порядке следования
действий:

Распределительный закон умножения

Распределительный закон умножения позволяет умножить


сумму на число или число на сумму.
145

Рассмотрим следующее выражение:

(3 + 5) × 2

Мы знаем, что сначала надо выполнить действие в скобках.


Выполняем:

(3 + 5) = 8

В главном выражении (3 + 5) × 2 выражение в скобках


заменим на полученную восьмёрку:

8 × 2 = 16

Получили ответ 16. Этот же пример можно решить с


помощью распределительного закона умножения. Для этого
каждое слагаемое, которое в скобках, нужно умножить на 2,
затем сложить полученные результаты:

Мы рассмотрели распределительный закон умножения


слишком развёрнуто и подробно. В школе этот пример
записали бы очень коротко. К такой записи тоже надо
привыкать. Выглядит она следующим образом:

(3 + 5) × 2 = 3 × 2 + 5 × 2 = 6 + 10 = 16

Или ещё короче:


146

(3 + 5) × 2 = 6 + 10 = 16

Теперь запишем распределительный закон умножения с


помощью переменных:

(a + b) × c = a × c + b × c

Давайте внимательно посмотрим на начало этого


распределительного закона умножения. Начало у него
выглядит так: (a + b) × c.

Если рассматривать выражение в скобках (a + b), как единое


целое, то это будет множимое, а переменная с будет
множителем, поскольку соединены они знаком умножения ×

Из переместительного закона умножения мы узнали, что


если множимое и множитель поменять местами, то
произведение не изменится.

Если множимое (a + b) и множитель c поменять местами, то


получим выражение c × (a + b). Тогда получится, что мы
умножаем переменную c на сумму (a + b). Для выполнения
такого умножения, опять же применяется
распределительный закон умножения. В данном случае
переменную c нужно умножить на каждое слагаемое в
скобках:

c × (a + b) = c × a + c × b
147

Пример 2. Найти значение выражения 5 × (3 + 2)

Умножим число 5 на каждое слагаемое в скобках и


полученные результаты сложим:

5 × (3 + 2) = 5 × 3 + 5 × 2 = 15 + 10 = 25

Пример 3. Найти значение выражения 6 × (5 + 2)

Умножим число 6 на каждое слагаемое в скобках и


полученные результаты сложим:

6 × (5 + 2) = 6 × 5 + 6 × 2 = 30 + 12 = 42

Если в скобках располагается не сумма, а разность, то


сначала нужно умножить множимое на каждое число, которое
в скобках. Затем из полученного первого числа вычесть
второе число. В принципе, ничего нового.

Пример 4. Найти значение выражения 5 × (6 − 2)

Умножим 5 на каждое число в скобках. Затем из полученного


первого числа вычтем второе число:

5 × (6 − 2) = 5 × 6 − 5 × 2 = 30 − 10 = 20

Пример 5. Найти значение выражения 7 × (3 − 2)

Умножим 7 на каждое число в скобках. Затем из полученного


первого числа вычтем второе число:

7 × (3 − 2) = 7 × 3 − 7 × 2 = 21 − 14 = 7


148

Задания для самостоятельного решения


Задание 1. Найдите значение выражения, используя
распределительный закон умножения:
3 × (7 + 8)
Показать решение
Задание 2. Найдите значение выражения, используя
распределительный закон умножения:
5 × (6 + 8)
Показать решение
Задание 3. Найдите значение выражения, используя порядок
выполнения действий:
4 × (5 + 4) + 9 × (3 + 2)
Показать решение
Задание 4. Найдите значение выражения, используя
распределительный закон умножения:
4 × (5 + 4) + 9 × (3 + 2)
Показать решение
Задание 5. Найдите значение выражения, используя
распределительный закон умножения:
16 × (2 + 7) + 5 × (4 + 1)
Показать решение
149

Делители и кратные

В данном уроке мы рассмотрим такие понятия как делители


и кратные.

Содержание урока

 Что такое делитель?


 Кратные числа
 Признаки делимости чисел
 Чётные и нечётные числа
 Простые и составные числа
 Разложение составного числа на простые множители
 Нахождение делителей числа
 Задания для самостоятельного решения

Что такое делитель?

Мы знаем, что делитель это число, показывающее на сколько


частей нужно разделить делимое. Например, в выражении 8 :
2 = 4, делителем является число 2. Это число показывает на
сколько частей нужно разделить число 8. После разделения
получается ответ 4. Как видно из примера, число 8 делится
на число 2 без остатка. Говорят, что число 2 является
делителем числа 8.

Пример 1. Число 2 является делителем числа 8, поскольку 8


делится на 2 без остатка:

8:2=4

Пример 2. Число 3 является делителем числа 9, поскольку 9


делится на 3 без остатка:
150

9:3=3

Пример 3. Число 4 не является делителем числа 10


поскольку 10 не делится на 4 без остатка:

10 : 4 = 2 (2 в остатке)

Определение. Делителем числа а называется число, на


которое число а делится без остатка.

Данное определение содержит переменную a. Подставим


вместо этой переменной любое число, например число 12 и
прочитаем определение:

Делителем числа 12 называется число, на которое 12


делится без остатка.

Попробуем перечислить эти числа:

1, 2, 3, 4, 6, 12

Все эти числа являются делителями числа 12, поскольку


число 12 делится на них без остатка. Покажем это:

12 : 1 = 12
12 : 2 = 6
12 : 3 = 4
12 : 4 = 3
12 : 6 = 2
12 : 12 = 1
151

Кратные числа

Если какое-нибудь число без остатка разделилось на другое,


то его называют кратным этого числа. Например, 6 без
остатка делится на 3. Поэтому 6 является кратным числа 3

6:3=2

Определение. Кратным числа а называется число,


которое делится без остатка на а.

Данное определение содержит переменную a. Подставим


вместо этой переменной любое число, например число 5 и
прочитаем определение:

Кратным числа 5 называется число, которое делится без


остатка на 5.

У любого числа бесконечно много кратных. Например,


первыми кратными числа 5, являются числа 5, 10, 15, 20, 25.
Все они кратны 5, поскольку делятся на 5 без остатка:

5:5=1
10 : 5 = 2
15 : 5 = 3
20 : 5 = 4
25 : 5 = 5

Признаки делимости чисел

Признаки делимости чисел используются для того, чтобы


ускорить процесс деления чисел. Существует множество
признаков делимости и других интересных алгоритмов,
значительно ускоряющих решение и освобождающих от
152

излишней волокиты. Рассмотрим наиболее популярные из


них.

Признак делимости на 10

Любое число, которое оканчивается нулем, делится без


остатка на 10. Чтобы получить частное, достаточно
отбросить цифру 0 в делимом.

Например, 380 : 10 = 38. Мы просто отбросили последний


ноль в числе 380.

В случае, если мы имеем выражение такого вида 385 : 10, то


получится 38 и 5 в остатке, поскольку 380 : 10 = 38, а пятерка
это остаток, который не разделился.

Таким образом, если число оканчивается цифрой 0, то оно


делится без остатка на 10. Если же оно оканчивается другой
цифрой, то оно не делится без остатка на 10. Остаток в этом
случае равен последней цифре числа. Действительно, в
примере 385 : 10 = 38 (5 в остатке), остаток равен последней
цифре в числе 385, то есть пятерке.

Признак делимости на 5 и на 2

Любое число, которое оканчивается нулем, делится без


остатка и на 5, и на 2.

Примеры:

10 : 5 = 2

100 : 5 = 20

100 : 2 = 50
153

Признак делимости на 5

Если число оканчивается цифрой 0 или 5, то оно делится без


остатка на 5.

Примеры:

355 : 5 = 71

200 : 5 = 40

475 : 5 = 95

Признак делимости на 3

Число делится на 3, если сумма цифр этого числа делится на


3. Например, рассмотрим число 27, сумма его цифр 2 + 7 = 9.
Девять, как мы знаем делится на 3, значит и 27 делится на 3:

27 : 3 = 9

Признак делимости на 9

Число делится на 9, если сумма его цифр делится на 9.


Например, рассмотрим число 18. Сумма его цифр 1 + 8 = 9.
Девять делится на девять, значит и 18 делится на 9

18 : 9 = 2

Рассмотрим число 846. Сумма его цифр 8 + 4 + 6 = 18. 


Восемнадцать делится на девять, значит и 846 делится на 9:
154

Чётные и нечётные числа

Чётным называется число, которое делится без остатка на


2. Например, число 20 является четным, поскольку оно
делится без остатка на 2:

20 : 2 = 10

Нечётным называется число, если при его делении на 2,


остаётся остаток 1. Например число 21 является нечетным,
поскольку после его деления на 2 остается остаток 1:

21 : 2 = 10 (1 в остатке)

Как распознать чётное число от нечетного, не выполняя


деления на 2? Очень просто. Из однозначных чисел чётными
являются числа 0, 2, 4, 6, 8, а нечетными являются 1, 3, 5, 7,
9. Если число оканчивается чётной цифрой, то это число
является чётным. Если число оканчивается нечетной
цифрой, то это число является нечетным.

Например, число 308 чётно, поскольку оно оканчивается


чётной цифрой. Число 1024 тоже четно, поскольку
оканчивается четной цифрой.

А числа 305 и 1027 являются нечётными, поскольку они


оканчиваются нечётными цифрами.
155

Простые и составные числа

Простым называется число, которое делится без остатка на


единицу и на само себя. Другими словами, имеет только два
делителя. Например, число 5 делится без остатка на
единицу и на само себя:

5:1=5

5:5=1

Значит, число 5 является простым числом.

Составным же называется число, которое имеет два и


более делителя. Например, число 4 составное, поскольку у
него два и более делителя:  4, 2 и 1

4:4=1

4:2=2

4:1=4

Значит, число 4 является составным числом.

Разложение составного числа на простые множители

Любое составное число можно разложить на простые


множители. Чем-то похожим мы занимались в уроке замены
в выражениях. Из этого урока мы узнали, что любое число,
входящее в выражение, можно заменить на то же самое, но
записанное в другом виде.
156

Например, число 6 можно записать в виде суммы 4 + 2 или в


виде частного 12 : 2 или в виде произведения 2 × 3.
Последнюю запись 2 × 3 можно назвать разложением числа
6 на простые множители.

Суть разложения числа на простые множители заключается


в том, чтобы представить это число в виде произведения
нескольких простых множителей.

Разложим число 4 на простые множители. Для этого соберем


данное число из других чисел, при этом соединим их знаком
умножения (×). Число 4 состоит из чисел 2 и 2. Эти два числа
и являются простыми множителями, из которых состоит
число 4

4=2×2

Разложим на множители число 6. Число 6 можно собрать из


чисел 2 и 3. Эти два числа и являются простыми
множителями, из которых состоит число 6

6=2×3

Разложим на множители число 8. Это число можно


разложить на множители 2 и 4, при этом множитель 4 можно
разложить на два множителя: 2 и 2. Поэтому вместо четвёрки
записываем её разложение:
157

Большие числа раскладываются таким же образом. Сначала


их раскладывают на большие множители, затем эти большие
множители раскладывают на маленькие. И так до тех пор,
пока каждый множитель не станет простым числом.

Например, разложим число 180 на простые множители.


Число 180 это два множителя 18 и 10

180 = 18 × 10

Теперь раскладываем множители 18 и 10 на другие


множители:

18 = 3 × 6

 10 = 5 × 2

Теперь раскладываем выделенную синюю шестерку. Это


последний большой множитель, который можно разложить
на простые множители:

6=2×3

Теперь собираем все простые множители вместе:

На множители можно разложить только составное число.


Простое число на множители не раскладывается. Именно
поэтому, когда разложение доходит до простых чисел, мы
эти простые числа дальше не раскладываем.
158

Есть и второй способ разложения на простые множители. Он


проще и хорошо подходит для больших чисел. Суть этого
способа заключается в том, что сначала проводится
вертикальная линия. Затем слева от этой линии
записываются делимые, а справа — делители, которые
впоследствии собирают во множители.

При разложении числа этим способом, используют признаки


делимости, такие как: признаки делимости на 2, на 3, на 5 и
другие.

Например, разложим предыдущее число 180 этим способом.

Проводим вертикальную линию и слева записываем первое


делимое 180

Теперь применяем признаки делимости. В первую очередь


проверяем делится ли 180 на 2. Если делится, то нужно
записать эту двойку справа от вертикальной линии.

180 делится на 2, поскольку 180 оканчивается нулём.


Записываем двойку справа от вертикальной линии:
159

Теперь делим 180 на 2 и получаем второе делимое 90.


Записываем это делимое слева от вертикальной линии:

Теперь делим 90. Снова применяем признаки делимости.


Проверяем делится ли 90 на 2.

90 делится на 2, поскольку 90 оканчивается нулём.


Записываем двойку справа от вертикальной линии:
160

Теперь делим 90 на 2, получаем третье делимое 45.


Записываем это делимое слева от вертикальной линии:

Теперь делим 45. Снова применяем признаки делимости.


Проверяем делится ли 45 на 2.

45 на 2 не делится. Тогда проверяем делится ли 45 на 3.

45 делится на 3, поскольку сумма цифр 4 и 5 делится на 3.


Записываем тройку справа от вертикальной линии:
161

Делим 45 на 3, получаем четвёртое делимое 15. Записываем


это делимое слева от вертикальной линии:

Теперь делим 15. Проверяем делится ли 15 на 2.

15 не делится на 2. Тогда проверяем делится ли 15 на 3.

15 на 3 делится, поскольку сумма цифр 1 и 5 делится на 3.


Записываем тройку справа от вертикальной линии:
162

Делим 15 на 3, получаем пятое делимое 5. Записываем


пятёрку слева от вертикальной линии:

Теперь делим 5. Проверяем делится ли 5 на 2.

5 не делится на 2. Тогда проверяем делится ли 5 на 3.

5 не делится на 3. Тогда проверяем делится ли 5 на 5.

5 делится на 5. Записываем эту пятёрку справа от


вертикальной линии:
163

Делим 5 на 5, получаем шестое делимое 1. Записываем эту


единицу слева от вертикальной линии:

На этом деление завершается, поскольку мы достигли


единицы. Делители, которые записывают справа от
вертикальной линии должны быть простыми числами.
Поэтому, когда делимое 5 не разделилось на 2, а затем не
разделилось на 3, мы попробовали разделить его на 5, не
пробуя разделить на 4, поскольку 4 является не простым, а
составным числом.

Теперь переписываем в один ряд все делители, которые


записаны справа от вертикальной линии. Они и будут
разложением числа 180 на простые множители. Желательно
164

записывать их, начиная с самых малых. Это позволяет


упорядочить их по возрастанию:

Не расстраивайтесь, если будете испытывать затруднения


при разложении чисел на простые множители. Эта тема
требует немного практики. Для тренировки можете разложить
на простые множители следующие числа: 256,  378,  512.

Нахождение делителей числа

В начале данного урока было сказано, что делителем


называется число, на которое другое число делится без
остатка.

Например, число 2 является делителем числа 6, поскольку


число 6 можно без остатка разделить на 2

6:2=3

Ещё делителем числа 6 является число 3

6:3=2

Ещё делителем числа 6 является число 1


165

6:1=6

Наконец, делителем числа 6 является само это число

6:6=1

Перечислим все делители числа 6

1, 2, 3, 6

Иногда возникает необходимость найти все делители какого-


нибудь числа. Чтобы понять, как это делается, рассмотрим
несколько примеров.

Пример 1. Найти делители числа 12

Во-первых, единица является делителем любого числа.


Пусть и у нас первым делителем числа 12 будет 1

Теперь раскладываем число 12 на простые множители:

Получили разложение 2 × 2 × 3. 

В процессе разложения числа 12 на простые множители, мы


делили его на числа 2 и 3. На них число 12 разделилось без
остатка, значит они тоже являются делителями числа 12.
Внесём эти два числа в нашу таблицу делителей:
166

Чтобы получить остальные делители числа 12, нужно найти


все возможные произведения его простых множителей
между собой. Получаемые в результате ответы и будут
остальными делителями числа 12.

Число 12 мы разложили на простые множители 2 × 2 × 3.


Найдём все возможные произведения этих простых
множителей между собой. Первое произведение это 2 × 2.
Это произведение равно 4

2 × 2 = 4

Занесём число 4 в нашу таблицу делителей

Следующее возможное произведение из простых


множителей числа 12 это произведение 2 × 3. Данное
произведение равно 6. Занесём число 6 в нашу таблицу
делителей:
167

Последнее возможное произведение из простых множителей


числа 12 это произведение из всех его множителей, а
именно 2 × 2 × 3. Это произведение равно 12. Занесём число
12 в нашу таблицу делителей:

Таким образом, делителями числа 12 являются числа 1, 2, 3,


4, 6, 12.

На основании приведённого примера можно сформировать


правило для нахождения делителей числа:

Чтобы найти делители числа, нужно:

 записать в качестве первого делителя единицу;


 разложить исходное число на простые множители и
выписать из полученных простых множителей те
множители, которые являются делителями
168

исходного числа (если множитель повторяется, то


выписать его нужно только один раз); 
 найти все возможные произведения полученных
простых множителей между собой. Получаемые в
результате ответы будут остальными делителями
исходного числа.

Пример 2. Найти делители числа 6

Первым делителем числа 6 запишем единицу:

Теперь разложим число 6 на простые множители:

Выпишем из полученного разложения те множители, которые


являются делителями числа 6. Видим, что это множители 2 и
3. Они будут следующими делителями числа 6. Допишем их к
нашим делителям:

1, 2, 3

Теперь найдём все возможные произведения простых


множителей числа 6. В данном случае имеется только одно
произведение, а именно 2 × 3. Это произведение равно 6.
Допишем число 6 к нашим делителям:

1, 2, 3, 6

Таким образом, делителями числа 6 являются числа 1, 2, 3,


6.
169

Задания для самостоятельного решения


Задание 1. Разложите число 256 на простые множители
Показать решение
Задание 2. Разложите число 52 на простые множители
Показать решение
Задание 3. Разложите число 98 на простые множители
Показать решение
Задание 4. Разложите число 116 на простые множители
Показать решение
Задание 5. Разложите число 228 на простые множители
Показать решение
170

НОД и НОК

Продолжаем изучать деление. В данном уроке мы


рассмотрим такие понятия, как НОД и НОК.

НОД — это наибольший общий делитель.

НОК — это наименьшее общее кратное.

Тема довольно скучная, но разобраться в ней нужно


обязательно. Не понимая этой темы, не получится
эффективно работать с дробями, которые являются
настоящей преградой в математике.

Содержание урока

 Наибольший общий делитель


 Второй способ нахождения НОД
 Третий способ нахождения НОД
 Нахождение НОД для нескольких чисел
 Наименьшее общее кратное
 Второй способ нахождения НОК
 Третий способ нахождения НОК
 Задания для самостоятельного решения

Наибольший общий делитель

Определение. Наибольшим общим делителем чисел a и


b называется наибольшее число, на которое a и b
делятся без остатка.

Чтобы хорошо понять это определение, подставим вместо


переменных a и b любые два числа. Например, вместо
171

переменной a подставим число 12, а вместо переменной b


— число 9. Теперь попробуем прочитать это определение:

Наибольшим общим делителем чисел 12 и 9 называется


наибольшее число, на которое 12 и 9 делятся без
остатка.

Из определения понятно, что речь идёт об общем делителе


чисел 12 и 9. Причем делитель является наибольшим из всех
существующих делителей. Этот наибольший общий
делитель (НОД) нужно найти.

Для нахождения наибольшего общего делителя двух чисел,


используется три способа. Первый способ довольно
трудоёмкий, но зато позволяет хорошо понять суть темы и
прочувствовать весь ее смысл.

Второй и третий способы довольны просты и дают


возможность быстро найти НОД. Рассмотрим все три
способа. А какой применять на практике — выбирать вам.

Первый способ заключается в поиске всех возможных


делителей двух чисел и в выборе наибольшего из них.
Рассмотрим этот способ на следующем примере: найти
наибольший общий делитель чисел 12 и 9.

Сначала найдём все возможные делители числа 12. Для


этого разделим 12 на все делители в диапазоне от 1 до 12.
Если делитель позволит разделить 12 без остатка, то мы
будем выделять его синим цветом и в скобках делать
соответствующее пояснение.
172

12 : 1 = 12
(12 разделилось на 1 без остатка, значит 1 является
делителем числа 12)

12 : 2 = 6
(12 разделилось на 2 без остатка, значит 2 является
делителем числа 12)

12 : 3 = 4
(12 разделилось на 3 без остатка, значит 3 является
делителем числа 12)

12 : 4 = 3
(12 разделилось на 4 без остатка, значит 4 является
делителем числа 12)

12 : 5 = 2 (2 в остатке)
(12 не разделилось на 5 без остатка, значит 5 не является
делителем числа 12)

12 : 6 = 2
(12 разделилось на 6 без остатка, значит 6 является
делителем числа 12)

12 : 7 = 1 (5 в остатке)
(12 не разделилось на 7 без остатка, значит 7 не является
делителем числа 12)

12 : 8 = 1 (4 в остатке)
(12 не разделилось на 8 без остатка, значит 8 не является
делителем числа 12)
173

12 : 9 = 1 (3 в остатке)
(12 не разделилось на 9 без остатка, значит 9 не является
делителем числа 12)

12 : 10 = 1 (2 в остатке)
(12 не разделилось на 10 без остатка, значит 10 не является
делителем числа 12)

12 : 11 = 1 (1 в остатке)
(12 не разделилось на 11 без остатка, значит 11 не является
делителем числа 12)

12 : 12 = 1
(12 разделилось на 12 без остатка, значит 12 является
делителем числа 12)

Теперь найдём делители числа 9. Для этого проверим все


делители от 1 до 9

9:1=9
(9 разделилось на 1 без остатка, значит 1 является
делителем числа 9)

9 : 2 = 4 (1 в остатке)
(9 не разделилось на 2 без остатка, значит 2 не является
делителем числа 9)

9:3=3
(9 разделилось на 3 без остатка, значит 3 является
делителем числа 9)

9 : 4 = 2 (1 в остатке)
(9 не разделилось на 4 без остатка, значит 4 не является
делителем числа 9)
174

9 : 5 = 1 (4 в остатке)
(9 не разделилось на 5 без остатка, значит 5 не является
делителем числа 9)

9 : 6 = 1 (3 в остатке)
(9 не разделилось на 6 без остатка, значит 6 не является
делителем числа 9)

9 : 7 = 1 (2 в остатке)
(9 не разделилось на 7 без остатка, значит 7 не является
делителем числа 9)

9 : 8 = 1 (1 в остатке)
(9 не разделилось на 8 без остатка, значит 8 не является
делителем числа 9)

9:9=1
(9 разделилось на 9 без остатка, значит 9 является
делителем числа 9)

Теперь выпишем делители обоих чисел. Числа выделенные


синим цветом и являются делителями. Их и выпишем:

Выписав делители, можно сразу определить какой является


наибольшим и общим.

Согласно определению, наибольшим общим делителем


чисел 12 и 9, является число, на которое 12 и 9 делятся без
остатка. Наибольшим и общим делителем чисел 12 и 9
является число 3
175

И число 12 и число 9 делятся на 3 без остатка:

12 : 3 = 4

9  : 3 = 3

Значит НОД (12 и 9) = 3

Второй способ нахождения НОД

Теперь рассмотрим второй способ нахождения наибольшего


общего делителя. Суть данного способа заключается в том,
чтобы разложить оба числа на простые множители и
перемножить общие из них.

Пример 1. Найти НОД чисел 24 и 18

Сначала разложим оба числа на простые множители:

Теперь перемножим их общие множители. Чтобы не


запутаться, общие множители можно подчеркнуть.
176

Смотрим на разложение числа 24. Первый его множитель это


2. Ищем такой же множитель в разложении числа 18 и видим,
что он там тоже есть. Подчеркиваем обе двойки:

Снова смотрим на разложение числа 24. Второй его


множитель тоже 2. Ищем такой же множитель в разложении
числа 18 и видим, что его там второй раз уже нет. Тогда
ничего не подчёркиваем.

Следующая двойка в разложении числа 24 также отсутствует


в разложении числа 18.

Переходим к последнему множителю в разложении числа 24.


Это множитель 3. Ищем такой же множитель в разложении
числа 18 и видим, что там он тоже есть. Подчеркиваем обе
тройки:

Итак, общими множителями чисел 24 и 18 являются


множители 2 и 3. Чтобы получить НОД, эти множители
необходимо перемножить:
177

2 × 3 = 6

Значит НОД (24 и 18) = 6

Третий способ нахождения НОД

Теперь рассмотрим третий способ нахождения наибольшего


общего делителя. Суть данного способа заключается в том,
что числа подлежащие поиску наибольшего общего делителя
раскладывают на простые множители. Затем из разложения
первого числа вычеркивают множители, которые не входят в
разложение второго числа. Оставшиеся числа в первом
разложении перемножают и получают НОД.

Пример 1. Найти НОД чисел 28 и 16.

В первую очередь, раскладываем числа 28 и 16 на простые


множители:

Получили два разложения:   и 

Теперь из разложения первого числа вычеркнем множители,


которые не входят в разложение второго числа. В
разложение второго числа не входит семёрка. Её и
вычеркнем из первого разложения:
178

Теперь перемножаем оставшиеся множители и получаем


НОД:

Число 4 является наибольшим общим делителем чисел 28 и


16. Оба этих числа делятся на 4 без остатка:

28 : 4 = 7

16 : 4 = 4

 НОД (28 и 16) = 4

Пример 2. Найти НОД чисел 100 и 40

Раскладываем на множители число 100

Раскладываем на множители число 40


179

Получили два разложения: 2 × 2 × 5 × 5 и 2 × 2 × 2 × 5

Теперь из разложения первого числа вычеркнем множители,


которые не входят в разложение второго числа. В
разложение второго числа не входит одна пятерка (там
только одна пятёрка). Её и вычеркнем из первого разложения

Перемножим оставшиеся числа:

Получили ответ 20. Значит число 20 является наибольшим


общим делителем чисел 100 и 40. Эти два числа делятся на
20 без остатка:

100 : 20 = 5

40 : 20 = 2

 НОД (100 и 40) = 20.

Пример 3. Найти НОД чисел 72 и 128

Раскладываем на множители число 72


180

Раскладываем на множители число 128

Получили два разложения: 2 × 2 × 2 × 3 × 3 и


2 × 2 × 2 × 2 × 2 × 2 × 2.

Теперь из разложения первого числа вычеркнем множители,


которые не входят в разложение второго числа. В
разложение второго числа не входят две тройки (там их
вообще нет). Их и вычеркнем из первого разложения:

Перемножим оставшиеся числа:


181

Получили ответ 8. Значит число 8 является наибольшим


общим делителем чисел 72 и 128. Эти два числа делятся на
8 без остатка:

72 : 8 = 9

128 : 8 = 16

 НОД (72 и 128) = 8

Нахождение НОД для нескольких чисел

Наибольший общий делитель можно находить и для


нескольких чисел, а не только для двух. Для этого числа,
подлежащие поиску наибольшего общего делителя,
раскладывают на простые множители, затем находят
произведение общих простых множителей этих чисел.

Например, найдём НОД для чисел 18,  24  и  36

Разложим на множители число 18

Разложим на множители число 24


182

Разложим на множители число 36

Получили три разложения:

Теперь найдём и подчеркнём общие множители:


183

Мы видим, что общие множители для чисел 18, 24 и 36 это


множители 2 и 3. Эти множители входят во все три
разложения. Перемножив эти множители, мы получим НОД,
который ищем:

2×3=6

Получили ответ 6. Значит число 6 является наибольшим


общим делителем чисел 18, 24 и 36. Эти три числа делятся
на 6 без остатка:

18 : 6 = 3

24 : 6 = 4

36 : 6 = 6

 НОД (18, 24 и 36) = 6

Пример 2. Найти НОД для чисел 12, 24, 36 и 42

Разложим на простые множители каждое число. Затем


найдём произведение общих простых множителей.

Разложим на множители число 12

Разложим на множители число 24


184

Разложим на множители число 36

Разложим на множители число 42

Получили четыре разложения:


185

Теперь найдём и подчеркнём общие множители:

Мы видим, что общие множители для чисел 12, 24, 36, и 42


это множители 2 и 3. Перемножив эти множители, мы
получим НОД, который ищем:

2×3=6

Получили ответ 6. Значит число 6 является наибольшим


общим делителем чисел 12, 24, 36 и 42. Эти числа делятся
на 6 без остатка:

12 : 6 = 2

24 : 6 = 4
186

36 : 6 = 6

42 : 6 = 7

 НОД (12, 24 , 36 и 42) = 6

Наименьшее общее кратное

Из предыдущего урока мы знаем, что если какое-то число без


остатка разделилось на другое, его называют кратным этого
числа.

Оказывается, кратное может быть общим у нескольких


чисел. И сейчас нас будет интересовать кратное двух чисел,
причем оно должно быть максимально маленьким.

Определение. Наименьшее общее кратное (НОК) чисел a


и b — это наименьшее число, которое кратно a и b.
Другими словами, это такое маленькое число, которое
делится без остатка на число a и число b.

Определение содержит две переменные a и b. Давайте


подставим вместо этих переменных любые два числа.
Например, вместо переменной a подставим число 9, а
вместо переменной b подставим число 12. Теперь попробуем
прочитать определение:

Наименьшее общее кратное (НОК) чисел 9 и 12 — это


наименьшее число, которое кратно 9 и 12. Другими
словами, это такое маленькое число, которое делится
без остатка на число 9 и на число 12.
187

Из определения понятно, что наименьшее общее кратное это


наименьшее число, которое делится без остатка на 9 и на 12.
Это наименьшее общее кратное требуется найти.

Для нахождения наименьшего общего кратного (НОК) можно


пользоваться тремя способами. Первый способ заключается
в том, что можно выписать первые кратные двух чисел, а
затем выбрать среди этих кратных такое число, которое
будет общим для обоих чисел и маленьким. Давайте
применим этот способ.

В первую очередь, найдем первые кратные для числа 9.


Чтобы найти кратные для 9, нужно эту девятку поочерёдно
умножить на числа от 1 до 9. Получаемые ответы будут
кратными для числа 9.

Итак, начнём. Кратные будем выделять синим цветом:

Теперь находим кратные для числа 12. Для этого поочерёдно


умножим число 12 на все числа 1 до 12:
188

Теперь выпишем кратные обоих чисел:

Теперь найдём общие кратные обоих чисел. Найдя, сразу


подчеркнём их:
189

Общими кратными для чисел 9 и 12 являются кратные 36 и


72. Наименьшим же из них является 36.

Значит наименьшее общее кратное для чисел 9 и 12 это


число 36. Данное число делится на 9 и 12 без остатка:

36 : 9 = 4

36 : 12 = 3

НОК (9 и 12) = 36

Второй способ нахождения НОК

Второй способ заключается в том, что числа для которых


ищется наименьшее общее кратное раскладываются на
простые множители. Затем выписываются множители,
входящие в первое разложение, и добавляют недостающие
множители из второго разложения. Полученные множители
перемножают и получают НОК.
190

Применим данный способ для предыдущей задачи. Найдём


НОК для чисел 9 и 12.

Разложим на множители число 9

Разложим на множители число 12

Выпишем первое разложение:

Теперь допишем множители из второго разложения, которых


нет в первом разложении. В первом разложении нет двух
двоек. Их и допишем:

Теперь перемножаем эти множители:

Получили ответ 36. Значит наименьшее общее кратное


чисел 9 и 12 это число 36. Данное число делится на 9 и 12
без остатка:

36 : 9 = 4
191

36 : 12 = 3

НОК (9 и 12) = 36

Говоря простым языком, всё сводится к тому, чтобы


организовать новое разложение куда входят оба разложения
сразу. Разложением первого числа 9 являлись множители 3 и
3, а разложением второго числа 12 являлись множители 2, 2
и 3.

Наша задача состояла в том, чтобы организовать новое


разложение куда входило бы разложение числа 9 и
разложение числа 12 одновременно. Для этого мы выписали
разложение первого числа и дописали туда множители из
второго разложения, которых не было в первом разложении.
В результате получили новое разложение 3 × 3 × 2 × 2.
Нетрудно увидеть воочию, что в него одновременно входят
разложение числа 9 и разложение числа 12

Пример 2. Найти НОК чисел 50 и 180


192

Разложим на множители число 50

Разложим на множители число 180

Выпишем первое разложение:

Теперь допишем множители из второго разложения, которых


нет первом разложении. В первом разложении нет ещё
одной двойки и двух троек. Их и допишем:

Теперь перемножаем эти множители:

Получили ответ 900. Значит наименьшее общее кратное


чисел 50 и 180 это число 900. Данное число делится на 50 и
180 без остатка:
193

900 : 50 = 18

900 : 180 = 5

НОК (50 и 180) = 900

Пример 3. Найти НОК чисел 8, 15 и 33

Разложим на множители число 8

Разложим на множители число 15

Разложим на множители число 33

Выпишем первое разложение:

Теперь допишем множители из второго и третьего


разложения, которых нет первом разложении. Допишем
194

множители 3 и 5 из второго разложения, и множитель 11 из


третьего разложения:

Теперь перемножаем эти множители:

Получили ответ 1320. Значит наименьшее общее кратное


чисел 8, 15 и 33 это число 1320. Данное число делится на 8,
15 и 33 без остатка:

1320 : 8 = 165

1320 : 15 = 88

1320 : 33 = 40

НОК (8, 15 и 33) = 1320

Третий способ нахождения НОК

Есть и третий способ нахождения наименьшего общего


кратного. Он работает при условии, что его ищут для двух
чисел и при условии, что уже найден наибольший общий
делитель этих чисел.

Данный способ разумнее использовать, когда одновременно


нужно найти НОД и НОК двух чисел.

К примеру, пусть требуется найти НОД и НОК чисел 24 и 12.


Сначала найдем НОД этих чисел:
195

Теперь для нахождения наименьшего общего кратного чисел


24 и 12, нужно перемножить эти два числа и полученный
результат разделить на их наибольший общий делитель.

Итак, перемножим числа 24 и 12

Разделим полученное число 288 на НОД чисел 24 и 12

Получили ответ 24. Значит наименьшее общее кратное


чисел 24 и 12 равно 24
196

НОК (24 и 12) = 24

Пример 2. Найти НОД и НОК чисел 36 и 48

Найдем НОД чисел 36 и 48

Перемножим числа 36 и 48

Разделим 1728 на НОД чисел 36 и 48


197

Получили 144. Значит наименьшее общее кратное чисел 36 и


48 равно 144

НОК (36 и 48) = 144

Для проверки можно найти НОК обычным вторым способом,


которым мы пользовались ранее. Если мы всё сделали
правильно, то должны получить 144

Не расстраивайтесь, если сразу не научитесь находить НОД


и НОК. Главное понимать, что это такое и как оно работает. А
ошибки вполне естественны на первых порах. Как говорят:
«На ошибках учимся».
198

Задания для самостоятельного решения


Задание 1. Найдите НОД чисел 12 и 16
Показать решение
Задание 2. Найдите НОК чисел 12 и 16
Показать решение
Задание 3. Найдите НОД чисел 40 и 32
Показать решение
Задание 4. Найдите НОК чисел 40 и 32
Показать решение
Задание 5. Найдите НОД чисел 54 и 86
Показать решение
Задание 6. Найдите НОК чисел 54 и 86
Показать решение
Задание 7. Найдите НОД чисел 98 и 35
Показать решение
Задание 8. Найдите НОК чисел 98 и 35
Показать решение
Задание 9. Найдите НОД чисел 112 и 82
Показать решение
Задание 10. Найдите НОК чисел 112 и 82
Показать решение
Задание 11. Найдите НОД чисел 24, 48, 64
Показать решение
199

Задание 12. Найдите НОК чисел 24, 48, 64


Показать решение
Задание 13. Найдите НОД чисел 18, 48, 96
Показать решение
Задание 14. Найдите НОК чисел 18, 48, 96
Показать решение
Задание 15. Найдите НОД чисел 28, 24, 76
Показать решение
Задание 16. Найдите НОК чисел 28, 24, 76
Показать решение
200

Дроби

Дроби это тема об которую спотыкается половина жителей


нашей планеты. Если спросить у людей с какой темы у них
начались проблемы с математикой, то большинство из них
ответят — с дробей.

Этих людей нельзя упрекнуть. Дроби действительно тема не


из простых. Тема дробей требует много терпения и
внимания, особенно если человек изучает её впервые.

Но есть и хорошие новости. Если вы наберётесь терпения и


освоите дроби, то уверяем, что дальнейшее изучение
математики станет для вас простым и интересным.

А если вы ещё хорошо изучили предыдущий урок, который


назывался деление, то можете быть уверены, что дроби вы
освоили уже наполовину.

Содержание урока

 Что такое дробь?


 Дробь означает деление
 Выделение целой части дроби
 Перевод смешанного числа в неправильную дробь
 Основное свойство дроби
 Сокращение дробей
 Второй способ сокращения дроби
 Задания для самостоятельного решения
201

Что такое дробь?

Если говорить простым языком, то дробь это часть чего-


либо. Это «чего-либо» может быть чем угодно — едой,
деньгами, числом. В народе дробь называют долей. Само
слово «дробь» тоже говорит за себя — дробь означает
дробление, деление, разделение.

Рассмотрим пример из жизни. Мы купили себе пиццу, чтобы


съесть её в течении дня. Допустим мы решили разделить её
на четыре части, чтобы съедать постепенно по одному
кусочку.

Посмотрите на этот рисунок. Представьте, что это наша


пицца, разделённая на четыре куска. Каждый кусок пиццы
это и есть дробь, потому что каждый кусок по отдельности
это часть пиццы.

Допустим мы съели один кусок. Как его записать? Очень


просто. Сначала рисуется маленькая линия:
202

Внизу этой линии записывается на сколько кусков пицца


была разделена. Пицца была разделена на четыре куска.
Значит внизу линии записывается четвёрка:

А сверху этой линии записывается сколько кусков пиццы


было съедено. Съеден был один кусок, значит сверху
записываем единицу:

Такие записи называют дробями. Дробь состоит из


числителя и знаменателя.

Число, которое записывается сверху, называется


числителем дроби.

Число, которое записывается снизу, называется


знаменателем дроби.

В нашем примере числитель дроби это единица, а


знаменатель дроби — четвёрка. Эту дробь можно прочитать
так: «одна четвёртая» либо «один кусок из
четырёх» либо «одна четвёртая доля» либо «четверть»
— всё это синонимы.

Теперь представьте, что мы съели ещё один кусок той же


самой пиццы, которая была разделена на четыре куска. Как
записать такую дробь?
203

Очень просто. Сверху записываем 2 (поскольку уже съедено


два куска), а внизу записываем 4 (поскольку всего кусков
было 4):

Эта дробь читается так: «две четвёртых» либо «два куска


из четырёх» либо «две четвёртые доли».

Теперь представьте, что пиццу мы разделили не на четыре


части, а на три.

Допустим мы съели один кусок этой пиццы. Как записать


такую дробь?

Очень просто. Опять же рисуется маленькая линия. Внизу


этой линии записывается число 3, поскольку пицца
разделена на три части, а сверху этой линии записывается
число 1, поскольку съеден один кусок:
204

Эта дробь читается так: «Одна третья» либо «Один кусок


из трёх» либо «Одна третья доля» либо «Треть».

Если мы съедим два куска пиццы, то такая дробь будет


называться «две третьих» и записываться следующим
образом:

Теперь представьте, что пиццу мы разделили на две части,


или как говорят в народе: «Пополам»:

Допустим, из этих двух кусков мы съели один кусок. Как


записать такую дробь?

Опять же рисуем линию. Внизу этой линии записываем число


2, поскольку пицца разделена на две части, а вверху
записываем число 1, поскольку съеден один кусок:
205

Эта дробь читается так: «одна вторая» либо «один кусок из


двух» либо «одна вторая доля» либо «половина».

Дроби, которые мы сейчас рассмотрели, называют


обыкновенными.

Вообще, дроби бывают двух видов: обыкновенные и


десятичные. На данный момент мы рассматриваем
обыкновенные дроби. Обыкновенная дробь это дробь,
которая состоит из числителя и знаменателя. Десятичные
дроби рассмотрим немного позже.

Знаменатель дроби — это число, которое показывает на


сколько равных частей можно что-либо разделить. Вернёмся
к нашей пицце. Поровну эта пицца может быть разделена и
на 2 части и на 3, и на 4, и на 5, и на 6. В зависимости от
того, на сколько частей мы будем делить пиццу, знаменатель
будет меняться.

На следующем рисунке представлены три пиццы, которые


разделены по разному. У первой пиццы знаменателем будет
2. У второй пиццы знаменателем будет 3. У третьей пиццы
знаменателем будет 4.
206

Числитель же показывает сколько частей взято от чего-либо.


К примеру, если разделить пиццу на две части, как на первом
рисунке, и взять одну часть для трапезы, то получится что

мы взяли (одну часть из двух), или как говорят в народе


«половину» пиццы.

С помощью переменных дробь можно записать так:

где a — это числитель, b — знаменатель.

Следующая вещь, которую важно знать это то, что


обыкновенные дроби бывают правильными и
неправильными.

Правильная дробь — это дробь, у которой числитель


меньше знаменателя. Например, следующие дроби
являются правильными:
207

Почему такие дроби называют правильными? Вспомним, что


дробь это часть чего-либо. Ведь будет логичнее, если эта
часть будет меньше того, откуда эта часть была взята.
Например, если пицца разделена на четыре части, и мы

возьмём (одну четвёртую), то наш кусок будет меньше,


чем все четыре куска вместе взятые (чем одна целая пицца).
Поэтому такие дроби называют правильными.

С неправильной дробью всё с точностью наоборот.


Неправильная дробь — это дробь, у которой числитель
больше знаменателя. Например, следующие дроби являются
неправильными:

Видно, что у этих дробей числитель больше


знаменателя. Почему же такие дроби называют
неправильными? Вспомним, что дробь это часть чего-либо.
Знаменатель показывает на сколько частей это чего-либо
разделено. А числитель показывает сколько этого чего-либо
взяли.

Теперь возьмём к примеру неправильную дробь   и


применим её к нашей пицце. В знаменателе стоит 2, значит
пицца разделена на две части, а в числителе стоит 9.
Получается, что взято девять кусков из двух. Но как можно
взять девять кусков, если их всего два? Ответ — никак.
Поэтому такие дроби называют неправильными.

Дробь, у которой числитель и знаменатель одинаковые, тоже


называют неправильной. Например:
208

Вообще, такие дроби даже не должны называться дробями.

И вот почему. Рассмотрим к примеру дробь . Применим её к


нашей пицце.

Допустим, мы хотим съесть пиццы.  В знаменателе стоит


число 2, значит пицца разделена на две части. И в числителе
стоит 2, значит взято две части. По сути, взята вся целая

пицца, и если мы съедим эту пиццы, то съедим не часть


пиццы, а всю пиццу целиком. Иными словами, съедим не
дробь, а целую часть пиццы. Поэтому дробь, у которой
числитель и знаменатель одинаковые, называют
неправильной.

Дробь означает деление

Черта в дроби, которая отделяет числитель от знаменателя,


означает деление. Она говорит, что числитель можно
разделить на знаменатель.

Например, рассмотрим дробь . Дробная черта говорит, что


четвёрку можно разделить на двойку. Мы знаем, что четыре
разделить на два будет два. Ставим знак равенства (=) и
записываем ответ:
209

Можно сделать вывод, что любое деление чисел можно


записать с помощью дробей. Например:

Это простейшие примеры. Видно, что у них отсутствует


остаток. С остатком немного сложнее, зато интереснее.
Поговорим об этом в следующей теме, которая называется
«выделение целой части дроби».

Выделение целой части дроби

Вычислим дробь . Пять разделить на два будет два и один


в остатке:

5 : 2 = 2 (1 в остатке)

Проверка: (2 × 2) + 1 = 4 + 1 = 5

Но сейчас мы имеем дело с дробями, значит и отвечать надо


в дробном виде. Чтобы хорошо понять, как это делается,
рассмотрим пример из жизни.
210

Представьте, что у вас есть 5 яблок и вы решили поделиться


ими со своим другом. Причём поделиться по-честному, чтобы
каждому досталось поровну. Как разделить эти 5 яблок?

Очевидно, что каждому из вас достанется по два яблока, а


оставшееся одно яблоко вы разрежете ножом пополам и
тоже разделите между собой:

Посмотрите внимательно на этот рисунок. На нём показано,


как пять яблок разделены между вами и вашим другом.
Очевидно, что каждому досталось по два целых яблока и по
половинке яблока.
211

Теперь возвращаемся к дроби и отвечаем на её вопрос.


Сколько будет пять разделить на два? Смотрим на наш
рисунок и отвечаем: если пять яблок разделить на двоих, то
каждому достанется два целых яблока и половинка яблока.
Так и записываем:

Схематически это выглядит так:

Процедуру, которую мы сейчас провели, называют


выделением целой части дроби.
212

В нашем примере мы выделили целую часть дроби   и

получили новую дробь .  Такую дробь называют


смешанной. Смешанная дробь — это дробь, у которой есть
целая часть и дробная.

В нашем примере целая часть это 2, а дробная часть это

Обязательно запомните эти понятия! А лучше запишите в


свою рабочую тетрадь.

Выделить целую часть можно только у неправильных


дробей. Напомним, что неправильная дробь это дробь, у
которой числитель больше знаменателя. Например,
следующие дроби являются неправильными, и у них
выделена целая часть:
213

Чтобы выделить целую часть, достаточно знать, как делить

числа уголком. Например, выделим целую часть у дроби .


Записываем уголком данное выражение и решаем:

После того, как решение примера завершается, новую дробь


собирают подобно детскому конструктору. Важно понимать,
что куда относить. Частное относят к целой части, остаток
относят в числитель дробной части, делитель относят в
знаменатель дробной части.

В принципе, если вы хорошо знаете таблицу умножения, и


можете быстро в уме выполнять элементарные вычисления,
то можно обойтись без записей уголком. В школах кстати,
именно этого и требуют — чтобы учащиеся не тратили время
на простые операции, а сразу записывали ответы.

Но если вы только начинаете изучать математику, советуем


записывать каждую мелочь.

Рассмотрим ещё один пример на выделение целой части.

Пусть требуется выделить целую часть дроби 


214

Записываем уголком данное выражение и решаем. Потом


собираем смешанную дробь:

Получили:

Перевод смешанного числа в неправильную дробь

Любое смешанное число получается в результате выделения


целой части в неправильной дроби. Например, рассмотрим

неправильную дробь . Если выделить в ней целую часть, то

получается

Но возможен и обратный процесс — любое смешанное число


можно перевести в неправильную дробь. Для этого целую
часть надо умножить на знаменатель дробной части и
полученный результат прибавить к числителю дробной
части. Полученный результат будет числителем новой
дроби, а знаменатель останется без изменений.

Например, переведём смешанное число в неправильную


дробь. Умножаем целую часть 2 на знаменатель дробной
части:
215

2×3=6

Затем к 6 прибавляем числитель дробной части:

6+1=7

Полученная семёрка будет числителем новой дроби, а


знаменатель 3 останется без изменений:

Подробное решение выглядит так:

А с помощью переменных перевод смешанного числа в


неправильную дробь можно записать так:

Пример 2. Перевести смешанное число в неправильную


дробь.

Умножаем целую часть смешанного числа на знаменатель


дробной части и прибавляем к числителю дробной части, а
знаменатель оставляем без изменений:
216

Основное свойство дроби

Основное свойство дроби говорит о том, что если числитель


и знаменатель дроби умножить или разделить на одно и то
же число, то получится равная ей дробь. Это означает, что
значение дроби не изменится.

Например, рассмотрим дробь .  Умножим её числитель и


знаменатель на одно и то же число, например на число 2

Получили новую дробь  .  Если верить основному свойству

дроби, то дроби   и  равны между собой. Так ли это?


Давайте проверим, нарисовав эти дроби в виде кусочков
пиццы:
217

Посмотрите внимательно на эти два рисунка. Первый

рисунок иллюстрирует дробь (один кусок из двух), а второй

иллюстрирует дробь   (два куска из четырёх). Если хорошо


присмотреться на эти куски, то можно убедиться, что у них
одинаковые размеры. Различие лишь в том, что разделаны
они по-разному. Первая пицца была разделана на два куска,
и с неё взяли один кусок. А вторая пицца была разделана на
четыре куска, и с неё взяли два куска.

Поэтому между дробями и  можно поставить знак


равенства (=), поскольку они равны одному и тому же
значению:

Теперь испытаем основное свойство дроби, разделив


числитель и знаменатель на одно и то же число.

Рассмотрим дробь . Давайте разделим её числитель и


знаменатель на одно и то же число, например на число 2

Получили новую дробь . Если верить основному свойству

дроби, то дроби  и равны между собой. Так ли это?


Давайте проверим,  нарисовав эти дроби в виде кусочков
пиццы:
218

Посмотрите внимательно на эти два рисунка. Первый

рисунок иллюстрирует дробь  (четыре куска из восьми), а

второй иллюстрирует дробь  (два куска из четырёх). Если


хорошо присмотреться на эти куски, то можно убедиться, что
у них одинаковые размеры. Различие лишь в том, что
разделаны они по-разному. Первая пицца была разделана на
восемь кусков, и с неё взяли четыре куска. А вторая пицца
была разделана на четыре куска, и с неё взяли два куска.

Поэтому между дробями  и  можно поставить знак


равенства (=), поскольку они равны одному и тому же
значению:
219

Теперь мы полностью проверили, как работает основное


свойство дроби, и убедились, что работает оно
замечательно.

Число, на которое умножается числитель и знаменатель,


называется дополнительным множителем. Запомните это
обязательно!

Сокращение дробей

Дроби можно сокращать. Сократить — значит сделать дробь

короче и проще для восприятия. Например, дробь выглядит

намного проще и красивее, чем дробь .

Если при решении примеров получается большая и


некрасивая дробь, то нужно попытаться её сократить.

Сокращение дроби опирается на основное свойство дроби.


Поэтому, прежде чем изучать сокращение дробей,
обязательно изучите основное свойство дроби.

Деление числителя и знаменателя на их наибольший общий


делитель называется сокращением дроби.

Пример 1. Сократить дробь

Итак, нужно разделить числитель и знаменатель дроби на


наибольший общий делитель чисел 2 и 4.
220

В данном случае дробь простая и для неё НОД ищется легко.


НОД чисел 2 и 4 это число 2. Значит, числитель и

знаменатель дроби  надо разделить на 2

В результате дробь обратилась в более простую дробь .


Значение исходной дроби при этом не изменилось, поскольку
сокращение подразумевает деление числителя и
знаменателя на одно и то же число. А это действие, как было
указано ранее, не меняет значение дроби.

На рисунке представлены дроби и в виде кусочков пиццы.


До сокращения и после сокращения они имеют одинаковые
размеры. Разница лишь в том, что раздéланы они по-
разному.
221

Пример 2. Сократим дробь

Чтобы сократить дробь , нужно числитель и знаменатель


этой дроби разделить на наибольший общий делитель чисел
20 и 40.

НОД чисел 20 и 40 это число 20. Поэтому делим числитель и

знаменатель дроби на 20

Пример 3. Сократим дробь

Чтобы сократить дробь , нужно числитель и знаменатель


этой дроби разделить на наибольший общий делитель чисел
32 и 36.

НОД чисел 32 и 36 это число 4. Поэтому делим числитель и

знаменатель дроби на 4

Если в числителе и знаменателе располагаются простые


числа, то такую дробь сократить нельзя — она не
сокращается. Такие дроби называют несократимыми.
Например, следующие дроби являются несократимыми:
222

Напомним, что простыми называются числа, которые


делятся только на единицу и самих себя.

Второй способ сокращения дроби

Второй способ является короткой версией первого способа.


Суть его заключается в том, что пропускается подробное
разъяснение того, на что был разделён числитель и
знаменатель.

К примеру, вернёмся к дроби . Эту дробь мы сократили на


4, то есть разделили числитель и знаменатель этой дроби на
число 4

Теперь представьте, что в данном выражении отсутствует

конструкция , и сразу записан ответ . Получится


следующее выражение:

Суть в том что число, на которое разделили числитель и


знаменатель, хранят в уме. В нашем случае числитель и
знаменатель делят на 4 — это число и будем хранить в уме.
223

Сначала делим числитель на число 4. Полученный ответ


записываем рядом с числителем, предварительно зачеркнув
его:

Затем таким же образом делим знаменатель на число 4.


Полученный ответ записываем рядом со знаменателем,
предварительно зачеркнув его:

Затем собираем новую дробь. В числитель отправляем


новое число 8 вместо 32, а в знаменатель отправляем новое
число 9 вместо 36

Происходит своего рода замена одной дроби на другую.


Значение новой дроби равно значению предыдущей дроби,
поскольку срабатывает основное свойство дроби, которое
говорит о том что если числитель и знаменатель дроби
умножить или разделить на одно и то же число, то получится
равная ей дробь.
224

Также, дроби можно сокращать, предварительно разложив


на простые множители числитель и знаменатель.

Например, сократим дробь , предварительно разложив на


простые множители числитель и знаменатель:

Итак, мы разложили числитель и знаменатель дроби  на


множители. Теперь применяем второй способ сокращения. В
числителе и в знаменателе выбираем по множителю и делим
выбранные множители на НОД этих множителей.

Давайте сократим по тройке в числителе и в знаменателе.


Для этого разделим эти тройки на 3 (на их наибольший
общий делитель). Получим следующее выражение:

Сократить можно ещё по тройке в числителе и в


знаменателе:

Дальше сокращать больше нéчего. Последнюю тройку в


знаменателе просто так сократить нельзя, поскольку в
числителе нет множителя, который можно было бы сократить
вместе с этой тройкой.

Записываем новую дробь, в числителе и в знаменателе


которой будут новые множители.
225

 Получили ответ . Значит, при сокращении дроби

получается новая дробь .

Не рекомендуется пользоваться вторым способом


сокращения дроби и способом разложения на простые
множители числителя и знаменателя, если человек только
нáчал изучать математику. Практика показывает, что это
оказывается сложным на первых этапах.

Поэтому, если испытываете затруднения при использовании


второго способа, то пользуйтесь старым добрым способом
сокращения: делите числитель и знаменатель дроби на их
наибольший общий делитель. Выражение в таком случае
получается простым, понятным и красивым. Так,
предыдущий пример может быть решён старым способом и
будет выглядеть так:

Сравните это выражение с выражением, которое мы


получили, когда пользовались вторым способом:

Первое выражение намного понятнее, аккуратнее и короче.


Не правда ли?
226

Задания для самостоятельного решения


Задание 1. Запишите в виде дроби следующий рисунок:

Показать решение
Задание 2. Запишите в виде дроби следующий рисунок:

Показать решение
Задание 3. Запишите в виде дроби следующий рисунок:
227

Показать решение
Задание 4. Запишите в виде дроби следующий рисунок:

Показать решение
Задание 5. Запишите в виде дроби следующий рисунок:
228

Показать решение
Задание 6. Выделите целые части в следующих дробях:

Показать решение
Задание 7. Выделите целые части в следующих дробях:

Показать решение
Задание 8. Переведите смешанные дроби в неправильные:

Показать решение
Задание 9. Переведите смешанные дроби в неправильные,
не расписывая как целая часть умножается на знаменатель
дробной части и полученный результат складывается с
числителем дробной части

Показать решение
Задание 10. Сократите следующую дробь на 3

Показать решение
Задание 11. Сократите следующую дробь на 3 вторым
способом
229

Показать решение
Задание 12. Сократите следующую дробь на 5

Показать решение
Задание 13. Сократите следующую дробь на 5 вторым
способом

Показать решение
Задание 14. Сократите следующие дроби:

Показать решение
Задание 15. Сократите следующие дроби вторым способом:

Показать решение
Задание 16. Запишите в виде дроби следующий рисунок:

Показать решение
230

Задание 17. Запишите в виде дроби следующий рисунок:

Показать решение
Задание 18. Запишите в виде дроби следующий рисунок:

Показать решение
Задание 19. Запишите в виде дроби следующий рисунок:
231

Показать решение
Задание 20. Запишите в виде дроби следующий рисунок:

Показать решение
Задание 21. Изобразите в виде рисунка следующую дробь:

Показать решение
Задание 22. Изобразите в виде рисунка следующую дробь:

Показать решение
Задание 23. Изобразите в виде рисунка следующую дробь:
232

Показать решение
Задание 24. Изобразите в виде рисунка следующую дробь:

Показать решение
Задание 25. Изобразите в виде рисунка следующую дробь:

Показать решение
Задание 26. Изобразите в виде рисунка следующую дробь:

Показать решение
Задание 27. Изобразите в виде рисунка следующую дробь:

Показать решение
Задание 28. Изобразите в виде рисунка следующую дробь:

Показать решение
Задание 29. Изобразите в виде рисунка следующую дробь:

Показать решение
233

Действия с дробями

Дроби можно складывать, вычитать, умножать и делить.


Также, дроби можно сравнивать между собой. В принципе,
всё что можно делать с обычными числами, можно делать и
с дробями.

Содержание урока

 Сложение дробей с одинаковыми знаменателями


 Сложение дробей с разными знаменателями
 Вычитание дробей с одинаковыми знаменателями
 Вычитание дробей с разными знаменателями
 Умножение дроби на число
 Умножение дробей
 Представление целого числа в виде дроби
 Обратные числа
 Деление дроби на число
 Деление числа на дробь
 Деление дробей
 Задания для самостоятельного решения

Сложение дробей с одинаковыми знаменателями

Сложение дробей бывает двух видов:

1. Сложение дробей с одинаковыми знаменателями;


2. Сложение дробей с разными знаменателями.

Сначала изýчим сложение дробей с одинаковыми


знаменателями. Тут всё просто. Чтобы сложить дроби с
234

одинаковыми знаменателями, нужно сложить их числители, а


знаменатель оставить без изменения.

Например, слóжим дроби    и  . Складываем числители, а


знаменатель оставляем без изменения:

Этот пример можно легко понять, если вспомнить про пиццу,

которая разделена на четыре части. Если к   пиццы

прибавить пиццы, то получится пиццы:

Пример 2. Сложить дроби и .

Опять же складываем числители, а знаменатель оставляем


без изменения:
235

В ответе получилась неправильная дробь .  Если наступает


конец задачи, то от неправильных дробей принято
избавляться. Чтобы избавится от неправильной дроби, нужно
выделить в ней целую часть. В нашем случае целая часть
выделяется легко — два разделить на два будет один:

Этот пример можно легко понять, если вспомнить про пиццу,

которая разделена на две части. Если к пиццы прибавить

еще пиццы, то получится одна целая пицца:

Пример 3. Сложить дроби    и  .


236

Опять же складываем числители, а знаменатель оставляем


без изменения:

Этот пример можно легко понять, если вспомнить про пиццу,

которая разделена на три части. Если к пиццы прибавить

ещё пиццы, то получится пиццы:

Пример 4. Найти значение выражения 

Этот пример решается точно также, как и предыдущие.


Числители необходимо сложить, а знаменатель оставить без
изменения:
237

Попробуем изобразить наше решение с помощью рисунка.

Если к   пиццы прибавить   пиццы и ещё прибавить   пиццы,

то получится 1 целая и ещё   пиццы.

Как видите в сложении дробей с одинаковыми


знаменателями нет ничего сложного. Достаточно понимать
следующие правила:

1. Чтобы сложить дроби с одинаковыми знаменателями,


нужно сложить их числители, а знаменатель оставить
без изменения;
2. Если в ответе получилась неправильная дробь, то нужно
выделить в ней целую часть.

Сложение дробей с разными знаменателями

Теперь научимся складывать дроби с разными


знаменателями. Когда складывают дроби, знаменатели этих
дробей должны быть одинаковыми. Но одинаковыми они
бывают не всегда.
238

Например, дроби   и  сложить можно, поскольку у них


одинаковые знаменатели.

А вот дроби    и    сразу сложить нельзя, поскольку у этих


дробей разные знаменатели. В таких случаях дроби нужно
приводить к одинаковому (общему) знаменателю.

Существует несколько способов приведения дробей к


одинаковому знаменателю. Сегодня мы рассмотрим только
один из них, поскольку остальные способы могут показаться
сложными для начинающего.

Суть этого способа заключается в том, что сначала ищется


наименьшее общее кратное (НОК) знаменателей обеих
дробей. Затем НОК делят на знаменатель первой дроби и
получают первый дополнительный множитель. Аналогично
поступают и со второй дробью — НОК делят на знаменатель
второй дроби и получают второй дополнительный
множитель.

Затем числители и знаменатели дробей умножаются на свои


дополнительные множители. В результате этих действий,
дроби у которых были разные знаменатели, обращаются в
дроби, у которых одинаковые знаменатели. А как складывать
такие дроби мы уже знаем.

Пример 1. Сложим дроби   и 

У этих дробей разные знаменатели, поэтому нужно привести


их к одинаковому (общему) знаменателю.
239

В первую очередь находим наименьшее общее кратное


знаменателей обеих дробей. Знаменатель первой дроби это
число 3, а знаменатель второй дроби — число 2.
Наименьшее общее кратное этих чисел равно 6

НОК (2 и 3) = 6

Теперь возвращаемся к дробям   и . Сначала разделим


НОК на знаменатель первой дроби и получим первый
дополнительный множитель. НОК это число 6, а знаменатель
первой дроби это число 3. Делим 6 на 3, получаем 2.

Полученное число 2 это первый дополнительный множитель.


Записываем его к первой дроби. Для этого делаем
небольшую косую линию над дробью и записываем над ней
найденный дополнительный множитель:

Аналогично поступаем и со второй дробью. Делим НОК на


знаменатель второй дроби и получаем второй
дополнительный множитель. НОК это число 6, а знаменатель
второй дроби — число 2. Делим 6 на 2, получаем 3.

Полученное число 3 это второй дополнительный множитель.


Записываем его ко второй дроби. Опять же делаем
небольшую косую линию над второй дробью и записываем
над ней найденный дополнительный множитель:
240

Теперь у нас всё готово для сложения. Осталось умножить


числители и знаменатели дробей на свои дополнительные
множители:

Посмотрите внимательно к чему мы пришли. Мы пришли к


тому, что дроби у которых были разные знаменатели,
превратились в дроби у которых одинаковые знаменатели. А
как складывать такие дроби мы уже знаем. Давайте
дорешаем этот пример до конца:

Таким образом, пример завершается.  К   прибавить   

получается  .

Попробуем изобразить наше решение с помощью рисунка.

Если к   пиццы прибавить   пиццы, то получится одна целая


пицца и еще одна шестая пиццы:
241

Приведение дробей к одинаковому (общему) знаменателю


также можно изобразить с помощью рисунка. Приведя

дроби   и   к общему знаменателю, мы получили дроби   и  .


Эти две дроби будут изображаться теми же кусками пицц.
Различие будет лишь в том, что в этот раз они будут
разделены на одинаковые доли (приведены к одинаковому
знаменателю).

Первый рисунок изображает дробь   (четыре кусочка из

шести), а второй рисунок изображает дробь   (три кусочка из

шести). Сложив эти кусочки мы получаем   (семь кусочков из


шести). Эта дробь неправильная, поэтому мы выделили в
242

ней целую часть. В результате получили   (одну целую


пиццу и еще одну шестую пиццы).

Отметим, что мы с вами расписали данный пример слишком


подробно. В учебных заведениях не принято писать так
развёрнуто. Нужно уметь быстро находить НОК обоих
знаменателей и дополнительные множители к ним, а также
быстро умножать найденные дополнительные множители на
свои числители и знаменатели. Находясь в школе, данный
пример нам пришлось бы записать следующим образом:

Но есть и обратная сторона медали. Если на первых этапах


изучения математики не делать подробных записей, то
начинают появляться вопросы рода «а откуда вон та
цифра?», «почему дроби вдруг превращаются совсем в
другие дроби?«.

Поэтому на первых этапах советуем записывать каждую


мелочь. Хвастаться можно лишь в будущем, когда будут
усвоены азы.

Чтобы легче было складывать дроби с разными


знаменателями, можно воспользоваться следующей
пошаговой инструкцией:

1. Найти НОК знаменателей дробей;


2. Разделить НОК на знаменатель каждой дроби и
получить дополнительный множитель для каждой дроби;
3. Умножить числители и знаменатели дробей на свои
дополнительные множители;
243

4. Сложить дроби, у которых одинаковые знаменатели;


5. Если в ответе получилась неправильная дробь, то
выделить её целую часть;

Пример 2. Найти значение выражения .

Воспользуемся инструкцией, которая приведена выше.

Шаг 1. Найти НОК знаменателей дробей

Находим НОК знаменателей обеих дробей. Знаменатели


дробей это числа 2, 3 и 4

Шаг 2. Разделить НОК на знаменатель каждой дроби и


получить дополнительный множитель для каждой дроби

Делим НОК на знаменатель первой дроби. НОК это число 12,


а знаменатель первой дроби это число 2. Делим 12 на 2,
получаем 6. Получили первый дополнительный множитель 6.
Записываем его над первой дробью:

Теперь делим НОК на знаменатель второй дроби. НОК это


число 12, а знаменатель второй дроби это число 3. Делим 12
244

на 3, получаем 4. Получили второй дополнительный


множитель 4. Записываем его над второй дробью:

Теперь делим НОК на знаменатель третьей дроби. НОК это


число 12, а знаменатель третьей дроби это число 4. Делим
12 на 4, получаем 3. Получили третий дополнительный
множитель 3. Записываем его над третьей дробью:

Шаг 3. Умножить числители и знаменатели дробей на


свои дополнительные множители

Умножаем числители и знаменатели на свои


дополнительные множители:

Шаг 4. Сложить дроби у которых одинаковые


знаменатели

Мы пришли к тому, что дроби у которых были разные


знаменатели, превратились в дроби, у которых одинаковые
(общие) знаменатели. Осталось сложить эти дроби.
Складываем:
245

Сложение не поместилось на одной строке, поэтому мы


перенесли оставшееся выражение на следующую строку.
Это допускается в математике. Когда выражение не
помещается на одну строку, его переносят на следующую
строку, при этом надо обязательно поставить знак равенства
(=) на конце первой строки и в начале новой строки. Знак
равенства на второй строке говорит о том, что это
продолжение выражения, которое было на первой строке.

Шаг 5. Если в ответе получилась неправильная дробь, то


выделить в ней целую часть

У нас в ответе получилась неправильная дробь. Мы должны


выделить у неё целую часть. Выделяем:

Получили ответ

Вычитание дробей с одинаковыми знаменателями

Вычитание дробей бывает двух видов:


246

1. Вычитание дробей с одинаковыми знаменателями


2. Вычитание дробей с разными знаменателями

Сначала изучим вычитание дробей с одинаковыми


знаменателями.

Чтобы вычесть из одной дроби другую, нужно из числителя


первой дроби вычесть числитель второй дроби, а
знаменатель оставить без изменения.

Например, найдём значение выражения  . Чтобы


решить этот пример, надо из числителя первой дроби
вычесть числитель второй дроби, а знаменатель оставить
без изменения. Так и сделаем:

Этот пример можно легко понять, если вспомнить про пиццу,

которая разделена на четыре части. Если от  пиццы

отрезать    пиццы, то получится   пиццы:


247

Пример 2. Найти значение выражения .

Опять же из числителя первой дроби вычитаем числитель


второй дроби, а знаменатель оставляем без изменения:

Этот пример можно легко понять, если вспомнить про пиццу,

которая разделена на три части. Если от  пиццы отрезать   

пиццы, то получится  пиццы:


248

Пример 3. Найти значение выражения

Этот пример решается точно также, как и предыдущие. Из


числителя первой дроби нужно вычесть числители
остальных дробей:

В ответе получилась неправильная дробь. Выделим в ней


целую часть:

Как видите в вычитании дробей с одинаковыми


знаменателями ничего сложного нет. Достаточно понимать
следующие правила:

1. Чтобы вычесть из одной дроби другую, нужно из


числителя первой дроби вычесть числитель второй
дроби, а знаменатель оставить без изменения;
249

2. Если в ответе получилась неправильная дробь, то нужно


выделить в  ней целую часть.

Вычитание дробей с разными знаменателями

Теперь научимся вычитать дроби у которых разные


знаменатели. Когда вычитают дроби их знаменатели должны
быть одинаковыми. Но одинаковыми они бывают не всегда.

Например, от дроби  можно вычесть дробь , поскольку у

этих дробей  одинаковые знаменатели. А вот от дроби   

нельзя вычесть дробь , поскольку у этих дробей разные


знаменатели. В таких случаях дроби нужно приводить к
одинаковому (общему) знаменателю.

Общий знаменатель находят по тому же принципу, которым


мы пользовались при сложении дробей с разными
знаменателями. В первую очередь находят НОК
знаменателей обеих дробей. Затем НОК делят на
знаменатель первой дроби и получают первый
дополнительный множитель, который записывается над
первой дробью. Аналогично НОК делят на знаменатель
второй дроби и получают второй дополнительный
множитель, который записывается над второй дробью.

Затем дроби умножаются на свои дополнительные


множители. В результате этих операций, дроби у которых
были разные знаменатели, обращаются в дроби, у которых
250

одинаковые знаменатели. А как вычитать такие дроби мы


уже знаем.

Пример 1. Найти значение выражения:

У этих дробей разные знаменатели, поэтому нужно привести


их к одинаковому (общему) знаменателю.

Сначала находим НОК знаменателей обеих дробей.


Знаменатель первой дроби это число 3, а знаменатель
второй дроби — число 4. Наименьшее общее кратное этих
чисел равно 12

НОК (3 и 4) = 12

Теперь возвращаемся к дробям  и

Найдём дополнительный множитель для первой дроби. Для


этого разделим НОК на знаменатель первой дроби. НОК это
число 12, а знаменатель первой дроби — число 3. Делим 12
на 3, получаем 4. Записываем четвёрку над первой дробью:

Аналогично поступаем и со второй дробью. Делим НОК на


знаменатель второй дроби. НОК это число 12, а знаменатель
второй дроби — число 4. Делим 12 на 4, получаем 3.
Записываем тройку над второй дробью:
251

Теперь у нас всё готово для вычитания. Осталось умножить


дроби на свои дополнительные множители:

Мы пришли к тому, что дроби у которых были разные


знаменатели, превратились в дроби у которых одинаковые
знаменатели. А как вычитать такие дроби мы уже знаем.
Давайте дорешаем этот пример до конца:

Получили ответ

Попробуем изобразить наше решение с помощью рисунка.

Если от   пиццы отрезать   пиццы, то получится   пиццы


252

Это подробная версия решения. Находясь в школе, нам


пришлось бы решить этот пример покороче. Выглядело бы
такое решение следующим образом:

Приведение дробей   и   к общему знаменателю также


может быть изображено с помощью рисунка. Приведя эти

дроби к общему знаменателю, мы получили дроби   и .


Эти дроби будут изображаться теми же кусочками пицц, но в
этот раз они будут разделены на одинаковые доли
(приведены к одинаковому знаменателю):
253

Первый рисунок изображает дробь  (восемь кусочков из

двенадцати), а второй рисунок — дробь  (три кусочка из


двенадцати). Отрезав от восьми кусочков три кусочка мы

получаем пять кусочков из двенадцати. Дробь   и описывает


эти пять кусочков.

Пример 2. Найти значение выражения

У этих дробей разные знаменатели, поэтому сначала нужно


привести их к одинаковому (общему) знаменателю.

Найдём НОК знаменателей этих дробей.

Знаменатели дробей это числа 10, 3 и 5. Наименьшее общее


кратное этих чисел равно 30

НОК (10, 3, 5) = 30

Теперь находим дополнительные множители для каждой


дроби. Для этого разделим НОК на знаменатель каждой
дроби.

Найдём дополнительный множитель для первой дроби. НОК


это число 30, а знаменатель первой дроби — число 10.
Делим 30 на 10, получаем первый дополнительный
множитель 3. Записываем его над первой дробью:
254

Теперь находим дополнительный множитель для второй


дроби. Разделим НОК на знаменатель второй дроби. НОК
это число 30, а знаменатель второй дроби — число 3. Делим
30 на 3, получаем второй дополнительный множитель 10.
Записываем его над второй дробью:

Теперь находим дополнительный множитель для третьей


дроби. Разделим НОК на знаменатель третьей дроби. НОК
это число 30, а знаменатель третьей дроби — число 5.
Делим 30 на 5, получаем третий дополнительный множитель
6. Записываем его над третьей дробью:

Теперь всё готово для вычитания. Осталось умножить дроби


на свои дополнительные множители:

Мы пришли  к тому, что дроби у которых были разные


знаменатели, превратились в дроби у которых одинаковые
(общие) знаменатели. А как вычитать такие дроби мы уже
знаем. Давайте дорешаем этот пример.

Продолжение примера не поместится на одной строке,


поэтому переносим продолжение на следующую строку. Не
забываем про знак равенства (=) на новой строке:
255

В ответе получилась правильная дробь, и вроде бы нас всё


устраивает, но она слишком громоздка и некрасива. Надо бы
сделать её проще. А что можно сделать? Можно сократить
эту дробь.

Чтобы сократить дробь , нужно разделить её числитель и


знаменатель на наибольший общий делитель (НОД) чисел 20
и 30.

Итак, находим НОД чисел 20 и 30:

Теперь возвращаемся к нашему примеру и делим числитель

и знаменатель дроби  на найденный НОД, то есть на 10


256

Получили ответ

Умножение дроби на число

Чтобы умножить дробь на число, нужно числитель данной


дроби умножить на это число, а знаменатель оставить без
изменений.

Пример 1. Умножить дробь  на число 1.

Умножим числитель дроби на число 1

Запись можно понимать, как взять половину 1 раз. К

примеру, если пиццы взять 1 раз, то получится  пиццы

Из законов умножения мы знаем, что если множимое и


множитель поменять местами, то произведение не

изменится. Если выражение  , записать как  , то


257

произведение по прежнему будет равно  . Опять же


срабатывает правило перемножения целого числа и дроби:

Эту запись можно понимать, как взятие половины от


единицы. К примеру, если имеется 1 целая пицца и мы

возьмем от неё половину, то у нас окажется   пиццы:

Пример 2. Найти значение выражения

Умножим числитель дроби на 4

В ответе получилась неправильная дробь. Выделим в ней


целую часть:
258

Выражение  можно понимать, как взятие двух четвертей

4 раза. К примеру, если  пиццы взять 4 раза, то получится


две целые пиццы

А если поменять множимое и множитель местами, то

получим выражение  . Оно тоже будет равно 2. Это


выражение можно понимать, как взятие двух пицц от четырех
целых пицц:
259

Число, которое умножается на дробь, и знаменатель дроби


разрешается сокращать, если они имеют общий делитель,
бóльший единицы.

Например, выражение можно вычислить двумя


способами.

Первый способ. Умножить число 4 на числитель дроби, а


знаменатель дроби оставить без изменений:
260

Второй способ. Умножаемую четвёрку и четвёрку,

находящуюся в знаменателе дроби , можно сократить.


Сократить эти четвёрки можно на 4, поскольку наибольший
общий делитель для двух четвёрок есть сама четвёрка:

Получился тот же результат 3. После сокращения четвёрок,


на их месте образуются новые числа: две единицы. Но
перемножение единицы с тройкой, и далее деление на
единицу ничего не меняет. Поэтому решение можно записать
покороче:

Сокращение может быть выполнено даже тогда, когда мы


решили воспользоваться первым способом, но на этапе
перемножения числа 4 и числителя 3 решили
воспользоваться сокращением:

А вот к примеру выражение можно вычислить только

первым способом — умножить число 7 на числитель дроби ,


а знаменатель оставить без изменений:
261

Связано это с тем, что число 7 и знаменатель дроби не


имеют общего делителя, бóльшего единицы, и
соответственно не сокращаются.

Некоторые ученики по ошибке сокращают умножаемое число


и числитель дроби. Делать этого нельзя. Например,
следующая запись не является правильной:

Сокращение дроби подразумевает, что и числитель и


знаменатель будет разделён на одно и тоже число. В

ситуации с выражением деление выполнено только в

числителе, поскольку записать  это всё равно, что

записать . Видим, что деление выполнено только в


числителе, а в знаменателе никакого деления не происходит.

Умножение дробей

Чтобы перемножить дроби, нужно перемножить их числители


и знаменатели. Если в ответе получится неправильная
дробь, нужно выделить в ней целую часть.
262

Пример 1. Найти значение выражения .

Умножаем числитель первой дроби на числитель второй


дроби, а знаменатель первой дроби на знаменатель второй
дроби:

Получили ответ  . Желательно сократить данную дробь.

Дробь можно сократить на 2. Тогда окончательное решение


примет следующий вид:

Выражение   можно понимать, как взятие   пиццы от


половины пиццы. Допустим, у нас есть половина пиццы:
263

Как взять от этой половины две третьих? Сначала нужно


поделить эту половину на три равные части:

И взять от этих трех кусочков два:

У нас получится   пиццы. Вспомните, как выглядит пицца,


разделенная на три части:
264

Один кусок от этой пиццы и взятые нами два кусочка будут


иметь одинаковые размеры:

Другими словами, речь идет об одном и том же размере

пиццы. Поэтому значение выражения   равно 


265

Пример 2. Найти значение выражения

Умножаем числитель первой дроби на числитель второй


дроби, а знаменатель первой дроби на знаменатель второй
дроби:

В ответе получилась неправильная дробь. Выделим в ней


целую часть:

Пример 3. Найти значение выражения 


266

Умножаем числитель первой дроби на числитель второй


дроби, а знаменатель первой дроби на знаменатель второй
дроби:

В ответе получилась правильная дробь, но будет хорошо,


если её сократить. Чтобы сократить эту дробь, нужно
числитель и знаменатель данной дроби разделить на
наибольший общий делитель (НОД) чисел 105 и 450.

Итак, найдём НОД чисел 105 и 450:

Теперь делим числитель и знаменатель нашего ответа на


НОД, который мы сейчас нашли, то есть на 15

Представление целого числа в виде дроби

Любое целое число можно представить в виде дроби.

Например, число 5 можно представить как  . От этого


267

пятёрка своего значения не поменяет, поскольку выражение 

  означает «число пять разделить на единицу», а это, как


известно равно пятёрке:

Обратные числа

Сейчас мы познакомимся с очень интересной темой в


математике. Она называется «обратные числа».

Определение. Обратным к числу a называется число,


которое при умножении на a даёт единицу.

Давайте подставим в это определение вместо переменной a


число 5 и попробуем прочитать определение:

Обратным к числу 5 называется число, которое при


умножении на 5 даёт единицу.

Можно ли найти такое число, которое при умножении на 5,


даёт единицу? Оказывается можно. Представим пятёрку в
виде дроби:

Затем умножить эту дробь на саму себя, только поменяем


местами числитель и знаменатель. Другими словами,

умножим дробь на саму себя, только перевёрнутую:


268

Что получится в результате этого? Если мы продолжим


решать этот пример, то получим единицу:

Значит обратным к числу 5, является число , поскольку при

умножении 5 на получается единица.

Обратное число можно найти также для любого другого


целого числа.

Примеры:

 обратным числа 2 является дробь

 обратным  числа 3 является дробь

 обратным числа 4 является дробь

Найти обратное число можно также для любой другой дроби.


Для этого достаточно перевернуть её.

Примеры:

 для дроби обратной дробью является дробь 

 для для дроби обратной дробью является дробь

 для дроби обратной дробью является дробь


269

Деление дроби на число

Допустим, у нас имеется половина пиццы:

Разделим её поровну на двоих. Сколько пиццы достанется


каждому?

Видно, что после разделения половины пиццы получилось

два равных кусочка, каждый из которых составляет   пиццы.

Значит каждому достанется по   пиццы.


270

Деление дробей выполняется с помощью обратных чисел.


Обратные числа позволяют заменить деление умножением.

Чтобы разделить дробь на число, нужно эту дробь


умножить на число, обратное делителю.

Пользуясь этим правилом, запишем деление нашей


половины пиццы на две части.

Итак, требуется разделить дробь   на число 2. Здесь

делимым является дробь  , а делителем число 2.

Чтобы разделить дробь   на число 2, нужно эту дробь


умножить на число, обратное делителю 2. Обратное

делителю 2 это дробь  . Значит нужно умножить   на 

Получили ответ  . Значит при делении половины на две


части получается четверть.

Попробуем понять механизм этого правила. Для этого


рассмотрим следующий простейший пример. Пусть у нас
имеется одна целая пицца:
271

Умножим её на 2. То есть повторим её два раза (или возьмём


два раза). В результате будем иметь две пиццы:

Теперь угостим этими пиццами двоих друзей. То есть


разделим две пиццы на 2. Тогда каждому достанется по
одной пицце:
272

Разделить две пиццы на 2 это всё равно, что взять половину

от этих пицц, то есть умножить число 2 на дробь 

В обоих случаях получился один и тот же результат.

Тоже самое происходило, когда мы делили половину пиццы

на две части. Чтобы разделить   на 2, мы умножили эту


дробь на число, обратное делителю 2. А обратное делителю

2 это дробь 
273

Пример 2. Найти значение выражения 

Умножим первую дробь на число, обратное делителю:

Допустим, имеется четверть пиццы и нужно разделить её на


двоих:

Если разделить эту четверть на две части, то каждая


получившаяся часть будет одной восьмой частью целой
пиццы:
274

Заменять деление умножением можно не только при работе


с дробями, но и с обычными числами. Например, все мы
знаем, что 10 разделить на 2 будет 5

10 : 2 = 5

Заменим в этом примере деление умножением. Чтобы


разделить число 10 на число 2, можно умножить число 10 на

число, обратное числу 2. А обратное числу 2 это дробь 

Как видно результат не изменился. Мы снова получили ответ


5.

Можно сделать вывод, что деление можно заменять


умножением при условии, что вместо делителя будет
подставлено обратное ему число.

Пример 3. Найти значение выражения

Умножим первую дробь на число, обратное делителю.

Обратное делителю число это дробь 


275

Допустим, имелось пиццы:

Как разделить такую пиццу на шестерых? Если каждый из


трех кусков разделить пополам, то можно получить 6 равных
кусков

Эти шесть кусков являются шестью кусками из двенадцати. А

один из этих кусков составляет  . Поэтому при делении   на

6 получается 
276

Деление числа на дробь

Правило деления числа на дробь такое же, как и правило


деления дроби на число.

Чтобы разделить число на дробь, нужно умножить это


число на дробь, обратную делителю.

Например, разделим число 1 на .

Чтобы разделить число 1 на , нужно это число 1 умножить

на дробь, обратную дроби  . А обратная дроби   это дробь 

Выражение  можно понимать, как определение


количества половин в одной целой пицце. Допустим, имеется
одна целая пицца:
277

Если зададим вопрос «сколько раз половина содержится в


этой пицце», то ответом будет 2. Действительно, половина
содержится в одной целой пицце два раза

Пример 2. Найти значение выражение 


278

Умножим число 2 на дробь, обратную делителю. А обратная

делителю дробь это дробь 

Допустим, у нас имеются две целые пиццы:

Если зададим вопрос «сколько раз половина содержится в


двух пиццах», то ответом будет 4. Действительно, половина
содержится в двух пиццах четыре раза:
279

Деление дробей

Чтобы разделить дробь на дробь, нужно первую дробь


умножить на дробь, обратную второй.

Например, разделим   на 

Чтобы разделить   на  , нужно   умножить на дробь,

обратную дроби  . А обратная дроби   это дробь 

Допустим, имеется половина пиццы:


280

Если зададим вопрос «сколько раз четверть пиццы


содержится в этой половине», то ответом будет 2.
Действительно, четверть пиццы содержится в половине
пиццы два раза:

Пример 1. Найти значение выражения 


281

Умножаем первую дробь на дробь, обратную второй. Грубо


говоря, умножаем первую дробь на перевёрнутую вторую:

Пример 2. Найти значение выражения

Умножаем первую дробь на дробь обратную второй:

Здесь советуем остановиться и потренироваться. Решите


несколько примеров, приведенных ниже. Можете
использовать материалы сайта, как справочник. Это
позволит вам научиться работать с литературой.

Каждая следующая тема будет более сложной, поэтому


нужно тренироваться.

Задания для самостоятельного решения:


Задание 1. Найдите значение выражения:

Показать решение
Задание 2. Найдите значение выражения:

Показать решение
Задание 3. Найдите значение выражения:
282

Показать решение
Задание 4. Найдите значение выражения:

Показать решение
Задание 5. Найдите значение выражения:

Показать решение
Задание 6. Найдите значение выражения:

Показать решение
Задание 7. Найдите значение выражения:

Показать решение
Задание 8. Найдите значение выражения:

Показать решение
Задание 9. Найдите значение выражения:

Показать решение
Задание 10. Найдите значение выражения:
283

Показать решение
Задание 11. Найдите значение выражения:

Показать решение
Задание 12. Найдите значение выражения:

Показать решение
Задание 13. Найдите значение выражения:

Показать решение
Задание 14. Найдите значение выражения:

Показать решение
284

Смешанные числа

В предыдущих уроках было сказано, что дробь, состоящая из


целой и дробной части, называется смешанной.

Все дроби, имеющие целую и дробную часть, носят одно


общее название — смешанные числа.

Смешанные числа так же как и обыкновенные дроби можно


складывать, вычитать, умножать и делить. В данном уроке
мы рассмотрим каждое из этих действий по отдельности.

Содержание урока

 Сложение целого числа и правильной дроби


 Сложение смешанных чисел
 Сложение целого и смешанного числа
 Вычитание дроби из целого числа
 Вычитание смешанного числа из целого числа
 Вычитание смешанных чисел
 Умножение целого числа на дробь
 Умножение смешанного числа на дробь
 Умножение смешанных чисел
 Деление целого числа на дробь
 Деление дроби на целое число
 Деление целого числа на смешанное число
 Деление смешанного числа на целое число
 Деление смешанных чисел
 Задания для самостоятельного решения
285

Сложение целого числа и правильной дроби

Встречаются задачи, в которых требуется сложить целое


число и правильную дробь. Например, сложить число 2 и

дробь . Чтобы решить этот пример, нужно число 2

представить в виде дроби  . Затем сложить дроби с


разными знаменателями:

А теперь внимательно посмотрим на этот пример. Смотрим


на его начало и на его конец. Начало у него выглядит

так:  , а конец так:  . Различие в том, что в первом

случае число 2 и дробь  соединяются знаком сложения, а во


втором случае они записаны вместе. На самом деле это одно

и то же. Дело в том, что  это свёрнутая форма записи

смешанного числа, а  — развёрнутая.

Когда перед нами смешанное число вида , мы должны


понимать, что знак сложения опущен.

Какой можно сделать вывод? Если потребуется сложить


целое число и правильную дробь, можно опустить плюс и
записать целое число и дробь вместе.

Значит значение выражения равно 


286

Если к двум целым пиццам прибавить половину пиццы, то


получится две целые пиццы и ещё половина пиццы:

Пример 2. Найти значение выражения

Представим число 3 в виде дроби . Затем сложим дроби с


разными знаменателями:

Это первый способ. Второй способ намного проще. Можно


поставить знак равенства и записать целую и дробную часть
вместе. То есть опустить знак сложения:

Пример 3. Найти значение выражения 


287

Можно записать вместе число 2 и дробь , но этот ответ не

будет окончательным, поскольку в дроби  можно выделить


целую часть.

Поэтому в данном примере сначала нужно выделить целую

часть в дроби   . Пять вторых это две целых и одна вторая:

Теперь в главном выражении   вместо дроби   запишем

смешанное число 

Получили новое выражение  . В этом

выражении смешанное число  запишем в развёрнутом


виде:

Применим сочетательный закон сложения. Сложим две


двойки, получим 4:
288

Теперь свернём полученное смешанное число:

Это окончательный ответ. Подробное решение этого


примера можно записать следующим образом:

Сложение смешанных чисел

Встречаются задачи, в которых требуется сложить


смешанные числа. Например, найти значение выражения

. Чтобы решить этот пример, нужно целые и дробные


части сложить по отдельности.

Для начала запишем смешанные числа в развёрнутом виде:

Применим сочетательный закон сложения. Сгруппируем


целые и дробные части по отдельности:

Вычислим целые части: 2 + 3 = 5. В главном выражении


заменяем выражение в скобках (2 + 3) на полученную
пятёрку:
289

Теперь вычислим дробные части. Это сложение дробей с


разными знаменателями. Как складывать такие дроби мы
уже знаем:

Получили   . Теперь в главном выражении

заменяем дробные части на полученную дробь

Теперь свернем полученное смешанное число:

Таким образом, значение выражения  равно .


Попробуем изобразить это решение в виде рисунка. Если к
двум целым и половине пиццы прибавить три целые и одну
восьмую пиццы, то получится пять целых пицц и ещё пять
восьмых пиццы:
290

Подобные примеры нужно решать быстро, не


останавливаясь на подробностях. Находясь в школе, нам
пришлось бы записать решение этого примера следующим
образом:

Если в будущем увидите такое короткое решение, не


пугайтесь. Вы уже понимаете, что откуда взялось.

Пример 2. Найти значение выражения

Запишем смешанные числа в развёрнутом виде:


291

Сгруппируем целые и дробные части по отдельности:

Вычислим целые части: 5 + 3 = 8. В главном выражении


заменяем выражение в скобках (5 + 3) на полученное число 8

Теперь вычислим дробные части:

Получили смешанное число . Теперь в главном

выражении заменяем выражение в скобках на

полученное смешанное число 

Получили выражение . В данном случае число 8 надо

прибавить к целой части смешанного числа  . Для этого

смешанное число можно временно развернуть, чтобы


было понятнее, что с чем складывать:

Сложим целые части. Получаем 9


292

Сворачиваем готовый ответ:

Таким образом, значение выражения  равно  .

Полное решение этого примера выглядит следующим


образом:

Для решения подобных примеров существует универсальное


правило. Выглядит оно следующим образом:

Чтобы сложить смешанные числа, надо:

 привести дробные части этих чисел к общему


знаменателю;
 отдельно выполнить сложение целых и дробных
частей.

Если при сложении дробных частей получилась


неправильная дробь, выделить целую часть в этой
дроби и прибавить ее к полученной целой части.

Применение готовых правил допустимо в том случае, если


суть темы полностью понятна. Решение по-шаблону,
поглядывая в другие подобные примеры, приводит к
ошибкам на обнаружение которых уходит дополнительное
293

время. Поэтому, сначала разумнее понять тему, а затем


пользоваться готовым правилом.

Пример 3. Найти значение выражения 

Воспользуемся готовым правилом. Приведём дробные части


к общему знаменателю, затем по отдельности сложим целые
и дробные части:

Сложение целого и смешанного числа

Встречаются задачи, в которых нужно сложить целое и


смешанное число. Например, сложить 2 и смешанное число

. В этом случае целые части складываются отдельно, а


дробная часть остаётся без изменения:

Здесь смешанная дробь была развёрнута в ходе решения,


затем целые части были сгруппированы и сложены. В конце
целая и дробная части были свёрнуты. В результате

получили ответ .

Попробуем изобразить это решение в виде рисунка. Если к


двум целым пиццам прибавить три целые и треть пиццы, то
получятся пять целых и треть пиццы:
294

Пример 2. Найти значение выражения

В этом примере, как и в предыдущем, нужно сложить целые


части:

Осталось свернуть целую и дробную части, но дело в том,

что дробная часть    представляет собой неправильную


дробь. Сначала нужно выделить целую часть в этой
неправильной дроби. Затем целую часть этой дроби
прибавить к 4, а дробную часть оставить без изменения.
Продолжим данный пример на новой строке:
295

Вычитание дроби из целого числа

Встречаются задачи, в которых требуется вычесть дробь из

целого числа. Например, вычесть из числа 1 дробь  . Чтобы


решить такой пример, нужно целое число 1 представить в

виде дроби  , и выполнить вычитание дробей с разными


знаменателями:

Если имеется одна целая пицца и мы вычтем из неё


половину пиццы, то у нас получится половина пиццы:
296

Пример 2. Найти значение выражения .

Представим число 2 в виде дроби , и выполним вычитание


дробей с разными знаменателями:

Если имеются две целые пиццы и мы вычтем из низ


половину, то останется одна целая и половина пиццы:

Такие примеры можно решать в уме. Достаточно суметь


воспроизвести их в своём воображении. К примеру, найдём

значение выражения  , не приводя на бумаге никаких


вычислений.

Представим, что число 3 это три пиццы:


297

Нужно вычесть из них  . Мы помним, что треть выглядит


следующим образом:

Теперь представим, во что превратятся три пиццы, если


отрезать от них эту треть

Получилось   (две целых и две трети пиццы).


298

Чтобы убедиться в правильности решения, можно найти

значение выражения   обычным методом, представив


число 3 в виде дроби, и выполнив вычитание дробей с
разными знаменателями:

Пример 3. Найти значение выражения

Представим число 3 в виде дроби . Затем выполним


вычитание дробей с разными знаменателями:

Вычитание смешанного числа из целого числа

Теперь мы готовы к тому, чтобы вычесть смешанное число из

целого числа. Найдём значение выражения .

Чтобы решить этот пример, число 5 нужно представить в

виде дроби, а смешанное число перевести в неправильную

дробь. После перевода смешанного числа  в неправильную

дробь, получим дробь . Теперь выполним вычитание дробей


с разными знаменателями:
299

Если из пяти целых пицц вычесть одну целую и половину


пиццы, то останутся три целые пиццы и половина пиццы:

Пример 2. Найти значение выражения

Представим 6 в виде дроби , а смешанное число , в виде

неправильной дроби. После перевода смешанного числа   

в неправильную дробь, получим дробь . Теперь выполним


вычитание дробей с разными знаменателями:
300

Примеры на вычитание дроби из числа или вычитание


смешанной дроби из числа опять же можно выполнять в уме.
Этот процесс легко поддаётся воображению.

К примеру, если нужно быстро найти значение

выражения  , то вовсе необязательно представлять


число 2 в виде дроби и выполнять вычитание дробей с
разными знаменателями. Число 2 можно вообразить, как две
целые пиццы и далее представить, как от одной из них
отрезали две третьих (два куска из трёх)

Тогда от той пиццы, от которой отрезали  останется 


пиццы. Плюс одна из пицц останется нетронутой. Получится
одна целая пицца и треть пиццы:

Если на рисунке вы закроете рукой две третьих пиццы (она


закрашена), то сразу всё поймёте.
301

Вычитание смешанных чисел

Встречаются задачи, в которых требуется вычесть из одного


смешанного числа другое смешанное число. Например,

найдём значение выражения:

Чтобы решить этот пример, нужно смешанные числа  и 


перевести в неправильные дроби, затем выполнить
вычитание дробей с разными знаменателями:

Если от трёх целых пицц вычесть две целые и треть пиццы,


то останутся одна целая и одна шестая пиццы:
302

Пример 2. Найти значение выражения

Переводим смешанные числа и в неправильные дроби и


выполняем вычитание дробей с разными знаменателями:

К вычитанию смешанных чисел мы ещё вернёмся. В


вычитании дробей есть немало тонкостей, которым новичок
пока не готов. Например, возможен случай, когда
уменьшаемое может оказаться меньше вычитаемого. Это
может вывести нас в мир отрицательных чисел, которых мы
ещё не изучали.

А пока изучим умножение смешанных чисел. Благо оно не


такое сложное, как сложение и вычитание.

Умножение целого числа на дробь

Любое целое число можно умножить на дробь. Для этого


достаточно умножить это число на числитель дроби.

Например, умножим число 5 на дробь . Чтобы решить этот

пример, нужно число 5 умножить на числитель дроби

В ответе получилась неправильная дробь. Выделим в ней


целую часть:
303

Если имеются пять целых пицц и мы возьмём от этого


количества половину, то у нас окажется две целые пиццы и
половина пиццы:

Пример 2. Найти значение выражения


304

Умножим число 3 на числитель дроби 

В ответе получилась неправильная дробь , но мы выделили


её целую часть и получили 2.

Также, можно было сократить эту дробь. Получился бы тот


же результат. Выглядело бы это следующим образом:

Если имеются три целые пиццы и мы возьмём от этого


количества две третьих, то у нас окажется две целые пиццы:
305

Пример 3. Найти значение выражения

Этот пример решается так же, как и предыдущие. Целое


число и числитель дроби нужно перемножить:

Пример 4. Найти значение выражения


306

Умножим число 3 на числитель дроби 

Умножение смешанного числа на дробь

Чтобы умножить смешанное число на дробь, нужно


смешанное число перевести в неправильную дробь, затем
выполнить перемножение обыкновенных дробей.

Пример 1. Найти значение выражения 

Переведём смешанное число   в неправильную дробь.

После перевода это число превратится в дробь  . Затем

можно будет умножить эту дробь на 

Допустим, имеются одна целая и половина пиццы:


307

Умножить эти куски на   означает взять от них две трети.


Чтобы взять от них две трети, сначала разделим их на три
равные части. Разделим пополам ту пиццу, которая слева.
Тогда у нас получится три равных куска:

Теперь если мы возьмем   (два куска из трёх имеющихся), то


получим одну целую пиццу. Для наглядности закрасим эти
два куска:

Поэтому значение выражения   было равно 1


308

Умножение смешанных чисел

Встречаются задачи, в которых требуется перемножить

смешанные числа. Например, перемножить  и . Чтобы


решить этот пример, нужно перевести эти смешанные числа
в неправильные дроби, затем выполнить умножение
неправильных дробей:

Попробуем разобраться в этом примере с помощью


рисунка. Допустим, имеются одна целая и половина пиццы:
309

Теперь разберемся со смешанным множителем  . Этот


множитель означает, что одну целую и половину пиццы

нужно взять 2 раза и еще   раза.

С множителем 2 всё понятно, он означает что одну целую и


половину пиццы нужно взять два раза. Давайте возьмём два
раза целую пиццу и половину:

Но ещё осталось взять   от изначальной целой пиццы и

половины, ведь множителем было смешанное число . Для


этого вернёмся к изначальной одной целой и половине
пиццы, и разделим их на равные части так, чтобы можно
было взять от них ровно половину. А половину мы сможем
взять, если разделим целую пиццу на четыре части, а
половину на две части:
310

Мы разделили нашу целую пиццу и половину на равные


части, и теперь можем сказать, что является половиной от

этих кусков. Половиной от этих кусков является   пиццы. Это


можно хорошо увидеть, если мы упорядочим наши равные
кусочки следующим образом:

А если смотреть на изначальную целую пиццу и половину с


точки зрения такого порядка, как на этом рисунке, то

половиной от них является  пиццы.

Поэтому значение выражения   равно 


311

Пример 2. Найти значение выражения

Переводим смешанные числа в неправильные дроби и


перемножаем эти неправильные дроби. Если в ответе
получится неправильная дробь, выделим в ней целую часть:
312

Деление целого числа на дробь

Чтобы разделить целое число на дробь, нужно это целое


число умножить на дробь, обратную делителю.

Например, разделим число 3 на дробь  . Здесь число 3 —

это делимое, а дробь   — делитель.

Чтобы решить этот пример, нужно число 3 умножить на

дробь, обратную дроби  . А обратная дробь для дроби   это

дробь . Поэтому умножаем число 3 на дробь 

Допустим, имеются три целые пиццы:

Если мы зададим вопрос «cколько раз  (половина пиццы)


содержится в трёх пиццах», то ответом будет «шесть раз».

Действительно, если мы разделим каждую пиццу пополам, то


у нас получится шесть половинок:
313

Поэтому значение выражения   равно 6.

Пример 2. Найти значение выражения 

Чтобы решить этот пример, нужно число 2 умножить на

дробь, обратную дроби  . А обратная дробь для дроби   это

дробь 

Допустим, имеются две целые пиццы:


314

Зададим вопрос «Сколько раз   пиццы содержится в этих


двух пиццах?» Чтобы ответить на этот вопрос, выделим

целую часть в дроби  . После выделения целой части в этой

дроби получим 

Теперь поставим вопрос так: «Сколько раз  (одна целая и


половина пиццы) содержится в двух пиццах?».

Чтобы ответить на этот вопрос, нужно найти в двух пиццах


такое количество пиццы, которое изображено на следующем
рисунке:

В двух пиццах одна целая и половина пиццы содержится


один раз. Это можно увидеть, если вторую пиццу разделить
пополам:
315

А оставшаяся половина это треть от , которая не


вместилась. Третью она является по той причине, что в
одной целой и половине пиццы целую часть пиццы можно
разделить пополам. Тогда каждый кусок будет третью от
этого количества:

Поэтому значение выражения   равно 

Пример 3. Найти значение выражения 


316

Чтобы решить этот пример, нужно число 5 умножить на

дробь, обратную дроби  . А обратная дробь для дроби   это

дробь . Поэтому умножаем число 5 на 

Дробь  это 2 целых и  . Проще говоря, две целые и четверть


пиццы:

А выражение   определяет сколько раз   содержится в

пяти целых пиццах. Ответом было смешанное число .

То есть  пиццы содержится в пяти целых пиццах  раза.

Давайте нащупаем в пяти пиццах два раза по 


317

Белым цветом осталось не выделено две четверти. Эти две

четверти представляют собой   от  , которые не


вместились. Двумя девятыми они являются по той причине,

что в  пиццы каждую целую пиццу можно разделить на


четыре части. Тогда каждый кусок будет девятой частью от
этого количества, а два куска соответственно двумя из
девяти:

Поэтому значение выражения   равно 

Деление дроби на целое число

Чтобы разделить дробь на целое число, нужно данную дробь


умножить на число, обратное делителю. Таким делением мы
занимались в прошлом уроке. Вспомним ещё раз.
318

Пример 1. Разделим дробь   на число 2

Чтобы разделить дробь   на 2, нужно данную дробь


умножить на число, обратное числу 2. А обратное числу 2 это

дробь 

Пусть имеется половина пиццы:

Разделим её поровну на две части. Тогда каждая


получившаяся часть будет одной четвертой пиццы:
319

Поэтому значение выражения   равно 

Пример 2. Найти значение выражения 

Чтобы решить этот пример, нужно дробь  умножить на

число, обратное числу 2. Обратное числу 2 это дробь

Пример 3. Найти значение выражения

Умножаем первую дробь на число, обратное числу 3.

Обратное числу 3 это дробь


320

Деление целого числа на смешанное число

Встречаются задачи, в которых требуется разделить целое

число на смешанное число. Например, разделим 2 на .

Чтобы решить этот пример, нужно делитель перевести в


неправильную дробь. Затем умножить число 2 на дробь,
обратную делителю.

Переведём делитель   в неправильную дробь, получим .

Затем умножим 2 на дробь, обратную дроби . Обратная для

дроби  это дробь

Допустим, имеются две целые пиццы:


321

Зададим вопрос «Сколько раз   (одна целая и половина


пиццы) содержится в двух целых пиццах?». Похожий пример
мы решали ранее, когда учились делить целое число на
дробь.

В двух пиццах одна целая и половина пиццы содержится


один раз. Это можно увидеть, если вторую пиццу разделить
пополам:

А оставшаяся половина это треть от , которая не


вместилась. Третью она является по той причине, что в
одной целой и половине пиццы целую часть пиццы можно
разделить пополам. Тогда каждый кусок будет третью от
этого количества:
322

Поэтому значение выражения   равно 

Пример 2. Найти значение выражения

Переводим делитель  в неправильную дробь, получаем .

Теперь умножаем число 5 на дробь, обратную дроби .

Обратная для дроби  это дробь 

Сначала мы получили ответ , затем сократили эту дробь

на 5, и получили , но этот ответ нас тоже не устроил,


поскольку он представлял собой неправильную дробь. Мы
выделили в этой неправильной дроби целую часть. В

результате получили ответ


323

Деление смешанного числа на целое число

Чтобы разделить смешанное число на целое число, нужно


смешанное число перевести в неправильную дробь, затем
умножить эту дробь на число, обратное делителю.

Например, разделим  на 2. Чтобы решить этот пример,

нужно делимое  перевести в неправильную дробь. Затем


умножить эту дробь на число, обратное делителю 2.

Переведём смешанное число   в неправильную дробь,

получим .

Теперь умножаем  на число, обратное числу 2. Обратное

числу 2 это дробь

Допустим, имеется одна целая и половина пиццы:

Разделим это количество пиццы поровну на две части. Для


этого сначала разделим на две части целую пиццу:
324

Затем разделим поровну на две части и половину:

Теперь если мы сгруппируем эти кусочки на две группы, то

получим по   пиццы в каждой группе:

Поэтому значение выражения   равно 


325

Пример 2. Найти значение выражения 

Переведём делимое в неправильную дробь, получим .

Теперь умножаем  на число, обратное числу 4. Обратное

числу 4 это дробь .

Деление смешанных чисел

Чтобы разделить смешанные числа, нужно перевести их в


неправильные дроби, затем выполнить обычное деление
дробей. То есть умножить первую дробь на дробь, обратную
второй.

Пример 1. Найти значение выражения

Переведём смешанные числа в неправильные дроби.


Получим следующее выражение:

Как решать дальше мы уже знаем. Первую дробь  нужно


умножить на дробь, обратную второй. Обратная для второй

дроби это дробь .

Дорешаем данный пример до конца:


326

Допустим, имеются две целые и половина пиццы:

Если зададим вопрос «Сколько раз   (одна целая и


четверть пиццы) содержится в двух целых и половине
пиццы», то ответом будет «два раза»:

Пример 2. Найти значение выражения


327

Переведём смешанные числа в неправильные дроби.


Получим следующее выражение:

Теперь умножаем первую дробь на дробь, обратную второй.

Обратная для дроби это дробь

Сначала мы получили дробь . Эту дробь мы сократили на

9. В результате получили дробь  , но такой ответ нас тоже не

устроил и мы выделили в дроби  целую часть. В результате

получили окончательный ответ .

Задания для самостоятельного решения


Задание 1. Найдите значение выражения:

Показать решение
Задание 2. Найдите значение выражения:

Показать решение
Задание 3. Найдите значение выражения:
328

Показать решение
Задание 4. Найдите значение выражения:

Показать решение
Задание 5. Найдите значение выражения:

Показать решение
329

Сравнение дробей

Продолжаем изучать дроби. Сегодня мы поговорим об их


сравнении. Тема интересная и полезная. Она позволит
новичку почувствовать себя учёным в белом халате.

Суть сравнения дробей заключается в том, чтобы узнать


какая из двух дробей больше или меньше.

Чтобы ответить на вопрос какая из двух дробей больше или


меньше, пользуются операциями отношения, такими как
больше (>) или меньше (<).

Ученые-математики уже позаботились о готовых правилах,


позволяющие сразу ответить на вопрос какая дробь больше,
а какая меньше. Эти правила можно смело применять.

Мы рассмотрим все эти правила и попробуем разобраться,


почему происходит именно так.

Содержание урока

 Сравнение дробей с одинаковыми знаменателями


 Сравнение дробей с одинаковыми числителями
 Сравнение дробей с разными числителями и разными
знаменателями
 Вычитание смешанных чисел. Сложные случаи.
 Задания для самостоятельного решения

Сравнение дробей с одинаковыми знаменателями

Дроби, которые нужно сравнить, попадаются разные. Самый


удачный случай это когда у дробей одинаковые знаменатели,
330

но разные числители. В этом случае применяют следующее


правило:

Из двух дробей с одинаковыми знаменателями больше


та дробь, у которой числитель больше. И
соответственно меньше будет та дробь, у которой
числитель меньше.

Например, сравним дроби  и  и ответим, какая из этих


дробей больше. Здесь одинаковые знаменатели, но разные

числители. У дроби  числитель больше, чем у дроби  .

Значит дробь   больше, чем . Так и отвечаем. Отвечать


нужно с помощью значка больше (>)

Этот пример можно легко понять, если вспомнить про пиццы,

которые разделены на четыре части. пиццы больше, чем


пиццы:
331

Каждый согласится с тем, что первая пицца больше, чем


вторая.

Сравнение дробей с одинаковыми числителями

Следующий случай, в который мы можем попасть, это когда


числители дробей одинаковые, но знаменатели разные. Для
таких случаев предусмотрено следующее правило:

Из двух дробей с одинаковыми числителями больше та


дробь, у которой знаменатель меньше. И соответственно
меньше та дробь, у которой знаменатель больше.

Например, сравним дроби и . У этих дробей одинаковые

числители. У дроби знаменатель меньше, чем у дроби

. Значит дробь больше, чем дробь . Так и отвечаем:


332

Этот пример можно легко понять, если вспомнить про пиццы,

которые разделены на три и четыре части. пиццы больше,

чем пиццы:

Каждый согласится с тем, что первая пицца больше, чем


вторая.

Сравнение дробей с разными числителями и разными


знаменателями

Нередко случается так, что приходиться сравнивать дроби с


разными числителями и разными знаменателями.

Например, сравнить дроби  и . Чтобы ответить на вопрос,


какая из этих дробей больше или меньше, нужно привести их
к одинаковому (общему) знаменателю. Затем можно будет
легко определить какая дробь больше или меньше.
333

Приведём дроби  и  к одинаковому (общему) знаменателю.


Найдём наименьшее общее кратное (НОК) знаменателей

обеих дробей. НОК знаменателей дробей  и  это число 6.

Теперь находим дополнительные множители для каждой

дроби. Разделим НОК на знаменатель первой дроби . НОК


это число 6, а знаменатель первой дроби это число 2. Делим
6 на 2, получаем дополнительный множитель 3. Записываем
его над первой дробью:

Теперь найдём второй дополнительный множитель.

Разделим НОК на знаменатель второй дроби . НОК это


число 6, а знаменатель второй дроби это число 3. Делим 6 на
3, получаем дополнительный множитель 2. Записываем его
над второй дробью:

Умножим дроби на свои дополнительные множители:

Мы пришли к тому, что дроби, у которых были разные


знаменатели, превратились в дроби, у которых одинаковые
334

знаменатели. А как сравнивать такие дроби мы уже знаем. Из


двух дробей с одинаковыми знаменателями больше та
дробь, у которой числитель больше:

Правило правилом, а мы попробуем разобраться почему

больше, чем . Для этого выделим целую часть в дроби .В

дроби ничего выделять не нужно, поскольку эта дробь уже


правильная.

После выделения целой части в дроби , получим


следующее выражение:

Теперь можно легко понять, почему больше, чем . 


Давайте нарисуем эти дроби в виде пицц:
335

2 целые пиццы и пиццы, больше чем пиццы.

Вычитание смешанных чисел. Сложные случаи.

Вычитая смешанные числа, иногда можно обнаружить, что


всё идёт не так гладко, как хотелось бы. Часто случается так,
что при решении какого-нибудь примера ответ получается не
таким, каким он должен быть.

При вычитании чисел уменьшаемое должно быть больше


вычитаемого. Только в этом случае будет получен
нормальный ответ.

Например, 10−8=2

10 — уменьшаемое

8 — вычитаемое

2 — разность
336

Уменьшаемое 10 больше вычитаемого 8, поэтому мы


получили нормальный ответ 2.

А теперь посмотрим, что будет если уменьшаемое окажется


меньше вычитаемого. Пример 5−7=−2

5 — уменьшаемое

7 — вычитаемое

−2 — разность

В этом случае мы выходим за пределы привычных для нас


чисел и попадаем в мир отрицательных чисел, где нам
ходить пока рано, а то и опасно. Чтобы работать с
отрицательными числами, нужна соответствующая
математическая подготовка, которую мы ещё не получили.

Если при решении примеров на вычитание вы обнаружите,


что уменьшаемое меньше вычитаемого, то можете пока
пропустить такой пример. Работать с отрицательными
числами допустимо только после их изучения.

С дробями ситуация та же самая. Уменьшаемое должно быть


больше вычитаемого. Только в этом случае можно будет
получить нормальный ответ. А чтобы понять больше ли
уменьшаемая дробь, чем вычитаемая, нужно уметь сравнить
эти дроби.

Например, решим пример .


337

Это пример на вычитание. Чтобы решить его, нужно


проверить больше ли уменьшаемая дробь, чем вычитаемая.

больше чем 

поэтому смело можем вернуться к примеру и решить его:

Теперь решим такой пример 

Проверяем больше ли уменьшаемая дробь, чем вычитаемая.


Обнаруживаем, что она меньше:

В этом случае разумнее остановиться и не продолжать


дальнейшее вычисление. Вернёмся к этому примеру, когда
изучим отрицательные числа.

Смешанные числа перед вычитанием тоже желательно

проверять. Например, найдём значение выражения .

Сначала проверим больше ли уменьшаемое смешанное


число, чем вычитаемое. Для этого переведём смешанные
числа в неправильные дроби:
338

Получили дроби с разными числителями и разными


знаменателями. Чтобы сравнить такие дроби, нужно
привести их к одинаковому (общему) знаменателю. Не будем
подробно расписывать, как это сделать. Если испытываете
затруднения, обязательно повторите действия с дробями.

После приведения дробей к одинаковому знаменателю,


получаем следующее выражение:

Теперь нужно сравнить дроби и . Это дроби с


одинаковыми знаменателями. Из двух дробей с одинаковыми
знаменателями больше та дробь, у которой числитель
больше.

У дроби числитель больше, чем у дроби  . Значит дробь

больше, чем дробь  .

А это значит, что уменьшаемое больше, чем вычитаемое


339

А значит мы можем вернуться к нашему примеру и смело


решить его:

Пример 3. Найти значение выражения

Проверим больше ли уменьшаемое, чем вычитаемое.

Переведём смешанные числа в неправильные дроби:

Получили дроби с разными числителями и разными


знаменателями. Приведем данные дроби к одинаковому
(общему) знаменателю:

Теперь сравним дроби   и . У дроби числитель меньше,

чем у дроби , значит дробь меньше, чем дробь

А это значит, что и уменьшаемое меньше, чем вычитаемое


340

А это гарантировано приведёт нас в мир отрицательных


чисел. Поэтому разумнее остановиться на этом месте и не
продолжать вычисление. Продолжим его, когда изучим
отрицательные числа.

Пример 4. Найти значение выражения

Проверим больше ли уменьшаемое, чем вычитаемое.

Переведём смешанные числа в неправильные дроби:

Получили дроби с разными числителями и разными


знаменателями. Приведем их к одинаковому (общему)
знаменателю:

Теперь нужно сравнить дроби    и  . У дроби  числитель

больше, чем у дроби . Значит дробь  больше, чем дробь


.
341

А это значит, что уменьшаемое больше, чем вычитаемое

Поэтому мы смело можем продолжить вычисление нашего


примера:

Сначала мы получили ответ . Эту дробь мы сократили на 2

и получили дробь , но такой ответ нас тоже не устроил и


мы выделили целую часть в этом ответе. В итоге получили

ответ .

Задания для самостоятельного решения


Задание 1. Сравнить дроби:

Показать решение
Задание 2. Сравнить дроби:

Показать решение
Задание 3. Сравнить дроби:
342

Показать решение
Задание 4. Сравнить дроби:

Показать решение
343

Единицы измерения

Этот урок не будет новым для новичков. Все мы слышали со


школы такие понятия как сантиметр, метр, километр. А когда
речь заходила о массе, обычно говорили грамм, килограмм,
тонна.

Сантиметры, метры и километры; граммы, килограммы и


тонны носят одно общее название — единицы измерения
физических величин.

В данном уроке мы рассмотрим наиболее популярные


единицы измерения, но не будем сильно углубляться в эту
тему, поскольку единицы измерения уходят в область
физики. Сегодня мы вынуждены изучить часть физики,
поскольку нам это необходимо для дальнейшего изучения
математики.

Содержание урока

 Единицы измерения длины


 Международная система единиц СИ
 Единицы измерения массы
 Единицы измерения времени

Единицы измерения длины

Для измерения длины предназначены следующие единицы


измерения:

 миллиметры;
 сантиметры;
 дециметры;
344

 метры;
 километры.

Самая маленькая единица измерения это миллиметр (мм).


Миллиметры можно увидеть даже воочию, если взять
линейку, которой мы пользовались в школе каждый день

Подряд идущие друг за другом маленькие линии это и есть


миллиметры.  Точнее, расстояние между этими линиями
равно одному миллиметру (1 мм):
345

Следующая единица измерения это сантиметр (см). На


линейке каждый сантиметр обозначен числом. К примеру
наша линейка, которая была на первом рисунке, имела длину
15 сантиметров. Последний сантиметр на этой линейке
выделен числом 15.

В одном сантиметре 10 миллиметров. Между одним


сантиметром и десятью миллиметрами можно поставить знак
равенства, поскольку они обозначают одну и ту же длину:

1 см = 10 мм

Вы можете сами убедиться в этом, если посчитаете


количество миллиметров на предыдущем рисунке. Вы
обнаружите, что количество миллиметров (расстояний между
линиями) равно 10.
346

Следующая единица измерения длины это дециметр (дм). В


одном дециметре десять сантиметров. Между одним
дециметром и десятью сантиметрами можно поставить знак
равенства, поскольку они обозначают одну и ту же длину:

1 дм = 10 см

Вы можете убедиться в этом, если посчитаете количество


сантиметров на следующем рисунке:
347

Вы обнаружите, что количество сантиметров равно 10.

Следующая единица измерения это метр (м). В одном метре


десять дециметров. Между одним метром и десятью
дециметрами можно поставить знак равенства, поскольку
они обозначают одну и ту же длину:

1 м = 10 дм

К сожалению, метр нельзя проиллюстрировать на рисунке,


потому что он достаточно великоват. Если вы хотите увидеть
метр в живую, возьмите рулетку. Она есть у каждого в доме.
На рулетке один метр будет обозначен как 100 см. Это
потому что в одном метре десять дециметров, а в десяти
дециметрах сто сантиметров:

1 м = 10 дм = 100 см

100 получается путём перевода одного метра в сантиметры.


Это отдельная тема, которую мы рассмотрим чуть позже. А
пока перейдём к следующей единице измерения длины,
которая называется километр.
348

Километр считается самой большой единицей измерения


длины. Есть конечно и другие более старшие единицы, такие
как мегаметр, гигаметр тераметр, но мы не будем их
рассматривать, поскольку для дальнейшего изучения
математики нам достаточно и километра.

В одном километре тысяча метров. Между одним


километром и тысячью метрами можно поставить знак
равенства, поскольку они обозначают одну и ту же длину:

1 км = 1000 м

В километрах измеряются расстояния между городами и


странами. К примеру, расстояние от Москвы до Санкт-
Петербурга около 714 километров.

Международная система единиц СИ

Международная система единиц СИ — это некоторый набор


общепринятых физических величин.

Основное предназначение международной системы единиц


СИ — достижение договоренностей между странами.

Мы знаем, что языки и традиции стран мира различны. С


этим  ничего не поделать. Но законы математики и физики
одинаково работают везде. Если в одной стране «дважды
два будет четыре», то и в другой стране «дважды два будет
четыре».

Основная проблема заключалась в том, что для каждой


физической величины существует несколько единиц
измерения. К примеру, мы сейчас узнали, что для измерения
349

длины существуют миллиметры, сантиметры, дециметры,


метры и километры. Если несколько ученых, говорящих на
разных языках, соберутся в одном месте для решения какой-
нибудь задачи, то такое большое многообразие единиц
измерения длины может породить между этими учеными
противоречия.

Один ученый будет заявлять, что в их стране длина


измеряется в метрах. Второй может сказать, что в их стране
длина измеряется в километрах. Третий может предложить
свою единицу измерения.

Поэтому была создана международная система единиц СИ.


СИ это аббревиатура от французского словосочетания Le
Système International d’Unités, SI (что в переводе на русский
означает — международная система единиц СИ). 

В СИ приведены наиболее популярные физические


величины и для каждой из них определена своя
общепринятая единица измерения. К примеру, во всех
странах при решении задач условились, что длину будут
измерять в метрах. Поэтому, при решении задач, если длина
дана в другой единице измерения (например, в километрах),
то её обязательно нужно перевести в метры. О том, как
переводить одну единицу измерения в другую, мы поговорим
немного позже. А пока нарисуем свою международную
систему единиц СИ.

Наш рисунок будет представлять собой таблицу физических


величин. Каждую изученную физическую величину мы будем
включать в нашу таблицу и указывать ту единицу измерения,
которая принята во всех странах. Сейчас мы изучили
единицы измерения длины и узнали, что в системе СИ для
350

измерения длины определены метры. Значит наша таблица


будет выглядеть так:

Единицы измерения массы

Масса – это величина, обозначающая количество вещества в


теле. В народе массу тела называют весом. Обычно, когда
что-либо взвешивают, говорят «это весит столько-то
килограмм», хотя речь идёт не о весе, а о массе этого тела.

Вместе с тем, масса и вес это разные понятия. Вес — это


сила с которой тело действует на горизонтальную опору. Вес
измеряется в ньютонах. А масса это величина,
показывающая количество вещества в этом теле.

Но ничего страшного нет в том, если вы назовёте массу тела


весом. Даже в медицине говорят «вес человека», хотя речь
идёт о массе человека. Главное быть в курсе, что это разные
понятия

Для измерения массы используются следующие единицы


измерения:

 миллиграммы;
 граммы;
 килограммы;
 центнеры;
351

 тонны.

Самая маленькая единица измерения это миллиграмм (мг).


Миллиграмм скорее всего вы никогда не примените на
практике. Их применяют химики и другие ученые, которые
работают с мелкими веществами. Для вас достаточно знать,
что такая единица измерения массы существует.

Следующая единица измерения это грамм (г). В граммах


принято измерять количество того или иного продукта при
составлении рецепта.

В одном грамме тысяча миллиграммов. Между одним


граммом и тысячью миллиграммами можно поставить знак
равенства, поскольку они обозначают одну и ту же массу:

1 г = 1000 мг

Следующая единица измерения это килограмм (кг).


Килограмм это общепринятая единица измерения. В ней
измеряется всё что угодно. Килограмм включен в систему
СИ. Давайте и мы включим в нашу таблицу СИ ещё одну
физическую величину. Она у нас будет называться «масса»:

В одном килограмме тысяча граммов. Между одним


килограммом и тысячью граммами можно поставить знак
равенства, поскольку они обозначают одну и ту же массу:
352

1 кг = 1000 г

Следующая единица измерения это центнер (ц). В


центнерах удобно измерять массу урожая, собранного с
небольшого участка или массу какого-нибудь груза.

В одном центнере сто килограммов. Между одним центнером


и ста килограммами можно поставить знак равенства,
поскольку они обозначают одну и ту же массу:

1 ц = 100 кг

Следующая единица измерения это тонна (т). В тоннах


обычно измеряются большие грузы и массы больших тел.
Например, масса космического корабля или автомобиля.

В одной тонне тысяча килограмм. Между одной тонной и


тысячью килограммами можно поставить знак равенства,
поскольку они обозначают одну и ту же массу:

1 т = 1000 кг

Единицы измерения времени

Что такое время думаем объяснять не нужно. Каждый знает


что из себя представляет время и зачем оно нужно. Если мы
откроем дискуссию на то, что такое время и попытаемся дать
ему определение, то начнем углубляться в философию, а
это нам сейчас не нужно. Лучше начнём с единиц измерения
времени.

Для измерения времени предназначены следующие единицы


измерения:
353

 секунды;
 минуты;
 часы;
 сутки.

Самая маленькая единица измерения это секунда (с). Есть


конечно и более маленькие единицы такие как
миллисекунды, микросекунды, наносекунды, но их мы
рассматривать не будем, поскольку на данный момент в этом
нет смысла.

В секундах измеряются различные показатели. Например, за


сколько секунд спортсмен пробежит 100 метров. Секунда
включена в международную систему единиц СИ для
измерения времени и обозначается как «с». Давайте и мы
включим в нашу таблицу СИ ещё одну физическую величину.
Она у нас будет называться «время»:

Следующая единица измерения времени это минута (м). В


одной минуте 60 секунд. Между одной минутой и
шестьюдесятью секундами можно поставить знак равенства,
поскольку они обозначают одно и то же время:

1 м = 60 с
354

Следующая единица измерения это час (ч). В одном часе 60


минут. Между одним часом и шестьюдесятью минутами
можно поставить знак равенства, поскольку они обозначают
одно и то же время:

1 ч = 60 м

К примеру, если мы изучали этот урок один час и нас спросят


сколько времени мы потратили на его изучение, мы можем
ответить двумя способами: «мы изучали урок один час» или
так «мы изучали урок шестьдесят минут». В обоих случаях,
мы ответим правильно.

Следующая единица измерения времени это сутки. В сутках


24 часа. Между одними сутками и двадцатью четырьмя
часами можно поставить знак равенства, поскольку они
обозначают одно и то же время:

1 сут = 24 ч
355

Применение дробей

Этот урок будет интересным и познавательным. Мы


научимся применять дроби для различных жизненных
случаев.

Содержание урока

 Нахождение дроби от числа


 Нахождение целого числа по дроби
 Деление меньшего числа на большее
 Какую часть одно число составляет от другого
 Задания для самостоятельного решения

Нахождение дроби от числа

Мы уже говорили, что дробь это часть от чего-либо. Эта

часть может быть чем угодно. Например,   от пиццы это


половина пиццы:
356

Но применение дробей не заканчивается на одной пицце.

Например, можно узнать сколько составляет  от десяти


сантиметров:

Как вы уже догадались от десяти сантиметров составляют

пять сантиметров. Ведь это простейшая дробь, которая


означает половину от чего-то. У нас было десять
сантиметров. Мы разделили эти десять сантиметров
пополам и получили пять сантиметров.

Попробуем узнать, сколько составляет от одного часа.

Вспоминаем, что час это 60 минут. Нам нужно найти  


(половину) от 60 минут. Нетрудно догадаться, что половина

от 60 минут это 30 минут. Значит  от одного часа составляет


30 минут или полчаса.

Попробуем найти от одного центнера. Центнер это 100 кг.

Требуется найти (половину) от 100 кг. Нетрудно догадаться,


357

что половина от 100 кг это 50 кг. Значит от одного центнера


составляют 50 кг.

Поскольку мы занимаемся математикой, значит в


большинстве случаев будем иметь дело с числами.

Например, найдём  от числа 12.

Итак, нужно найти половину от числа 12. Нетрудно


догадаться, что половиной от числа 12 является число 6.

Значит  числа 12 составляет число 6.

Чтобы легче было находить дробь от числа, можно


пользоваться следующим правилом:

Чтобы найти дробь от числа, нужно это число разделить


на знаменатель дроби, и полученный результат
умножить на числитель дроби.

Попробуем проследить весь процесс работы этого правила.


Для примера возьмём десять сантиметров:

Пусть требуется найти  от этих десяти сантиметров. Читаем


первую часть правила:

Чтобы найти дробь от числа, нужно это число разделить

на знаменатель дроби 
358

Итак, делим десять сантиметров на знаменатель дроби .


Знаменатель этой дроби равен числу 2. Поэтому делим
десять сантиметров на 2

10 см : 2 = 5 см

Читаем вторую часть правила:

и полученный результат умножить на числитель дроби 

Итак, умножаем пять сантиметров на числитель дроби .


Числитель дроби в данном случае единица. Поэтому
умножаем пять сантиметров на единицу:

5 см × 1 = 5 см

Мы нашли от десяти сантиметров. Видим, что  от десяти


сантиметров составляют пять сантиметров:

Почему же после деления числа на знаменатель дроби


приходиться умножать полученный результат на числитель
дроби? Дело в том, что знаменатель дроби показывает на
сколько частей что-либо разделено, а числитель показывает
сколько частей было взято.
359

В нашем примере десять сантиметров были разделены на


две части (пополам), и из этих частей была взята одна часть.
Умножая одну часть на числитель дроби, мы тем самым
указываем сколько частей мы берём от чего-то. То есть

умножив пять сантиметров на числитель дроби , мы тем


самым указали, что берем одну часть из двух.

Пример 2. Найти  от 10 см.

Применим правило нахождения дроби от числа:

Чтобы найти дробь от числа, нужно это число разделить


на знаменатель дроби, и полученный
результат умножить на числитель дроби.

Сначала делим 10 сантиметров на знаменатель дроби

10 см : 5 = 2 см

Получили два сантиметра. Этот результат нужно умножить

на числитель дроби 

2 см × 2 = 4 см

Мы нашли от десяти сантиметров. Видим, что  от десяти


сантиметров составляют четыре сантиметра.

Весь процесс решения можно увидеть на следующем


рисунке:
360

Сначала десять сантиметров были разделены на пять


равных частей. Затем было взято две части из этих пяти
частей:

Пример 3.  Найти  от числа 56.

Чтобы найти   от числа 56, нужно это число разделить на

знаменатель дроби , и полученный результат умножить на

числитель дроби .

Итак, сначала делим число 56 на знаменатель дроби

56 : 8 = 7

Теперь умножаем полученное результат на числитель дроби


361

7 × 3 = 21

Получили ответ 21. Значит  от числа 56 составляет 21.

Пример 4. Найти   от одного часа.

Один час это 60 минут. Задание можно понимать, как

нахождение  от 60 минут.

Сначала разделим 60 минут на знаменатель дроби

60 мин : 4 = 15 мин

Теперь умножим полученные 15 минут на числитель дроби

15 мин × 2 = 30 мин

Получили в ответе 30 минут. Значит  от одного часа


составляют тридцать минут или полчаса.

Пример 5. Найти   от одного метра.

Один метр это сто сантиметров. Сначала разделим 100 см на

знаменатель дроби

100 см : 5 = 20 см
362

Теперь умножим полученные 20 см на числитель дроби

20 см × 4 = 80 см

Получили ответ 80 см. Значит  от одного метра составляют


80 см.

Нахождение целого числа по дроби

Зная часть числа и сколько это составляет от целого числа,


можно найти изначальное целое число. Это обратная задача
к той, которую мы рассматривали в предыдущей теме. Там
мы искали дробь от числа, деля это число на знаменатель
дроби, и полученный результат умножая на числитель дроби.

А сейчас наоборот, зная дробь и сколько это составляет от


числа, найти изначальное целое число.

Например, если  длины линейки составляют шесть


сантиметров и нам говорят найти длину всей линейки, то мы
должны понимать, что от нас требуют найти изначальное

целое число (длину всей линейки) по дроби . Давайте


решим эту задачу.

Требуется найти длину всей линейки по дроби . Известно,

что  длины всей линейки составляют 6 см.

Мы уже знаем каким образом получились эти 6 см. Имелась


какая-то длина, её разделили на пять частей, поскольку
363

знаменатель дроби  это число 5. Затем было взято две

части от пяти частей, поскольку числитель дроби  это число


2.

Чтобы узнать длину всей линейки, сначала нужно узнать


длину одной части. Как это узнать? Попробуем догадаться,
внимательно изучив следующий рисунок:

Если две части длины линейки составляют 6 см, то нетрудно


догадаться, что одна часть составляет 3 см. А чтобы
получить эти 3 см, надо 6 разделить на 2

6 см : 2 = 3 см

Итак, мы нашли длину одной части. Одна часть из пяти или  


длины линейки составляет 3 см. Если частей всего пять, то
для нахождения длины линейки, нужно взять три сантиметра
пять раз. Другими словами, умножить 3 см на число 5

3 см × 5 = 15

Мы нашли длину линейки. Она составляет 15 сантиметров.


Это можно увидеть на следующем рисунке.
364

Видно, что пять частей из пяти или   составляют пятнадцать


сантиметров.

Чтобы легче было находить число по его дроби, можно


пользоваться следующим правилом:

Чтобы найти число по его дроби, нужно известное число


разделить на числитель дроби, и полученный результат
умножить на знаменатель дроби.

Пример 2. Число 20 это   от всего числа. Найдите это число.

Знаменатель дроби  показывает, что число, которое мы

должны найти, разделено на пять частей. Если   этого числа


составляет число 20, то для нахождения всего числа,

сначала нужно найти   (одну часть из пяти) от всего числа.

Для этого 20 надо разделить на числитель дроби

20 : 4 = 5
365

Мы нашли  от всего числа. Эта часть равна 5. Чтобы найти


всё число, нужно полученный результат 5 умножить на

знаменатель дроби

5 × 5 = 25

Мы нашли  от всего числа. Другими словами, нашли всё


число, которое от нас требовали найти. Это число 25.

Пример 3. Десять минут это   времени приготовления каши.


Найдите общее время приготовления каши.

Знаменатель дроби  показывает, что общее время

приготовления каши разделено на три части. Если  времени


приготовления каши составляет десять минут, то для
нахождения общего времени приготовления, нужно сначала

найти   времени приготовления. Для этого 10 нужно

разделить на числитель дроби

10 мин : 2 = 5 мин

Мы нашли  времени приготовления каши.  времени


приготовления каши составляют пять минут. Для нахождения
общего времени приготовления, нужно 5 минут умножить на

знаменатель дроби
366

5 мин × 3 = 15 мин

Мы нашли  времени приготовления каши, то есть нашли


общее время приготовления. Оно составляет 15 минут.

Пример 4.     массы мешка цемента составляет 30 кг. Найти


общую массу мешка.

Знаменатель дроби показывает, что общая масса мешка

разделена на четыре части. Если массы мешка составляет


30 кг то для того, чтобы найти общую массу мешка нужно

сначала найти массы мешка. Для этого 30 надо разделить

на числитель дроби .

30кг : 2 = 15кг

Мы нашли массы мешка. массы мешка составляет 15 кг.


Теперь, чтобы найти общую массу мешка, надо 15кг

умножить на знаменатель дроби

15кг × 4 = 60кг

Мы нашли массы мешка. Другими словами, нашли общую


массу мешка. Общая масса мешка цемента составляет 60 кг.
367

Деление меньшего числа на большее

В жизни часто возникают ситуации, когда требуется


разделить меньшее число на большее. Например,
представим ситуацию. Имеется трое друзей:

И требуется поровну разделить между ними два яблока. Как


это сделать? Друзей трое, а яблок всего два. Мы попали в
ситуацию в которой требуется разделить меньшее число на
большее (два яблока на троих).

Для таких случаев предусмотрено следующее правило:

При делении меньшего числа на большее получается


дробь, в числителе которой делимое, а в знаменателе –
делитель.

Давайте применим это правило. Оно говорит, что при


делении меньшего числа на большее получается дробь, в
числителе которой делимое, а в знаменателе делитель.
368

Делимое у нас это два яблока. Записываем в числителе


число 2:

А делитель у нас это трое друзей (вспоминаем, что делитель


показывает на сколько частей надо разделить делимое).
Записываем тройку в знаменателе нашей дроби:

Забавно, но дробь  это ответ к нашей задаче. Каждому

другу достанется яблока. Почему так произошло?

Чтобы разделить два яблока на троих, надо разрезать ножом


каждое яблоко на три части и раскидать поровну эти куски
между тремя друзьями:
369

Как видно на рисунке, каждое яблоко было разделено на три


части и раскидано поровну на троих друзей. Каждому другу

досталось яблока (два кусочка из трёх).


370

Какую часть одно число составляет от другого

Иногда возникает необходимость узнать какую часть первое


число составляет от второго. Для таких случаев
предусмотрено следующее правило:

Чтобы узнать какую часть первое число составляет от


второго, надо первое число разделить на второе. 

Например, яблоко разделили на пять одинаковых долек.


Какую часть яблока составляют две дольки?

Чтобы ответить на этот вопрос, надо первое число разделить


на второе. Первое число это 2, второе — 5. Получается

дробь .

Значит две дольки из пяти долек составляют две пятых. Это


можно увидеть на следующем рисунке:
371

Итак, две дольки яблока из пяти составляют две пятых.

Возникает вопрос, а как узнать какое число первое, а какое


второе? Для этого нужно посмотреть на вопрос, который
поставлен в задаче. То число, которое указано в вопросе
задачи, оно и будет первым числом. Например, в
предыдущей задаче вопрос был поставлен так:

«Какую часть яблока составляют две такие дольки?»

Если внимательно присмотреться к вопросу, то можно


обнаружить, что в нём указано число 2. Оно и стало первым
числом.

Иногда в вопросе мелькает сразу два числа. Например:


какую часть составляет число 2 от числа 10?
372

В этом случае первым числом будет то, которое в вопросе


расположено раньше. В данном случае первое число это 2, а

второе 10. Делим 2 на 10, получаем дробь . Значит число 2

от числа 10 составляет  (две десятых).

Дробь означает, что число 10 разделено на десять частей,


и от этих десяти частей взято две части.

Также, эту дробь можно сократить на 2. После сокращения

дроби на 2 получаем дробь .

Дробь  тоже может послужить ответом к задаче. Она будет


означать, что число 10 разделено на пять частей, и от этих
пяти частей взята одна часть.

Таким образом, число 2 составляет (одну пятую) от числа


10.

Пример 3. Какую часть составляет число 5 от числа 15?

Делим первое число на второе. Первое число 5, а второе 15.

Делим 5 на 15, получаем дробь . Эту дробь можно


сократить на 5
373

Получили аккуратную дробь . Значит ответ будет выглядеть


следующим образом:

Число 5 составляет  (одну третью) от числа 15.

Это можно даже проверить. Для этого нужно найти от числа


15. Если мы всё сделали правильно, то должны получить
число 5.

Итак, найдём от числа 15. Как находить дробь от числа мы


уже знаем

15 : 3 = 5

5×1=5

Получили ответ 5. Значит задача была решена правильно.

Пример 4. Какую часть 3 см составляют от 12 см?

Делим первое число на второе. Первое число это 3, а второе

12. Получаем дробь . Эту дробь можно сократить на 3

Получили ответ  .  Значит 3 см составляют (одну четвёртую)


от 12 см.
374

Проверим правильно ли мы решили эту задачу. Для этого

найдём от 12 см. Если мы всё сделали правильно, то


должны получить 3 см.

Делим 12 на знаменатель дроби

12 см : 4 = 3 см

Умножаем полученные 3 см на числитель дроби

3 см × 1 = 3 см

Получили ответ 3 см. Значит задача была решена


правильно.

Задания для самостоятельного решения

Задание 1. Найдите от числа 30.


Показать решение

Задание 2. Найдите от числа 30.


Показать решение
375

Задание 3. Найдите от числа 30.


Показать решение

Задание 4. Найдите от числа 48.


Показать решение

Задание 5. Найдите от числа 48.


Показать решение

Задание 6. Найдите от 120 см.


Показать решение

Задание 7. Найдите от 150 см.


Показать решение
Задание 8. Найдите целое число по дроби, если известно,

что этого числа составляет число 16.


Показать решение
Задание 9. Найдите целое число по дроби, если известно,

что этого числа составляет число 32.


Показать решение
Задание 10. Найдите целое число по дроби, если известно,

что этого числа составляет число 150.


Показать решение
376

Задание 11. Найдите длину пути от дома до школы, если

известно, что этого пути составляют 4 км.


Показать решение

Задание 12. Найдите длину рулетки, если известно, что этой


рулетки составляют 100 см.
Показать решение
377

Десятичные дроби

Мы уже говорили, что дроби бывают обыкновенные и


десятичные. На данный момент мы немного изучили
обыкновенные дроби. Мы узнали, что обыкновенные дроби
бывают правильные и неправильные. Также мы узнали, что
обыкновенные дроби можно сокращать, складывать,
вычитать умножать и делить. И ещё мы узнали, что бывают
так называемые смешанные числа, которые состоят из целой
и дробной части.

Мы ещё не до конца изучили обыкновенные дроби. Есть


немало тонкостей и деталей, о которых следует поговорить,
но уже сегодня мы начнём изучать десятичные дроби,
поскольку обыкновенные и десятичные дроби достаточно
часто приходится сочетать. То есть при решении задач
приходиться работать с обоими видов дробей.

Этот урок возможно покажется сложным и непонятным. Это


вполне нормально. Такого рода уроки требуют, чтобы их
именно изучали, а не просматривали поверхностно.

Содержание урока

 Выражение величин в дробном виде


 Перевод смешанных чисел в десятичные дроби
 Перевод обыкновенных дробей в десятичные дроби
 Перевод неправильных дробей в десятичную дробь
 Перевод десятичной дроби в смешанное число
 Перевод десятичной дроби в обыкновенную дробь
378

Выражение величин в дробном виде

Иногда удобно бывает показать что-либо в дробном виде.


Например, одна десятая часть дециметра записывается так:

Это выражение означает, что один дециметр был разделён


на десять равных частей, и от этих десяти частей была взята
одна часть. А одна часть из десяти в данном случае равна
одному сантиметру:

Рассмотрим следующий пример. Пусть требуется показать 6


см и ещё 3 мм в сантиметрах в дробном виде.

Итак, 6 целых сантиметров у нас уже есть:

Но осталось еще 3 миллиметра. Как показать эти 3


миллиметра, при этом в сантиметрах? На помощь приходят
379

дроби. Один сантиметр это десять миллиметров. Три


миллиметра это три части из десяти. А три части из десяти

записываются как см

Выражение см означает, что один сантиметр был разделён


на десять равных частей, и от этих десяти частей взяли три
части.

В результате имеем шесть целых сантиметров и три десятых


сантиметра:

Цифра 6 показывает число целых сантиметров, а дробь —


число дробных. Эта дробь читается как «шесть целых и три
десятых сантиметра».
380

Дроби, в знаменателе которых присутствуют числа 10, 100,


1000 можно записывать без знаменателя. Сначала пишут
цéлую часть, а потом числитель дробной части. Целая часть
отделяется от числителя дробной части запятой.

Например, запишем без знаменателя. Сначала


записываем целую часть. Целая часть это 6

Целая часть записана. Сразу же после написания целой


части ставим запятую:

6,

И теперь записываем числитель дробной части. В

смешанном числе числитель дробной части это число 3.


Записываем после запятой тройку:

6,3

Любое число, которое представляется в таком виде,


называется десятичной дробью.

Поэтому показать 6 см и ещё 3 мм в сантиметрах можно с


помощью десятичной дроби:

6,3 см

Выглядеть это будет следующим образом:


381

На самом деле десятичные дроби это те же самые


обыкновенные дроби и смешанные числа. Особенность таких
дробей заключается в том, что в знаменателе их дробной
части содержатся числа 10, 100, 1000 или 10000.

Как и смешанное число, десятичная дробь имеет цéлую

часть и дробную. Например, в смешанном числе целая

часть это 6, а дробная часть это .

В десятичной дроби 6,3 целая часть это число 6, а дробная

часть это числитель дроби , то есть число 3.

Бывает и так, что обыкновенные дроби в знаменателе


которых числа 10, 100, 1000 даны без целой части.

Например, дробь дана без целой части. Чтобы записать


такую дробь как десятичную, сначала записывают 0, затем
ставят запятую и записывают числитель дробной части.

Дробь без знаменателя будет записана следующим


образом:

0,5

Читается как «ноль целых, пять десятых».


382

Перевод смешанных чисел в десятичные дроби

Когда мы записываем смешанные числа без знаменателя,


мы тем самым перевóдим их в десятичные дроби. При
переводе обыкновенных дробей в десятичные дроби нужно
знать несколько моментов, о которых мы сейчас поговорим.

После того как записана целая часть, обязательно нужно


посчитать количество нулей в знаменателе дробной части,
поскольку количество нулей дробной части и количество
цифр после запятой в десятичной дроби должно быть
одинаковым. Что это значит? Рассмотрим следующий

пример: перевести смешанное число в десятичную дробь.

Сначала записываем целую часть и ставим запятую:

3,

И можно бы сразу записать числитель дробной части и


десятичная дробь готова, но обязательно нужно посчитать
сколько нулей содержится в знаменателе дробной части.

Итак, посчитаем количество нулей в дробной части

смешанного числа .  Видим, что в знаменателе дробной


части один ноль. Значит в десятичной дроби после запятой
будет одна цифра и это цифра будет числитель дробной

части смешанного числа , то есть число 2

3,2
383

Таким образом, смешанное число при переводе в


десятичную дробь обращается в 3,2. Эта десятичная дробь
читается так:

«Три целых, две десятых»

«Десятых» потому что в дробной части смешанного числа

содержится число 10.

Пример 2. Перевести смешанное число в десятичную


дробь.

Записываем цéлую часть и ставим запятую:

5,

И можно бы сразу записать числитель дробной части и


получить десятичную дробь 5,3 но правило говорит, что
после запятой должно быть столько цифр сколько нулей в

знаменателе дробной части смешанного числа  . А мы

видим, что в знаменателе дробной части   два нуля.


Значит в нашей десятичной дроби после запятой должно
быть две цифры, а не одна.

В таких случаях числитель дробной части нужно немного


видоизменить: добавить ноль перед числителем, то есть
перед числом 3
384

Теперь можно довести дело до конца. Записываем после


запятой числитель дробной части:

5,03

Видим, что количество цифр после запятой и количество

нулей в знаменателе дробной части смешанного числа   


одинаково.

Десятичная дробь 5,03 читается так:

«Пять целых, три сотых»

«Сотых» потому что в знаменателе дробной части

смешанного числа содержится число 100.

Пример 3. Перевести смешанное число в десятичную


дробь.

Из предыдущих примеров мы узнали, что для успешного


перевода смешанного числа в десятичную дробь, количество
цифр в числителе дробной части и количество нулей в 
знаменателе дробной части должно быть одинаковым.

Перед переводом смешанного числа в десятичную


дробь, его дробную часть нужно немного видоизменить, а
именно сделать так, чтобы количество цифр в числителе
дробной части и количество нулей в знаменателе дробной
части было одинаковым.
385

В первую очередь смóтрим на количество нулей в


знаменателе дробной части. Видим, что там три нуля:

Наша задача организовать в числителе дробной части три


цифры. Одна цифра у нас уже есть — это цифра 2. Осталось
добавить ещё две цифры. Ими будут два нуля. Добавим их
перед цифрой 2. В результате количество нулей в
знаменателе и количество цифр в числителе станет
одинаковым:
386

Теперь можно заняться переводом этого смешанного числа в


десятичную дробь. Записываем сначала цéлую часть и
ставим запятую:

3,

и сразу записываем числитель дробной части

3,002

Видим, что количество цифр после запятой и количество

нулей в знаменателе дробной части смешанного числа


одинаково.

Десятичная дробь 3,002 читается так:

«Три целых, две тысячных»


387

«Тысячных» потому что в знаменателе дробной части

смешанного числа    содержится число 1000.

Перевод обыкновенных дробей в десятичные дроби

Обыкновенные дроби, у которых в знаменателе числа 10,


100, 1000 или  10000, тоже можно перевести в десятичные
дроби. Поскольку у обыкновенной дроби целая часть
отсутствует, сначала записывают 0, затем ставят запятую и
записывают числитель дробной части.

Здесь также количество нулей в знаменателе и количество


цифр в числителе должно быть одинаковым. Поэтому
следует быть внимательным.

Пример 1. Перевести обыкновенную дробь в десятичную


дробь.

Целая часть отсутствует, значит сначала записываем 0 и


ставим запятую:

0,

Теперь смóтрим на количество нулей в знаменателе. Видим,


что там один ноль. И в числителе одна цифра. Значит можно
спокойно продолжить десятичную дробь, записав после
запятой цифру 5

0,5
388

В полученной десятичной дроби 0,5 количество цифр после

запятой и количество нулей в знаменателе дроби


одинаково. Значит дробь переведена правильно.

Десятичная дробь 0,5 читается так:

«Ноль целых, пять десятых»

Пример 2. Перевести обыкновенную дробь в десятичную


дробь.

Целая часть отсутствует. Записываем сначала 0 и стáвим


запятую:

0,

Теперь смóтрим на количество нулей в знаменателе. Видим,


что там два нуля. А в числителе только одна цифра. Чтобы
сделать количество цифр и количество нулей одинаковым,
добавим в числителе перед цифрой 2 один ноль. Тогда

дробь примет вид  . Теперь количество нулей в


знаменателе и количество цифр в числителе одинаково.
Значит можно продолжить десятичную дробь:

0,02

В полученной десятичной дроби 0,02 количество цифр после

запятой и количество нулей в знаменателе дроби 


одинаково. Значит дробь переведена правильно.
389

Десятичная дробь 0,02 читается так:

«Ноль целых, две сотых».

Пример 3. Перевести обыкновенную дробь в


десятичную дробь.

Записываем 0 и стáвим запятую:

0,

Теперь посчитаем количество нулей в знаменателе дроби

. Видим, что там пять нулей, а в числителе только одна


цифра. Чтобы сделать количество нулей в знаменателе и
количество цифр в числителе одинаковым, нужно в
числителе перед цифрой 5 дописать четыре нуля:

Теперь можно продолжить десятичную дробь. Записываем

после запятой числитель дроби

0,00005

В полученной десятичной дроби 0,00005 количество цифр


после запятой и количество нулей в знаменателе дроби 

одинаково. Значит дробь переведена правильно.

Десятичная дробь 0,00005 читается так:

«Ноль целых, пять стотысячных».


390

Перевод неправильных дробей в десятичную дробь

Неправильная дробь это дробь, у которой числитель больше


знаменателя.

Бывают неправильные дроби, у которых в знаменателе


содержатся числа 10, 100, 1000 или 10000. Такие дроби
можно переводить в десятичные. Но перед переводом в
десятичную дробь, у таких дробей необходимо выделять
цéлую часть.

Пример 1. Перевести неправильную дробь  в десятичную.

Дробь является неправильной. Чтобы перевести такую


дробь в десятичную, нужно в первую очередь выделить у нее
цéлую часть. Вспоминаем, как выделять целую часть у
неправильных дробей. Если забыли, советуем вернуться к
этой теме и хорошенько изучить её.

Итак, выделим целую часть в неправильной дроби .


Напомним, что дробь означает деление — в данном случае
деление числа 112 на число 10. Деление нужно выполнить с
остатком:
391

Посмóтрим на этот рисунок и соберём новое смешанное


число, подобно детскому конструктору. Частное 11 будет
целой частью, остаток 2 — числителем дробной части,
делитель 10 — знаменателем дробной части:

Мы получили смешанное число . Его и переведём в


десятичную дробь. А как переводить такие числа в
десятичные дроби мы уже знаем. Сначала записываем
целую часть и ставим запятую:

11,
392

Теперь посчитаем количество нулей в знаменателе дробной


части. Видим, что там один ноль. И в числителе дробной
части одна цифра. Значит количество нулей в знаменателе
дробной части  и количество цифр в числителе дробной
части одинаково. Это даёт нам возможность сразу записать
после запятой числитель дробной части:

11,2

Значит, неправильная дробь при переводе в десятичную


обращается в 11,2

Десятичная дробь 11,2 читается так:

«Одиннадцать целых, две десятых».

Пример 2. Перевести неправильную дробь   в десятичную


дробь.

Это неправильная дробь, поскольку числитель больше


знаменателя. Но её можно перевести в десятичную дробь,
поскольку в знаменателе содержится число 100.

В первую очередь выделим целую часть этой дроби. Для


этого разделим уголком 450 на 100:
393

Соберём новое смешанное число — получим . Теперь


переведём его в десятичную дробь. Записываем целую часть
и ставим запятую:

4,

Теперь посчитаем количество нулей в знаменателе дробной


части и количество цифр в числителе дробной части. Видим,
что количество нулей в знаменателе  и количество цифр в
числителе одинаково. Это даёт нам возможность сразу
записать числитель дробной части после запятой:

4,50

Значит неправильная дробь  при переводе в десятичную


обращается в 4,50

При решении задач, если в конце десятичной дроби


оказываются нули, их можно отбросить. Давайте и мы
отбросим ноль в нашем ответе. Тогда мы получим 4,5

Это одна из интересных особенностей десятичных дробей.


Она заключается в том, что нули которые стоят в конце
дроби, не придают этой дроби никакого веса. Другими
словами, десятичные дроби 4,50 и 4,5 равны и между ними
можно поставить знак равенства:

4,50 = 4,5

Возникает вопрос «а почему так происходит?» Ведь на вид


4,50 и 4,5 разные дроби. Весь секрет кроется в основном
свойстве дроби, котором мы изучали ранее. Мы попробуем
доказать, почему равны десятичные дроби 4,50 и 4,5, но
394

после изучения следующей темы, которая называется


«перевод десятичной дроби в смешанное число».

Перевод десятичной дроби в смешанное число

Любая десятичная дробь может быть обратно переведена в


смешанное число. Для этого достаточно уметь читать
десятичные дроби.

Например, переведём 6,3 в смешанное число. 6,3 это шесть


целых и три десятых. Записываем сначала шесть целых:

и рядом три десятых:

Пример 2. Перевести десятичную дробь 3,002 в смешанное


число

3,002 это три целых и две тысячных. Записываем сначала


три целых

и рядом записываем две тысячных:

3
395

Пример 3. Перевести десятичную дробь 4,50 в смешанное


число

4,50 это четыре целых и пятьдесят сотых. Записываем


четыре целых

и рядом пятьдесят сотых:

Кстати, давайте вспомним последний пример из предыдущей


темы. Мы сказали, что десятичные дроби 4,50 и 4,5 равны.
Также мы сказали, что ноль можно отбросить. Докажем, что
десятичные 4,50 и 4,5 равны. Для этого переведем обе
десятичные дроби в смешанные числа.

После перевода в смешанное число десятичная дробь 4,50

обращается в , а десятичная дробь 4,5 обращается в

Имеем два смешанных числа   и  . Переведём эти


смешанные числа в неправильные дроби:

Теперь имеем две дроби    и  . Теперь вспоминаем


основное свойство дроби, которое говорит о том, что при
умножении (или делении) числителя и знаменателя дроби на
одно и то же число, значение дроби не меняется.
396

Давайте разделим числитель и знаменатель первой дроби

на число 10

Получили , а это есть вторая дробь. Значит и  равны


между собой и равны одному и тому же значению:

  = 

Попробуйте на калькуляторе разделить сначала 450 на 100,


а затем 45 на 10. Забавная штука получится.

Перевод десятичной дроби в обыкновенную дробь

Любая десятичная дробь может быть обратно переведена в


обыкновенную дробь. Для этого опять же достаточно уметь
читать десятичные дроби. Например, переведём 0,3 в
обыкновенную дробь. 0,3 это ноль целых и три десятых.
Записываем сначала ноль целых:

и рядом три десятых 0 . Ноль по традиции не записывают,

поэтому окончательный ответ будет не 0 , а просто .

Пример 2. Перевести десятичную дробь 0,02 в


обыкновенную дробь.
397

0,02 это ноль целых и две сотых. Ноль не записываем,


поэтому сразу записываем две сотых

Пример 3. Перевести 0,00005 в обыкновенную дробь

0,00005 это ноль целых и пять сто тысячных. Ноль не


записываем, поэтому сразу записываем пять сто тысячных 

Пример 4. Перевести 3,5 в обыкновенную дробь

Сначала переведём данную десятичную дробь в смешанное


число:

Теперь смешанное число переведём в неправильную


(обыкновенную) дробь:

Пример 5. Перевести 1,25 в обыкновенную дробь

Сначала переведём данную десятичную дробь в смешанное


число:
398

Теперь смешанное число  переведём в неправильную


(обыкновенную) дробь:
399

Действия с десятичными дробями

Десятичные дроби можно складывать, вычитать, умножать и


делить. Также, десятичные дроби можно сравнивать между
собой.

В этом уроке мы рассмотрим каждую из этих операций по


отдельности.

Содержание урока

 Сложение десятичных дробей


 Разряды в десятичных дробях
 Вычитание десятичных дробей
 Умножение десятичных дробей
 Умножение десятичной дроби на обычное число
 Умножение десятичных дробей на 10, 100, 1000
 Умножение десятичных дробей на 0,1 0,01 и 0,001
 Деление меньшего числа на большее. Продвинутый
уровень.
 Деление чисел без остатка
 Деление десятичной дроби на обычное число
 Деление десятичной дроби на десятичную дробь
 Деление десятичной дроби на 10, 100, 1000
 Деление десятичной дроби на 0,1, 0,01 и 0,001
 Задания для самостоятельного решения

Сложение десятичных дробей

Как мы знаем, десятичная дробь состоит из целой и дробной


части. При сложении десятичных дробей, целые и дробные
части складываются по отдельности.
400

Например, сложим десятичные дроби 3,2 и 5,3. Десятичные


дроби удобнее складывать в столбик.

Запишем сначала эти две дроби в столбик, при этом целые


части обязательно должны быть под целыми, а дробные под
дробными. В школе это требование называют «запятая под
запятой».

Запишем дроби в столбик так, чтобы запятая оказалась под


запятой:

Складываем дробные части: 2 + 3 = 5. Записываем пятёрку в


дробной части нашего ответа:

Теперь складываем целые части: 3 + 5 = 8. Записываем


восьмёрку в целой части нашего ответа:

Теперь отделяем запятой целую часть от дробной. Для этого


опять же соблюдаем правило «запятая под запятой»:
401

Получили ответ 8,5. Значит, выражения 3,2 + 5,3 равно 8,5

3,2 + 5,3 = 8,5

На самом деле не всё так просто как кажется на первый


взгляд. Здесь тоже имеются свои подводные камни, о
которых мы сейчас поговорим.

Разряды в десятичных дробях

У десятичных дробей, как и у обычных чисел, есть свои


разряды. Это разряды десятых, разряды сотых, разряды
тысячных. При этом разряды начинаются после запятой.

Первая цифра после запятой отвечает за разряд десятых,


вторая цифра после запятой за разряд сотых, третья цифра
после запятой за разряд тысячных.

Разряды в десятичных дробях хранят в себе нéкоторую


полезную информацию. В частности, они сообщают сколько в
десятичной дроби десятых частей, сотых частей и тысячных
частей.

Например, рассмотрим десятичную дробь 0,345

Позиция, где находится тройка, называется разрядом


десятых
402

Позиция, где находится четвёрка, называется разрядом


сотых

Позиция, где находится пятёрка, называется разрядом


тысячных

Посмотрим на данный рисунок. Видим, что в разряде


десятых располагается тройка. Это говорит о том, что в

десятичной дроби 0,345 содержится три десятых   .

Смотрим дальше. В разряде сотых располагается четвёрка.


Это говорит о том, что в десятичной дроби 0,345 содержится

четыре сотых   .

Смотрим дальше. В разряде тысячных находится пятёрка.


Это говорит о том, что в десятичной дроби 0,345 содержится

пять тысячных .

Если мы сложим дроби ,    и  то получим


изначальную десятичную дробь 0,345
403

Сначала мы получили ответ , но перевели его в


десятичную дробь и получили 0,345.

При сложении десятичных дробей соблюдаются те же


правила что и при сложении обычных чисел. Сложение
десятичных дробей происходит по разрядам: десятые части
складываются с десятыми частями, сотые с сотыми,
тысячные с тысячными.

Поэтому при сложении десятичных дробей требуют


соблюдать правило «запятая под запятой». Запятая под
запятой обеспечивает тот самый порядок, в котором десятые
части складываются с десятыми, сотые с сотыми, тысячные с
тысячными.

Пример 1. Найти значение выражения 1,5 + 3,4

Записываем в столбик данное выражение, соблюдая


правило «запятая под запятой»:

В первую очередь складываем дробные части 5 + 4 = 9.


Записываем девятку в дробной части нашего ответа:
404

Теперь складываем целые части 1 + 3 = 4. Записываем


четвёрку в целой части нашего ответа:

Теперь отделяем запятой целую часть от дробной. Для этого


опять же соблюдаем правило «запятая под запятой»:

Получили ответ 4,9. Значит значение выражения 1,5 + 3,4


равно 4,9

1,5 + 3,4 = 4,9

Пример 2. Найти значение выражения: 3,51 + 1,22

Записываем в столбик данное выражение, соблюдая


правило «запятая под запятой»
405

В первую очередь складываем дробную часть, а именно


сотые части 1+2=3. Записываем тройку в сотой части нашего
ответа:

Теперь складываем десятые части 5+2=7. Записываем


семёрку в десятой части нашего ответа:

Теперь складываем целые части 3+1=4. Записываем


четвёрку в целой части нашего ответа:

Отделяем запятой целую часть от дробной, соблюдая


правило «запятая под запятой»:

Получили ответ 4,73. Значит значение выражения 3,51 + 1,22


равно 4,73

3,51 + 1,22 = 4,73


406

Как и в обычных числах, при сложении десятичных дробей


может произойти переполнение разряда. В этом случае в
ответе записывается одна цифра, а остальные переносят на
следующий разряд.

Пример 3. Найти значение выражения 2,65 + 3,27

Записываем в столбик данное выражение:

Складываем сотые части 5+7=12. Число 12 не поместится в


сотой части нашего ответа. Поэтому в сотой части
записываем цифру 2, а единицу переносим на следующий
разряд:

Теперь складываем десятые части 6+2=8 плюс единица,


которая досталась от предыдущей операции, получим 9.
Записываем цифру 9 в десятой части нашего ответа:

Теперь складываем целые части 2+3=5. Записываем цифру 5


в целой части нашего ответа:
407

Отделяем запятой целую часть от дробной:

Получили ответ 5,92. Значит значение выражения 2,65 + 3,27


равно 5,92

2,65 + 3,27 = 5,92

Пример 4. Найти значение выражения 9,5 + 2,8

Записываем в столбик данное выражение

Складываем дробные части 5 + 8 = 13. Число 13 не


поместится в дробной часть нашего ответа, поэтому сначала
записываем цифру 3, а единицу переносим на следующий
разряд, точнее переносим её к целой части:
408

Теперь складываем целые части 9+2=11 плюс единица,


которая досталась от предыдущей операции, получаем 12.
Записываем число 12 в целой части нашего ответа:

Отделяем запятой целую часть от дробной:

Получили ответ 12,3. Значит значение выражения 9,5 + 2,8


равно 12,3

9,5 + 2,8 = 12,3

При сложении десятичных дробей количество цифр после


запятой в обеих дробях должно быть одинаковым. Если
цифр не хватает, то эти места в дробной части заполняются
нулями.

Пример 5. Найти значение выражения: 12,725 + 1,7

Прежде чем записывать в столбик данное выражение,


сделаем количество цифр после запятой в обеих дробях
одинаковым. В десятичной дроби 12,725 после запятой три
цифры, а в дроби 1,7 только одна. Значит в дроби 1,7 в конце
нужно добавить два нуля. Тогда получим дробь 1,700. Теперь
409

можно записать в столбик данное выражение и начать


вычислять:

Складываем тысячные части 5+0=5. Записываем цифру 5 в


тысячной части нашего ответа:

Складываем сотые части 2+0=2. Записываем цифру 2 в


сотой части нашего ответа:

Складываем десятые части 7+7=14. Число 14 не поместится


в десятой части нашего ответа. Поэтому сначала
записываем цифру 4, а единицу переносим на следующий
разряд:

Теперь складываем целые части 12+1=13 плюс единица,


которая досталась от предыдущей операции, получаем 14.
Записываем число 14 в целой части нашего ответа:
410

Отделяем запятой целую часть от дробной:

Получили ответ 14,425. Значит значение выражения


12,725+1,700 равно 14,425

12,725+ 1,700 = 14,425

Вычитание десятичных дробей

При вычитании десятичных дробей нужно соблюдать те же


правила что и при сложении: «запятая под запятой» и
«равное количества цифр после запятой».

Пример 1. Найти значение выражения 2,5 − 2,2

Записываем в столбик данное выражение, соблюдая


правило «запятая под запятой»:

Вычисляем дробную часть 5−2=3. Записываем цифру 3 в


десятой части нашего ответа:
411

Вычисляем целую часть 2−2=0. Записываем ноль в целой


части нашего ответа:

Отделяем запятой целую часть от дробной:

Получили ответ 0,3. Значит значение выражения 2,5 − 2,2


равно 0,3

2,5 − 2,2 = 0,3

Пример 2. Найти значение выражения 7,353 — 3,1

В этом выражении разное количество цифр после запятой. В


дроби 7,353 после запятой три цифры, а в дроби 3,1 только
одна. Значит в дроби 3,1 в конце нужно добавить два нуля,
чтобы сделать количество цифр в обеих дробях одинаковым.
Тогда получим 3,100.

Теперь можно записать в столбик данное выражение и


вычислить его:
412

Получили ответ 4,253. Значит значение выражения 7,353 −


3,1 равно 4,253

7,353 — 3,1 = 4,253

Как и в обычных числах, иногда придётся занимать единицу у


соседнего разряда, если вычитание станет невозможным.

Пример 3. Найти значение выражения 3,46 − 2,39

Вычитаем сотые части 6−9. От число 6 не вычесть число 9.


Поэтому нужно занять единицу у соседнего разряда. Заняв
единицу у соседнего разряда число 6 обращается в число 16.
Теперь можно вычислить сотые части 16−9=7. Записываем
семёрку в сотой части нашего ответа:

Теперь вычитаем десятые части. Поскольку мы заняли в


разряде десятых одну единицу, то цифра, которая там
располагалась, уменьшилась на одну единицу. Другими
словами, в разряде десятых теперь не цифра 4, а цифра 3.
Вычислим десятые части 3−3=0. Записываем ноль в десятой
части нашего ответа:
413

Теперь вычитаем целые части 3−2=1. Записываем единицу в


целой части нашего ответа:

Отделяем запятой целую часть от дробной:

Получили ответ 1,07. Значит значение выражения 3,46−2,39


равно 1,07

 3,46−2,39=1,07

Пример 4. Найти значение выражения 3−1,2

В этом примере из целого числа вычитается десятичная


дробь. Запишем данное выражение столбиком так, чтобы
целая часть десятичной дроби 1,2 оказалась под числом 3

Теперь сделаем количество цифр после запятой


одинаковым. Для этого после числа 3 поставим запятую и
допишем один ноль:
414

Теперь вычитаем десятые части: 0−2. От нуля не вычесть


число 2. Поэтому нужно занять единицу у соседнего разряда.
Заняв единицу у соседнего разряда, 0 обращается в число
10. Теперь можно вычислить десятые части 10−2=8.
Записываем восьмёрку в десятой части нашего ответа:

Теперь вычитаем целые части. Раньше в целой


располагалось число 3, но мы заняли у него одну единицу. В
результате оно обратилось в число 2. Поэтому из 2 вычитаем
1. 2−1=1. Записываем единицу в целой части нашего ответа:

Отделяем запятой целую часть от дробной:

Получили ответ 1,8. Значит значение выражения 3−1,2 равно


1,8

3 − 1,2 = 1,8
415

Умножение десятичных дробей

Умножение десятичных дробей это просто и даже


увлекательно. Чтобы перемножить десятичные дроби, нужно
перемножить их как обычные числа, не обращая внимания на
запятые.

Получив ответ, необходимо отделить запятой целую часть от


дробной. Чтобы сделать это, надо посчитать количество
цифр после запятой в обеих дробях, затем в ответе
отсчитать справа столько же цифр и поставить запятую.

Пример 1. Найти значение выражения 2,5 × 1,5

Перемножим эти десятичные дроби как обычные числа, не


обращая внимания на запятые. Чтобы не обращать
внимания на запятые, можно на время представить, что они
вообще отсутствуют:

Получили 375. В этом числе необходимо отделить запятой


целую часть от дробной. Для этого нужно посчитать
количество цифр после запятой в дробях 2,5 и 1,5. В первой
416

дроби после запятой одна цифра, во второй дроби тоже


одна. Итого две цифры.

Возвращаемся к числу 375 и начинаем двигаться справа


налево. Нам нужно отсчитать две цифры справа и поставить
запятую:

Получили ответ 3,75. Значит значение выражения 2,5 × 1,5


равно 3,75

2,5 × 1,5 = 3,75

Пример 2. Найти значение выражения 12,85 × 2,7

Перемножим эти десятичные дроби, не обращая внимания


на запятые:
417

Получили 34695. В этом числе нужно отделить запятой


целую часть от дробной. Для этого необходимо посчитать
количество цифр после запятой в дробях 12,85 и 2,7. В дроби
12,85 после запятой две цифры, в дроби 2,7 одна цифра —
итого три цифры.

Возвращаемся к числу 34695 и начинаем двигаться справа


налево. Нам нужно отсчитать три цифры справа и поставить
запятую:

Получили ответ 34,695. Значит значение выражения 12,85 ×


2,7 равно 34,695
418

12,85 × 2,7 = 34,695

Умножение десятичной дроби на обычное число

Иногда возникают ситуации, когда требуется умножить


десятичную дробь на обычное число.

Чтобы перемножить десятичную дробь и обычное число,


нужно перемножить их, не обращая внимания на запятую в
десятичной дроби. Получив ответ, необходимо отделить
запятой целую часть от дробной. Для этого нужно посчитать
количество цифр после запятой в десятичной дроби, затем в
ответе отсчитать справа столько же цифр и поставить
запятую.

Например, умножим 2,54 на 2

Умножаем десятичную дробь 2,54 на обычное число 2, не


обращая внимания на запятую:

Получили число 508. В этом числе нужно отделить запятой


целую часть от дробной. Для этого необходимо посчитать
количество цифр после запятой в дроби 2,54. В дроби 2,54
после запятой две цифры.
419

Возвращаемся к числу 508 и начинаем двигаться справа


налево. Нам нужно отсчитать две цифры справа и поставить
запятую:

Получили ответ 5,08. Значит значение выражения 2,54 × 2


равно 5,08

2,54 × 2 = 5,08

Умножение десятичных дробей на 10, 100, 1000

Умножение десятичных дробей на 10, 100 или 1000


выполняется таким же образом, как и умножение десятичных
дробей на обычные числа. Нужно выполнить умножение, не
обращая внимания на запятую в десятичной дроби, затем в
ответе отделить целую часть от дробной, отсчитав справа
столько же цифр, сколько было цифр после запятой в
десятичной дроби.

Например, умножим 2,88 на 10


420

Умножим десятичную дробь 2,88 на 10, не обращая


внимания на запятую в десятичной дроби:

Получили 2880. В этом числе нужно отделить запятой целую


часть от дробной. Для этого необходимо посчитать
количество цифр после запятой в дроби 2,88. Видим, что в
дроби 2,88 после запятой две цифры.

Возвращаемся к числу 2880 и начинаем двигаться справа


налево. Нам нужно отсчитать две цифры справа и поставить
запятую:

Получили ответ 28,80. Отбросим последний ноль — получим


28,8. Значит значение выражения 2,88×10 равно 28,8
421

2,88 × 10 = 28,8

Есть и второй способ умножения десятичных дробей на 10,


100, 1000. Этот способ намного проще и удобнее. Он
заключается в том, что запятая в десятичной дроби
передвигается вправо на столько цифр, сколько нулей во
множителе.

Например, решим предыдущий пример 2,88×10 этим


способом. Не приводя никаких вычислений, сразу же
смотрим на множитель 10. Нас интересует сколько в нём
нулей. Видим, что в нём один ноль. Теперь в дроби 2,88
передвигаем запятую вправо на одну цифру, получим 28,8.

2,88 × 10 = 28,8

Попробуем умножить 2,88 на 100. Сразу же смотрим на


множитель 100. Нас интересует сколько в нём нулей. Видим,
что в нём два нуля. Теперь в дроби 2,88 передвигаем
запятую вправо на две цифры, получаем 288

2,88 × 100 = 288

Попробуем умножить 2,88 на 1000. Сразу же смотрим на


множитель 1000. Нас интересует сколько в нём нулей.
Видим, что в нём три нуля. Теперь в дроби 2,88 передвигаем
запятую вправо на три цифры. Третьей цифры там нет,
поэтому мы дописываем ещё один ноль. В итоге получаем
2880.
422

2,88 × 1000 = 2880

Умножение десятичных дробей на 0,1  0,01 и 0,001

Умножение десятичных дробей на 0,1,  0,01 и 0,001


происходит таким же образом, как и умножение десятичной
дроби на десятичную дробь. Необходимо перемножить
дроби, как обычные числа, и в ответе поставить запятую,
отсчитав столько цифр справа, сколько цифр после запятой в
обеих дробях.

Например, умножим 3,25 на 0,1

Умножаем эти дроби, как обычные числа, не обращая


внимания на запятые:

Получили 325. В этом числе нужно отделить запятой целую


часть от дробной. Для этого необходимо посчитать
количество цифр после запятой в дробях 3,25 и 0,1. В дроби
3,25 после запятой две цифры, в дроби 0,1 одна цифра.
Итого три цифры.

Возвращаемся к числу 325 и начинаем двигаться справа


налево. Нам нужно отсчитать три цифры справа и поставить
запятую. Отсчитав три цифры мы обнаруживаем, что цифры
423

закончились. В этом случае нужно дописать один ноль и


поставить запятую:

Получили ответ 0,325. Значит значение выражения 3,25 × 0,1


равно 0,325

3,25 × 0,1 = 0,325

Есть и второй способ умножения десятичных дробей на 0,1, 


0,01  и 0,001. Этот способ намного проще и удобнее. Он
заключается в том, что запятая в десятичной дроби
передвигается влево на столько цифр, сколько нулей во
множителе.

Например, решим предыдущий пример 3,25 × 0,1 этим


способом. Не приводя никаких вычислений сразу же смотрим
на множитель 0,1. Нас интересует сколько в нём нулей.
Видим, что в нём один ноль. Теперь в дроби 3,25
передвигаем запятую влево на одну цифру. Передвинув
424

запятую на одну цифру влево мы видим, что перед тройкой


больше нет никаких цифр. В этом случае дописываем один
ноль и ставим запятую. В результате получаем 0,325

3,25 × 0,1 = 0,325

Попробуем умножить 3,25 на 0,01. Сразу же смотрим на


множитель 0,01. Нас интересует сколько в нём нулей. Видим,
что в нём два нуля. Теперь в дроби 3,25 передвигаем
запятую влево на две цифры, получаем 0,0325

3,25 × 0,01 = 0,0325

Попробуем умножить 3,25 на 0,001. Сразу же смотрим на


множитель 0,001. Нас интересует сколько в нём нулей.
Видим, что в нём три нуля. Теперь в дроби 3,25 передвигаем
запятую влево на три цифры, получаем 0,00325

3,25 × 0,001 = 0,00325

Нельзя путать умножение десятичных дробей на 0,1,  0,001 и


0,001 с умножением на 10, 100, 1000. Типичная ошибка
большинства людей.

При умножении на 10, 100, 1000 запятая переносится вправо


на столько же цифр сколько нулей во множителе.

А при умножении на 0,1,  0,01 и 0,001 запятая переносится


влево на столько же цифр сколько нулей во множителе.

Если на первых порах это сложно запомнить, можно


пользоваться первым способом, в котором умножение
выполняется как с обычными числами. В ответе нужно будет
425

отделить целую часть от дробной, отсчитав справа столько


же цифр, сколько цифр после запятой в обеих дробях.

Деление меньшего числа на большее. Продвинутый уровень.

В одном из предыдущих уроков мы сказали, что при делении


меньшего числа на большее получается дробь, в числителе
которой делимое, а в знаменателе – делитель.

Например, чтобы разделить одно яблоко на двоих, нужно в


числитель записать 1 (одно яблоко), а в знаменатель

записать 2 (двое друзей). В результате получим дробь  .

Значит каждому другу достанется по   яблока. Другими

словами, по половине яблока. Дробь   это ответ к задаче


«как разделить одно яблоко на двоих»
426

Оказывается, можно решать эту задачу и дальше, если


разделить 1 на 2. Ведь дробная черта в любой дроби

означает деление, а значит и в дроби   это деление


разрешено. Но как? Мы ведь привыкли к тому, что делимое
всегда больше делителя. А здесь наоборот, делимое меньше
делителя.

Всё станет ясным, если вспомнить, что дробь означает


дробление, деление, разделение. А значит и единица может
быть раздроблена на сколько угодно частей, а не только на
две части.

При разделении меньшего числа на большее получается


десятичная дробь, в которой целая часть будет 0 (нулевой).
Дробная часть же может быть любой.

Итак, разделим 1 на 2. Решим этот пример уголком:


427

Единицу на два просто так нацело не разделить. Если задать


вопрос «сколько двоек в единице», то ответом будет 0.
Поэтому в частном записываем 0 и ставим запятую:

Теперь как обычно умножаем частное на делитель, чтобы


вытащить остаток:

Настал момент, когда единицу можно дробить на две части.


Для этого справа от полученной единички дописываем ещё
один ноль:

Получили 10. Делим 10 на 2, получаем 5. Записываем


пятёрку в дробной части нашего ответа:

Теперь вытаскиваем последний остаток, чтобы завершить


вычисление. Умножаем 5 на 2, получаем 10
428

Получили ответ 0,5. Значит дробь   равна 0,5

Половину яблока   можно записать и с помощью десятичной


дроби 0,5. Если сложить эти две половинки (0,5 и 0,5), мы
опять получим изначальное одно целое яблоко:
429

Этот момент также можно понять, если представить, как 1 см


делится на две части. Если 1 сантиметр разделить на 2
части, то получится 0,5 см

Пример 2. Найти значение выражения 4 : 5

Сколько пятёрок в четвёрке? Нисколько. Записываем в


частном 0 и ставим запятую:

Умножаем 0 на 5, получаем 0. Записываем ноль под


четвёркой. Сразу же вычитаем этот ноль из делимого:
430

Теперь начнём дробить (делить) четвёрку на 5 частей. Для


этого справа от 4 дописываем ноль и делим 40 на 5,
получаем 8. Записываем восьмёрку в частном.

Завершаем пример, умножив 8 на 5, и получив 40:

Получили ответ 0,8. Значит значение выражения 4 : 5 равно


0,8

Пример 3. Найти значение выражения 5 : 125

Сколько чисел 125 в пятёрке? Нисколько. Записываем 0 в


частном и ставим запятую:

Умножаем 0 на 125, получаем 0. Записываем 0 под пятёркой.


Сразу же вычитаем из пятёрки 0
431

Теперь начнём дробить (делить) пятёрку на 125 частей. Для


этого справа от этой пятёрки запишем ноль:

Делим 50 на 125. Сколько чисел 125 в числе 50? Нисколько.


Значит в частном опять записываем 0

Умножаем 0 на 125, получаем 0. Записываем этот ноль под


50. Сразу же вычитаем 0 из 50

Теперь делим число 50 на 125 частей. Для этого справа от 50


запишем ещё один ноль:

Делим 500 на 125. Сколько чисел 125 в числе 500. В числе


500 четыре числа 125. Записываем четвёрку в частном:
432

Завершаем пример, умножив 4 на 125, и получив 500

Получили ответ 0,04. Значит значение выражения 5 : 125


равно 0,04

Деление чисел без остатка

В уроке деление мы научились делить числа с остатком.


Например, чтобы разделить 9 на 5, мы поступали
следующим образом:

и далее говорили, что «девять разделить на пять будет


один и четыре в остатке».

Теперь мы получили необходимые знания, чтобы разделить


9 на 5 без остатка. Наша задача раздробить остаток 4 на 5
433

частей. Другими словами, разделить меньшее число на


большее.

Итак, поставим в частном после единицы запятую, тем


самым указывая, что деление целых частей закончилось и
мы приступаем к дробной части:

Допишем ноль к остатку 4

Теперь делим 40 на 5, получаем 8. Записываем восьмёрку в


частном:

Что делать дальше мы уже знаем. Вытаскиваем остаток


(если есть). Умножаем восьмёрку на делитель 5, и
записываем полученный результат под 40:
434

40−40=0. Получили 0 в остатке. Значит деление на этом


полностью завершено. При делении 9 на 5 получается
десятичная дробь 1,8:

9 : 5 = 1,8

Пример 2. Разделить 84 на 5 без остатка

Сначала разделим 84 на 5 как обычно с остатком:

Получили в частном 16 и еще 4 в остатке. Теперь разделим


этот остаток на 5. Поставим в частном запятую, а к остатку 4
допишем 0

Теперь делим 40 на 5, получаем 8. Записываем восьмерку в


частном после запятой:
435

и завершаем пример, проверив есть ли еще остаток:

Деление десятичной дроби на обычное число

Десятичная дробь, как мы знаем состоит из целой и дробной


части. При делении десятичной дроби на обычное число в
первую очередь нужно:

 разделить целую часть десятичной дроби на это число;


 после того, как целая часть будет разделена, нужно в
частном сразу же поставить запятую и продолжить
вычисление, как в обычном делении.

Например, разделим 4,8 на 2

Запишем этот пример уголком:


436

Теперь разделим целую часть на 2. Четыре разделить на два


будет два. Записываем двойку в частном и сразу же ставим
запятую:

Теперь умножаем частное на делитель и смотрим есть ли


остаток от деления:

4−4=0. Остаток равен нулю. Ноль пока не записываем,


поскольку решение не завершено. Далее продолжаем
вычислять, как в обычном делении. Сносим 8 и делим её на 2

8 : 2 = 4. Записываем четвёрку в частном и сразу умножаем


её на делитель:

Получили ответ 2,4. Значение выражения 4,8 : 2 равно 2,4


437

 Пример 2. Найти значение выражения 8,43 : 3

Делим 8 на 3, получаем 2. Сразу же ставим запятую после


двойки:

Теперь умножаем частное на делитель 2 × 3 = 6. Записываем


шестёрку под восьмёркой и находим остаток:

Далее продолжаем вычислять, как в обычном делении.


Сносим 4

Делим 24 на 3, получаем 8. Записываем восьмёрку в


частном. Сразу же умножаем её на делитель, чтобы найти
остаток от деления:
438

24−24=0. Остаток равен нулю. Ноль пока не записываем.


Сносим последнюю тройку из делимого и делим на 3,
получим 1. Сразу же умножаем 1 на 3, чтобы завершить этот
пример:

Получили ответ 2,81. Значит значение выражения 8,43 : 3


равно 2,81

Деление десятичной дроби на десятичную дробь

Чтобы разделить десятичную дробь на десятичную дробь,


надо в делимом и в делителе перенести запятую вправо на
столько же цифр, сколько их после запятой в делителе, и
затем выполнить деление на обычное число.

Например, разделим 5,95 на 1,7

Запишем уголком данное выражение

Теперь в делимом и в делителе перенесём запятую вправо


на столько же цифр, сколько их после запятой в делителе. В
делителе после запятой одна цифра. Значит мы должны в
439

делимом и в делителе перенести запятую вправо на одну


цифру. Переносим:

После перенесения запятой вправо на одну цифру


десятичная дробь 5,95 обратилась в дробь 59,5. А
десятичная дробь 1,7 после перенесения запятой вправо на
одну цифру обратилась в обычное число 17. А как делить
десятичную дробь на обычное число мы уже знаем.
Дальнейшее вычисление не составляет особого труда:

Запятая переносится вправо с целью облегчить деление. Это


допускается по причине того, что при умножении или
делении делимого и делителя на одно и то же число, частное
не меняется. Что это значит?

Это одна из интересных особенностей деления. Его


называют свойством частного. Рассмотрим выражение 9 : 3 =
3. Если в этом выражении делимое и делитель умножить или
разделить на одно и то же число, то частное 3 не изменится.

Давайте умножим делимое и делитель на 2, и посмотрим, что


из этого получится:

(9 × 2) : (3 × 2) = 18 : 6 = 3
440

Как видно из примера, частное не поменялось.

Тоже самое происходит, когда мы переносим запятую в


делимом и в делителе. В предыдущем примере, где мы
делили 5,91 на 1,7 мы перенесли в делимом и делителе
запятую на одну цифру вправо. После переноса запятой,
дробь 5,91 преобразовалась в дробь 59,1 а дробь 1,7
преобразовалась в обычное число 17. На самом деле
здесь происходило умножение на 10. Вот как это выглядело:

5,91 × 10 = 59,1

1,7 × 10 = 17

Поэтому от количества цифр после запятой в делителе


зависит то, на что будет умножено делимое и делитель.
Другими словами, от количества цифр после запятой в
делителе будет зависеть то, на сколько цифр в делимом и в
делителе запятая будет перенесена вправо.

Деление десятичной дроби на 10, 100, 1000

Деление десятичной дроби на 10, 100, или 1000


осуществляется таким же образом, как и деление десятичной
дроби на обычное число. Например, разделим 2,1 на 10.
Решим этот пример уголком:
441

Но есть и второй способ. Он более лёгкий. Суть этого


способа в том, что запятая в делимом переносится влево на
столько цифр, сколько нулей в делителе.

Решим предыдущий пример этим способом. 2,1 : 10.


Смотрим на делитель. Нас интересует сколько в нём нулей.
Видим, что там один ноль. Значит в делимом 2,1 нужно
перенести запятую влево на одну цифру. Переносим запятую
влево на одну цифру и видим, что там больше не осталось
цифр. В этом случае перед цифрой дописываем ещё один
ноль. В итоге получаем 0,21

2,1 : 10 = 0,21

Попробуем разделить 2,1 на 100. В числе 100 два нуля.


Значит в делимом 2,1 надо перенести запятую влево на две
цифры:

2,1 : 100 = 0,021

Попробуем разделить 2,1 на 1000. В числе 1000 три нуля.


Значит в делимом 2,1 надо перенести запятую влево на три
цифры:
442

2,1 : 1000 = 0,0021

Деление десятичной дроби на 0,1,  0,01  и  0,001

Деление десятичной дроби на 0,1,  0,01, и 0,001


осуществляется таким же образом, как и деление десятичной
дроби на десятичную дробь. В делимом и в делителе надо
перенести запятую вправо на столько цифр, сколько их после
запятой в делителе.

Например, разделим 6,3 на 0,1. В первую очередь перенесём


запятые в делимом и в делителе вправо на столько же цифр,
сколько их после запятой в делителе. В делителе после
запятой одна цифра. Значит переносим запятые в делимом и
в делителе вправо на одну цифру.

После перенесения запятой вправо на одну цифру,


десятичная дробь 6,3 превращается в обычное число 63, а
десятичная дробь 0,1 после перенесения запятой вправо на
одну цифру превращается в единицу. А разделить 63 на 1
очень просто:

63 : 1 = 63

Значит значение выражения 6,3 : 0,1 равно 63

6,3 : 0,1 = 63

Но есть и второй способ. Он более лёгкий. Суть этого


способа в том, что запятая в делимом переносится вправо на
столько цифр, сколько нулей в делителе.

Решим предыдущий пример этим способом. 6,3 : 0,1.


Смотрим на делитель. Нас интересует сколько в нём нулей.
443

Видим, что там один ноль. Значит в делимом 6,3 нужно


перенести запятую вправо на одну цифру. Переносим
запятую вправо на одну цифру и получаем 63

6,3 : 0,1 = 63

Попробуем разделить 6,3 на 0,01. В делителе 0,01 два нуля.


Значит в делимом 6,3 надо перенести запятую вправо на две
цифры. Но в делимом после запятой только одна цифра. В
этом случае в конце нужно дописать ещё один ноль. В
результате получим 630

6,3 : 0,01 = 630

Попробуем разделить 6,3 на 0,001. В делителе 0,001 три


нуля. Значит в делимом 6,3 надо перенести запятую вправо
на три цифры:

6,3 : 0,001 = 6300

Задания для самостоятельного решения


Задание 1. Выполните сложение:
0,6 + 0,3
Показать решение
Задание 2. Выполните сложение:
1,2 + 5,3
Показать решение
Задание 3. Выполните сложение:
444

1,6 + 0,4
Показать решение
Задание 4. Выполните сложение:
0,8 + 0,5
Показать решение
Задание 5. Выполните вычитание:
0,9 − 0,4
Показать решение
Задание 6. Выполните вычитание:
2 − 0,3
Показать решение
Задание 7. Выполните вычитание:
9 − 7,8
Показать решение
Задание 8. Выполните вычитание:
4 − 1,8
Показать решение
Задание 9. Выполните умножение:
3,2 × 1,8
Показать решение
Задание 10. Выполните умножение:
9,3 × 5,8
Показать решение
445

Задание 11. Выполните умножение:


0,23 × 0,07
Показать решение
Задание 12. Выполните умножение:
3,14 × 0,25
Показать решение
Задание 13. Выполните деление:
9,36 : 6
Показать решение
Задание 14. Выполните деление:
0,169 : 13
Показать решение
446

Применение десятичных дробей

Десятичные дроби имеют широкий спектр применения. Их


применяют в экономике, медицине, машиностроении и во
многих других отраслях. В данном уроке мы рассмотрим
некоторые элементарные операции, которые могут
пригодиться в будущем.

Содержание урока

 Сравнение десятичных дробей


 Нахождение десятичной дроби от числа
 Нахождение числа по десятичной дроби

Сравнение десятичных дробей

Чтобы сравнить две десятичные дроби, нужно в обеих


дробях сделать количество цифр после запятой одинаковым,
приписáв к одной из них нули. Затем отбросить запятые в
обеих дробях и сравнить получившиеся числа.

Например, сравним дроби 5,345 и 5,36. В первой дроби


после запятой три цифры, а во второй только две. В конце
второй дроби нужно приписать ещё один ноль, чтобы
количество цифр после запятой в обеих дробях стало
одинаковым.

Припишем в конце второй дроби ноль, тогда получим дроби


5,345 и 5,360. Теперь отбросим запятые в обеих дробях,
получим 5345 и 5360. Ну и  сравниваем их как обычные
числа. 5345 меньше, чем 5360

5345 < 5360


447

Значит и дробь 5,345 меньше, чем дробь 5,36

5,345 < 5,36

Пример 2. Сравнить десятичные дроби 6,782 и 6,71

Сделаем количество цифр в обеих дробях одинаковым:

6,782
6,710

Отбросим запятые:

6782
6710

6782 больше, чем 6710

6782 > 6710

Значит и дробь 6,782 больше, чем дробь 6,71

6,782 > 6,71

Нахождение десятичной дроби от числа

В прошлых уроках мы находили обыкновенную дробь от


числа. Для этого мы делили число на знаменатель дроби и
полученный результат умножáли на числитель дроби.

Например, чтобы найти  от числа 9, нужно число 9

разделить на знаменатель дроби и полученный результат


умножить на числитель этой же дроби
448

9:3=3
3×2=6

Значит  от числа 9 составляет 6.

Но находить можно и десятичные дроби от числа.


Нахождение десятичной дроби от числа намного проще.
Чтобы найти десятичную дробь от числа, достаточно это
число умножить на данную дробь.

Например, найдём 0,5 от числа 12. Чтобы найти 0,5 от числа


12, достаточно умножить 12 на 0,5

Получили ответ 6. Значит 0,5 от числа 12 составляет число 6.

Проверим правильно ли мы нашли 0,5 от числа 12. Сначала


переведём десятичную дробь 0,5 в обыкновенную дробь. 0,5
это ноль целых и пять десятых. Ноль не пишем, а
записываем сразу пять десятых:

Cделаем эту дробь более простой для нашей работы. Для


этого сократим её на 5
449

Получили дробь   . Теперь находим  от числа 12. Нетрудно

догадаться, что  от числа 12 это число 6. Значит и


десятичная дробь 0,5 от числа 12 была найдена правильно.

Пример 2. Найти 0,4 от одного метра

Один метр это 100 см. Чтобы найти 0,4 от 100 см, нужно 100
см умножить на 0,4. А чтобы умножить 100 см на 0,4 нужно в
0,4 перенести запятую вправо на две цифры:

100 × 0,4 = 40

Значит 0,4 от одного метра составляют 40 см.

Десятичную дробь также можно найти от десятичной дроби.


Например, найдем 0,5 от 2,5. Для этого 2,5 нужно умножить
на 0,5

2,5 × 0,5 = 1,25

Нахождение числа по десятичной дроби

В прошлых уроках мы находили число по обыкновенной


дроби. Чтобы найти всё число по его дроби мы делили
известное число на числитель дроби и полученный результат
умножали на знаменатель дроби.

Например, если   числа составляет 6, то для нахождения


всего числа, нужно 6 разделить на 2 и полученный результат
умножить на 4
450

6:2=3

3 × 4 = 12

Значит если всё число равно 12.

Находить число можно и по десятичной дроби. Нахождение


числа по десятичной дроби намного проще. Чтобы найти
число по десятичной дроби, достаточно это число разделить
на данную дробь.

Пример 1. 0,6 всего числа составляет 12, найти всё число.


Чтобы найти всё число, достаточно 12 разделить на 0,6.

Чтобы разделить 12 на 0,6 нужно в делимом и в делителе


перенести запятую вправо на одну цифру. Тогда получим
выражение 120 : 6. А это выражение вычисляется легко:

120 : 6 = 20

Значит, если 0,6 всего числа составляет 12, то всё число это
20.

Пример 2. Велосипедист проехал 3 км, что составляет 0,2


всего пути, который должен проехать велосипедист. Какой
путь должен проехать велосипедист?

Если 0,2 всего пути составляют 3 км, то для того чтобы найти
весь путь, нужно 3 разделить на 0,2. Чтобы разделить число
3 на 0,2 нужно в делимом и в делителе перенести запятую
451

вправо на одну цифру. Тогда получим выражение 30 : 2. А


это выражение вычисляется легко:

30 : 2 = 15

Значит весь путь, который должен проехать велосипедист


составляет 15 км.
452

Округление чисел

Сегодня мы рассмотрим довольно скучную тему, без


понимания которой двигаться дальше не представляется
возможным. Эта тема называется «округление чисел» или
по-другому «приближённые значения чисел».

Содержание урока

 Приближённые значения
 Округление чисел
 Первое правило округления
 Второе правило округления
 Округление десятичных дробей

Приближённые значения

Приближённые (или приблизительные) значения


применяются тогда, когда точное значение чего-то найти
невозможно, или же не важно чтобы это значение было
точным для исследуемого предмета.

Например, на словах можно сказать, что в городе проживает


полмиллиона человек, но это высказывание не будет
истинным, поскольку количество человек в городе меняется
— люди приезжают и уезжают, рождаются и умирают.
Поэтому правильнее будет сказать, что в городе проживает
приблизительно полмиллиона человек.

Ещё пример. В девять утра начинаются занятия. Мы вышли


из дома в 8:30. Через некоторое время по дороге мы
встретили своего товарища, который спросил у нас сколько
сейчас времени. Когда мы выходили из дома было 8:30, на
453

дорогу мы потратили какое-то неизвестное время. Мы не


знаем сколько сейчас времени, поэтому отвечаем товарищу:
«сейчас приблизительно около девяти часов».

В математике приближенные значения указываются с


помощью специального знака. Выглядит он следующим
образом:

Читается как «приближённо (приблизительно) равно».

Чтобы указать приближённое (приблизительное) значение,


прибегают к такому действию как округление чисел.

Округление чисел

Для нахождения приближенного значения применяется такое


действие как округление чисел.

Слово «округление» говорит само за себя. Округлить число


значит сделать его круглым. Круглым называется число,
которое оканчивается нулём. Например, следующие числа
являются круглыми:

10, 20, 30, 100, 300, 700, 1000

Любое число можно сделать круглым. Процедуру, при


которой число делают круглым, называют округлением
числá.

Мы уже занимались «округлением» чисел, когда делили


большие числа. Напомним, что для этого мы оставляли без
454

изменения цифру, образующую старший разряд, а


остальные цифры заменяли нулями. Но это были лишь
наброски, которые мы делали для облегчения деления.
Своего рода лайфхак. По факту, это даже не являлось
округлением чисел. Именно поэтому в начале данного
абзаца мы взяли слово округление в кавычки.

На самом деле, суть округления заключается в том, чтобы


найти ближайшее значение от исходного. При этом, число
может быть округлено до определённого разряда — до
разряда десятков, разряда сотен, разряда тысяч.

Рассмотрим простой пример на округление. Дано число 17.


Требуется округлить его до разряда десятков.

Не забегая вперёд попробуем понять, что означает


«округлить до разряда десятков». Когда говорят округлить
число 17, то надо понимать, что от нас требуют найти
ближайшее круглое число от числá 17. Причём в ходе этого
поиска возможно изменения коснутся и той цифры, которая
располагается в разряде десятков числá 17 (т.е цифры 1).

Предстáвим числа от 10 до 20 с помощью следующего


рисунка:

На рисунке видно, что для числá 17 ближайшее круглое


число это число 20. Значит ответ к задаче таким и будет: «17
приближённо равно 20″

17 ≈ 20
455

Мы нашли приближённое значение для 17, то есть округлили


его до разряда десятков. Видно, что после округления в
разряде десятков появилась новая цифра 2.

Попробуем найти приближённое число для числа 12. Для


этого снова предстáвим числа от 10 до 20 с помощью
рисунка:

На рисунке видно, что ближайшее круглое число для 12 это


число 10. Значит ответ к задаче таким и будет: 12
приближённо равно 10

12 ≈ 10

Мы нашли приближённое значение для 12, то есть округлили


его до разряда десятков. В этот раз цифра 1, которая стояла
в разряде десятков в числе 12, не пострадала от округления.
Почему так получилось мы расскажем позже.

Попробуем найти ближайшее число для числá 15. Снова


предстáвим числа от 10 до 20 с помощью рисунка:

На рисунке видно, что число 15 одинаково удалено от


круглых чисел 10 и 20. Возникает вопрос: которое из этих
круглых чисел будет приближённым значением для числа
15? Для таких случаев условились принимать бóльшее число
за приближённое. 20 больше чем 10, поэтому приближённое
значение для 15 будет число 20
456

15 ≈ 20

Округлять можно и большие числа. Естественно, для них


делать рисунки и изображать числа не представляется
возможным. Для них существует свой способ. Например,
округлим число 1456 до разряда десятков.

Итак, мы должны округлить 1456 до разряда десятков.


Разряд десятков начинается на пятёрке:

Теперь о существовании первых цифр 1 и 4 временно


забываем. Остается число 56

Теперь смотрим, какое круглое число находится ближе к


числу 56. Очевидно, что ближайшее круглое число для 56 это
число 60. Значит заменяем число 56 на число 60
457

Значит при округлении числа 1456 до разряда десятков


полýчим 1460

1456 ≈ 1460

Видно, что после округления числа 1456 до разряда


десятков, изменения коснулись и самогó разряда десятков. В
новом полученном числе в разряде десятков теперь
располагается цифра 6, а не 5.

Округлять числа можно не только до разряда десятков.


Округлять число можно до разряда сотен, тысяч, десятков
тысяч и так далее.

После того, как станóвится ясно, что округление это ни что


иное как поиск ближáйшего числá, можно применять готовые
правила, которые значительно облегчают округление чисел.
458

Первое правило округления

В предыдущих примерах мы видели, что округляя число до


определенного разряда, младшие разряды заменяются
нулями. Цифры, которые заменяются нулями, называют
отбрасываемыми цифрами.

Первое правило округления выглядит следующим образом:

Если при округлении чисел первая из отбрасываемых


цифр 0, 1, 2, 3 или 4, то сохраняемая цифра остаётся без
изменений.

Например, округлим число 123 до разряда десятков.

В первую очередь находим сохраняемую цифру. Для этого


надо прочитать самó задание. В разряде, о котором
говорится в задании и находится сохраняемая цифра. В
задании сказано: округлить число 123 до разряда десятков.

Видим, что в разряде десятков нахóдится двойка. Значит


сохраняемой цифрой является цифра 2

Теперь находим первую из отбрасываемых цифр. Первой из


отбрасываемых цифр является та цифра, которая следует
после сохраняемой цифрой. Видим, что первая цифра после
459

двойки это цифра 3. Значит цифра 3 является первой


отбрасываемой цифрой.

Теперь применяем правило округления. Оно говорит, что


если при округлении чисел первая из отбрасываемых цифр
0, 1, 2, 3 или 4, то сохраняемая цифра остаётся без
изменений.

Так и делаем. Оставляем без изменения сохраняемую


цифру, а все младшие разряды заменяем нулями. Другими
словами, всё что следует после цифры 2 заменяем нулями
(точнее нулём):

123 ≈ 120

Значит при округлении числа 123 до разряда десятков,


получаем приближённое ему число 120.
460

Теперь попробуем округлить то же самое число 123, но уже


до разряда сотен.

Нам требуется округлить число 123 до разряда сотен. Снова


ищем сохраняемую цифру. В этот раз сохраняемой цифрой
является 1, поскольку мы округляем число до разряда сотен.

Теперь находим первую из отбрасываемых цифр. Первой из


отбрасываемых цифр является та цифра, которая следует
после сохраняемой цифрой. Видим, что первая цифра после
единицы это цифра 2. Значит цифра 2 является первой
отбрасываемой цифрой:
461

Теперь применим правило. Оно говорит, что если при


округлении чисел первая из отбрасываемых цифр 0, 1, 2, 3
или 4, то сохраняемая цифра остаётся без изменений.

Так и делаем. Оставляем без изменения сохраняемую


цифру, а все младшие разряды заменяем нулями. Другими
словами, всё что следует после цифры 1 заменяем нулями:

123 ≈ 100

Значит при округлении числа 123 до разряда сотен,


получаем приближённое ему число 100.

Пример 3. Округлить число 1234 до разряда десятков.

Здесь сохраняемая цифра это 3. А первая отбрасываемая


цифра это 4. Согласно правилу, если при округлении чисел
первая из отбрасываемых цифр 0, 1, 2, 3 или 4, то
сохраняемая цифра остаётся без изменений.

Значит оставляем сохраняемую цифру 3 без изменений, а


всё что располагается после неё заменяем нулём:

1234 ≈ 1230

Пример 4. Округлить число 1234 до разряда сотен.

Здесь сохраняемая цифра это 2. А первая отбрасываемая


цифра это 3. Согласно правилу, если при округлении чисел
первая из отбрасываемых цифр 0, 1, 2, 3 или 4, то
сохраняемая цифра остаётся без изменений.
462

Значит оставляем сохраняемую цифру 2 без изменений, а


всё что располагается после неё заменяем нулями:

1234 ≈ 1200

Пример 3. Округлить число 1234 до разряда тысяч.

Здесь сохраняемая цифра это 1. А первая отбрасываемая


цифра это 2. Согласно правилу, если при округлении чисел
первая из отбрасываемых цифр 0, 1, 2, 3 или 4, то
сохраняемая цифра остаётся без изменений.

Значит оставляем сохраняемую цифру 1 без изменений, а


всё что располагается после неё заменяем нулями:

1234 ≈ 1000

Второе правило округления

Второе правило округления выглядит следующим образом:

Если при округлении чисел первая из отбрасываемых


цифр 5, 6, 7, 8 или 9, то сохраняемая цифра
увеличивается на единицу.

Например, округлим число 675 до разряда десятков.

В первую очередь находим сохраняемую цифру. Для этого


надо прочитать само задание. В разряде, о котором
говорится в задании и находится сохраняемая цифра. В
задании сказано: округлить число 675 до разряда десятков.
463

Видим, что в разряде десятков находится семёрка. Значит


сохраняемой цифрой является цифра 7

Теперь находим первую из отбрасываемых цифр. Первой из


отбрасываемых цифр является та цифра, которая следует
после сохраняемой цифрой. Видим, что первая цифра после
семёрки это цифра 5. Значит цифра 5 является первой
отбрасываемой цифрой.

Теперь применяем второе правило округления. Оно говорит,


что если при округлении чисел первая из отбрасываемых
464

цифр 5, 6, 7, 8 или 9, то сохраняемая цифра увеличивается


на единицу.

У нас первая из отбрасываемых цифр это 5. Значит мы


должны увеличить на единицу сохраняемую цифру 7, а всё
что следует после неё заменить нулём:

675 ≈ 680

Значит при округлении числа 675 до разряда десятков,


получаем приближённое ему число 680.

Теперь попробуем округлить то же самое число 675, но уже


до разряда сотен.

Нам требуется округлить число 675 до разряда сотен. Снова


ищем сохраняемую цифру. В этот раз сохраняемой цифрой
является 6, поскольку мы округляем число до разряда сотен:

Теперь находим первую из отбрасываемых цифр. Первой из


отбрасываемых цифр является та цифра, которая следует
после сохраняемой цифрой. Видим, что первая цифра после
шестёрки это цифра 7. Значит цифра 7 является первой
отбрасываемой цифрой:
465

Теперь применяем второе правило округления. Оно говорит,


что если при округлении чисел первая из отбрасываемых
цифр 5, 6, 7, 8 или 9, то сохраняемая цифра увеличивается
на единицу.

У нас первая из отбрасываемых цифр это 7. Значит мы


должны увеличить на единицу сохраняемую цифру 6, а всё
что следует после неё заменить нулями:

675 ≈ 700

Значит при округлении числа 675 до разряда сотен,


получаем приближённое ему число 700.

Пример 3. Округлить число 9876 до разряда десятков.

Здесь сохраняемая цифра это 7. А первая отбрасываемая


цифра это 6. Согласно правилу, если при округлении чисел
466

первая из отбрасываемых цифр 5, 6, 7, 8 или 9, то


сохраняемая цифра увеличивается на единицу.

Значит увеличиваем на единицу сохраняемую цифру 7, а всё


что располагается после неё заменяем нулём:

9876 ≈ 9880

Пример 4. Округлить число 9876 до разряда сотен.

Здесь сохраняемая цифра это 8. А первая отбрасываемая


цифра это 7. Согласно правилу, если при округлении чисел
первая из отбрасываемых цифр 5, 6, 7, 8 или 9, то
сохраняемая цифра увеличивается на единицу.

Значит увеличиваем на единицу сохраняемую цифру 8, а всё


что располагается после неё заменяем нулями:

9876 ≈ 9900

Пример 5. Округлить число 9876 до разряда тысяч.

Здесь сохраняемая цифра это 9. А первая отбрасываемая


цифра это 8. Согласно правилу, если при округлении чисел
первая из отбрасываемых цифр 5, 6, 7, 8 или 9, то
сохраняемая цифра увеличивается на единицу.

Значит увеличиваем на единицу сохраняемую цифру 9, а всё


что располагается после неё заменяем нулями:

9876 ≈ 10000
467

Пример 6. Округлить число 2971 до сотен.

При округлении этого числа до сотен следует быть


внимательным, поскольку сохраняемая цифра здесь 9, а
первая отбрасываемая цифра это 7. Значит цифра 9 должна
увеличиться на единицу. Но дело в том, что после
увеличения девятки на единицу получится 10, а это цифра не
вместится в разряд сотен нового числа.

В этом случае, в разряде сотен нового числа надо записать


0, а единицу перенести на следующий разряд и сложить с
цифрой, которая там находится. Далее заменить все цифры
после сохраняемой нулями:

2971 ≈ 3000

Округление десятичных дробей

При округлении десятичных дробей следует быть особенно


внимательным, поскольку десятичная дробь состоит из
целой и дробной части. И каждая из этих двух частей имеет
свои разряды:

Разряды целой части:

 разряд единиц;
 разряд десятков;
 разряд сотен;
 разряд тысяч.
468

Разряды дробной части:

 разряд десятых;
 разряд сотых;
 разряд тысячных

Рассмотрим десятичную дробь 123,456 — сто двадцать три


целых четыреста пятьдесят шесть тысячных. Здесь целая
часть это 123, а дробная часть 456. При этом у каждой из
этих частей есть свои разряды. Очень важно не путать их:

Для целой части применяются те же правила округления, что


и для обычных чисел. Отличие в том, что после округления
целой части и замены нулями всех цифр после сохраняемой
цифры, дробная часть полностью отбрасывается.

Например, округлим дробь 123,456 до разряда десятков.


Именно до разряда десятков, а не разряда десятых. Очень
важно не перепутать эти разряды. Разряд десятков
располагается в целой части, а разряд десятых в дробной.

Итак, мы должны округлить 123,456 до разряда десятков.


Сохраняемая цифра здесь это 2, а первая из отбрасываемых
цифр это 3
469

Согласно правилу, если при округлении чисел первая из


отбрасываемых цифр 0, 1, 2, 3 или 4, то сохраняемая цифра
остаётся без изменений.

Значит сохраняемая цифра останется без изменений, а всё


остальное заменится нулём. А что делать с дробной частью?
Её просто отбрасывают (убирают):

123,456 ≈ 120

Теперь попробуем округлить ту же самую дробь 123,456 до


разряда единиц. Сохраняемая цифра здесь будет 3, а
первая из отбрасываемых цифр это 4, которая находится в
дробной части:
470

Согласно правилу, если при округлении чисел первая из


отбрасываемых цифр 0, 1, 2, 3 или 4, то сохраняемая цифра
остаётся без изменений.

Значит сохраняемая цифра останется без изменений, а всё


остальное заменится нулём. Оставшаяся дробная часть
будет отброшена:

123,456 ≈ 123,0

Ноль, который остался после запятой тоже можно отбросить.


Значит окончательный ответ будет выглядеть следующим
образом:

123,456 ≈ 123,0 ≈ 123

Теперь займёмся округлением дробных частей. Для


округления дробных частей справедливы те же правила, что
и для округления целых частей. Попробуем округлить дробь
471

123,456 до разряда десятых. В разряде десятых


располагается цифра 4, значит она является сохраняемой
цифрой, а первая отбрасываемая цифра это 5, которая
находится в разряде сотых:

Согласно правилу, если при округлении чисел первая из


отбрасываемых цифр 5, 6, 7, 8 или 9, то сохраняемая цифра
увеличивается на единицу.

Значит сохраняемая цифра 4 увеличится на единицу, а


остальная часть заменится нулями

123,456 ≈ 123,500

Попробуем округлить ту же самую дробь 123,456 до разряда


сотых. Сохраняемая цифра здесь это 5, а первая из
отбрасываемых цифр это 6, которая находится в разряде
тысячных:
472

Согласно правилу, если при округлении чисел первая из


отбрасываемых цифр 5, 6, 7, 8 или 9, то сохраняемая цифра
увеличивается на единицу.

Значит сохраняемая цифра 5 увеличится на единицу, а


остальная часть заменится нулями

123,456 ≈ 123,460
473

Периодические дроби

Существуют дроби, у которых в дробной части некоторые


цифры бесконечно повторяются. Выглядят эти дроби
следующим образом:

0,66666666666666…

0,33333333333333…

0,68181818181818…

Дроби такого вида называют периодическими. В данном


уроке мы попробуем разобраться, что это за дроби и как с
ними работать.

Содержание урока

 Получаем периодическую дробь


 Виды периодических дробей
 Избавляемся от хвоста
 Перевод чистой периодической дроби в обыкновенную
дробь
 Перевод смешанной периодической дроби в
обыкновенную дробь

Получаем периодическую дробь

Попробуем разделить 1 на 3. Не будем подробно


останавливаться на том, как это сделать. Этот момент
подробно описан в уроке действия с десятичными дробями, в
теме деление меньшего числа на большее. Продвинутый
уровень.
474

Итак, делим 1 на 3

Видно, что мы постоянно получаем остаток 1, далее


приписываем к нему 0 и делим 10 на 3. И это повторяется
вновь и вновь. В результате в дробной части каждый раз
получается цифра 3. Деление 1 на 3 будет выполняться
бесконечно, поэтому разýмнее будет остановиться на
достигнутом.

Такие дроби называют периодическими, поскольку у них


присутствует период цифр, который бесконечно повторяется.
Период цифр может состоять из нескольких цифр, а может
состоять из одной как в нашем примере.

В примере, который мы рассмотрели выше, период в дроби


0,33333 это цифра 3. Обычно такие дроби записывают
сокращённо. Сначала записывают цéлую часть, затем ставят
запятую и в скобках указывают период (цифру, которая
повторяется).
475

В нашем примере повторяется цифра 3, она является


периодом в дроби 0,33333. Поэтому сокращённая запись
будет выглядеть так:

0, (3)

Читается как «ноль целых и три в периоде»

Пример 2. Разделить 5 на 11

Это тоже периодическая дробь. Период данной дроби это


цифры 4 и 5, эти цифры повторяются бесконечно.
Сокращённая запись будет выглядеть так:

0, (45)

Читается как «ноль целых и сорок пять в периоде»


476

Пример 3. Разделить 15 на 13

Здесь период состоит из нескольких цифр, а именно из цифр


153846. Для наглядности период отделён синей линией.
477

Сокращённая запись для данной периодической дроби будет


выглядеть так:

1, (153846)

Читается как: «одна целая сто пятьдесят три тысячи


восемьсот сорок шесть в периоде».

Пример 4. Разделить 471 на 900

В этом примере период начинается не сразу, а после цифр 5


и 2.  Сокращённая запись для данной периодической дроби
будет выглядеть так:

0, 52 (3)

Читается как: «ноль целых пятьдесят две сотых и три в


периоде».
478

Виды периодических дробей

Периодические дроби бывают двух видов: чистые и


смéшанные.

Если в периодической дроби период начинается сразу после


запятой, то такую периодическую дробь называют чистой.
Например, следующие периодические дроби являются
чистыми:

0, (3)

0, (6)

0, (5)

Видно, что в этих дробях период начинается сразу после


запятой.

Если же в периодической дроби период начинается не сразу,


а после некоторого количества не повторяющихся цифр, то
такую периодическую дробь называют смéшанной.
Например, следующие периодические дроби являются
смéшанными:

0,52 (3)

0,16 (5)

0,31 (6)

Видно, что в этих дробях период начинается не сразу, а


после некоторого количества не повторяющихся цифр.
479

Избавляемся от хвоста

Подобно тому, как ящерица избавляется от хвоста, мы


можем избавить периодическую дробь от повторяющегося
периода. Для этого достаточно округлить эту периодическую
дробь до нýжного разряда.

Например, округлим периодическую дробь 0, (3) до разряда


сотых. Чтобы увидеть сохраняемую и отбрасываемую цифру,
временно запишем дробь 0, (3) не в сокращённом виде, а в
полном:

Вспоминаем правило округления. Если при округлении чисел


первая из отбрасываемых цифр 0, 1, 2, 3 или 4, то
сохраняемая цифра остаётся без изменений.

Значит периодическая дробь 0, (3) при округлении до сотых


обращается в дробь 0,33

0, (3) ≈ 0,33
480

Округлим периодическую дробь 6,31 (6) до разряда


тысячных.

Запишем эту дробь в полном виде, чтобы увидеть


сохраняемую и отбрасываемую цифру:

Вспоминаем правило округления. Если при округлении чисел


первая из отбрасываемых цифр 5, 6, 7, 8 или 9, то
сохраняемая цифра увеличивается на единицу.

Значит периодическая дробь 6,31 (6) при округлении до


тысячных обращается в дробь 6,317

6,31 (6) ≈ 6,317

Перевод чистой периодической дроби в обыкновенную дробь

Перевод периодической дроби в обыкновенную это


операция, которую мы будем применять довольно редко. Тем
481

не менее, для общего развития желательно изучить и этот


момент. А начнём мы с перевода чистой периодической
дроби в обыкновенную дробь.

Мы уже говорили, что если период в периодической дроби


начинается сразу после запятой, то такую дробь называют
чистой.

Чтобы перевести чистую периодическую дробь в


обыкновенную дробь, нужно в числитель обыкновенной
дроби записать период периодической дроби, а в
знаменатель обыкновенной дроби записать некоторое
количество девяток. При этом, количество девяток должно
быть равно количеству цифр в периоде периодической
дроби.

В качестве примера, рассмотрим чистую периодическую


дробь 0, (3) — ноль целых и три в периоде. Попробуем
перевести её в обыкновенную дробь.

Правило гласит, что в первую очередь в числитель


обыкновенной дроби нужно записать период периодической
дроби.

Итак, записываем в числителе период дроби 0, (3) то есть


тройку:

А в знаменатель нужно записать некоторое количество


девяток. При этом,  количество девяток должно быть равно
количеству цифр в периоде периодической дроби 0, (3).
482

В периодической дроби 0, (3) период состоит из одной


цифры 3. Значит в знаменателе обыкновенной дроби
записываем одну девятку:

Полученную дробь можно сократить на 3, тогда получим


следующее:

Получили обыкновенную дробь  .

Таким образом, при переводе периодической дроби 0, (3) в

обыкновенную дробь получается

 Пример 2. Перевести периодическую дробь 0, (45) в


обыкновенную дробь.

Здесь период составляет две цифры 4 и 5. Записываем эти


две цифры в числитель обыкновенной дроби:

А в знаменатель записываем некоторое количество девяток.


Количество девяток должно быть равно количеству цифр в
периоде периодической дроби 0, (45).
483

В периодической дроби 0, (45) период состоит из двух цифр 4


и 5. Значит в знаменателе обыкновенной дроби записываем
две девятки:

Полученную дробь    можно сократить эту дробь на 9, тогда


получим следующее:

Таким образом, при переводе периодической дроби 0, (45) в

обыкновенную дробь получается 

Перевод смешанной периодической дроби в обыкновенную


дробь

Чтобы перевести смешанную периодическую дробь в


обыкновенную дробь, нужно в числителе записать разность в
которой уменьшаемое это цифры, стоящие после запятой в
периодической дроби, а вычитаемое — цифры, стоящие
между запятой и первым периодом периодической дроби.

В знаменателе же нужно записать некоторое количество


девяток и нулей. При этом, количество девяток должно быть
равно количеству цифр в периоде периодической дроби, а
количество нулей должно быть равно количеству цифр
между запятой и периодом периодической дроби.
484

Например, переведём смешанную периодическую дробь 0,31


(6) в обыкновенную дробь.

Сначала запишем в числителе разность. Уменьшаемым


будут все цифры, стоящие после запятой (включая и
период), а вычитаемым будут цифры, стоящие между
запятой и периодом:

Итак, записываем в числителе разность:

А в знаменателе запишем некоторое количество девяток и


нулей. Количество девяток должно быть равно количеству
цифр в периоде периодической дроби 0,31 (6)

В дроби 0,31 (6) период состоит из одной цифры. Значит в


знаменатель дроби записываем одну девятку:
485

Теперь дописываем количество нулей. Количество нулей


должно быть равно количеству цифр между запятой и
периодом периодической дроби.

В дроби 0,31 (6) между запятой и периодом располагается


две цифры. Значит в знаменателе дроби должно быть два
нуля. Дописываем их:

Получили выражение, которое вычисляется легко:

Получили ответ 

Таким образом, при переводе периодической дроби 0,31 (6) в

обыкновенную дробь, получается

Пример 2. Перевести смешанную периодическую дробь 0,72


(62) в обыкновенную дробь

Сначала запишем в числителе разность. Уменьшаемым


будут все цифры, стоящие после запятой (включая и
период), а вычитаемым будут цифры, стоящие между
запятой и периодом:
486

Итак, записываем в числителе разность:

А в знаменателе запишем некоторое количество девяток и


нулей. Количество девяток должно быть равно количеству
цифр в периоде периодической дроби 0,72 (62)

В дроби 0,72 (62) период состоит из двух цифр. Значит в


знаменатель дроби записываем две девятки:

Теперь дописываем количество нулей. Количество нулей


должно быть равно количеству цифр между запятой и
периодом периодической дроби.
487

В дроби 0,72 (62) между запятой и периодом располагаются


две цифры. Значит в знаменателе дроби должно быть два
нуля. Дописываем их:

  

Получили выражение, которое вычисляется легко:

Получили ответ  

Значит при переводе периодической дроби 0,72 (62) в

обыкновенную дробь, получается 


488

Перевод единиц

В этом уроке мы научимся переводить физические величины


из одной единицы измерения в другую.

Содержание урока

 Перевод единиц измерения длины


 Перевод единиц измерения массы
 Перевод единиц измерения времени

Перевод единиц измерения длины

Из прошлых уроков мы знаем, что основные единицы


измерения длины это:

 миллиметры;
 сантиметры;
 дециметры;
 метры;
 километры.

Любая величина, которая характеризует длину, может быть


переведена из одной единицы измерения в другую.

Кроме того, при решении задач по физике, обязательно


нужно соблюдать требования международной системы СИ.
То есть если длина дана не в метрах, а в другой единице
измерения, то её обязательно нужно перевести в метры,
поскольку метр является единицей измерения длины в
системе СИ.
489

Чтобы переводить длину из одной единицы измерения в


другую, нужно знать из чего состоит та или иная единица
измерения. То есть нужно знать, что к примеру один
сантиметр состоит из десяти миллиметров или один
километр состоит из тысячи метров.

Покажем на простом примере, как можно рассуждать при


переводе длины из одной единицы измерения в другую.
Предположим, что имеется 2 метра и нужно перевести их в
сантиметры.

Сначала нужно узнать сколько сантиметров содержится в


одном метре. В одном метре содержится сто сантиметров:

1 м = 100 см

Если в 1 метре содержится 100 сантиметров, то сколько


сантиметров будет содержаться в двух метрах? Ответ
напрашивается сам — 200 см. А эти 200 см получаются, если
2 умножить на 100.

Значит, чтобы перевести 2 метра в сантиметры, нужно 2


умножить на 100

2 × 100 = 200 см

Теперь попробуем перевести те же 2 метра в километры.


Сначала надо узнать сколько метров содержится в одном
километре. В одном километре содержится тысяча метров:

1 км = 1000 м

Если один километр содержит 1000 метров, то километр


который содержит только 2 метра будет намного меньше.
Чтобы его получить нужно 2  разделить на 1000
490

2 : 1000 = 0,002 км

Поначалу бывает трудно запомнить, какое действие


применять для перевода единиц — умножение или деление.
Поэтому на первых порах удобно пользоваться следующей
схемой:

Суть данной схемы заключается в том, что при переходе из


старшей единицы измерения в младшую применяется
умножение. И наоборот, при переходе из младшей единицы
измерения в более старшую применяется деление.

Стрелки, которые направлены вниз и вверх указывают на то,


что осуществляется переход из старшей единицы измерения
в младшую и переход из младшей единицы измерения в
более старшую соответственно. В конце стрелки указывается
какую операцию применить: умножение или деление.

Например, переведём 3000 метров в километры, пользуясь


данной схемой.

Итак, мы должны перейти из метров в километры. Другими


словами, перейти из младшей единицы измерения в более
старшую (километр старше метра). Смотрим на схему и
видим, что стрелка указывающая переход из младших
491

единиц в более старшие, направлена вверх и в конце


стрелки указано, что мы должны применить деление:

Теперь нужно узнать, сколько метров содержится в одном


километре. В одном километре содержится 1000 метров. А
чтобы узнать, сколько километров составляют 3000 таких
метров, нужно 3000 разделить на 1000

3000 : 1000 = 3 км

Значит, при переводе 3000 метров в километры, получим 3


километра.

Попробуем перевести те же 3000 метров в дециметры. Здесь


мы должны перейти из старших единиц в младшие
(дециметр младше метра). Смотрим на схему и видим, что
стрелка указывающая переход из старших единиц в
младшие, направлена вниз и в конце стрелки указано, что мы
должны применить умножение:
492

Теперь нужно узнать, сколько дециметров в одном метре. В


одном метре 10 дециметров.

1 м = 10 дм

А чтобы узнать сколько таких дециметров в трёх тысячах


метрах, нужно 3000 умножить на 10

3000 × 10 = 30 000 дм

Значит при переводе 3000 метров в дециметры, получим


30000 дециметров.

Перевод единиц измерения массы

Из прошлых уроков мы знаем, что основные единицы


измерения массы это:

 миллиграммы;
 граммы;
 килограммы;
 центнеры;
 тонны.
493

Любая величина, которая характеризует массу, может быть


переведена из одной единицы измерения в другую.

Кроме того, при решении задач по физике, обязательно


нужно соблюдать требования международной системы СИ.
То есть если масса дана не в килограммах, а в другой
единице измерения, то её обязательно нужно перевести в
килограммы, поскольку килограмм является единицей
измерения массы в системе СИ.

Чтобы переводить массу из одной единицы измерения в


другую, нужно знать из чего состоит та или иная единица
измерения. То есть нужно знать, что к примеру один
килограмм состоит из тысячи граммов или один центнер
состоит из ста килограммов.

Покажем на простом примере, как можно рассуждать при


переводе массы из одной единицы измерения в другую.
Предположим, что имеется 3 килограмма и нужно перевести
их в граммы.

Сначала нужно узнать сколько граммов содержится в одном


килограмме. В одном килограмме содержится тысяча
граммов:

1 кг = 1000 г

Если в 1 килограмме 1000 граммов, то сколько граммов будут


содержаться в трёх таких килограммах? Ответ
напрашивается сам — 3000 граммов. А эти 3000 граммов
получаются путем умножения 3 на 1000. Значит, чтобы
перевести 3 килограмма в граммы, нужно 3 умножить на 1000

3 × 1000 = 3000 г
494

Теперь попробуем перевести те же 3 килограмма в тонны.


Сначала нужно узнать сколько килограммов содержатся в
одной тонне. В одной тонне содержится тысяча килограмм:

1 т = 1000 кг

Если одна тонна содержит 1000 килограмм, то тонна которая


содержит только 3 килограмма будет намного меньше. Чтобы
её получить нужно 3 разделить на 1000

3 : 1000 = 0,003 т

Как и в случае с переводом единиц измерения длины, на


первых порах удобно пользоваться следующей схемой:

Данная схема позволит быстро сориентироваться какое


действие выполнить для перевода единиц — умножение или
деление.

Например, переведём 5000 килограмм в тонны, пользуясь


данной схемой.

Итак, мы должны перейти из килограммов в тонны. Другими


словами, перейти из младшей единицы измерения в более
старшую (тонна старше килограмма). Смотрим на схему и
видим, что стрелка указывающая переход из младших
495

единиц в более старшие, направлена вверх и в конце


стрелки указано, что мы должны применить деление:

Теперь нужно узнать сколько килограмм содержатся в одной


тонне. В одной тонне содержится 1000 килограмм. А чтобы
узнать, сколько тонн составляет 5000 килограмм, нужно 5000
разделить на 1000

5000 : 1000 = 5 т

Значит, при переводе 5000 килограмм в тонны, получается 5


тонн.

Попробуем перевести 6 килограммов в граммы. В данном


случае мы переходим из старшей единицы измерения в
младшую. Поэтому будем применять умножение.

Сначала надо узнать сколько граммов содержится в одном


килограмме. В одном килограмме содержится тысяча
граммов:

1 кг = 1000 г

Если в 1 килограмме 1000 граммов, то в шести таких


килограммах будет в шесть раз больше граммов. Значит
6 нужно умножить на 1000
496

6 × 1000 = 6000 г

Значит, при переводе 6 килограммов в граммы, получим 6000


грамм.

Перевод единиц измерения времени

Из прошлых уроков мы знаем, что основные единицы


измерения времени это:

 секунды;
 минуты;
 часы;
 сутки.

Любая величина, которая характеризует время, может быть


переведена из одной единицы измерения в другую.

Кроме того, при решении задач по физике, обязательно


нужно соблюдать требования международной системы СИ.
То есть если время дано не в секундах, а в другой единице
измерения, то его обязательно нужно перевести в секунды,
поскольку секунда является единицей измерения времени в
системе СИ.

Чтобы переводить время из одной единицы измерения в


другую, нужно знать из чего состоит та или иная единица
измерения времени. То есть нужно знать, что к примеру один
час состоит из шестидесяти минут или одна минута состоит
из шестидесяти секунд и т.д.
497

Покажем на простом примере, как можно рассуждать при


переводе времени из одной единицы измерения в другую.
Предположим, что требуется перевести 2 минуты в секунды.

Сначала надо узнать сколько секунд содержится в одной


минуте. В одной минуте содержатся шестьдесят секунд:

1 мин = 60 с

Если в 1 минуте 60 секунд, то сколько секунд будет в двух


таких минутах? Ответ напрашивается сам — 120 секунд. А
эти 120 секунд получаются путём умножения 2 на 60. Значит,
чтобы перевести 2 минуты в секунды, нужно 2 умножить на
60

2 × 60 = 120 с

Теперь попробуем перевести те же 2 минуты в часы.


Поскольку мы переводим минуты в часы, то сначала надо
узнать сколько минут содержится в одном часе. В одном часе
содержится шестьдесят минут:

1 ч = 60 м

Если один час содержит 60 минут, то час который содержит


только 2 минуты будет намного меньше. Чтобы его получить
нужно 2 минуты разделить на 60
498

При делении 2 на 60 получается периодическая дробь 0,0


(3). Эту дробь можно округлить до разряда сотых. Тогда
получим ответ 0,03

2 : 60= 0,03 ч

При переводе единиц измерения времени также применима


схема, подсказывающая что применять — умножение или
деление:

Например, переведём 25 минут в часы, пользуясь данной


схемой.
499

Итак, мы должны перейти из минут в часы. Другими словами,


перейти из младшей единицы измерения в более старшую
(часы старше минут). Смотрим на схему и видим, что стрелка
указывающая переход из младших единиц в более старшие,
направлена вверх и в конце стрелки указано, что мы должны
применить деление:

Теперь нужно узнать, сколько минут содержится в одном


часе. В одном часе содержится 60 минут. А час, который
содержит только  25 минут будет намного меньше. Чтобы его
найти, нужно 25 разделить на 60
500

При делении 25 на 60 получается периодическая дробь 0,41


(6). Эту дробь можно округлить до разряда сотых. Тогда
получим ответ 0,42

25 : 60 = 0,42 ч
501

Соотношения

Соотношением называют некоторую взаимосвязь между


сущностями нашего мира. Это могут быть числа, физические
величины, предметы, продукты, явления, действия и даже
люди.

В повседневной жизни, когда речь заходит о соотношениях,


мы говорим «соотношения того-то и того-то». Например,
если в вазе лежит 4 яблока и 2 груши, то мы говорим
«соотношения яблок и груш» или если поменять местами
яблоки и груши, то «соотношения груш и яблок».

В математике соотношение чаще употребляется как


«отношение того-то к тому-то».  Например, соотношение
четырёх яблок и двух груш, которые мы рассматривали
выше, в математике будет читаться как «отношение
четырех яблок к двум грушам» или если поменять местами
яблоки и груши, то «отношение двух груш к четырем
яблокам».

Соотношение выражается, как a к b (где вместо a и b любые


числа), но чаще можно встретить запись, которая составлена
с помощью двоеточия как a : b. Прочитать эту запись можно
различными способами:

 aкb
 a относится к b
 отношение a к b

Запишем соотношение четырех яблок и двух груш с


помощью символа соотношения:
502

4:2

Это соотношение можно прочитать как «четыре к двум»


либо «соотношение четырех яблок и двух груш» либо
«четыре яблока относится к двум грушам»

Если же поменяем местами яблоки и груши, то будем иметь


соотношение 2 : 4. Это соотношение можно прочитать как
«два к четырем» либо «две груши к четырем яблокам»
либо «две груши относятся к четырем яблокам».

В дальнейшем соотношение мы будем называть


отношением.

Содержание урока


o Что такое отношение?
o Сравнение величин
o Свойство отношения
o Несколько членов отношения

Что такое отношение?

Отношение, как было сказано ранее, записывается в


виде a:b. Также его можно записать в виде дроби . А мы
знаем, что такая запись в математике означает деление.
Тогда результатом выполнения отношения будет частное
чисел a и b.

Отношением в математике называют частное двух чисел.

Отношение позволяет узнать сколько количества одной


сущности приходится на единицу другой.  Вернемся к
503

отношению четырех яблок к двум грушам (4 : 2). Это


отношение позволит нам узнать, сколько яблок приходится
на единицу груши. Под единицей подразумевается одна
груша. Сначала запишем отношение 4 : 2 в виде дроби:

Данное отношение представляет собой деление числа 4 на


число 2. Если выполнить это деление, мы получим ответ на
вопрос сколько яблок приходится на единицу груши

Получили 2. Значит четыре яблока и две груши (4 : 2)


соотносятся (взаимосвязаны друг с другом) так, что на одну
грушу приходится два яблока
504

На рисунке показано, как четыре яблока и две груши


соотносятся между собой. Видно, что на каждую грушу
приходятся два яблока.

Отношение  можно перевернуть, записав как . Тогда у нас


получится соотношение двух груш и четырех яблок или
«отношение двух груш к четырем яблокам». Это отношение
покажет, сколько груш приходится на единицу яблока. Под
единицей яблока подразумевается одно яблоко.

Чтобы найти значение дроби нужно вспомнить, как делить


меньшее число на большее
505

Получили 0,5.  Переведём эту десятичную дробь в


обыкновенную:

Сократим полученную обыкновенную дробь на 5

Получили ответ (половину груши). Значит две груши и


четыре яблока (2 : 4) соотносятся (взаимосвязаны друг с
другом) так, что на одно яблоко приходится половина груши
506

На рисунке показано, как две груши и четыре яблока


соотносятся между собой. Видно, что на каждое яблоко
приходится половинка груши.

Числа, из которых составлено отношение, называют


членами отношения. Например, в отношении 4 : 2 членами
являются числа 4 и 2.

Рассмотрим другие примеры соотношений. Для


приготовления чего-либо составляется рецепт. Рецепт
строят из соотношений между продуктами. Например, для
приготовления овсяной каши обычно требуется стакан
хлопьев на два стакана молока или воды. Получается
соотношение 1 : 2 («один к двум» или «один стакан хлопьев
на два стакана молока»).
507

Преобразуем соотношение 1 : 2 в дробь, получим .


Вычислив эту дробь, получим 0,5. Значит один стакан
хлопьев и два стакана молока соотносятся (взаимосвязаны
друг с другом) так, что на один стакан молока приходится
половина стакана хлопьев.

Если перевернуть соотношение 1 : 2 то получится


соотношение 2 : 1 («два к одному» или «два стакана молока
на один стакан хлопьев»). Преобразуем соотношение 2 : 1 в

дробь, получим . Вычислив эту дробь, получим 2. Значит


два стакана молока и один стакан хлопьев соотносятся
(взаимосвязаны друг с другом) так, что на один стакан
хлопьев приходятся два стакана молока.

Пример 2. В классе 15 школьников. Из них 5 – это мальчики,


10 – девочки. Можно записать соотношение девочек и
мальчиков 10 : 5 и преобразовать это соотношение в дробь

. Вычислив эту дробь получим 2. То есть девочки и


мальчики соотносятся между собой так, что на каждого
мальчика приходятся две девочки
508

На рисунке показано, как десять девочек и пять мальчиков


соотносятся между собой. Видно, что на каждого мальчика
приходятся две девочки.

Соотношение не всегда можно обращать в дробь и находить


частное. В некоторых случаях это будет нелогично.

Так, если перевернуть отношение  получится , а это уже


отношение мальчиков к девочкам. Если вычислить эту дробь
получается 0,5. Получается, что пять мальчиков относятся к
десяти девочкам так, что на каждую девочку приходится
половина мальчика. Математически это конечно верно, но с
точки зрения реальности не совсем разумно, ибо мальчик это
живой человек и его нельзя просто так взять и разделить, как
грушу или яблоко.
509

Умение построить правильное отношение — важный навык


при решении задач. Так в физике, отношение пройденного
расстояния ко времени есть скорость движения.

Расстояние обозначается через переменную S, время —


через переменную t, скорость — через переменную v. Тогда
фраза «отношение пройденного пути ко времени есть
скорость движения» будет описываться следующим
выражением:

Предположим, что автомобиль проехал 100 километров за 2


часа. Тогда отношение пройденных ста километров к двум
часам будет скоростью движения автомобиля:

Скоростью принято называть расстояние, пройденное телом


за единицу времени. Под единицей времени
подразумевается 1 час, 1 минута или 1 секунда. А
отношение, как было сказано ранее, позволяет узнать
сколько количества одной сущности приходится на единицу
другой. В нашем примере отношение ста километров к двум
часам показывает сколько километров приходится на один
час движения. Видим, что на каждый час движения
приходятся 50 километров
510

Поэтому скорость измеряется в км/ч, м/мин, м/с. Символ


дроби ( / ) указывает на отношение расстояния ко времени:
километров в час, метров в минуту и метров в секунду
соответственно.

Пример 2. Отношение стоимости товара к его количеству


есть цена одной единицы товара

Если мы взяли в магазине 5 шоколадных батончиков и их


общая стоимость составила 100 рублей, то мы можем
определить цену одного батончика. Для этого нужно найти
отношение ста рублей к количеству батончиков. Тогда
получим, что на один батончик приходятся 20 рублей

Сравнение величин

Ранее мы узнали, что отношение между величинами разной


природы образуют новую величину. Так, отношение
511

пройденного расстояния ко времени есть скорость движения.


Отношение стоимости товара к его количеству есть цена
одной единицы товара.

Но отношение можно использовать и для сравнения величин.


Результат выполнения такого отношения есть число,
показывающее во сколько раз первая величина больше
второй или какую часть первая величина составляет от
второй.

Чтобы узнать во сколько раз первая величина больше


второй, в числитель отношения нужно записать
большую величину, а в знаменатель меньшую величину.

Чтобы узнать какую часть первая величина составляет


от второй, в числитель отношения нужно записать
меньшую величину, а в знаменатель большую величину.

Рассмотрим числа 20 и 2. Давайте узнаем во сколько раз


число 20 больше числа 2. Для этого находим отношение
числа 20 к числу 2. В числителе отношения записываем
число 20, а в знаменателе — число 2

Значение данного отношения равно десяти

Отношение числа 20 к числу 2 есть число 10. Это число


показывает во сколько раз число 20 больше числа 2. Значит
число 20 больше числа 2 в десять раз.
512

Пример 2. В классе 15 школьников. 5 из них это мальчики, 10


– девочки. Определить во сколько раз девочек больше
мальчиков.

Записываем отношение девочек к мальчикам. В числителе


отношения записываем количество девочек, в знаменатель
отношения — количество мальчиков:

Значение данного отношения равно 2. Значит в классе из 15


человек девочек в два раза больше мальчиков.

Здесь уже не стоит вопрос о том, сколько девочек


приходятся на одного мальчика. В данном случае

отношение  используется для сравнения  количества


девочек с количеством мальчиков.

Пример 3. Какую часть число 2 составляет от числа 20.

Находим отношение числа 2 к числу 20. В числителе


отношения записываем число 2, а в знаменателе — число 20

Чтобы найти значение данного отношения, нужно вспомнить,


как делить меньшее число на большее
513

Значение отношения числа 2 к числу 20 есть число 0,1

В данном случае десятичную дробь 0,1 можно перевести в


обыкновенную. Такой ответ будет проще для восприятия:

Значит число 2 от числа 20 составляет одну десятую часть.

Можно сделать проверку. Для этого найдём от числа 20.


Если мы всё сделали правильно, то должны получить число
2

20 : 10 = 2

2×1=2

Получили число 2. Значит одна десятая часть от числа 20


есть число 2. Отсюда делаем вывод, что задача решена
верно.
514

Пример 4. В классе 15 человек. 5 из них это мальчики, 10 –


девочки. Определить какую часть от общего количества
школьников составляют мальчики.

Записываем отношение мальчиков к общему количеству


школьников. В числителе отношения записываем пять
мальчиков, в знаменателе — общее количество школьников.
Общее количество школьников это 5 мальчиков плюс 10
девочек, поэтому в знаменателе отношения записываем
число 15

Чтобы найти значение данного отношения, нужно вспомнить,


как делить меньшее число на большее. В данном случае
число 5 нужно разделить на число 15

При делении 5 на 15 получается периодическая дробь.


Переведём эту дробь в обыкновенную
515

Сократим полученную дробь на 3

Получили окончательный ответ . Значит мальчики


составляют одну треть от всего класса

На рисунке видно, что в классе из 15 школьников треть


класса составляют 5 мальчиков.

Если для проверки найти  от 15 школьников, то мы получим


5 мальчиков

15 : 3 = 5

5×1=5

Пример 5. Во сколько раз число 35 больше числа 5 ?


516

Записываем отношение числа 35 к числу 5. В числитель


отношения нужно записать число 35, в знаменатель — число
5, но не наоборот

Значение данного отношения равно 7. Значит число 35 в


семь раз больше числа 5.

Пример 6. В классе 15 человек. 5 из них это мальчики, 10 –


девочки. Определить какую часть от общего количества
составляют девочки.

Записываем отношение девочек к общему количеству


школьников. В числителе отношения записываем десять
девочек, в знаменателе — общее количество школьников.
Общее количество школьников это 5 мальчиков плюс 10
девочек, поэтому в знаменателе отношения записываем
число 15

Чтобы найти значение данного отношения, нужно вспомнить,


как делить меньшее число на большее. В данном случае,
число 10 нужно разделить на число 15
517

При делении 10 на 15 получается периодическая дробь.


Переведём эту дробь в обыкновенную

Сократим полученную дробь на 3

Получили окончательный ответ . Значит девочки


составляют две трети от всего класса
518

На рисунке видно, что в классе из 15 школьников две трети


класса составляют 10 девочек.

Если для проверки найти  от 15 школьников, то получим 10


девочек

15 : 3 = 5

5 × 2 = 10

Пример 7. Какую часть 10 см составляют от 25 см

Записываем отношение десяти сантиметров к двадцати пяти


сантиметрам. В числителе отношения записываем 10 см, в
знаменателе — 25 см

Чтобы найти значение данного отношения, нужно вспомнить,


как делить меньшее число на большее. В данном случае
число 10 нужно разделить на число 25

Переведём полученную десятичную дробь в обыкновенную


519

Сократим полученную дробь на 2

Получили окончательный ответ . Значит 10 см составляют   


от 25 см.

Пример 8. Во сколько раз 25 см больше 10 см

Записываем отношение двадцати пяти сантиметров к десяти


сантиметрам. В числителе отношения записываем 25 см, в
знаменателе — 10 см

Найдём значение данного отношения

Получили ответ 2,5. Значит 25 см больше 10 см в 2,5 раза (в


два с половиной раза)

Важное замечание. При нахождении отношения


одноименных физических величин эти величины
обязательно должны быть выражены в одной единице
измерения, в противном случае ответ будет неверным.
520

Например, если мы имеем дело с двумя длинами и хотим


узнать во сколько раз первая длина больше второй или
какую часть первая длина составляет от второй, то обе
длины сначала нужно выразить в одной единице измерения.

Пример 9. Во сколько раз 150 см больше 1 метра?

Сначала сделаем так, чтобы обе длины были выражены в


одной единице измерения. Для этого переведем 1 метр в
сантиметры. Один метр это сто сантиметров

1 м = 100 см

Теперь находим отношение ста пятидесяти сантиметров к


ста сантиметрам. В числителе отношения записываем 150
сантиметров, в знаменателе — 100 сантиметров

Найдём значение данного отношения

Получили ответ 1,5. Значит 150 см больше 100 см в 1,5 раза


(в полтора раза).

А если бы не стали переводить метры в сантиметры и сразу


попытались найти отношение 150 см к одному метру, то у нас
получилось бы следующее:
521

Получилось бы, что 150 см больше одного метра в сто


пятьдесят раз, а это неверно. Поэтому обязательно нужно
обращать внимание на единицы измерения физических
величин, которые участвуют в отношении. Если эти
величины выражены в разных единицах измерения, то для
нахождения отношения этих величин, нужно перейти к одной
единице измерения.

Пример 10. В прошлом месяце зарплата человека


составляла 25000 рублей, а в текущем месяце зарплата
выросла до 27000 рублей. Определить во сколько раз
выросла зарплата

Записываем отношение двадцати семи тысяч к двадцати


пяти тысячам. В числителе отношения записываем 27000, в
знаменателе — 25000

Найдём значение данного отношения


522

Получили ответ 1,08. Значит зарплата выросла в 1,08 раза. В


будущем, когда мы познакомимся с процентами, такие
показатели как зарплата будем выражать в процентах.

Пример 11. Ширина многоквартирного дома 80 метров, а


высота 16 метров. Во сколько раз ширина дома больше его
высоты?

Записываем отношение ширины дома к его высоте:

Значение данного отношения равно 5. Значит ширина дома в


пять раз больше его высоты.

Свойство отношения

Отношение не изменится если его члены умножить или


разделить на одно и тоже число.

Это одно из важнейших свойств отношения следует из


свойства частного. Мы знаем, что если делимое и делитель
умножить или разделить на одно и то же число, то частное не
изменится. А поскольку отношение является ничем иным как
делением, то свойство частного работает и для него.

Вернемся к отношению девочек к мальчикам (10 : 5). Данное


отношение показало, что на каждого мальчика приходится
две девочки. Проверим, как работает свойство отношения, а
именно попробуем умножить или разделить его члены на
одно и то же число.
523

В нашем примере удобнее разделить члены отношения  на


их наибольший общий делитель (НОД).

НОД членов 10 и 5 это число 5. Поэтому можно разделить

члены отношения на число 5

Получили новое отношение . Это есть отношение два к


одному (2:1). Данное отношение, как и прошлое отношение
10:5 показывает, что на одного мальчика приходятся две
девочки.

На рисунке показано отношение 2 : 1 (два к одному). Как и в


прошлом отношении 10 : 5 на одного мальчика приходятся
две девочки. Другими словами, отношение не изменилось.
524

Пример 2. В одном классе 10 девочек и 5 мальчиков. В


другом классе 20 девочек и 10 мальчиков. Во сколько раз в
первом классе девочек больше мальчиков? Во сколько раз
во втором классе девочек больше мальчиков?

В обоих классах девочек  в два раза больше мальчиков,

поскольку отношения     и     равны одному и тому же


числу.

Свойство отношения позволяет строить различные модели,


которые имеют схожие параметры с реальным объектом.
Предположим, что многоквартирный дом имеет ширину 30
метров и высоту 10 метров.

Чтобы нарисовать на бумаге похожий дом, нужно рисовать


его в таком же отношении 30 : 10.

Разделим оба члена этого отношения на число 10. Тогда


получим отношение 3 : 1. Это отношение равно 3, как и
предыдущее отношение равно 3
525

Переведем метры в сантиметры. 3 метра это 300


сантиметров, а 1 метр это 100 сантиметров

3 м = 300 см

1  м = 100 см

Имеем отношение 300 см : 100 см. Разделим члены этого


отношения на 100. Получим отношение 3 см : 1 см. Теперь
можно нарисовать дом с шириной 3 см и высотой 1 см

Конечно нарисованный дом намного меньше реального


дома, но неизменным осталось отношение ширины и высоты.
Это позволило нам нарисовать дом, максимально похожий
на реальный
526

Отношение можно понимать и другим образом. Изначально


было сказано, что у реального дома ширина составляет 30
метров, а высота 10 метров. Итого получается 30+10, то есть
40 метров.

Эти 40 метров можно понимать, как 40 частей. Отношение 30


: 10 говорит о том, что 30 частей приходится на ширину, а 10
частей на высоту.

Далее члены отношения 30 : 10 были разделены на 10. В


результате получилось отношение 3 : 1. Это отношение
можно понимать, как 4 части, три из которых приходится на
ширину, одна — на высоту. В этом случае обычно требуется
узнать сколько конкретно метров приходится на ширину и
высоту.

Другими словами, нужно узнать сколько метров приходится


на 3 части и сколько метров приходится на 1 часть. Сначала
надо узнать сколько метров приходится на одну часть. Для
этого общие 40 метров нужно разделить на 4, поскольку в
отношении 3 : 1 всего четыре части

40 м : 4 = 10 м

Далее с помощью умножения определяют сколько метров


приходятся на ширину и высоту. Члены, которые даны в
отношении используют в качестве сомножителя.

Определим сколько метров приходится на ширину:

10 м × 3 = 30 м

Определим сколько метров приходится на высоту:

10 м × 1 = 10 м
527

Несколько членов отношения

Если в отношении дано несколько членов, то их можно


понимать как части от чего-либо.

Пример 1. Куплено 18 яблок. Эти яблоки разделили между


мамой, папой и дочкой в отношении 2 : 1 : 3. Сколько яблок
получил каждый?

Отношение 2 : 1 : 3 говорит о том, что мама получила 2


части, папа — 1 часть, дочка — 3 части. Другими словами,
каждый член отношения 2 : 1 : 3 это определенная часть от
18 яблок:

Если сложить члены отношения 2 : 1 : 3, то можно узнать


сколько всего частей имеется:

2 + 1 + 3 = 6 (частей)

Узнаем сколько яблок приходится на одну часть. Для этого


18 яблок разделим на 6

18 : 6 = 3 (яблока на одну часть)

Теперь определим сколько яблок получил каждый. Умножая


три яблока на каждый член отношения 2 : 1 : 3, можно
определить сколько яблок получила мама, сколько получил
папа и сколько получила дочка.
528

Узнаем сколько яблок получила мама:

3 × 2 = 6 (яблок)

Узнаем сколько яблок получил папа:

3 × 1 = 3 (яблока)

Узнаем сколько яблок получила дочка:

3 × 3 = 9 (яблок)

Пример 2. Новое серебро (альпака) — это сплав никеля,


цинка и меди в отношении 3 : 4 : 13. Сколько килограммов
каждого металла нужно взять, чтобы получить 4 кг нового
серебра?

4 килограмма нового серебра будет содержать 3 части


никеля, 4 части цинка и 13 частей меди. Сначала узнаем
сколько всего частей будет в четырех килограммах серебра:

3 + 4 + 13 = 20 (частей)

Определим сколько килограммов будет приходиться на одну


часть:

4 кг : 20 = 0,2 кг

Определим сколько килограммов никеля будет содержáться


в 4 кг нового серебра. В отношении 3 : 4 : 13 указано, что три
части сплава содержат никель. Поэтому умножаем 0,2 на 3:

0,2 кг × 3 = 0,6 кг никеля


529

Теперь определим сколько килограммов цинка будет


содержáться в 4 кг нового серебра. В отношении 3 : 4 : 13
указано, что четыре части сплава содержат цинк. Поэтому
умножаем 0,2 на 4:

0,2 кг × 4 = 0,8 кг цинка

Теперь определим сколько килограммов меди будет


содержáться в 4 кг нового серебра. В отношении 3 : 4 : 13
указано, что тринадцать частей сплава содержат медь.
Поэтому умножаем 0,2 на 13:

0,2 кг × 13 = 2,6 кг меди

Значит, чтобы получить 4 кг нового серебра, нужно взять 0,6


кг никеля, 0,8 кг цинка и 2,6 кг меди.

Пример 3. Латунь — это сплав меди и цинка, массы которых


относятся как 3 : 2. Для изготовления куска латуни требуется
120 г меди. Сколько требуется цинка для изготовления этого
куска латуни?

Определим сколько граммов сплава приходится на одну


часть. В условии сказано, что для изготовления куска латуни
требуется 120 г меди. Также сказано, что три части сплава
содержат медь. Если разделить 120 на 3, мы узнаем сколько
граммов сплава приходится на одну часть:

120 : 3 = 40 граммов на одну часть

Теперь определим сколько требуется цинка для


изготовления куска латуни. Для этого 40 граммов умножим на
530

2, поскольку в отношении 3 : 2 указано, что две части


содержат цинк:

40 г × 2 = 80 граммов цинка

Пример 4. Взяли два сплава золота и серебра. В одном


количество этих металлов находится в отношении 1 : 9, а в
другом 2 : 3. Сколько нужно взять каждого сплава, чтобы
получить 15 кг нового сплава, в котором золото и серебро
относилось бы как 1 : 4?

Решение

15 кг нового сплава должны состоять в отношении 1 : 4. Это


отношение говорит о том, что на одну часть сплава будет
приходиться золото, а на четыре части будет приходиться
серебро. Всего же частей пять. Схематически это можно
представить следующим образом

Определим массу одной части. Для этого сначала сложим


все части (1 и 4), затем массу сплава разделим на
количество этих частей
531

1+4=5
15 кг : 5 = 3 кг

Одна часть сплава будет иметь массу 3 кг. Тогда в 15 кг


нового сплава будет содержáться 3 × 1 = 3 кг золота и
серебра 3 × 4 = 12 кг серебра.

Поэтому для получения сплава массой 15 кг нам нужно 3 кг


золота и 12 кг серебра.

Теперь ответим на вопрос задачи — «Сколько нужно взять


каждого сплава?»

Первого сплава мы возьмем 10 кг, поскольку золото и


серебро в нём находятся в отношении 1 : 9. То есть этот
первый сплав даст нам 1 кг золота и 9 кг серебра.

Второго сплава мы возьмем 5 кг, поскольку золото и серебро


находятся в нём в отношении 2 : 3. То есть этот второй сплав
даст нам 2 кг золота и 3 кг серебра.
532

Пропорция

Продолжаем изучать соотношения. В данном уроке мы


познакомимся с пропорцией.

Содержание урока

 Что такое пропорция?


 Основное свойство пропорции

Что такое пропорция?

Пропорцией называют равенство двух отношений.

Например, отношение   равно отношению 

Данная пропорция читается следующим образом:

Десять так относится к пяти, как два относится к


одному

Дроби, из которых составлена пропорция, всегда равны.

Например, если в пропорции выполнить деление в


обеих дробях, то получится число 2 в обеих частях:

Предположим, что в классе 10 девочек и 5 мальчиков


533

Запишем отношение десяти девочек к пяти мальчикам:

10 : 5

Преобразуем данное отношение в дробь

Выполнив деление в этой дроби, мы получим 2. То есть


десять девочек так будут относиться к пяти мальчикам, что
на одного мальчика будет приходиться две девочки
534

Теперь рассмотрим другой класс в котором две девочки и


один мальчик
535

Запишем отношение двух девочек к одному мальчику:

2:1

Преобразуем данное отношение в дробь:

Выполнив деление в этой дроби, мы снова получим 2. То


есть две девочки так будут относиться к одному мальчику,
что на этого одного мальчика будут приходиться две
девочки:
536

Можно сделать вывод, что отношение   пропорционально

отношению  . Поэтому оно и читалось как «десять так


относится к пяти, как два относится к одному».

В нашем примере десять девочек так относятся к пяти


мальчикам, как и две девочки относятся к одному мальчику.

Пример 2. Рассмотрим отношение 12 девочек к 3 мальчикам


537

а также отношение 12 девочек к 2 мальчикам


538

Данные отношения не являются пропорциональными.

Другими словами, мы не можем записать, что  ,


поскольку первое отношение, как видно на рисунке
показывает, что на одного мальчика приходятся четыре
девочки, а второе отношение показывает, что на одного
мальчика приходятся шесть девочек.

Поэтому отношение   не пропорционально отношению  .

Из рассмотренных примеров видно, что пропорция


составляется из дробей. Первая рассмотренная нами

пропорция   состоит из двух дробей. Если выполнить


539

деление в этих дробях, то получим, что 2=2. Понятно, что 2


равно 2.

Вторая рассмотренная нами пропорция была  . Мы


пришли к выводу, что она составлена неправильно, поэтому

поставили между дробями   и   знак не равно (≠). Если


выполнить деление в этих дробях, получим числа 4 и 6.
Понятно, что 4 не равно 6.

Рассмотрим пропорцию  . Данная пропорция составлена

правильно, поскольку отношения    и    равны между собой:

Можно проверить это, выполнив деление в этих дробях, то


есть разделить 4 на 2, а 8 на 4. В результате с двух сторон
получатся двойки. А 2 равно 2

2=2

Все числа, находящиеся в пропорции (числители и


знаменатели обеих дробей) называются членами
пропорции. Эти члены подразделяются на два вида:
крайние члены и средние члены.

В нашей пропорции    крайние члены это 4 и 4, а


средние члены это 2 и 8
540

Почему крайние члены называют крайними, а средние


средними? Если записать пропорцию не в дробном, а в
обычном виде, то сразу станет всё понятно:

4:2=8:4

Числа 4 и 4 располагаются с краю, поэтому их назвали


крайними, а числа 2 и 8 располагаются посередине, поэтому
их назвали средними:

С помощью переменных пропорцию можно записать так:

Данное выражение можно прочесть следующим образом:

a так относится к b, как c относится к d


541

Смысл данного предложения уже понятен. Речь идет о


членах, участвующих в соотношении. a и d — это крайние
члены пропорции, b и c — средние члены пропорции.

Основное свойство пропорции

Основное свойство пропорции выглядит следующим


образом:

Произведение крайних членов пропорции равно


произведению её средних членов.

Мы знаем, что произведение это ни что иное, как обычное


умножение. Чтобы проверить правильно ли составлена
пропорция, нужно перемножить её крайние и средние члены.
Если произведение крайних членов будет равно
произведению средних членов, то такая пропорция
составлена правильно.

Например, проверим правильно ли составлена пропорция

. Для этого перемножим её крайние и средние члены.


Легко заметить, что крайние и средние члены пропорции
располагаются «крест-накрест», поэтому в умножении нет
ничего сложного. Перемножаем члены пропорции «крест-
накрест»:
542

4 × 4 = 16 — произведение крайних членов пропорции равно


16.

2 × 8 = 16 — произведение средних членов пропорции так же


равно 16.

4×4=2×8

16 = 16

4 × 4 = 2 × 8 — произведение крайних членов равно

произведению средних членов. Значит пропорция 


составлена правильно.

Пример 2. Проверить правильно ли составлена пропорция

Проверим равно ли произведение крайних членов пропорции


произведению её средних членов. Перемножим члены
пропорции крест-накрест:

2 × 6 = 12 — произведение крайних членов пропорции равно


12

3 × 1 = 3 — произведение средних членов пропорции равно 3


543

2×6≠3×1

12 ≠ 3

2 × 6 ≠ 3 × 1 — произведение крайних членов пропорции НЕ


равно произведению её средних членов. Значит

пропорция  составлена неправильно.

Поэтому в пропорции   разумнее заменить знак


равенства (=) на знак не равно (≠)
544

Расстояние, скорость, время

В этом уроке мы рассмотрим три физические величины, а


именно расстояние, скорость и время.

Содержание урока

 Расстояние
 Скорость
 Время
 Взаимосвязь скорости, времени, расстояния

Расстояние

Расстояние мы уже изучали в уроке единицы измерения.


Говоря простым языком, расстояние это длина от одного
пункта до другого. (Пример: расстояние от дома до школы 2
километра).

Имея дело с большими расстояниями, в основном они будут


измеряться в метрах и километрах. Расстояние обозначается
латинской буквой S. Можно обозначить и другой буквой, но
буква S общепринята.

Скорость

Скоростью называют расстояние, пройденное телом за


единицу времени. Под единицей времени подразумевается 1
час, 1 минута или 1 секунда.

Предположим, что двое школьников решили проверить, кто


быстрее добежит от двора до спортплощадки. Расстояние от
двора до спортплощадки 100 метров. Первый школьник
545

добежал за 25 секунд. Второй за 50 секунд. Кто добежал


быстрее?

Быстрее добежал тот, кто за 1 секунду пробежал бóльшее


расстояние. Говорят, что у него скорость движения больше.
В данном случае скорость школьников это расстояние,
которое они пробегают за 1 секунду.

Чтобы найти скорость, нужно расстояние разделить на время


движения.  Давайте найдём скорость первого школьника. Для
этого разделим 100 метров на время движения первого
школьника, то есть на 25 секунд:

100 м : 25 с = 4

Если расстояние дано в метрах, а время движения в


секундах, то скорость измеряется в метрах в секунду (м/с).
Если расстояние дано в километрах, а время движения в
часах, скорость измеряется в километрах в час (км/ч). 

У нас расстояние дано в метрах, а время в секундах. Значит


скорость измеряется в метрах в секунду (м/с)

100м : 25с = 4 (м/с)

Итак, скорость движения первого школьника составляет 4


метра в секунду (м/с).

Теперь найдем скорость движения второго школьника. Для


этого разделим расстояние на время движения второго
школьника, то есть на 50 секунд:

100 м : 50 c = 2 (м/с)
546

Значит скорость движения второго школьника составляет 2


метра в секунду (м/с).

Скорость движения первого школьника — 4 (м/с)


Скорость движения второго школьника — 2 (м/с)

4 (м/с) > 2 (м/с)

Скорость первого школьника больше. Значит он добежал до


спортплощадки быстрее. Скорость обозначается латинской
буквой v.

Время

Иногда возникает ситуация, когда требуется узнать за какое


время тело преодолеет то или иное расстояние.

Например, от дома до спортивной секции 1000 метров. Мы


должны доехать туда на велосипеде. Наша скорость будет
500 метров в минуту (500м/мин). За какое время мы доедем
до спортивной секции?

Если за одну минуту мы будем проезжать 500 метров, то


сколько таких минут с пятью ста метрами будет в 1000
метрах?

Очевидно, что надо разделить 1000 метров на то расстояние,


которое мы будем проезжать за одну минуту, то есть на 500
метров. Тогда мы получим время, за которое доедем до
спортивной секции:

1000 : 500 = 2 (мин)


547

Время движения обозначается маленькой латинской буквой


t.

Взаимосвязь скорости, времени, расстояния

Скорость принято обозначать маленькой латинской буквой v,


время движения – маленькой буквой t, пройденное
расстояние – маленькой буквой s. Скорость, время и
расстояние связаны между собой.

Если известны скорость и время движения, то можно найти


расстояние. Оно равно скорости, умноженной на время:

s=v×t

Например, мы вышли из дома и направились в магазин. Мы


дошли до магазина за 10 минут. Наша скорость была 50
метров в минуту. Зная свою скорость и время, мы можем
найти расстояние.

Если за одну минуту мы прошли 50 метров, то сколько таких


пятьдесят метров мы пройдем за 10 минут? Очевидно, что
умножив 50 метров на 10, мы определим расстояние от дома
до магазина:
548

v = 50 (м/мин)

t = 10 минут

s = v × t = 50 × 10 = 500 (метров до магазина)

Если известно время и расстояние, то можно найти скорость:

v=s:t

Например, расстояние от дома до школы 900 метров.


Школьник дошел до этой школы за 10 минут. Какова была его
скорость?

Скорость движения школьника это расстояние, которое он


проходит за одну минуту. Если за 10 минут он преодолел 900
метров, то какое расстояние он преодолевал за одну минуту?

Чтобы ответить на этот, нужно разделить расстояние на


время движения школьника:

s = 900 метров

t = 10 минут

v = s : t = 900 : 10 = 90 (м/мин)

Если известна скорость и расстояние, то можно найти время:

t=s:v

Например, от дома до спортивной секции 500 метров. Мы


должны дойти до неё пешком. Наша скорость будет 100
549

метров в минуту (100 м/мин). За какое время мы дойдем до


спортивной секции?

Если за одну минуту мы будем проходить 100 метров, то


сколько таких минут со ста метрами будет в 500 метрах?

Чтобы ответить на этот вопрос нужно 500 метров разделить


на расстояние, которое мы будем проходить за одну минуту,
то есть на 100. Тогда мы получим время, за которое мы
дойдем до спортивной секции:

s = 500 метров

v = 100 (м/мин)

t = s : v = 500 : 100 = 5 (минут до спортивной секции)


550

Прямая и обратная пропорциональность

Пропорциональность — это взаимосвязь между двумя


величинами, при которой изменение одной из них влечет за
собой изменение другой во столько же раз.

Пропорциональность бывает прямой и обратной. В данном


уроке мы рассмотрим каждую из них.

Содержание урока

 Прямая пропорциональность
 Обратная пропорциональность

Прямая пропорциональность

Предположим, что автомобиль двигается со скоростью 50


км/ч. Мы помним, что скорость это расстояние, пройденное
за единицу времени (1 час, 1 минуту или 1 секунду). В нашем
примере автомобиль двигается со скоростью 50 км/ч, то есть
за один час он будет проезжать расстояние, равное
пятидесяти километрам.

Изобразим на рисунке расстояние, пройденное автомобилем


за 1 час
551

Пусть автомобиль проехал еще один час с той же скоростью,


равной пятидесяти километрам в час. Тогда получится,
что автомобиль проедет 100 км

Как видно из примера, увеличение времени в два раза


привело к увеличению пройденного расстояния во столько
же раз, то есть в два раза.

Такие величины, как время и расстояние называют прямо


пропорциональными. А взаимосвязь между такими
величинами называют прямой пропорциональностью.

Прямой пропорциональностью называют взаимосвязь


между двумя величинами, при которой увеличение
одной из них влечет за собой увеличение другой во
столько же раз.
552

и наоборот, если одна величина уменьшается в


определенное число раз, то другая уменьшается во столько
же раз.

Предположим, что изначально планировалось проехать на


автомобиле 100 км за 2 часа, но проехав 50 км, водитель
решил отдохнуть. Тогда получится, что уменьшив расстояние
в два раза, время уменьшится во столько же раз. Другими
словами, уменьшение пройденного расстояния приведет к
уменьшению времени во столько же раз.

Интересная особенность прямо пропорциональных величин


заключается в том, что их отношение всегда постоянно. То
есть при изменении значений прямо пропорциональных
величин, их отношение остается неизменным.

В рассмотренном примере расстояние сначала было равно


50 км, а время одному часу. Отношение расстояния ко
времени есть число 50.

Но мы увеличили время движения в 2 раза, сделав его


равным двум часам. В результате пройденное расстояние
увеличилось во столько же раза, то есть стало равно 100 км.
Отношение ста километров к двум часам опять же есть число
50

Число 50 называют коэффициентом прямой


пропорциональности. Он показывает сколько расстояния
553

приходится на час движения. В данном случае коэффициент


играет роль скорости движения, поскольку скорость это
отношение пройденного расстояния ко времени.

Из прямо пропорциональных величин можно составлять

пропорции. К примеру, отношения   и   составляют


пропорцию:

Это отношение можно прочитать следующим образом:

Пятьдесят километров так относятся к одному часу,


как сто километров относятся к двум часам.

Пример 2. Стоимость и количество купленного товара


являются прямо пропорциональными величинами. Если 1 кг
конфет стоит 30 рублей, то 2 кг этих же конфет обойдутся в
60 рублей, 3 кг в 90 рублей. С увеличением стоимости
купленного товара, его количество увеличивается во столько
же раз.

Поскольку стоимость товара и его количество являются


прямо пропорциональными величинами, то их отношение
всегда постоянно.

Запишем чему равно отношение тридцати рублей к одному


килограмму
554

Теперь запишем чему равно отношение шестидесяти рублей


к двум килограммам. Это отношение опять же будет равно
тридцати:

Здесь коэффициентом прямой пропорциональности является


число 30. Этот коэффициент показывает сколько рублей
приходится на килограмм конфет. В данном примере
коэффициент играет роль цены одного килограмма товара,
поскольку цена это отношение стоимости товара на его
количество.

Обратная пропорциональность

Рассмотрим следующий пример. Расстояние между двумя


городами 80 км. Мотоциклист выехал из первого города, и со
скоростью 20 км/ч доехал до второго города за 4 часа.

Если скорость мотоциклиста составила 20 км/ч это значит,


что каждый час он проезжал расстояние равное двадцати
километрам. Изобразим на рисунке расстояние, пройденное
мотоциклистом, и время его движения:
555

На обратном пути скорость мотоциклиста была 40 км/ч, и на


тот же путь он затратил 2 часа.

Легко заметить, что при изменении скорости, время


движения изменилось во столько же раз. Причем изменилось
в обратную сторону — то есть скорость увеличилась, а время
наоборот уменьшилось.
556

Такие величины, как скорость и время называют обратно


пропорциональными. А взаимосвязь между такими
величинами называют обратной пропорциональностью.

Обратной пропорциональностью называют взаимосвязь


между двумя величинами, при которой увеличение
одной из них влечет за собой уменьшение другой во
столько же раз.

и наоборот, если одна величина уменьшается в


определенное число раз, то другая увеличивается во столько
же раз.

К примеру, если на обратном пути скорость мотоциклиста


составила бы 10 км/ч, то те же 80 км он преодолел бы за 8
часов:

Как видно из примера, уменьшение скорости привело к


увеличению времени движения во столько же раз.

Особенность обратно пропорциональных величин


заключается в том, что их произведение всегда постоянно.
557

То есть при изменении значений обратно пропорциональных


величин, их произведение остается неизменным.

В рассмотренном примере расстояние между городами было


равно 80 км. При изменении скорости и времени движения
мотоциклиста, это расстояние всегда оставалось
неизменным

Мотоциклист мог проехать это расстояние со скоростью 20


км/ч за 4 часа, и со скоростью 40 км/ч за 2 часа, и со
скоростью 10 км/ч за 8 часов. Во всех случаях произведение
скорости и времени было равно 80 км
558
559

Проценты

Процент это один из интересных и часто применяемых на


практике инструментов. Проценты частично или полностью
применяются в любой науке, на любой работе и даже в
повседневном общении. Человек, хорошо разбирающийся в
процентах, создаёт впечатление умного и образованного. В
данном уроке мы узнаем, что такое процент и какие действия
можно с ним выполнять.

Содержание урока

 Что такое процент?


 Как найти процент?
 Второй способ нахождения процента
 Нахождения числа по его проценту
 Задания для самостоятельного решения

Что такое процент?

В повседневной жизни дроби   встречаются наиболее


часто. Они даже получили свои названия: половина, треть и
четверть соответственно.
560

Но есть ещё одна дробь, которая тоже встречается часто.

Это дробь (одна сотая). Данная дробь получила название

процент. А что означает дробь одна сотая ? Эта дробь


означает, что чего-либо разделено на сто частей и оттуда
взята одна часть. Значит процентом является одна сотая
часть чего-либо.

Процентом называется одна сотая часть чего-либо

Например,  от одного метра составляет 1 см. Один метр


разделили на сто частей, и взяли одну часть (вспоминаем,
что 1 метр это 100 см). А одна часть из этих ста частей
составляет 1 см. Значит один процент от одного метра
составляет 1 см.

от одного метра уже составляет 2 сантиметра. В этот раз


один метр разделили на сто частей и взяли оттуда не одну, а
две части. А две части из ста составляют два сантиметра.
561

Значит два процента от одного метра составляет 2


сантиметра.

Еще пример,    от одного рубля составляет одну копейку.


Рубль разделили на сто частей, и взяли оттуда одну часть. А
одна часть из этих ста частей составляет одну копейку.
Значит один процент от одного рубля составляет одну
копейку.

Проценты встречались настолько часто, что люди заменили

дробь  на специальный значок, который выглядит


следующим образом:

Эта запись читается как «один процент». Она заменяет

собой дробь  . Также она заменяет собой десятичную

дробь 0,01 потому что если перевести обычную дробь     в


десятичную дробь, то мы получим 0,01. Стало быть между
этими тремя выражениями можно поставить знак равенства:

1% =  = 0,01

Два процента в дробном виде будут записаны как  , в виде


десятичной дроби как 0,02 а с помощью специального значка
два процента записывается как 2%.

2% =  = 0,02
562

Как найти процент?

Принцип нахождения процента такой же, как и обычное


нахождение дроби от числа. Чтобы найти процент от чего-
либо, нужно это чего-либо разделить на 100 частей и
полученное число умножить на нужный процент.

Например, найти 2% от 10 см.

Что означает запись 2% ? Запись 2% заменяет собой запись

. Если перевести это задание на более понятый язык, то


оно будет выглядеть следующим образом:

Найти     от 10 см

А как решать подобные задания мы уже знаем. Это обычное


нахождение дроби от числа. Чтобы найти дробь от числа,
нужно это число разделить на знаменатель дроби, и
полученный результат умножить на числитель дроби.

Итак, делим число 10 на знаменатель дроби 

Получили 0,1. Теперь 0,1 умножаем на числитель дроби 

0,1 × 2 = 0,2
563

Получили ответ 0,2. Значит 2% от 10 см составляет 0,2 см. А


если перевести 0,2 сантиметра в миллиметры, то получим 2
миллиметра:

0,2 см = 2 мм

Значит 2% от 10 см составляют 2 мм.

Пример 2. Найти 50% от 300 рублей.

Чтобы найти 50% от 300 рублей, нужно эти 300 рублей


разделить на 100, и полученный результат умножить на 50.

Итак, делим 300 рублей на 100

300 : 100 = 3

Теперь полученный результат умножаем на 50

3 × 50 = 150 руб.

Значит 50% от 300 рублей составляет 150 рублей.

Если на первых порах сложно привыкнуть к записи со


значком %, можно заменять эту запись на обычную дробную
запись.

Например, те же 50% можно заменить на запись  . Тогда

задание будет выглядеть так: Найти  от 300 рублей, а


решать такие задачи для нас пока проще

300 : 100 = 3

3 × 50 = 150
564

В принципе, ничего сложного здесь нет. Если возникают


сложности, советуем остановиться и заново изучить дроби и
как их можно применять.

Пример 3. Швейная фабрика выпустила 1200 костюмов. Из


них 32% составляют костюмы нового фасона. Сколько
костюмов нового фасона выпустила фабрика?

Здесь нужно найти 32% от 1200. Найденное число будет


ответом к задаче. Воспользуемся правилом нахождения
процента. Разделим 1200 на 100 и полученный результат
умножим на искомый процент, т.е. на 32

1200 : 100 = 12

12 × 32 = 384

Ответ: 384 костюмов нового фасона выпустила фабрика.

Второй способ нахождения процента

Второй способ нахождения процента намного проще и


удобнее. Он заключается в том, что число от которого
ищется процент сразу умножит на нужный процент,
выраженный в виде десятичной дроби.

Например, решим предыдущую задачу этим способом. Найти


50% от 300 рублей.

Запись 50% заменяет собой запись  , а если перевести

эти  в десятичную дробь, то мы получим 0,5


565

Теперь для нахождения 50% от 300, достаточно будет


умножить число 300 на десятичную дробь 0,5

300 × 0,5 = 150

Кстати, по этому же принципу работает механизм


нахождения процента на калькуляторах. Чтобы найти
процент с помощью калькулятора, нужно ввести в
калькулятор число от которого ищется процент, затем нажать
клавишу умножения и ввести искомый процент. Затем нажать
клавишу процента %

Нахождения числа по его проценту

Зная процент от числа, можно узнать всё число. Например,


предприятие выплатило нам 60000 рублей за работу, и это
составляет 2% от общей прибыли, полученной
предприятием. Зная свою долю, и сколько процентов она
составляет, мы можем узнать общую прибыль.

Сначала нужно узнать сколько рублей составляет один


процент. Как это сделать? Попробуйте догадаться
внимательно изучив следующий рисунок:
566

Если два процента от общей прибыли составляют 60 тысяч


рублей, то нетрудно догадаться, что один процент
составляет 30 тысяч рублей. А чтобы получить эти 30 тысяч
рублей, нужно 60 тысяч разделить на 2

60 000 : 2 = 30 000

Мы нашли один процент от общей прибыли, т.е. . Если


одна часть это 30 тысяч, то для определения ста частей,
нужно 30 тысяч умножить на 100

30 000 × 100 = 3 000 000

Мы нашли общую прибыль. Она составляет три миллиона.

Попробуем сформировать правило нахождения числа по его


проценту.

Чтобы найти число по его проценту, нужно известное


число разделить на данный процент, и полученный
результат умножить на 100.
567

Пример 2. Число 35 это 7% от какого-то неизвестного числа.


Найти это неизвестное число.

Читаем первую часть правила:

Чтобы найти число по его проценту, нужно известное


число разделить на данный процент

У нас известное число это 35, а данный процент это 7.


Разделим 35 на 7

35 : 7 = 5

Читаем вторую часть правила:

и полученный результат умножить на 100

У нас полученный результат это число 5. Умножим 5 на 100

5 × 100 = 500

500 это неизвестное число, которое требовалось найти.


Можно сделать проверку. Для этого находим 7% от 500. Если
мы всё сделали правильно, то должны получить 35

500 : 100 = 5

5 × 7 = 35

Получили 35. Значит задача была решена правильно.

Принцип нахождения числа по его проценту такой же, как и


обычное нахождение целого числа по его дроби. Если
проценты на первых порах смущают и сбивают с толку, то
запись с процентом можно заменять на дробную запись.
568

Например, предыдущая задача может быть изложена так:

число 35 это от какого-то неизвестного числа. Найти это


неизвестное число. Как решать такие задачи мы уже знаем.
Это нахождение числа по дроби. Для нахождения числа по
дроби, мы это число делим на числитель дроби и
полученный результат умножаем на знаменатель дроби. В
нашем примере число 35 нужно разделить на 7 и полученный
результат умножить на 100

35 : 7 = 5

5 × 100 = 500

В будущем мы будем решать задачи на проценты, часть из


которых будут сложными. Чтобы на первых порах не
усложнять обучение, достаточно уметь находить процент от
числа, и число по проценту.

Задания для самостоятельного решения


Задание 1. Найдите 20% от числа 200
Показать решение
Задание 2. Найдите 34% от числа 1050
Показать решение
Задание 3. Найдите 25% от числа 80
Показать решение
Задание 4. Найдите 185% от числа 1,5
Показать решение
Задание 5. Найдите 150% от числа 1150
569

Показать решение
Задание 6. Представьте выражение 15% в виде
обыкновенной дроби
Показать решение
Задание 7. Представьте выражение 25% в виде
обыкновенной дроби
Показать решение
Задание 8. Представьте выражение 125% в виде
обыкновенной дроби
Показать решение
Задание 9. Число 12 это 60% от какого-то числа. Найдите это
число.
Показать решение
Задание 10. Число 40 это 20% от какого-то числа. Найдите
это число.
Показать решение
570

Отрицательные числа

Отрицательные числа — это числа со знаком минус (−),


например −1, −2, −3. Читается как: минус один, минус два,
минус три.

Примером применения отрицательных чисел является


термометр, показывающий температуру тела, воздуха, почвы
или воды. В зимнее время, когда на улице очень холодно,
температура бывает отрицательной (или как говорят в
народе «минусовой»).

Например, −10 градусов холода:


571

Обычные же числа, которые мы рассматривали ранее такие


как 1, 2, 3 называют положительными. Положительные числа
— это числа со знаком плюс (+).

При записи положительных чисел знак + не записывают,


поэтому мы и видим привычные для нас числа 1, 2, 3. Но
следует иметь ввиду, что эти положительные числа выглядят
так: +1, +2, +3.

Содержание урока

 Координатная прямая
572

 Сравнение отрицательных и положительных чисел


 Задания для самостоятельного решения

Координатная прямая

Координатная прямая это прямая линия, на которой


располагаются все числа: и отрицательные и
положительные. Выглядит следующим образом:

Здесь показаны только числа от −5 до 5. На самом деле


координатная прямая бесконечна. На рисунке представлен
лишь её небольшой фрагмент.

Числа на координатной прямой отмечают в виде точек. На


рисунке жирная чёрная точка является началом отсчёта.
Начало отсчёта начинается с нуля. Слева от начала отсчёта
отмечают отрицательные числа, а справа — положительные.

Координатная прямая продолжается бесконечно по обе


стороны. Бесконечность в математике обозначается
символом ∞. Отрицательное направление будет
обозначаться символом −∞, а положительное символом +∞.
Тогда можно сказать, что на координатной прямой
располагаются все числа от минус бесконечности до плюс
бесконечности:

(−∞; +∞)

Каждая точка на координатной прямой имеет своё имя и


координату. Имя — это любая латинская буква. Координата
— это число, которое показывает положение точки на этой
573

прямой. Проще говоря, координата это то самое число,


которое мы хотим отметить на координатной прямой.

Например, точка А(2) читается как «точка А с координатой


2« и будет обозначаться на координатной прямой
следующим образом:

Здесь A — это имя точки, 2 — координата точки A.

Пример 2. Точка B(4) читается как «точка B с координатой


4« и будет обозначаться на координатной прямой так:

Здесь B — это имя точки, 4 — координата точки B.

Пример 3. Точка M(−3) читается как «точка M с


координатой минус три» и будет обозначаться на
координатной прямой так:

Здесь M — это имя точки, −3 — координата точки M.

Точки можно обозначать любыми буквами. Но общепринято


обозначать их большими латинскими буквами. Более того,
574

начало отчёта, которое по другому называют началом


координат принято обозначать большой латинской буквой O

Легко заметить, что отрицательные числа лежат левее


относительно начала отсчёта, а положительные числа
правее.

Существуют такие словосочетания как «чем левее, тем


меньше» и «чем правее, тем больше». Наверное, вы уже
догадались о чём идёт речь. При каждом шаге влево число
будет уменьшаться в меньшую сторону. И при каждом шаге
вправо число будет увеличиваться. Стрелка, направленная
вправо, указывает на положительное направление отсчёта.

Сравнение отрицательных и положительных чисел

Правило 1. Любое отрицательное число меньше любого


положительного числа.

Например, сравним два числа: −5 и 3. Минус пять меньше,


чем три, несмотря на то, что пятёрка бросается в глаза в
первую очередь, как цифра большая, чем три.

Связано это с тем, что −5 является отрицательным числом, а


3 — положительным. На координатной прямой можно
увидеть, где располагаются числа −5 и 3
575

Видно, что −5 лежит левее, а 3 правее. А мы говорили, что


«чем левее, тем меньше». И правило говорит, что любое
отрицательное число меньше любого положительного числа.
Отсюда следует, что

−5 < 3

«Минус пять меньше, чем три»

Правило 2. Из двух отрицательных чисел меньше то,


которое располагается левее на координатной прямой.

Например, сравним числа −4 и −1. Минус четыре меньше,


чем минус единица.

Связано это опять же с тем, что на координатной прямой −4


располагается левее, чем −1

Видно, что −4 лежит левее, а −1 правее. А мы говорили, что


«чем левее, тем меньше». И правило говорит, что из двух
576

отрицательных чисел меньше то, которое располагается


левее на координатной прямой. Отсюда следует, что

−4 < −1

Минус четыре меньше, чем минус единица

Правило 3. Ноль больше любого отрицательного числа.

Например, сравним 0 и −3. Ноль больше, чем минус три.


Связано это с тем, что на координатной прямой 0
располагается правее, чем −3

Видно, что 0 лежит правее, а −3 левее. А мы говорили, что


«чем правее, тем больше». И правило говорит, что ноль
больше любого отрицательного числа. Отсюда следует, что

0 > −3

Ноль больше, чем минус три

Правило 4. Ноль меньше любого положительного числа.


577

Например, сравним 0 и 4. Ноль меньше, чем 4. Это в


принципе ясно и так. Но мы попробуем увидеть это воочию,
опять же на координатной прямой:

Видно, что на координатной прямой 0 располагается левее, а


4 правее. А мы говорили, что «чем левее, тем меньше». И
правило говорит, что ноль меньше любого положительного
числа. Отсюда следует, что

0<4

Ноль меньше, чем четыре

Задания для самостоятельного решения


Задание 1. Сравните числа −2 и 1
Показать решение
Задание 2. Сравните числа −5 и −2
Показать решение
Задание 3. Сравните числа −5 и −16
Показать решение
Задание 4. Сравните числа 15 и 20
Показать решение
578

Задание 5. Сравните числа −7 и 0


Показать решение
Задание 6. Сравните числа 5 и 0
Показать решение
Задание 7. Сравните числа 5 и 7
Показать решение
579

Модуль числа

Мóдуль числá a — это расстояние от начала координат до


точки А(a).

Чтобы понять это определение, подставим вместо


переменной a любое число, например 3, и снова прочитаем
его:

Мóдуль числá 3 — это расстояние от начала координат


до точки А(3).

То есть модуль это ни что иное как обычное расстояние.


Давайте попробуем увидеть расстояние от начала координат
до точки А(3)

Расстояние от начала координат до точки А(3) составляет 3


(три единицы или три шага).

Модуль числа обозначает двумя вертикальными линиями,


например:

Модуль числа 3 обозначается так: |3|

Модуль числа 4 обозначается так: |4|

Модуль числа 5 обозначается так: |5|

Мы искали модуль числа 3 и выяснили, что он равен 3. Так и


записываем:
580

|3| = 3

Читается как «Модуль числа три равен три»

Теперь попробуем найти модуль числа −3. Опять же


возвращаемся к определению и подставляем в него число
−3. Только вместо точки A используем новую точку B. Точку
A мы уже использовали в первом примере.

Модулем числа −3 называют расстояние от начала


координат до точки B(−3).

Расстояние от одного пункта до другого не может быть


отрицательным. Модуль это тоже расстояние, поэтому тоже
не может быть отрицательным.

Модуль числа −3 равен 3. Расстояние от начала координат


до точки B(−3) равно трём единицам:

|−3| = 3

Читается как «Модуль числа минус три равен три»

Модуль числа 0 равен 0, так как точка с координатой 0


совпадает  с началом координат. То есть расстояние от
начала координат до точки O(0) равно нулю:
581

|0| = 0

«Модуль нуля равен нулю»

Сделаем выводы:

 Модуль числа не может быть отрицательным;


 Для положительного числа и нуля модуль равен самомý
числу, а для отрицательного – противоположному числу;
 Противоположные числа имеют равные модули.

Противоположные числа

Числа, отличающиеся только знаками называют


противоположными.

Например, числа −2 и 2 являются противоположными. Они


отличаются только знаками. У числá −2 знак минуса, а у
числá 2 знак плюса, но мы его не видим, поскольку плюс как
говорилось ранее, не записывают.

Еще примеры противоположных чисел:

−1 и 1

−3 и 3

−5 и 5

−9 и 9
582

Противоположные числа имеют равные модули. Например,


найдём модули чисел −3 и 3

|−3| и |3|

3=3

На рисунке видно, что расстояние от начала координат до


точек A(−3) и B(3) одинаково равно трём шагам.
583

Что такое множество?

Множество — это набор каких-либо объектов. Объекты, из


которых состоит множество, называются элементами этого
множества.

Например: множество школьников, множество машин,


множество чисел.

В математике множество рассматривается намного шире.


Мы не будем сильно углубляться в эту тему, поскольку она
относится к высшей математике и на первых порах может
создавать трудности для обучения. Мы рассмотрим только ту
часть темы, с которой уже имели дело.

Содержание урока

 Обозначения
 Множество натуральных чисел
 Множество целых чисел
 Множество рациональных чисел

Обозначения

Множество чаще всего обозначают заглавными


буквами латинского алфавита, а его элементы — строчными.
При этом элементы заключаются в фигурные скобки.

Например, если наших друзей зовут Том, Джон и Лео, то мы


можем задать множество друзей, элементами которого будут
Том, Джон и Лео.
584

Обозначим множество наших друзей через заглавную


латинскую букву F (friends), затем поставим знак равенства и
в фигурных скобках перечислим наших друзей:

F = { Том, Джон, Лео }

Пример 2. Запишем множество делителей числа 6.

Обозначим через любую заглавную латинскую букву данное


множество, например, через букву D

затем поставим знак равенства и в фигурных скобках


перечислим элементы данного множества, то есть
перечислим делители числа 6

D = { 1, 2, 3, 6 }

Если какой-то элемент принадлежит заданному множеству,


то эта принадлежность указывается с помощью знака
принадлежности ∈. К примеру, делитель 2 принадлежит
множеству делителей числа 6 (множеству D). Записывается
это так:

2∈D

Читается как «2 принадлежит множеству делителей числа


Если какой-то элемент не принадлежит заданному


множеству, то эта не принадлежность указывается с
помощью зачёркнутого знака принадлежности ∉. К примеру,
585

делитель 5 не принадлежит множеству D. Записывается это


так:

5∉D

Читается как «5 не принадлежит множеству делителей


числа 6»

Кроме того, множество можно записывать прямым


перечислением элементов, без заглавных букв. Это может
быть удобным, если множество состоит из небольшого
количества элементов. Например, зададим множество из
одного элемента. Пусть этим элементом будет наш друг Том:

{ Том }

Зададим множество, которое состоит из одного числа 2

{2}

Зададим множество, которое состоит из двух чисел: 2 и 5

{ 2, 5 }

Множество натуральных чисел

Это первое множество с которым мы начали работать.


Натуральными числами называют числа 1, 2, 3 и т.д.

Натуральные числа появились из-за потребности людей


сосчитать те иные объекты. Например, посчитать количество
кур, коров, лошадей. Натуральные числа возникают
естественным образом при счёте.
586

В прошлых уроках, когда мы употребляли слово «число»,


чаще всего подразумевалось именно натуральное число.

В математике множество натуральных чисел обозначается


заглавной латинской буквой N.

Например, укажем, что число 1 принадлежит множеству


натуральных чисел. Для этого записываем  число 1, затем с
помощью знака принадлежности ∈ указываем, что единица
принадлежит множеству N

1∈N

Читается как: «единица принадлежит множеству


натуральных чисел»

Множество целых чисел

Множество целых чисел включает в себя все положительные


и отрицательные числа, а также число 0.

Множество целых чисел обозначается заглавной латинской


буквой Z.

Укажем, к примеру, что число −5 принадлежит множеству


целых чисел:

−5 ∈ Z

Укажем, что 10 принадлежит множеству целых чисел:

10 ∈ Z

Укажем, что 0 принадлежит множеству целых чисел:


587

0∈Z

В будущем все положительные и отрицательные числа мы


будем называть одним словосочетанием — целые числа.

Множество рациональных чисел

Рациональные числа, это те самые обыкновенные дроби,


которые мы изучаем по сей день.

Рациональное число — это число, которое может быть


представлено в виде дроби , где a — числитель дроби, b —
знаменатель.

В роли числителя и знаменателя могут быть любые числа, в


том числе и целые (за исключением нуля, поскольку на нуль
делить нельзя).

Например, представим, что вместо a стоит число 10, а


вместо b — число 2

10 разделить на 2 равно 5. Видим, что число 5 может быть

представлено в виде дроби  , а значит число 5 входит во


множество рациональных чисел.
588

Легко заметить, что число 5 также относится и ко множеству


целых чисел. Стало быть множество целых чисел входит во
множество рациональных чисел. А значит, во множество
рациональных чисел входят не только обыкновенные дроби,
но и целые числа вида −2, −1, 0, 1, 2.

Теперь представим, что вместо a стоит число 12, а вместо b


— число 5.

12 разделить на 5 равно 2,4. Видим, что десятичная дробь

2,4 может быть представлена в виде дроби , а значит она


входит во множество рациональных чисел. Отсюда делаем
вывод, что во множество рациональных чисел входят не
только обыкновенные дроби и целые числа, но и десятичные
дроби.

Мы вычислили дробь    и получили ответ 2,4. Но мы могли


бы выделить в этой дроби целую часть:

При выделении целой части в дроби , получается

смешанное число . Видим, что смешанное число   тоже


589

может быть представлено в виде дроби .  Значит во


множество рациональных чисел входят и смешанные числа.

В итоге мы приходим к выводу, что множество рациональных


чисел содержат в себе:

 целые числа
 обыкновенные дроби
 десятичные дроби
 смешанные числа

Множество рациональных чисел обозначается заглавной


латинской буквой Q.

Например укажем, что дробь принадлежит множеству

рациональных чисел. Для этого записываем саму дробь ,


затем с помощью знака принадлежности ∈ указываем, что

дробь принадлежит множеству рациональных чисел:

∈Q

Укажем, что десятичная дробь 4,5 принадлежит множеству


рациональных чисел:

4,5 ∈ Q

Укажем, что смешанное число    принадлежит множеству


рациональных чисел:
590

 ∈ Q

Вводный урок по множествам завершён. В будущем мы


рассмотрим множества намного лучше, а пока
рассмотренного в данном уроке будет достаточно.
591

Сложение и вычитание целых чисел

В данном уроке мы изýчим сложение и вычитание целых


чисел.

Напомним, что целые числа — это все положительные и


отрицательные числа, а также число 0. Например,
следующие числа являются целыми:

−3, −2, −1, 0, 1, 2, 3

Положительные числа легко складываются и вычитаются,


умножаются и делятся. К сожалению, этого нельзя сказать об
отрицательных числах, которые смущают многих новичков
своими минусами перед каждой цифрой.

Содержание урока

 Примеры сложения и вычитания целых чисел


 Правила сложения и вычитания целых чисел
 Задания для самостоятельного решения

Примеры сложения и вычитания целых чисел

Первое чему следует научиться это складывать и вычитать


целые числа с помощью координатной прямой. Совсем
необязательно рисовать координатную прямую. Достаточно
воображать её в своих мыслях и видеть, где располагаются
отрицательные числа и где положительные.

Рассмотрим следующее простейшее выражение

1+3

Значение данного выражения равно 4


592

1+3=4

Этот пример можно понять с помощью координатной прямой.


Для этого из точки, где располагается число 1, нужно
сдвинуться вправо на три шага. В результате мы окажемся в
точке, где располагается число 4. На рисунке можно увидеть,
как это происходит:

Знак плюса в выражении 1 + 3 указывает нам, что нужно


двигаться вправо в сторону увеличения чисел.

Пример 2. Найдём значение выражения 1 − 3

Значение данного выражения равно −2

1 − 3 = −2

Этот пример опять же можно понять с помощью


координатной прямой. Для этого из точки, где располагается
число 1 нужно сдвинуться влево на три шага. В результате
мы окажемся в точке, где располагается отрицательное
число −2. На рисунке можно увидеть, как это происходит:
593

Знак минуса в выражении 1 − 3 указывает нам, что нужно


двигаться влево в сторону уменьшения чисел.

Вообще, если осуществляется сложение, то нужно двигаться


вправо в сторону увеличения. Если же осуществляется
вычитание, то нужно двигаться влево в сторону уменьшения.

Пример 3. Найти значение выражения −2 + 4

Значение данного выражения равно 2

−2 + 4 = 2

Этот пример опять же можно понять с помощью


координатной прямой. Для этого из точки, где располагается
отрицательное число −2 нужно сдвинуться вправо на четыре
шага. В результате мы окажемся в точке, где располагается
положительное число 2

Видно, что мы сдвинулись из точки где располагается


отрицательное число −2 в правую сторону на четыре шага, и
оказались в точке, где располагается положительное число
2.

Пример 4. Найти значение выражения −1 − 3

Значение данного выражения равно −4


594

−1 − 3 = −4

Этот пример опять же можно решить с помощью


координатной прямой. Для этого из точки, где располагается
отрицательное число −1 нужно сдвинуться влево на три
шага. В результате мы окажемся в точке, где располагается
отрицательное число −4

Видно, что мы сдвинулись из точки где располагается


отрицательное число −1 в левую сторону на три шага, и
оказались в точке, где располагается отрицательное число
−4.

Пример 5. Найти значение выражения −2 + 2

Значение данного выражения равно 0

−2 + 2 = 0

Этот пример можно решить с помощью координатной


прямой. Для этого из точки, где располагается
отрицательное число −2 нужно сдвинуться вправо на два
шага. В результате мы окажемся в точке, где располагается
число 0
595

Видно, что мы сдвинулись из точки где располагается


отрицательное число −2 в правую сторону на два шага и
оказались в точке, где располагается число 0.

Правила сложения и вычитания целых чисел

Чтобы сложить или вычесть целые числа, вовсе


необязательно каждый раз воображать координатную
прямую, и тем более рисовать её. Можно воспользоваться
готовыми правилами.

Применяя правила, нужно обращать внимания на знак


операции и знаки чисел, которые нужно сложить или
вычесть. От этого будет зависеть какое правило применять.

Пример 1. Найти значение выражения −2 + 5

Здесь к отрицательному числу прибавляется положительное


число. Другими словами, осуществляется сложение чисел с
разными знаками, потому что −2 это отрицательное число, а
5 — положительное. Для таких случаев применяется
следующее правило:

Чтобы сложить числа с разными знаками, нужно из


большего модуля вычесть меньший модуль, и перед
полученным ответом поставить знак того числа, модуль
которого больше.
596

Итак, посмотрим какой модуль больше:

Модуль числа 5 больше, чем модуль числа −2. Правило


требует из большего модуля вычесть меньший. Поэтому мы
должны из 5 вычесть 2, и перед полученным ответом
поставить знак того числа, модуль которого больше.

У числа 5 модуль больше, поэтому знак этого числа и будет в


ответе. То есть ответ будет положительным:

−2 + 5 = 5 − 2 = 3

Обычно записывают покороче: −2 + 5 = 3

Пример 2. Найти значение выражения 3 + (−2)

Здесь как и в предыдущем примере, осуществляется


сложение чисел с разными знаками. 3 это положительное
число, а −2 — отрицательное. Обратите внимание, что число
−2 заключено в скобки, чтобы сделать выражение понятнее.
Это выражение намного проще для восприятия, чем
выражение 3 + −2.

Итак, применим правило сложения чисел с разными знаками.


Как и в прошлом примере, из большего модуля вычитаем
меньший модуль и перед ответом ставим знак того числа,
модуль которого больше:
597

3 + (−2) = |3| − |−2| = 3 − 2 = 1

Модуль числа 3 больше, чем модуль числа −2, поэтому мы из


3 вычли 2, и перед полученным ответом поставили знак того
числа, модуль которого больше. У числа 3 модуль больше,
поэтому знак этого числа и поставлен в ответе. То есть ответ
положительный.

Обычно записывают покороче 3 + (−2) = 1

Пример 3. Найти значение выражения 3 − 7

В этом выражении из меньшего числа вычитается большее.


Для такого случая применяется следующее правило:

Чтобы из меньшего числа вычесть большее, нужно из


большего числа вычесть меньшее, и перед полученным
ответом поставить минус.

3 − 7 = 7 − 3 = −4

В этом выражении есть небольшая загвоздка. Вспомним, что


знак равенства (=) ставится между величинами и
выражениями тогда, когда они равны между собой.

Значение выражения 3 − 7 как мы узнали равно −4. Это


означает, что любые преобразования которые мы будем
совершать в данном выражении, должны быть равны −4

Но мы видим, что на втором этапе располагается выражение


7 − 3, которое не равно −4.
598

Чтобы исправить эту ситуацию, выражение 7 − 3 нужно взять


в скобки и перед этой скобкой поставить минус:

3 − 7 = − (7 − 3) = − (4) = −4

В этом случае равенство будет соблюдаться на каждом


этапе:

После того, как выражение вычислено, скобки можно убрать,


что мы и сделали.

Поэтому, чтобы быть более точным, решение должно


выглядеть так:

3 − 7 = − (7 − 3) = − (4) = − 4

Данное правило можно записать с помощью переменных.


Выглядеть оно будет следующим образом:

a − b = − (b − a)

Большое количество скобок и знаков операций могут


усложнять решение, казалось бы совсем простой задачи,
599

поэтому целесообразнее научиться записывать такие


примеры коротко, например 3 − 7 = − 4.

На самом деле сложение и вычитание целых чисел сводится


только к сложению. Это означает, что если требуется
осуществить вычитание чисел, эту операцию можно
заменить сложением.

Итак, знакомимся с новым правилом:

Вычесть одно число из другого означает прибавить к


уменьшаемому такое число, которое будет
противоположно вычитаемому.

Например, рассмотрим простейшее выражение 5 − 3. На


начальных этапах изучения математики мы ставили знак
равенства и записывали ответ:

5−3=2

Но сейчас мы прогрессируем в изучении, поэтому надо


приспосабливаться к новым правилам. Новое правило
говорит, что вычесть одно число из другого означает
прибавить к уменьшаемому такое число, которое будет
противоположно вычитаемому.

На примере выражения 5 − 3 попробуем понять это правило.


Уменьшаемое в данном выражении это 5, а вычитаемое это
3. Правило говорит, что для того, чтобы из 5 вычесть 3 ,
нужно к 5 прибавить такое число, которое будет
противоположно 3. Противоположное для числа 3 это число
−3. Записываем новое выражение:

5 + (−3)
600

А как находить значения для таких выражений мы уже знаем.


Это сложение чисел с разными знаками, которое мы
рассмотрели ранее. Чтобы сложить числа с разными
знаками, мы из большего модуля вычитаем меньший модуль,
и перед полученным ответом поставить знак того числа,
модуль которого больше:

5 + (−3) = |5| − |−3| = 5 − 3 = 2

Модуль числа 5 больше, чем модуль числа −3. Поэтому мы


из 5 вычли 3 и получили 2. У числа 5 модуль больше,
поэтому знак этого числа и поставили в ответе. То есть ответ
положителен.

Поначалу быстро заменять вычитание сложением удаётся не


всем. Это связано с тем, что положительные числа
записываются без знака плюс.

Например, в выражении 3 − 1  знак минуса, указывающий на


вычитание, является знаком операции и не относится к
единице. Единица в данном случае является положительным
числом, и у неё есть свой знак плюса, но мы его не видим,
поскольку плюс перед положительными числами не
записывают.

А стало быть, для наглядности данное выражение можно


записать следующим образом:

(+3) − (+1)

Для удобства числа со своим знаками заключают в скобки. В


таком случае заменить вычитание сложением намного
проще.
601

В выражении (+3) − (+1) вычитаемое это число (+1), а


противоположное ему число это (−1).

Заменим вычитание сложением и вместо вычитаемого (+1)


записываем противоположное ему число (−1)

(+3) − (+1) = (+3) + (−1)

Дальнейшее вычисление не составит особого труда.

(+3) − (+1) = (+3) + (−1) = |3| − |−1| = 3 − 1 = 2

На первый взгляд покажется, какой смысл в этих лишних


телодвижениях, если можно старым добрым методом
поставить знак равенства и сразу записать ответ 2. На самом
деле это правило ещё не раз нас выручит.

Решим предыдущий пример 3 − 7, используя правило


вычитания. Сначала приведём выражение к понятному виду,
расставив каждому числу свои знаки.

У тройки знак плюса, поскольку она является положительным


числом. Минус, указывающий на вычитание не относится к
семёрке. У семёрки знак плюса, поскольку она является
положительным числом:

(+3) − (+7)

Заменим вычитание сложением:

(+3) − (+7) = (+3) + (−7)

Дальнейшее вычисление не составляет труда:

(+3) − (−7) = (+3) + (-7) = −(|−7| − |+3|) = −(7 − 3) = −(4) = −4


602

Пример 7. Найти значение выражения −4 − 5

Приведём выражение к понятному виду:

(−4) − (+5)

Перед нами снова операция вычитания. Эту операцию нужно


заменить сложением. К уменьшаемому (−4) прибавим число,
противоположное вычитаемому (+5). Противоположное
число для вычитаемого (+5) это число (−5).

(−4) − (+5) = (−4) + (−5)

Мы пришли к ситуации, где нужно сложить отрицательные


числа. Для таких случаев применяется следующее правило:

Чтобы сложить отрицательные числа, нужно сложить их


модули, и перед полученным ответом поставить минус.

Итак, сложим модули чисел, как от нас требует правило, и


поставим перед полученным ответом минус:

(−4) − (+5) = (−4) + (−5) = |−4| + |−5| = 4 + 5 = −9

Запись с модулями необходимо заключить в скобки и перед


этими скобками поставить минус. Так мы обеспечим минус,
который должен стоять перед ответом:

(−4) − (+5) = (−4) + (−5) = −(|−4| + |−5|) = −(4 + 5) = −(9) = −9

Решение для данного примера можно записать покороче:

−4 − 5 = −(4 + 5) = −9

или ещё короче:

−4 − 5 = −9
603

Пример 8. Найти значение выражения −3 − 5 − 7 − 9

Приведём выражение к понятному виду. Здесь все числа,


кроме числа −3 являются положительными, поэтому у них
будут знаки плюса:

(−3) − (+5) − (+7) − (+9)

Заменим вычитания сложениями. Все минусы, кроме минуса,


стоящего перед тройкой, поменяются на плюсы, и все
положительные числа поменяются на противоположные:

(−3) − (+5) − (+7) − (+9) = (−3) + (−5) + (−7) + (−9)

Теперь применим правило сложения отрицательных чисел.


Чтобы сложить отрицательные числа, нужно сложить их
модули и перед полученным ответом поставить минус:

(−3) − (+5) − (+7) − (+9) = (−3) + (−5) + (−7) + (−9) =

= −( |−3| + |−5| + |−7| + |−9| ) = −(3 + 5 + 7 + 9) = −(24) = −24

Решение данного примера можно записать покороче:

−3 − 5 − 7 − 9 = −(3 + 5 + 7 + 9) = −24

или ещё короче:

−3 − 5 − 7 − 9 = −24

Пример 9. Найти значение выражения −10 + 6 − 15 + 11 − 7

Приведём выражение к понятному виду:


604

(−10) + (+6) − (+15) + (+11) − (+7)

Здесь сразу две операции: сложение и вычитание. Сложение


оставляем без изменения, а вычитание заменяем
сложением:

(−10) + (+6) − (+15) + (+11) − (+7) = (−10) + (+6) + (−15) + (+11)


+ (−7)

Соблюдая порядок действий, выполним поочерёдно каждое


действие, опираясь на ранее изученные правила. Записи с
модулями можно пропустить:

Первое действие:

(−10) + (+6) = − (10 − 6) = − (4) = − 4

Второе действие:

(−4) + (−15) = − (4 + 15) = − (19) = − 19

Третье действие:

(−19) + (+11) = − (19 − 11) = − (8) = −8

Четвёртое действие:

(−8) + (−7) = − (8 + 7) = − (15) = − 15

Таким образом, значение выражения −10 + 6 − 15 + 11 − 7


равно −15

Примечание. Приводить выражение к понятному виду,


заключая числа в скобки, вовсе необязательно. Когда
происходит привыкание к отрицательным числам, это
605

действие можно пропустить, поскольку оно отнимает время и


может запутать.

Итак, для сложения и вычитания целых чисел необходимо


запомнить следующие правила:

Чтобы сложить числа с разными знаками, нужно из


большего модуля вычесть меньший модуль, и перед
полученным ответом поставить знак того числа, модуль
которого больше.

Чтобы из меньшего числа вычесть большее, нужно из


большего числа вычесть меньшее и перед полученным
ответом поставить минус.

Вычесть одно число из другого означает, прибавить к


уменьшаемому такое число, которое противоположно
вычитаемому.

Чтобы сложить отрицательные числа, нужно сложить их


модули, и перед полученным ответом поставить минус.

Задания для самостоятельного решения


Задание 1. Найдите значение выражения:
−50 + 40
Показать решение
Задание 2. Найдите значение выражения:
25 + (−5)
Показать решение
Задание 3. Найдите значение выражения:
606

−20 + 60
Показать решение
Задание 4. Найдите значение выражения:
20 + (−8)
Показать решение
Задание 5. Найдите значение выражения:
30 + (−50)
Показать решение
Задание 6. Найдите значение выражения:
27 + (−19)
Показать решение
Задание 7. Найдите значение выражения:
−17 + (−12) + (−8)
Показать решение
Задание 8. Найдите значение выражения:
−6 − 4
Показать решение
Задание 9. Найдите значение выражения:
−6 − (−4)
Показать решение
Задание 10. Найдите значение выражения:
−15 − (−15)
Показать решение
607

Задание 11. Найдите значение выражения:


−11 − (−14)
Показать решение
Задание 12. Найдите значение выражения:
−3 + 2 − (−1)
Показать решение
Задание 13. Найдите значение выражения:
−5 − 6 − 3
Показать решение
608

Умножение и деление целых чисел

При умножении и делении целых чисел применяется


несколько правил. В данном уроке мы рассмотрим каждое из
них.

При умножении и делении целых чисел следует обращать


внимание на знаки чисел. От них будет зависеть какое
правило применять. Необходимо также изучить несколько
законов умножения и деления. Изучение этих правил
позволит избежать некоторых досадных ошибок в будущем.

Содержание урока

 Законы умножения
 Умножение целых чисел
 Законы деления
 Деление целых чисел

Законы умножения

Некоторые из законов математики мы рассматривали в уроке


законы математики. Но мы рассмотрели не все законы. В
математике немало законов и разумнее будет изучать их
последовательно по мере необходимости.

Для начала вспомним из чего состоит умножение.


Умножение состоит из трёх параметров: множимого,
множителя и произведения. Например, в выражении
3 × 2 = 6, число 3 — это множимое, число 2 — множитель,
число 6 — произведение.
609

Множимое показывает, что именно мы увеличиваем. В


нашем примере мы увеличиваем число 3.

Множитель показывает во сколько раз нужно увеличить


множимое. В нашем примере множитель это число 2. Этот
множитель показывает во сколько раз нужно увеличить
множимое 3. То есть в ходе операции умножения число 3
будет увеличено в два раза.

Произведение это собственно результат операции


умножения. В нашем примере произведение это число 6. Это
произведение является результатом умножения 3 на 2.

Выражение 3 × 2 также можно понимать, как сумму двух


троек. Множитель 2 в таком случае будет показывать сколько
раз нужно повторить число 3:

Таким образом, если число 3 повторить два раза подряд,


получится число 6.
610

Переместительный закон умножения

Множимое и множитель называют одним общим словом –


сомножители. Переместительный закон умножения
выглядит следующим образом:

От перестановки мест сомножителей произведение не


меняется.

Проверим так ли это. Умножим к примеру 3 на 5. Здесь 3 и 5


это сомножители.

3 × 5 = 15

Теперь поменяем местами сомножители:

5 × 3 = 15

В обоих случаях мы получаем ответ 15, поэтому между


выражениями 3 × 5 и 5 × 3 можно поставить знак равенства,
поскольку они равны одному тому же значению:

3×5=5×3

15 = 15

А с помощью  переменных переместительный закон


умножения можно записать так:

a×b=b×a

где a и b — сомножители
611

Сочетательный закон умножения

Этот закон говорит о том, что если выражение состоит из


нескольких сомножителей, то произведение не будет
зависеть от порядка действий.

К примеру, выражение 3 × 2 × 4 состоит из нескольких


сомножителей. Чтобы его вычислить, можно перемножить 3 и
2, затем полученное произведение умножить на оставшееся
число 4. Выглядеть это будет так:

3 × 2 × 4 = (3 × 2) × 4 = 6 × 4 = 24

Это был первый вариант решения. Второй вариант состоит в


том, чтобы перемножить 2 и 4, затем полученное
произведение умножить на оставшееся число 3. Выглядеть
это будет так:

3 × 2 × 4 = 3 × (2 × 4) = 3 × 8 = 24

В обоих случаях мы получаем ответ 24. Поэтому между


выражениями (3 × 2) × 4 и 3 × (2 × 4) можно поставить знак
равенства, поскольку они равны одному и тому же значению:

(3 × 2) × 4 = 3 × (2 × 4)

24 = 24

а с помощью переменных сочетательный закон умножения


можно записать так:

a × b × c = (a × b) × c = a × (b × c)

где вместо a, b, c могут стоять любые числа.


612

Распределительный закон умножения

Распределительный закон умножения позволяет умножить


сумму на число. Для этого каждое слагаемое этой суммы
умножается на это число, затем полученные результаты
складывают.

Например, найдём значение выражения (2 + 3) × 5

Выражение находящееся в скобках является суммой. Эту


сумму нужно умножить на число 5. Для этого каждое
слагаемое этой суммы, то есть числа 2 и 3 нужно умножить
на число 5, затем полученные результаты сложить:

(2 + 3) × 5 = 2 × 5 + 3 × 5 = 10 + 15 = 25

Значит значение выражения (2 + 3) × 5 равно 25.

С помощью переменных распределительный закон


умножения записывается так:

(a + b) × c = a × c + b × c
613

где вместо a, b, c могут стоять любые числа.

Закон умножения на ноль

Этот закон говорит о том, что если в любом умножении


имеется хотя бы один ноль, то в ответе получится ноль.

Произведение равно нулю, если хотя бы один из


сомножителей равен нулю.

Например, выражение 0 × 2 равно нулю

0×2=0

В данном случае число 2 является множителем и показывает


во сколько раз нужно увеличить множимое. То есть во
сколько раз увеличить ноль. Буквально это выражение
читается так: «увеличить ноль в два раза». Но как можно
увеличить ноль в два раза, если это ноль? Ответ — никак.

Иными словами, если «ничего» увеличить в два раза или


даже в миллион раз, всё равно получится «ничего».

И если в выражении 0 × 2 поменять местами сомножители,


опять же получится ноль. Это мы знаем из предыдущего
переместительного закона:

0×2=2×0

0=0

Примеры применения закона умножения на ноль:

5×0=0
614

5×5×5×0=0

2 × 5  × 0 × 9  × 1 = 0

В последних двух примерах имеется несколько


сомножителей. Увидев в них ноль, мы сразу в ответе
поставили ноль, применив закон умножения на ноль.

Мы рассмотрели основные законы умножения. Теперь


рассмотрим самó умножение целых чисел.

Умножение целых чисел

Пример 1. Найти значение выражения −5 × 2

Это умножение чисел с разными знаками. −5 является


отрицательным числом, а 2 – положительным. Для таких
случаев нужно применять следующее правило:

Чтобы перемножить числа с разными знаками, нужно


перемножить их модули, и перед полученным ответом
поставить минус.

−5 × 2 = − (|−5| × |2|) = − (5 × 2) = − (10) = −10

Обычно записывают короче:  −5 × 2 = −10

Любое умножение может быть представлено в виде суммы


чисел. Например, рассмотрим выражение 2 × 3. Оно равно 6.

2×3=6

Множителем в данном выражение является число 3. Этот


множитель показывает во сколько раз нужно увеличить
615

двойку. Но выражение 2 × 3 также можно понимать как сумму


трёх двоек:

То же самое происходит и с выражением −5 × 2. Это


выражение может быть представлено в виде суммы

А выражение (−5) + (−5) равно −10. Мы это знаем из


прошлого урока. Это сложение отрицательных чисел.
Напомним, что результат сложения отрицательных чисел
есть отрицательное число.

Пример 2. Найти значение выражения 12 × (−5)

Это умножение чисел с разными знаками. 12 –


положительное число, (−5) – отрицательное. Опять же
применяем предыдущее правило. Перемножаем модули
чисел и перед полученным ответом ставим минус:

12 × (−5) = − (|12| × |−5|) = − (12 × 5) = − (60) = −60

Обычно решение записывают покороче:

12 × (−5) = −60
616

Пример 3. Найти значение выражения 10 × (−4) × 2

Это выражение состоит из нескольких сомножителей.


Сначала перемножим 10 и (−4), затем полученное число
умножим на 2. Попутно применим ранее изученные правила:

Первое действие:

10 × (−4) = −(|10| × |−4|) = −(10 × 4) = (−40) = −40

Второе действие:

−40 × 2 = −(|−40 | × | 2|) = −(40 × 2) = −(80) = −80

Значит значение выражения 10 × (−4) × 2 равно −80

Запишем решение покороче:

10 × (−4) × 2 = −40 × 2 = −80

Пример 4. Найти значение выражения (−4) × (−2)

Это умножение отрицательных чисел. В таких случаях нужно


применять следующее правило:

Чтобы перемножить отрицательные числа, нужно


перемножить их модули и перед полученным ответом
поставить плюс

(−4) × (−2) = |−4| × |−2| = 4 × 2 = 8

Плюс по традиции не записываем, поэтому просто


записываем ответ 8.
617

Запишем решение покороче (−4) × (−2) = 8

Возникает вопрос почему при умножении отрицательных


чисел вдруг получается положительное число. Давайте
попробуем доказать, что (−4) × (−2) равно 8 и ни чему
другому.

Сначала запишем следующее выражение:

4 × (−2)

Заключим его в скобки:

( 4 × (−2) )

Прибавим к этому выражению наше выражение (−4) × (−2).


Его тоже заключим в скобки:

( 4 × (−2) ) + ( (−4) × (−2) )

Всё это приравняем к нулю:

(4 × (−2)) + ((−4) × (−2)) = 0

Теперь начинается самое интересное. Суть в том, что мы


должны вычислить левую часть этого выражения, и в
результате получить 0.

Итак, первое произведение (4 × (−2)) равно −8. Запишем в


нашем выражении число −8 вместо произведения (4 × (−2))

−8 + ((−4) × (−2)) = 0

Теперь вместо второго произведения временно поставим


многоточие

−8 + … = 0
618

Теперь внимательно посмотрим на выражение −8 + … = 0.


Какое число должно стоять вместо многоточия, чтобы
соблюдалось равенство? Ответ напрашивается сам. Вместо
многоточия должно стоять положительное число 8 и никакое
другое. Только так будет соблюдаться равенство. Ведь
−8 + 8 равно 0.

Возвращаемся к выражению −8 + ((−4) × (−2)) = 0 и вместо


произведения ((−4) × (−2)) записываем число 8

−8 + 8 = 0

Пример 5. Найти значение выражения  −2 × (6 + 4)

Применим распределительный закон умножения, то есть


умножим число  −2 на каждое слагаемое суммы (6 + 4)

−2 × (6 + 4) = −2 × 6 + (−2) × 4

Теперь выполним умножение, и сложим полученные


результаты. Попутно применим ранее изученные правила.
Запись с модулями можно пропустить, чтобы не
загромождать выражение

Первое действие:

−2 × 6 = −12

Второе действие:

−2 × 4 = −8

Третье действие:

−12 + (−8) = −20


619

Значит значение выражения −2 × (6 + 4) равно −20

Запишем решение покороче:

−2 × (6 + 4) = (−12) + (−8) = −20

Пример 6. Найти значение выражения (−2) × (−3) × (−4)

Выражение состоит из нескольких сомножителей. Сначала


перемножим числа −2 и −3, и полученное произведение
умножим на оставшееся число −4. Запись с модулями
пропустим, чтобы не загромождать выражение

Первое действие:

(−2) × (−3) = 6

Второе действие:

6 × (−4) = −(6 × 4) = −24

Значит значение выражения (−2) × (−3) × (−4) равно −24

Запишем решение покороче:

(−2) × (−3) × (−4) = 6 × (−4) = −24

Законы деления

Прежде чем делить целые числа, необходимо изучить два


закона деления.

В первую очередь, вспомним из чего состоит деление.


Деление состоит из трёх параметров: делимого, делителя и
620

частного. Например, в выражении 8 : 2 = 4,  8 – это делимое,


2 – делитель, 4 – частное.

Делимое показывает, что именно мы делим. В нашем


примере мы делим число 8.

Делитель показывает на сколько частей нужно разделить


делимое. В нашем примере делитель это число 2. Этот
делитель показывает на сколько частей нужно разделить
делимое 8. То есть в ходе операции деления, число 8 будет
разделено на две части.

Частное – это собственно результат операции деления. В


нашем примере частное это число 4. Это частное является
результатом деления 8 на 2.

Далее рассмотрим законы деления.

На ноль делить нельзя

Любое число запрещено делить на ноль.

Дело в том, что деление это действие, обратное умножению.


Данную фразу можно понимать в прямом смысле. Например,
если 2 × 5 = 10, то 10 : 5 = 2.
621

Видно, что второе выражение записано в обратном порядке.


Если к примеру, у нас имеется два яблока и мы захотим
увеличить их в пять раз, то мы запишем 2 × 5 = 10.
Получится десять яблок. Затем, если мы захотим обратно
уменьшить эти десять яблок до двух, то мы
запишем 10 : 5 = 2

Точно так же можно поступать и с другими выражениями.


Если к примеру, 2 × 6 = 12, то мы можем обратно вернуться к
изначальному числу 2. Для этого достаточно записать
выражение 2 × 6 = 12 в обратном порядке, разделяя 12 на 6

12 : 6 = 2

Теперь рассмотрим выражение 5 × 0. Мы знаем из законов


умножения, что произведение равно нулю, если хотя бы один
из сомножителей равен нулю. Значит и выражение 5 × 0
равно нулю

5×0=0

Если записать это выражение в обратном порядке, то


получим:

0:0=5

Сразу в глаза бросается ответ 5, который получается в


результате деления ноль на ноль. Это невозможно.
622

В обратном порядке можно записать и другое похожее


выражение, например 2 × 0 = 0

0:0=2

В первом случае, разделив ноль на ноль мы получили 5, а во


втором случае 2. То есть каждый раз деля ноль на ноль, мы
можем получить разные значения, а это недопустимо.

Второе объяснение заключается в том, что разделить


делимое на делитель означает найти такое число, которое
при умножении на делитель даст делимое.

Например выражение 8 : 2 означает найти такое число,


которое при умножении на 2 даст 8

…×2=8

Здесь вместо многоточия должно стоять число, которое при


умножении на 2 даст ответ 8. Чтобы найти это число,
достаточно записать это выражение в обратном порядке:

8:2=4

Получили число 4. Запишем его вместо многоточия:

4×2=8

Теперь представим, что нужно найти значение выражения 5 :


0. В данном случае 5 – это делимое, 0 – делитель. Разделить
5 на 0 означает найти такое число, которое при умножении
на 0 даст 5

…×0=5
623

Здесь вместо многоточия должно стоять число, которое при


умножении на 0 даст ответ 5. Но не существует числа,
которое при умножении на ноль даёт 5.

Выражение … × 0 = 5 противоречит закону умножения на


ноль, который утверждает, что произведение равно нулю,
когда хотя бы один из сомножителей равен нулю.

А значит записывать выражение … × 0 = 5 в обратном


порядке, деля 5 на 0 нет никакого смысла. Поэтому и
говорят, что на ноль делить нельзя.

С помощью переменных данный закон записывается


следующим образом:

,  при b ≠ 0

Это выражение можно прочитать так:

Число a можно разделить на число b, при условии, что b


не равно нулю.

Свойство частного

Этот закон говорит о том, что если делимое и делитель


умножить или разделить на одно и то же число, то частное не
изменится.

Например, рассмотрим выражение 12 : 4. Значение этого


выражения равно 3

12 : 4 = 3
624

Попробуем умножить делимое и делитель на одно и то же


число, например на число 4. Если верить свойству частного,
мы опять должны получить в ответе число 3

(12 × 4) : (4 × 4)
(12 × 4) : (4 × 4) = 48 : 16 = 3

Получили ответ 3.

Теперь попробуем не умножить, а разделить делимое и


делитель на число 4

(12 : 4) : (4 : 4)
(12 : 4) : (4 : 4) = 3 : 1 = 3

Получили ответ 3.

Видим, что если делимое и делитель умножить или


разделить на одно и то же число, то частное не меняется.
625

Мы рассмотрели два закона деления. Далее рассмотрим


деление целых чисел.

Деление целых чисел

Пример 1. Найти значение выражения 12 : (−2)

Это деление чисел с разными знаками. 12 — положительное


число, (−2) – отрицательное. Чтобы решить этот пример,
нужно модуль делимого разделить на модуль делителя,
и перед полученным ответом поставить минус.

12 : (−2) = −(|12| : |−2|) = −(12 : 2) = −(6) = −6

Обычно записывают покороче:

12 : (−2) = −6

Пример 2. Найти значение выражения −24 : 6

Это деление чисел с разными знаками. −24 – это


отрицательное число, 6 – положительное. Опять же модуль
делимого делим на модуль делителя, и перед
полученным ответом ставим минус.

−24 : 6 = −(|−24| : |6|) = −(24 : 6) = −(4) = −4

Запишем решение покороче:

−24 : 6 = −4

Пример 3. Найти значение выражения −45 : (−5)


626

Это деление отрицательных чисел. Чтобы решить этот


пример, нужно модуль делимого разделить на модуль
делителя, и перед полученным ответом поставить знак
плюс.

−45 : (−5) = |−45| : |−5| = 45 : 5 = 9

Запишем решение покороче:

−45 : (−5) = 9

Пример 4. Найти значение выражения −36 : (−4) : (−3)

Согласно порядку действий, если в выражении присутствует


только умножение или деление, то все действия нужно
выполнять слева направо в порядке их следования.

Разделим −36 на (−4), и полученное число разделим на −3

Первое действие:

−36 : (−4) = |−36| : |−4| = 36 : 4 = 9

Второе действие:

9 : (−3) = −(|9| : |−3|) = −(9 : 3) = −(3) = −3

Запишем решение покороче:

−36 : (−4) : (−3) = 9 : (−3) = −3


627

Рациональные числа

Тема рациональных чисел достаточно обширна. О ней можно


говорить бесконечно и писать целые труды, каждый раз
удивляясь новым фишкам.

Чтобы не допускать в будущем ошибок, в данном уроке мы


немного углубимся в тему рациональных чисел, почерпнём
из неё необходимые сведения и двинемся дальше.

Содержание урока

 Что такое рациональное число


 Рациональные числа на координатной прямой
 Минус перед рациональным числом
 Противоположные рациональные числа
 Перевод смешанных чисел в неправильные дроби

Что такое рациональное число

Рациональное число — это число, которое может быть


представлено в виде дроби  , где a — это числитель дроби,
b — знаменатель дроби. Причем b не должно быть нулём,
поскольку деление на ноль не допускается.

К рациональным числам относятся следующие категории


чисел:

 целые числа (например −2, −1, 0 1, 2 и т.д.)

 обыкновенные дроби (например ,  ,    и т.п.)


628

 смешанные числа (например ,  ,    и т.п.)

 десятичные дроби (например 0,2 и т.п.)

 бесконечные периодические дроби (например 0,(3) и т.п.)

Каждое число из этой категории может быть представлено в


виде дроби .

Примеры:

Пример 1. Целое число 2 может быть представлено в виде

дроби . Значит число 2 относится не только к целым числам,


но и к рациональным.

Пример 2. Смешанное число может быть представлено в

виде дроби . Данная дробь получается путём перевода


смешанного числа в неправильную дробь

Значит смешанное число относится к рациональным


числам.

Пример 3. Десятичная дробь 0,2 может быть представлена в

виде дроби . Данная дробь получилась путём перевода


десятичной дроби 0,2 в обыкновенную дробь. Если
629

испытываете затруднения на этом моменте, повторите тему


десятичных дробей.

Поскольку десятичная дробь 0,2 может быть представлена в

виде дроби , значит она тоже относится к рациональным


числам.

Пример 4. Бесконечная периодическая дробь 0, (3) может

быть представлена в виде дроби . Данная дробь получается


путём перевода чистой периодической дроби в
обыкновенную дробь. Если испытываете затруднения на
этом моменте, повторите тему периодические дроби.

Поскольку бесконечная периодическая дробь 0, (3) может

быть представлена в виде дроби  , значит она тоже


относится к рациональным числам.

В дальнейшем, все числа которые можно представить в виде


дроби, мы всё чаще будем называть одним словосочетанием
— рациональные числа.

Рациональные числа на координатной прямой

Координатную прямую мы рассматривали, когда изучали


отрицательные числа. Напомним, что это прямая линия на
которой лежат множество чисел. Выглядит следующим
образом:
630

На этом рисунке приведен небольшой фрагмент


координатной прямой от −5 до 5.

Отметить на координатной прямой целые числа вида 2, 0, −3


не составляет особого труда.

Намного интереснее дела обстоят с остальными числами: с


обыкновенными дробями, смешанными числами,
десятичными дробями и т.д. Эти числа лежат между целыми
числами и этих чисел бесконечно много.

Например, отметим на координатной прямой рациональное

число . Данное число располагается ровно между нулём и


единицей

Попробуем понять, почему дробь  вдруг расположилась


между нулём и единицей.

Как уже говорилось выше, между целыми числами лежат


остальные числа — обыкновенные дроби, десятичные дроби,
смешанные числа и т.д. К примеру, если увеличить участок
координатной прямой от 0 до 1, то можно увидеть
следующую картину
631

Видно, что между целыми числами 0 и 1 лежат уже другие


рациональные числа, которые являются знакомыми для нас

десятичными дробями. Здесь же видна наша дробь ,


которая расположилась там же, где и десятичная дробь 0,5.
Внимательное рассмотрение этого рисунка даёт ответ на

вопрос почему дробь расположилась именно там.

Дробь означает разделить 1 на 2. А если разделить 1 на 2,


то мы получим 0,5

Десятичную дробь 0,5 можно замаскировать и под другие


дроби. Из основного свойства дроби мы знаем, что если
числитель и знаменатель дроби умножить или разделить на
одно и то же число, то значение дроби не изменится.
632

Если числитель и знаменатель дроби умножить на любое

число, например на число 4, то мы получим новую дробь  , а

эта дробь также как и  равна 0,5

А значит на координатной прямой дробь можно

расположить там же, где и располагалась дробь

Пример 2. Попробуем отметить на координатной

рациональное число . Данное число располагается ровно


между числами 1 и 2

Значение дроби равно 1,5


633

Если увеличить участок координатной прямой от 1 до 2, то


мы увидим следующую картину:

Видно, что между целыми числами 1 и 2 лежат уже другие


рациональные числа, которые являются знакомыми для нас

десятичными дробями. Здесь же видна наша дробь ,


которая расположилась там же, где и десятичная дробь 1,5.

Мы увеличивали определенные отрезки на координатной


прямой, чтобы увидеть остальные числа, лежащие на этом
отрезке. В результате, мы обнаруживали десятичные дроби,
которые имели после запятой одну цифру.

Но это были не единственные числа, лежащие на этих


отрезках. Чисел, лежащих на координатной прямой
бесконечно много.

Нетрудно догадаться, что между десятичными дробями,


имеющими после запятой одну цифру, лежат уже другие
634

десятичные дроби, имеющие после запятой две цифры.


Другими словами, сотые части отрезка.

К примеру, попробуем увидеть числа, которые лежат между


десятичными дробями 0,1 и 0,2

Ещё пример. Десятичные дроби, имеющие две цифры после


запятой и лежащие между нулём и рациональным числом 0,1
выглядят так:

Пример 3. Отметим на координатной прямой рациональное

число . Данное рациональное число будет располагаться


очень близко к нулю

Значение дроби равно 0,02


635

Если мы увеличим отрезок от 0 до 0,1 то увидим где точно

расположилось рациональное число

Видно, что наше рациональное число расположилось там


же, где и десятичная дробь 0,02.

Пример 4. Отметим на координатной прямой рациональное


число 0, (3)

Рациональное число 0, (3) является бесконечной


периодической дробью. Его дробная часть никогда не
заканчивается, она бесконечная

0,33333….и так далее до бесконечности..

И поскольку у числа 0,(3) дробная часть является


бесконечной, это означает, что мы не сможем найти точное
место на координатной прямой, где это число располагается.
Мы можем лишь указать это место приблизительно.
636

Рациональное число 0,33333… будет располагаться очень


близко к обычной десятичной дроби 0,3

Данный рисунок не показывает точное место расположения


числа 0,(3). Это лишь иллюстрация, показывающая как
близко может располагаться периодическая дробь 0,(3) к
обычной десятичной дроби 0,3.

Пример 5. Отметим на координатной прямой рациональное

число . Данное рациональное число будет располагаться


посередине между числами 2 и 3

это есть 2 (две целых) и (одна вторая). Дробь по


другому ещё называют «половиной». Поэтому мы отметили
на координатной прямой два целых отрезка и ещё половину
отрезка.

Если перевести смешанное число в неправильную дробь,

то получим обыкновенную дробь . Эта дробь на


637

координатной прямой будет располагаться там же, где и

дробь

Значение дроби равно 2,5

Если увеличить участок координатной прямой от 2 до 3, то


мы увидим следующую картину:

Видно, что наше рациональное число  расположилось там


же, где и десятичная дробь 2,5

Минус перед рациональным числом

В предыдущем уроке, который назвался умножение и


деление целых чисел мы научились делить целые числа. В
638

роли делимого и делителя могли стоять как положительные,


так и отрицательные числа.

Рассмотрим простейшее выражение

(−6) : 2 = −3

В данном выражении делимое (−6) является отрицательным


числом.

Теперь рассмотрим второе выражение

6 : (−2) = −3

Здесь уже отрицательным числом является делитель (−2).


Но в обоих случаях мы получаем один и тот же ответ −3.

Учитывая, что любое деление можно записать в виде дроби,


мы можем рассмотренные выше примеры также записать в
виде дроби:

А поскольку в обоих случаях значение дроби одинаково,


минус стоящий либо в числителе либо в знаменателе можно
сделать общим, поставив его перед дробью
639

Поэтому между выражениями      и    и    можно


поставить знак равенства, потому что они несут одно и то же
значение

В дальнейшем работая с дробями, если минус будет нам


встречаться в числителе или в знаменателе, мы будем
делать этот минус общим, ставя его перед дробью.

Противоположные рациональные числа

Как и целое число, рациональное число имеет своё


противоположное число.

Например, для рационального числа противоположным

числом является . Располагается оно на координатной

прямой симметрично расположению   относительно начала


координат. Другими словами, оба этих числа равноудалены
от начала координат

Перевод смешанных чисел в неправильные дроби

Мы знаем что для того, чтобы перевести смешанное число в


неправильную дробь, нужно целую часть умножить на
знаменатель дробной части и прибавить к числителю
640

дробной части. Полученное число будет числителем новой


дроби, а знаменатель остаётся прежним..

Например, переведём смешанное число   в неправильную


дробь

Умножим целую часть на знаменатель дробной части и


прибавим числитель дробной части:

(2 × 2) + 1

Вычислим данное выражение:

(2 × 2) + 1 = 4 + 1 = 5

Полученное число 5 будет числителем новой дроби, а


знаменатель останется прежним:

Полностью данная процедура записывается следующим


образом:

Чтобы вернуть изначальное смешанное число, достаточно

выделить целую часть в дроби

Но этот способ перевода смешанного числа в неправильную


дробь применим только в том случае, если смешанное число
641

является положительным. Для отрицательного числа данный


способ не сработает.

Рассмотрим дробь . Выделим в этой дроби целую часть.

Получим

Чтобы вернуть изначальную дробь нужно перевести

смешанное число   в неправильную дробь. Но если мы


воспользуемся старым правилом, а именно умножим целую
часть на знаменатель дробной части и к полученному числу
прибавим числитель дробной части, то получим следующее
противоречие:

Мы получили дробь , а должны были получить дробь .

Делаем вывод, что смешанное число в неправильную


дробь переведено неправильно:
642

Чтобы правильно перевести отрицательное смешанное


число в неправильную дробь, нужно целую часть умножить
на знаменатель дробной части, и из полученного числа
вычесть числитель дробной части. В этом случае у нас всё
встанет на свои места

Отрицательное смешанное число является

противоположным для смешанного числа . Если

положительное смешанное число располагается в правой


части и выглядит так

то отрицательное смешанное число будет располагаться

в левой части симметрично относительное начала


координат

И если читается как «две целых и одна вторая», то


читается как «минус две целых и минус одна вторая».
643

Поскольку числа −2 и располагаются в левой части


координатной прямой — они оба являются отрицательными.

Любое смешанное число можно записать в развёрнутом

виде. Положительное смешанное число в развёрнутом

виде записывается как .

А отрицательное смешанное число записывается как

Теперь мы можем понять, почему смешанное число


расположилось в левой части координатной прямой. Минус
перед двойкой указывает, что мы сдвинулись от нуля на два
шага влево, в результате оказались в точке, где находится
число −2

Затем, начиная от числа −2 сдвинулись ещё влево на

шага. А поскольку значение равно −0,5 то наш шаг будет


половиной от полного шага.
644

В итоге, мы окажемся посередине между числами −3 и −2

Пример 2. Выделить в неправильной дроби целую часть,


затем полученное смешанное число обратно перевести в
неправильную дробь

Выполним первую часть задания, а именно выделим в

неправильной дроби целую часть

Выполним вторую часть задания, а именно переведём

полученное смешанное число в неправильную дробь. Для


этого умножим целую часть на знаменатель дробной части и
из полученного числа вычтем числитель дробной части:
645

Если нет желания путаться и привыкать к новому правилу, то


можно  смешанное число заключить в скобки, а минус
оставить за скобкой. Тогда можно будет применить старое
доброе правило: умножить целую часть на знаменатель
дробной части и к полученному числу прибавить числитель
дробной части.

Выполним предыдущее задание этим способом, а именно

переведём смешанное число в неправильную дробь


646

Сравнение рациональных чисел

Продолжаем изучать рациональные числа. В данном уроке


мы научимся сравнивать их.

Из предыдущих уроков мы узнали, что чем правее число


располагается на координатной прямой, тем оно больше. И
соответственно, чем левее располагается число на
координатной прямой, тем оно меньше.

Например, если сравнивать числа 4 и 1, то можно сразу


ответить, что 4 больше чем 1. Это вполне логичное
утверждение и каждый с этим согласится.

В качестве доказательства можно привести координатную


прямую. На ней видно, что четвёрка лежит правее единицы

4>1

Для этого случая есть и правило, которое при желании


можно использовать. Выглядит оно следующим образом:

Из двух положительных чисел больше то число, модуль


которого больше.

Чтобы ответить на вопрос какое число больше, а какое


меньше, сначала нужно найти модули этих чисел, сравнить
эти модули, а потом уже ответить на вопрос.

Например, сравним те же числа 4 и 1, применяя


вышеприведенное  правило
647

Находим модули чисел:

|4| = 4

|1| = 1

Сравниваем найденные модули:

4>1

Отвечаем на вопрос:

4>1

Для отрицательных чисел существует другое правило,


выглядит оно следующим образом:

Из двух отрицательных чисел больше то число, модуль


которого меньше.

Например, сравним числа −3 и −1

Находим модули чисел

|−3| = 3

|−1| = 1

Сравниваем найденные модули:

3>1

Отвечаем на вопрос:

−3 < −1

Нельзя путать модуль числа с самим числом. Частая ошибка


многих новичков. К примеру, если модуль числа −3 больше,
648

чем модуль числа −1, это не означает, что число −3 больше,


чем число −1.

Число −3 меньше, чем число −1. Это можно понять, если


воспользоваться координатной прямой

Видно, что число −3 лежит левее, чем −1. А мы знаем, что


чем левее, тем меньше.

Если сравнивать отрицательное число с положительным, то


ответ будет напрашиваться сам. Любое отрицательное число
будет меньше любого положительного числа. Например, −4
меньше, чем 2

−4 < 2

Видно, что −4 лежит левее, чем 2. А мы знаем, что «чем


левее, тем меньше».

Здесь в первую очередь нужно смотреть на знаки чисел.


Минус перед числом будет говорить о том, что число
отрицательное. Если знак числа отсутствует, то число
положительное, но вы можете записать его для наглядности.
Напомним, что это знак плюса

−4 < +2
649

Мы рассмотрели в качестве примера целые числа, вида


−4, −3 −1, 2. Сравнить такие числа, а также изобразить на
координатной прямой не составляет особого труда.

Намного сложнее сравнивать другие виды чисел, такие как


обыкновенные дроби, смешанные числа и десятичные дроби,
некоторые из которых являются отрицательными. Здесь уже
в основном придётся применять правила, потому что точно
изобразить такие числа на координатной прямой не всегда
возможно. В некоторых случаях, число надо будет
видоизменять, чтобы сделать его более простым для
сравнения и восприятия.

Пример 1. Сравнить рациональные числа

Итак, требуется сравнить отрицательное число


с положительным. Любое отрицательное число меньше
любого положительного числа. Поэтому не теряя времени

отвечаем, что    меньше, чем

Пример 2. Сравнить рациональные числа   и 

Требуется сравнить два отрицательных числа. Из двух


отрицательных чисел больше то, модуль которого меньше.

Находим модули чисел:


650

Сравниваем найденные модули:

Согласно правилу, из двух отрицательных чисел больше то

число, модуль которого меньше. Значит рациональное

больше, чем  , потому что модуль числа меньше, чем

модуль числа

Пример 3. Сравнить числа 2,35 и

Требуется сравнить положительное число с отрицательным.


Любое положительное число больше любого отрицательного
числа. Поэтому не теряя времени отвечаем что 2,35 больше,

чем

2,35 > 

Пример 4. Сравнить рациональные числа   и


651

Требуется сравнить два отрицательных числа. Из двух


отрицательных чисел больше то число, модуль которого
меньше.

Находим модули чисел:

Сравниваем найденные модули. Но сначала приведём их к


понятному виду, чтобы проще было сравнить, а именно
переведём в неправильные дроби и приведём к общему
знаменателю

Согласно правилу, из двух отрицательных чисел больше то

число, модуль которого меньше. Значит рациональное 

больше, чем  , потому что модуль числа  меньше, чем

модуль числа
652

Пример 5. Сравнить рациональные числа 0 и

Требуется сравнить ноль с отрицательным числом. Ноль


больше любого отрицательного числа, поэтому не теряя

времени отвечаем, что 0 больше, чем

Пример 6. Сравнить рациональные числа 0 и 

Требуется сравнить ноль с положительным числом. Ноль


меньше любого положительного числа, поэтому не теряя

времени отвечаем, что 0 меньше, чем

Пример 7. Сравнить рациональные числа 4,53 и 4,403

Требуется сравнить два положительных числа. Из двух


положительных чисел больше то число, модуль которого
больше.

Сделаем в обеих дробях количество цифр после запятой


одинаковым. Для этого в дроби 4,53 припишем в конце один
ноль

4,530

Далее применим правило сравнения положительных чисел.


653

Находим модули чисел

|4,530| = 4,530

|4,403| = 4,403

Сравниваем найденные модули:

Согласно правилу, из двух положительных чисел больше то


число, модуль которого больше. Значит рациональное число
4,53 больше, чем 4,403 потому что модуль числа 4,53
больше, чем модуль числа 4,403

4,53 > 4,403

Пример 8. Сравнить рациональные числа  и

Требуется сравнить два отрицательных числа. Из двух


отрицательных чисел больше то число, модуль которого
меньше.

Находим модули чисел:


654

Сравниваем найденные модули. Но сначала приведём их к


понятному виду, чтобы проще было сравнить, а именно

переведём смешанное число  в неправильную дробь,


затем приведём обе дроби к общему знаменателю:

Согласно правилу, из двух отрицательных чисел больше то

число, модуль которого меньше. Значит рациональное 

больше, чем  , потому что модуль числа  меньше, чем

модуль числа 

Сравнивать десятичные дроби намного проще, чем


обыкновенные дроби и смешанные числа. В некоторых
случаях, посмотрев на целую часть такой дроби, можно сразу
ответить на вопрос какая дробь больше, а какая меньше.

Чтобы сделать это, нужно сравнить модули целых частей.


Это позволит быстро ответить на вопрос в задаче. Ведь как
известно, целые части в десятичных дробях имеют вес
больший, чем дробные.
655

Пример 9. Сравнить рациональные числа 15,4 и 2,1256

Модуль целой части дроби 15,4 больше, чем модуль целой


части дроби 2,1256

|15| = 15

|2| = 2

15 > 2

поэтому и дробь 15,4 больше, чем дробь 2,1256

15,4 > 2,1256

Другими словами, нам не пришлось тратить время на


дописывание нулей дроби 15,4 и сравнивать получившиеся
дроби, как обычные числа

15,4000   2,1256

154000 > 21256

Правила сравнения остаются всё теми же. В нашем случае


мы сравнивали положительные числа.

Пример 10. Сравнить рациональные числа −15,2 и −0,152

Требуется сравнить два отрицательных числа. Из двух


отрицательных чисел больше то число, модуль которого
меньше. Но мы сравним только модули целых частей

|−15| = 15

|−0| = 0
656

15 > 0

Видим, что модуль целой части дроби −15,2 больше, чем


модуль целой части дроби −0,152.

А значит рациональное −0,152 больше, чем −15,2 потому что


модуль целой части числа −0,152 меньше, чем модуль целой
части числа −15,2

 −0,152 > −15,2

Пример 11. Сравнить рациональные числа −3,4 и −3,7

Требуется сравнить два отрицательных числа. Из двух


отрицательных чисел больше то число, модуль которого
меньше. Но мы сравним только модули целых частей. Но
проблема в том, что модули целых чисел равны:

|−3| = 3

|−3| = 3

3=3

В этом случае придётся пользоваться старым методом:


найти модули рациональных чисел и сравнить эти модули

|−3,4| = 3,4

|−3,7| = 3,7

Сравниваем найденные модули:


657

Согласно правилу, из двух отрицательных чисел больше то


число, модуль которого меньше. Значит рациональное −3,4
больше, чем −3,7 потому что модуль числа −3,4 меньше, чем
модуль числа −3,7

−3,4 > −3,7

Пример 12. Сравнить рациональные числа 0,(3) и 

Требуется сравнить два положительных числа. Причем


сравнить периодическую дробь с простой дробью.

Переведём периодическую дробь 0,(3) в обыкновенную

дробь и сравним её с дробью  . После перевода


периодической дроби 0,(3) в обыкновенную, она обращается

в дробь

Находим модули чисел:


658

Сравниваем найденные модули. Но сначала приведём их к


понятному виду, чтобы проще было сравнить, а именно

приведём к общему знаменателю:

Согласно правилу, из двух положительных чисел больше то


число, модуль которого больше. Значит рациональное число

больше, чем 0,(3) потому что модуль числа больше, чем


модуль числа 0,(3)

0,(3)  < 
659

Сложение и вычитание рациональных чисел

В данном уроке рассматривается сложение и вычитание


рациональных чисел. Тема относится к категории сложных.
Здесь необходимо использовать весь арсенал полученных
ранее знаний.

Правила сложения и вычитания целых чисел справедливы и


для рациональных чисел. Напомним, что рациональными
называют числа, которые могут быть представлены в виде
дроби  ,  где a – это числитель дроби, b – знаменатель
дроби. При этом, b не должно быть нулём.

В данном уроке дроби и смешанные числа мы всё чаще


будем называть одним общим словосочетанием —
рациональные числа.

Навигация по уроку:

 Сложение рациональных чисел с разными знаками


 Сложение отрицательных рациональных чисел
 Несколько дробей
 Десятичные дроби
 Задания для самостоятельного решения

Пример 1. Найти значение выражения: 

Заключим каждое рациональное число в скобки вместе со


своими знаками. Учитываем, что плюс который дан в
выражении, является знаком операции и не относится к
660

дроби . У этой дроби свой знак плюса, который невидим по


причине того, что его не записывают. Но мы запишем его для
наглядности:

Это сложение рациональных чисел с разными знаками.


Чтобы сложить рациональные числа с разными знаками,
нужно из большего модуля вычесть меньший модуль, и
перед полученным ответом поставить знак того
рационального числа, модуль которого больше. А чтобы
понять какой модуль больше, а какой меньше, нужно суметь
сравнить модули этих дробей до их вычисления:

Модуль рационального числа    больше, чем модуль

рационального числа  . Поэтому мы из  вычли . Получили

ответ  . Затем сократив эту дробь на 2, получили

окончательный ответ .

Некоторые примитивные действия, такие как заключение


чисел в скобки и проставление модулей, можно пропустить.
Данный пример вполне можно записать покороче:
661

Пример 2. Найти значение выражения: 

Заключим каждое рациональное число в скобки вместе со


своими знаками. Учитываем, что минус, стоящий между

рациональными числами  и   является знаком операции и

не относится к дроби . У этой дроби свой знак плюса,


который невидим по причине того, что его не записывают. Но
мы запишем его для наглядности:

Заменим вычитание сложением. Напомним, что для


этого нужно к уменьшаемому прибавить число,
противоположное вычитаемому:

Получили сложение отрицательных рациональных чисел.


Чтобы сложить отрицательные рациональные числа,
нужно сложить их модули и перед полученным ответом
поставить минус:

Запишем решение данного примера покороче:


662

Примечание. Заключать в скобки каждое рациональное


число вовсе необязательно. Делается это для удобства,
чтобы хорошо видеть какие знаки имеют рациональные
числа.

Пример 3. Найти значение выражения: 

В этом выражении у дробей разные знаменатели. Чтобы


облегчить себе задачу, приведём эти дроби к общему
знаменателю. Не будем подробно останавливаться на том
как это сделать. Если испытываете с этим затруднения,
обязательно повторите урок действия с дробями.

После приведения дробей к общему знаменателю


выражение примет следующий вид:

Заключим каждое рациональное число в скобки вместе


своими знаками:

Это сложение рациональных чисел с разными знаками.


Вычитаем из большего модуля меньший модуль, и перед
полученным ответом ставим знак того рационального числа,
модуль которого больше:
663

Запишем решение данного примера покороче:

Пример 4. Найти значение выражения  

Заключим каждое рациональное число в скобки вместе со


своими знаками:

Вычислим данное выражение в следующем порядке: слóжим

рациональные числа   и  , затем из полученного

результата вычтем рациональное число 

Первое действие:

Второе действие:
664

Таким образом, значение выражения   равно 

Пример 5. Найти значение выражения: 

Представим целое число −1 в виде дроби , а смешанное

число   переведём в неправильную дробь:

Приведём данные дроби к общему знаменателю. После их


приведения к общему знаменателю, они примут следующий
вид:

Заключим каждое рациональное число в скобки вместе со


своими знаками:

Получили сложение рациональных чисел с разными знаками.


Вычитаем из большего модуля меньший модуль, и перед
полученным ответом ставим знак того рационального числа,
модуль которого больше:
665

Получили ответ  .

Есть и второй способ решения. Он заключается в том, чтобы


сложить отдельно целые части.

Итак, вернёмся к изначальному выражению:

Заключим каждое число в скобки. Для этого смешанное

число  временно развернём:

Вычислим целые части:

(−1) + (+2) = 1

В главном выражении вместо (−1) + (+2) запишем


полученную единицу:

Полученное выражение   свернём. Для этого запишем

единицу и дробь вместе:

Запишем решение этим способом покороче:


666

Пример 6. Найти значение выражения

Переведём смешанное число   в неправильную дробь.


Остальную часть перепишем без изменения:

Заключим каждое рациональное число в скобки вместе со


своими знаками:

Заменим вычитание сложением:

Получили сложение отрицательных рациональных чисел.


Слóжим модули этих чисел и перед полученным ответом
поставим минус:

Запишем решение данного примера покороче:


667

Пример 7. Найти значение выражение

Представим целое число −5 в виде дроби  , а смешанное

число   переведём в неправильную дробь:

Приведём данные дроби к общему знаменателю. После их


приведения к общему знаменателю, они примут следующий
вид:

Заключим каждое рациональное число в скобки вместе со


своими знаками:

Заменим вычитание сложением:

Получили сложение отрицательных рациональных чисел.


Слóжим модули этих чисел и перед полученным ответом
поставим минус:
668

Таким образом, значение выражения   равно .

Решим данный пример вторым способом. Вернемся к


изначальному выражению:

Запишем смешанное число в развёрнутом виде.


Остальное перепишем без изменений:

Заключим каждое рациональное число в скобки вместе


своими знаками:

Заменим вычитание сложением там, где это можно:

Вычислим целые части:

В главном выражении вместо запишем полученное


число −7
669

Выражение   является развёрнутой формой записи

смешанного числа .  Запишем число −7 и дробь  


вместе, образуя окончательный ответ:

Запишем это решение покороче:

Пример 8. Найти значение выражения

Переведём смешанные числа в неправильные дроби:

Заключим каждое рациональное число в скобки вместе


своими знаками:

Заменим вычитание сложением:

Получили сложение отрицательных рациональных чисел.


Слóжим модули этих чисел и перед полученным ответом
поставим минус:
670

Таким образом, значение выражения   равно 

Данный пример можно решить и вторым способом. Он


заключается в том, чтобы сложить целые и дробные части по
отдельности. Вернёмся к изначальному выражению:

Заключим каждое рациональное число в скобки вместе со


своими знаками:

Заменим вычитание сложением:

Получили сложение отрицательных рациональных чисел.


Слóжим модули этих чисел и перед полученным ответом
поставим минус. Но в этот раз слóжим по отдельности целые

части (−1 и −2), и дробные   и 


671

Запишем это решение покороче:

Пример 9. Найти выражения выражения

Переведём смешанные числа в неправильные дроби:

Заключим рациональное число   в скобки вместе своим

знаком. Рациональное число   в скобки заключать не


нужно, поскольку оно уже в скобках:
672

Приведём данные дроби в общему знаменателю. После их


приведения к общему знаменателю, они примут следующий
вид:

Получили сложение отрицательных рациональных чисел.


Слóжим модули этих чисел и перед полученным ответом
поставим минус:

Таким образом, значение выражения   равно 

Теперь попробуем решить этот же пример вторым способом,


а именно сложением целых и дробных частей по
отдельности.

В этот раз, в целях получения короткого решения, попробуем


пропустить некоторые действия, такие как: запись
смешанного числа в развёрнутом виде и замена вычитания
сложением:
673

Обратите внимание, что дробные части были приведены к


общему знаменателю.

Пример 10. Найти значение выражения

Заменим вычитание сложением:

В получившемся выражении нет отрицательных чисел,


которые являются основной причиной допущения ошибок. А
поскольку нет отрицательных чисел, мы можем убрать плюс
перед вычитаемым, а также убрать скобки:

Получилось простейшее выражение, которое вычисляется


легко. Вычислим его любым удобным для нас способом:

Пример 11. Найти значение выражения

Это сложение рациональных чисел с разными знаками.


Вычтем из большего модуля меньший модуль, и перед
полученными ответом поставим знак того рационального
числа, модуль которого больше:
674

Пример 12. Найти значение выражения

Выражение состоит из нескольких рациональных чисел.


Согласно порядку действий, в первую очередь необходимо
выполнить действия в скобках.

Сначала вычислим выражение , затем выражение

Полученные результаты слóжим .

Первое действие:

Второе действие:

Третье действие:

Ответ: значение выражения  равно 


675

Пример 13. Найти значение выражения

Переведём смешанные числа в неправильные дроби:

Заключим рациональное число  в скобки вместе со своим

знаком. Рациональное число   заключать в скобки не


нужно, поскольку оно уже в скобках:

Приведём данные дроби в общему знаменателю. После их


приведения к общему знаменателю, они примут следующий
вид:

Заменим вычитание сложением:

Получили сложение рациональных чисел с разными знаками.


Вычтем из большего модуля меньший модуль, и перед
полученными ответом поставим знак того рационального
числа, модуль которого больше:
676

Таким образом, значение выражения равно

Рассмотрим сложение и вычитание десятичных дробей,


которые тоже относятся к рациональным числам и которые
могут быть как положительными, так и отрицательными.

Пример 14. Найти значение выражения −3,2 + 4,3

Заключим каждое рациональное число в скобки вместе со


своими знаками. Учитываем, что плюс который дан в
выражении, является знаком операции и не относится к
десятичной дроби 4,3. У этой десятичной дроби свой знак
плюса, который невидим по причине того, что его не
записывают. Но мы его запишем для наглядности:

(−3,2) + (+4,3)

Это сложение рациональных чисел с разными знаками.


Чтобы сложить рациональные числа с разными знаками,
нужно из большего модуля вычесть меньший модуль, и
перед полученным ответом поставить знак того
рационального числа, модуль которого больше. А чтобы
понять какой модуль больше, а какой меньше, нужно суметь
сравнить модули этих десятичных дробей до их вычисления:

(−3,2) + (+4,3) = |+4,3| − |−3,2| = 1,1


677

Модуль числа 4,3 больше, чем модуль числа −3,2 поэтому


мы из 4,3 вычли 3,2. Получили ответ 1,1. Ответ положителен,
поскольку перед ответом должен стоять знак того
рационального числа, модуль которого больше. А модуль
числа 4,3 больше, чем модуль числа −3,2

Таким образом, значение выражения −3,2 + (+4,3) равно 1,1

Этот пример можно записать покороче:

−3,2 + (+4,3) = 1,1

Пример 15. Найти значение выражения 3,5 + (−8,3)

Это сложение рациональных чисел с разными знаками. Как и


в прошлом примере из большего модуля вычитаем меньший
и перед ответом ставим знак того рационального числа,
модуль которого больше:

3,5 + (−8,3) = −(|−8,3| − |3,5|) = −(8,3 − 3,5) = −(4,8) = −4,8

Таким образом, значение выражения 3,5 + (−8,3) равно −4,8

Этот пример можно записать покороче:

 3,5 + (−8,3) = −4,8

Пример 16. Найти значение выражения −7,2 + (−3,11)

Это сложение отрицательных рациональных чисел. Чтобы


сложить отрицательные рациональные числа, нужно сложить
их модули и перед полученным ответом поставить минус.
678

Запись с модулями можно пропустить, чтобы не


загромождать выражение:

−7,2 + (−3,11) = −7,20 + (−3,11) = −(7,20 + 3,11) = −(10,31) =


−10,31

Таким образом, значение выражения −7,2 + (−3,11) равно


−10,31

Этот пример можно записать покороче:

−7,2 + (−3,11) = −10,31

Пример 17. Найти значение выражения −0,48 + (−2,7)

Это сложение отрицательных рациональных чисел. Слóжим


их модули и перед полученным ответом поставим минус.
Запись с модулями можно пропустить, чтобы не
загромождать выражение:

−0,48 + (−2,7) = (−0,48) + (−2,70) = −(0,48 + 2,70) = −(3,18) =


−3,18

Пример 18. Найти значение выражения −4,9 − 5,9

Заключим каждое рациональное число в скобки вместе со


своими знаками. Учитываем, что минус который
располагается между рациональными числами −4,9 и 5,9
является знаком операции и не относится к числу 5,9. У этого
рационального числа свой знак плюса, который невидим по
причине того, что он не записывается. Но мы запишем его
для наглядности:
679

(−4,9) − (+5,9)

Заменим вычитание сложением:

(−4,9) + (−5,9)

Получили  сложение отрицательных рациональных чисел.


Слóжим их модули и перед полученным ответом поставим
минус:

(−4,9) + (−5,9) = −(4,9 + 5,9) = −(10,8) = −10,8

Таким образом, значение выражения −4,9 − 5,9 равно −10,8

Запишем решение этого примера покороче:

−4,9 − 5,9 = −10,8

Пример 19. Найти значение выражения 7 − 9,3

Заключим в скобки каждое число вместе со своими знаками

(+7) − (+9,3)

Заменим вычитание сложением

(+7) + (−9,3)

Получили сложение рациональных чисел с разными знаками.


Вычтем из большего модуля меньший модуль, и перед
ответом поставим знак того числа, модуль которого больше:

(+7) + (−9,3) = −(9,3 − 7) = −(2,3) = −2,3

Таким образом, значение выражения 7 − 9,3 равно −2,3


680

Запишем решение этого примера покороче:

7 − 9,3 = −2,3

Пример 20. Найти значение выражения −0,25 − (−1,2)

Заменим вычитание сложением:

−0,25 + (+1,2)

Получили сложение рациональных чисел с разными знаками.


Вычтем из большего модуля меньший модуль, и перед
ответом поставим знак того числа, модуль которого больше:

−0,25 + (+1,2) = 1,2 − 0,25 = 0,95

Запишем решение этого примера покороче:

−0,25 − (−1,2) = 0,95

Пример 21. Найти значение выражения −3,5 + (4,1 − 7,1)

Выполним действия в скобках, затем слóжим полученный


ответ с числом −3,5

Первое действие:

4,1 − 7,1 = (+4,1) − (+7,1) = (+4,1) + (−7,1) = −(7,1 − 4,1) =


−(3,0) = −3,0

Второе действие:

−3,5 + (−3,0) = −(3,5 + 3,0) = −(6,5) = −6,5

Ответ: значение выражения −3,5 + (4,1 − 7,1) равно −6,5.


681

Пример 22. Найти значение выражения (3,5 − 2,9) − (3,7 −


9,1)

Выполним действия в скобках. Затем из числа, которое


получилось в результате выполнения первых скобок, вычтем
число, которое получилось в результате выполнения вторых
скобок:

Первое действие:

3,5 − 2,9 = (+3,5) − (+2,9) = (+3,5) + (−2,9) = 3,5 − 2,9 = 0,6

Второе действие:

3,7 − 9,1 = (+3,7) − (+9,1) = (+3,7) + (−9,1) = −(9,1 − 3,7) =


−(5,4) = −5,4

Третье действие

0,6 − (−5,4) = (+0,6) + (+5,4) = 0,6 + 5,4 = 6,0 = 6

Ответ: значение выражения (3,5 − 2,9) − (3,7 − 9,1) равно 6.

Пример 23. Найти значение выражения −3,8 + 17,15 − 6,2 −


6,15

Заключим в скобки каждое рациональное число вместе со


своими знаками

(−3,8) + (+17,15) − (+6,2) − (+6,15)

Заменим вычитание сложением там, где это можно:

(−3,8) + (+17,15) + (−6,2) + (−6,15)


682

Выражение состоит из нескольких слагаемых. Согласно


сочетательному закону сложения, если выражение состоит
из нескольких слагаемых, то сумма не будет зависеть от
порядка действий. Это значит, что слагаемые можно
складывать в любом порядке.

Не будем изобретать велосипед, а слóжим все слагаемые


слева направо в порядке их следования:

Первое действие:

(−3,8) + (+17,15) = 17,15 − 3,80 = 13,35

Второе действие:

13,35 + (−6,2) = 13,35 − −6,20 = 7,15

Третье действие:

7,15 + (−6,15) = 7,15 − 6,15 = 1,00 = 1

Ответ: значение выражения −3,8 + 17,15 − 6,2 − 6,15 равно 1.

Пример 24. Найти значение выражения

Переведём десятичную дробь −1,8 в смешанное число.


Остальное перепишем без изменения:

Далее вычисляем данное выражение, применяя ранее


изученные правила:
683

Пример 25. Найти значение выражения

Заменим вычитание сложением. Попутно переведём


десятичную дробь (−4,4) в неправильную дробь

В получившемся выражении нет отрицательных чисел. А


поскольку нет отрицательных чисел, мы можем убрать плюс
перед вторым числом, и убрать скобки. Тогда получим
простое выражение на сложение, которое решается легко

Пример 26. Найти значение выражения

Переведём смешанное число в неправильную дробь, а


десятичную дробь −0,85 в обыкновенную дробь. Получим
следующее выражение:

Получили  сложение отрицательных рациональных чисел.


Слóжим их модули и перед полученным ответом поставим
минус:
684

Пример 27. Найти значение выражения

Переведём обе дроби в неправильные дроби. Чтобы


перевести десятичную дробь 2,05 в неправильную дробь,
можно перевести ее сначала в смешанное число, а затем в
неправильную дробь:

После перевода обеих дробей в неправильные дроби,


получим следующее выражение:

Получили сложение рациональных чисел с разными знаками.


Вычтем из большего модуля меньший модуль и перед
полученным ответом поставим знак того числа, модуль
которого больше:
685

Пример 28. Найти значение выражения 

Заменим вычитание сложением. Далее переведём


десятичную дробь в обыкновенную дробь. Затем вычислим
получившееся выражение, применяя ранее изученные
правила:

Пример 29. Найти значение выражения

Переведём десятичные дроби −0,25 и −1,25 в обыкновенные


дроби, остальное перепишем без изменения. Получим
следующее выражение:

Можно сначала заменить вычитание сложением там, где это


можно и сложить рациональные числа одно за другим.

Есть и второй вариант: сначала сложить рациональные

числа и , а затем из полученного результата

вычесть  . Этим вариантом и воспользуемся.

Первое действие:
686

Второе действие:

Ответ: значение выражения  равно −2.

Пример 30. Найти значение выражения

Переведём десятичные дроби в обыкновенные. Остальное


перепишем без изменения:

Получили сумму из нескольких слагаемых. Если сумма


состоит из нескольких слагаемых, то выражение можно
вычислять в любом порядке. Это следует из сочетательного
закона сложения.

Поэтому мы можем организовать наиболее удобный для нас


вариант. В первую очередь можно сложить первое и

последнее слагаемое, а именно рациональные числа   и 

. У этих чисел одинаковые знаменатели, а значит это


освободит нас от необходимости приводить их к нему.
687

Первое действие:

Полученное число можно сложить со вторым слагаемым, а

именно с рациональным числом . У рациональных

чисел и    одинаковые знаменатели в дробных


частях, что опять же является преимуществом для нас

Второе действие:

Ну и слóжим полученное число −7 с последним слагаемым, а

именно с рациональным числом . Удобно то, что при


вычислении данного выражения, семёрки исчезнут,
поскольку их сумма будет равна нулю:

Третье действие:

Ответ: значение выражения  равно

Задания для самостоятельного решения


688

Задание 1. Найдите значение выражения:

Показать решение
Задание 2. Найдите значение выражения:

Показать решение
Задание 3. Найдите значение выражения:

Показать решение
Задание 4. Найдите значение выражения:

Показать решение
Задание 5. Найдите значение выражения:

Показать решение
Задание 6. Найдите значение выражения:

Показать решение
Задание 7. Найдите значение выражения:

Показать решение
Задание 8. Найдите значение выражения:

Показать решение
Задание 9. Найдите значение выражения:
689

Показать решение
Задание 10. Найдите значение выражения:

Показать решение
690

Умножение и деление рациональных чисел

В данном уроке рассматривается умножение и деление


рациональных чисел.

Содержание урока

 Умножение рациональных чисел


 Деление рациональных чисел
 Многоэтажные дроби
 Использование переменных
 Задания для самостоятельного решения

Умножение рациональных чисел

Правила умножения целых чисел справедливы и для


рациональных чисел. Иными словами, чтобы умножать
рациональные числа, нужно уметь умножать целые числа.

Необходимо также знать основные законы умножения такие


как переместительный закон умножения, сочетательный
закон умножения, распределительный закон умножения и
закон умножения на ноль.

Пример 1. Найти значение выражения

Это умножение рациональных чисел с разными знаками.


Чтобы перемножить рациональные числа с разными
знаками, нужно перемножить их модули и перед
полученным ответом поставить минус.
691

Чтобы хорошо увидеть, что мы имеем дело с числами, у


которых разные знаки, заключим каждое рациональное число
в скобки вместе со своими знаками

Модуль числа равен , а модуль числа равен .


Перемножив полученные модули, как положительные дроби,

мы получили ответ , но перед ответом поставили минус, как


от нас требовало правило. Чтобы обеспечить перед ответом
этот минус, умножение модулей выполнялось в скобках,
перед которыми и поставлен минус.

Таким образом, значение выражения   равно

Короткое решение выглядит следующим образом:

Пример 2. Найти значение выражения 


692

Это умножение рациональных чисел с разными знаками.


Перемножим модули этих чисел и перед полученным
ответом поставим минус:

Решение для данного примера можно записать покороче:

Пример 3. Найти значение выражения

Это умножение отрицательных рациональных чисел. Чтобы


перемножить отрицательные рациональные числа,
нужно перемножить их модули и перед полученным
ответом поставить плюс

Решение для данного примера можно записать покороче:

Пример 4. Найти значение выражения


693

Это умножение отрицательных рациональных чисел.


Перемножим модули этих чисел и перед полученным
ответом поставим плюс

Решение для данного примера можно записать покороче:

Пример 5. Найти значение выражения

Это умножение рациональных чисел с разными знаками.


Перемножим модули этих чисел и перед полученным
ответом поставим минус

Короткое решение будет выглядеть значительно проще:

Пример 6. Найти значение выражения


694

Переведём смешанное число в неправильную дробь.


Остальное перепишем, как есть

Получили умножение рациональных чисел с разными


знаками. Перемножим модули этих чисел и перед
полученным ответом поставим минус. Запись с модулями
можно пропустить, чтобы не загромождать выражение

Решение для данного примера можно записать покороче

Пример 7. Найти значение выражения 

Это умножение рациональных чисел с разными знаками.


Перемножим модули этих чисел и перед полученным
ответом поставим минус
695

Сначала в ответе получилась неправильная дробь , но мы


выделили в ней цéлую часть. Обратите внимание, что целая

часть была выделена от модуля дроби . Получившееся

смешанное число было заключено в скобки, перед


которыми поставлен минус. Это сделано для того, чтобы
выполнялось требование правила. А правило требовало,
чтобы перед полученным ответом стоял минус.

Решение для данного примера можно записать покороче:

Пример 8. Найти значение выражения


696

Выражение состоит из нескольких сомножителей. Согласно


сочетательному закону умножения, если выражение состоит
из нескольких сомножителей, то произведение не будет
зависеть от порядка действий. Это позволяет нам вычислить
данное выражение в любом порядке.

Сначала перемножим    и и полученное число


перемножим с оставшимся числом 5. Запись с модулями
пропустим, чтобы не загромождать выражение.

Первое действие:

Второе действие:

Ответ: значение выражения равно −2.

Пример 9. Найти значение выражения:

Переведём смешанные числа в неправильные дроби:

Получили умножение отрицательных рациональных чисел.


Перемножим модули этих чисел и перед полученным
697

ответом поставим плюс. Запись с модулями можно


пропустить, чтобы не загромождать выражение

Пример 10. Найти значение выражения

Выражение состоит из нескольких сомножителей. Согласно


сочетательному закону умножения, если выражение состоит
из нескольких сомножителей, то произведение не будет
зависеть от порядка действий. Это позволяет нам вычислить
данное выражение в любом порядке.

Не будем изобретать велосипед, а вычислим данное


выражение слева направо в порядке следования
сомножителей. Запись с модулями пропустим, чтобы не
загромождать выражение

Первое действие:

Второе действие:

Третье действие:
698

Четвёртое действие:

Ответ: значение выражения

равно

Пример 11. Найти значение выражения

Вспоминаем закон умножения на ноль. Этот закон гласит, что


произведение равно нулю, если хотя бы один из
сомножителей равен нулю.

В нашем примере один из сомножителей равен нулю,


поэтому не теряя времени отвечаем, что значение

выражения равно нулю:

Пример 12. Найти значение выражения

Произведение равно нулю, если хотя бы один из


сомножителей равен нулю.
699

В нашем примере один из сомножителей равен нулю,


поэтому не теряя времени отвечаем, что значение

выражения равно нулю:

Пример 13. Найти значение выражения

Можно воспользоваться порядком действий и сначала

вычислить выражение в скобках и полученный ответ

перемножить с дробью .

Ещё можно воспользоваться распределительным законом

умножения — умножить каждое слагаемое суммы на

дробь и полученные результаты сложить. Этим способом и


воспользуемся.

Согласно порядку действий, если в выражении присутствует


сложение и умножение, то в первую очередь нужно
выполнять умножение. Поэтому в получившемся новом
выражении возьмём в скобки те дроби, которые должны быть
перемножены. Так мы хорошо увидим, какие действия
выполнить раньше, а какие позже:
700

Далее вычисляем выражение по действиям. Сначала


вычислим выражения в скобках, и полученные результаты
сложим

Первое действие:

Второе действие:

Третье действие:

Ответ: значение выражения   равно 

Решение для данного примера можно записать значительно


короче. Выглядеть оно будет следующим образом:

Видно, что данный пример можно было решить даже в уме.


Поэтому следует развивать в себе навык анализа выражения
до начала его решения. Вполне вероятно, что его можно
решить в уме и сэкономить много времени и нервов. А на
контрольных и экзаменах, как известно время очень дорого
стоит.
701

Пример 14. Найти значение выражения −4,2 × 3,2

Это умножение рациональных чисел с разными знаками.


Перемножим модули этих чисел и перед полученным
ответом поставим минус

Обратите внимание, как перемножались модули


рациональных чисел. В данном случае, чтобы перемножить
модули рациональных чисел, потребовалось суметь
перемножить десятичные дроби.

Пример 15. Найти значение выражения −0,15 × 4

Это умножение рациональных чисел с разными знаками.


Перемножим модули этих чисел и перед полученным
ответом поставим минус
702

Обратите внимание, как перемножались модули


рациональных чисел. В данном случае, чтобы перемножить
модули рациональных чисел, потребовалось суметь
перемножить десятичную дробь и целое число.

Пример 16. Найти значение выражения −4,2 × (−7,5)

Это умножение отрицательных рациональных чисел.


Перемножим модули этих чисел и перед полученным
ответом поставим плюс

Деление рациональных чисел

Деление рациональных чисел свóдится к умножению этих же


чисел. Для этого первую дробь умножают на дробь,
703

обратную второй. Затем применяются правила умножения


рациональных чисел.

Пример 1. Выполнить деление:

Умнóжим первую дробь на дробь, обратную второй.

Обратная для второй дроби это дробь . На неё и умножим


первую дробь:

Получили умножение рациональных чисел с разными


знаками. А как вычислять такие выражения мы уже знаем.
Для этого нужно перемножить модули данных рациональных
чисел и перед полученным ответом поставить минус.

Дорешаем данный пример до конца. Запись с модулями


можно пропустить, чтобы не загромождать выражение

Таким образом, значение выражения   равно 

Подробное решение выглядит следующим образом:

Короткое решение можно записать так:


704

Пример 2. Выполнить деление 

Умнóжим первую дробь на дробь, обратную второй.

Обратная для второй дроби это дробь . На неё и


умножим первую дробь:

Короткое решение можно записать так:

Пример 3. Выполнить деление

Умнóжим первую дробь на дробь, обратную второй.

Обратная для второй дроби это дробь

Получили умножение отрицательных рациональных чисел.


Выполним данное умножение. Запись с модулями можно
пропустить, чтобы не загромождать выражение:
705

Пример 4. Выполнить деление

В данном случае нужно первое число −3 умножить на дробь,

обратную дроби .

Обратная для дроби это дробь . Затем следует применить


правило умножения рациональных чисел с разными знаками:

Пример 6. Выполнить деление

Умнóжим первую дробь на число, обратное числу 4.

Обратное числу 4 это дробь . На неё и умножим первую

дробь

Пример 5. Выполнить деление

Умнóжим первую дробь на число, обратное числу −3

Обратное числу −3 это дробь


706

Пример 6. Найти значение выражение −14,4 : 1,8

Это деление рациональных чисел с разными знаками. Чтобы


вычислить данное выражение, нужно модуль делимого
разделить на модуль делителя и перед полученным ответом
поставить минус

Обратите внимание, как модуль делимого был разделён на


модуль делителя. В данном случае потребовалось суметь
разделить десятичную дробь на другую десятичную дробь.

Если нет желания работать с десятичными дробями (а это


бывает часто), то эти десятичные дроби можно перевести в
смешанные числа, затем перевести эти смешанные числа в
неправильные дроби, а затем заняться непосредственно
делением.

Вычислим предыдущее выражение −14,4 : 1,8 этим


способом. Переведём десятичные дроби в смешанные
числа:
707

Теперь переведём полученные смешанные числа в


неправильные дроби:

Теперь можно заняться непосредственно делением, а

именно разделить дробь    на дробь  . Для этого нужно


первую дробь умножить на дробь, обратную второй:

Пример 7. Найти значение выражения

Переведём десятичную дробь −2,06 в неправильную дробь, и


умножим эту дробь на дробь, обратную второй:0
708

Пример 8. Найти значение выражения −7,2 : (−0,6)

Это деление отрицательных рациональных чисел. Чтобы


выполнить данное деление, нужно первую дробь умножить
на дробь обратную второй.

Перенесём в обеих дробях запятую на одну цифру вправо,


получим деление −72:(−6)

Многоэтажные дроби

Часто можно встретить выражение, в котором деление


дробей записано с помощью дробной черты. Например,

выражение   может быть записано следующим


образом:

В чём же разница между выражениями   и   ? На


самом деле разницы никакой. Эти два выражения несут одно
и то же значение и между ними можно поставить знак
равенства:
709

В первом случае знак деления представляет собой


двоеточие и выражение записано в одну строку. Во втором
случае деление дробей записано с помощью дробной черты.
В результате получается дробь, которую в народе
договорились называть многоэтажной.

При встрече с такими многоэтажными выражениями, нужно


применять те же правила деления обыкновенных дробей.
Первую дробь необходимо умножать на дробь, обратную
второй.

Использовать в решении подобные дроби крайне неудобно,


поэтому можно записать их в понятном виде, используя в
качестве знака деления не дробную черту, а двоеточие.

Например, запишем многоэтажную дробь в понятном


виде. Для этого сначала нужно разобраться, где первая
дробь и где вторая, потому что сделать это правильно
удаётся не всегда. В многоэтажных дробях имеется
несколько дробных черт, которые могут запутать. Главная
дробная черта, которая отделяет первую дробь от второй,
обычно бывает длиннее остальных.

После определения главной дробной черты можно без труда


понять, где первая дробь и где вторая:
710

И далее можно воспользоваться методом деления дробей —


умножить первую дробь на дробь, обратную второй.

Пример 2. Запишем в понятном виде многоэтажную дробь

Находим главную дробную черту (она самая длинная) и


видим, что осуществляется деление целого числа −3 на

обыкновенную дробь

А если бы мы по ошибке приняли вторую дробную черту за


главную (ту, что короче), то получилось бы, что мы делим
711

дробь   на целое число 5 В этом случае, даже если


это выражение вычислить верно, задача будет решена
неправильно, поскольку делимым в данном случае является

число −3, а делителем — дробь .

Пример 3. Запишем в понятном виде многоэтажную дробь

Находим главную дробную черту (она самая длинная) и

видим, что осуществляется деление дроби на целое число


2

А если бы мы по ошибке приняли первую дробную черту за


главную (ту, что короче), то получилось бы, что мы делим

целое число −5 на дробь В этом случае, даже если это


выражение вычислить верно, задача будет решена
неправильно, поскольку делимым в данном случае является

дробь , а делителем — целое число 2.


712

Несмотря на то, что многоэтажные дроби неудобны в работе,


сталкиваться мы с ними будем очень часто, особенно при
изучении высшей математики.

Естественно, на перевод многоэтажной дроби в понятный


вид уходит дополнительное время и место. Поэтому можно
воспользоваться более быстрым методом. Данный метод
удобен и на выходе позволяет получить готовое выражение,
в котором первая дробь уже умножена на дробь, обратную
второй.

Реализуется этот метод следующим образом:

Если дробь четырехэтажная, например как  , то цифру


находящуюся на первом этаже поднимают на самый верхний
этаж. А цифру, находящуюся на втором этаже поднимают на
третий этаж. Полученные цифры нужно соединить значками
умножения ( × )

В результате, минуя промежуточную запись мы

получаем новое выражение  , в котором первая


дробь уже умножена на дробь, обратную второй. Удобство да
и только!
713

Чтобы не допускать ошибок при использовании данного


метода, можно руководствоваться следующим правилом:

С первого на четвёртый. Со второго на третий.

В правиле речь идет об этажах. Цифру с первого этажа


нужно поднимать на четвертый этаж. А цифру со второго
этажа нужно поднимать на третий этаж.

Попробуем вычислить многоэтажную дробь  пользуясь


вышеприведённым правилом.

Итак, цифру находящуюся на первом этаже поднимаем на


четвёртый этаж, а цифру находящуюся на втором этаже
поднимаем на третий этаж

В результате, минуя промежуточную запись мы

получаем новое выражение , в котором первая дробь уже


умножена на дробь, обратной второй. Далее можно
воспользоваться имеющимися знаниями:
714

Попробуем вычислить многоэтажную дробь пользуясь


новой схемой.

Здесь имеется только первый, второй и четвёртый этажи.


Третий этаж отсутствует. Но мы не отходим от основной
схемы: цифру с первого этажа поднимаем на четвёртый
этаж. А поскольку третий этаж отсутствует, то цифру
находящуюся на втором этаже оставляем, как есть

В результате, минуя промежуточную запись  мы

получили новое выражение   , в котором первое число


−3 уже умножено на дробь, обратную второй. Далее можно
воспользоваться имеющимися знаниями:
715

Попробуем вычислить многоэтажную дробь , пользуясь


новой схемой.

Здесь имеется только второй, третий и четвёртый этажи.


Первый этаж отсутствует. Поскольку первый этаж
отсутствует, подниматься на четвёртый этаж нечему, но зато
мы можем поднять цифру со второго этажа на третий:

В результате, минуя промежуточную запись  мы

получили новое выражение   , в котором первая дробь


уже умножена на число, обратное делителю. Далее можно
воспользоваться имеющимися знаниями:
716

Использование переменных

Если выражение сложное и вам кажется, что оно запутает


вас в процессе решения задачи, то часть выражения можно
занести в переменную и далее работать с этой переменной.

Математики часто так и делают. Сложную задачу разбивают


на более лёгкие подзадачи и решают их. Затем собирают
решённые подзадачи в одно единое целое. Это творческий
процесс и этому учатся годами, упорно тренируясь.

Использование переменных оправдано, при работе с


многоэтажными дробями. Например:

Найти значение выражения 

Итак, имеется дробное выражение в числителе и в


знаменателе котором дробные выражения. Другими словами,
перед нами снова многоэтажная дробь, которую мы так не
любим.

Выражение, находящееся в числителе  можно занести в


переменную с любым названием, например:

Но в математике в подобном случае переменным принято


давать название из больших латинских букв. Давайте не
будем нарушать эту традицию, и обозначим первое
выражение через большую латинскую букву A
717

А выражение, находящееся в знаменателе    можно


обозначить через большую латинскую букву B

Теперь наше изначальное выражение    принимает вид 

. То есть мы сделали замену числового выражения на


буквенное, предварительно занеся числитель и знаменатель
в переменные A и B.

Теперь мы можем отдельно вычислить значения переменной


A и значение переменной B. Готовые значения мы вставим в

выражение  .

Найдём значение переменной A

Найдём значение переменной B


718

Теперь подставим в главное выражения    вместо


переменных A и B их значения:

Мы получили многоэтажную дробь в которой можно


воспользоваться схемой «с первого на четвёртый, со второго
на третий», то есть цифру находящуюся на первом
этаже поднять на четвёртый этаж, а цифру находящуюся на
втором этаже поднять на третий этаж. Дальнейшее
вычисление не составит особого труда:

Таким образом, значение выражения    равно −1.

Конечно, мы рассмотрели простейший пример, но нашей


целью было узнать, как можно использовать переменные для
облегчения себе задачи, чтобы свести к минимуму
допущение ошибок.

Отметим также, что решение для данного примера можно


записать не применяя переменные. Выглядеть оно будет как
719

Это решение более быстрое и короткое и в данном случае


его целесообразнее так и записать, но если выражение
окажется сложным, состоящим из нескольких параметров,
скобок, корней и степеней, то желательно вычислять его в
несколько этапов, занося часть его выражений в
переменные.

Задания для самостоятельного решения


Задание 1. Выполните умножение:

Показать решение
Задание 2. Выполните умножение:

Показать решение
Задание 3. Выполните умножение:

Показать решение
Задание 4. Выполните умножение:

Показать решение
Задание 5. Выполните умножение:
720

Показать решение
Задание 6. Выполните умножение:

Показать решение
Задание 7. Выполните умножение:

Показать решение
Задание 8. Выполните умножение:

Показать решение
Задание 9. Выполните деление:

Показать решение
Задание 10. Выполните деление:

Показать решение
Задание 11. Выполните деление:

Показать решение
721

Задание 12. Выполните деление:

Показать решение
Задание 13. Выполните деление:

Показать решение
Задание 14. Выполните деление:

Показать решение
Задание 15. Выполните деление:

Показать решение
Задание 16. Выполните деление:

Показать решение
Задание 17. Выполните деление:

Показать решение
Задание 18. Вычислите выражение:
722

Показать решение
Задание 19. Вычислите выражение:

Показать решение
Задание 20. Вычислите выражение:

Показать решение
Задание 21. Вычислите выражение:

Показать решение
Задание 22. Вычислите выражение:

Показать решение
Задание 23. Запишите следующую многоэтажную дробь с
помощью двоеточия и вычислите:

Показать решение
Задание 24. Запишите следующую многоэтажную дробь с
помощью двоеточия и вычислите:
723

Показать решение
Задание 25. Запишите следующую многоэтажную дробь с
помощью двоеточия и вычислите:

Показать решение
Задание 26. Используя метод «С первого на четвёртый, со
второго на третий», запишите следующую дробь в виде
умножения и вычислите:

Показать решение
Задание 27. Используя метод «С первого на четвёртый, со
второго на третий», запишите следующую дробь в виде
умножения и вычислите:

Показать решение
Задание 28. Используя метод «С первого на четвёртый, со
второго на третий», запишите следующую дробь в виде
умножения и вычислите:
724

Показать решение
Задание 29. Используя метод «С первого на четвёртый, со
второго на третий», запишите следующую дробь в виде
умножения и вычислите:

Показать решение
Задание 30. Найдите значение выражения:

Показать решение
Задание 31. Найдите значение выражения:

Показать решение
Задание 32. Найдите значение выражения:
725

Показать решение
Задание 33. Найдите значение выражения:

Показать решение
726

Дополнительные сведения о дробях

В этом уроке мы коснёмся тех моментов, о которых не


упоминали при изучении дробей, посчитав что на первых
порах они создают трудности для обучения.

Содержание урока

 Правильные и неправильные дроби


 Приведение дробей к общему знаменателю
 Нахождение дроби от числа
 Нахождение числа по дроби
 Десятичная точка в дробях
 Составные выражения

Правильные и неправильные дроби

В самом начале своего пути при изучении дробей мы узнали,


что правильная дробь — это та дробь, у которой числитель
меньше знаменателя.

В школьной литературе можно встретить другое


определение правильной дроби. Выглядит оно следующим
образом:

Правильная дробь всегда меньше единицы.

Как понять данное определение? Дробь сама по себе


указывает на то, что какой-либо объект разделен на
несколько частей. И это всегда один единственный объект.
Под единицей именно это и подразумевается.

Например, пусть у нас имеется одна пицца:


727

В данном случае она и является единицей.

Если мы отрежем от этой пиццы половину, то есть (одну


вторую пиццы), то наш кусок будет меньше, чем вся целая
пицца:

В этом и заключается суть фразы «правильная дробь всегда


меньше единицы».
728

Наша половинка пиццы является дробью   и она меньше


одной целой пиццы, то есть меньше единицы:

Это выражение можно доказать. Если мы вычислим дробь ,


то получим десятичную дробь 0,5. А это рациональное число
меньше единицы:

На координатной прямой можно увидеть, как располагаются


эти числа:

Видно, что рациональное число 0,5 располагается левее,


чем 1. А мы помним, что чем левее число располагается на
координатной прямой, тем оно меньше.

С неправильными дробями всё было наоборот.


Неправильной дробью мы назвали ту дробь, у которой
числитель больше знаменателя.

Но в школьной литературе можно встретить другое


определение неправильной дроби. Выглядит оно следующим
образом:

Неправильная дробь всегда больше единицы или равна ей.


729

Например, рассмотрим неправильную дробь . Выделим в

этой дроби целую часть, получим . Изобразим эту


смешанную дробь в виде одной целой пиццы и ещё
половинки пиццы:

Вместе одна целая пицца и ещё половина пиццы больше,


чем просто одна целая пицца
730

В этом и заключается суть фразы «неправильная дробь


всегда больше единицы».

Одна целая пицца и ещё половина пиццы описывается

смешанной дробью и эта смешанная дробь больше


единицы:

Переведём смешанную дробь обратно в неправильную


дробь, чтобы не противоречить правилу. Ведь речь в данном
случае идёт о неправильных дробях:

что схематически будет выглядеть так:


731

Выражение можно доказать. Если мы вычислим дробь


, то получим десятичную дробь 1,5. А это рациональное
число больше единицы:

На координатной прямой можно увидеть, как располагаются


эти числа:

Видно, что рациональное число 1,5 располагается правее,


чем 1. А мы помним, что чем правее число располагается на
координатной прямой, тем оно больше.

Неправильной также называется дробь равная единице.


Речь в данном случае идет о тех дробях, у которых
числитель и знаменатель равны.
732

Рассмотрим дробь . Изобразим её в виде двух одинаковых


кусочков пиццы:

Фактически речь идёт не о дроби, а об одной целой пицце:


733

В этом и заключается суть фразы «неправильная дробь


может равняться единице».

Любое целое число отличное от нуля (не равное нулю)


можно представить в виде неправильной дроби со
знаменателем 1. Например, числа 3, 5, 9, 12 можно
представить в виде неправильных дробей со знаменателем 1

Представление объекта в виде единицы позволяет проще


решать задачи. Рассмотрим примеры.

Пример 1. Куплен один шоколадный батончик. От него


отрезали треть. Сколько батончика осталось?

Осталось две трети батончика. Сам батончик можно описать


цифрой 1, далее из этой единицы вычесть треть:
734

Не приводя на бумаге никаких вычислений, можно ответить


на вопрос подобной задачи. Сказано «отрезали треть» —
значит сразу нужно обратить внимание на то, что
знаменатель равен 3.

Если отрезали одну часть из трёх, то сколько частей должно


остаться? Верно, две части. Поэтому и ответ «две части из
трёх» или «две трети».

Пример 2. Куплен один пирог. От него отрезали две шестых.


Сколько пирога осталось?

Осталось четыре шестых пирога. Сам пирог можно описать


цифрой 1, далее из этой единицы вычесть две шестых:
735

Приведение дробей к общему знаменателю

Чтобы привести дроби к общему знаменателю, мы находили


НОК (наименьшее общее кратное) знаменателей этих
дробей. Затем делили найденный НОК на знаменатель
первой дроби и получали  дополнительный множитель для
первой дроби.

То же самое мы делали и для второй дроби — делили НОК


на знаменатель второй дроби и получали дополнительный
множитель для второй дроби.

Затем дроби умножались на свои дополнительные


множители. В результате они обращались в дроби, у которых

одинаковые знаменатели. К примеру, выражение   


вычисляется следующим образом:
736

Но есть и другой способ приведения дробей к общему


знаменателю. Этим способом часто пользуются школьники
и ленивые студенты. Суть этого способа заключается в том,
что роль дополнительных множителей берут на себя
знаменатели обеих дробей, причем происходит это «крест-
накрест» — знаменатель первой дроби становится
дополнительным множителем второй дроби, а знаменатель
второй дроби становится дополнительным множителем
первой дроби.

Вычислим предыдущее выражение этим способом.


Знаменатель первой дроби 2 становится дополнительным
множителем второй дроби, а знаменатель второй дроби 6
становится дополнительным множителем первой дроби:

Далее числитель и знаменатель каждой дроби умножаем на


свой дополнительный множитель и вычисляем:

Преимущество данного способа в том, что не нужно находить


НОК знаменателей обеих дробей. В процессе вычисления
всё выравнивается само. Единственный недостаток
737

заключается в том, что выражение становится более


длинным и корявым.

Сравните выражения, которые мы вычислили сначала


первым способом, а затем вторым:

Выражение, вычисленное первым способом, намного


аккуратнее и короче, нежели второе.

Вторым способом мы будем пользоваться при изучении


алгебры. В алгебре работать с буквенными выражениями
приходиться чаще, чем с числовыми.

К примеру, если перед нами будет стоять задача привести

буквенное выражение    к общему знаменателю, то у нас


не будет другого выхода, кроме как воспользоваться
методом «крест-накрест», то есть использовать второй
способ, который мы сейчас рассмотрели:
738

Нахождение дроби от числа

Чтобы найти дробь от числа, мы делим это число на


знаменатель искомой дроби и полученный результат
умножаем на числитель искомой дроби.

Например, чтобы найти    от 10 сантиметров, нужно 10


разделить на 5, и полученный результат умножить на 2

10 : 5 = 2

2×2=4

Получили ответ 4. Значит от десяти сантиметров


составляют 4 сантиметра. Схематически это выглядит
примерно так:
739

Но есть и второй вариант решения. Для нахождения от

десяти сантиметров, достаточно умножить 10 на . Тогда мы


получим тот же результат, как и в прошлый раз, но получим
мы его в одно действие:

Поэтому можно взять на заметку следующее правило


нахождения дроби от числа:

Чтобы найти дробь от числа, нужно это число умножить на


искомую дробь.

Пример 2. Найти от двух часов.

Два часа это 120 минут. Чтобы найти от 120 минут, нужно

120 умножить на дробь

Значит от двух часов составляют 80 минут.

Нахождение числа по дроби

Чтобы найти всё число по его дроби, мы делили это число на


числитель имеющейся дроби и полученный результат
умножали на знаменатель имеющейся дроби.
740

Например, зная что  рулетки составляет 12 см, мы можем


найти длину всей рулетки. Для этого 12 нужно разделить на
2, и полученный результат умножить на 3

12 : 2 = 6

6 × 3 = 18

Получили 18. Значит длина всей рулетки равна 18 см.

Но есть и второй вариант решения. Для нахождения длины

всей рулетки, достаточно 12 разделить на дробь .  Тогда мы


получим тот же результат, как и в прошлый раз, но получим
мы его в одно действие:

Поэтому можно взять на заметку следующее правило


нахождения числа по дроби:

Чтобы найти число по дроби, нужно это число разделить на


данную дробь.

Пример 2.    всего пути составляет 6 км. Найти длину всего


пути.

Чтобы найти длину всего пути, достаточно 6 разделить на

дробь
741

Получили ответ 15. Значит длина всего пути составляет 15


километров.

Десятичная точка в дробях

Запятую в десятичной дроби, которая отделяет целую часть


от дробной, по-другому называют десятичной точкой.

Дело в том, что в некоторых источниках целая часть от


дробной отделяется именно точкой, а не запятой. Например:

2.5 (две целых пять десятых)

15.65 (пятнадцать целых шестьдесят пять сотых)

Точка часто используется для записи десятичных дробей на


компьютере — в программировании и при работе в
математических пакетах. В остальных случаях: на письме и
при подготовке документов, в десятичных дробях чаще
используется запятая, а не точка.

Мы используем в десятичных дробях запятую, а не точку,


поэтому разумнее называть эту запятую десятичной
запятой.

Но десятичную запятую большинство людей тоже называют


десятичной точкой. Что в принципе не является ошибкой,
потому как речь всё равно идёт о разделителе, котором
отделяет целую часть от дробной.
742

Давайте и мы будем называть свою запятую в десятичных


дробях десятичной точкой. Это словосочетание
проговаривается легче и приятнее на слух.

Десятичная точка используется для увеличения или


уменьшения дроби в 10, 100, 1000 и более раз. При
увеличении десятичной дроби, десятичная точка
передвигается вправо, а при уменьшении — влево. Чтобы
быстро запомнить это, можно воспользоваться фразами
«чем правее, тем больше» и «чем левее, тем меньше».

Пример 1. Увеличить десятичную дробь 6,3 в десять раз.

Чтобы увеличить десятичную дробь 6,3 в десять раз,


достаточно передвинуть десятичную точку вправо на одну
цифру, получим 63.

Пример 2. Уменьшить десятичную дробь 6,3 в десять раз.

Для уменьшения дроби 6,3 в десять раз достаточно


передвинуть десятичную точку влево на одну цифру,
получим 0,63

На вопрос «как узнать на сколько цифр передвигать


десятичную точку?», нужно смотреть во сколько
увеличивается (или уменьшается) десятичная дробь. Если
дробь нужно увеличить (или уменьшить) в десять раз, то
десятичная точка сдвигается на одну цифру.

Если дробь нужно увеличить (или уменьшить) в сто раз, то


десятичная точка сдвигается на две цифры.
743

Если дробь нужно увеличить (или уменьшить) в тысячу раз,


то десятичная точка сдвигается на три цифры. В общем, всё
зависит от количества нулей во множителе.

Например, увеличить дробь в десять раз означает умножить


её на 10. Мы помним, что для того чтобы умножить
десятичную дробь на 10, нужно в этой дроби передвинуть
запятую вправо на одну цифру (поскольку в числе 10 один
ноль). Теперь можно не заучивать подобные правила. Такое
умножение можно легко выполнить, передвинув десятичную
точку.

Пример 3. Увеличить десятичную дробь 6,3 в тысячу раз.

Чтобы увеличить десятичную дробь 6,3 в тысячу раз,


достаточно передвинуть десятичную точку вправо на три
цифры, получим 6300. Если после запятой не хватает цифр,
то вместо недостающих цифр записывают нули, что мы и
сделали.

Пример 4. Уменьшить десятичную дробь 12,5 в сто раз.

Для уменьшения дроби 12,5 в сто раз, достаточно


передвинуть десятичную точку влево на две цифры, получим
0,125

Десятичную точку можно использовать не только в


десятичных дробях. Её можно использовать для увеличения
(уменьшения) и других чисел в 10, 100 или в 1000 раз.
744

Возьмём к примеру целое число 325 и поставим в конце


точку, получим 325 с точкой. Воспользуемся в этот раз
точкой, так как её легче изобразить на рисунке:

Попробуем уменьшить это число в десять раз. Для этого


достаточно будет передвинуть точку влево на одну цифру,
получим 32.5

Попробуем увеличить число 123 в тысячу раз. Для этого


достаточно передвинуть десятичную точку на три цифры
вправо, получим 123000.

Попробуем уменьшить число 123 в тысячу раз. Для этого


достаточно передвинуть десятичную точку на три цифры
влево, получим 0,123

Попробуем уменьшить число 65 в тысячу раз. Для этого


достаточно передвинуть десятичную точку на три цифры
влево, получим 0,065
745

Попробуем увеличить число 65 в сто раз. Для этого


достаточно передвинуть десятичную точку на две цифры
вправо, получим 6500.

Составные выражения

Встречаются задачи, в которых требуется вычислить


выражение составленное из нескольких дробей. Например,

Такое выражение вычисляется согласно порядку действий. В


данном случае вычисление будет выполнено
последовательно слева направо:

Если из   пиццы вычесть   пиццы, затем прибавить   пиццы,

затем вычесть   пиццы, то останется   пиццы


746

Если вам тяжело понять данный пример, попробуйте


самостоятельно решить его на бумаге, делая
соответствующие рисунки к каждой дроби.

Пример 2. Найти значение выражения 

В данном примере сначала необходимо выполнить


умножение затем сложение и вычитание

Если  пиццы увеличить в два раза, то получится одна целая


пицца

Затем если к   пиццы прибавить эту целую пиццу, а затем из

полученного результата вычесть   пиццы, то получится   


пиццы
747

Пример 3. Найти значение выражения 

Сначала желательно вычислить выражения, находящиеся в


числителях обеих дробей, а именно выражения 2−1 и 1+1,

Дальнейшее вычисление не составляет особого труда   

плюс   равно 

Конечно, можно было записать в одном числителе


выражения, находящиеся в числителях обеих дробях. От
этого ответ не изменился бы:
748

Но в некоторых случаях возможны подвохи, особенно если


из одной дроби вычитается другая. Следующий пример
демонстрирует это.

Пример 4. Найти значение выражения 

Вычислим выражения, находящиеся в числителях обеих


дробей, а именно выражения 2+1 и 2−1

Ну и нетрудно догадаться, что   равно   или   (при

условии, что дробь  будет сокращена на 2)

Все логично. Если из   пиццы вычесть   пиццы, то

получится   пиццы.
749

Теперь попробуем решить данный пример, записав в одном


числителе оба выражения, находящиеся в числителях обеих
дробей:

Получается совсем другой ответ. Этот ответ не является


правильным. Давайте посмотрим, что представляет собой

выражение   .

Для начала запишем его следующим образом:

Теперь попробуем проследить весь процесс вычисления

этого выражения. Предположим, что имелось   пиццы

К ней добавили еще   пиццы


750

Затем из получившейся   пиццы вычитается

Затем из получавшейся   пиццы вычитают еще   пиццы


751

Получился 0, то есть пицца исчезла. Но мы знаем, что

должно было остаться   пиццы. Поэтому при вычислении


дробных выражений следует быть внимательным, особенно
при вычитании выражений, содержащих в числителе другие
выражения.

Если хочется сэкономить время и записать в числителе оба


выражения, находящиеся в числителях обеих дробей, то
второй числитель нужно взять в скобки. Это спасёт от
ошибки:

Пример 5. Найти выражения 

Вычислим выражения, находящиеся в числителях обеих


дробей:
752

Приведем полученные дроби к общему знаменателю и как


обычно вычислим полученное выражение:

Если из   вычесть   пиццы, то получится   пиццы

Пример 6. Найти значение выражения 

В первую очередь необходимо выполнить умножение:

Далее выполняется сложение:


753
754

Буквенные выражения

Буквенное выражение (или выражение с переменными) —


это математическое выражение, которое состоит из чисел,
букв и знаков математических операций. Например,
следующее выражение является буквенным:

a+b+4

С помощью буквенных выражений можно записывать законы,


формулы, уравнения и функции. Умение манипулировать
буквенными выражениями — залог хорошего знания алгебры
и высшей математики.

Любая серьёзная задача в математике свóдится к решению


уравнений. А чтобы уметь решать уравнения, нужно уметь
работать с буквенными выражениями.

Чтобы работать с буквенными выражениями, нужно хорошо


изучить базовую арифметику: сложение, вычитание,
умножение, деление, основные законы математики, дроби,
действия с дробями, пропорции. И не просто изучить, а
понять досконально.

Содержание урока

 Переменные
 Коэффициенты
 Как определить коэффициент
 Слагаемые в буквенных выражениях
 Подобные слагаемые
 Упрощение выражений
 Тождества. Тождественно равные выражения
755

 Задания для самостоятельного решения.

Переменные

Буквы, которые содержатся в буквенных выражениях,


называются переменными.

Например, в выражении a + b + 4 переменными являются


буквы a и b. Если вместо этих переменных подставить любые
числа, то буквенное выражение a + b + 4 обратится в
числовое выражение, значение которого можно будет найти.

Числа, которые подставляют вместо переменных называют


значениями переменных. Например, изменим значения
переменных a и b. Для изменения значений используется
знак равенства

a = 2, b = 3

Мы изменили значения переменных a и b. Переменной a


присвоили значение 2, переменной b присвоили значение 3.
В результате буквенное выражение a + b + 4 обращается в
обычное числовое выражение 2 + 3 + 4, значение которого
можно найти:

2+3+4=9

Когда происходит умножение переменных, то они


записываются вместе. Например, запись ab означает то же
самое, что и запись a × b. Если подставить вместо
переменных a и b числа 2 и 3, то мы получим 6

2×3=6
756

Слитно также можно записать умножение числа на


выражение в скобках. Например, вместо a × (b + c) можно
записать a(b + c). Применив распределительный закон
умножения, получим a(b + c) = ab + ac.

Коэффициенты

В буквенных выражениях часто можно встретить запись, в


которой число и переменная записаны вместе, например 3a.
На самом деле это короткая запись умножения числа 3 на
переменную a и эта запись выглядит как 3 × a.

Другими словами, выражение 3a является произведением


числа 3 и переменной a. Число 3 в этом произведении
называют коэффициентом. Этот коэффициент показывает
во сколько раз будет увеличена переменная a. Данное
выражение можно прочитать как «a три раза» или «трижды
а«, или «увеличить значение переменной a в три раза», но
наиболее часто читается как «три a«

К примеру, если переменная a равна 5, то значение


выражения 3a будет равно 15.

3 × 5 = 15

Говоря простым языком, коэффициент это число, которое


стоит перед буквой (перед переменной).

Букв может быть несколько, например 5abc. Здесь


коэффициентом является число 5. Данный коэффициент
показывает, что произведение переменных abc
увеличивается в пять раз. Это выражение можно прочитать
757

как «abc пять раз» либо «увеличить значение выражения abc


в пять раз», либо «пять abc«.

Если вместо переменных abc подставить числа 2, 3 и 4, то


значение выражения 5abc будет равно 120

5 × 2 × 3 × 4 = 120

Можно мысленно представить, как сначала перемнóжились


числа 2, 3 и 4, и полученное значение увеличилось в пять
раз:

Знак коэффициента отнóсится только к коэффициенту, и


не отнóсится к переменным!

Рассмотрим выражение −6b. Минус, стоящий перед


коэффициентом 6, отнóсится только к коэффициенту 6, и не
отнóсится к переменной b. Понимание этого факта позвóлит
не ошибаться в будущем со знаками.

Найдем значение выражения −6b при b = 3.

−6b это короткая форма записи от −6 × b. Для наглядности


запишем выражение −6b в развёрнутом виде и подставим
значение переменной b
758

−6b = −6 × b = −6 × 3 = −18

Пример 2. Найти значение выражения −6b при b = −5

Запишем выражение −6b в развёрнутом виде

−6b = −6 × b

и далее подставим значение переменной b

−6b = −6 × b = −6 × (−5) = 30

Пример 3. Найти значение выражения −5a + b при a = 3 и b


=2

−5a + b это короткая форма записи от −5 × a + b, поэтому


для наглядности запишем выражение −5 × a + b в
развёрнутом виде и подстáвим значения переменных a и b

−5a + b = −5 × a + b = −5 × 3 + 2 = −15 + 2 = −13

Иногда буквы записаны без коэффициента, например a или


ab. В этом случае коэффициентом является единица:

1a, 1ab

но единицу по традиции не записывают, поэтому просто


пишут a или ab

Если перед буквой стоит минус, то коэффициентом является


число −1. Например, выражение −a на самом деле выглядит
759

как −1a. Это произведение минус единицы и переменной a.


Оно получилось следующим образом:

−1 × a = −1a

Здесь крóется небольшой подвох. В выражении −a минус,


стоящий перед переменной a на самом деле относится к
невидимой единице, а не к переменной a. Поэтому при
решении задач следует быть внимательным.

К примеру, если дано выражение −a и нас прóсят найти его


значение при a = 2, то в школе мы подставляли двойку
вместо переменной a и получали ответ −2, не особо
зацикливаясь на том, как это получалось. На самом деле
происходило умножение минус единицы на положительное
число 2

−a = −1 × a

−1 × a = −1 × 2 = −2

Если дано выражение −a и требуется найти его значение при


a = −2, то мы подставляем −2 вместо переменной a

−a = −1 × a

−1 × a = −1 × (−2) = 2

Чтобы не допускать ошибок, первое время невидимые


единицы можно записывать явно.

Пример 4. Найти значение выражения abc при a=2, b=3 и


c=4
760

Выражение abc это короткая форма записи от 1×a×b×c. Для


наглядности запишем выражение abc в развёрнутом виде и
подставим значения переменных a, b и c

1 × a × b × c = 1 × 2 × 3 × 4 = 24

Пример 5. Найти значение выражения abc при a=−2, b=−3 и


c=−4

Запишем выражение abc в развёрнутом виде и подставим


значения переменных a, b и c

1 × a × b × c = 1 × (−2) × (−3) × (−4) = −24

Пример 6. Найти значение выражения −abc при a=3, b=5 и


c=7

Выражение −abc это короткая форма записи


от −1×a×b×c. Для наглядности запишем выражение −abc в
развёрнутом виде и подставим значения переменных a, b и c

−abc = −1 × a × b × c = −1 × 3 × 5 × 7 = −105

Пример 7. Найти значение выражения −abc при a=−2, b=−4 и


c=−3

Запишем выражение −abc в развёрнутом виде:

−abc = −1 × a × b × c

Подставим значение переменных a, b и c


761

−abc = −1 × a × b × c = −1 × (−2) × (−4) × (−3) = 24

Как определить коэффициент

Иногда требуется решить задачу, в которой требуется


определить коэффициент выражения. В принципе, данная
задача очень простá. Достаточно уметь правильно умножать
числа.

Чтобы определить коэффициент в выражении, нужно


отдельно перемножить числа, входящие в это выражение, и
отдельно перемножить буквы. Получившийся числовой
сомножитель и будет коэффициентом.

Пример 1. Определить коэффициент в выражении:


7m×5a×(−3)×n

Выражение состоит из нескольких сомножителей. Это можно


отчетливо увидеть, если записать выражение в развёрнутом
виде. То есть произведения 7m и 5a записать в виде 7×m и
5×a

7 × m × 5 × a × (−3) × n

Применим сочетательный закон умножения, который


позволяет перемножать сомножители в любом порядке. А
именно, отдельно перемнóжим числа и отдельно
перемнóжим буквы (переменные):

−3 × 7 × 5 × m × a × n = −105man

Коэффициент равен −105. После завершения буквенную


часть желательно расположить в алфавитном порядке:
762

−105amn

Пример 2. Определить коэффициент в выражении:


−a×(−3)×2

Перемножим отдельно числа и буквы:

−a × (−3 ) × 2 = −3 × 2 × (−a) = −6 × (−a) = 6a

Коэффициент равен 6.

Пример 3. Определить коэффициент в выражении:

Перемножим отдельно числа и буквы:

Коэффициент равен −1. Обратите внимание, что единица не


записана, поскольку коэффициент 1 принято не записывать.

Эти казалось бы простейшие задачи могут сыграть с нами


очень злую шутку. Часто выясняется, что знак коэффициента
поставлен не верно: либо пропущен минус либо наоборот он
поставлен зря. Чтобы избежать этих досадных ошибок, тема
умножения целых чисел должна быть изучена на хорошем
уровне.
763

Слагаемые в буквенных выражениях

При сложении нескольких чисел получается сумма этих


чисел. Числа, которые складывают называют слагаемыми.
Слагаемых может быть несколько, например:

1+2+3+4+5

Когда выражение состоит из слагаемых, вычислять его


намного проще, поскольку складывать легче, чем вычитать.
Но в выражении может присутствовать не только сложение,
но и вычитание, например:

1+2−3+4−5

В этом выражении числа 3 и 5 являются вычитаемыми, а не


слагаемыми. Но нам ничего не мешает, заменить вычитание
сложением. Тогда мы снова получим выражение, состоящее
из слагаемых:

1 + 2 + (−3) + 4 + (−5)

Не суть, что числа −3 и −5 теперь со знаком минус. Главное,


что все числа в данном выражении соединены знаком
сложения, то есть выражение является суммой.

Оба выражения 1 + 2 − 3 + 4 − 5 и 1 + 2 + (−3) + 4 + (−5)


равны одному и тому значению — минус единице:

1 + 2 − 3 + 4 − 5 = −1

1 + 2 + (−3) + 4 + (−5) = −1

Таким образом, значение выражения не пострадает от того,


что мы где-то заменим вычитание сложением.
764

Заменять вычитание сложением можно и в буквенных


выражениях. Например, рассмотрим следующее выражение:

7a + 6b − 3c + 2d − 4s

Заменим вычитание сложением там, где это можно:

7a + 6b + (−3c) + 2d + (−4s)

При любых значениях переменных a, b, c, d и s выражения


7a + 6b − 3c + 2d − 4s и 7a + 6b + (−3c) + 2d + (−4s) будут
равны одному и тому же значению.

Вы должны быть готовы к тому, что учитель в школе или


преподаватель в институте может называть слагаемыми
даже те числа (или переменные), которые ими не являются.

Например, если на доске будет записана разность a − b, то


учитель не будет говорить, что a — это уменьшаемое, а b —
вычитаемое. Обе переменные он назовет одним общим
словом — слагаемые. А всё потому, что выражение вида
a − b математик видит, как сумму a + (−b). В таком случае
выражение становится суммой, а переменные a и (−b)
станóвятся слагаемыми.

Подобные слагаемые

Подобные слагаемые — это слагаемые, которые имеют


одинаковую буквенную часть.

Например, рассмотрим выражение 7a + 6b + 2a. Слагаемые


7a и 2a имеют одинаковую буквенную часть — переменную a.
Значит слагаемые 7a и 2a являются подобными.
765

Обычно подобные слагаемые складывают, чтобы упростить


выражение или решить какое-нибудь уравнение. Это
действие называют приведéнием подобных слагаемых.

Чтобы привести подобные слагаемые, нужно сложить


коэффициенты этих слагаемых, и полученный результат
умножить на общую буквенную часть.

Например, приведём подобные слагаемые в выражении


3a + 4a + 5a. В данном случае подобными являются все
слагаемые. Слóжим их коэффициенты и результат умножим
на общую буквенную часть — на переменную a

3a + 4a + 5a = (3 + 4 + 5)×a = 12a

Подобные слагаемые обычно привóдят в уме и результат


записывают сразу:

3a + 4a + 5a = 12a

Также, можно рассуждать следующим образом:

Было 3 переменные a, к ним прибавили еще 4 переменные a


и ещё 5 переменных a. В итоге получили 12 переменных a
766

Если подсчитать на рисунке количество переменных a, то


насчитается 12.

Рассмотрим несколько примеров на приведение подобных


слагаемых. Учитывая, что данная тема очень важна, на
первых порах будем записывать подробно каждую мелочь.
Несмотря на то, что здесь всё очень просто, большинство
людей допускают множество ошибок. В основном по
невнимательности, а не по незнанию.

Пример 1. Привести подобные слагаемые в выражении 3a +


2a + 6a + 8a

Сложим коэффициенты в данном выражении и полученный


результат умножим на общую буквенную часть:

3a + 2a + 6a + 8a= (3 + 2 + 6 + 8) × a = 19a

Конструкцию (3 + 2 + 6 + 8) × a можно не записывать,


поэтому сразу запишем ответ
767

3a + 2a + 6a + 8a = 19a

Пример 2. Привести подобные слагаемые в выражении 2a +


a

Второе слагаемое a записано без коэффициента, но на


самом деле перед ним стоит коэффициент 1, который мы не
видим по причине того, что его не записывают. Стало быть,
выражение выглядит следующим образом:

2a + 1a

Теперь приведем подобные слагаемые. То есть сложим


коэффициенты и результат умножим на общую буквенную
часть:

2a + 1a = (2 + 1) × a = 3a

Запишем решение покороче:

2a + a = 3a

Приводя подобные слагаемые в выражении 2a+a, можно


рассуждать и по-другому:

Было 2 переменные a, добавили ещё одну переменную a, в


итоге получилось 3 переменные a.
768

Пример 3. Привести подобные слагаемые в выражении 2a −


a

Заменим вычитание сложением:

2a + (−a)

Второе слагаемое (−a) записано без коэффициента, но на


самом деле оно выглядит как (−1a). Коэффициент −1 опять
же невидимый по причине того, что его не записывают.
Стало быть, выражение выглядит следующим образом:

2a + (−1a)

Теперь приведем подобные слагаемые. Сложим


коэффициенты и результат умножим на общую буквенную
часть:

2a + (−1a) = (2 + (−1)) × a = 1a = a

Обычно записывают короче:


769

2a − a = a

Приводя подобные слагаемые в выражении 2a−a можно


рассуждать и по-другому:

Было 2 переменные a, вычли одну переменную a, в итоге


осталась одна единственная переменная a

Пример 4. Привести подобные слагаемые в выражении 6a −


3a + 4a − 8a

Заменим вычитание сложение там, где это можно:

6a − 3a + 4a − 8a = 6a + (−3a) + 4a + (−8a)

Теперь приведем подобные слагаемые. Сложим


коэффициенты и результат умножим на общую буквенную
часть

(6 + (−3) + 4 + (−8)) × a = −1a = −a

Запишем решение покороче:


770

6a − 3a + 4a − 8a = −a

Встречаются выражения, которые содержат несколько


различных групп подобных слагаемых. Например,
3a + 3b + 7a + 2b. Для таких выражений справедливы те же
правила, что и для остальных, а именно складывание
коэффициентов и умножение полученного результата на
общую буквенную часть. Но чтобы не допустить
ошибок, удобно разные группы слагаемых подчеркнуть
разными линиями.

Например, в выражении 3a + 3b + 7a + 2b те слагаемые,


которые содержат переменную a, можно подчеркнуть одной
линией, а те слагаемые которые содержат переменную b,
можно подчеркнуть двумя линиями:

Теперь можно привести подобные слагаемые. То есть


сложить коэффициенты и полученный результат умножить
на общую буквенную часть. Сделать это нужно для обеих
групп слагаемых: для слагаемых, содержащих переменную a
и для слагаемых содержащих переменную b.

3a + 3b + 7a + 2b = (3+7)×a + (3 + 2)×b = 10a + 5b

Опять же повторимся, выражение несложное, и подобные


слагаемые можно приводить в уме:

3a + 3b + 7a + 2b = 10a + 5b
771

Пример 5. Привести подобные слагаемые в выражении 5a −


6a −7b + b

Заменим вычитание сложение там, где это можно:

5a − 6a −7b + b = 5a + (−6a) + (−7b) + b

Подчеркнём подобные слагаемые разными линиями.


Слагаемые, содержащие переменные a подчеркнем одной
линией, а слагаемые содержащие переменные b,
подчеркнем двумя линиями:

Теперь можно привести подобные слагаемые. То есть


сложить коэффициенты и полученный результат умножить
на общую буквенную часть:

5a + (−6a) + (−7b) + b = (5 + (−6))×a + ((−7) + 1)×b = −a + (−6b)

Если в выражении содержатся обычные числа без буквенных


сомножителей, то они складываются отдельно.

Пример 6. Привести подобные слагаемые в выражении 4a +


3a − 5 + 2b + 7

Заменим вычитание сложением там, где это можно:

4a + 3a − 5 + 2b + 7 = 4a + 3a + (−5) + 2b + 7

Приведем подобные слагаемые. Числа −5 и 7 не имеют


буквенных сомножителей, но они являются подобными
слагаемыми — их необходимо просто сложить. А слагаемое
772

2b останется без изменений, поскольку оно единственное в


данном выражении, имеющее буквенный сомножитель b, и
его не с чем складывать:

4a + 3a + (−5) + 2b + 7 = (4 + 3)×a + 2b + (−5) + 7 = 7a + 2b + 2

Запишем решение покороче:

4a + 3a − 5 + 2b + 7 = 7a + 2b + 2

Слагаемые можно упорядочивать, чтобы те слагаемые,


которые имеют одинаковую буквенную часть, располагались
в одной части выражения.

Пример 7. Привести подобные слагаемые в выражении


5t+2x+3x+5t+x

Поскольку выражение является суммой из нескольких


слагаемых, это позволяет нам вычислять его в любом
порядке. Поэтому слагаемые, содержащие переменную t,
можно записать в начале выражения, а слагаемые
содержащие переменную x в конце выражения:

5t + 5t + 2x + 3x + x

Теперь можно привести подобные слагаемые:

5t + 5t + 2x + 3x + x = (5+5)×t + (2+3+1)×x = 10t + 6x

Запишем решение покороче:

5t + 2x + 3x + 5t + x = 10t + 6x
773

Сумма противоположных чисел равна нулю. Это правило


работает и для буквенных выражений. Если в выражении
встретятся одинаковые слагаемые, но с противоположными
знаками, то от них можно избавиться на этапе приведения
подобных слагаемых. Иными словами, просто вычеркнуть их
из выражения, поскольку их сумма равна нулю.

Пример 8. Привести подобные слагаемые в выражении 3t −


4t − 3t + 2t

Заменим вычитание сложением там, где это можно:

3t − 4t − 3t + 2t = 3t + (−4t) + (−3t) + 2t

Слагаемые 3t и (−3t) являются противоположными. Сумма


противоположных слагаемых равна нулю. Если убрать этот
ноль из выражения, то значение выражения не изменится,
поэтому мы его и уберём. А уберём мы его обычным
вычеркиванием слагаемых 3t и (−3t)

В итоге у нас останется выражение (−4t) + 2t. В данном


выражении можно привести подобные слагаемые и получить
окончательный ответ:

(−4t) + 2t = ((−4) + 2)×t = −2t

Запишем решение покороче:


774

Упрощение выражений

Часто можно встретить задание, в котором сказано


«упростите выражение» и далее приводится выражение,
которое требуется упростить. Упростить выражение
значит сделать его прóще и корóче.

На самом деле мы уже занимались упрощением выражений,


когда сокращали дроби. После сокращения дробь
становилась короче и проще для восприятия.

Рассмотрим следующий пример. Упростить выражение .

Это задание буквально можно понять так: «Примените к


данному выражению любые допустимые действия, но
сделайте его прóще».

В данном случае можно осуществить сокращение дроби, а


именно разделить числитель и знаменатель дроби на 2:

Что ещё можно сделать? Можно вычислить

полученную дробь . Тогда мы получим десятичную дробь


0,5

В итоге дробь упростилась до 0,5.


775

Первый вопрос, который нужно себе задавать при решении


подобных задач, должен быть: «а что можно сделать?».
Потому что есть действия, которые можно делать, и есть
действия, которые делать нельзя.

Ещё один важный момент, о котором нужно помнить,


заключается в том что значение выражение не должно
измениться после упрощения выражения. Вернемся к

выражению . Данное выражение представляет собой


деление, которое можно выполнить. Выполнив это деление,
мы получаем значение данного выражения, которое равно
0,5

Но мы упростили выражение и получили новое упрощённое

выражение . Значение нового упрощённого выражения по-


прежнему равно 0,5

Но выражение мы тоже попытались упростить, вычислив


его. В итоге получили окончательный ответ 0,5.
776

Таким образом, как бы мы не упрощали выражение, значение


получаемых выражений по-прежнему равно 0,5. Значит
упрощение выполнялось верно на каждом этапе. Именно к
этому нужно стремиться при упрощении выражений —
значение выражения не должно пострадать от наших
действий.

Часто требуется упрощать буквенные выражения. Для них


справедливы те же правила упрощения, что и для числовых
выражений. Можно выполнять любые допустимые действия,
лишь бы не изменилось значение выражения.

Рассмотрим несколько примеров.

Пример 1. Упростить выражение 5,21s × t × 2,5

Чтобы упростить данное выражение, можно отдельно


перемножить числа и отдельно перемножить буквы. Это
задание очень похоже на то, которое мы рассматривали,
когда учились определять коэффициент:

5,21s × t × 2,5 = 5,21 × 2,5 × s × t = 13,025 × st = 13,025st

Таким образом, выражение 5,21s × t × 2,5 упростилось до


13,025st.

Пример 2. Упростить выражение −0,4 × (−6,3b) × 2

Второе произведение (−6,3b) можно перевести в понятный


для нас вид, а именно записать в виде (−6,3)×b, затем
отдельно перемножить числа и отдельно перемножить
буквы:
777

−0,4 × (−6,3b) × 2 = −0,4 × (−6,3) × b × 2 = 5,04b

Таким образом, выражение −0,4 × (−6,3b) × 2 упростилось до


5,04b

Пример 3. Упростить выражение

Распишем данное выражение более подробно, чтобы


хорошо увидеть, где числа, а где буквы:

Теперь отдельно перемножим числа и отдельно перемножим


буквы:

Таким образом, выражение упростилось до −abc.


Данное решение можно записать покороче:

При упрощении выражений, дроби можно сокращать в


процессе решения, а не в самом конце, как мы это делали с
обычными дробями. Например, если в ходе решения мы

наткнёмся на выражение вида , то вовсе


778

необязательно вычислять числитель и знаменатель и делать


что-то вроде этого:

Дробь можно сократить, выбирая по множителю в числителе


и в знаменателе и сокращать эти множители на их
наибольший общий делитель. Другими словами,
использовать короткую версию сокращения дроби, в которой
мы не расписываем подробно на что был разделен
числитель и знаменатель.

Например, в числителе множитель 12 и в знаменателе


множитель 4 можно сократить на 4. Четвёрку храним в уме, а
разделив 12 и 4 на эту четвёрку, ответы записываем рядом с
этими числами, предварительно зачеркнув их

Далее в числителе множитель 9 и в знаменателе множитель


3 можно сократить на 3

Далее в числителе множитель 6 и в знаменателе множитель


2 можно сократить на 2
779

Теперь можно перемножить получившиеся маленькие


множители. В данном случае их немного и можно
перемножить в уме:

Со временем можно обнаружить, что решая ту или иную


задачу, выражения начинают «толстеть», поэтому
желательно приучиться к быстрым вычислениям. То, что
можно вычислить в уме, нужно вычислять в уме. То, что
можно быстро сократить, нужно быстро сокращать.

Пример 4. Упростить выражение

Перемножим отдельно числа и отдельно буквы:

Таким образом, выражение упростилось до

Пример 5. Упростить выражение

Перемножим отдельно числа и отдельно буквы:


780

Таким образом, выражение упростилось до mn.

Пример 6. Упростить выражение

Запишем данное выражение более подробно, чтобы хорошо


увидеть, где числа, а где буквы:

Теперь отдельно перемножим числа и отдельно буквы. Для


удобства вычислений десятичную дробь −6,4 и смешанное

число можно перевести в обыкновенные дроби:

Таким образом, выражение  упростилось


до
781

Решение для данного примера можно записать значительно


короче. Выглядеть оно будет следующим образом:

Пример 7. Упростить выражение

Перемножим отдельно числа и отдельно буквы. Для

удобства вычисления смешанное число и десятичные


дроби 0,1 и 0,6 можно перевести в обыкновенные дроби:

Таким образом, выражение упростилось до


abcd. Если пропустить подробности, то данное решение
можно записать значительно короче:

Обратите внимание на то, как сократилась дробь. Новые


множители, которые получаются в результате сокращения
предыдущих множителей, тоже допускается сокращать.
782

Теперь поговорим о том, чего делать нельзя. При упрощении


выражений категорически нельзя перемножать числа и
буквы, если выражение является суммой, а не
произведением.

Например, если требуется упростить выражение 5a + 4b, то


нельзя записывать следующим образом:

Это равносильно тому, что если бы нас попросили сложить


два числа, а мы бы их перемножали вместо того, чтобы
складывать.

При подстановке любых значений переменных a и b


выражение 5a  +4b обращается в обыкновенное числовое
выражение. Предположим, что переменные a и b имеют
следующие значения:

a = 2, b = 3

Тогда значение выражения будет равно 22

5a + 4b = 5 × 2 + 4 × 3 = 10 + 12 = 22

Сначала выполняется умножение, а затем полученные


результаты складывают. А если бы мы попытались упростить
данное выражение, перемножив числа и буквы, то
получилось бы следующее:

5a + 4b = 5 × 4 × a × b = 20ab

20ab = 20 × 2 × 3 = 120
783

Получается совсем другое значение выражения. В первом


случае получилось 22, во втором случае 120. Это означает,
что упрощение выражения 5a + 4b было выполнено неверно.

После упрощения выражения, его значение не должно


изменяться при одних и тех же значениях переменных. Если
при подстановке в изначальное выражение любых значений
переменных получается одно значение, то после упрощения
выражения должно получаться то же самое значение, что и
до упрощения.

С выражением 5a + 4b на самом деле ничего делать нельзя.


Оно не упрощается.

Если в выражении содержатся подобные слагаемые, то их


можно сложить, если нашей целью является упрощение
выражения.

Пример 8. Упростить выражение 0,3a−0,4a+a

Чтобы упростить данное выражение можно привести


подобные слагаемые:

0,3a − 0,4a + a = 0,3a + (−0,4a) + a = (0,3 + (−0,4) + 1)×a = 0,9a

или покороче: 0,3a − 0,4a + a = 0,9a

Таким образом, выражение 0,3a−0,4a+a упростилось до 0,9a

Пример 9. Упростить выражение −7,5a − 2,5b + 4a


784

Чтобы упростить данное выражение можно привести


подобные слагаемые:

−7,5a − 2,5b + 4a = −7,5a + (−2,5b) + 4a = ((−7,5) + 4)×a +


(−2,5b) = −3,5a + (−2,5b)

или покороче −7,5a − 2,5b + 4a = −3,5a + (−2,5b)

Слагаемое (−2,5b) осталось без изменений, поскольку его не


с чем было складывать.

Пример 10. Упростить выражение

Чтобы упростить данное выражение можно привести


подобные слагаемые:

Коэффициент был переведён в неправильную дробь для


удобства вычисления.

Таким образом, выражение упростилось до

Пример 11. Упростить выражение


785

Чтобы упростить данное выражение можно привести


подобные слагаемые:

Таким образом, выражение упростилось до .

В данном примере целесообразнее было бы сложить первый


и последний коэффициент в первую очередь. В этом случае
мы получили бы короткое решение. Выглядело бы оно
следующим образом:

Пример 12. Упростить выражение

Чтобы упростить данное выражение можно привести


подобные слагаемые:
786

Таким образом, выражение упростилось до

Слагаемое осталось без изменения, поскольку его не с


чем было складывать.

Данное решение можно записать значительно короче.


Выглядеть оно будет следующим образом:

В коротком решении пропущены этапы замены вычитания


сложением и подробная запись, как дроби приводились к
общему знаменателю.

Ещё одно различие заключается в том, что в подробном

решении ответ выглядит как , а в коротком как


787

. На самом деле, это одно и то же выражение.


Различие в том, что в первом случае вычитание заменено
сложением, поскольку в начале когда мы записывали
решение в подробном виде, мы везде где можно заменили
вычитание сложением, и эта замена сохранилась и для
ответа.

Тождества. Тождественно равные выражения

После того как мы упростили какое-нибудь выражение, оно


станóвится проще и короче. Чтобы проверить верно ли
упрощено выражение, достаточно подстáвить любые
значения переменных сначала в предыдущее выражение,
которое требовалось упростить, а затем в новое, которое
упростили. Если значение в обоих выражениях будет
одинаковым, то это означает, что выражение упрощено
верно.

Рассмотрим простейший пример. Пусть требуется упростить


выражение 2a × 7b. Чтобы упростить данное выражение,
можно по-отдельности перемнóжить числа и буквы:

2a × 7b = 2 × 7 × a × b = 14ab

Проверим верно ли мы упростили выражение. Для этого


подставим любые значения переменных a и b сначала в
первое выражение, которое требовалось упростить, а затем
во второе, которое упростили.

Пусть значения переменных a, b будут следующими:


788

a=4
b=5

Подстáвим их в первое выражение 2a × 7b

2a × 7b = 2 × 4 × 7 × 5 = 280

Теперь подстáвим те же значения переменных в выражение,


которое получилось в результате упрощения выражения
2a × 7b, а именно в выражение 14ab

14ab = 14 × 4 × 5 = 280

Видим, что при a = 4 и b = 5 значение первого выражения


2a × 7b и значение второго выражения 14ab равны

2a × 7b = 2 × 4 × 7 × 5 = 280

14ab = 14 × 4 × 5 = 280

То же самое произойдет и для любых других значений.


Например, пусть a = 1 и b = 2

2a × 7b = 2 × 1 × 7 × 2 = 28

14ab = 14 × 1 × 2 = 28

Таким образом, выражения 2a × 7b и 14ab при любых


значениях переменных равны одному и тому же значению.
Такие выражения называют тождественно равными.

Делаем вывод, что между выражениями 2a × 7b и


14ab можно поставить знак равенства, поскольку они равны
одному и тому же значению:

2a × 7b = 14ab
789

Равенством называют любое выражение, которые соединено


знаком равенства (=).

А равенство вида 2a × 7b = 14ab называют тождеством.

Тождеством называют равенство, которое верно при


любых значениях переменных.

Другие примеры тождеств:

a+b=b+a

a(b + c) = ab + ac

a(bc) = (ab)c

Да, законы математики, которые мы изучали, являются


тождествами.

Верные числовые равенства тоже являются тождествами.


Например:

2+2=4

3+3=5+1

10 = 7 + 2 + 1

Решая сложную задачу, чтобы облегчить себе вычисление,


сложное выражение заменяют на более простое выражение,
тождественно равное предыдущему. Такую замену называют
тождественным преобразованием выражения или просто
преобразованием выражения.
790

Например, мы упростили выражение 2a × 7b, и получили


более простое выражение 14ab. Это упрощение можно
называть тождественным преобразованием.

Часто можно встретить задание, в котором сказано


«докажите, что равенство является тождеством» и
далее приводится равенство, которое требуется доказать.
Обычно это равенство состоит из двух частей: левой и
правой части равенства. Наша задача состоит в том, чтобы
выполнить тождественные преобразования с одной из частей
равенства и получить другую часть. Либо выполнить
тождественные преобразования с обеими частями равенства
и сделать так, чтобы в обеих частях равенства оказались
одинаковые выражения.

Например, докажем, что равенство 0,5a × 5b = 2,5ab


является тождеством.

Упростим левую часть этого равенства. Для


этого перемножим числа и буквы по отдельности:

0,5 × 5 × a × b = 2,5ab

2,5ab = 2,5ab

В результате небольшого тождественного преобразования,


левая часть равенства стала равна правой части равенства.
Значит мы доказали, что равенство 0,5a × 5b = 2,5ab
является тождеством.

Из тождественных преобразований мы научились


складывать, вычитать, умножать и делить числа, сокращать
дроби, приводить подобные слагаемые, а также упрощать
некоторые выражения.
791

Но это далеко не все тождественные преобразования,


которые существуют в математике. Тождественных
преобразований намного больше. В будущем мы ещё не раз
в этом убедимся.

Задания для самостоятельного решения:

Задание 1. Найдите значение выражения при и


Показать решение
Задание 2. Найдите значение выражения при

Показать решение

Задание 3. Найдите значение выражения при и

и
Показать решение
Задание 4. Найдите значение выражения при и

Показать решение
Задание 5. Запишите в виде буквенного выражения
следующую последовательность действий:

 Число a умножить на три, и из этого произведения


вычесть пятнадцать
 Число t умножить на девять, и к полученному
произведению прибавить тридцать пять

Показать решение
792

Задание 6. Приведите подобные слагаемые в следующем


выражении:

Показать решение
Задание 7. Приведите подобные слагаемые в следующем
выражении:

Показать решение
Задание 8. Приведите подобные слагаемые в следующем
выражении:

Показать решение
Задание 9. Приведите подобные слагаемые в следующем
выражении:

Показать решение
Задание 10. Приведите подобные слагаемые в следующем
выражении:

Показать решение
Задание 11. Упростите выражение:

Показать решение
Задание 12. Упростите выражение:
793

Показать решение
Задание 13. Упростите выражение:

Показать решение
Задание 14. Упростите выражение:

Показать решение
Задание 15. Упростите выражение:

Показать решение
Задание 16. Упростите выражение:

Показать решение
Задание 17. Упростите выражение:

Показать решение
Задание 18. Упростите выражение:

Показать решение
Задание 19. Упростите выражение:

Показать решение
794

Задание 20. Упростите выражение:

Показать решение
795

Вынесение общего множителя за скобки

Продолжаем разбираться с основами алгебры. Сегодня мы


поработаем с распределительным законом умножения, а
именно рассмотрим такое действие как вынесение общего
множителя за скобки.

Содержание урока

 Основной принцип
 Как происходит вынесение общего множителя за скобки
 Вынесение минуса за скобки
 Вынесение общего множителя за скобки в буквенном
выражении
 Задания для самостоятельного решения

Основной принцип

Распределительный закон умножения позволяет умножить


число на сумму (или сумму на число). Например, чтобы найти
значение выражения 3 × (4 + 5) можно умножить число 3 на
каждое слагаемое в скобках и сложить полученные
результаты:

3 × (4 + 5) = 3 × 4 + 3 × 5 = 12 + 15

Число 3 и выражение в скобках можно поменять местами


(это следует из переместительного закона умножения). Тогда
каждое слагаемое, которое в скобках, будет умножено на
число 3

 (4 + 5) × 3 = 4 × 3 + 5 × 3= 12 + 15
796

Пока не будем вычислять конструкцию 3 × 4 + 3 × 5 и


складывать полученные результаты 12 и 15. Оставим
выражение в виде 3(4 + 5) = 3 × 4 + 3 × 5. Ниже оно нам
потребуется именно в таком виде, чтобы понять суть
вынесения общего множителя за скобки.

Распределительный закон умножения иногда называют


внесением множителя во внутрь скобок. В выражении
3 × (4 + 5) множитель 3 был за скобками. Умножив его на
каждое слагаемое в скобках, мы по сути внесли его во внутрь
скобок. Для наглядности можно так и записать, хоть и не
принято так записывать:

3 (4 + 5) = (3 × 4 + 3 × 5)

Поскольку в выражении 3 × (4 + 5) число 3 умножается на


каждое слагаемое в скобках, это число является общим
множителем для слагаемых 4 и 5

Как говорилось ранее, умножив этот общий множитель на


каждое слагаемое в скобках, мы вносим его во внутрь скобок.
Но возможен и обратный процесс — общий множитель
можно обратно вынести за скобки. В данном случае в
выражении 3 × 4 + 3 × 5 общий множитель виден, как на
ладони — это множитель 3. Его и нужно вынести за скобки.
Для этого сначала записывается сам множитель 3

3
797

и рядом в скобках записывается выражение 3 × 4 + 3 × 5 но


уже без общего множителя 3, поскольку он вынесен за скобки

3(4 + 5)

В результате вынесения общего множителя за скобки


получается выражение 3(4 + 5). Это выражение
тождественно равно предыдущему выражению 3 × 4 + 3 × 5

3(4 + 5) = 3 × 4 + 3 × 5

Если вычислить обе части полученного равенства, то


получим тождество:

3(4 + 5) = 3 × 4 + 3 × 5

27 = 27

Как происходит вынесение общего множителя за скобки

Вынесение общего множителя за скобки по сути является


обратной операцией внесению общего множителя во внутрь
скобок.

Если при внесении общего множителя внутрь скобок, мы


умножаем этот множитель на каждое слагаемое в скобках, то
при вынесении этого множителя обратно за скобки, мы
должны разделить каждое слагаемое в скобках на этот
множитель.

В выражении 3 × 4 + 3 × 5, которое было рассмотрено выше,


так и происходило. Каждое слагаемое было разделено на
общий множитель 3. Произведения 3 × 4 и 3 × 5 и являются
798

слагаемыми, поскольку если их вычислить, мы получим


сумму 12 + 15

Теперь мы можем детально увидеть, как происходит


вынесение общего множителя за скобки:

Видно, что общий множитель 3 сначала вынесен за скобки,


затем в скобках происходит деление каждого слагаемого на
этот общий множитель.

Деление каждого слагаемого на общий множитель можно


выполнять не только разделяя числитель на знаменатель,
как это было показано выше, но и сокращая эти дроби. В
обоих случаях получится один и тот же результат:
799

Мы рассмотрели простейший пример вынесения общего


множителя за скобки, чтобы понять основной принцип.

Но не всё так просто, как кажется на первый взгляд. После


того, как число умножено на каждое слагаемое в скобках,
полученные результаты складывают, и общий множитель
пропадает из виду.

Вернёмся к нашему примеру 3(4 + 5). Применим


распределительный закон умножения, то есть умножим
число 3 на каждое слагаемое в скобках и сложим полученные
результаты:

3 × (4 + 5) = 3 × 4 + 3 × 5 = 12 + 15

После того, как вычислена конструкция 3 × 4 + 3 × 5, мы


получаем новое выражение 12 + 15. Видим, что общий
множитель 3 пропал из виду. Теперь в полученном
выражении 12 + 15 попробуем обратно вынести общий
множитель за скобки, но чтобы вынести этот общий
множитель его сначала нужно найти.

Обычно при решении задач встречаются именно такие


выражения, в которых общий множитель сначала нужно
найти, прежде чем его выносить.

Чтобы в выражении 12 + 15 вынести общий множитель за


скобки, нужно найти наибольший общий делитель (НОД)
800

слагаемых 12 и 15. Найденный НОД и будет общим


множителем.

Итак, найдём НОД слагаемых 12 и 15. Напомним, что для


нахождения НОД необходимо разложить исходные числа на
простые множители, затем выписать первое разложение и
убрать из него множители, которые не входят в разложение
второго числа. Оставшиеся множители нужно перемножить и
получить искомый НОД. Если испытываете затруднения на
этом моменте, обязательно повторите этот урок.

НОД слагаемых 12 и 15 это число 3. Данное число является


общим множителем слагаемых 12 и 15. Его и нужно выносить
за скобки. Для этого сначала записываем сам множитель 3 и
рядом в скобках записываем новое выражение, в котором
каждое слагаемое выражения 12 + 15 разделено на общий
множитель 3
801

Ну и дальнейшее вычисление не составляет особого труда.


Выражение в скобках легко вычисляется — двенадцать
разделить на три будет четыре, а пятнадцать
разделить на три будет пять:

Таким образом, при вынесении общего множителя за скобки


в выражении 12 + 15 получается выражение 3(4 + 5).
Подробное решение выглядит следующим образом:

В коротком решении пропускают запись в которой показано,


как каждое слагаемое разделено на общий множитель:

Пример 2. Вынести общий множитель за скобки в выражении


15 + 20
802

Наибольший общий делитель слагаемых 15 и 20 это число 5.


Данное число является общим множителем слагаемых 15 и
20. Его и вынесем за скобки:

Получили выражение 5(3 + 4).

Получившееся выражение 5(3 + 4) можно проверить. Для


этого достаточно умножить пятёрку на каждое слагаемое в
скобках. Если мы всё сделали правильно, то должны
получить выражение 15 + 20

Пример 3. Вынести общий множитель за скобки в выражении


18 + 24 + 36

Найдём НОД слагаемых 18, 24 и 36. Чтобы найти НОД


нескольких чисел, нужно разложить эти числа на простые
множители, затем найти произведение общих множителей:
803

НОД слагаемых 18, 24 и 36 это число 6. Данное число


является общим множителем слагаемых 18, 24 и 36. Его и
вынесем за скобки:

Проверим получившееся выражение. Для этого умножим


число 6 на каждое слагаемое в скобках. Если мы всё сделали
правильно, то должны получить выражение 18 + 24 + 36

Пример 4. Вынести общий множитель за скобки в выражении


13 + 5

Слагаемые 13 и 5 являются простыми числами. Они


раскладываются только на единицу и самих себя:
804

Это значит, что у слагаемых 13 и 5 нет общих множителей,


кроме единицы. Соответственно, нет смысла выносить эту
единицу за скобки, поскольку это ничего не даст. Покажем
это:

Пример 5. Вынести общий множитель за скобки в выражении


195 + 156 + 260

Найдём НОД слагаемых 195, 156 и 260

НОД слагаемых 195, 156 и 260 это число 13. Данное число
является общим множителем для слагаемых 195, 156 и 260.
Его и вынесем за скобки:
805

Проверим получившееся выражение. Для этого умножим 13


на каждое слагаемое в скобках. Если мы всё сделали
правильно, то должны получить выражение 195 + 156 + 260

Выражение, в котором требуется вынести общий множитель


за скобки, может быть не только суммой чисел, но и
разностью. Например, вынесем общий множитель за скобки в
выражении 16 − 12 − 4. Наибольшим общим делителем
чисел 16, 12 и 4 это число 4. Данное число и вынесем за
скобки:

Проверим получившееся выражение. Для этого умножим


четвёрку на каждое число в скобках. Если мы всё сделали
правильно, то должны получить выражение 16 − 12 − 4

Пример 6. Вынести общий множитель за скобки в выражении


72 + 96 − 120

Найдём НОД чисел 72, 96 и 120


806

НОД для 72, 96 и 120 это число 24. Данное число является
общим множителем слагаемых 195, 156 и 260. Его и вынесем
за скобки:

Проверим получившееся выражение. Для этого умножим 24


на каждое число в скобках. Если мы всё сделали правильно,
то должны получить выражение 72+96−120

Общий множитель, выносимый за скобки, может быть и


отрицательным. Например, вынесем общий множитель за
скобки в выражении −6 − 3. Вынести общий множитель за
807

скобки в таком выражении можно двумя способами.


Рассмотрим каждый из них.

Способ 1.

Заменим вычитание сложением:

−6 + (−3)

Теперь находим общий множитель. Общим множителем


данного выражения будет наибольший общий делитель
модулей слагаемых −6 и −3.

Модуль первого слагаемого это 6. А модуль второго


слагаемого это 3. НОД(6 и 3) равен 3. Данное число является
общим множителем слагаемых 6 и 3. Его и вынесем за
скобки:

Выражение полученное таким способом получилось не очень


аккуратным. Много скобок и отрицательных чисел не
придают выражению простоту. Поэтому можно
воспользоваться вторым способом, суть которого
заключается в том, чтобы вынести за скобки не 3, а −3.

Способ 2.

Как и в прошлый раз заменяем вычитание сложением

−6 + (−3)

В этот раз мы вынесем за скобки не 3, а  −3


808

Выражение полученное в этот раз выглядит намного проще.


Запишем решение покороче, чтобы сделать его ещё проще:

Разрешать выносить отрицательный множитель за скобки


связано с тем, что разложение чисел −6 и (−3) можно
записать двумя видами: сначала сделать множимое
отрицательным, а множитель положительным:

−6 = −2 × 3

−3 = −1 × 3

во втором случае множимое можно сделать положительным,


а множитель отрицательным:

−6 = 2 × (−3)

−3 = 1 × (−3)

А значит мы вольны выносить за скобки тот сомножитель,


который захотим.

Пример 8. Вынести общий множитель за скобки в выражении


−20 − 16 − 2

Заменим вычитание сложением

−20 − 16 − 2 = −20 + (−16) + (−2)


809

Наибольшим общим делителем слагаемых −20, −16 и −2


является число 2. Это число является общим множителем
этих слагаемых. Посмотрим, как это выглядит:

−20 = −10 × 2

−16 = −8 × 2

−2 = −1 × 2

Но приведенные разложения можно заменить на


тождественно равные разложения. Различие будет в том, что
общим множителем будет не 2, а −2

−20 = 10 × (−2)

−16 = 8 × (−2)

−2 = 1 × (−2)

Поэтому для удобства за скобки можно вынести не 2, а −2

Запишем приведенное решение покороче:

А если бы мы вынесли за скобки 2, то получилось бы не


совсем аккуратное выражение:
810

Пример 9. Вынести общий множитель за скобки в выражении


−30 − 36 − 42

Заменим вычитание сложением:

−30 + (−36) + (−42)

Наибольшим общим делителем слагаемых −30, −36 и −42


это число 6. Данное число является общим множителем для
этих слагаемых. Но за скобки мы вынесем не 6, а −6
поскольку числа −30, −36 и −42 можно представить так:

−30 = 5 × (−6)

−36 = 6 × (−6)

−42 = 7 × (−6)

Вынесение минуса за скобки

При решении задач иногда может быть полезным вынесение


минуса за скобки. Это позволяет упростить выражение и
сделать его проще.
811

Рассмотрим следующий пример. Вынести минус за скобки в


выражении −15 + (−5) + (−3)

Для наглядности заключим данное выражение в скобки, ведь


речь идёт о том, чтобы вынести минус за эти скобки

( −15 + (−5) + (−3) )

Итак, чтобы вынести минус за скобки, нужно записать перед


скобками минус и в скобках записать все слагаемые, но с
противоположными знаками. Знаки операций (то есть плюсы)
оставляем без изменений:

−(15 + 5 + 3)

Мы вынесли минус за скобки в выражении −15 + (−5) + (−3) и


получили −(15 + 5 + 3). Оба выражения равны одному и тому
же значению −23

−15 + (−5) + (−3) = −23

−(15 + 5 + 3) = −(23) = −23

Поэтому между выражениями −15 + (−5) + (−3) и −(15 + 5 + 3)


можно поставить знак равенства, потому что они равны
одному и тому же значению:

−15 + (−5) + (−3) = −(15 + 5 + 3)

−23 = −23

На самом деле при вынесении минуса за скобки опять же


срабатывает распределительный закон умножения:

a(b + c) = ab + ac
812

Если поменять местами левую и правую часть этого


тождества, то получится, что сомножитель a вынесен за
скобки

 ab + ac = a(b+c)

Тоже самое происходит, когда мы выносим общий множитель


в других выражениях и когда выносим минус за скобки.

Очевидно, что при вынесении минуса за скобки, выносится


не минус, а минус единица. Ранее мы говорили, что
коэффициент 1 принято не записывать.

Поэтому и образуется перед скобками минус, а знаки


слагаемых которые были в скобках меняют свой знак на
противоположный, поскольку каждое слагаемое разделено
на минус единицу.

Вернёмся к предыдущему примеру и детально увидим, как на


самом деле выносился минус за скобки

Пример 2. Вынести минус за скобки в выражении −3 + 5 + 11

Ставим минус и рядом в скобках записываем выражение


−3 + 5 + 11 с противоположным знаком у каждого слагаемого:

−3 + 5 + 11 = −(3 − 5 − 11)
813

Как и в прошлом примере, здесь за скобки вынесен не минус,


а минус единица. Подробное решение выглядит следующим
образом:

Сначала получилось выражение −1(3 + (−5) + (−11)), но мы


раскрыли в нём внутренние скобки и получили выражение
−(3 − 5 − 11). Раскрытие скобок это тема следующего урока,
поэтому если данный пример вызывает у вас затруднения,
можете пока пропустить его.

Вынесение общего множителя за скобки в буквенном


выражении

Выносить общий множитель за скобки в буквенном


выражении намного интереснее.

Для начала рассмотрим простейший пример. Пусть имеется


выражение 3a + 2a. Вынесем общий множитель за скобки.

В данном случае, общий множитель виден невооруженным


глазом — это множитель a. Его и вынесем за скобки. Для
этого записываем сам множитель a и рядом в скобках
записываем выражение 3a + 2a, но уже без множителя a
поскольку он вынесен за скобки:
814

Как и в случае с числовым выражением, здесь происходит


деление каждого слагаемого на вынесенный общий
множитель. Выглядит это так:

В обеих дробях переменные a были сокращены на a. Вместо


них в числителе и в знаменателе получились единицы.
Единицы получились по причине того, что вместо
переменной a может стоять любое число. Эта переменная
располагалась и в числителе и в знаменателе. А если в
числителе и в знаменателе располагаются одинаковые
числа, то наибольший общий делитель для них будет само
это число.

Например, если вместо переменной a подставить число 4, то

конструкция примет следующий вид: .


Тогда четвёрки в обеих дробях можно будет сократить на 4:

Получается то же самое, что и раньше, когда вместо


четвёрок стояла переменная a.
815

Поэтому не следует пугаться при виде сокращения


переменных. Переменная это полноправный множитель,
пусть даже выраженный буквой. Такой множитель можно
выносить за скобки, сокращать и выполнять другие действия,
которые допустимы к обычным числам.

Буквенное выражение содержит не только числа, но и буквы


(переменные). Поэтому общий множитель, который
выносится за скобки часто бывает буквенным множителем,
состоящим из числа и буквы (коэффициента и переменной).
К примеру, следующие выражения являются буквенными
множителями:

3a, 6b, 7ab, a, b, c

Прежде чем выносить такой множитель за скобки, нужно


определиться, какое число будет в числовой части общего
множителя и какая переменная будет в буквенной части
общего множителя. Другими словами, нужно узнать какой
коэффициент будет у общего множителя и какая переменная
будет в него входить.

Рассмотрим выражение 10a + 15a. Попробуем вынести в нём


общий множитель за скобки. Сначала определимся из чего
будет состоять общий множитель, то есть узнаем его
коэффициент и какая переменная будет в него входить.

Коэффициентом общего множителя должен быть


наибольший общий делитель коэффициентов буквенного
выражения 10a + 15a. Коэффициентами данного выражения
являются числа 10 и 15, а их наибольший общий делитель
это число 5. Значит число 5 будет коэффициентом общего
множителя, выносимого за скобки.
816

Теперь определимся какая переменная будет входить в


общий множитель. Для этого нужно посмотреть на
выражение 10a + 15a и найти буквенный сомножитель,
который входит во все слагаемые. В данном случае, это
сомножитель a. Этот сомножитель входит в каждое
слагаемое выражения 10a + 15a. Значит переменная a будет
входить в буквенную часть общего множителя, выносимого
за скобки:

Теперь осталось вынести общий множитель 5a за скобки.


Для этого разделим каждое слагаемое выражения 10a + 15a
на 5a. Для наглядности коэффициенты и числа будем
отделять знаком умножения (×)

Проверим получившееся выражение. Для этого умножим 5a


на каждое слагаемое в скобках. Если мы всё сделали
правильно, то получим выражение 10a + 15a

Буквенный множитель не всегда можно вынести за скобки.


Иногда общий множитель состоит только из числа, поскольку
ничего подходящего для буквенной части в выражении не
находится.
817

Например, вынесем общий множитель за скобки в


выражении 2a − 2b. Здесь общим множителем будет только
число 2, а среди буквенных сомножителей общих
множителей в выражении нет. Поэтому в данном случае
будет вынесен только множитель 2

Пример 2. Вынести общий множитель выражении


3x + 9y + 12

Коэффициентами данного выражения являются числа 3, 9 и


12, их НОД равен 3. Значит коэффициентом общего
множителя, выносимого за скобки, будет число 3. А среди
буквенных сомножителей (переменных) нет общего
множителя. Поэтому окончательный общий множитель это 3

Пример 3. Вынести общий множитель за скобки в выражении


8x + 6y + 4z + 10 + 2

Коэффициентами данного выражения являются числа 8, 6, 4,


10 и 2, их НОД равен 2. Значит коэффициентом общего
множителя, выносимого за скобки, будет число 2. А среди
буквенных сомножителей нет общего множителя. Поэтому
окончательный общий множитель это 2
818

Пример 4. Вынести общий множитель 6ab + 18ab + 3abc

Коэффициентами данного выражения являются числа 6, 18 и


3, их НОД равен 3. Значит коэффициентом общего
множителя, выносимого за скобки, будет число 3. В
буквенную часть общего множителя будут входить
переменные a и b, поскольку в выражении 6ab + 18ab + 3abc
эти две переменные входят в каждое слагаемое. Поэтому
окончательный общий множитель это 3ab

При подробном решении выражение становится громоздким


и даже непонятным. В данном примере это более чем
заметно. Это связано с тем, что мы сокращаем множители в
числителе и в знаменателе. Лучше всего делать это в уме и
сразу записывать результаты деления. Тогда выражение
станет коротким и аккуратным:

Как и в случае с числовым выражением в буквенном


выражении общий множитель может быть и отрицательным.
819

Например, вынесем общий множитель за скобки в


выражении −3a − 2a.

Для удобства заменим вычитание сложением

−3a − 2a = −3a + (−2a)

Общим множителем в данном выражении является


множитель a. Но за скобки можно вынести не только a, но и
−a. Его и вынесем за скобки:

Получилось аккуратное выражение −a(3+2). Не следует


забывать, что множитель −a на самом деле выглядел как −1a
и после сокращения в обеих дробях переменных a, в
знаменателях остались минус единицы. Поэтому в итоге и
получаются положительные ответы в скобках

Пример 6. Вынести общий множитель за скобки в выражении


−6x − 6y

Заменим вычитание сложением

−6x−6y = −6x+(−6y)

Вынесем за скобки −6
820

Запишем решение покороче:

−6x − 6y = −6(x + y)

Пример 7. Вынести общий множитель за скобки в выражении


−2a − 4b − 6c

Заменим вычитание сложением

−2a-4b-6c = −2a + (−4b) + (−6c)

Вынесем за скобки −2

Запишем решение покороче:

−2a − 4b − 6c = −2(a + 2b + 3c)

Вынесение общего множителя за скобки это очень важная


тема. В данном уроке рассмотрены только азы и простейшие
примеры. Мы ещё вернемся к этой теме, когда будем изучать
многочлены.

Обязательно изучите данный урок, поскольку при изучении


многочленов потребуется выносить за скобки сложный
множитель, состоящий из степеней.
821

Задания для самостоятельного решения


Задание 1. Вынесите общий множитель за скобки в
следующем выражении:

Показать решение
Задание 2. Вынесите общий множитель за скобки в
следующем выражении:

Показать решение
Задание 3. Вынесите общий множитель за скобки в
следующем выражении:

Показать решение
Задание 4. Вынесите общий множитель за скобки в
следующем выражении:

Показать решение
Задание 5. Вынесите общий множитель за скобки в
следующем выражении:

Показать решение
Задание 6. Вынесите общий множитель за скобки в
следующем выражении:

Показать решение
Задание 7. Вынесите общий множитель за скобки в
следующем выражении:
822

Показать решение
Задание 8. Вынесите общий множитель за скобки в
следующем выражении:

Показать решение
Задание 9. Вынесите общий множитель за скобки в
следующем выражении:

Показать решение
Задание 10. Вынесите общий множитель за скобки в
следующем выражении:

Показать решение
Задание 11. Вынесите общий множитель за скобки в
следующем выражении:

Показать решение
Задание 12. Вынесите общий множитель за скобки в
следующем выражении:

Показать решение
Задание 13. Вынесите общий множитель за скобки в
следующем выражении:

Показать решение
823

Задание 14. Вынесите общий множитель за скобки в


следующем выражении:

Показать решение
Задание 15. Вынесите общий множитель за скобки в
следующем выражении:

Показать решение
Задание 16. Вынесите минус за скобки в следующем
выражении:

Показать решение
Задание 17. Вынесите минус за скобки в следующем
выражении:

Показать решение
Задание 18. Вынесите минус за скобки в следующем
выражении:

Показать решение
Задание 19. Вынесите общий множитель за скобки в
следующем выражении:

Показать решение
Задание 20. Вынесите общий множитель за скобки в
следующем выражении:
824

Показать решение
825

Раскрытие скобок

Продолжаем изучать основы алгебры. В данном уроке мы


научимся раскрывать скобки в выражениях. Раскрыть скобки
означает избавить выражение от этих скобок.

Чтобы раскрывать скобки, нужно выучить наизусть два


правила. При регулярных занятиях раскрывать скобки можно
с закрытыми глазами, и про те правила которые требовалось
заучивать наизусть, можно благополучно забыть.

Содержание урока

 Первое правило раскрытия скобок


 Второе правило раскрытия скобок
 Механизм раскрытия скобок
 Задания для самостоятельного решения

Первое правило раскрытия скобок

Рассмотрим следующее выражение:

8 + (−9 + 3)

Значение данного выражения равно 2. Раскроем скобки в


данном выражении. Раскрыть скобки означает избавиться от
них, не влияя на значение выражения. То есть после
избавления от скобок значение выражения 8 + (−9 + 3) по
прежнему должно быть равно двум.

Первое правило раскрытия скобок выглядит следующим


образом:
826

При раскрытии скобок, если перед скобками стоит плюс, то


этот плюс опускается вместе со скобками.

Итак, мы видим что в выражении 8 + (−9 + 3) перед скобками


стоит плюс. Этот плюс нужно опустить вместе со скобками.
Иными словами, скобки исчезнут вместе с плюсом, который
перед ними стоял. А то, что было в скобках запишется без
изменений:

Мы получили выражение без скобок 8−9+3. Данное


выражение равно 2, как и предыдущее выражение со
скобками было равно 2.

8 + (−9 + 3) = 2

8−9+3=2

Таким образом, между выражениями 8+(−9+3) и 8−9+3 можно


поставить знак равенства, поскольку они равны одному и
тому же значению:

8 + (−9 + 3) = 8 − 9 + 3
827

2=2

Пример 2. Раскрыть скобки в выражении 3 + (−1 − 4)

Перед скобками стоит плюс, значит этот плюс опускается


вместе со скобками. То, что было в скобках останется без
изменений:

3 + (−1 − 4) = 3 − 1 − 4

Пример 3. Раскрыть скобки в выражении 2 + (−1)

Перед скобками стоит плюс, значит этот плюс опускается


вместе со скобками. То, что было в скобках останется без
изменений:

2 + (−1) = 2 − 1

В данном примере раскрытие скобок стало своего рода


обратной операцией замене вычитания сложением. Как это
понимать?

В выражении 2 − 1 происходит вычитание, но его можно


заменить сложением. Тогда получится выражение 2 + (−1).
Но если в выражении 2 + (−1) раскрыть скобки, то получится
изначальное 2 − 1.

Поэтому первое правило раскрытия скобок можно


использовать для упрощения выражений после каких-нибудь
преобразований. То есть избавить его от скобок и сделать
проще.

Например, упростим выражение 2a + a− 5b + b.


828

Чтобы упростить данное выражение, можно привести


подобные слагаемые. Напомним, что для приведения
подобных слагаемых, нужно сложить коэффициенты
подобных слагаемых и результат умножить на общую
буквенную часть:

Получили выражение 3a + (−4b). В этом выражении раскроем


скобки. Перед скобками стоит плюс, поэтому используем
первое правило раскрытия скобок, то есть опускаем скобки
вместе с плюсом, который стоит перед этими скобками:

3a + (−4b) = 3a − 4b

Таким образом, выражение 2a+a−5b+b упрощается до 3a−4b.

Раскрыв одни скобки, по пути могут встретиться другие. К


ним применяем те же правила, что и к первым. Например,
раскроем скобки в следующем выражении:

2 + (−3 + 1) + 3 + (−6)

Здесь два места, где нужно раскрыть скобки. В данном


случае применимо первое правило раскрытия скобок, а
именно опускание скобок вместе с плюсом, который стоит
перед этими скобками:

2 + (−3 + 1) + 3 + (−6) = 2 − 3 + 1 + 3 − 6

Пример 3. Раскрыть скобки в выражении 6+(−3)+(−2)


829

В обоих местах, где имеются скобки, перед ними стоит плюс.


Здесь опять же применяется первое правило раскрытия
скобок:

6 + (−3) + (−2) = 6 − 3 − 2

Иногда первое слагаемое в скобках записано без знака.


Например, в выражении 1+(2+3−4) первое слагаемое в
скобках 2 записано без знака. Возникает вопрос, а какой знак
будет стоять перед двойкой после того, как скобки и плюс,
стоящий перед скобками опустятся? Ответ напрашивается
сам — перед двойкой будет стоять плюс.

На самом деле даже будучи в скобках перед двойкой стоит


плюс, но мы его не видим по причине того, что его не
записывают. Мы уже говорили, что полная запись
положительных чисел выглядит как +1, +2, +3. Но плюсы по
традиции не записывают, поэтому мы и видим привычные
для нас положительные числа 1, 2, 3.

Поэтому, чтобы раскрыть скобки в выражении 1+(2+3−4),


нужно как обычно опустить скобки вместе с плюсом, стоящим
перед этими скобками, но первое слагаемое которое было в
скобках записать со знаком плюс:

1 + (2 + 3 − 4) = 1 + 2 + 3 − 4

Пример 4. Раскрыть скобки в выражении −5 + (2 − 3)

Перед скобками стоит плюс, поэтому применяем первое


правило раскрытия скобок, а именно опускаем скобки вместе
830

с плюсом, который стоит перед этими скобками. Но первое


слагаемое, которое в скобках записываем со знаком плюс:

−5 + (2 − 3) = −5 + 2 − 3

Пример 5. Раскрыть скобки в выражении (−5)

Перед скобками стоит плюс, но он не записан по причине


того, что до него не было других чисел или выражений. Наша
задача убрать скобки, применив первое правило раскрытия
скобок, а именно опустить скобки вместе с этим плюсом
(даже если он невидим)

(−5) = −5

Пример 6. Раскрыть скобки в выражении 2a + (−6a + b)

Перед скобками стоит плюс, значит этот плюс опускается


вместе со скобками. То, что было в скобках запишется без
изменений:

2a + (−6a + b) = 2a −6a + b

Пример 7. Раскрыть скобки в выражении 5a + (−7b + 6c) + 3a


+ (−2d)

В данном выражении имеется два места, где нужно раскрыть


скобки. В обоих участках перед скобками стоит плюс, значит
этот плюс опускается вместе со скобками. То, что было в
скобках запишется без изменений:
831

5a + (−7b + 6c) + 3a + (−2d) = 5a −7b + 6c + 3a − 2d

Второе правило раскрытия скобок

Теперь рассмотрим второе правило раскрытия скобок. Оно


применяется тогда, когда перед скобками стоит минус.

Если перед скобками стоит минус, то этот минус опускается


вместе со скобками, но слагаемые, которые были в скобках,
меняют свой знак на противоположный.

Например, раскроем скобки в следующем выражении

5 − (−2 − 3)

Видим, что перед скобками стоит минус. Значит нужно


применить второе правило раскрытия, а именно опустить
скобки вместе с минусом, стоящим перед этими скобками.
При этом слагаемые, которые были в скобках, поменяют свой
знак на противоположный:
832

Мы получили выражение без скобок 5 + 2 + 3. Данное


выражение равно 10, как и предыдущее выражение со
скобками было равно 10.

5 − (−2 − 3) = 10

5 + 2 + 3 = 10

Таким образом, между выражениями 5−(−2−3) и 5+2+3 можно


поставить знак равенства, поскольку они равны одному и
тому же значению:

5 − (−2 − 3) = 5 + 2 + 3

10 = 10

Пример 2. Раскрыть скобки в выражении 6 − (−2 − 5)

Перед скобками стоит минус, поэтому применяем второе


правило раскрытия скобок, а именно опускаем скобки вместе
с минусом, который стоит перед этими скобками. При этом
слагаемые, которые были в скобках, записываем с
противоположными знаками:

6 − (−2 − 5) = 6 + 2 + 5

Пример 3. Раскрыть скобки в выражении 2 − (7 + 3)

Перед скобками стоит минус, поэтому применяем второе


правило раскрытия скобок:

2 − (7 + 3) = 2 − 7 − 3
833

Пример 4. Раскрыть скобки в выражении −(−3 + 4)

Перед скобками стоит минус, поэтому применяем второе


правило раскрытия скобок:

−(−3 + 4) = 3 − 4

Пример 5. Раскрыть скобки в выражении −(−8 − 2) + 16 + (−9


− 2)

Здесь два места, где нужно раскрыть скобки. В первом


случае нужно применить второе правило раскрытия скобок, а
когда очередь доходит до выражения +(−9 − 2) нужно
применить первое правило:

−(−8 − 2) + 16 + (−9 − 2) = 8 + 2 + 16 − 9 − 2

Пример 6. Раскрыть скобки в выражении −(−a − 1)

Перед скобками стоит минус, поэтому применяем второе


правило раскрытия скобок:

−(−a − 1) = a + 1

Пример 7. Раскрыть скобки в выражении −(4a + 3)

Перед скобками стоит минус, поэтому применяем второе


правило раскрытия скобок:

−(4a + 3) = −4a − 3
834

Пример 8. Раскрыть скобки в выражении a − (4b + 3) + 15

Перед скобками стоит минус, поэтому применяем второе


правило раскрытия скобок:

a − (4b + 3) + 15 = a − 4b − 3 + 15

Пример 9. Раскрыть скобки в выражении 2a + (3b − b) − (3c +


5)

Здесь два места, где нужно раскрыть скобки. В первом


случае нужно применить первое правило раскрытия скобок, а
когда очередь доходит до выражения −(3c+5) нужно
применить второе правило:

2a + (3b − b) − (3c + 5) = 2a + 3b − b − 3c − 5

Пример 10. Раскрыть скобки в выражении −a − (−4a) + (−6b)


− (−8c + 15)

Здесь три места, где нужно раскрыть скобки. Вначале нужно


применить второе правило раскрытия скобок, затем первое,
а затем опять второе:

−a − (−4a) + (−6b) − (−8c + 15) = −a + 4a − 6b + 8c − 15

Механизм раскрытия скобок

Правила раскрытия скобок, которые мы сейчас рассмотрели,


основаны на распределительном законе умножения:

a(b+c) = ab + ac
835

На самом деле раскрытием скобок называют ту процедуру,


когда общий множитель умножают на каждое слагаемое в
скобках. В результате такого умножения скобки исчезают.
Например, раскроем скобки в выражении 3×(4+5)

3 × (4 + 5) = 3 × 4 + 3 × 5

Поэтому, если нужно умножить число на выражение в


скобках (или выражение в скобках умножить на число) надо
говорить раскроем скобки.

Но как связан распределительный закон умножения с


правилами раскрытия скобок, которые мы рассматривали
ранее?

Дело в том, что перед любыми скобками стоит общий


множитель. В примере 3×(4+5) общий множитель это 3. А в
примере a(b+c) общий множитель это переменная a.

Если перед скобками нет чисел или переменных, то общим


множителем является 1 или −1, в зависимости от того, какой
знак стоит перед скобками. Если перед скобками стоит плюс,
значит общим множителем является 1. Если перед скобками
стоит минус, значит общим множителем является −1.

К примеру, раскроем скобки в выражении −(3b−1). Перед


скобками стоит минус, поэтому нужно воспользоваться
вторым правилом раскрытия скобок, то есть опустить скобки
вместе с минусом, стоящим перед скобками. А выражение,
которое было в скобках, записать с противоположными
знаками:

−(3b − 1) = −3b + 1
836

Мы раскрыли скобки, воспользовавшись правилом раскрытия


скобок. Но эти же скобки можно раскрыть, воспользовавшись
распределительным законом умножения. Для этого сначала
записываем перед скобками общий множитель 1, который не
был записан:

−1(3b −1)

Минус, который раньше стоял перед скобками относился к


этой единице. Теперь можно раскрыть скобки, применяя
распределительный закон умножения. Для этого общий
множитель −1 нужно умножить на каждое слагаемое в
скобках и полученные результаты сложить.

Для удобства заменим разность, находящуюся в скобках на


сумму:

−1(3b −1) = −1( 3b + (−1) )

Далее умножаем общий множитель −1 на каждое слагаемое


в скобках:

−1(3b −1) = −1(3b + (−1)) = −1 × 3b + (−1) × (−1) = −3b + 1

Как и в прошлый раз мы получили выражение −3b+1. Каждый


согласится с тем, что в этот раз затрачено больше времени
на решение столь простейшего примера. Поэтому разумнее
пользоваться готовыми правилами раскрытия скобок,
которые мы рассматривали в данном уроке:

−(3b − 1) = −3b + 1

Но не мешает знать, как эти правила работают.


837

В данном уроке мы научились ещё одному тождественному


преобразованию. Вместе с раскрытием скобок, вынесением
общего за скобки и приведением подобных слагаемых можно
немного расширить круг решаемых задач. Например:

Раскрыть скобки и привести подобные слагаемые в


следующем выражении:

Здесь нужно выполнить два действия — сначала раскрыть


скобки, а потом привести подобные слагаемые. Итак, по
порядку:

1) Раскрываем скобки:

2) Приводим подобные слагаемые:

В получившемся выражении −10b+(−1) можно раскрыть


скобки:

Пример 2. Раскрыть скобки и привести подобные слагаемые


в следующем выражении:

1) Раскроем скобки:
838

2) Приведем подобные слагаемые. В этот раз для


экономии времени и места, не будем записывать, как
коэффициенты умножаются на общую буквенную часть

Пример 3. Упростить выражение 8m+3m и найти его


значение при m=−4

1) Сначала упростим выражение. Чтобы упростить


выражение 8m+3m, можно вынести в нём общий множитель
m за скобки:

8m+3m = m(8+3)

2) Находим значение выражения m(8+3) при m=−4. Для этого


в выражение m(8+3) вместо переменной m подставляем
число −4

m (8 + 3) = −4 (8 + 3) = −4 × 8 + (−4) × 3 = −32 + (−12) = −44

Задания для самостоятельного решения


Задание 1. Раскройте скобки в следующем выражении:

Показать решение
Задание 2. Раскройте скобки в следующем выражении:

Показать решение
Задание 3. Раскройте скобки в следующем выражении:
839

Показать решение
Задание 4. Раскройте скобки в следующем выражении:

Показать решение
Задание 5. Раскройте скобки в следующем выражении:

Показать решение
Задание 6. Раскройте скобки в следующем выражении:

Показать решение
Задание 7. Раскройте скобки в следующем выражении:

Показать решение
Задание 8. Раскройте скобки в следующем выражении:

Показать решение
Задание 9. Раскройте скобки в следующем выражении:

Показать решение
Задание 10. Раскройте скобки в следующем выражении:

Показать решение
Задание 11. Раскройте скобки в следующем выражении:

Показать решение
840

Задание 12. Раскройте скобки в следующем выражении:

Показать решение
Задание 13. Раскройте скобки в следующем выражении:

Показать решение
Задание 14. Раскройте скобки в следующем выражении:

Показать решение
Задание 15. Раскройте скобки в следующем выражении:

Показать решение
Задание 16. Раскройте скобки в следующем выражении:

Показать решение
Задание 17. Раскройте скобки в следующем выражении:

Показать решение
Задание 18. Раскройте скобки в следующем выражении:

Показать решение
Задание 19. Раскройте скобки в следующем выражении:
841

Показать решение
Задание 20. Раскройте скобки в следующем выражении:

Показать решение
Задание 21. Раскройте скобки в следующем выражении:

Показать решение
Задание 22. Раскройте скобки и приведите подобные
слагаемые в следующем выражении:

Показать решение
Задание 23. Раскройте скобки и приведите подобные
слагаемые в следующем выражении:

Показать решение
842

Простейшие задачи по математике

Рассмотрим несколько простейших задач по уже пройденным


темам. Для этого нам потребуется минимальная
математическая подготовка. В частности, уметь складывать,
вычитать, умножать и делить, находить доли от чисел, уметь
строить соотношения и выполнять элементарные
тождественные преобразования.

Задачи, приведенные в данном уроке, достаточно легкие для


восприятия и понимания. Потребуется только небольшая
сноровка, чтобы понять каким из изученных инструментов
воспользоваться для решения поставленной задачи. Изучить
что-либо это одно дело, а вот применить на практике —
другое.

Содержание урока

 Запись выражений содержащих сложение и вычитание


 Запись выражений содержащих умножение и деление
 Определить стоимость, длину, массу, время, скорость
 Графическое описание задачи
 Нахождение НОД и НОК
 Перевод единиц измерения
 Задачи для самостоятельного решения
 Список сокращений

Запись выражений содержащих сложение и вычитание

Задача 1. В чашке для фруктов в которой лежало пять яблок,


положили еще три яблока. Через некоторое время с чашки
843

взяли два яблока. На чашке осталось шесть яблок. Записать


выражение, описывающее это движение яблок.

Решение

5+3−2=6

Задача 2. На поле обработано 20 грядок моркови и 15 грядок


свеклы. Итого обработано 35 грядок. Записать следующие
выражения в которых содержатся сложение и вычитание:

1. Выражение, описывающее сколько грядок обработано


всего. Число отвечающее за общее количество грядок
расположить в левой части равенства;
2. Выражение, описывающее, что свеклы обработано на
пять грядок меньше, чем моркови. Число отвечающее за
свеклу расположить в левой части равенства;
3. Выражение, описывающее, что моркови обработано на
пять грядок больше, чем свеклы. Число отвечающее за
морковь расположить в левой части равенства;

Решение

35 = 20 + 15

15 = 20 − 5

20 = 15 + 5

Задача 3. Автомобиль за 3 дня проехал 980 км. В пятницу и


субботу он проехал 725 км. Сколько километров проезжал
844

автомобиль в каждый из этих дней, если в субботу он


проехал больше, чем в воскресенье на 123 км?

Решение

Узнáем сколько километров проехал автомобиль в


воскресенье. Для этого из общего пути (980 км) вычтем путь
который автомобиль проехал в пятницу и субботу (725 км)

980 − 725 = 255 км в воскресенье

В условии сказано, что в субботу автомобиль проехал на 123


км больше, чем в воскресенье. Поэтому к пути, который
автомобиль проехал в воскресенье (255 км) нужно прибавить
123 км. Так мы получим путь, который автомобиль проехал в
субботу

255 + 123 = 378 км в субботу

Теперь узнаем сколько километров автомобиль проехал в


пятницу. Для этого вычтем из общего пути (980) те пути,
которые автомобиль проехал в субботу и воскресенье. Для
удобства эти два пути можно сложить и полученный
результат вычесть из 980

980 − (378 + 255) = 980 − 633 = 347 км в пятницу

Теперь проверим, правильно ли решена задача. Для этого


сложим все пути и посмотрим равна ли сумма 980 км

347 + 378 + 255 = 980

980 = 980
845

Задача 4. За три рабочие смены фабрика изготовила 1680


метров ткани. Первая и вторая смены изготовили вместе 970
метров ткани, вторая и третья — 1060 метров. Сколько
метров ткани изготовила каждая смена?

Решение

Узнаем сколько метров ткани изготовила первая смена. Для


этого вычтем из общего количества метров то количество,
которое изготовила вторая и третья смены (1060м)

1680 − 1060 = 620м (изготовила первая смена)

Узнаем сколько метров ткани изготовила третья смена. Для


этого вычтем из общего количества метров то количество,
которое изготовила первая и вторая смены (970м)

1680 − 970 = 710м (изготовила третья смена)

Узнаем сколько метров ткани изготовила вторая смена. Для


этого вычтем из общего количества метров то количество,
которое изготовила первая и третья смены (620м и 710м).
Для удобства эти два количества можно сложить и
полученный результат вычесть из 1680

1680 − (620 + 710) = 1680 − 1330 = 350м (изготовила вторая


смена)

Проверим правильно ли решена задача. Для этого сложим


все количества и посмотрим равна ли сумма 1680м

620 + 350 + 710 = 1680

1680 = 1680
846

Задача 5. Имеется следующее равенство

5+5+2=8+4

Обе части равенства (и левая и правая) равны 12. Прибавим


к левой части равенства число 3

5+5+2+3≠8+4

Равенство сразу нарушилось. Левая часть не равна правой


части, поскольку левая часть теперь равна 15, а правая 12.
Чтобы сохранить равенство, прибавим к правой части
равенства число 3

5+5+2+3=8+4+3

Знак «неравно» исчез. Теперь к числу 4, которое


располагается в правой части равенства прибавим единицу

5 + 5 + 2 + 3 ≠ 8 + (4 + 1) + 3

Равенство снова нарушилось. Левая часть не равна правой


части, поскольку правая часть теперь равна 16, а левая 15.

Попробуем снова «восстановить справедливость» между


левой и правой частью. Для этого можно прибавить единицу
к левой части. Но можно также вычесть единицу из любого
числа, располагающегося в правой части. Давайте не трогая
левую часть, вычтем единицу из числа 8, которое в правой
части:

5 + 5 + 2 + 3 = (8 − 1) + (4 + 1) + 3
847

Задача 6. На сколько единиц число 78 больше, чем число


63?

Решение

Вычтем из 78 число 63

78 − 63 = 15

Число 78 больше числа 63 на 15 единиц.

Задача 7. На сколько единиц число 88 меньше числа 105

Решение

Вычтем из 105 число 88

105 − 88 = 17

Число 88 меньше числа 105 на 17 единиц

Задача 8. На сколько число 65 больше, чем число 48 и


меньше чем число 95

Решение

Вычтем из 65 число 48

65 − 48 = 17

Вычтем из 95 число 65

95 − 65 = 30
848

Число 65 больше числа 48 на 17 единиц, и меньше числа 95


на 30 единиц.

Запись выражений содержащих умножение и деление

Задача 1. В три коробки поровну разложили 90 чашек. В


результате в каждой коробке оказалось 30 чашек. Записать
выражение, которое описывает, что 90 чашек разложено в 3
коробки. Далее выполнить действие в этом выражении.

Решение

90 : 3

90 : 3 = 30

30 = 30

Задача 2. В коробки упаковали 60 тарелок по 6 тарелок в


каждую. В результате получилось 10 коробок. Записать
выражение, которое описывает, что 60 тарелок разложено в
коробки по 6 тарелок в каждую. Далее выполнить действие в
этом выражении.

Решение

60 : 6

60 : 6 = 10

10 = 10
849

Задача 3. Имеется 10 коробок с тарелками. В каждой по 6


тарелок. Если расфасовать все коробки, получается 60
тарелок. Записать выражение, которое описывает, что в
результате расфасовки всех коробок получается 60 тарелок.

Решение

6 × 10 = 60 тарелок

Здесь следует сделать небольшое замечание. При


построении выражения, содержащего произведение,
желательно заранее разобраться, что будет множимым, а
что множителем. По традиции сначала записывают
мнóжимое, а потом мнóжитель.

Например, если нужно увеличить число 2 в три раза, то


нужно записывать 2 × 3. В этом выражении мнóжимым
является число 2, а мнóжителем число 3.

В условии задачи было сказано, что после расфасовки


получилось 60 тарелок. Значит конечной целью является
получение этих сáмых 60 тарелок. А их можно получить
путём увеличения шести тарелок в десять раз. То есть
умножить 6 на множитель 10. Значит роль мнóжимого играют
тарелки, а роль мнóжителя — коробки. В результате получим
тарелки, количество которых будет увеличено в 10 раз от
изначального.
850

Не следует думать, что на рисунке представлены все 60


тарелок. Это всего лишь модель, описывающая что в
результате умножения получаются тарелки, а не коробки.

Конечно, от перестановки мест сомножителей произведение


не меняется, но если поставить число 10 на первое место, а
число 6 на второе, то получится выражение 10 × 6. В этом
выражении роль мнóжимого играют коробки, а роль
мнóжителя — тарелки. Тогда получится не 60 тарелок, а 60
коробок, что будет нарушать логику задачи:

Как и с предыдущим рисунком, не следует думать, что на нем


представлены все 60 коробок, получающиеся в результате
умножения. Это всего лишь модель, описывающая что в
результате умножения получаются коробки, а не тарелки.

Как было сказано выше, от перестановки мест сомножителей


произведение не меняется, и мы можем записывать
сомножители в любом порядке, поскольку ответ к задаче не
851

поменяется. Тем не менее, слежение за порядком


сомножителей позволяет хорошо осмыслить задачу и понять
её суть.

С другой стороны, традиция записывать множимое первым,


сохранилась наверное только в нашей стране. В
большинстве других стран сначала записывают множитель, а
затем множимое. И это даже правильнее с точки зрения
настоящей математики.

Например, если нам встретится запись 5 см, то мы читаем


«пять сантиметров», а не «сантиметров пять». Пятерка в
данном случае является множителем — числом, которым
увеличивают один сантиметр в пять раз. Под сокращением
«см» подразумевается 1 сантиметр:

5 см = 5 × 1 см

В повседневном общении мы часто употребляем


произведение, порой не замечая этого. И произносим мы
сначала множитель, а затем множимое. Примеры: «пять
конфет», «сто рублей», «десять тюльпанов». Мы не говорим
«конфет пять», «рублей сто», «тюльпанов десять».

Вспомните урок «Буквенные выражения». Этот урок являлся


базовым для изучения алгебры. В нем мы затронули понятия
коэффициента — множителя, стоящего перед переменной.
Этот коэффициент мы записывали раньше переменной,
например 3a, 2x, 7y. Мы не записывали a3, x2, y7. Первая
запись правильнее, и она более аккуратнее и красивее с
точки зрения эстетики. В последствии, при изучении алгебры
и высшей математики, вы чаще будете замечать, что
множитель стоит на первом месте.
852

Большинство учителей, воспитанных по советским


учебникам, скорее всего снизят вам оценку, если вы будете
записывать сначала множитель, а затем множимое. При
решении задач, дабы избежать нападок со стороны этих
учителей и других педагогов, советуем пользоваться старой
схемой умножения:
множимое × множитель = произведение. А в
последствии переходя к алгебре, множитель можно
записывать раньше.

Задача 4. В тетради 18 листов. Сколько можно сделать


тетрадей из 54 таких же листов?

Решение

18 листов это одна тетрадь. Чтобы узнать сколько таких


тетрадей можно сделать из 54 листов, нужно эти 54 листа
сгруппировать по 18 листов. Для этого необходимо
разделить 54 на 18. Это позволяет узнать сколько тетрадей
можно сделать из 54 листов:

54 : 18 = 3 тетради

Задача 5. Суммарно (вместе) в нескольких одинаковых


тетрадях 72 листа. Каждая тетрадь имеет 18 листов.
Запишите выражение, позволяющее узнать сколько всего
тетрадей имеется.

Решение
853

Если в одной тетради 18 листов, то для того чтобы узнать


сколько таких же 18 листов (целых тетрадей) в 72 листах,
нужно 72 разделить на 18

72 : 18 = 4 (тетради)

Задача 6. Суммарно (вместе) в трёх одинаковых тетрадях 75


листов. Сколько листов в одной тетради?

Решение

Тетради все одинаковые. Если разделить 75 на количество


тетрадей, то есть на 3, мы узнаем сколько листов приходится
на одну тетрадь:

75 : 3 = 25 листов

Задача 7. Отцу 46 лет, сыну 23 года. Отец вдвое старше


сына. Записать выражение которое описывает, что отец
вдвое старше сына.

Решение

Записываем возраст отца и через знак равенства пишем, что


возраст отца вдвое больше возраста сына:

46 = 23 × 2

Выполним действие в правой части равенства, чтобы


удостовериться в правильности выражения — значок
равенства должен оправдывать свое положение:

46 = 23 × 2
854

46 = 46

Задача 8. Маме 36 лет, дочери 12 лет. Дочь младше матери


в три раза. Записать выражение, описывающее что дочь
втрое младше матери.

Решение

Записываем возраст дочери и через знак равенства пишем,


что она младше матери в три раза

12 = 36 : 3

Выполним действие в правой части равенства — получим


тождество:

12 = 12

Определить стоимость, длину, массу, время, скорость

Задача 1. Девять блокнотов стоят 162 рубля. Сколько стоят


пять таких же блокнотов?

Чтобы решать подобные задачи, сначала нужно определить


стоимость одной единицы товара. Далее воспользоваться
умножением и определить стоимость нескольких единиц
товара. В данном случае 162 рубля нужно поровну раскидать
на 9 блокнотов. Так мы узнáем сколько стóит один блокнот:

162 : 9 = 18 руб.

Получили стоимость одной единицы товара. То есть


стоимость одного блокнота составляет 18 рублей. Далее,
855

чтобы узнать стоимость пяти таких же блокнотов, нужно 18


умножить на множитель 5

18 × 5 = 90 руб.

Задача 2. Восемь журналов стоят 176 рублей. Определить


сколько журналов можно купить на 66 рублей.

Определим стоимость одного журнала. Для этого разделим


общую цену 176 рублей на восемь журналов:

176 : 8 = 22 руб.

22 рубля — стоимость одного журнала. Определим сколько


журналов можно купить на 66 рублей. Для этого узнаем
сколько раз 66 рублей содержит по 22 рубля. Другими
словами, разделим 66 рублей на стоимость одного журнала:

66 : 22 = 3 журнала.

Задача 3. Из 6 рулонов ткани сшили 3 рубашки. Определить


сколько рулонов пошлó на одну рубашку

Решение

Чтобы определить сколько рулонов пошлó на одну рубашку,


нужно 6 рулонов разделить на сшитое количество рубашек,
то есть на 3

6:3=2
856

Задача 4. Из 15 метров тюля сшили 5 занавесок. Определить


сколько занавесок можно сшить из 42 метров тюля.

Решение

Узнáем сколько метров тюля уходит на шитье одной


занавески. Для этого разделим 15 метров на количество
сшитых занавесок, то есть на 5

15 : 5 = 3 метра

Три метра уходит на шитье одной занавески. Чтобы узнать


сколько занавесок можно сшить из 42 метров тюля, нужно
эти 42 метра разделить на 3 метра

42 : 3 = 14 занавесок

Задача 5. Из 3 кг муки испекли 6 булок хлеба. Определить


сколько булок можно испечь из 30 кг муки.
857

Решение

Узнáем сколько килограммов муки уходит на одну булку


хлеба. Для этого разделим 3 килограмма на 6 булок

3 : 6 = 0,5 кг на булку

На одну булку уходит 0,5 кг муки. Узнаем сколько булок


можно получить из 30 кг муки. Для этого 30 кг разделим на
0,5

30 : 0,5 = 60 булок

Задача 6. Куплено 5 одинаковых пакетов с картофелем,


общая масса которых 15 кг. Определить массу одного пакета

Решение

15 : 5 = 3 кг

Задача 7. Отец купил 5 одинаковых пакетов с картофелем,


общая масса которых 20 кг. Сын помог отцу донести 2
пакета. Сколько килограммов картофеля нёс сын?

Решение

Масса одного пакета:

20 кг : 5 = 4 кг

Масса пакетов, которые нес сын:

4 кг × 2 = 8 кг.
858

Задача 8. Скорость вертолета 250 км/ч, а скорость самолета


в 4 раза больше. Определить скорость самолета.

Решение

Если скорость самолета в 4 раза больше скорости


вертолета, достаточно умножить скорость вертолета на 4.
Так мы получим скорость самолета:

250 км/ч × 4 = 1000 км/ч

Задача 9. На поезде за 6 часов проехали 390 км, каждый час


поровну. Определить сколько километров проезжали на этом
поезде за 1 час.

Решение

Чтобы определить сколько километров поезд проезжал за


один час, достаточно 390 разделить на 6, понимай раскидать
поровну расстояние на затраченное время:

390 км : 6 = 65 км каждый час

Задача 10. Тюльпан стоит 25 рублей. Какое наибольшее


число тюльпанов можно купить на 160 рублей?

Решение

Разделим 160 на 25
859

На 160 рублей можно купить максимум 6 тюльпанов, плюс


останется сдача в 10 рублей.

Графическое описание задачи

Некоторые задачи полезно описáть графически в виде схем,


рисунков или таблиц. Это позволяет быстрее найти решение
и понять суть задачи. Бывают также задачи, которые трудно
решить, если перед глазами нет её графического
представления.

Графическое описание задачи это творческий процесс и


здесь всё зависит от вашей фантазии.

Решим несколько простейших задач, которые можно описáть


с помощью рисунков.

Задача 1. В мастерской было 2 куска материи длиной 96


метров и 84 метра. Из них сшили пальто. Из второго куска
вышло на 3 пальто меньше, чем из первого куска. Сколько
пальто сшито из каждого куска?

Решение

Сначала нужно узнать сколько метров материи расходуется


на одно пальто. Узнав это число, мы смогли бы разделить на
это число 96 метров и 84 метра, и таким образом узнать
сколько пальто сшито из каждого куска. Но как это сделать?
860

Узнáем на сколько 96 метров больше, чем 84 метра. Для


этого из 96 вычтем 84

96 м − 84 м = 12 м 

96 метров больше, чем 84 метра на 12 метров. В условии


задачи сказано, что из второго куска вышло на 3 пальто
меньше, чем из первого куска. Это значит, что из первого
куска наоборот вышло на три куска больше, чем из второго.
На эти три пальто приходятся найденные нами 12 метров
материи. Если мы разделим 12 метров на эти 3 пальто, то
узнаем сколько метров материи расходуется на одно пальто:

12 : 3 = 4 метра на одно пальто

Теперь можно узнать сколько пальто сшито из каждого куска


материи. Для этого поочередно разделим числа 96 и 84 на 4
метра

96 : 4 = 24 пальто из первого куска

84 : 4 = 21 пальто из второго куска

Можно сделать проверку, действительно ли из второго куска


вышло на 3 пальто меньше, чем из первого куска. Вычтем из
24 число 21

24 − 21 = 3

Эту задачу можно изобразить графически. Представим 96


метров и 84 метра в виде двух линий — вторая короче
первой:
861

Нарисуем красную линию поверх этих двух линий так, чтобы


эта линия пересекла 96 метров на участке где заканчиваются
84 метра:

Теперь на первом куске материи после красной линии


нарисуем три пальто. Ведь из первого куска материи вышло
на 3 пальто больше:
862

Ну а дальше при внимательном рассмотрении рисунка можно


понять, что нужно сделать. Остаток первой материи (после
красной линии) нужно поровну разделить на 3 пальто и тем
самым получить число метров, расходуемых на одно пальто.
Остаток первой материи (после красной линии) можно найти
путем вычитания из 96 метров 84 метра.
863

Теперь решать задачу намного удобнее. 12 метров делят на


3 пальто и определяют сколько метров материи расходуется
на одно пальто.

Затем оба куска материи делят на 4 и определяют сколько


пальто сшито из каждого куска.

Задача 2. За 4 часа теплоход прошел 136 км. Сколько


километров он пройдет за 8 часов, двигаясь с той же
скоростью?

Решение

Найдем скорость теплохода. Вспоминаем, что скорость это


расстояние пройденное телом (человеком, машиной,
теплоходом) за 1 час, 1 минуту или 1 секунду. Чтобы найти
скорость, нужно пройденное расстояние разделить на время
движения:

136 : 4 = 34 км/ч
864

Значит за один час теплоход проходит 34 километра. В


задаче сказано, что теплоход двигается с одинаковой
скоростью. Это позволяет нам узнать сколько таких 34
километра он пройдет восемь раз (за 8 часов)

34 км × 8 = 272 км

К этой задаче также можно сделать рисунок. Особенно он


полезен был бы в случае, если вспомнить что такое скорость
так и не удалось.

Нарисуем 136 километров в виде линии:

Сверху над этой линией нарисуем четыре линии, которые


будут изображать 4 часа. Они в свою очередь будут нам
подсказывать сколько километров теплоход проходит в
течении часа

Поскольку скорость теплохода одинаковая в каждом часе, мы


можем дорисовать еще 4 часа, тем самым делая себе
подсказку с пройденным расстоянием:
865

Внимательное рассмотрение рисунка позволяет понять, что


расстояние увеличилось в два раза. Мы можем записать
произведение 136×2 равное 272 километрам. Можем также
дорисовать нижнюю линию, изображающую пройденное
расстояние:

136 × 2 = 272 км
866

Задача 3. Путь от одной станции до другой товарный поезд


прошел за 9 часов, а пассажирский за 6 часов. Чему равна
скорость пассажирского поезда, если скорость товарного
поезда равна 40 км/ч?

Решение

Вопрос задачи состоит в том, чтобы найти скорость


пассажирского поезда. Чтобы найти скорость пассажирского
поезда, нужно пройденное им расстояние разделить на
время его движения. Но дело в том, что пройденное
расстояние нам неизвестно. Известно лишь его время
движения — 6 часов.

Зато в задаче есть подсказка, что товарный поезд прошел то


же самое расстояние за 9 часов и его скорость была 40 км/ч.
Это позволяет нам узнать расстояние между станциями.
Если за один час товарный поезд проходит 40 километров, то
за 9 часов он прошел в девять раз больше:

40 км × 9 = 360 км.

Теперь нам известно расстояние между станциями. Оно


равно 360 километрам. Это позволяет без проблем найти
скорость движения пассажирского поезда. Напомним, что для
этого нужно пройденное расстояние (360км) разделить на
время движения пассажирского поезда (6ч)

360 : 6 = 60 км/ч

Нарисуем схему к данной задаче. В первую очередь


изобразим время движения товарного поезда в виде девяти
867

линий. Эти же линии будут изображать сколько километров


товарный поезд проходит в течении часа:

Ниже этих линий нарисуем сплошную линию, которая будет


изображать пройденное этим поездом расстояние

Ниже от сплошной линии, изображающей расстояние,


нарисуем шесть линий, которые изображают время движения
пассажирского поезда:
868

Если внимательно посмотреть на получившийся рисунок,


можно понять, что делать дальше. Можно сложить все
расстояния, пройденные товарным поездом в течении
девяти часов движения (40 км в каждом часе) и получить
длину всего пути. Далее полученный путь разделить на
время движения пассажирского поезда (6 часов) и получить
скорость его движения.
869

Задача 4. В школе-интернате, имеющей 250 воспитанников,


запасено 22500 кг картофеля на 180 дней. На сколько дней
сделан запас картофеля в другой школе-интернате, если при
той же норме на 160 воспитанников запасено 9600 кг
картофеля?

Решение

Сначала разберемся с первой школой. В ней 250


воспитанников. Для них запасено 22500 кг картофеля на 180
дней. Сделаем рисунок к задаче в виде таблицы. Таблицы
это еще один полезный инструмент, который позволяет
упорядочить данные в задаче и быстрее найти решение к
ней

Определим сколько килограммов картофеля приходится на


один день. Для этого разделим 22500 кг на 180 дней

22500 кг : 180 д = 125 кг на один день

Дополним нашу таблицу новым значением. Вместо


вопросительного знака вставим найденные 125 килограммов,
которые приходятся на один день
870

Итак, в день 250 воспитанникам выделяется 125


килограммов картофеля.

Определим сколько килограммов картофеля приходится на


одного воспитанника в день. Для этого 125 килограммов
разделим на 250 воспитанников

125 кг : 250 в = 0,5 кг в день на одного воспитанника

Дополним нашу таблицу новым значением. Вместо


вопросительного знака вставим найденные 0,5 килограммов,
которые приходятся на одного воспитанника в день
871

Итак, 0.5 кг это масса картофеля, выделяемого на одного


воспитанника в день. Эта же масса является нормой
картофеля на одного воспитанника в день

В задаче сказано, что во второй школе при этой же норме


выделено 9600 кг картофеля на 160 воспитанников. Если мы
умножим 0,5 килограммов картофеля на 160 воспитанников,
то узнаем сколько килограммов картофеля приходится на
160 воспитанников в день

0,5 кг × 160 в = 80 кг картофеля на 160 воспитанников

А если мы разделим 9600 килограммов на 80 килограммов,


то узнаем на сколько дней хватит выделенного картофеля во
второй школе. То есть получим ответ к задаче, узнав на
сколько дней сделан запас картофеля во второй школе-
интернате

9600 кг : 80 кг = 120 дней


872

Нахождение НОД и НОК

Задача 1. Имеется 42 конфеты. Записать следующие


выражения, содержащие деление:

 Выражение в котором 42 конфеты разделены на 2 детей;


 Выражение в котором 42 конфеты разделены на 3 детей;
 Выражение в котором 42 конфеты разделены на 7 детей;
 Выражение в котором 42 конфеты разделены на 14
детей;
 Выражение в котором 42 конфеты разделены на 21
детей;
 Выражение в котором 42 конфеты разделены на 42
детей;

Решение

 42 : 2 = 21
 42 : 3 = 14
 42 : 7 = 6
 42 : 14 = 3
 42 : 21 = 2
 42 : 42 = 1

Задача 2. На улице играются трое детей. У нас имеются 24


конфеты марки «ласточка» и 18 конфет марки
«буревестник». Нужно раздать эти конфеты детям так, чтобы
каждому досталось конфет поровну.

Решение
873

Сначала разделим 24 конфеты марки «ласточка» на троих


детей:

24 : 3 = 8 (конфет марки «ласточка» каждому ребенку)

Далее разделим 18 конфет марки «буревестник» на тех же


троих детей:

18 : 3 = 6 (конфет марки «буревестник» каждому ребенку)

Каждому ребенку досталось 8 конфет марки «ласточка» и 6


конфет марки «буревестник».

Задача 3. Имеется 24 конфеты марки ласточка и 18 конфет


марки буревестник. Какое наибольшее число одинаковых
подарков можно составить из этих конфет?

Решение

Эта задача похожа на предыдущую. В предыдущей мы


разделили эти конфеты на троих детей. Но эти конфеты
могли быть разделены не только на троих, но и на двоих,
четверых, шестерых.

В данном случае, вопрос состоит в том, чтобы определить


наибольшее число детей на которых можно было бы
разделить эти конфеты. При этом разделить так, чтобы
каждому ребенку досталось конфет поровну и чтобы у нас
ничего не осталось.

В таких случаях нужно находить наибольший общий


делитель (НОД) двух  чисел — в нашем случае чисел 24 и
874

18. Напомним, что НОД это наибольшее число, на которое


оба числа делятся без остатка. А это то, что нам нужно.

Найдем НОД чисел 24 и 18

Значит из 24 конфет марки ласточка и 18 конфет марки


буревестник можно составить 6 одинаковых подарков.

Мы можем раскидать (разделить) конфеты каждой марки на


шесть подарков и узнать сколько конфет каждой марки будет
в каждом подарке:

24 : 6 = 4 (конфеты марки «ласточка»)

18 : 6 = 3 (конфеты марки «буревестник»)

В каждом подарке будет 4 конфеты марки ласточка и 3


конфеты марки буревестник.

Задача 4. Имеется 60 яблок и 40 груш. Какому наибольшему


количеству детей можно поровну раздать эти фрукты?
Сколько яблок и груш получит каждый ребенок?
875

Решение

Дети в данном случае являются наибольшем общим


делителем. Наша задача найти этот НОД, чтобы раздать
поровну 60 яблок и 40 груш.

Наибольшим общим делителем чисел 60 и 40 является


число 20. Значит 60 яблок и 40 груш могут быть поровну
розданы 20 детям.

Раздадим 60 яблок:

60 : 20 = 3 (яблока каждому ребенку)

Раздадим 40 груш:

40 : 20 = 2 (груши каждому ребенку)

Каждый ребенок получил три яблока и две груши.

Задача 5. Материал ткани продается по 3 метра либо по 4


метра. Какое наименьшее число метров должно быть в
рулоне, чтобы материал можно было продать без остатка?

Решение

Чтобы продать материал без остатка, число его метров


должно без остатка делится на 3 и на 4 метра. Мы знаем, что
число которое без остатка делится на оба числа, называется
наименьшим общим кратным эти чисел. В этой задаче
именно этот случай. Чтобы ответить на вопрос задачи, нужно
найти НОК чисел 3 и 4.
876

НОК чисел 3 и 4 равен 12. Значит наименьшее число метров


материала должно быть 12, чтобы его можно было продать
без остатка.

Задача 6. Маленькая коробка вмещает 24 карандаша, а


большая 30 карандашей. Определить наименьшее число
карандашей, которые могут быть разложены, как в маленькие
коробки, так и в большие.

Решение

Наименьшее общее кратное чисел 24 и 30 равно 120. Значит


120 карандашей могут быть разложены, как в маленькие
коробки, так и в большие.

В случае, если мы разложим 120 карандашей в маленькие


коробки, нам потребуется 5 маленьких коробок:

120 : 24 = 5

В случае, если мы разложим 120 карандашей в большие


коробки, нам потребуется 4 большие коробки:

120 : 30 = 4

Перевод единиц измерения

Задача 1. Выразить 3 минуты в секундах.

В одной минуте 60 секунд, а в трех минутах 180 секунд:

60 × 3 = 180 (секунд)

3 минуты = 180 секунд


877

Задача 2. Выразить 10 минут в секундах.

В одной минуте 60 секунд, а в десяти минутах 600 секунд:

60 × 10 = 600 (секунд)

10 минут = 600 секунд

Задача 3. Выразить 8 минут в секундах.

В одной минуте 60 секунд, а в восьми минутах 480 секунд:

60 × 8 = 480 (секунд)

8 минут = 480 секунд

Задача 4. Выразить 2 минуты и 3 секунды в секундах.

Две минуты это 120 секунд плюс 3 секунды, которые никуда


переводить не нужно:

60 × 2 = 120

120 + 3 = 123 (секунды)

2 мин 3 с = 123 с

Задача 5. Выразить 15 минут и 47 секунд в секундах.

15 минут это 900 секунд плюс 47 секунд, которые никуда


переводить не нужно:
878

60 × 15 = 900 секунд

900 + 47 = 947 секунд

15 мин 47 с = 947 с

Задача 6. Выразить 3 центнера и 45 килограмм в


килограммах.

Сначала переведем 3 центнера в килограммы. Один центнер


составляет 100 килограмм, а три центнера составляют 300
килограмм. Плюс у нас имеется еще 45 килограмм. Поэтому
нужно сложить 300 и 45. В итоге получим 345 килограмм:

Задача 7. Выразить 4 килограмма и 654 грамма в граммах

Сначала переведем 4 килограмма в граммы. 1 килограмм


составляет 1000 грамм, а 4 килограмма составляют 4000
грамм. Плюс у нас имеется еще 654 грамма. Поэтому нужно
сложить 4000 и 654. В итоге получим 4654 грамма:

Задача 8. Выразить 20 секунд в минутах. Ответ записать в


виде обыкновенной дроби:
879

Выражение означает, что минута разделена на 60 равных


частей (на 60 секунд), и из этих частей взято 20 частей.

Задача 9. Выразить 15 секунд в минутах. Ответ записать в


виде обыкновенной дроби:

Задача 10. Выразить 45 минут в часах. Ответ записать в


виде обыкновенной дроби:

Выражение означает, что час разделен на 60 частей, и из


этих 60 частей взято 45 частей. А 45 частей из 60 составляет
45 минут.

Задача 11. Масса лошади 500 кг. Выразить эту массу в


тоннах. Ответ записать в виде обыкновенной дроби:

Задача 12. Записать 6 минут и 15 секунд в виде смешанного


числа
880

6 минут это целая часть смешанного числа. 15 секунд это

пятнадцать секунд из шестидесяти секунд, поэтому будет


дробной частью

Задача 13. Записать 43 метра и 5 сантиметров в виде


смешанного числа

43 метра это целая часть смешанного числа. 5 сантиметров

это пять сантиметров из ста сантиметров, поэтому будет


дробной частью смешанного числа:

Задача 14. К 6 ц 87 кг прибавить 2 ц 45 кг

Запишем задание в виде выражения:

Для удобства переведем данные величины в килограммы,


получим 687 кг и 245 кг. Сложим эти килограммы, получим
932 кг. Затем распишем центнеры и остатки килограмм по
отдельности:

Второй способ с помощью смешанных чисел:


881

Задача 15. Найти значение следующего выражения:

Решение

Решим эту задачу с помощью смешанных чисел:

Вычитание смешанных чисел было выполнено следующим


образом: целые части были вычислены отдельно от
дробных. Так, от трех вычесть два получилась единица,
которая стала целой частью получившегося смешанного

числа. Значение выражения стало равно дроби

, которая стала дробной частью смешанного числа.

Но выражение могло быть вычислено путем


перевода смешанных чисел в неправильные дроби.
Выглядело бы это решение следующим образом:

Конечно это решение выглядит более громоздким, чем


первое решение. Первое выглядит намного короче и
аккуратнее:
882

Но надо иметь ввиду, что при решении подобных задач


иногда случается так, что дробная часть уменьшаемого
оказывается меньше дробной части вычитаемого. В таком
случае, целесообразнее искать обходные пути, чтобы
сделать решение более короче. В противном случае, задача
может разрастись на половину листа.

Задача 16. Найти значение следующего выражения:

Решение

Теперь попробуем решить эту задачу с помощью смешанных


чисел

Мы видим, что дробная часть уменьшаемого меньше


дробной части вычитаемого. Если попытаться вычислить
целые и дробные части по отдельности, то мы получим
довольно громоздкое решение:
883

А если сначала перевести смешанные числа в неправильные


дроби, то решение можно записать намного короче:

Также можно воспользоваться еще одним методом, которым


часто пользуются в школах. Суть метода в том, что с целой
части уменьшаемого снимают одну единицу, и представляют
ее в виде дроби с таким же знаменателем, как у дробной
части уменьшаемого. Затем эту дробь складывают с дробной
частью уменьшаемого.

Запишем уменьшаемое в следующем виде:

С целой части 43 сняли одну единицу — целая часть теперь


представляет собой сумму чисел 42 и 1. Теперь единицу
представляем в виде дроби с таким же знаменателем, как и у

дроби

Теперь складываем дроби и — получаем следующее


выражение:
884

Уменьшаемое, которое было раньше теперь приняло

вид . Мы видим, что проблема уменьшаемого исчезла


— оно больше вычитаемого. Это позволяет нам вычислить
целые и дробные части по отдельности и получить короткое
решение:

При решении столь простых задач, вовсе необязательно


прибегать к дробям и тем более к таким способам, которые
мы сейчас рассматриваем. Ваша цель — уметь применить
полученные знания на практике.

Задача 17. Найти значение следующего выражения:

Решение

Задача 18. Найти значение следующего выражения:

Решение:

Задача 19. Выполнить сложение:


885

Решение

Получили 1325 минут. Выразим этот ответ в часах и минутах.


Для этого «вытащим» часы из 1325.

60 минут это один час. А чтобы понять сколько таких 60


минут (часов) содержится в 1325 минутах, нужно 1325
разделить на 60

Получаем ответ, что 1325 минут это 22 часа и 5 минут

Задача 18. Первый в мире советский искусственный спутник


Земли имел массу 83 кг 600 г. Масса второго искусственного
спутника была на 424 кг 700 г. больше массы первого и на
818 кг 700 г меньше массы третьего искусственного спутника
Земли. Определить массу третьего искусственного спутника
Земли.

Решение
886

Найдем массу второго спутника. Она больше массы первого


спутника на 424 кг 700 г. Прибавим к массе первого спутника
еще 424 кг и 700 граммов

Далее сказано, что масса второго спутника меньше на 818 кг


и 700 граммов. Если к массе второго спутника прибавить 818
кг и 700 граммов, мы получим массу третьего спутника

Значит третий искусственный спутник имеет массу 1327


килограмм.

Задачи для самостоятельного решения


Задача 1. Две одинаковые бочки наполнены водой. Когда из
них взяли 16 вёдер воды, по 9 литров в каждом, в первой
бочке осталось 34 ведра воды, а во второй 20 вёдер. Сколько
литров воды взяли из каждой бочки?
Показать решение
Задача 2. 15 растений дикой редьки дают в год 180000
семен. Сколько семян в год дадут 80 растений дикой редки?
Показать решение
887

Задача 3. За 4 кг сахара и 5 кг яблок заплатили 6,8 р. Сколько


стоит 1 кг сахара, если 1 кг яблок стоит 0,6 р?
Показать решение
Задача 4. Для библиотеки требуется переплести 3240 книг.
Одна мастерская сможет выполнить заказ за 12 дней, другая
за 15 дней, а третья за 20 дней. За сколько дней выполнят
этот заказ три мастерские, работая одновременно?
Показать решение
Задача 5. Хозяйка купила 4,5 кг крупы по цене 12,8 р. за
килограмм. Сколько крупы по цене на 3,2 р за килограмм
большей можно купить на эти деньги?
Показать решение
Задача 6. Два маляра покрасили вместе 144 рамы. Один из
них работал 6 дней, по 7 ч в день, а другой — 5 дней, по 6 ч в
день. Сколько рам покрасил каждый маляр, если за 1 ч
работы они красили одинаковое количество рам?
Показать решение
Задача 7. В хлебный отдел магазина привезли 10 лотков
чёрного хлеба и 14 лотков белого хлеба. Количество буханок
на одном лотке и количество буханок на другом лотке
одинаковое. Всего в отдел привезли 288 единиц хлеба (и
чёрного и белого). Сколько буханок чёрного хлеба и сколько
батонов белого хлеба привезли в хлебный отдел?
Показать решение
Задача 8. За 7 м шёлка заплатили на 450 р. больше, чем за 4
м такого же шёлка. Сколько стоит 1 м шёлка?
Показать решение
888

Задача 9. В шкафу стоят 5 литровых банок с пшеном и 3


литровые банки с горохом, причем пшена на 2210 г больше,
чем гороха. Сколько в шкафу пшена и сколько гороха?
Показать решение
Задача 10. На мельницу привезли 6360 кг пшеницы. При
размоле пшеницы отходы составили 860 кг. Муку насыпали
поровну в мешки и погрузили на три машины. На первую
погрузили 28 мешков, на вторую — 32 мешка, а на третью —
40 мешков. Сколько килограммов муки погрузили на каждую
машину?
Показать решение
Список сокращений

В задачах часто приводятся различные сокращения для


больших чисел. Взамен нулей могут употребляются
сокращения тыс. млн, млрд.

Пример 1. Вместо записи 1000 используется сокращение 1


тыс.

Пример 2. Вместо записи 2000 используется запись 2 тыс.

Пример 3. Вместо записи 1000000 используется запись 1


млн.

Пример 4. Вместо записи 1000000000 используется запись 1


млрд.

Сокращения тыс., млн, млрд заменяют собой множители


1000, 1000000, 1000000000 соответственно. Чтобы записать
число полностью без сокращений, достаточно умножить его
на соответствующий множитель.
889

К примеру, если на письме указано число 5,5 тыс., то для его


полного написания нужно умножить его на 1000

5,5 × 1000 = 5500

Пример 2. Записать число 1500 тыс. без сокращения

Сокращение тыс. указывает нам на множитель 1000.


Умножим на него 1500

1500 × 1000 = 1 500 000

Пример 3. Записать число 1500,5 тыс. без сокращения

1500,5 × 1000 = 1 500 500

Пример 4. Записать число 11,2 тыс. без сокращения

11,2 × 1000 = 11 200

Пример 5. Записать 1,5 млн без сокращения

Сокращение млн указывает нам на множитель 1000000.


Умножим на него 1,5

1,5 × 1000000 = 1500000

Пример 6. Зарплата отца составляет 32,7 тыс. руб.,


зарплаты матери — 28,2 тыс. руб. Записать эти числа без
сокращений.
890

32,7 × 1000 = 32700 рублей

28,2 × 1000 = 28200 рублей

Полезно также обратить внимание на то, что сокращение


тыс. употребляется с точкой на конце. Остальные же
сокращения (млн, млрд, трлн) употребляются без точки.
891

Задачи на дроби

Продолжаем изучать элементарные задачи по математике.


Данный урок посвящен задачам на дроби.

Прежде чем решать задачи на дроби, необходимо


досконально изучить все темы, касающиеся дробей. Ниже
приведен список уроков, которые можно повторить.

Каждая задача, приведенная в данном уроке, относится к


категории элементарных. Если какая-то задача непонятна,
это указывает на то, что предыдущий материал усвоен
недостаточно хорошо.

Предварительные навыки

 Дроби
 Действия с дробями
 Смешанные числа
 Применение дробей
 Десятичные дроби
 Действия с десятичными дробями
 Применение десятичных дробей
 Дополнительные сведения о дробях

Содержание урока

 Задачи на дроби
 Задачи для самостоятельного решения
892

Задачи на дроби

Задача 1. В классе  школьников составляют отличники.


Какую часть составляют остальные? Сделать графическое
описание задачи. Рисунок может быть любым.

Решение

Если   составляют отличники, то   составляют остальные

Задача 2. В классе   школьников составляют отличники,  

составляют хорошисты,   составляют троечники. Сделать


графическое описание задачи. Рисунок может быть любым.
893

Задача 3. В классе 24 школьника.   школьников

составляют отличники,   составляют хорошисты,   


составляют троечники. Сколько в классе отличников,
хорошистов и троечников?

Решение

24 : 6 × 1 = 4 × 1 = 4 (отличника)

24 : 6 × 3 = 4 × 3 = 12 (хорошистов)

24 : 6 × 2 = 4 × 2 = 8 (троечников)

Проверка

4 + 12 + 8 = 24 (школьника)

24 = 24

Задача 4. В классе  школьников составляют отличники,   


составляют хорошисты. Какую часть составляют троечники?
894

Решение

Школьники разделены на 6 частей. На одну из частей


приходятся отличники, на три части — хорошисты. Нетрудно
догадаться, что на остальные две части приходятся

троечники. Значит   школьников составляют троечники

Не приводя рисунков можно сложить дроби   и  , и

полученный результат вычесть из дроби  , которая выражает


всю часть школьников. Другими словами, сложить
отличников и хорошистов, затем вычесть этих отличников и
хорошистов из общего количества школьников
895

Задача 5. В классе 16 школьников. Из них  составляют

отличники,   составляют хорошисты. Сколько отличников и


хорошистов в классе? Сделать графическое описание
задачи. Рисунок может быть любым.

Решение

16 : 4 × 1 = 4 × 1 = 4 (отличника)

16 : 16 × 12 = 1 × 12 = 12 (хорошистов)


896

Задача 6. В классе 16 школьников. Из них   составляют

отличники,   составляют хорошисты,  составляют


троечники. Сколько отличников, хорошистов и троечников в
классе? Сделать графическое описание задачи. Рисунок
может быть любым.

Решение

16 : 8 × 1 = 2 × 1 = 2 (отличника)

16 : 16 × 10 = 1 × 10 = 10 (хорошистов)

16 : 4 = 4 (троечника)
897

Задача 7. Из зерен пшеницы производят полтавскую крупу,

масса которой составляет  массы зерна пшеницы, а


остальное составляют кормовые отходы. Сколько можно
получить полтавской крупы и кормовых отходов из 500
центнеров пшеницы
898

Решение

Найдем от 500 центнеров:

Теперь найдем массу кормовых отходов. Для этого вычтем


из 500 ц массу полтавской крупы:

Значит из 500 центнеров зерен пшеницы можно получить 320


центнеров полтавской крупы и 180 центнеров кормовых
отходов.

Задача 8. Килограмм сахара стоит 88 рублей. Сколько стоит

кг сахара? кг? кг? кг?

Решение

1) кг это половина одного килограмма. Если один килограмм


стоит 88 рублей, то половина килограмма будет стоит
половину от 88, то есть 44 рубля. Если найти половину от 88
рублей, мы получим 44 рубля

88 : 2 = 44

44 × 1 = 44 рубля

2) кг это четверть килограмма. Если один килограмм стоит


88 рублей, то четверть килограмма будет стоит четверти от
899

88 рублей, то есть 22 рубля. Если найти  от 88 рублей, мы


получим 22 рубля

88 : 4 = 22

22 × 1 = 22 рубля

3) Дробь означает, что килограмм разделен на восемь


частей, и оттуда взято три части. Если один килограмм стоит
88 рублей, то стоимость трех восьми килограмм будут стоить

 от 88 рублей. Если найти  от 88 рублей, мы получим 33


рубля.

4) Дробь  означает, что килограмм разделен на восемь


частей, и оттуда взято одиннадцать частей. Но невозможно
взять одиннадцать частей, если их только восемь. Мы имеем
дело с неправильной дробью. Сначала выделим в ней целую
часть:

Одиннадцать восьмых это один целый килограмм и


килограмма. Теперь мы можем по отдельности найти
стоимость одного целого килограмма и стоимость трёх
восьмых килограммов. Один килограмм, как было указано

выше стоит 88 рублей.  Стоимость кг мы также находили и

получили 33 рубля. Значит  кг сахара будет стоит 88+33


рубля, то есть 121 рубль.
900

Стоимость можно найти не выделяя целой части. Для этого

достаточно найти от 88.

88 : 8 = 11

11 × 11 = 121

Но выделив целую часть можно хорошо понять, как

сформировалась цена на кг сахара.

Задача 9. Финики содержат  сахара и  минеральных


солей. Сколько граммов каждого из веществ содержится в 4
кг фиников?

Решение

Узнаем сколько граммов сахара содержится в одном


килограмме фиников. Один килограмм это тысяча грамм.

Найдем от 1000 грамм:

1000 : 25 = 40

40 × 18 = 720 г

В одном килограмме фиников содержится 720 грамм сахара.


Чтобы узнать сколько грамм сахара содержится в четырех
килограммах, нужно 720 умножить на 4

720 × 4 = 2880 г
901

Теперь узнаем сколько минеральных солей содержится в 4


килограммах фиников. Но сначала узнаем сколько
минеральных солей содержится в одном килограмме. Один

килограмм это тысяча грамм. Найдем  от 1000 грамм:

1000 : 200 = 5

5 × 3 = 15 г

В одном килограмме фиников содержится 15 грамм


минеральных солей. Чтобы узнать сколько грамм
минеральных солей содержится в четырех килограммах,
нужно 15 умножить на 4

15 × 4 = 60 г

Значит в 4 кг фиников содержится 2880 грамм сахара и 60


грамм минеральных солей.

Решение для данной задачи можно записать значительно


короче, двумя выражениями:

Суть в том, что от 4 килограмм нашли и полученные 2,88


перевели в граммы, умножив на 1000. Тоже самое сделали и

для минеральных солей — от 4 кг нашли и получившиеся


килограммы перевели в граммы, умножив на 1000. Обратите
также внимание на то, что дробь от числа найдена
902

упрощенным способом — прямым умножением числа на


дробь.

Задача 10. Поезд прошел 840 км, что составляет  его пути.
Какое расстояние ему осталось пройти? Каково расстояние
всего пути?

Решение

В задаче говорится, что 840 км это от его пути. Знаменатель

дроби указывает на то, что весь путь разделен на семь


равных частей, а числитель указывает на то, что четыре
части этого пути уже пройдено и составляют 840 км.
Поэтому, разделив 840 км на 4, мы узнаем сколько
километров приходится на одну часть:

840 : 4 = 210 км.

А поскольку весь путь состоит из семи частей, то расстояние


всего пути можно найти, умножив 210 на 7:

210 × 7 = 1470 км.

Теперь ответим на второй вопрос задачи — какое расстояние


осталось пройти поезду? Если длина пути 1470 км, а
пройдено 840, то оставшийся путь равен 1470−840, то есть
630

1470 − 840 = 630


903

Задача 11. Одна из групп, покорившая горную вершину


Эверест, состояла из спортсменов, проводников и
носильщиков. Спортсменов в группе было 25, число

проводников составляло  числа спортсменов, а число


спортсменов и проводников вместе лишь 9/140 числа
носильщиков. Сколько было носильщиков в этой
экспедиции?

Решение

Спортсменов группе 25. Проводников составляет числа

спортсменов. Найдем от 25 и узнаем сколько в группе


проводников:

25 : 5 × 4 = 20

Спортсменов и проводников вместе — 45 человек. Это число

составляет от числа носильщиков. Зная что от числа


носильщиков это 45 человек, мы можем найти общее число
носильщиков. Для этого найдем число по дроби:

45 : 9 × 140 = 5 × 140 = 700

Задача 12. В школу привезли 900 новых учебников, из них

учебники по математике составляли  всех книг, учебники по

русскому языку  всех книг, а остальные книги были по


литературе. Сколько привезли книг по литературе
904

Узнаем сколько составляют учебники по математике:

900 : 25 × 8 = 288 (книг по математике)

Узнаем сколько учебников по русскому языку:

900 : 100 × 33 = 297 (книг по русскому языку)

Узнаем сколько учебников по литературе. Для этого из


общего числа книг вычтем учебники по математике и по
русскому:

900 – (288+297) = 900 – 585 = 315

Проверка

288 + 297 + 315 = 900

900 = 900

Задача 13. В первый день продали  , а во второй день   


поступившего в магазин винограда. Какую часть винограда
продали за два дня?

Решение

За два дня продали   винограда. Эта часть получается путем

сложения дробей   и 


905

Можно представить поступивший в магазин виноград в виде

шести гроздей. Тогда винограда это две грозди, винограда

— три грозди, а винограда это пять гроздей из шести,


проданные за два дня. Ну и нетрудно увидеть, что осталась

одна гроздь, выраженная дробь  (одна гроздь из шести)


906

Задача 14. Вера в первый день прочитала   книги, а во

второй день на  меньше. Какую часть книги прочитала Вера


во второй день? Успела ли она прочитать книгу за два дня?

Решение

Определим часть книги, прочитанной во второй день.

Сказано, что во второй день прочитано на   меньше, чем в

первый день. Поэтому из   нужно вычесть 

Во второй день Вера прочитала   книги. Теперь ответим на


второй вопрос задачи — успела ли Вера прочитать книгу за
два дня? Сложим то, что Вера прочитала в первый и во
второй день:

За два дня Вера прочитала   книги, но осталось ещё   


книги. Значит Вера не успела прочитать всю книгу за два дня.

Сделаем проверку. Предположим что книга, которую читала

Вера, имела 180 страниц. В первый день она прочла   книги.

Найдем   от 180 страниц


907

180 : 9 × 5 = 100 (страниц)

Во второй день Вера прочитала на   меньше, чем в первый.

Найдем   от 180 страниц, и вычтем полученный результат из


100 листов, прочитанных в первый день

180 : 6 × 1 = 30 × 1 = 30 (страниц)

100 − 30 = 70 (страниц во второй день)

Проверим, являются ли 70 страниц   частью книги:

180 : 18 × 7 = 10 × 7 = 70 (страниц)

Теперь ответим на второй вопрос задачи — успела ли Вера


прочитать все 180 страниц за два дня. Ответ — не успела,
поскольку за два дня она прочла только 170 страниц

100 + 70 = 170 (страниц)

Осталось прочесть еще 10 страниц. В задаче в роли остатка

у нас была дробь  . Проверим являются ли 10 страниц   


частью книги?

180 : 18 × 1 = 10 × 1 = 10 (страниц)


908

Задача 15. В одном пакете  кг, а в другом на  кг меньше.


Сколько килограммов конфет в двух пакетах вместе?

Решение

Определим массу второго пакета. Она на   кг меньше, чем


масса первого пакета. Поэтому из массы первого пакета
вычтем массу второго:

Масса второго пакета   кг. Определим массу обоих пакетов.


Сложим массу первого и массу второго:

Масса обоих пакетов   кг. А   килограмма это 800 граммов.


Можно решать такую задачу, работая с дробями, складывая
и вычитая их. Также можно сначала найти число по данным в
909

задаче дробям и приступить к решению. Так   килограмма

это 500 граммов, а   кг это 200 граммов

1000 : 2 × 1 = 500 × 1 = 500 г

1000 : 5 × 1 = 200 × 1 = 200 г

Во втором пакете на 200 граммов меньше, поэтому чтобы


определить массу второго пакета, нужно из 500 г вычесть 200
г

500 − 200 = 300 г

Ну и напоследок сложить массы обоих пакетов:

500 + 300 = 800 г

Задача 16. Туристы прошли путь от турбазы до озера за 4

дня. В первый день они прошли всего пути, во второй


оставшегося пути, а в третий и четвертый дни проходили по
12 км. Чему равна длина всего пути от турбазы до озера?

Решение

В задаче сказано, что во второй день туристы прошли

оставшегося пути. Дробь означает, что оставшийся путь


разделен на 7 равных частей, из них туристы прошли три

части, но осталось пройти остальные . На эти приходится


то расстояние, которое туристы прошли в третий и четвертый
910

день, то есть 24 км (по 12 км в каждом дне). Нарисуем


наглядную схему, иллюстрирующую второй, третий и
четвертый дни:

В третий и четвертый день туристы прошли 24 км и это

составляет   от пути, пройденного во второй, третий и

четвертый дни. Зная, что   составляют 24 км, мы можем


найти весь путь, пройденный во второй, третий и четвертый
день:

24 : 4 × 7 = 6 × 7 = 42 км

Во второй, третий и четвертый день туристы прошли 42 км.

Теперь найдем   от этого пути. Так мы узнаем сколько


километров туристы прошли во второй день:

42 : 7 × 3 = 6 × 3 = 18 км

Теперь возвращаемся к началу задачи. Сказано, что в

первый день туристы прошли   всего пути. Весь путь


разделен на четыре части, и на первую часть приходится
путь, пройденный в первый день. А путь, который приходится
911

на остальные три части, мы уже нашли — это 42 километра,


пройденные во второй, третий и четвертый дни. Нарисуем
наглядную схему, иллюстрирующую первый и остальные три
дня:

Зная, что   пути составляют 42 километра, мы можем найти


длину всего пути:

42 : 3 × 4 = 56 км

Значит длина пути от турбазы до озера составляет 56


километров. Сделаем проверку. Для этого сложим все пути,
пройденные туристами в каждый из четырех дней.

Сначала найдем путь пройденный в первый день:

56 : 4 × 1 = 14 (в первый день)

14 + 18 + 12 + 12 = 56

56 = 56

Задача из арифметики известного среднеазиатского


математика Мухаммеда ибн-Мусы ал-Хорезми (IX век н. э.)
912

«Найти число, зная, что если отнять от него одну треть и


одну четверть, то получится 10»

Изобразим число, которое мы хотим найти, в виде отрезка,


разделенного на три части. В первой части отрезка отметим
треть, во второй — четверть, оставшаяся третья часть будет
изображать число 10.

Сложим треть и четверть:

Теперь изобразим отрезок, разделенный на 12 частей.

Отметим на нем дробь   , остальные пять частей пойдут на


число 10:
913

Зная, что пять двенадцатых числа составляют число 10, мы


можем найти всё число:

10 : 5 × 12 = 2 × 12 = 24

Мы нашли всё число — оно равно 24.

Эту задачу можно решить не приводя рисунков. Для этого,


сначала нужно сложить треть и четверть. Затем из единицы,
которая играет роль неизвестного числа, вычесть результат
сложения трети и четверти. Затем по полученной дроби
определить всё число:

Задача 17. Семья, состоящая из четырех человек, в месяц


зарабатывает 80 тысяч рублей. Бюджет распланирован
914

следующим образом:   на еду,   на коммунальные

услуги,   на Интернет и ТВ,  на лечение и походы по

врачам,  на пожертвование в детский дом,  на

проживание в съемной квартире,   в копилку. Сколько денег


выделено на еду, коммунальные услуги, на Интернет и ТВ,
на лечение и походы по врачам, пожертвование на детский
дом, на проживание в съемной квартире, и на копилку?

Решение

80 : 40 × 7 = 14 (тыс. на еду)

80 : 20 × 1 = 4 × 1 = 4 тыс. (на коммунальные услуги)

80 : 20 × 1 = 4 × 1 = 4 тыс. (на Интернет и ТВ)

80 : 20 × 3 = 4 × 3 = 12 тыс. (на лечение и походы по врачам)

80 : 10 × 1 = 8 × 1 = 8 тыс. (на пожертвование в детский дом)

80 : 20 × 3 = 4 × 3 = 12 тыс. (на проживание в съемной


квартире)

80 : 40 × 13 = 2 × 13 = 26 тыс. (в копилку)

Проверка

14 + 4 + 4 + 12 + 8 + 12 + 26 = 80

80 = 80
915

Задача 18. Туристы во время похода за первый час прошли

 км, а за второй на  км больше. Сколько километров


прошли туристы за два часа?

Решение

Найдем числа по дробям.   это три целых километра и


семь десятых километра, а семь десятых километра это 700
метров:

 это один целый километр и одна пятая километра, а одна


пятая километра это 200 метров

Определим длину пути, пройденного туристами за второй


час. Для этого к 3 км 700 м нужно прибавить 1 км 200 м

3 км 700 м + 1 км 200 м = 3700м + 1200м =  4900м = 4 км 900


м

Определим длину пути, пройденного туристами за два часа:

3 км 700 м + 4 км 900 = 3700м + 4900м = 8600м = 8 км 600 м

Значит за два часа туристы прошли 8 километров и еще 600


метров. Решим эту задачу с помощью дробей. Так её можно
значительно укоротить
916

Получили ответ   километра. Это восемь целых


километров и шесть десятых километра, а шесть десятых
километра это шестьсот метров

Задача 19. Геологи прошли долину, расположенную между

горами, за три дня. В первый день они прошли , во второй  


всего пути и в третий оставшиеся 28 км. Вычислить длину
пути, проходящего по долине.

Решение

Изобразим путь в виде отрезка, разделенного на три части. В

первой части отметим   пути, во второй части   пути, в


третьей части оставшиеся 28 километров:
917

Сложим части пути, пройденные в первый и во второй день:

За первый и второй дни геологи прошли   всего пути. На

остальные   пути приходятся 28 километров, пройденные

геологами в третий день. Зная, что 28 километров это   всего


пути, мы можем найти длину пути, проходящего по долине:

28 : 4 × 9 = 7 × 9 = 63 км

Проверка
918

63 : 9 × 5 = 7 × 5 = 35

63 : 9 × 4 = 7 × 4 = 28

35 + 28 = 63

63 = 63

Задача 20. Для приготовления крема использовали сливки,


сметану и сахарную пудру. Сметану и сливки составляют
844,76 кг, а сахарная пудра и сливки 739,1 кг. Сколько в
отдельности сливок, сметаны и сахарной пудры содержится
в 1020,85 кг крема?

Решение

сметана и сливки — 844,76 кг


сахарная пудра и сливки — 739,1 кг

Вытащим из 1020,85 кг крема сметану и сливки (844,76 кг).


Так мы найдем массу сахарной пудры:

1020,85 кг — 844,76 кг = 176,09 (кг сахарной пудры)

Вытащим из сахарной пудры и сливок сахарную пудру


(176,09 кг). Так мы найдем массу сливок:

739,1 кг — 176,09 кг = 563,01 (кг сливок)

Вытащим сливки из сметаны и сливок. Так мы найдем массу


сметаны:

844,76 кг — 563,01 кг = 281,75 (кг сметаны)

176,09 (кг сахарная пудра)


919

563,01 (кг сливки)

281,75 (кг сметана)

Проверка

176,09 кг + 563,01 кг + 281,75 кг = 1020,85 кг

1020,85 кг = 1020,85 кг

Задача 21. Масса бидона, заполненного молоком равна 34


кг. Масса бидона, заполненного наполовину, равна 17,75 кг.
Какова масса пустого бидона?

Решение

Вычтем из массы бидона, заполненного молоком, массу


бидона заполненного наполовину. Так мы получим массу
содержимого бидона, заполненного наполовину, но уже без
учета массы бидона:

34 кг − 17,75 кг = 16,25 кг

16,25 это масса содержимого бидона заполненного


наполовину. Умножим эту массу на 2, получим массу бидона
заполненного полностью:

16,25 кг × 2 = 32,5 кг

32,5 кг это масса содержимого бидона. Чтобы вычислить


массу пустого бидона, нужно из 34 кг вычесть массу его
содержимого, то есть 32,5 кг

34 кг − 32,5 кг = 1,5 кг


920

Ответ: масса пустого бидона составляет 1,5 кг.

Задача 22. Сливки составляют 0,1 массы молока, а


сливочное масло составляет 0,3 массы сливок. Сколько
сливочного масла можно получить из суточного надоя
коровы, равного 15 кг молока?

Решение

Определим сколько килограмм сливок можно получить с 15 кг


молока. Для этого найдем 0,1 часть от 15 кг.

15 × 0,1 = 1,5 (кг сливок)

Теперь определим сколько сливочного масла можно


получить с 1,5 кг сливок. Для этого найдем 0,3 часть от 1,5 кг

1,5 кг × 0,3 = 0,45 (кг сливочного масла)

Ответ: из 15 кг молока можно получить 0,45 кг сливочного


масла.

Задача 23. 100 кг клея для линолеума содержат  55 кг


асфальта, 15 кг канифоли, 5 кг олифы и 25 кг бензина. Какую
часть этого клея образует каждая из его составляющих?

Решение

Представим, что 100 кг клея как 100 частей. Тогда на 55


частей приходится асфальт, на 15 частей — канифоль, на 5
частей — олифа, на 25 частей — бензин. Запишем эти части
в виде дробей, и по возможности сократим получающиеся
дроби:
921

Ответ:   клея составляет асфальт,   составляет

канифоль,   составляет олифа,   составляет бензин.

Задачи для  самостоятельного решения

Задача 1. В одном пакете  кг конфет, а в другом на  кг.


меньше. Какова масса двух пакетов вместе
Показать решение

Задача 2. Чтобы побывать в театре, Тане потребовалось .

На дорогу туда и обратно у нее ушло   ч. Сколько времени


длилось театральное представление?
Показать решение

Задача 3. В первый час лыжник прошел всего расстояния,

которое он должен пройти, во второй всего пути, а в третий


оставшуюся часть пути. Какую часть всего расстояния
прошел лыжник в третий час?
922

Показать решение
Задача 4. Все мальчики класса приняли участие в школьных

соревнованиях: часть вошла в футбольную команду, часть

в баскетбольную, часть состязалась по прыжкам в длину,


остальные учащиеся класса – в бегу. На какую часть бегунов
было больше (или меньше), чем футболистов?
баскетболистов?
Показать решение

Задача 5. На выставке художественных работ представлена

живопись, скульптура и графика. всех работ составляет

скульптура, – живопись, оставшуюся часть – графика. Какую


часть всех работ составляет графика?

Показать решение

Задача 6. Рабочие отремонтировали дорогу длиной 820 м за

три дня. Во вторник они отремонтировали этой дороги, а в

среду оставшейся части. Сколько метров дороги


отремонтировали рабочие в четверг?

Показать решение

Задача 7. В книге три рассказа. Наташа прочла первый

рассказ за ч, на чтение второго рассказа она потратила на

ч больше, а чтение третьего рассказа заняло на ч меньше,


923

чем чтение первого и второго рассказов вместе. Сколько


времени ушло у Наташи на чтение всей книги?

Показать решение

Задача 8. Из одной тонны хлопка-сырца можно изготовить


3400 м ткани, 1,05 ц пищевого масла и 0,225 т жмыха.
Сколько метров ткани, пищевого масла и жмыха можно
получить из 32,4 ц хлопка-сырца?

Показать решение
Задача 9. Какой путь прошли туристы, если 0,2 всего пути
составляют 12 км?
Показать решение
Задача 10. Михаил прочитал 0,7 книги, что составило 56
страниц. Сколько всего страниц в книге? Сколько страниц
осталось прочитать?
Показать решение

Задача 11. зданий в городе составляют жилые дома. из


них — многоэтажные. Какую часть всех зданий в городе
составляют многоэтажные жилые дома?
Показать решение

Задача 12. От веревки длина которой м, нужно отрезать м.


Как это сделать, не производя измерений?
924

Задачи на проценты

Продолжаем изучать элементарные задачи по математике.


Данный урок посвящен задачам на проценты. Мы
рассмотрим несколько задач, а также затронем те моменты,
которые не упоминали ранее при изучении процентов,
посчитав что на первых порах они создают трудности для
обучения.

Большинство задач на проценты сводятся к тому, чтобы


найти процент от числá, найти число по проценту, выразить в
процентах какую-либо часть, либо выразить в процентном
соотношении взаимосвязь между несколькими объектами,
числами, величинами.

Предварительные навыки

 Соотношения
 Проценты
 Простейшие задачи по математике
 Задачи на дроби

Содержание урока

 Способы нахождения процента


 Выражение числа в процентах
 Аналоги в виде дробей
 Уменьшение и увеличение процентов
 Задача на нахождение процентного соотношения
 Сравнение величин в процентах
 Задачи на концентрацию, сплавы и смеси
 Задачи для самостоятельного решения
925

Способы нахождения процента

Процент можно находить различными способами. Самый


популярный способ — разделить число на 100 и умножить
полученный результат на искомое количество процентов.

Например, чтобы найти 60% от 200 рублей, нужно сначала


эти 200 рублей разделить на сто равных частей:

200 руб : 100 = 2 руб.

Когда мы делим число на 100, мы тем самым находим один


процент от этого числа. Так, разделив 200 рублей на 100
частей, мы автоматически нашли 1% от двухсот рублей, то
есть узнали сколько рублей прихóдится на одну часть. Как
видно из примера, на одну часть (на один процент)
приходится 2 рубля.

1% от 200 рублей — 2 рубля

Зная сколько рублей приходится на одну часть (на 1%),


можно узнать сколько рублей приходится на две части, на
три, на четыре, на пять и т.д. То есть можно найти любое
количество процентов. Для этого достаточно умножить эти 2
рубля на искомое количество частей (процентов). Давайте
найдём шестьдесят частей (60%)

2 руб × 60 = 120 руб.

Найдём 5%

2 руб × 5 = 10 руб.

Найдем 90%
926

2 руб × 90 = 180 руб.

Найдем 100%

2 руб × 100 = 200 руб.

100% это все сто частей и они составляют все 200 рублей.

Второй способ заключается в том, чтобы представить


проценты в виде обыкновенной дроби и найти эту дробь от
того числа, откуда требуется найти процент.

Например, найдем те же 60% от 200 рублей. Сначала


предстáвим 60% в виде обыкновенной дроби. 60% это
шестьдесят частей из ста, то есть шестьдесят сотых:

Теперь задание можно понимать как «найти  от 200


рублей«. Это нахождение дроби от числа, которое мы
изучали ранее. Напомним, что для нахождения дроби от
числа, нужно это число разделить на знаменатель дроби и
полученный результат умножить на числитель дроби

200 : 100 = 2

2 × 60 = 120

Либо умножить число на дробь (быстрый способ нахождения


дроби от числа):
927

Третий способ заключается в том, чтобы представить


процент в виде десятичной дроби и умножить число на эту
десятичную дробь.

Например, найдем те же 60% от 200 рублей. Для начала


представляем 60% в виде дроби. 60% процентов это
шестьдесят частей из ста

Выполним деление в этой дроби. Перенесем запятую в числе


60 на две цифры влево:

Теперь находим 0,60 от 200 рублей. Для нахождения


десятичной дроби от числа, нужно это число умножить на
десятичную дробь:

200 × 0,60 = 120 руб.

Приведенный способ нахождения процента является


наиболее удобным, особенно если человек привык
пользоваться калькулятором. Этот способ позволяет найти
процент в одно действие.

Как правило выразить процент в десятичной дроби не


составляет особого труда. Достаточно приписать «ноль
целых» перед процентной долей, если процентная доля
представляет собой двузначное число, или приписать «ноль
целых» и еще один ноль, если процентная доля
представляет собой однозначное число. Примеры:
928

60% = 0,60 — приписали ноль целых перед числом 60,


поскольку число 60 является двузначным

6% = 0,06 — приписали ноль целых и еще один ноль перед


числом 6, поскольку число 6 является однозначным.

При делении на 100 мы воспользовались методом


передвижения запятой на две цифры влево. В ответе 0,60
ноль, стоящий после цифры 6, сохранился. Но если
выполнить это деление уголком, ноль исчезает —
получается ответ 0,6

Надо помнить, что десятичные дроби 0,60 и 0,6 равны


одному и тому же значению:

0,60 = 0,6

В том же «уголке» можно продолжать деление бесконечно,


каждый раз приписывая к остатку ноль, но это будет
бессмысленным действием:
929

Выражать проценты в виде десятичной дроби можно не


только делением на 100, но и умножением. Значок процента
(%) сам по себе заменяет собой множитель 0,01. А если
учитывать, что число процентов и значок процента записаны
слитно, то между ними располагается «невидимый» знак
умножения (×).

Так, запись 45% на самом деле выглядит следующим


образом:

Заменим знак процента на множитель 0,01

Данное умножение на 0,01 выполнятся путем перемещения


запятой на две цифры влево:
930

Задача 1. Бюджет семьи составляет 75 тыс. рублей в месяц.


Из них 70% — деньги, заработанные папой. Какую часть
заработала мама?

Решение

Всего процентов 100. Если папа заработал 70% денег, то


остальные 30% денег заработала мама.

Задача 2. Бюджет семьи составляет 75 тыс. рублей в месяц.


Из них 70% — деньги, заработанные папой, а 30% — деньги,
заработанные мамой. Сколько денег заработал каждый?

Решение

Найдем 70 и 30 процентов от 75 тыс. рублей. Так мы


определим сколько денег заработал каждый. Для удобства
70% и 30% запишем в виде десятичных дробей:

75 × 0,70 = 52,5 (тыс. руб. заработал папа)

75 × 0,30 = 22,5 (тыс. руб. заработала мама)

Проверка

52,5 + 22,5 = 75

75 = 75

Ответ: 52,5 тыс. руб. заработал папа, 22,5 руб. заработала


мама.
931

Задача 3. При остывании хлеб теряет до 4% своей массы в


результате испарения воды. Сколько килограммов испарится
при остывании 12 тонн хлеба.

Решение

Переведем 12 тонн в килограммы. В одной тонне тысяча


килограмм, а в 12 тоннах в 12 раз больше:

1000 × 12 = 12 000 кг

Теперь найдем 4% от 12000. Полученный результат и будет


ответом к задаче:

12 000 × 0,04 = 480 кг

Ответ: при остывании 12 тонн хлеба испарится 480


килограмм.

Задача 4. Яблоки при сушке теряют 84% своей массы.


Сколько получится сушенных яблок из 300 кг свежих?

Найдем 84% от 300 кг

300 : 100 × 84 = 252 кг

300 кг свежих яблок в результате сушки потеряют 252 кг


своей массы. Чтобы ответить на вопрос сколько получится
сушенных яблок, нужно из 300 вычесть 252

300 − 252 = 48 кг

Ответ: из 300 кг свежих яблок получится 48 кг сушенных.


932

Задача 5. В семенах сои содержится 20% масла. Сколько


масла содержится в 700 кг сои?

Решение

Найдем 20% от 700 кг

700 × 0,20 = 140 кг

Ответ: в 700 кг сои содержится 140 кг масла

Задача 6. Гречневая крупа содержит 10% белков, 2,5%


жиров и 60% углеводов. Сколько этих продуктов содержится
в 14,4 ц гречневой крупы?

Решение

Переведем 14,4 центнера в килограммы. В одном центнере


100 килограмм, в 14,4 центнерах в 14,4 раз больше

100 × 14,4 = 1440 кг

Найдем 10%, 2,5% и 60% от 1440 кг

1440 × 0,10 = 144 (кг белков)

1440 × 0,025 = 36 (кг жиров)

1440 × 0,60 = 864 (кг углеводов)

Ответ: в 14,4 ц гречневой крупы содержится 144 кг


белков, 36 кг жиров, 864 кг углеводов.
933

Задача 7. Для лесопитомника школьники собрали 60 кг


семян дуба, акации, липы и клена. Желуди составляли 60%,
семена клена 15%, семена липы 20% всех семян, а
остальное составляли семена акации. Сколько килограммов
семян акации было собрано школьниками?

Решение

Примем за 100% семена дуба, акации, липы и клена. Вычтем


из этих 100% проценты, выражающие семена дуба, липы и
клена. Так мы узнаем сколько процентов составляют семена
акации:

100% − (60% + 15% + 20%) = 100% − 95% = 5%

Теперь находим семена акации:

60 × 0,05 = 3 кг

Ответ: школьниками было собрано 3 кг семян акации.

Проверка:

60 × 0,60 = 36

60 × 0,15 = 9

60 × 0,20 = 12

60 × 0,05 = 3

36 + 9 + 12 + 3 = 60

60 = 60
934

Задача 8. Купил человек продукты. Молоко стоит 60 рублей,


что составляет 48% от стоимости всех покупок. Определить
общую сумму денег, потраченных на продукты.

Решение

Это задача на нахождение числа по его проценту, то есть по


его известной части. Такую задачу можно решать двумя
способами. Первый заключается в том, чтобы выразить
известное число процентов в виде десятичной дроби и найти
неизвестное число по этой дроби

Выразим 48% в виде десятичной дроби

48% : 100 = 0,48

Зная, что 0,48 составляет 60 рублей, мы можем определить


сумму всех покупок. Для этого нужно найти неизвестное
число по десятичной дроби:

60 : 0,48 = 125 рублей

Значит, общая сумма денег, затраченных на продукты


составляет 125 рублей.

Второй способ заключается в том, чтобы сначала узнать


сколько денег приходится на один процент, затем
полученный результат умножить на 100

48% это 60 рублей. Если мы разделим 60 рублей на 48, то


узнаем сколько рублей приходится на 1%

60 : 48% = 1,25 рублей


935

На 1% приходится 1,25 рублей. Всего процентов 100. Если


мы умножим 1,25 рублей на 100, получим общую сумму
денег, затраченных на продукты

1,25 × 100 = 125 рублей

Задача 9. Из свежих слив выходит 35% сушенных. Сколько


надо взять свежих слив, чтобы получить 140 кг сушенных?
Сколько получится сушенных слив из 600 кг свежих?

Решение

Выразим 35% в виде десятичной дроби и найдем


неизвестное число по этой дроби:

35% = 0,35

140 : 0,35 = 400 кг

Чтобы получить 140 кг сушенных слив, нужно взять 400 кг


свежих.

Ответим на второй вопрос задачи — сколько получится


сушенных слив из 600 кг свежих? Если из свежих слив
выходит 35% сушенных, то достаточно найти эти 35% от 600
кг свежих слив

600 × 0,35 = 210 кг

Ответ: чтобы получить 140 кг сушенных слив, нужно взять


400 кг свежих. Из 600 кг свежих слив получится 210 кг
сушенных.
936

Задача 10. Усвоение жиров организмом человека составляет


95%. За месяц ученик употребил 1,2 кг жиров. Сколько жиров
может быть усвоено его организмом?

Решение

Переведем 1,2 кг в граммы

1,2 × 1000 = 1200 г

Найдем 95% от 1200 г

1200 × 0,95 = 1140 г

Ответ: 1140 г жиров может быть усвоено организмом


ученика.

Выражение чисел в процентах

Процент, как было сказано ранее, можно представить в виде


десятичной дроби. Для этого достаточно разделить число
этих процентов на 100. Например, представим 12% в виде
десятичной дроби:

Замечание. Мы сейчас не находим процент от чего-то, а


просто записываем его в виде десятичной дроби.

Но возможен и обратный процесс. Десятичная дробь может


быть представлена в виде процента. Для этого нужно
умножить эту дробь на 100 и поставить знак процента (%)

Представим десятичную дробь 0,12 в виде процентов


937

0,12 × 100 = 12%

Это действие называют выражением числа в процентах


или выражением чисел в сотых долях.

Умножение и деление являются обратными операциями. К


примеру, если 2 × 5 = 10, то 10 : 5 = 2

Точно так же деление можно записать в обратном порядке.


Если 10 : 5 = 2, то 2 × 5 = 10:

Тоже самое происходит, когда мы выражаем десятичную


дробь в виде процентов. Так, 12% были выражены в виде
десятичной дроби следующим образом: 12 : 100 = 0,12 но
потом эти же 12% были «возвращены» с помощью
умножения, записав выражение 0,12 × 100 = 12%.

Аналогично можно выразить в процентах любые другие


числа, в том числе и целые. Например, выразим в процентах
число 3. Умножим данное число на 100 и к полученному
результату добавим знак процента:

3 × 100 = 300%


938

Большие проценты вида 300% поначалу могут сбивать с


толку, поскольку человек привык считать 100%
максимальной долей. Из дополнительных сведений о дробях
мы знаем, что один целый объект можно обозначать через
единицу. К примеру, если имеется целый не разрезанный
торт, то его можно обозначить через 1

Этот же торт можно обозначить как 100% торта. В этом


случае и единица и 100% будут обозначать один и тот же
целый торт:

Разрежем торт пополам. В этом случае единица обратится в


десятичное число 0,5 (поскольку это половина единицы), а
100% обратятся в 50% (поскольку 50 это половина от сотни)
939

Вернем обратно целый торт, единицу и 100%

Изобразим ещё два таких торта с такими же обозначениями:

Если один торт является единицей, то три торта являются


тремя единицами. Каждый торт является целым
стопроцентным. Если сложить эти три сотни получится 300%.

Поэтому при переводе целых чисел в проценты, мы


умножаем эти числа на 100.

Задача 2. Выразить в процентах число 5

5 × 100 = 500%

Задача 3. Выразить в процентах число 7


940

7 × 100 = 700%

Задача 4. Выразить в процентах число 7,5

7,5 × 100 = 750%

Задача 5. Выразить в процентах число 0,5

0,5 × 100 = 50%

Задача 6. Выразить в процентах число 0,9

0,9 × 100 = 90%

Пример 7. Выразить в процентах число 1,5

1,5 × 100 = 150%

Пример 8. Выразить в процентах число 2,8

2,8 × 100 = 280%

Задача 9. Джордж идет со школы домой. Первые пятнадцать


минут он прошел 0,75 пути. В остальное время он прошел
оставшиеся 0,25 пути. Выразите в процентах части пути,
пройденные Джорджом.

Решение
941

0,75 × 100 = 75%

0,25 × 100 = 25%

Задача 10. Джона угостили половиной яблока. Выразите эту


половину в процентах.

Решение

Половина яблока записывается в виде дроби 0,5. Чтобы


выразить эту дробь в процентах, умножим её на 100 и к
полученному результату добавим знак процента

0,5 × 100 = 50%

Аналоги в виде дробей

Величина, выраженная в процентах, имеет свой аналог в


виде обычной дроби. Так, аналогом для 50% является дробь

. Пятьдесят процентов также можно назвать словом


«половина».

Аналогом для 25% является дробь . Двадцать пять


процентов также можно назвать словом «четверть».

Аналогом для 20% является дробь . Двадцать процентов


также можно назвать словами «пятая часть».

Аналогом для 40% является дробь .


942

Аналогом для 60% является дробь

Пример 1. Пять сантиметров это 50% от дециметра или или


же просто половина. Во всех случаях речь идет об одной и
той же величине — пяти сантиметрах из десяти

Пример 2. Два с половиной сантиметра это 25% от

дециметра или или же просто четверть

Пример 3. Два сантиметра это 20% от дециметра или 


943

Пример 4. Четыре сантиметра это 40% от дециметра или 

Пример 5. Шесть сантиметров это 60% от дециметра или


944

Уменьшение и увеличение процентов

При увеличении или уменьшении величины, выраженной в


процентах употребляется предлог «на».

Примеры:

 Увеличить на 50% — означает увеличить величину в 1,5


раза;
 Увеличить на 100% — означает увеличить величину в 2
раза;
 Увеличить на 200% — означает увеличить в 3 раза;
 Уменьшить на 50% — означает уменьшить величину в 2
раза;
 Уменьшить на 80% — означает уменьшить в 5 раз.

Пример 1. Десять сантиметров увеличили на 50%. Сколько


сантиметров получилось?

Чтобы решать подобные задачи, нужно исходную величину


принимать за 100%. Исходная величина это 10 см. 50% от
них составляют 5 см

Изначальные 10 см увеличили на 50% (на 5 см), значит


получилось 10+5 см, то есть 15 см
945

Аналогом же увеличения десяти сантиметров на 50%


является множитель 1,5. Если умножить на него 10 см
получится 15 см

10 × 1,5 = 15 см

Поэтому выражения «увеличить на 50%» и «увеличить в 1,5


раза» говорят об одном и том же.

Пример 2. Пять сантиметров увеличили на 100%. Сколько


сантиметров получилось?

Примем исходные пять сантиметров за 100%. Сто процентов


от этих пяти сантиметров будут сами 5 см. Если увеличить 5
см на эти же 5 см, то получится 10 см
946

Аналогом же увеличения пяти сантиметров на 100%


является множитель 2. Если умножить на него 5 см
получится 10 см

5 × 2 = 10 см

Поэтому выражения «увеличить на 100%» и «увеличить в 2


раза» говорят об одном и том же.

Пример 3. Пять сантиметров увеличили на 200%. Сколько


сантиметров получилось?

Примем исходные пять сантиметров за 100%. Двести


процентов это два раза по сто процентов. То есть 200% от 5
947

см будут составлять 10 см (по 5 см на каждые 100%). Если


увеличить 5 см на эти 10 см, то получится 15 см

Аналогом же увеличения пяти сантиметров на 200%


является множитель 3. Если умножить на него 5 см
получится 15 см

5 × 3 = 15 см

Поэтому выражения «увеличить на 200%» и «увеличить в 3


раза» говорят об одном и том же.
948

Пример 4. Десять сантиметров уменьшили на 50%. Сколько


сантиметров осталось?

Примем исходные 10 см за 100%. Пятьдесят процентов от 10


см составляют 5 см. Если уменьшить 10 см на эти 5 см,
останется 5 см

Аналогом же уменьшения десяти сантиметров на 50%


является делитель 2. Если разделить на него 10 см, то
получится 5 см

10 : 2 = 5 см

Поэтому выражения «уменьшить на 50%» и «уменьшить в 2


раза» говорят об одном и том же.

Пример 5. Десять сантиметров уменьшили на 80%. Сколько


сантиметров осталось?
949

Примем исходные 10 см за 100%. Восемьдесят процентов от


10 см составляют 8 см. Если уменьшить 10 см на эти 8 см,
останется 2 см

Аналогом же уменьшения десяти сантиметров на 80%


является делитель 5. Если разделить на него 10 см, то
получится 2 см

10 : 5 = 2 см

Поэтому выражения «уменьшить на 80%» и «уменьшить в 5


раз» говорят об одном и том же.

При решении задач на уменьшение и увеличение процентов,


можно умножать/делить величину на указанный в задаче
множитель.

Задача 1. Насколько процентов изменилась величина, если


она увеличилась в 1,5 раза?
950

Величину о которой говорится в задаче можно обозначить


как 100%. Далее умножить эти 100% на множитель 1,5

100% × 1,5 = 150%

Теперь из полученных 150% вычтем изначальные 100% и


получим ответ к задаче:

150% − 100% = 50%

Задача 2. Насколько процентов изменилась величина, если


она уменьшилась в 4 раза?

В этот раз будет происходить уменьшение величины,


поэтому будем выполнять деление. Величину о которой
говорится в задаче обозначим как 100%. Далее разделим эти
100% на делитель 4

100% : 4 = 25%

Из изначальных 100% вычтем полученные 25% и получим


ответ к задаче:

100% − 25% = 75%

Значит, при уменьшении величины в 4 раза она уменьшилась


на 75%.

Задача 3. Насколько процентов изменилась величина, если


она уменьшилась в 5 раз?

Величину о которой говорится в задаче обозначим как 100%.


Далее разделим эти 100% на делитель 5
951

100% : 5 = 20%

Из изначальных 100% вычтем полученные 20% и получим


ответ к задаче:

100% − 20% = 80%

Значит, при уменьшении величины в 5 раз она уменьшилась


на 80%.

Задача 4. Насколько процентов изменилась величина, если


она уменьшилась в 10 раз?

Величину о которой говорится в задаче обозначим как 100%.


Далее разделим эти 100% на делитель 10

100% : 10 = 10%

Из изначальных 100% вычтем полученные 10% и получим


ответ к задаче:

100% − 10% = 90%

Значит, при уменьшении величины в 10 раз она уменьшилась


на 90%.

Задача на нахождение процентного соотношения

Чтобы выразить что-либо в процентном соотношении,


сначала нужно записать дробь, показывающую какую часть
первое число составляет от второго, затем выполнить
деление в этой дроби и полученный результат выразить в
процентах.
952

Например, пусть имеется пять яблок. При этом два яблока


являются красными, три — зелеными. Выразим красные и
зеленые яблоки в процентном соотношении.

Сначала нужно узнать какую часть составляют красные


яблоки. Всего яблок пять, а красных два. Значит, два из пяти
или две пятых составляют красные яблоки:

Зеленых же яблок три. Значит, три из пяти или три пятых


составляют зеленые яблоки:

Имеем две дроби     и    . Выполним деление в этих дробях


953

Получили десятичные дроби 0,4 и 0,6. Теперь выразим в


процентах эти десятичные дроби:

0,4 × 100 = 40%

0,6 × 100 = 60%

Значит, 40% составляют красные яблоки, 60% — зеленые.

А все пять яблок составляют 40%+60%, то есть 100%


954

Задача 2. Двум сыновьям мама дала 200 рублей. Младшему


брату мама дала 80 рублей, а старшему 120 рублей.
Выразите в процентном соотношении деньги, данные
каждому брату.

Решение

Младший брат получил 80 рублей из 200 рублей.


Записываем дробь восемьдесят двухсотых:

Старший брат получил 120 рублей из 200 рублей.


Записываем дробь сто двадцать двухсотых:

Имеем дроби    и  . Выполним деление в этих дробях


955

Выразим в процентах полученные результаты:

0,4 × 100 = 40%

0,6 × 100 = 60%

Значит, 40% денег получил младший брат, а 60% — старший.

Некоторые дроби, показывающие какую часть первое число


составляет от второго, можно сокращать.

Так дроби     и     можно было бы сократить. От этого


ответ к задаче не изменился бы:

Задача 3. Бюджет семьи составляет 75 тыс. рублей в месяц.


Из них 52,5 тыс. руб. — деньги, заработанные папой. 22,5
тыс. руб. — деньги, заработанные мамой. Выразите в
процентах деньги, заработанные папой и мамой.

Решение
956

Данная задача, как и предыдущая, является задачей на


нахождение процентного соотношения.

Выразим в процентах деньги, заработанные папой. Он


заработал 52,5 тыс. рублей из 75 тыс. рублей

Выполним деление в этой дроби:

Выразим полученный результат в процентах:

0,7 × 100 = 70%

Значит, папа заработал 70% денег. Далее нетрудно


догадаться, что остальные 30% денег заработала мама.
Ведь 75 тыс. рублей это все 100% денег. Для уверенности
сделаем проверку. Мама заработала 22,5 тыс. руб. из 75 тыс.
руб. Записываем дробь, выполняем деление и выражаем в
процентах полученный результат:

Задача 4. Школьник тренируется делать подтягивания на


перекладине. В прошлом месяце он мог делать 8
подтягиваний за подход. В этом месяце он может делать 10
подтягиваний за подход. На сколько процентов он увеличил
количество подтягиваний?
957

Решение

Узнаем на сколько больше подтягиваний школьник делает в


текущем месяце, чем в прошлом

10 − 8 = 2

Узнаем какую часть два подтягивания составляют от восьми


подтягиваний. Для этого найдем отношение 2 к 8

Выполним деление в этой дроби

Выразим полученный результат в процентах:

0,25 × 100 = 25%

Значит, школьник увеличил количество подтягиваний на 25%.

Эту задачу можно решить и вторым, более быстрым методом


— узнать во сколько раз 10 подтягиваний больше, чем 8
подтягиваний и полученный результат выразить в процентах.

Чтобы узнать во сколько раз десять подтягиваний больше


восьми подтягиваний, нужно найти отношение 10 к 8
958

Выполним деление в получившейся дроби

Выразим полученный результат в процентах:

1,25 × 100 = 125%

Показатель подтягиваний в текущем месяце составляет


125%. Данное высказывание нужно понимать именно как
«составляет 125%», а не как «показатель увеличился на
125%». Это два разных высказывания, выражающих
различные количества.

Высказывание «составляет 125%» нужно понимать как


«восемь подтягиваний, которые составляют 100% плюс два
подтягивания, которые составляют 25% от восьми
подтягиваний». Графически это выглядит следующим
образом:
959

А высказывание «увеличился на 125%» нужно понимать как


«к текущим восьми подтягиваниях, которые составляли 100%
добавились еще 100% (еще 8 подтягиваний) плюс еще 25%
(2 подтягивания)». Итого получается 18 подтягиваний

100% + 100% + 25% = 8 + 8 + 2 = 18 подтягиваний

Графически это высказывание выглядит следующим


образом:

Всего же получается 225%. Если найти 225% от восьми


подтягиваний, мы получим 18 подтягиваний

8 × 2,25 = 18
960

Задача 5. В прошлом месяце зарплата составляла 19,2 тыс.


руб. В текущем месяце она составила 20,16 тыс. руб. На
сколько процентов повысилась зарплата?

Эту задачу как и предыдущую можно решать двумя


способами. Первый заключается в том, чтобы сначала узнать
на сколько рублей увеличилась зарплата. Далее узнать
какую часть эта прибавка составляет от зарплаты прошлого
месяца

Узнаем на сколько рублей повысилась зарплата:

20,16 − 19,2 = 0,96 тыс. руб.

Узнаем какую часть 0,96 тыс. руб. составляет от 19,2. Для


этого найдем отношение 0,96 к 19,2

Выполним деление в получившейся дроби. По пути


вспомним, как выполняется деление десятичных дробей:

Выразим полученный результат в процентах:

0,05 × 100 = 5%
961

Значит, зарплата повысилась на 5%.

Решим задачу вторым способом. Узнаем во сколько раз


20,16 тыс. руб. больше, чем 19,2 тыс. руб. Для этого найдем
отношение 20,16 к 19,2

Выполним деление в получившейся дроби:

Выразим полученный результат в процентах:

1,05 × 100 = 105%

Зарплата составляет 105%. То есть сюда входят 100%,


которые составляли 19,2 тыс. руб., плюс 5% которые
составляют 0,96 тыс. руб.

100% + 5% = 19,2 + 0,96

Задача 6. Цена ноутбука в этом месяце повысилась на 5%.


Какова его цена, если в прошлом месяце он стоил 18,3 тыс.
рублей?

Решение
962

Найдем 5% от 18,3:

18,3 × 0,05 = 0,915

Прибавим эти 5% к 18,3:

18,3 + 0,915 = 19,215 тыс. руб.

Ответ: цена ноутбука составляет 19,215 тыс. руб.

Задача 7. Цена ноутбука в этом месяце снизилась на 10%.


Какова его цена, если в прошлом месяце он стоил 16,3 тыс.
рублей?

Решение

Найдем 10% от 16,3:

16,3 × 0,10 = 1,63

Вычтем эти 10% из 16,3:

16,3 − 1,63 = 14,67 (тыс. рублей)

Подобные задачи можно записывать кратко:

16,3 − (16,3 × 0,10) = 14,67 (тыс. рублей)

Ответ: цена ноутбука составляет 14,67 тыс. рублей.

Задача 8. В прошлом месяце цена ноутбука составляла 21


тыс. рублей. В этом месяце цена повысилась до 22,05 тыс.
рублей. На сколько процентов повысилась цена?

Решение
963

Определим насколько рублей повысилась цена

22,05 − 21 = 1,05 (тыс. руб)

Узнаем какую часть 1,05 тыс. руб. составляет от 21 тыс. руб.

Выразим полученный результат в процентах

0,05 × 100 = 5%

Ответ: цена ноутбука повысилась на 5%

Задача 8. Рабочий должен был изготовить по плану 600


деталей, а он изготовил 900 деталей. На сколько процентов
он выполнил план?

Решение

Узнаем во сколько раз 900 деталей больше, чем 600


деталей. Для этого найдем отношение 900 к 600

Значение данной дроби равно 1,5. Выразим это значение в


процентах:

1,5 × 100 = 150%

Значит, рабочий выполнил план на 150%. То есть выполнил


его на все 100%, изготовив 600 деталей. Затем изготовил
еще 300 деталей, что составляет 50% от изначального
плана.
964

Ответ: рабочий выполнил план на 150%.

Сравнение величин в процентах

Мы уже много раз сравнивали величины различными


способами. Первым нашим инструментом была разность.
Так, к примеру чтобы сравнить 5 рублей и 3 рубля, мы
записывали разность 5−3. Получив ответ 2, можно было
сказать, что «пять рублей больше трех рублей на два
рубля».

Получаемый в результате вычитания ответ в повседневной


жизни называют не «разностью», а «разницей».

Так, разница между пятью и тремя рубля составляет два


рубля.

Следующим инструментом, которым мы воспользовались


для сравнения величин, было отношение. Отношение
позволяло нам узнать во сколько раз первое число больше
второго (или сколько раз первое число содержит второе).

Так, к примеру десять яблок больше двух яблок в пять раз.


Или по другому, десять яблок содержит два яблока пять раз.
Данное сравнение можно записать с помощью отношения

Но величины можно сравнить и в процентах. Например, цену


двух товаров сравнивать не в рублях, а оценивать, насколько
цена одного товара больше или меньше цены другого в
процентах.
965

Для сравнения величин в процентах, одну из них нужно


обозначить как 100%, а вторую исходя из условий задачи.

Например, узнаем на сколько процентов десять яблок


больше, чем восемь яблок.

За 100% нужно обозначить ту величину с которой мы что-


либо сравниваем. Мы сравниваем 10 яблок с 8 яблоками.
Значит, за 100% обозначаем 8 яблок:

Теперь наша задача сравнить на сколько процентов 10 яблок


больше, чем эти 8 яблок. 10 яблок это 8+2 яблока. Значит,
добавив к восьми яблокам ещё два яблока, мы увеличим
100% еще на какое-то число процентов. Чтобы узнать на
какое именно, определим сколько процентов от восьми яблок
составляют два яблока

Добавив эти 25% к восьми яблокам, мы получим 10 яблок. А


10 яблок это 8+2, то есть 100% и еще 25%. Итого получаем
125%
966

Значит, десять яблок больше восьми яблок на 25%.

Теперь решим обратную задачу. Узнаем насколько


процентов восемь яблок меньше, чем десять яблок. Сразу
напрашивается ответ, что восемь яблок меньше на 25%.
Однако это не так.

Мы сравниваем восемь яблок с десятью яблоками. Мы


договорились, что за 100% будем брать то, с чем
сравниваем. Поэтому в этот раз за 100% берем 10 яблок:

Восемь яблок это 10−2, то есть уменьшив 10 яблок на 2


яблока, мы уменьшим их на какое-то число процентов. Чтобы
узнать на какое именно, определим сколько процентов от
десяти яблок составляют два яблока
967

Отняв эти 20% от десяти яблок, мы получим 8 яблок. А 8


яблок это 10−2, то есть 100% и минус 20%. Итого получаем
80%

Значит, восемь яблок меньше десяти яблок на 20%.

Задача 2. На сколько процентов 5000 рублей больше, чем


4000 рублей?

Решение

Примем 4000 рублей за 100%. 5 тысяч больше 4 тысяч на 1


тысячу. Значит, увеличив четыре тысячи на одну тысячу, мы
увеличим четыре тысячи на какое-то количество процентов.
Узнаем на какое именно. Для этого определим какую часть
одна тысяча составляет от четырех тысяч:

Выразим полученный результат в процентах:

0,25 × 100 = 25%


968

1000 рублей от 4000 рублей составляют 25%. Если


прибавить эти 25% к 4000, то получится 5000 рублей. Значит,
5000 рублей на 25% больше, чем 4000 рублей

Задача 3. На сколько процентов 4000 рублей меньше, чем


5000 рублей?

В этот раз сравниваем 4000 с 5000. Примем 5000 за 100%.


Пять тысяч больше четырех тысяч на одну тысячу рублей.
Узнаем какую часть одна тысяча составляет от пяти тысяч

Тысяча от пяти тысяч составляет 20%. Если вычесть эти 20%


от 5000 рублей, то получим 4000 рублей.

Значит, 4000 рублей меньше 5000 рублей на 20%


969

Задачи на концентрацию, сплавы и смеси

Допустим, возникло желание приготовить какой-нибудь сок. У


нас в распоряжении имеется вода и малиновый сироп

Нальем 200 мл воды в стакан:


970

Добавим 50 мл малинового сиропа и размешаем полученную


жидкость. В результате у нас получится 250 мл малинового
сока (200 мл воды + 50 мл сиропа = 250 мл сока)

Какую часть от получившегося сока составляет малиновый


сироп?
971

Малиновый сироп составляет сока. Вычислим это


отношение, получим число 0,20. Это число показывает
количество растворённого сиропа в получившемся соке.
Назовём это число концентрацией сиропа.

Концентрацией растворённого вещества называют


отношение количества растворённого вещества или
его массы к объему раствора.

Концентрация обычно выражается в процентах. Давайте


выразим концентрацию сиропа в процентах:

0,20 × 100 = 20%

Таким образом, концентрация сиропа в малиновом соке


составляет 20%.

Вещества в растворе могут быть неоднородными. Например,


смешаем 3 л воды и 200 г соли.

Масса 1 л воды составляет 1 кг. Тогда масса 3 л воды будет


составлять 3 кг. Переведем 3 кг в граммы, получим
3 кг = 3000 г.

Теперь в 3000 г воды опустим 200 г соли и смешаем


полученную жидкость. В результате получится соленный
раствор, общая масса которого будет составлять 3000+200,
то есть 3200 г. Найдем концентрацию соли в полученном
растворе. Для этого найдём отношение массы растворенной
соли к массе раствора
972

Значит, при смешивании 3 л воды и 200 г соли получится


6,25%-й раствор соли.

Аналогично может быть определено количество вещества в


сплаве или в смеси. Например, сплав содержит олово
массой 210 г, и серебро массой 90 г. Тогда масса сплава
будет составлять 210+90, то есть 300 г. Олова в сплаве

будет содержаться  , а серебра . В процентном


соотношении олова будет 70%, а серебра 30%

При смешивании двух растворов получается новый раствор,


состоящий из первого и второго растворов. У нового
раствора концентрация вещества может быть другой.
Полезным навыком является умение решать задачи на
концентрацию, сплавы и смеси. В общем итоге смысл таких
задач заключается в отслеживании изменений, которые
происходят при смешивании растворов различной
концентрации.

Смешаем два малиновых сока. Первый сок объемом 250 мл


содержит 12,8% малинового сиропа. А второй сок объемом
300 мл содержит 15% малинового сиропа. Сольем эти два
сока в большой стакан и смешаем. В результате получим
новый сок объемом 550 мл.
973

Теперь определим концентрацию сиропа в полученном соке.


Первый слитый сок объемом 250 мл содержал 12,8% сиропа.
А 12,8% от 250 мл это 32 мл. Значит, первый сок содержал
32 мл сиропа.

Второй слитый сок объемом 300 мл содержал 15% сиропа. А


15% от 300 мл это 45 мл. Значит, второй сок содержал 45 мл
сиропа.

Сложим количества сиропов:

32 мл + 45 мл = 77 мл

Эти 77 мл сиропа содержатся в новом соке, объем которого


составляет 550 мл. Определим концентрацию сиропа в этом
соке. Для этого найдём отношение 77 мл растворённого
сиропа к объему сока 550 мл:
974

Значит, при смешивании 12,8%-го малинового сока объемом


250 мл и 15%‍-го малинового сока объемом 300 мл,
получается 14%-й малиновый сок объемом 550 мл.

Задача 1. Имеются 3 раствора морской соли в воде: первый


раствор содержит 10% соли, второй содержит 15% соли и
третий — 20% соли. Смешали 130 мл первого раствора, 200
мл второго раствора и 170 мл третьего раствора.
Определите сколько процентов составляет морская соль в
полученном растворе.

Решение

Определим объем полученного раствора:

130 мл + 200 мл + 170 мл = 500 мл

Поскольку в первом растворе было 130 × 0,10 = 13 мл


морской соли, во втором растворе 200 × 0,15 = 30 мл
морской соли, а в третьем — 170 × 0,20 = 34 мл морской
соли, то в полученном растворе будет содержаться
13 + 30 + 34 = 77 мл морской соли.

Определим концентрацию морской соли в полученном


растворе. Для этого найдём отношение 77 мл морской соли к
объему раствора 500 мл

Значит, в полученном растворе содержится 15,4% морской


соли.
975

Задача 2. Сколько граммов воды надо добавить к 50 г


раствора, содержащего 8% соли, чтобы получить 5%-й
раствор?

Решение

Заметим, что если к имеющемуся раствору добавить воды,


то количество соли в нём не изменится. Изменится только её
процентное содержание, поскольку добавление воды в
раствор приведёт к изменению его массы.

Нам нужно добавить такое количество воды при котором


восемь процентов соли стали бы пятью процентами.

Определим сколько граммов соли содержится в 50 г


раствора. Для этого найдем 8% от 50

50 г × 0,08 = 4 г

8% от 50 г составляют 4 г. Другими словами, на восемь


частей из ста приходятся 4 грамма соли. Давайте сделаем
так, чтобы эти 4 грамма приходились не на восемь частей, а
на пять частей, то есть на 5%

4 грамма — 5%

Теперь зная, что на 5% раствора приходятся 4 грамма, мы


можем найти массу всего раствора. Для этого нужно найти
число по его проценту:

4 г : 5 = 0,8 г
0,8 г × 100 = 80 г

80 граммов раствора это масса при которой 4 грамма соли


будут приходиться на 5% раствора. А для получения этих 80
976

граммов, нужно к изначальным 50 граммам добавить 30


граммов воды.

Значит, для получения 5%-го раствора соли, нужно к


имеющемуся раствору добавить 30 г воды.

Задача 2. Виноград содержит 91% влаги, а изюм – 7%.


Сколько килограммов винограда требуется для получения 21
килограмма изюма?

Решение

Виноград состоит из влаги и чистого вещества. Если в


свежем винограде содержится 91% влаги, то на остальные
9% будет приходиться чистое вещество этого винограда:

Изюм же содержит 93% чистого вещества и 7% влаги:

Заметим, что в процессе превращения винограда в изюм,


исчезает только влага этого винограда. Чистое вещество
остаётся без изменения. После того, как виноград
превратится в изюм, в получившемся изюме будет 7% влаги
и 93% чистого вещества.
977

Определим сколько чистого вещества содержится в 21 кг


изюма. Для этого найдем 93% от 21 кг

21 кг × 0,93 = 19,53 кг

Теперь вернемся к первому рисунку. Наша задача состояла в


том, чтобы определить сколько винограда нужно взять для
получения 21 кг изюма. Чистое вещество массой 19,53 кг
будет приходиться на 9% винограда:

Теперь зная, что 9% чистого вещества составляют 19,53 кг,


мы можем определить сколько винограда требуется для
получения 21 кг изюма. Для этого нужно найти число по его
проценту:

19,53 кг : 9 = 2,17 кг
2,17 кг × 100 = 217 кг

Значит, для получения 21 кг изюма нужно взять 217 кг


винограда.

Задача 3. В сплаве олова и меди медь составляет 85%.


Сколько надо взять сплава, чтобы в нём содержалось 4,5 кг
олова?

Решение
978

Если в сплаве медь составляет 85%, то на остальные 15%


будет приходиться олово:

Спрашивается сколько надо взять сплава, чтобы в нем


содержалось 4,5 олова. Поскольку олова в сплаве
содержится 15%, то 4,5 кг олова и будут приходиться на эти
15%.

А зная, что 4,5 кг сплава составляют 15% мы можем


определить массу всего сплава. Для этого нужно найти число
по его проценту:

4,5 кг : 15 = 0,3 кг
0,3 кг × 100 = 30 кг

Значит, сплава нужно взять 30 кг, чтобы в нём содержалось


4,5 кг олова.

Задача 4. Смешали некоторое количество 12%-го раствора


соляной кислоты с таким же количеством 20%-го раствора
этой же кислоты. Найти концентрацию получившейся
соляной кислоты.

Решение

Изобразим на рисунке первый раствор в виде прямой линии


и выделим на нём 12%
979

Поскольку количество растворов одинаково, рядом можно


изобразить такой же рисунок, иллюстрирующий второй
раствор с содержанием соляной кислоты 20%

У нас получилось двести частей раствора (100% + 100%),


тридцать две части из которых составляют соляную кислоту
(12% + 20%)

Определим какую часть 32 части составляют от 200 частей

Значит, при смешивании 12%-го раствора соляной кислоты с


таким же количеством 20%-го раствора этой же кислоты
получится 16%-й раствор соляной кислоты.

Для проверки представим, что масса первого раствора была


2 кг. Масса второго раствора так же будет составлять 2 кг.
Тогда при смешивании этих растворов получится 4 кг
раствора. В первом растворе соляной кислоты было
980

2 × 0,12 = 0,24 кг, а во втором — 2 × 0,20 = 0,40 кг. Тогда в


новом растворе соляной кислоты будет 0,24 + 0,40 = 0,64 кг.
Концентрация соляной кислоты составит 16%

Задачи для самостоятельного решения


Задание 1. Выразите в виде обыкновенной дроби следующие
части:

Показать решение
Задание 2. Выразите в виде обыкновенной дроби следующие
части:

Показать решение
Задание 3. Выразите в виде десятичной дроби следующие
части:

Показать решение
Задание 4. Изобразите графически следующие части:

Показать решение
Задание 5. Опишите следующий рисунок в виде процентов:
981

Показать решение
Задание 6. Опишите следующий рисунок в виде процентов:

Показать решение
Задание 7. Опишите следующий рисунок в виде процентов:

Показать решение
Задание 8. Опишите следующий рисунок в виде процентов:

Показать решение
Задание 9. Опишите следующий рисунок в виде процентов:

Показать решение
982

Задача 10. Число 50 увеличили на 20%. Найти новое


значение числа.
Показать решение

Задача 11. Число увеличили на 60%. Найти новое значение


числа.
Показать решение

Задача 12. Ответьте на следующие вопросы:

1) Потратили 80 % суммы. Сколько процентов этой суммы


осталось?
2) Мужчины составляют 75 % всех работников завода.
Сколько процентов работников завода составляют
женщины?
3) Девочки составляют 40 % класса. Сколько процентов
класса составляют мальчики?

Показать решение

Задача 13. Ответьте на следующие вопросы:

1) В магазин привезли 2500 кг помидоров. В первый день


продали 30% всех помидоров. Сколько килограммов
помидоров осталось продать?
2) В школе 400 учащихся, 52 % этого числа составляют
девочки. Сколько мальчиков в школе?

Показать решение
Задача 14. Число увеличили на 25%. На сколько процентов
надо уменьшить новое число, чтобы получилось исходное?
Показать решение
983

Задача 15. Число уменьшили на 50%. На сколько надо


увеличить новое число, чтобы получилось исходное?
Показать решение
Задача 16. В прошлом месяце в городе произошло 15 ДТП.
В этом месяце этот показатель снизился до 6. На сколько
процентов снизилось количество ДТП?
Показать решение
Задача 17. Смешали 8 кг 18%-го раствора некоторого
вещества с 12 кг 8%-го раствора этого же вещества. Найдите
концентрацию получившегося раствора.
Показать решение
Задача 18. Смешали некоторое количество 11%-го раствора
некоторого вещества с таким же количеством 19% раствора
этого же вещества. Найдите концентрацию получившегося
раствора.
Показать решение
Задача 19. За последние 3 месяца цены на продукты питания
росли в среднем на 10% за каждый месяц. На сколько
процентов выросли цены за 3 месяца?
Показать решение
984

Задачи на движение

Продолжаем изучать элементарные задачи по математике.


Данный урок посвящен задачам на движение.

Предварительные навыки

 Расстояние, скорость, время


 Простейшие задачи по математике
 Задачи на дроби
 Задачи на проценты

Содержание урока

 Задача на нахождение расстояния/скорости/времени


 Скорость сближения
 Скорость удаления
 Задача на движение объектов в одном направлении
 Задача на движение по реке
 Задачи для самостоятельного решения

Задача на нахождение расстояния/скорости/времени

Задача 1. Автомобиль двигается со скоростью 80 км/ч.


Сколько километров он проедет за 3 часа?

Решение

Если за один час автомобиль проезжает 80 километров, то за


3 часа он проедет в три раза больше. Чтобы найти
расстояние, нужно скорость автомобиля (80км/ч) умножить
на время движения (3ч)

80 × 3 = 240 км
985

Ответ: за 3 часа автомобиль проедет 240 километров.

Задача 2. На автомобиле за 3 часа проехали 180 км с одной


и той же скоростью. Чему равна скорость автомобиля?

Решение

Скорость — это расстояние, пройденное телом за единицу


времени. Под единицей подразумевается 1 час, 1 минута или
1 секунда.

Если за 3 часа автомобиль проехал 180 километров с одной


и той же скоростью, то разделив 180 км на 3 часа мы
определим расстояние, которое проезжал автомобиль за
один час. А это есть скорость движения. Чтобы определить
скорость, нужно пройденное расстояние разделить на время
движения:

180 : 3 = 60 км/ч
986

Ответ: скорость автомобиля составляет 60 км/ч

Задача 3. За 2 часа автомобиль проехал 96 км, а


велосипедист за 6 часов проехал 72 км. Во сколько раз
автомобиль двигался быстрее велосипедиста?

Решение

Определим скорость движения автомобиля. Для этого


разделим пройденное им расстояние (96км) на время его
движения (2ч)

96 : 2 = 48 км/ч

Определим скорость движения велосипедиста. Для этого


разделим пройденное им расстояние (72км) на время его
движения (6ч)

72 : 6 = 12 км/ч

Узнаем во сколько раз автомобиль двигался быстрее


велосипедиста. Для этого найдем отношение 48 к 12
987

Ответ: автомобиль двигался быстрее велосипедиста в 4


раза.

Задача 4. Вертолет преодолел расстояние в 600 км со


скоростью 120 км/ч. Сколько времени он был в полете?

Решение

Если за 1 час вертолет преодолевал 120 километров, то


узнав сколько таких 120 километров в 600 километрах, мы
определим сколько времени он был в полете. Чтобы найти
время, нужно пройденное расстояние разделить на скорость
движения

600 : 120 = 5 часов

Ответ: вертолет был в пути 5 часов.

Задача 5. Вертолет летел 6 часов со скоростью 160 км/ч.


Какое расстояние он преодолел за это время?

Решение
988

Если за 1 час вертолет преодолевал 160 км, то за 6 часов, он


преодолел в шесть раз больше. Чтобы определить
расстояние, нужно скорость движения умножить на время

160 × 6 = 960 км

Ответ: за 6 часов вертолет преодолел 960 км.

Задача 6. Расстояние от Перми до Казани, равное 723 км,


автомобиль проехал за 13 часов. Первые 9 часов он ехал со
скоростью 55 км/ч. Определить скорость автомобиля в
оставшееся время.

Решение

Определим сколько километров автомобиль проехал за


первые 9 часов. Для этого умножим скорость с которой он
ехал первые девять часов (55км/ч) на 9

55 × 9 = 495 км

Определим сколько осталось проехать. Для этого вычтем из


общего расстояния (723км) расстояние, пройденное за
первые 9 часов движения
989

723 − 495 = 228 км

Эти 228 километров автомобиль проехал за оставшиеся 4


часа. Чтобы определить скорость автомобиля в оставшееся
время, нужно 228 километров разделить на 4 часа:

228 : 4 = 57 км/ч

Ответ: скорость автомобиля в оставшееся время составляла


57 км/ч

Скорость сближения

Скорость сближения — это расстояние, пройденное двумя


объектами навстречу друг другу за единицу времени.

Например, если из двух пунктов навстречу друг другу


отправятся два пешехода, причем скорость первого будет
100 м/м, а второго — 105 м/м, то скорость сближения будет
составлять 100 + 105, то есть 205 м/м. Это значит, что
каждую минуту расстояние между пешеходами будет
уменьшáться на 205 метров

Чтобы найти скорость сближения, нужно сложить


скорости объектов.
990

Предположим, что пешеходы встретились через три минуты


после начала движения. Зная, что они встретились через три
минуты, мы можем узнать расстояние между двумя пунктами.

Каждую минуту пешеходы преодолевали расстояние равное


двухсот пяти метрам. Через 3 минуты они встретились.
Значит умножив скорость сближения на время движения,
можно определить расстояние между двумя пунктами:

205 × 3 = 615 метров

Можно и по другому определить расстояние между пунктами.


Для этого следует найти расстояние, которое прошел каждый
пешеход до встречи.
991

Так, первый пешеход шел со скоростью 100 метров в минуту.


Встреча состоялась через три минуты, значит за 3 минуты он
прошел 100 × 3 метров

100 × 3 = 300 метров

А второй пешеход шел со скоростью 105 метров в минуту. За


три минуты он прошел 105 × 3 метров

105 × 3 = 315 метров

Теперь можно сложить полученные результаты и таким


образом определить расстояние между двумя пунктами:

300 м + 315 м = 615 м

Задача 1. Из двух населенных пунктов навстречу друг другу


выехали одновременно два велосипедиста. Скорость
первого велосипедиста 10 км/ч, а скорость второго — 12 км/ч.
Через 2 часа они встретились. Определите расстояние
между населенными пунктами

Решение

Найдем скорость сближения велосипедистов

10 км/ч + 12 км/ч = 22 км/ч

Определим расстояние между населенными пунктами. Для


этого скорость сближения умножим на время движения

22 × 2 = 44 км
992

Решим эту задачу вторым способом. Для этого найдем


расстояния, пройденные велосипедистами и сложим
полученные результаты.

Найдем расстояние, пройденное первым велосипедистом:

10 × 2 = 20 км

Найдем расстояние, пройденное вторым велосипедистом:

12 × 2 = 24 км

Сложим полученные расстояния:

20 км + 24 км = 44 км

Ответ: расстояние между населенными пунктами составляет


44 км.
993

Задача 2. Из двух населенных пунктов, расстояние между


которыми 60 км, навстречу друг другу выехали одновременно
два велосипедиста. Скорость первого велосипедиста 14 км/ч,
а скорость второго — 16 км/ч. Через сколько часов они
встретились?

Решение

Найдем скорость сближения велосипедистов:

14 км/ч + 16 км/ч = 30 км/ч

За один час расстояние между велосипедистами


уменьшается на 30 километров. Чтобы определить через
сколько часов они встретятся, нужно расстояние между
населенными пунктами разделить на скорость сближения:

60 : 30 = 2 часа

Значит велосипедисты встретились через два часа


994

Ответ: велосипедисты встретились через 2 часа.

Задача 3. Из двух населенных пунктов, расстояние между


которыми 56 км, навстречу друг другу выехали одновременно
два велосипедиста. Через два часа они встретились. Первый
велосипедист ехал со скоростью 12 км/ч. Определить
скорость второго велосипедиста.

Решение

Определим расстояние пройденное первым велосипедистом.


Как и второй велосипедист в пути он провел 2 часа. Умножив
скорость первого велосипедиста на 2 часа, мы сможем
узнать сколько километров он прошел до встречи

12 × 2 = 24 км

За два часа первый велосипедист прошел 24 км. За один час


он прошел 24:2, то есть 12 км. Изобразим это графически
995

Вычтем из общего расстояния (56 км) расстояние,


пройденное первым велосипедистом (24 км). Так мы
определим сколько километров прошел второй
велосипедист:

56 км − 24 км = 32 км

Второй велосипедист, как и первый провел в пути 2 часа.


Если мы разделим пройденное им расстояние на 2 часа, то
узнаем с какой скоростью он двигался:

32 : 2 = 16 км/ч

Значит скорость второго велосипедиста составляет 16 км/ч.

Ответ: скорость второго велосипедиста составляет 16 км/ч.

Скорость удаления

Скорость удаления — это расстояние, которое


увеличивается за единицу времени между двумя объектами,
двигающимися в противоположных направлениях.

Например, если два пешехода отправятся из одного и того


же пункта в противоположных направлениях, причем
скорость первого будет 4 км/ч, а скорость второго 6 км/ч, то
скорость удаления будет составлять 4+6, то есть 10 км/ч.
Каждый час расстояние между двумя пешеходами будет
увеличиться на 10 километров.

Чтобы найти скорость удаления, нужно сложить скорости


объектов.
996

Так, за первый час расстояние между пешеходами будет


составлять 10 километров. На следующем рисунке можно
увидеть, как это происходит

Видно, что первый пешеход прошел свои 4 километра за


первый час. Второй пешеход также прошел свои 6
километров за первый час. Итого за первый час расстояние
между ними стало 4+6, то есть 10 километров.

Через два часа расстояние между пешеходами будет


составлять 10×2, то есть 20 километров. На следующем
рисунке можно увидеть, как это происходит:
997

Задача 1. От одной станции отправились одновременно в


противоположных направлениях товарный поезд и
пассажирский экспресс. Скорость товарного поезда
составляла 40 км/ч, скорость экспресса 180 км/ч. Какое
расстояние будет между этими поездами через 2 часа?

Решение

Определим скорость удаления поездов. Для этого сложим их


скорости:

40 + 180 = 220 км/ч

Получили скорость удаления поездов равную 220 км/ч.


Данная скорость показывает, что за час расстояние между
поездами будет увеличиваться на 220 километров. Чтобы
узнать какое расстояние будет между поездами через два
часа, нужно 220 умножить на 2
998

220 × 2 = 440 км

Ответ: через 2 часа расстояние будет между поездами будет


440 километров.

Задача 2. Из пункта одновременно в противоположных


направлениях отправились велосипедист и мотоциклист.
Скорость велосипедиста 16 км/ч, а скорость мотоциклиста —
40 км/ч. Какое расстояние будет между велосипедистом и
мотоциклистом через 2 часа?

Решение

Определим скорость удаления велосипедиста и


мотоциклиста. Для этого сложим их скорости:

16 км/ч + 40 км/ч = 56 км/ч

Определим расстояние, которое будет между


велосипедистом и мотоциклистом через 2 часа. Для этого
скорость удаления (56км/ч) умножим на 2 часа

56 × 2 = 112 км
999

Ответ: через 2 часа расстояние между велосипедистом и


мотоциклистом будет 112 км.

Задача 3. Из пункта одновременно в противоположных


направлениях отправились велосипедист и мотоциклист.
Скорость велосипедиста 10 км/ч, а скорость мотоциклиста —
30 км/ч. Через сколько часов расстояние между ними будет
80 км?

Решение

Определим скорость удаления велосипедиста и


мотоциклиста. Для этого сложим их скорости:

10 км/ч + 30 км/ч = 40 км/ч

За один час расстояние между велосипедистом и


мотоциклистом увеличивается на 40 километров. Чтобы
узнать через сколько часов расстояние между ними будет 80
км, нужно определить сколько раз 80 км содержит по 40 км
1000

80 : 40 = 2

Ответ: через 2 часа после начала движения, между


велосипедистом и мотоциклистом будет 80 километров.

Задача 4. Из пункта одновременно в противоположных


направлениях отправились велосипедист и мотоциклист.
Через 2 часа расстояние между ними было 90 км. Скорость
велосипедиста составляла 15 км/ч. Определить скорость
мотоциклиста

Решение

Определим расстояние, пройденное велосипедистом за 2


часа. Для этого умножим его скорость (15 км/ч) на 2 часа

15 × 2 = 30 км
1001

На рисунке видно, что велосипедист прошел по 15


километров в каждом часе. Итого за два часа он прошел 30
километров.

Вычтем из общего расстояния (90 км) расстояние,


пройденное велосипедистом (30 км). Так мы определим
сколько километров прошел мотоциклист:

90 км − 30 км = 60 км

Мотоциклист за два часа прошел 60 километров. Если мы


разделим пройденное им расстояние на 2 часа, то узнаем с
какой скоростью он двигался:

60 : 2 = 30 км/ч

Значит скорость мотоциклиста составляла 30 км/ч.


1002

Ответ: скорость мотоциклиста составляла 30 км/ч.

Задача на движение объектов в одном направлении

В предыдущей теме мы рассматривали задачи в которых


объекты (люди, машины, лодки) двигались либо навстречу
другу другу либо в противоположных направлениях. При
этом мы находили различные расстояния, которые
изменялись между объектами в течении определенного
времени. Эти расстояния были либо скоростями сближения
либо скоростями удаления.

В первом случае мы находили скорость сближения — в


ситуации, когда два объекта двигались навстречу друг другу.
За единицу времени расстояние между объектами
уменьшалось на определенное расстояние
1003

Во втором случае мы находили скорость удаления — в


ситуации, когда два объекта двигались в противоположных
направлениях. За единицу времени расстояние между
объектами увеличивалось на определенное расстояние

Но объекты также могут двигаться в одном направлении,


причем с различной скоростью. Например, из одного пункта
одновременно могут выехать велосипедист и мотоциклист,
причем скорость велосипедиста может составлять 20
километров в час, а скорость мотоциклиста — 40 километров
в час
1004

На рисунке видно, что мотоциклист впереди велосипедиста


на двадцать километров. Связано это с тем, что в час он
преодолевает на 20 километров больше, чем велосипедист.
Поэтому каждый час расстояние между велосипедистом и
мотоциклистом будет увеличиваться на двадцать
километров.

В данном случае 20 км/ч являются скоростью удаления


мотоциклиста от велосипедиста.

Через два часа расстояние, пройденное велосипедистом


будет составлять 40 км. Мотоциклист же проедет 80 км,
отдалившись от велосипедиста еще на двадцать километров
— итого расстояние между ними составит 40 километров
1005

Чтобы найти скорость удаления при движении в одном


направлении, нужно из большей скорости вычесть меньшую
скорость.

В приведенном выше примере, скорость удаления


составляет 20 км/ч. Её можно найти путем вычитания
скорости велосипедиста из скорости мотоциклиста. Скорость
велосипедиста составляла 20 км/ч, а скорость мотоциклиста
— 40 км/ч. Скорость мотоциклиста больше, поэтому из 40
вычитаем 20

40 км/ч − 20 км/ч = 20 км/ч

Задача 1. Из города в одном и том же направлении выехали


легковой автомобиль и автобус. Скорость автомобиля 120
км/ч, а скорость автобуса 80 км/ч. Какое расстояние будет
между ними через 1 час? 2 часа?

Решение
1006

Найдем скорость удаления. Для этого из большей скорости


вычтем меньшую

120 км/ч − 80 км/ч = 40 км/ч

Каждый час легковой автомобиль отдаляется от автобуса на


40 километров. За один час расстояние между автомобилем
и автобусом будет 40 км. За 2 часа в два раза больше:

40 × 2 = 80 км

Ответ: через один час расстояние между автомобилем и


автобусом будет 40 км, через два часа — 80 км.

Рассмотрим ситуацию в которой объекты начали свое


движение из разных пунктов, но в одном направлении.

Пусть имеется дом, школа и аттракцион. От дома до школы


700 метров

Два пешехода отправились в аттракцион в одно и то же


время. Причем первый пешеход отправился в аттракцион от
дома со скоростью 100 метров в минуту, а второй пешеход
отправился в аттракцион от школы со скоростью 80 метров
в минуту. Какое расстояние будет между пешеходами через
2 минуты? Через сколько минут после начала движения
первый пешеход догонит второго?
1007

Ответим на первый вопрос задачи — какое расстояние будет


между пешеходами через 2 минуты?

Определим расстояние, пройденное первым пешеходом за 2


минуты. Он двигался со скоростью 100 метров в минуту. За
две минуты он пройдет в два раза больше, то есть 200
метров

100 × 2 = 200 метров

Определим расстояние, пройденное вторым пешеходом за 2


минуты. Он двигался со скоростью 80 метров в минуту. За
две минуты он пройдет в два раза больше, то есть 160
метров

80 × 2 = 160 метров

Теперь нужно найти расстояние между пешеходами


1008

Чтобы найти расстояние между пешеходами, можно к


расстоянию от дома до школы (700м) прибавить расстояние,
пройденное вторым пешеходом (160м) и из полученного
результата вычесть расстояние, пройденное первым
пешеходом (200м)

700 м + 160 м = 860 м

860 м − 200 м = 660 м

Либо из расстояния от дома до школы (700м) вычесть


расстояние, пройденное первым пешеходом (200м), и к
полученному результату прибавить расстояние, пройденное
вторым пешеходом (160м)

700 м − 200 м = 500 м

500 м + 160 м = 660 м

Таким образом, через две минуты расстояние между


пешеходами будет составлять 660 метров
1009

Попробуем ответить на следующий вопрос задачи: через


сколько минут после начала движения первый пешеход
догонит второго?

Давайте посмотрим какой была ситуация в самом начале


пути — когда пешеходы еще не начали своё движение
1010

Как видно на рисунке, расстояние между пешеходами в


начале пути составляло 700 метров. Но уже через минуту
после начала движения расстояние между ними будет
составлять 680 метров, поскольку первый пешеход двигается
на 20 метров быстрее второго:

100 м × 1 = 100 м

80 м × 1 = 80 м

700 м + 80 м − 100 м = 780 м − 100 м = 680 м

Через две минуты после начала движения, расстояние


уменьшится еще на 20 метров и будет составлять 660
метров. Это был наш ответ на первый вопрос задачи:

100 м × 2 = 200 м

80 м × 2 = 160 м
1011

700 м + 160 м − 200м = 860 м − 200 м = 660 м

Через три минуты расстояние уменьшится еще на 20 метров


и будет уже составлять 640 метров:

100 м × 3 = 300 м

80 м × 3 = 240 м

700 м + 240 м − 300м = 940 м − 300 м = 640 м


1012

Мы видим, что с каждой минутой первый пешеход будет


приближáться ко второму на 20 метров, и в конце концов
догонит его. Можно сказать, что скорость равная двадцати
метрам в минуту является скоростью сближения пешеходов.
Правила нахождения скорости сближения и удаления при
движении в одном направлении идентичны.

Чтобы найти скорость сближения при движении в одном


направлении, нужно из большей скорости вычесть меньшую.

А раз изначальные 700 метров с каждой минутой


уменьшаются на одинаковые 20 метров, то мы можем узнать
сколько раз 700 метров содержат по 20 метров, тем самым
определяя через сколько минут первый пешеход догонит
второго

700 : 20 = 35

Значит через 35 минут после начала движения первый


пешеход догонит второго. Для интереса узнаем сколько
метров прошел к этому времени каждый пешеход. Первый
1013

двигался со скоростью 100 метров в минуту. За 35 минут он


прошел в 35 раз больше

100 × 35 = 3500 м

Второй шел со скоростью 80 метров в минуту. За 35 минут он


прошел в 35 раз больше

80 × 35 = 2800 м

Первый прошел 3500 метров, а второй 2800 метров. Первый


прошел на 700 метров больше, поскольку он шел от дома.
Если вычесть эти 700 метров из 3500, то мы получим 2800 м

Рассмотрим ситуацию в которой объекты движутся в одном


направлении, но один из объектов начал своё движение
раньше другого.

Пусть имеется дом и школа. Первый пешеход отправился в


школу со скоростью 80 метров в минуту. Через 5 минут вслед
за ним в школу отправился второй пешеход со скоростью 100
1014

метров в минуту. Через сколько минут второй пешеход


догонит первого?

Второй пешеход начал свое движение через 5 минут. К этому


времени первый пешеход уже отдалился от него на какое-то
расстояние. Найдём это расстояние. Для этого умножим его
скорость (80 м/м) на 5 минут

80 × 5 = 400 метров

Первый пешеход отдалился от второго на 400 метров.


Поэтому в момент, когда второй пешеход начнет свое
движение, между ними будут эти самые 400 метров.

Но второй пешеход двигается со скоростью 100 метров в


минуту. То есть двигается на 20 метров быстрее первого
пешехода, а значит с каждой минутой расстояние между
ними будет уменьшáться на 20 метров. Наша задача узнать
через сколько минут это произойдет.

Например, уже через минуту расстояние между пешеходами


будет составлять 380 метров. Первый пешеход к своим 400
метрам пройдет еще 80 метров, а второй пройдет 100
метров
1015

Принцип здесь такой-же, как и в предыдущей задаче.


Расстояние между пешеходами в момент движения второго
пешехода необходимо разделить на скорость сближения
пешеходов. Скорость сближения в данном случае равна
двадцати метрам. Поэтому, чтобы определить через сколько
минут второй пешеход догонит первого, нужно 400 метров
разделить на 20

400 : 20 = 20

Значит через 20 минут второй пешеход догонит первого.

Задача 2. Из двух сел, расстояние между которыми 40 км,


одновременно в одном направлении выехали автобус и
велосипедист. Скорость велосипедиста 15 км/ч, а скорость
автобуса 35 км/ч. Через сколько часов автобус догонит
велосипедиста?
1016

Решение

Найдем скорость сближения

35 км/ч − 15 км/ч = 20 км/ч

Определим через часов автобус догонит велосипедиста

40 : 20 = 2

Ответ: автобус догонит велосипедиста через 2 часа.

Задача на движение по реке

Суда двигаются по реке с различной скоростью. При этом


они могут двигаться, как по течению реки, так и против
течения. В зависимости от того, как они двигаются (по или
против течения), скорость будет меняться.

Предположим, что скорость реки составляет 3 км/ч. Если


спустить лодку на реку, то река унесет лодку со скоростью 3
км/ч.

Если спустить лодку на стоячую воду, в которой отсутствует


течение, то и лодка будет стоять. Скорость движения лодки в
этом случае будет равна нулю.

Если лодка плывет по стоячей воде, в которой отсутствует


течение, то говорят, что лодка плывет с собственной
скоростью.

Например, если моторная лодка плывет по стоячей воде со


скоростью 40 км/ч, то говорят что собственная скорость
моторной лодки составляет 40 км/ч.
1017

Как определить скорость судна?

Если судно плывет по течению реки, то к собственной


скорости судна нужно прибавить скорость течения реки.

Например, если моторная лодка плывет со скоростью 30 км/ч


по течению реки, и скорость течения реки составляет 2 км/ч,
то к собственной скорости моторной лодки (30 км/ч)
необходимо прибавить скорость течения реки (2 км/ч)

30 км/ч + 2 км/ч = 32 км/ч

Течение реки можно сказать помогает моторной лодке


дополнительной скоростью равной двум километрам в час.

Если судно плывет против течения реки, то из


собственной скорости судна нужно вычесть скорость
течения реки.

Например, если моторная лодка плывет со скоростью 30 км/ч


против течения реки, и скорость течения реки составляет 2
км/ч, то из собственной скорости моторной лодки (30 км/ч)
необходимо вычесть скорость течения реки (2 км/ч)

30 км/ч − 2 км/ч = 28 км/ч

Течение реки в этом случае препятствует моторной лодке


свободно двигаться вперед, снижая её скорость на два
километра в час.

Задача 1. Скорость катера 40 км/ч, а скорость течения реки 3


км/ч. С какой скоростью катер будет двигаться по течению
реки? Против течения реки?
1018

Ответ:

Если катер будет двигаться по течения реки, то скорость его


движения составит 40 + 3, то есть 43 км/ч.

Если катер будет двигаться против течения реки, то скорость


его движения составит 40 − 3, то есть 37 км/ч.

Задача 2. Скорость теплохода в стоячей воде — 23 км/ч.


Скорость течения реки — 3 км/ч. Какой путь пройдет
теплоход за 3 часа по течению реки? Против течения?

Решение

Собственная скорость теплохода составляет 23 км/ч. Если


теплоход будет двигаться по течению реки, то скорость его
движения составит 23 + 3, то есть 26 км/ч. За три часа он
пройдет в три раза больше

26 × 3 = 78 км

Если теплоход будет двигаться против течения реки, то


скорость его движения составит 23 − 3, то есть 20 км/ч. За
три часа он пройдет в три раза больше

20 × 3 = 60 км

Задача 3. Расстояние от пункта А до пункта B лодка


преодолела за 3 часа 20 минут, а расстояние от пункта B до
А — за 2 часа 50 минут. В каком направлении течет река: от
А к В или от В к А, если известно, что скорость яхты не
менялась?
1019

Решение

Скорость яхты не менялась. Узнаем на какой путь она


затратила больше времени: на путь от А до В или на путь от
В до А. Тот путь, который затратил больше времени будет
тем путем, течение реки которого шло против яхты

3 часа 20 минут больше, чем 2 часа 50 минут. Это значит, что


течение реки снизило скорость яхты и это отразилось на
времени пути. 3 часа 20 минут это время, затраченное на
путь от от А до В. Значит река течет от пункта B к пункту А

Задача 4. За какое время при движении против течения реки


теплоход пройдет 204 км, если его собственная скорость
15 км/ч, а скорость течения в 5 раз меньше собственной
скорости теплохода?

Решение

Требуется найти время за которое теплоход пройдет 204


километра против течения реки. Собственная скорость
теплохода составляет 15 км/ч. Двигается он против течения
реки, поэтому нужно определить его скорость при таком
движении.

Чтобы определить скорость против течения реки, нужно из


собственной скорости теплохода (15 км/ч) вычесть скорость
движения реки. В условии сказано, что скорость течения реки
в 5 раз меньше собственной скорости теплохода, поэтому
сначала определим скорость течения реки. Для этого
уменьшим 15 км/ч в пять раз

15 : 5 = 3 км/ч
1020

Скорость течения реки составляет 3 км/ч. Вычтем эту


скорость из скорости движения теплохода

15 км/ч − 3 км/ч = 12 км/ч

Теперь определим время за которое теплоход пройдет 204


км при скорости 12 км/ч. В час теплоход проходит 12
километров. Чтобы узнать за сколько часов он пройдет 204
километра, нужно определить сколько раз 204 километра
содержит по 12 километров

204 : 12 = 17 ч

Ответ: теплоход пройдет 204 километра за 17 часов

Задача 5. Двигаясь по течению реки, за 6 часов лодка


прошла 102 км. Определите собственную скорость лодки,
если скорость течения – 4 км/ч.

Решение

Узнаем с какой скоростью лодка двигалась по реке. Для этого


пройденное расстояние (102км) разделим на время
движения (6ч)

102 : 6 = 17 км/ч

Определим собственную скорость лодки. Для этого из


скорости по которой она двигалась по реке (17 км/ч) вычтем
скорость течения реки (4 км/ч)

17 − 4 = 13 км/ч
1021

Задача 6. Двигаясь против течения реки, за 5 часов лодка


прошла 110 км. Определите собственную скорость лодки,
если скорость течения – 4 км/ч.

Решение

Узнаем с какой скоростью лодка двигалась по реке. Для этого


пройденное расстояние (110км) разделим на время
движения (5ч)

110 : 5 = 22 км/ч

Определим собственную скорость лодки. В условии сказано,


что она двигалась против течения реки. Скорость течения
реки составляла 4 км/ч. Это значит, что собственная скорость
лодки была уменьшена на 4. Наша задача прибавить эти 4
км/ч и узнать собственную скорость лодки

22 + 4 = 26 км/ч

Ответ: собственная скорость лодки составляет 26 км/ч

Задача 7. За какое время при движении против течения реки


лодка
пройдет 56 км, если скорость течения – 2 км/ч, а её
собственная скорость на 8 км/ч больше скорости течения?

Решение

Найдем собственную скорость лодки. В условии сказано, что


она на 8 км/ч больше скорости течения. Поэтому для
определения собственной скорости лодки, к скорости
течения (2 км/ч) прибавим еще 8 км/ч
1022

2 км/ч + 8 км/ч = 10 км/ч

Лодка движется против течения реки, поэтому из


собственной скорости лодки (10 км/ч) вычтем скорость
движения реки (2 км/ч)

10 км/ч − 2 км/ч = 8 км/ч

Узнаем за какое время лодка пройдет 56 км. Для этого


расстояние (56км) разделим на скорость движения лодки:

56 : 8 = 7 ч

Ответ: при движении против течения реки лодка пройдет 56


км за 7 часов

Задачи для самостоятельного решения


Задача 1. Сколько времени потребуется пешеходу, чтобы
пройти 20 км, если скорость его равна 5 км/ч?
Показать решение
Задача 2. Из пункта А в пункт В велосипедист ехал 5 часов
со скоростью 16 км/ч, а обратно он ехал по тому же пути со
скоростью 10 км/ч. Сколько времени потратил велосипедист
на обратный путь?
Показать решение
Задача 3. Велосипедист ехал 6 ч с некоторой скоростью.
После того как он проехал ещё 11 км с той же скоростью, его
путь стал равным 83 км. С какой скоростью ехал
велосипедист?
Показать решение
1023

Задача 4. Двигаясь против течения реки, расстояние в 72 км


теплоход проходит за 4ч, а плот такое же расстояние
проплывает за 36 ч. За сколько часов теплоход проплывет
расстояние 110 км, если будет плыть по течению реки?
Показать решение
Задача 5. Из одного пункта одновременно в
противоположных направлениях выехали два
велосипедиста. Один из них ехал со скоростью 11 км/ч, а
второй со скоростью 13 км/ч. Какое расстояние будет между
ними через 4 часа?
Показать решение
Задача 6. От двух пристаней одновременно навстречу друг
другу отошли два теплохода, и через 6 часов они
встретились. Какое расстояние до встречи прошел каждый
теплоход и какое расстояние между пристанями, если один
теплоход шел со скоростью 21 км/ч, а другой — со скоростью
24 км/ч?
Показать решение
Задача 7. Одновременно из Москвы и Уфы вышли два
поезда. Через 16 часов они встретились. Московский поезд
шел со скоростью 51 км/ч. С какой скоростью шел поезд,
вышедший из Уфы, если расстояние между Москвой и Уфой
1520 км? Какое расстояние было между поездами через 5
часов после их встречи?
Показать решение
Задача 8. Из одного пункта одновременно в
противоположных направлениях отправились два автобуса.
Скорость одного автобуса 48 км/ч, другого на 6 км/ч больше.
1024

Через сколько часов расстояние между автобусами будет


равно 510 км?
Показать решение
Задача 9. Расстояние от Ростова-на-Дону до Москвы 1230
км. Из Москвы и Ростова навстречу друг другу вышли два
поезда. Поезд из Москвы идет со скоростью 63 км/ч, а

скорость ростовского поезда составляет скорости


московского поезда. На каком расстоянии от Ростова
встретятся поезда?
Показать решение
Задача 10. От двух пристаней, расстояние между которыми
75 км, навстречу друг другу одновременно отошли две
моторные лодки. Одна шла со скоростью 16 км/ч, а скорость
другой составляла 75% скорости первой лодки. Какое
расстояние будет между лодками через 2 ч?
Показать решение
Задача 11. Легковая машина, скорость которой 62 км/ч,
догоняет грузовую машину, скорость которой 47 км/ч. Через
сколько времени и на каком расстоянии от начала движения
легковая автомашина догонит грузовую, если
первоначальное расстояние между ними было 60 км?
Показать решение
Задача 12. Из одного пункта в одном направлении
одновременно выезжали два мотоциклиста. Скорость одного
35 км/ч, а скорость другого составляла 80% скорости первого
мотоциклиста. Какое расстояние будет между ними через 5
часов?
1025

Показать решение
Задача 13. Мотоциклист, скорость которого 43 км/ч, догоняет
велосипедиста, скорость которого 13 км/ч. Через сколько
часов мотоциклист догонит велосипедиста, если
первоначальное расстояние между ними было 120 км?
Показать решение
Задача 14. Велосипедист, скорость которого 12 км/ч,
догоняет велосипедиста, скорость которого составляет 75 %
его скорости. Через 6 часов второй велосипедист догнал
велосипедиста, ехавшего первым. Какое расстояние было
между велосипедистами первоначально?
Показать решение
Задача 15. Автомобиль и автобус выехали одновременно из
одного пункта в одном направлении. Скорость автомобиля 53
км/ч, скорость автобуса 41 км/ч. Через сколько часов после
выезда автомобиль будет впереди автобуса на 48 км?
Показать решение
1026

Производительность

Продолжаем изучать элементарные задачи по математике.


Сегодня мы рассмотрим очень интересную физическую
величину — производительность.

Содержание урока

 Что такое сила?


 Что такое работа?
 Производительность
 Задачи на совместную работу
 Задачи для самостоятельного решения

Что такое сила?

Сила — это физическое явление, способное изменять форму


материальных тел, вызывать их движение, менять
направление и скорость движения этих тел или приводить
тело в состояние покоя.

Примеры сил:

 ребята слепили снеговика, а хулиганы его разрушили.


Получается, что хулиганы приложили к снеговику
свою силу, тем самым вызвали изменение формы
снеговика;

 на дворе стояла тележка. Прохожий случайно задел её и


тележка сдвинулась с места. Получается, что прохожий
применил силу к тележке и вызвал её движение;
1027

 далее тот же прохожий остановил тележку, чтобы она


далеко не уехала. Получается, что прохожий применил
силу, тем самым привел тележку в состояние покоя.

Сила является физической величиной — мерой воздействия


на тело других тел. Сила обозначается заглавной латинской
буквой F.

Что такое работа?

Работа — это количественная мера действия силы на тело.


Работа зависит от количества силы, приложенной на тело и
от направления этой силы, а также от перемещения данного
тела.

Например, если мы попробуем сдвинуть шкаф с места и он


сдвинется, то можно сказать, что мы совершили работу,
поскольку сила, которую мы приложили, привела к тому, что
шкаф совершил перемещение на некоторое расстояние.

Если же мы, к примеру, попробуем толкнуть стену, то стена с


места не сдвинется, а значит и работа не будет совершена,
поскольку сила была приложена, но эта сила не вызвала
никакого перемещения стены.

Работа обозначается заглавной латинской  буквой A.

Производительность

Производительностью называют работу, выполненную за


единицу времени. Под единицей подразумевается 1 час, 1
1028

минута или 1 секунда. Производительность обозначается


латинской буквой v

Рассмотрим следующий пример. Два пекаря пекли булочки.


Первый пекарь испёк 40 булочек за 10 минут, а второй 15
булочек за 5 минут. Как узнать, кто из пекарей работал
быстрее, первый или второй?

Работал быстрее тот, кто за одну минуту выпекает больше


булочек. Говорят, что у него производительность больше.
Для нахождения производительности предусмотрено
следующее правило:

Чтобы найти производительность, надо выполненную


работу разделить на время работы.

Также, можно воспользоваться формулой:

где v — производительность, A — выполненная работа, t —


время работы.

Вернемся к нашей задаче. Зная правило или формулу


нахождения производительности, можно определить сколько
булочек приходится на одну минуту.

Найдём производительность первого пекаря. Разделим


работу, которую он выполнил, на время которое он на нее
затратил. Выполненная работа это количество испеченных
им булочек, то есть 40, а время — 10 минут

40 : 10 = 4 булочки в минуту
1029

Аналогично найдём производительность второго пекаря.


Разделим 15 на 5

15 : 5 = 3 булочки в минуту

4>3

Первый пекарь в минуту выпекает больше булочек чем


второй, значит его производительность выше. Отсюда
делаем вывод, что работает он быстрее второго пекаря.

Также можно воспользоваться формулой нахождения


производительности. В этом случае решение принимает
следующий вид:

Под буквой v можно делать метки, указывающие для


кого/чего мы находим производительность.

Задача 2. Тому нужно за 2 дня прочитать книгу, в которой


100 страниц. В первый день он читал 4 часа со скоростью 12
страниц в час. С какой скоростью ему надо читать
оставшуюся часть книги, если у него есть  на это 4 часа?

Узнаем сколько страниц Том прочитал в первый день. Он


читал 12 страниц в час. Чтению в первый день он посвятил 4
часа, поэтому для нахождения количества прочитанных
страниц в первый день, нужно 12 умножить на 4
1030

12 × 4 = 48 страниц прочитано в первый день

Узнаем сколько страниц осталось прочесть. Вычтем из


общего количества страниц (100) количество прочитанных
страниц (48)

100 − 48 = 52 страницы осталось прочесть

Осталось прочесть 52 страницы. Теперь найдем такую


производительность, при которой Том сможет прочесть 52
страницы за 4 часа. Раскидаем 52 страницы на 4 часа
поровну

52 : 4 = 13 страниц в час

Ответ: чтобы прочитать оставшуюся часть книги за 4 часа,


Том должен читать ее со скоростью 13 страниц в час.

Замечание. В некоторых источниках слово


«производительность» может быть заменено на слова
«скорость», «эффективность», «продуктивность»,
«плодотворность».

Задача 3. Один насос работал 4 часа, выкачивая 158 вёдер


воды в час, а другой — 3 часа, выкачивая 169 вёдер воды в
час. Определить какой из насосов выкачал больше вёдер.

Решение

Определим сколько всего вёдер выкачал каждый насос по


отдельности. Для этого умножим их производительность на
время их работы:

158 в/ч × 4 = 632 вёдер выкачал первый насос


1031

169 в/ч × 3 = 507 вёдер выкачал второй насос

632 > 507

Ответ: первый насос выкачала больше вёдер, чем второй.

Задача 4. За 2 часа насос выкачал 80 литров воды.


Определить сколько литров он выкачает за 5 часов.

Решение

Сначала нужно определить сколько литров воды насос


выкачивает за час. Для этого 80 литров разделим на 2 часа
— получим 40 литров

80 : 2 = 40 литров в час

За один час насос выкачивает 40 литров воды. За 5 часов


выкачает в пять раз больше

40 × 5 = 200 литров

Ответ: за 5 часов насос выкачает 200 литров воды.

Если известны производительность и время работы, то


можно найти выполненную работу. Выполненная работа
равна производительности умноженной на время работы:

A = v × t

Например, если производительность пекаря составляет 50


булочек в час, и он проработал 4 часа, то можно найти всю
выполненную работу за эти четыре часа. Для этого
1032

производительность (50 бул/ч) нужно умножить на время его


работы (4ч)

50 × 4 = 200 булочек

Если известны работа и производительность, то можно найти


время работы. Время работы равно отношению выполненной
работы к производительности:

Например, если в неделю бригада отстраивает 2 этажа, то


можно узнать сколько недель потребуется для отстройки 8
этажей. Чтобы определить время отстройки восьми этажей,
нужно выполненную работу (8 этажей) разделить на
производительность (2 эт./нед):

8 : 2 = 4 нед.

Либо с помощью формулы, приведенной выше:

Если в неделю строится 2 этажа, то 8 этажей будет


отстроено за четыре недели. В данном случае вся работа
1033

была равна восьми. Производительность была равна двум,


поскольку по определению производительность есть работа,
выполненная за единицу времени – в нашем случае два
этажа за неделю.

Задача 6. Принтер работает с производительностью 70


стр./ч. Сколько страниц он напечатает за 5 часов?

Решение

Если в час принтер печатает 70 страниц, то за 5 часов он


напечатает в 5 раз больше:

70 × 5 = 350 страниц

Также, решение можно записать с помощью формулы


нахождения работы. В данном случае, количество
напечатанных страниц являются выполненной работой:

A = v × t = 70 × 5 = 350 страниц

A = 350 страниц

Задача 7. Принтер напечатал 350 страниц за 5 часов. С


какой производительностью он работал?
1034

Решение

Если в течении пяти часов принтер напечатал 350 страниц,

то в течении часа он печатал  . То есть работал с


производительностью 70 страниц в час:

350 : 5 = 70 стр./ч.

Либо с помощью формулы нахождения производительности:

Задача 8. Принтер работал с производительностью 70


страниц в час и напечатал 350 страниц. Определить время
работы принтера.

Решение

Выражение «работал с производительностью 70 страниц в


час» означает, что в каждом часе принтер печатал по 70
страниц. И это продолжалось до тех пор, пока он не
напечатал 350 страниц. Очевидно, что разделив 350 страниц
по 70, мы определим время работы принтера, то есть узнаем
сколько часов он работал

350 : 70 = 5 ч.

Либо с помощью формулы нахождения времени:


1035

Задача 9. Машинистка в первый день напечатала 48 страниц


рукописи, а во второй день — на 12 страниц больше, чем в
первый. На всю работу в эти 2 дня она затратила 9 часов.
Сколько часов работала она в каждый из этих дней, если
производительность её не менялась ?

Решение

Определим сколько страниц напечатала машинистка во


второй день. В условии сказано, что напечатала она на 12
страниц больше, чем в первый:

48 + 12 = 60 страниц во второй день.

Определим сколько страниц машинистка напечатала за два


дня:

48 + 60 = 108 страниц за два дня.

На эту работу машинистка затратила 9 часов. Также сказано,


что производительность её не менялась. Если мы разделим
выполненную работу (108) на время выполнения (9), то
определим производительность машинистки:

108 : 9 = 12 страниц в час.

Теперь мы можем определить сколько часов работала


машинистка в каждый из двух дней. Для этого поочередно
разделим выполненные работы в каждом из двух дней на
производительность:

48 : 12 = 4 часа работала машинистка в первый день

60 : 12 = 5 часов работала машинистка во второй день.


1036

Задача 10. Джон решил 10 примеров за 5 минут. С какой


производительностью он решал эти примеры?

10 примеров это выполненная Джоном работа. 5 минут —


время работы. Разделим выполненную работу на время
работы и определим производительность Джона:

10 : 5 = 2 примера в минуту.

Производительность Джона равна двум примерам в минуту.

Задача 11. Джон решил несколько примеров за 5 минут. С


какой производительностью он решил эти примеры?

Это та же самая задача, что и предыдущая, но в ней работа


не выражена каким-либо числом. Сказано лишь то, что Джон
выполнил эту работу за 5 минут. Поэтому, конкретную
производительность в такой задаче узнать нельзя. Но можно
воспользоваться дробями. Обозначим выполненную работу
через единицу. Тогда производительность работы Джона
будет выражаться дробью – частью примеров, решенных за
единицу времени. Если вы изучили задачи на дроби, то
должны понимать о чем идёт речь.

Итак, обозначим выполненную работу через единицу:

A=1
1037

Мы знаем, что для нахождения производительности,


выполненную работу нужно разделить на время. Время
работы у нас равно пяти минутам. Поэтому, единицу делим
на пять минут:

Дробь   выражает  часть работы, выполненную Джоном за


единицу времени. Если мы вернемся к предыдущей задаче,
где выполненная работа была равна десяти примерам и
найдем одну пятую от этой работы, то получим 2

Выражать выполненную работу через единицу часто


приходится при решении задач на совместную работу.

Задачи на совместную работу

Задача 1. Первый мастер за 2 часа изготавливает 64 детали,


а второй за 3 часа – 72 детали. За сколько часов они
изготовят 336 деталей?

В данной задаче речь идет о совместной работе.


Необходимо определить производительность обоих
1038

мастеров и найти время за которое они изготовят 336


деталей.

Для начала определим производительность первого


мастера:

64 : 2 = 32 дет./час

Определим производительность второго мастера:

72 : 3 = 24 дет./час

Определим совместную производительность мастеров. Для


этого сложим количество деталей, которые они
изготавливают по отдельности за единицу времени. То есть
сложим их производительности:

32 дет./час  + 24 дет./час = 56 дет./час

Вместе за один час мастера изготавливают 56 деталей.


Чтобы узнать за сколько часов они изготовят 336 деталей,
нужно определить сколько раз 336 содержит по 56

336 : 56 = 6 часов
1039

Задача 2. Первый мастер может покрасить забор за 20


минут, а второй мастер – за 30 минут. За сколько минут,
работая вместе, они могут покрасить забор?

Решение

В данной задаче, в отличие от предыдущей, работа не


выражена каким-либо числом. Сказано лишь то, что эту
работу первый мастер может выполнить за 20 минут, а
второй за 30 минут.

В такой ситуации можно воспользоваться дробями. Мы


можем обозначить всю работу (покраску забора) через
единицу.

Итак, обозначим работу (покраску забора) через единицу:

A=1

Производительность первого мастера будет выражáться

дробью . То есть за одну минуту он покрасит одну


двадцатую часть забора. Единица это вся работа, а двадцать
минут это время работы. Запишем производительность
первого мастера с помощью формулы нахождения
производительности:
1040

А производительность второго мастера будет выражáться

дробью . То есть за одну минуту он покрасит одну


тридцатую часть забора:

Определим общую производительность мастеров. Для этого


сложим дроби, выражающие производительность первого и
второго мастеров:

это дробь, выражающая общую производительность обоих

мастеров. То есть за одну минуту мастера вместе покрасят   


часть забора.

Определим время за которое мастера покрасят забор


вместе. Для этого воспользуемся формулой нахождения
времени: разделим выполненную работу на общую
производительность мастеров. Выполненная работа у нас

выражена единицей, а производительность — дробью 


1041

Ответ: работая вместе, мастера покрасят забор за 12 минут.

Задача 3. Первый рабочий может выполнить заказ за 8


часов, а второй за 6 часов. Два часа они работали вместе, а
заканчивал работу один второй рабочий. Сколько времени
потребовалось для выполнения этого заказа?

Решение

Обозначим всю работу через единицу

A=1

Тогда первый рабочий за один час может выполнить   часть

работы, а второй рабочий   часть работы. А вместе за один

час они могут выполнить   часть работы

Рабочие работали вместе два часа, поэтому умножим часть


работы, выполняемую ими за один час на 2:
1042

Остальную часть работы, а именно   работы заканчивал


один второй рабочий:

Второй рабочий за один час мог выполнить   часть работы.


Чтобы определить время за которое он завершил

оставшуюся   часть работы, воспользуемся формулой


нахождения времени.

Переменная A теперь равна  , переменная v — 

Теперь определим общее время заказа. Первые два часа


рабочие работали вместе, остальную часть работы второй
рабочий выполнил за два с половиной часа, отсюда имеем
4,5 ч.
1043

2 + 2,5 = 4,5 ч.

Ответ: для выполнения заказа потребовалось 4,5 ч.

Задача 4. Одна труба наполняет бассейн за 6 ч, а другая –


за 4 ч. За
сколько часов наполняют бассейн обе трубы, работая
вместе?

Решение

Обозначим работу (наполнение бассейна) через единицу

A=1

Тогда первая труба за один час выполнит   часть работы, а

вторая труба —   часть работы. Работая вместе за один час

они выполнят   часть работы:

Определим время за которое обе трубы наполняют бассейн,


работая вместе:

2,4 это два целых часа и четыре десятых часа

2,4 = 2 ч + 0,4 ч
1044

А четыре десятых часа это 24 минуты

60 мин. × 0,4 = 24 мин.

Ответ: работая вместе обе трубы наполнят бассейн за 2 ч 24


мин.

Задачи для самостоятельного решения


Задача 1. Первая бригада может выполнить некоторое
задание за 12 часов, вторая – за 4 часа. За сколько часов они
выполнят задание, если будут работать вместе?
Показать решение
Задача 2. Лошадь съедает копну сена за 1 сутки, корова
может съесть такую же копну за 3 суток, а овца за 6 суток. За
какое время съедят эту копну лошадь, корова и овца вместе.
Показать решение
Задача 3. Сосуд наполняется шлангом за 12 мин, а полный
сосуд опорожняется при открытии крана за 20 мин. За какое
время наполнится пустой сосуд, если одновременно открыть
кран и вливать в него воду через шланг?
Показать решение
Задача 4. Через первую трубу бассейн можно заполнить за
20 ч, через вторую за 30 ч. Какая часть бассейна заполнится
через обе трубы за 1 ч?
Показать решение
Задача 5. На прокладку траншеи требуется затратить 10 ч.
Экскаватор проработал 8 ч, после чего ему осталось пройти
50 м. Найти общую длину траншеи.
1045

Показать решение
Задача 6. Ванна заполняется холодной водой за 6 мин 40 с,
горячей – за 8 мин. Кроме того, если из полной ванны вынуть
пробку, вода вытечет за 13 мин 20 с. Сколько времени
понадобится, чтобы наполнить ванну полностью, при
условии, что открыты оба крана, но ванна не заткнута
пробкой?.
Показать решение
1046

Элементы статистики

Продолжаем изучать элементарные задачи по математике.


Сегодня мы поговорим о статистике.

Статистика — это раздел математики в котором изучаются


вопросы сбора, измерения и анализа информации,
представленной в числовой форме. Происходит слово
статистика от латинского слова status (состояние или
положение дел).

Так, с помощью статистики мы можем узнать свое положение


дел, касающихся финансов. С начала месяца можно вести
дневник расходов и по окончании месяца, воспользовавшись
статистикой, узнать сколько денег в среднем мы тратили
каждый день или какая потраченная сумма была наибольшей
в этом месяце либо узнать какую сумму мы тратили
наиболее часто.

На основе этой информации можно провести анализ и


сделать определенные выводы: следует ли в следующем
месяце немного сбавить аппетит, чтобы тратить меньше
денег, либо наоборот позволить себе не только хлеб с водой,
но и колбасу.

Содержание урока

 Выборка. Объем. Размах


 Среднее арифметическое
 Средняя скорость движения
 Мода и медиана
 Частота
1047

 Относительная частота

Выборка. Объем. Размах

Что такое выборка? Если говорить простым языком, то это


отобранная нами информация для исследования. Например,
мы можем сформировать следующую выборку — суммы
денег, потраченных в каждый из шести дней. Давайте
нарисуем таблицу в которую занесем расходы за шесть дней

Выборка состоит из n-элементов. Вместо переменной n


может стоять любое число. У нас имеется шесть элементов,
поэтому переменная n равна 6

n=6

Элементы выборки обозначаются с помощью переменных с


индексами  . Последний   элемент является шестым
элементом выборки, поэтому вместо n будет стоять число 6.

Обозначим элементы нашей выборки через


переменные 
1048

Количество элементов выборки называют объемом


выборки. В нашем случае объем равен шести.

Размахом выборки называют разницу между самым


большим и маленьким элементом выборки.

В нашем случае, самым большим элементом выборки


является элемент 250, а самым маленьким — элемент 150.
Разница между ними равна 100

Среднее арифметическое

Понятие среднего значения часто используется в


повседневной жизни.

Примеры:

 средняя зарплата жителей страны;


 средний балл учащихся;
 средняя скорость движения;
 средняя производительность труда.
1049

Речь идет о среднем арифметическом — результате деления


суммы элементов выборки на их количество.

Среднее арифметическое — это результат деления


суммы элементов выборки на их количество.

Вернемся к нашему примеру

Узнаем сколько в среднем мы тратили в каждом из шести


дней:

Средняя скорость движения

При изучении задач на движение мы определяли скорость


движения следующим образом: делили пройденное
расстояние на время. Но тогда подразумевалось, что тело
движется с постоянной скоростью, которая не менялась на
протяжении всего пути.

В реальности, это происходит довольно редко или не


происходит совсем. Тело, как правило, движется с различной
скоростью.

Когда мы ездим на автомобиле или велосипеде, наша


скорость часто меняется. Когда впереди нас помехи, нам
1050

приходиться сбавлять скорость. Когда же трасса свободна,


мы ускоряемся. При этом за время нашего ускорения
скорость изменяется несколько раз.

Речь идет о средней скорости движения. Чтобы её


определить нужно сложить скорости движения, которые были
в каждом часе/минуте/секунде и результат разделить на
время движения.

Задача 1. Автомобиль первые 3 часа двигался со скоростью


66,2 км/ч, а следующие 2 часа — со скоростью 78,4 км/ч. С
какой средней скоростью он ехал?

Сложим скорости, которые были у автомобиля в каждом часе


и разделим на время движения (5ч)

Значит автомобиль ехал со средней скоростью 71,08 км/ч.

Определять среднюю скорость можно и по другому —


сначала найти расстояния, пройденные с одной скоростью,
затем сложить эти расстояния и результат разделить на
время. На рисунке видно, что первые три часа скорость у
автомобиля не менялась. Тогда можно найти расстояние,
пройденное за три часа:

66,2 × 3 = 198,6 км.


1051

Аналогично можно определить расстояние, которое было


пройдено со скоростью 78,4 км/ч. В задаче сказано, что с
такой скоростью автомобиль двигался 2 часа:

78,4 × 2 = 156,8 км.

Сложим эти расстояния и результат разделим на 5

Задача 2. Велосипедист за первый час проехал 12,6 км, а в


следующие 2 часа он ехал со скоростью 13,5 км/ч.
Определить среднюю скорость велосипедиста.

Скорость велосипедиста в первый час составляла 12,6 км/ч.


Во второй и третий час он ехал со скоростью 13,5.
Определим среднюю скорость движения велосипедиста:

Мода и медиана

Модой называют элемент, который встречается в выборке


чаще других.

Рассмотрим следующую выборку: шестеро спортсменов, а


также время в секундах за которое они пробегают 100 метров
1052

Элемент 14 встречается в выборке чаще других, поэтому


элемент 14 назовем модой.

Рассмотрим еще одну выборку. Тех же спортсменов, а также


смартфоны, которые им принадлежат

Элемент iphone встречается в выборке чаще других, значит


элемент iphone является модой. Говоря простым языком,
носить iphone модно.

Конечно элементы выборки в этот раз выражены не числами,


а другими объектами (смартфонами), но для общего
представления о моде этот пример вполне приемлем.

Рассмотрим следующую выборку: семеро спортсменов, а


также их рост в сантиметрах:

Упорядочим данные в таблице так, чтобы рост спортсменов


шел по возрастанию. Другими словами, построим
спортсменов по росту:
1053

Выпишем рост спортсменов отдельно:

180, 182, 183, 184, 185, 188, 190

В получившейся выборке 7 элементов. Посередине этой


выборки располагается элемент 184. Слева и справа от него
по три элемента. Такой элемент как 184 называют медианой
упорядоченной выборки.

Медианой упорядоченной выборки называют элемент,


располагающийся посередине.

Отметим, что данное определение справедливо в случае,


если количество элементов упорядоченной выборки
является нечётным.

В рассмотренном выше примере, количество элементов


упорядоченной выборки было нечётным. Это позволило нам
быстро указать медиану

Но возможны случаи, когда количество элементов выборки


чётно.

К примеру, рассмотрим выборку в которой не семеро


спортсменов, а шестеро:
1054

Построим этих шестерых спортсменов по росту:

Выпишем рост спортсменов отдельно:

180, 182, 184, 186, 188, 190

В данной выборке не получается указать элемент, который


находился бы посередине. Если указать элемент 184 как
медиану, то слева от этого элемента будут располагаться
два элемента, а справа — три. Если как медиану указать
элемент 186, то слева от этого элемента будут
располагаться три элемента, а справа — два.

В таких случаях для определения медианы выборки, нужно


взять два элемента выборки, находящихся посередине и
найти их среднее арифметическое. Полученный результат
будет являться медианой.

Вернемся к нашим спортсменам. В упорядоченной


выборке 180, 182, 184, 186, 188, 190 посередине
располагаются элементы 184 и 186

Найдем среднее арифметическое элементов 184 и 186


1055

Элемент 185 является медианой выборки, несмотря на то,


что этот элемент не является членом исходной и
упорядоченной выборки. Спортсмена с ростом 185 нет среди
остальных спортсменов. Рост в 185 см используется в
данном случае для статистики, чтобы можно было сказать о
том, что срединный рост спортсменов составляет 185 см.

Поэтому более точное определение медианы зависит от


количества элементов в выборке.

Если количество элементов упорядоченной выборки


нечётно, то медианой выборки называют элемент,
располагающийся посередине.

Если количество элементов упорядоченной выборки


чётно, то медианой выборки называют среднее
арифметическое двух чисел, располагающихся
посередине этой выборки.

Медиана и среднее арифметическое по сути являются


«близкими родственниками», поскольку и то и другое
используют для определения среднего значения. Например,
для предыдущей упорядоченной выборки 180, 182, 184, 186,
188, 190 мы определили медиану, равную 185. Этот же
результат можно получить путем определения среднего
арифметического элементов 180, 182, 184, 186, 188, 190

Но медиана в некоторых случаях отражает более реальную


ситуацию. Например, рассмотрим следующий пример:
1056

Было подсчитано количество имеющихся очков у каждого


спортсмена. В результате получилась следующая выборка:

0, 1, 1, 1, 2, 1, 2, 3, 5, 4, 5, 0, 1, 6, 1

Определим среднее арифметическое для данной выборки —


получим значение 2,2

По данному значению можно сказать, что в среднем у


спортсменов 2,2 очка

Теперь определим медиану для этой же выборки.


Упорядочим элементы выборки и укажем элемент,
находящийся посередине:

0, 0, 1, 1, 1, 1, 1, 1, 2, 2, 3, 4, 5, 5, 6

В данном примере медиана лучше отражает реальную


ситуацию, поскольку половина спортсменов имеет не более
одного очка.

Частота

Частота это число, которое показывает сколько раз в


выборке встречается тот или иной элемент.

Предположим, что в школе проходят соревнования по


подтягиваниям. В соревнованиях участвует 36 школьников.
Составим таблицу в которую будем заносить число
подтягиваний, а также число участников, которые выполнили
столько подтягиваний.
1057

По таблице можно узнать сколько человек выполнило 5, 10


или 15 подтягиваний. Так, 5 подтягиваний выполнили четыре
человека, 10 подтягиваний выполнили восемь человек, 15
подтягиваний выполнили три человека.

Количество человек, повторяющих одно и то же число


подтягиваний в данном случае являются частотой. Поэтому
вторую строку таблицы переименуем в название «частота»:

Такие таблицы называют таблицами частот.

Частота обладает следующим свойством: сумма частот


равна общему числу данных в выборке.

Это означает, что сумма частот равна общему числу


школьников, участвующих в соревнованиях, то есть тридцати
шести. Проверим так ли это. Сложим частоты, приведенные
в таблице:

4 + 5 + 10 + 8 + 6 + 3 = 36
1058

Относительная частота

Относительная частота это в принципе та же самая частота,


которая была рассмотрена ранее, но только выраженная в
процентах.

Относительная частота равна отношению частоты на общее


число элементов выборки.

Вернемся к нашей таблице:

Пять подтягиваний выполнили 4 человека из 36. Шесть


подтягиваний выполнили 5 человек из 36. Восемь
подтягиваний выполнили 10 человек из 36 и так далее.
Давайте заполним таблицу с помощью таких отношений:

Выполним деление в этих дробях:

Выразим эти частоты в процентах. Для этого умножим их на


100. Умножение на 100 удобно выполнить передвижением
запятой на две цифры вправо:
1059

Теперь можно сказать, что пять подтягиваний выполнили


11% участников, 6 подтягиваний выполнили 14% участников,
8 подтягиваний выполнили 28% участников и так далее.
1060

Общие сведения об уравнениях

Уравнения — одна из сложных тем для усвоения, но при


этом они являются достаточно мощным инструментом для
решения большинства задач.

С помощью уравнений описываются различные процессы,


протекающие в природе. Уравнения широко применяются в
других науках: в экономике, физике, биологии и химии.

В данном уроке мы попробуем понять суть простейших


уравнений, научимся выражать неизвестные и решим
несколько уравнений. По мере усвоения новых материалов,
уравнения будут усложняться, поэтому понять основы очень
важно.

Предварительные навыки

 Основные операции
 Выражения
 Буквенные выражения
 Вынесение общего множителя за скобки
 Раскрытие скобок

Содержание урока

 Что такое уравнение?


 Выразить одно через другое
 Правила нахождения неизвестных
 Компоненты
 Равносильные уравнения
 Умножение на минус единицу
1061

 Приравнивание к нулю
 Альтернатива правилам нахождения неизвестных
 Когда корней несколько
 Когда корней бесконечно много
 Когда корней нет
 Буквенные уравнения
 Линейные уравнения с одним неизвестным
 Задания для самостоятельного решения

Что такое уравнение?

Уравнение — это равенство, содержащее в себе


переменную, значение которой требуется найти. Это
значение должно быть таким, чтобы при его подстановке в
исходное уравнение получалось верное числовое равенство.

Например выражение 3 + 2 = 5 является равенством. При


вычислении левой части получается верное числовое
равенство 5 = 5.

А вот равенство 3 + x = 5 является уравнением, поскольку


содержит в себе переменную x, значение которой можно
найти. Значение должно быть таким, чтобы при подстановке
этого значения в исходное уравнение, получилось верное
числовое равенство.

Другими словами, мы должны найти такое значение, при


котором знак равенства оправдал бы свое местоположение
— левая часть должна быть равна правой части.

Уравнение 3 + x = 5 является элементарным. Значение


переменной x равно числу 2. При любом другом значении
равенство соблюдáться не будет
1062

Говорят, что число 2 является корнем или решением


уравнения 3 + x = 5

Корень или решение уравнения — это значение


переменной, при котором уравнение обращается в верное
числовое равенство.

Корней может быть несколько или не быть совсем. Решить


уравнение означает найти его корни или доказать, что
корней нет.

Переменную, входящую в уравнение, иначе называют


неизвестным. Вы вправе называть как вам удобнее. Это
синонимы.

Примечание. Словосочетание «решить уравнение» говорит


самó за себя. Решить уравнение означает «уравнять»
равенство — сделать его сбалансированным, чтобы левая
часть равнялась правой части.

Выразить одно через другое

Изучение уравнений по традиции начинается с того, чтобы


научиться выражать одно число, входящее в равенство,
через ряд других. Давайте не будем нарушать эту традицию
и поступим также.

Рассмотрим следующее выражение:

8+2
1063

Данное выражение является суммой чисел 8 и 2. Значение


данного выражения равно 10

8 + 2 = 10

Получили равенство. Теперь можно выразить любое число


из этого равенства через другие числа, входящие в это же
равенство. К примеру, выразим число 2.

Чтобы выразить число 2, нужно задать вопрос: «что нужно


сделать с числами 10 и 8, чтобы получить число 2». Понятно,
что для получения числа 2, нужно из числа 10 вычесть число
8.

Так и делаем. Записываем число 2 и через знак равенства


говорим, что для получения этого числа 2 мы из числа 10
вычли число 8:

2 = 10 − 8

Мы выразили число 2 из равенства 8 + 2 = 10. Как видно из


примера, ничего сложного в этом нет.

При решении уравнений, в частности при выражении одного


числа через другие, знак равенства удобно заменять на
слово «есть». Делать это нужно мысленно, а не в самом
выражении.

Так, выражая число 2 из равенства 8 + 2 = 10 мы получили


равенство 2 = 10 − 8. Данное равенство можно прочесть так:

2 есть 10 − 8

То есть знак = заменен на слово «есть». Более того,


равенство 2 = 10 − 8 можно перевести с математического
1064

языка на полноценный человеческий язык. Тогда его можно


будет прочитать следующим образом:

Число 2 есть разность числа 10 и числа 8

или

Число 2 есть разница между числом 10 и числом 8.

Но мы ограничимся лишь заменой знака равенства на слово


«есть», и то будем делать это не всегда. Элементарные
выражения можно понимать и без перевода математического
языка на язык человеческий.

Вернём получившееся равенство 2 = 10 − 8 в


первоначальное состояние:

8 + 2 = 10

Выразим в этот раз число 8. Что нужно сделать с


остальными числами, чтобы получить число 8? Верно, нужно
из числа 10 вычесть число 2

8 = 10 − 2

Вернем получившееся равенство 8 = 10 − 2 в


первоначальное состояние:

8 + 2 = 10

В этот раз выразим число 10. Но оказывается, что десятку


выражать не нужно, поскольку она уже выражена.
Достаточно поменять местами левую и правую часть, тогда
получится то, что нам нужно:

10 = 8 + 2
1065

Пример 2. Рассмотрим равенство 8 − 2 = 6

Выразим из этого равенства число 8. Чтобы выразить число


8 остальные два числа нужно сложить:

8=6+2

Вернем получившееся равенство 8 = 6 + 2 в первоначальное


состояние:

8−2=6

Выразим из этого равенства число 2. Чтобы выразить число


2, нужно из 8 вычесть 6

2 = 8 − 6

Пример 3. Рассмотрим равенство 3 × 2 = 6

Выразим число 3. Чтобы выразить число 3, нужно 6


разделить 2

Вернем получившееся равенство  в первоначальное


состояние:

3×2=6

Выразим из этого равенства число 2. Чтобы выразить число


2, нужно 6 разделить 3
1066

Пример 4. Рассмотрим равенство 

Выразим из этого равенства число 15. Чтобы выразить число


15, нужно перемножить числа 3 и 5

15 = 3 × 5

Вернем получившееся равенство 15 = 3 × 5 в


первоначальное состояние:

Выразим из этого равенства число 5. Чтобы выразить число


5, нужно 15 разделить 3

Правила нахождения неизвестных

Рассмотрим несколько правил нахождения неизвестных.


Возможно, они вам знакомы, но не мешает повторить их ещё
раз. В дальнейшем их можно будет забыть, поскольку мы
научимся решать уравнения, не применяя эти правила.

Вернемся к первому примеру, который мы рассматривали в


предыдущей теме, где в равенстве 8 + 2 = 10 требовалось
выразить число 2.
1067

В равенстве 8 + 2 = 10 числа 8 и 2 являются слагаемыми, а


число 10 — суммой.

Чтобы выразить число 2, мы поступили следующим образом:

2 = 10 − 8

То есть из суммы 10 вычли слагаемое 8.

Теперь представим, что в равенстве 8 + 2 = 10 вместо числа


2 располагается переменная x

8 + x = 10

В этом случае равенство 8 + 2 = 10 превращается в


уравнение 8 + x = 10, а переменная x берет на себя роль так
называемого неизвестного слагаемого

Наша задача найти это неизвестное слагаемое, то есть


решить уравнение 8 + x = 10. Для нахождения неизвестного
слагаемого предусмотрено следующее правило:

Чтобы найти неизвестное слагаемое, нужно из суммы


вычесть известное слагаемое.

Что мы в принципе и сделали, когда выражали двойку в


равенстве 8 + 2 = 10. Чтобы выразить слагаемое 2, мы из
суммы 10 вычли другое слагаемое 8

2 = 10 − 8
1068

А сейчас, чтобы найти неизвестное слагаемое x, мы должны


из суммы 10 вычесть известное слагаемое 8:

x = 10 − 8

Если вычислить правую часть получившегося равенства, то


можно узнать чему равна переменная x

x=2

Мы решили уравнение. Значение переменной x равно 2. Для


проверки значение переменной x отправляют в исходное
уравнение 8 + x = 10 и подставляют вместо x. Так
желательно поступать с любым решённым уравнением,
поскольку нельзя быть точно уверенным, что уравнение
решено правильно:

В результате получается верное числовое равенство. Значит


уравнение решено правильно.

Это же правило действовало бы в случае, если неизвестным


слагаемым было бы первое число 8.

x + 2 = 10
1069

В этом уравнении x — это неизвестное слагаемое, 2 —


известное слагаемое, 10 — сумма. Чтобы найти неизвестное
слагаемое x, нужно из суммы 10 вычесть известное
слагаемое 2

x = 10 − 2

x=8

Вернемся ко второму примеру из предыдущей темы, где в


равенстве 8 − 2 = 6 требовалось выразить число 8.

В равенстве 8 − 2 = 6 число 8 это уменьшаемое, число 2 —


вычитаемое, число 6 — разность

Чтобы выразить число 8, мы поступили следующим образом:

8=6+2

То есть сложили разность 6 и вычитаемое 2.

Теперь представим, что в равенстве 8 − 2 = 6 вместо числа 8


располагается переменная x
1070

x − 2 = 6

В этом случае переменная x берет на себя роль так


называемого неизвестного уменьшаемого

Для нахождения неизвестного уменьшаемого предусмотрено


следующее правило:

Чтобы найти неизвестное уменьшаемое, нужно к


разности прибавить вычитаемое.

Что мы и сделали, когда выражали число 8 в равенстве


8 − 2 = 6. Чтобы выразить уменьшаемое 8, мы к разности 6
прибавили вычитаемое 2.

А сейчас, чтобы найти неизвестное уменьшаемое x, мы


должны к разности 6 прибавить вычитаемое 2

x=6+2

Если вычислить правую часть, то можно узнать чему равна


переменная x

x=8

Теперь представим, что в равенстве 8 − 2 = 6 вместо числа 2


располагается переменная x

8−x=6
1071

В этом случае переменная x берет на себя роль


неизвестного вычитаемого

Для нахождения неизвестного вычитаемого предусмотрено


следующее правило:

Чтобы найти неизвестное вычитаемое, нужно из


уменьшаемого вычесть разность.

Что мы и сделали, когда выражали число 2 в


равенстве 8 − 2 = 6. Чтобы выразить число 2, мы из
уменьшаемого 8 вычли разность 6.

А сейчас, чтобы найти неизвестное вычитаемое x, нужно


опять же из уменьшаемого 8 вычесть разность 6

x = 8 − 6

Вычисляем правую часть и находим значение x

x=2

Вернемся к третьему примеру из предыдущей темы, где в


равенстве 3 × 2 = 6 мы пробовали выразить число 3.

В равенстве 3 × 2 = 6 число 3 — это множимое, число 2 —


множитель, число 6 — произведение

Чтобы выразить число 3 мы поступили следующим образом:


1072

То есть разделили произведение 6 на множитель 2.

Теперь представим, что в равенстве 3 × 2 = 6 вместо числа 3


располагается переменная x

x × 2 = 6

В этом случае переменная x берет на себя роль


неизвестного множимого.

Для нахождения неизвестного множимого предусмотрено


следующее правило:

Чтобы найти неизвестное множимое, нужно


произведение разделить на множитель.

Что мы и сделали, когда выражали число 3 из равенства


3 × 2 = 6. Произведение 6 мы разделили на множитель 2.

А сейчас для нахождения неизвестного множимого x, нужно


произведение 6 разделить на множитель 2.

Вычисление правой части позволяет нам найти значение


переменной x

x=3
1073

Это же правило применимо в случае, если переменная x


располагается вместо множителя, а не множимого.
Представим, что в равенстве 3 × 2 = 6 вместо числа
2 располагается переменная x.

В этом случае переменная x берет на себя роль


неизвестного множителя. Для нахождения неизвестного
множителя предусмотрено такое же, что и для нахождения
неизвестного множимого, а именно деление произведения на
известный множитель:

Чтобы найти неизвестный множитель, нужно


произведение разделить на множимое.

Что мы и сделали, когда выражали число 2 из равенства


3 × 2 = 6. Тогда для получения числа 2 мы разделили
произведение 6 на множимое 3.

А сейчас для нахождения неизвестного множителя x мы


разделили произведение 6 на множимое 3.

Вычисление правой части равенства   позволяет узнать


чему равно x

x=2

Множимое и множитель вместе называют сомножителями.


Поскольку правила нахождения множимого и множителя
1074

совпадают, мы можем сформулировать общее правило


нахождения неизвестного сомножителя:

Чтобы найти неизвестный сомножитель, нужно


произведение разделить на известный сомножитель.

Например, решим уравнение 9 × x = 18. Переменная x


является неизвестным сомножителем. Чтобы найти этот
неизвестный сомножитель, нужно произведение 18
разделить на известный сомножитель 9

Отсюда  .

Решим уравнение x × 3 = 27. Переменная x является


неизвестным сомножителем. Чтобы найти этот неизвестный
сомножитель, нужно произведение 27 разделить на
известный сомножитель 3

Отсюда  .

Вернемся к четвертому примеру из предыдущей темы, где в

равенстве   требовалось выразить число 15. В этом


равенстве число 15 — это делимое, число 5 — делитель,
число 3 — частное.
1075

Чтобы выразить число 15 мы поступили следующим


образом:

15 = 3 × 5

То есть умножили частное 3 на делитель 5.

Теперь представим, что в равенстве  вместо числа 15


располагается переменная x

В этом случае переменная x берет на себя роль


неизвестного делимого.
1076

Для нахождения неизвестного делимого предусмотрено


следующее правило:

Чтобы найти неизвестное делимое, нужно частное


умножить на делитель.

Что мы и сделали, когда выражали число 15 из

равенства  . Чтобы выразить число 15, мы умножили


частное 3 на делитель 5.

А сейчас, чтобы найти неизвестное делимое x, нужно


частное 3 умножить на делитель 5

x = 3 × 5

Вычислим правую часть получившегося равенства. Так мы


узнаем чему равна переменная x.

x = 15

Теперь представим, что в равенстве   вместо числа 5


располагается переменная x.

В этом случае переменная x берет на себя роль


неизвестного делителя.
1077

Для нахождения неизвестного делителя предусмотрено


следующее правило:

Чтобы найти неизвестный делитель, нужно делимое


разделить на частное.

Что мы и сделали, когда выражали число 5 из

равенства  . Чтобы выразить число 5, мы разделили


делимое 15 на частное 3.

А сейчас, чтобы найти неизвестный делитель x, нужно


делимое 15 разделить на частное 3

Вычислим правую часть получившегося равенства. Так мы


узнаем чему равна переменная x.

x=5

Итак, для нахождения неизвестных мы изучили следующие


правила:
1078

 Чтобы найти неизвестное слагаемое, нужно из


суммы вычесть известное слагаемое;
 Чтобы найти неизвестное уменьшаемое, нужно к
разности прибавить вычитаемое;
 Чтобы найти неизвестное вычитаемое, нужно из
уменьшаемого вычесть разность;
 Чтобы найти неизвестное множимое, нужно
произведение разделить на множитель;
 Чтобы найти неизвестный множитель, нужно
произведение разделить на множимое;
 Чтобы найти неизвестное делимое, нужно частное
умножить на делитель;
 Чтобы найти неизвестный делитель, нужно делимое
разделить на частное.

Компоненты

Компонентами мы будем называть числа и переменные,


входящие в равенство

Так, компонентами сложения являются слагаемые и сумма

Компонентами вычитания являются уменьшаемое,


вычитаемое и разность
1079

Компонентами умножения являются множимое, множитель


и произведение

Компонентами деления являются делимое, делитель и


частное

В зависимости от того, с какими компонентами мы будем


иметь дело, будут применяться соответствующие правила
нахождения неизвестных. Эти правила мы изучили в
предыдущей теме. При решении уравнений желательно
знать эти правило наизусть.

Пример 1. Найти корень уравнения 45 + x = 60

45 — слагаемое, x — неизвестное слагаемое, 60 — сумма.


Имеем дело с компонентами сложения. Вспоминаем, что для
нахождения неизвестного слагаемого, нужно из суммы
вычесть известное слагаемое:

x = 60 − 45

Вычислим правую часть, получим значение x равное 15

x = 15

Значит корень уравнения 45 + x = 60 равен 15.


1080

Чаще всего неизвестное слагаемое необходимо привести к


виду при котором его можно было бы выразить.

Пример 2. Решить уравнение 

Здесь в отличие от предыдущего примера, неизвестное


слагаемое нельзя выразить сразу, поскольку оно содержит
коэффициент 2. Наша задача привести это уравнение к виду
при котором можно было бы выразить x

В данном примере мы имеем дело с компонентами сложения


— слагаемыми и суммой. 2x — это первое слагаемое, 4 —
второе слагаемое, 8 — сумма.

При этом слагаемое 2x содержит переменную x. После


нахождения значения переменной x слагаемое 2x примет
другой вид. Поэтому слагаемое 2x можно полностью принять
за неизвестное слагаемое:

Теперь применяем правило нахождения неизвестного


слагаемого. Вычитаем из суммы известное слагаемое:

Вычислим правую часть получившегося уравнения:

Мы получили новое уравнение . Теперь мы имеем дело


с компонентами умножения: множимым, множителем и
1081

произведением. 2 — множимое, x — множитель,


4 — произведение

При этом переменная x является не просто множителем, а


неизвестным множителем

Чтобы найти этот неизвестный множитель, нужно


произведение разделить на множимое:

Вычислим правую часть, получим значение переменной x

Для проверки найденный корень отправим в исходное


уравнение  и подставим вместо x

Получили верное числовое равенство. Значит уравнение


решено правильно.
1082

Пример 3. Решить уравнение 3x + 9x + 16x = 56

Cразу выразить неизвестное x нельзя. Сначала нужно


привести данное уравнение к виду при котором его можно
было бы выразить.

Приведем подобные слагаемые в левой части данного


уравнения:

Имеем дело с компонентами умножения. 28 — множимое,


x — множитель, 56 — произведение. При этом x является
неизвестным множителем. Чтобы найти неизвестный
множитель, нужно произведение разделить на множимое:

Отсюда x равен 2

Равносильные уравнения

В предыдущем примере при решении уравнения


3x + 9x + 16x = 56, мы привели подобные слагаемые в левой
части уравнения. В результате получили новое уравнение
28x = 56. Старое уравнение 3x + 9x + 16x = 56 и
получившееся новое уравнение 28x = 56 называют
равносильными уравнениями, поскольку их корни
совпадают.
1083

Уравнения называют равносильными, если их корни


совпадают.

Проверим это. Для уравнения 3x + 9x + 16x = 56 мы нашли


корень равный 2. Подставим этот корень сначала в
уравнение 3x + 9x + 16x = 56, а затем в уравнение 28x = 56,
которое получилось в результате приведения подобных
слагаемых в левой части предыдущего уравнения. Мы
должны получить верные числовые равенства

Согласно порядку действий, в первую очередь выполняется


умножение:

Подставим корень 2 во второе уравнение 28x = 56

Видим, что у обоих уравнений корни совпадают. Значит


уравнения 3x + 9x + 16x = 56 и 28x = 56 действительно
являются равносильными.

Для решения уравнения 3x + 9x + 16x = 56 мы


воспользовались одним из тождественных преобразований
— приведением подобных слагаемых. Правильное
тождественное преобразование уравнения позволило нам
1084

получить равносильное уравнение 28x = 56, которое проще


решать.

Из тождественных преобразований на данный момент мы


умеем только сокращать дроби, приводить подобные
слагаемые, выносить общий множитель за скобки, а также
раскрывать скобки. Существуют и другие преобразования,
которые следует знать. Но для общего представления о
тождественных преобразованиях уравнений, изученных нами
тем вполне хватает.

Рассмотрим некоторые преобразования, которые позволяют


получить равносильное уравнение

Если к обеим частям уравнения прибавить одно и то же


число, то получится уравнение равносильное данному.

и аналогично:

Если из обеих частей уравнения вычесть одно и то же


число, то получится уравнение равносильное данному.

Другими словами, корень уравнения не изменится, если к


обеим частям данного уравнения прибавить (или вычесть из
обеих частей) одно и то же число.

Пример 1. Решить уравнение

Вычтем из обеих частей уравнения число 10

Приведем подобные слагаемые в обеих частях:


1085

Получили уравнение 5x = 10. Имеем дело с компонентами


умножения. Чтобы найти неизвестный сомножитель x, нужно
произведение 10 разделить на известный сомножитель 5.

Отсюда  .

Вернемся к исходному уравнению  и подставим


вместо x найденное значение 2

Получили верное числовое равенство. Значит уравнение


решено правильно.

Решая уравнение мы вычли из обеих частей


уравнения число 10. В результате получили равносильное
уравнение . Корень этого уравнения, как и уравнения
 так же равен 2

Пример 2. Решить уравнение 4(x + 3) = 16

Раскроем скобки в левой части равенства:


1086

Вычтем из обеих частей уравнения число 12

Приведем подобные слагаемые в обеих частях уравнения:

В левой части останется 4x, а в правой


части число 4

Получили уравнение 4x = 4. Имеем дело с компонентами


умножения. Чтобы найти неизвестный сомножитель x, нужно
произведение 4 разделить на известный сомножитель 4

Отсюда 

Вернемся к исходному уравнению 4(x + 3) = 16 и подставим


вместо x найденное значение 1

Получили верное числовое равенство. Значит уравнение


решено правильно.
1087

Решая уравнение 4(x + 3) = 16 мы вычли из обеих частей


уравнения число 12. В результате получили равносильное
уравнение 4x = 4. Корень этого уравнения, как и уравнения
4(x + 3) = 16 так же равен 1

Пример 3. Решить уравнение

Раскроем скобки в левой части равенства:

Прибавим к обеим частям уравнения число 8

Приведем подобные слагаемые в обеих частях уравнения:

В левой части останется 2x, а в правой части число 9

В получившемся уравнении 2x = 9 выразим неизвестное


слагаемое x

Отсюда 

Вернемся к исходному уравнению   и подставим


вместо x найденное значение 4,5
1088

Получили верное числовое равенство. Значит уравнение


решено правильно.

Решая уравнение  мы прибавили к обеим частям


уравнения число 8. В результате получили равносильное
уравнение . Корень этого уравнения, как и уравнения
 так же равен 4,5

Следующее правило, которое позволяет получить


равносильное уравнение, выглядит следующим образом

Если в уравнении перенести слагаемое из одной части в


другую, изменив его знак, то получится уравнение
равносильное данному.

То есть корень уравнения не изменится, если мы перенесем


слагаемое из одной части уравнения в другую, изменив его
знак. Это свойство является одним из важных и одним из
часто используемых при решении уравнений.

Рассмотрим следующее уравнение:


1089

Корень данного уравнения равен 2. Подставим вместо x этот


корень и проверим получается ли верное числовое
равенство

Получается верное равенство. Значит число 2 действительно


является корнем уравнения .

Теперь попробуем поэкспериментировать со слагаемыми


этого уравнения, перенося их из одной части в другую,
изменяя знаки.

Например, слагаемое 3x располагается в левой части


равенства. Перенесём его в правую часть, изменив знак на
противоположный:

Получилось уравнение 12 = 9x − 3x. Приведем подобные


слагаемые в правой части данного уравнения:
1090

Имеем дело с компонентами умножения. Переменная x


является неизвестным сомножителем. Найдём этот
известный сомножитель:

Отсюда x = 2. Как видим, корень уравнения не изменился.


Значит уравнения 12 + 3x = 9x и 12 = 9x − 3x являются
равносильными.

На самом деле данное преобразование является


упрощенным методом предыдущего преобразования, где к
обеим частям уравнения прибавлялось (или вычиталось)
одно и то же число.

Мы сказали, что в уравнении 12 + 3x = 9x слагаемое 3x было


перенесено в правую часть, изменив знак. В реальности же
происходило следующее: из обеих частей уравнения вычли
слагаемое 3x

Затем в левой части были приведены подобные слагаемые и


получено уравнение 12 = 9x − 3x. Затем опять были
приведены подобные слагаемые, но уже в правой части, и
получено уравнение 12 = 6x.

Но так называемый «перенос» более удобен для подобных


уравнений, поэтому он и получил такое широкое
1091

распространение. Решая уравнения, мы часто будем


пользоваться именно этим преобразованием.

Равносильными также являются уравнения 12 + 3x = 9x и


3x − 9x = −12. В этот раз в уравнении 12 + 3x = 9x слагаемое
12 было перенесено в правую часть, а слагаемое 9x в левую.
Не следует забывать, что знаки этих слагаемых были
изменены во время переноса

Следующее правило, которое позволяет получить


равносильное уравнение, выглядит следующим образом:

Если обе части уравнения умножить или разделить на


одно и то же число, не равное нулю, то получится
уравнение равносильное данному.

Другими словами, корни уравнения не изменятся, если обе


его части умножить или разделить на одно и то же число. Это
действие часто применяется тогда, когда нужно решить
уравнение содержащее дробные выражения.

Сначала рассмотрим примеры, в которых обе части


уравнения будут умножаться на одно и то же число.

Пример 1. Решить уравнение 


1092

При решении уравнений, содержащих дробные выражения,


сначала  принято упростить это уравнение.

В данном случае мы имеем дело именно с таким


уравнением. В целях упрощения данного уравнения обе его
части можно умножить на 8:

Мы помним, что для умножения дроби на число, нужно


числитель данной дроби умножить на это число. У нас
имеются две дроби и каждая из них умножается на число 8.
Наша задача умножить числители дробей на это число 8

Теперь происходит самое интересное. В числителях и


знаменателях обеих дробей содержится множитель 8,
который можно сократить на 8. Это позволит нам избавиться
от дробного выражения:

В результате останется простейшее уравнение

Ну и нетрудно догадаться, что корень этого уравнения равен


4
1093

Вернемся к исходному уравнению    и подставим


вместо x найденное значение 4

Получается верное числовое равенство. Значит уравнение


решено правильно.

При решении данного уравнения мы умножили обе его части


на 8. В результате получили уравнение . Корень

этого уравнения, как и уравнения   равен 4. Значит


эти уравнения равносильны.

Множитель на который умножаются обе части уравнения


принято записывать перед частью уравнения, а не после неё.

Так, решая уравнение  , мы умножили обе части на


множитель 8 и получили следующую запись:

От этого корень уравнения не изменился, но если бы мы


сделали это находясь в школе, то нам сделали
бы замечание, поскольку в алгебре множитель принято
записывать перед тем выражением, с которым он
перемножается. Поэтому умножение обеих частей
1094

уравнения   на множитель 8 желательно переписать


следующим образом:

Пример 2. Решить уравнение 

Умнóжим обе части уравнения на 15

В левой части множители 15 можно сократить на 15, а в


правой части множители 15 и 5 можно сократить на 5

Перепишем то, что у нас осталось:

Раскроем скобки в правой части уравнения:

Перенесем слагаемое x из левой части уравнения в правую


часть, изменив знак. А слагаемое 15 из правой части
уравнения перенесем в левую часть, опять же изменив знак:

Приведем подобные слагаемые в обеих частях, получим


1095

Имеем дело с компонентами умножения. Переменная x


является неизвестным сомножителем. Найдём этот
известный сомножитель:

Отсюда 

Вернемся к исходному уравнению  и


подставим вместо x найденное значение 5

Получается верное числовое равенство. Значит уравнение


решено правильно. При решении данного уравнения мы
умножили обе го части на 15. Далее выполняя
тождественные преобразования, мы получили уравнение

10 = 2x. Корень этого уравнения, как и уравнения  


равен 5. Значит эти уравнения равносильны.

Пример 3. Решить уравнение 

Умнóжим обе части уравнения на 3


1096

В левой части можно сократить две тройки, а правая часть


будет равна 18

Останется простейшее уравнение  . Имеем дело с


компонентами умножения. Переменная x является
неизвестным сомножителем. Найдём этот известный
сомножитель:

Отсюда 

Вернемся к исходному уравнению   и подставим вместо


x найденное значение 9

Получается верное числовое равенство. Значит уравнение


решено правильно.

Пример 4. Решить уравнение 

Умнóжим обе части уравнения на 6


1097

В левой части уравнения раскроем скобки. В правой части


множитель 6 можно поднять в числитель:

Сократим в обеих частях уравнениях то, что можно


сократить:

Перепишем то, что у нас осталось:

Раскроем скобки в обеих частях уравнения:

Воспользуемся переносом слагаемых. Слагаемые,


содержащие неизвестное x, сгруппируем в левой части
уравнения, а слагаемые свободные от неизвестных — в
правой:

Приведем подобные слагаемые в обеих частях:

Теперь найдем значение переменной x. Для этого разделим


произведение 28 на известный сомножитель 7

Отсюда x = 4.
1098

Вернемся к исходному уравнению   и


подставим вместо x найденное значение 4

Получилось верное числовое равенство. Значит уравнение


решено правильно.

Пример 5. Решить уравнение 

Раскроем скобки в обеих частях уравнения там, где это


можно:

Умнóжим обе части уравнения на 15

Раскроем скобки в обеих частях уравнения:

Сократим в обеих частях уравнения, то что можно сократить:


1099

Перепишем то, что у нас осталось:

Раскроем скобки там, где это можно:

Воспользуемся переносом слагаемых. Слагаемые,


содержащие неизвестное, сгруппируем в левой части
уравнения, а слагаемые, свободные от неизвестных — в
правой. Не забываем, что во время переноса, слагаемые
меняют свои знаки на противоположные:

Приведем подобные слагаемые в обеих частях уравнения:

Найдём значение x

В получившемся ответе можно выделить целую часть:

Вернемся к исходному уравнению и подставим вместо x

найденное значение 
1100

Получается довольно громоздкое выражение. Воспользуемся


переменными. Левую часть равенства занесем в переменную
A, а правую часть равенства в переменную B

Наша задача состоит в том, чтобы убедиться равна ли левая


часть правой. Другими словами, доказать равенство A = B

Найдем значение выражения, находящегося в переменной А.


1101

Значение переменной А равно  . Теперь найдем значение


переменной B. То есть значение правой части нашего

равенства. Если и оно равно , то уравнение будет


решено верно
1102

Видим, что значение переменной B, как и значение

переменной A равно  . Это значит, что левая часть равна


правой части. Отсюда делаем вывод, что уравнение решено
правильно.

Теперь попробуем не умножать обе части уравнения на одно


и то же число, а делить.

Рассмотрим уравнение 30x + 14x + 14 = 70x − 40x + 42.


Решим его обычным методом: слагаемые, содержащие
неизвестные, сгруппируем в левой части уравнения, а
слагаемые, свободные от неизвестных — в правой. Далее
1103

выполняя известные тождественные преобразования,


найдем значение x

Подставим найденное значение 2 вместо x в исходное


уравнение:

Теперь попробуем разделить все слагаемые уравнения


30x + 14x + 14 = 70x − 40x + 42 на какое-нибудь
число. Замечаем, что все слагаемые этого уравнения имеют
общий множитель 2. На него и разделим каждое слагаемое:

Выполним сокращение в каждом слагаемом:

Перепишем то, что у нас осталось:

Решим это уравнение, пользуясь известными


тождественными преобразованиями:
1104

Получили корень 2. Значит уравнения


15x + 7x + 7 = 35x − 20x + 21
и 30x + 14x + 14 = 70x − 40x + 42 равносильны.

Деление обеих частей уравнения на одно и то же число


позволяет освобождать неизвестное от коэффициента. В
предыдущем примере когда мы получили уравнение 7x = 14,
нам потребовалось разделить произведение 14 на известный
сомножитель 7. Но если бы мы в левой части освободили
неизвестное от коэффициента 7, корень нашелся бы сразу.
Для этого достаточно было разделить обе части на 7

Этим методом мы тоже будем пользоваться часто.

Умножение на минус единицу

Если обе части уравнения умножить на минус единицу,


то получится уравнение равносильное данному.

Это правило следует из того, что от умножения (или


деления) обеих частей уравнения на одно и то же число,
1105

корень данного уравнения не меняется. А значит корень не


поменяется если обе его части умножить на −1.

Данное правило позволяет поменять знаки всех


компонентов, входящих в уравнение. Для чего это нужно?
Опять же, чтобы получить равносильное уравнение, которое
проще решать.

Рассмотрим уравнение . Чему равен корень этого


уравнения?

Прибавим к обеим частям уравнения число 5

Приведем подобные слагаемые:

А теперь вспомним про коэффициент буквенного выражения.


Что же представляет собой левая часть уравнения 
. Это есть произведение минус единицы и переменной x

То есть минус, стоящий перед переменной x, относится не к


самой переменной x, а к единице, которую мы не видим,
поскольку коэффициент 1 принято не записывать. Это
означает, что уравнение   на самом деле выглядит
следующим образом:

Имеем дело с компонентами умножения. Чтобы найти х,


нужно произведение −5 разделить на известный
сомножитель −1.
1106

или разделить обе части уравнения на −1, что еще проще

Итак, корень уравнения  равен 5. Для проверки


подставим его в исходное уравнение. Не забываем, что в
исходном уравнении минус стоящий перед переменной x
относится к невидимой единице

Получилось верное числовое равенство. Значит уравнение


решено верно.

Теперь попробуем умножить обе части уравнения   


на минус единицу:

После раскрытия скобок в левой части образуется


выражение  , а правая часть будет равна 10

Корень этого уравнения, как и уравнения   равен 5


1107

Значит уравнения   и   равносильны.

Пример 2. Решить уравнение 

В данном уравнении все компоненты являются


отрицательными. С положительными компонентами работать
удобнее, чем с отрицательными, поэтому поменяем знаки
всех компонентов, входящих в уравнение . Для
этого умнóжим обе части данного уравнения на −1.

Понятно, что от умножения на −1 любое число поменяет свой


знак на противоположный. Поэтому саму процедуру
умножения на −1 и раскрытие скобок подробно не
расписывают, а сразу записывают компоненты уравнения с
противоположными знаками.

Так, умножение уравнения  на −1 можно записать


подробно следующим образом:

либо можно просто поменять знаки всех компонентов:


1108

Получится то же самое, но разница будет в том, что мы


сэкономим себе время.

Итак, умножив обе части уравнения  на −1, мы


получили уравнение  . Решим данное уравнение. Из
обеих частей вычтем число 4 и разделим обе части на 3

Когда корень найден, переменную обычно записывают в


левой части, а её значение в правой, что мы и сделали.

Пример 3. Решить уравнение 

Умнóжим обе части уравнения на −1. Тогда все компоненты


поменяют свои знаки на противоположные:

Из обеих частей получившегося уравнения вычтем 2x и


приведем подобные слагаемые:
1109

Прибавим к обеим частям уравнения единицу и приведем

подобные слагаемые: 

Приравнивание к нулю

Недавно мы узнали, что если в уравнении перенести


слагаемое из одной части в другую, изменив его знак, то
получится уравнение равносильное данному.

А что будет если перенести из одной части в другую не одно


слагаемое, а все слагаемые? Верно, в той части откуда
забрали все слагаемые останется ноль. Иными словами, не
останется ничего.

В качестве примера рассмотрим уравнение .


Решим данное уравнение, как обычно — слагаемые,
содержащие неизвестные сгруппируем в одной части, а
числовые слагаемые, свободные от неизвестных оставим в
другой. Далее выполняя известные тождественные
преобразования, найдем значение переменной x
1110

Теперь попробуем решить это же уравнение, приравняв все


его компоненты к нулю. Для этого перенесем все слагаемые
из правой части в левую, изменив знаки:

Приведем подобные слагаемые в левой части:

Прибавим к обеим частям 77, и разделим обе части на 7

Альтернатива правилам нахождения неизвестных

Очевидно, что зная о тождественных преобразованиях


уравнений, можно не заучивать наизусть правила
нахождения неизвестных.

К примеру, для нахождения неизвестного в уравнении   


мы произведение 10 делили на известный сомножитель 2
1111

Но если в уравнении   обе части разделить на 2 корень


найдется сразу. В левой части уравнения в числителе
множитель 2 и в знаменателе множитель 2 сократятся на 2. А
правая часть будет  равна 5

Уравнения вида   мы решали выражая неизвестное


слагаемое:

Но можно воспользоваться тождественными


преобразованиями, которые мы сегодня изучили. В
уравнении слагаемое 4 можно перенести в правую
часть, изменив знак:

Далее разделить обе части на 2

В левой части уравнения сократятся две двойки. Правая


часть будет равна 2. Отсюда  .
1112

Либо можно было из обеих частей уравнения вычесть 4.


Тогда получилось бы следующее:

В случае с уравнениями вида   удобнее делить


произведение на известный сомножитель. Сравним оба
решения:

Первое решение намного короче и аккуратнее. Второе


решение можно значительно укоротить, если выполнить
деление в уме.

Тем не менее, необходимо знать оба метода, и только затем


использовать тот, который больше нравится.

Когда корней несколько

Уравнение может иметь несколько корней. Например


уравнение x(x + 9) = 0 имеет два корня: 0 и −9.
1113

В уравнении x(x + 9) = 0 нужно было найти такое значение


x при котором левая часть была бы равна нулю. В левой
части этого уравнения содержатся выражения x и (x + 9),
которые являются сомножителями. Из законов умножения
мы знаем, что произведение равно нулю, если хотя бы один
из сомножителей равен нулю (или первый сомножитель или
второй).

То есть в уравнении x(x + 9) = 0 равенство будет


достигаться, если x будет равен нулю или (x + 9) будет равно
нулю.

x = 0 или x + 9 = 0

Приравняв к нулю оба этих выражения, мы сможем найти


корни уравнения x(x + 9) = 0. Первый корень, как видно из
примера, нашелся сразу. Для нахождения второго корня
нужно решить элементарное уравнение x + 9 = 0. Несложно
догадаться, что корень этого уравнения равен −9. Проверка
показывает, что корень верный:

−9 + 9 = 0

Пример 2. Решить уравнение

Данное уравнение имеет два корня: 1 и 2. Левая часть


уравнения является произведение выражений (x − 1) и
1114

(x − 2). А произведение равно нулю, если хотя бы один из


сомножителей равен нулю (или сомножитель (x − 1) или
сомножитель (x − 2)).

Найдем такое x при котором выражения (x − 1) или


(x − 2) обращаются в нули:

Подставляем по-очереди найденные значения в исходное


уравнение  и убеждаемся, что при этих
значениях левая часть равняется нулю:

Когда корней бесконечно много

Уравнение может иметь бесконечно много корней. То есть


подставив в такое уравнение любое число, мы получим
верное числовое равенство.

Пример 1. Решить уравнение 

Корнем данного уравнения является любое число. Если


раскрыть скобки в левой части уравнения и привести
подобные слагаемые, то получится равенство 14 = 14. Это
равенство будет получаться при любом x
1115

Пример 2. Решить уравнение 

Корнем данного уравнения является любое число. Если


раскрыть скобки в левой части уравнения, то получится
равенство 10x + 12 = 10x + 12. Это равенство будет
получаться при любом x

Когда корней нет

Случается и так, что уравнение вовсе не имеет решений, то


есть не имеет корней. Например уравнение не имеет
корней, поскольку при любом значении x, левая часть
уравнения не будет равна правой части. Например, пусть
. Тогда уравнение примет следующий вид

Пусть

Пример 2. Решить уравнение 

Раскроем скобки в левой части равенства:

Приведем подобные слагаемые:


1116

Видим, что левая часть не равна правой части. И так будет


при любом значении y. Например, пусть y = 3.

Буквенные уравнения

Уравнение может содержать не только числа с


переменными, но и буквы.

Например, формула нахождения скорости является


буквенным уравнением:

Данное уравнение описывает скорость движения тела при


равноускоренном движении.

Полезным навыком является умение выразить любой


компонент, входящий в буквенное уравнение. Например,

чтобы из уравнения   определить расстояние, нужно


выразить переменную s.

Умнóжим обе части уравнения  на t


1117

В правой части переменные t сократим на t и перепишем то,


что у нас осталось:

В получившемся уравнении левую и правую часть поменяем


местами:

У нас получилась формула нахождения расстояния, которую


мы изучали ранее.

Попробуем из уравнения   определить время. Для этого


нужно выразить переменную t.

Умнóжим обе части уравнения на t

В правой части переменные t сократим на t и перепишем то,


что у нас осталось:

В получившемся уравнении v × t = s обе части разделим на


v
1118

В левой части переменные v сократим на v и перепишем то,


что у нас осталось:

У нас получилась формула определения времени, которую


мы изучали ранее.

Предположим, что скорость поезда равна 50 км/ч

v = 50 км/ч

А расстояние равно 100 км

s = 100 км

Тогда буквенное уравнение примет следующий вид

Из этого уравнения можно найти время. Для этого нужно


суметь выразить переменную t. Можно воспользоваться
правилом нахождения неизвестного делителя, разделив
делимое на частное и таким образом определить значение
переменной t

либо можно воспользоваться тождественными


преобразованиями. Сначала умножить обе части
уравнения на t
1119

Затем разделить обе части на 50

Пример 2. Дано буквенное уравнение . Выразите из


данного уравнения x

Вычтем из обеих частей уравнения a

Разделим обе части уравнения на b

Теперь, если нам попадется уравнение вида a + bx = c, то у


нас будет готовое решение. Достаточно будет подставить в
него нужные значения. Те значения, которые будут
подставляться вместо букв a, b, c принято называть
параметрами. А уравнения вида
a + bx = c называют уравнением с параметрами. В
зависимости от параметров, корень будет меняться.
1120

Решим уравнение 2 + 4x = 10. Оно похоже на буквенное


уравнение a + bx = c.  Вместо того, чтобы выполнять
тождественные преобразования, мы можем воспользоваться
готовым решением. Сравним оба решения:

Видим, что второе решение намного проще и короче.

Для готового решения необходимо сделать небольшое


замечание. Параметр b не должен быть равным нулю (b ≠ 0),
поскольку деление на ноль на допускается.

Пример 3. Дано буквенное уравнение 


. Выразите из данного уравнения x

Раскроем скобки в обеих частях уравнения

Воспользуемся переносом слагаемых. Параметры,


содержащие переменную x, сгруппируем в левой части
уравнения, а параметры свободные от этой переменной — в
правой.
1121

В левой части вынесем за скобки множитель x

Разделим обе части на выражение a − b

В левой части числитель и знаменатель можно сократить


на a − b. Так окончательно выразится переменная x

Теперь, если нам попадется уравнение вида


a(x − c) = b(x + d), то у нас будет готовое решение.
Достаточно будет подставить в него нужные значения.

Допустим нам дано уравнение 4(x − 3) = 2(x + 4). Оно похоже


на уравнение a(x − c) = b(x + d). Решим его двумя
способами: при помощи тождественных преобразований и
при помощи готового решения:

Для удобства вытащим из


уравнения 4(x − 3) = 2(x + 4) значения параметров a, b, c, d.
Это позволит нам не ошибиться при подстановке:
1122

Как и в прошлом примере знаменатель здесь не должен быть


равным нулю (a − b ≠ 0). Если нам встретится уравнение
вида a(x − c) = b(x + d) в котором параметры a и b будут
одинаковыми, мы сможем не решая его сказать, что у
данного уравнения корней нет, поскольку разность
одинаковых чисел равна нулю.

Например, уравнение 2(x − 3) = 2(x + 4) является
уравнением вида a(x − c) = b(x + d). В уравнении
2(x − 3) = 2(x + 4) параметры a и b одинаковые. Если мы
начнём его решать, то придем к тому, что левая часть не
будет равна правой части:

Пример 4. Дано буквенное уравнение  . Выразите из


данного уравнения x

Приведем левую часть уравнения к общему знаменателю:


1123

Умнóжим обе части на a

В левой части x вынесем за скобки

Разделим обе части на выражение (1 − a)

Линейные уравнения с одним неизвестным

Рассмотренные в данном уроке уравнения называют


линейными уравнениями первой степени с одним
неизвестным.

Если уравнение дано в первой степени, не содержит деления


на неизвестное, а также не содержит корней из неизвестного,
то его можно назвать линейным. Мы еще не изучали степени
1124

и корни, поэтому чтобы не усложнять себе жизнь, слово


«линейный» будем понимать как «простой».

Большинство уравнений, решенных в данном уроке, в


конечном итоге сводились к простейшему уравнению, в
котором нужно было произведение разделить на известный
сомножитель. Таковым к примеру является уравнение
2(x + 3) = 16. Давайте решим его.

Раскроем скобки в левой части уравнения, получим 2x + 6


= 16. Перенесем слагаемое 6 в правую часть, изменив знак.
Тогда получим 2x = 16 − 6. Вычислим правую часть,
получим 2x = 10. Чтобы найти x, разделим
произведение 10 на известный сомножитель 2. Отсюда x = 5.

Уравнение 2(x + 3) = 16 является линейным. Оно свелось к


уравнению 2x = 10, для нахождения корня которого
потребовалось разделить произведение на известный
сомножитель. Такое простейшее уравнение называют
линейным уравнением первой степени с одним
неизвестным в каноническом виде. Слово «канонический»
является синонимом слов «простейший» или «нормальный».

Линейное уравнение первой степени с одним


неизвестным в каноническом виде называют уравнение
вида ax = b.

Полученное нами уравнение 2x = 10 является линейным


уравнением первой степени с одним неизвестным в
каноническом виде. У этого уравнения первая степень, одно
неизвестное, оно не содержит деления на неизвестное и не
содержит корней из неизвестного, и представлено оно в
каноническом виде, то есть в простейшем виде при котором
1125

легко можно определить значение x. Вместо параметров a и


b в нашем уравнении содержатся числа 2 и 10. Но подобное
уравнение может содержать и другие числа: положительные,
отрицательные или равные нулю.

Если в линейном уравнении a = 0 и b = 0, то уравнение имеет


бесконечно много корней. Действительно, если a равно
нулю и b равно нулю, то линейное уравнение ax = b примет
вид 0x = 0. При любом значении x левая часть будет равна
правой части.

Если в линейном уравнении a = 0 и b ≠ 0, то уравнение


корней не имеет. Действительно, если a равно нулю и b
равно какому-нибудь числу, не равному нулю, скажем числу
5, то уравнение ax = b примет вид 0x = 5. Левая часть будет
равна нулю, а правая часть пяти. А ноль не равен пяти.

Если в линейном уравнении a ≠ 0, и b равно любому числу,


то уравнение имеет один корень. Он определяется делением
параметра b на параметр a

Действительно, если a равно какому-нибудь числу, не


равному нулю, скажем числу 3, и b равно какому-нибудь

числу, скажем числу 6, то уравнение   примет вид .


Отсюда  .

Существует и другая форма записи линейного уравнения


первой степени с одним неизвестным. Выглядит она
следующим образом: ax − b = 0. Это то же самое уравнение,
что и ax = b, но параметр b перенесен в левую часть с
1126

противоположным знаком. Такие уравнение мы тоже решали


в данном уроке. Например, уравнение 7x − 77 = 0. Уравнение
вида ax − b = 0 называют линейным уравнением первой
степени с одним неизвестным в общем виде.

В будущем после изучения рациональных выражений, мы


рассмотрим такие понятия, как посторонние корни и потеря
корней. А пока рассмотренного в данном уроке будет
достаточным.

Задания для самостоятельного решения


Задание 1. Используя метод переноса слагаемого, решите
следующее уравнение:

Показать решение
Задание 2. Используя метод прибавления (или вычитания)
числа к обеим частям, решите следующее уравнение:

Показать решение
Задание 3. Решите уравнение:

Показать решение
Задание 4. Решите уравнение:

Показать решение
Задание 5. Решите уравнение:

Показать решение
1127

Задание 6. Решите уравнение:

Показать решение
Задание 7. Решите уравнение:

Показать решение
Задание 8. Решите уравнение:

Показать решение
Задание 9. Решите уравнение:

Показать решение
Задание 10. Решите уравнение:

Показать решение
Задание 11. Решите уравнение:

Показать решение
Задание 12. Решите уравнение:

Показать решение
Задание 13. Решите уравнение:
1128

Показать решение
Задание 14. Решите уравнение:

Показать решение
Задание 15. Решите уравнение:

Показать решение
Задание 16. Решите уравнение:

Показать решение
Задание 17. Решите уравнение:

Показать решение
Задание 18. Решите уравнение:

Показать решение
Задание 19. Решите уравнение:

Показать решение
Задание 20. Решите уравнение:

Показать решение
1129

Задание 21. Решите уравнение:

Показать решение
Задание 22. Решите уравнение:

Показать решение
Задание 23. Решите уравнение:

Показать решение
Задание 24. Решите уравнение:

Показать решение
Задание 25. Решите уравнение:

Показать решение
Задание 26. Решите уравнение:

Показать решение
Задание 27. Решите уравнение:

Показать решение
1130

Задание 28. Решите уравнение:

Показать решение
Задание 29. Решите уравнение:

Показать решение
Задание 30. Решите уравнение:

Показать решение
Задание 31. Решите уравнение:

Показать решение
Задание 32. В следующем буквенном уравнении выразите
переменную x:

Показать решение
Задание 33. В следующем буквенном уравнении выразите
переменную x:

Показать решение
Задание 34. В следующем буквенном уравнении выразите
переменную x:
1131

Показать решение
Задание 35. В следующем буквенном уравнении выразите
переменную x:

Показать решение
Задание 36. В следующем буквенном уравнении выразите
переменную y:

Показать решение
Задание 37. В следующем буквенном уравнении выразите
переменную z:

Показать решение
1132

Решение задач с помощью уравнений

Решение задачи обычно свóдится к тому, чтобы путем


логических рассуждений и вычислений найти значение какой-
нибудь величины. Например, найти скорость, время,
расстояние, массу какого-нибудь предмета или количество
чего-то.

Такую задачу можно решить с помощью уравнения. Для


этого искомое значение обозначают через переменную,
затем путем логических рассуждений составляют и решают
уравнение. Решив уравнение, производят проверку на то,
удовлетворяет ли решение уравнения условиям задачи.

Содержание урока

 Запись выражений, содержащих неизвестное


 Примеры решения задач
 Задачи для самостоятельного решения

Запись выражений, содержащих неизвестное

Решение задачи сопровождается составлением уравнения к


этой задаче. На начальном этапе изучения задач желательно
научиться составлять буквенные выражения, описывающие
ту или иную жизненную ситуацию. Этот этап не является
сложным и его можно изучать в процессе решения самой
задачи.

Рассмотрим несколько ситуаций, которые можно записать с


помощью математического выражения.
1133

Задача 1. Возраст отца x лет. Мама на два года младше.


Сын младше отца в 3 раза. Запишите возраст каждого с
помощью выражений.

Решение:

Задача 2. Возраст отца x лет, мама на 2 года младше отца.


Сын младше отца в 3 раза, дочь младше матери в 3 раза.
Запишите возраст каждого с помощью выражений.

Решение:
1134

Задача 3. Возраст отца x лет, мама на 3 года младше отца.


Сын младше отца в 3 раза, дочь младше матери в 3 раза.
Сколько лет каждому, если общий возраст отца, мамы, сына
и дочери составляет 92 года?

Решение:

В данной задаче помимо записи выражений, необходимо


вычислить возраст каждого члена семьи.

Сначала запишем возраст каждого члена семьи с помощью


выражений. За переменную x примем возраст отца, и далее
пользуясь этой переменной составим остальные выражения:

Теперь определим возраст каждого члена семьи. Для этого


нам нужно составить и решить уравнение. Все компоненты
1135

уравнения у нас уже готовы. Осталось только собрать их


воедино.

Общий возраст в 92 года получился путем сложения


возрастов папы, мамы, сына и дочери:

Для каждого возраста мы составили математическое


выражение. Эти выражения и будут компонентами нашего
уравнения. Давайте соберем наше уравнение согласно
данной схеме и таблице, которая была приведена выше. То
есть слова папа, мама, сын, дочь заменим на
соответствующее им в таблице выражение:

Выражение, отвечающее за возраст мамы x − 3, для


наглядности было взято в скобки.

Теперь решим получившееся уравнение. Для начала можно


раскрыть скобки там, где это можно:

Чтобы освободить уравнение от дробей, умножим обе части


на 3
1136

Решим получившееся уравнение, пользуясь известными


тождественными преобразованиями:

Мы нашли значение переменной x. Эта переменная отвечала


за возраст отца. Значит возраст отца составляет 36 лет.

Зная возраст отца, можно вычислить возрасты остальных


членов семьи. Для этого нужно подставить значение
переменной x в те выражения, которые отвечают за возраст
конкретного члена семьи.

В задаче было сказано, что мама на 3 года младше отца. Ее


возраст мы обозначили через выражение x−3. Значение
переменной x теперь известно, и чтобы вычислить возраст
мамы, нужно в выражении x − 3 вместо x подставить
найденное значение 36

x − 3 = 36 − 3 = 33 года маме.

Аналогично определяется возраст остальных членов семьи:


1137

Проверка:

Задача 4. Килограмм яблок стоит x рублей. Запишите


выражение, вычисляющее сколько килограмм яблок можно
купить на 300 рублей.

Решение
1138

Если килограмм яблок стоит x рублей, то на 300 рублей

можно купить килограмм яблок.

Пример. Килограмм яблок стоит 50 рублей. Тогда на 300

рублей можно купить , то есть 6 килограмм яблок.

Задача 5. На x рублей было куплено 5 кг яблок. Запишите


выражение, вычисляющее сколько рублей стоит один
килограмм яблок.

Решение

Если за 5 кг яблок было уплачено x рублей, то один

килограмм будет стоит  рублей

Пример. За 300 рублей было куплено 5 кг яблок. Тогда один

килограмм яблок будет стоит  , то есть 60 рублей.

Задача 6. Том, Джон и Лео на перемене пошли в столовую и


купили по бутерброду и по кружке кофе. Бутерброд стоит x
рублей, а кружка кофе — 15 рублей. Определите стоимость
бутерброда, если известно, что за всё было уплачено 120
рублей?

Решение

Конечно, данная задача проста как три копейки и ее можно


решить не прибегая к уравнению. Для этого из 120 рублей
1139

нужно вычесть стоимость трех кружек кофе (15 × 3), и


полученный результат разделить на 3

Но наша цель — составить уравнение к задаче и решить это


уравнение. Итак, стоимость бутерброда x рублей. Куплено их
всего три. Значит увеличив стоимость в три раза, мы
получим выражение описывающее сколько рублей было
уплачено за три бутерброда

3x — стоимость трех бутербродов

А стоимость трех кружек кофе можно записать как 15 × 3. 15


это стоимость одной кружки кофе, а 3 множитель (Том,  Джон
и Лео), увеличивающий эту стоимость в три раза.

По условию задачи за все уплачено 120 рублей. У нас уже


появляется примерная схема, что нужно делать:

Выражения, описывающие стоимость трех бутербродов и


трех кружек кофе, у нас уже готовы. Это выражения 3x и
15 × 3. Пользуясь схемой составим уравнение и решим его:
1140

Итак, стоимость одного бутерброда составляет 25 рублей.

Задача решается верно только в том случае, если уравнение


к ней составлено правильно. В отличие от обычных
уравнений, по которым мы учимся находить корни, уравнения
для решения задач имеют своё конкретное применение.
Каждый компонент такого уравнения может быть описан в
словесной форме. Составляя уравнение, обязательно нужно
понимать для чего мы включаем в его состав тот или иной
компонент и зачем он нужен.

Также необходимо помнить, что уравнение это равенство,


после решения которого левая часть должна будет
равняться правой части. Составленное уравнение не должно
противоречить этой идее.

Представим, что уравнение это весы с двумя чашами и


экраном, показывающим состояние весов.

В данный момент экран показывает знак равенства. Понятно


почему левая чаша равна правой чаше — на чашах ничего
1141

нет. Состояние весов и отсутствие на чашах чего-либо


запишем с помощью следующего равенства:

0=0

Положим на левую чашу весов арбуз:

Левая чаша перевесила правую чашу и экран забил тревогу,


показав знак не равно ( ≠ ). Этот знак говорит о том, что
левая чаша не равна правой чаше.

Теперь попробуем решить задачу. Пусть требуется узнать


сколько весит арбуз, который лежит на левой чаше. Но как
это узнать? Ведь наши весы предназначены только для
проверки равна ли левая чаша правой.

На помощь приходят уравнения. Вспомним, что уравнение по


определению есть равенство, содержащее в себе
переменную значение которой требуется найти. Весы в
данном случае играют роль этого самого уравнения, а масса
арбуза это переменная, значение которой нужно найти. Наша
цель правильно составить это уравнение. Понимай,
1142

выровнять весы так, чтобы можно было вычислить массу


арбуза.

Чтобы выровнять весы, на правую чашу можно положить


какой-нибудь тяжелый предмет. Например, положим туда
гирю массой 7 кг.

Теперь наоборот правая чаша перевесила левую. Экран по


прежнему показывает, что чаши не равны.

Попробуем на левую чашу положить гирю массой 4 кг


1143

Теперь весы выровнялись. На рисунке видно, что левая чаша


на уровне правой чаши. А экран показывает знак равенства.
Этот знак говорит о том, что левая чаша равна правой чаше.

Таким образом мы получили уравнение — равенство,


содержащее неизвестное. Левая чаша — это левая часть
уравнения, состоящая из компонентов 4 и переменной x
(массы арбуза), а правая чаша — это правая часть
уравнения, состоящая из компонента 7.
1144

Ну и нетрудно догадаться, что корень уравнения 4 + x = 7


равен 3. Значит масса арбуза равна 3 кг.

Аналогично дела обстоят и с другими задачами. Чтобы найти


какое-нибудь неизвестное значение, к левой или к правой
части уравнения добавляют различные элементы:
слагаемые, множители, выражения. В школьных задачах эти
элементы бывают уже даны. Остается только правильно
структурировать их и построить уравнение. Мы же в данном
примере занимались подбором, пробуя гири разной массы,
чтобы вычислить массу арбуза.

Естественно, те данные которые даны в задаче сначала


нужно привести к виду, при котором их можно включить в
уравнение. Поэтому, как говорят «хочешь не хочешь, а
думать придётся».

Рассмотрим следующую задачу. Возраст отца равен


возрасту сына и дочери вместе. Сын вдвое старше дочери и
на двадцать лет моложе отца. Сколько лет каждому?

Возраст дочери можно обозначить через x. Если сын вдвое


старше дочери, то его возраст будет обозначаться как 2x. В
условии задачи сказано, что вместе возраст дочери и сына
равен возрасту отца. Значит возраст отца будет
обозначаться суммой x + 2x
1145

В выражении можно привести подобные слагаемые.


Тогда возраст отца будет обозначаться как 3x

Теперь составим уравнение. Нам нужно получить равенство


в котором можно найти неизвестное x. Воспользуемся
весами. На левую чашу положим возраст отца (3x), а на
правую чашу возраст сына (2x)

Понятно почему левая чаша перевесила правую и почему


экран показывает знак ( ≠ ). Ведь логично, что возраст отца
больше возраста сына.

Но нам нужно уравнять весы, чтобы можно было вычислить


неизвестное x. Для этого к правой чаше нужно прибавить
какое-нибудь число. Какое именно число указано в задаче. В
условии было сказано, что сын моложе отца на 20 лет.
Значит 20 лет это то самое число, которое нужно положить
на весы.

Весы выровнятся, если мы эти 20 лет добавим на правую


чашу весов. Иными словами, вырастим сына до возраста
отца
1146

Теперь весы выровнялись. Получилось уравнение ,


которое решается легко:

В начале решения данной задачи через переменную x мы


обозначили возраст дочери. Теперь мы нашли значение этой
переменной. Дочери 20 лет.

Далее было сказано, что сын двое старше дочери, значит


сыну (20 × 2), то есть 40 лет.

Ну и наконец вычислим возраст отца. В задаче было сказано,


что он равен сумме возрастов сына и дочери, то
есть (20 + 40) лет.
1147

Вернемся к середине задачи и обратим внимание на один


момент. Когда мы положили на весы возраст отца и возраст
сына, левая чаша перевесила правую

Но мы решили эту проблему, добавив на правую чашу еще


20 лет. В результате весы выровнялись и мы получили
равенство 
1148

Но можно было не добавлять к правой чаше эти 20 лет, а


вычесть их из левой. Мы получили бы равенство и в таком
случае

В этот раз получается уравнение . Корень


уравнения по прежнему равен 20
1149

То есть уравнения и являются


равносильными. А мы помним, что у равносильных
уравнений корни совпадают. Если внимательно посмотреть
на эти два уравнения, то можно увидеть что второе
уравнение получено путем переноса числа 20 из правой
части в левую с противоположным знаком. А это действие,
как было указано в предыдущем уроке, не меняет корней
уравнения.

Также нужно обратить внимание на то, что в начале решения


задачи возрасты каждого члена семьи можно было
обозначить через другие выражения.

Скажем возраст сына обозначить через x и поскольку он двое

старше дочери, то возраст дочери обозначить через 


(понимай сделать её младше сына в два раза). А возраст
отца поскольку он является суммой возрастов сына и дочери

обозначить через выражение . Ну и напоследок для


построения логически правильного уравнения, к возрасту
сына нужно  прибавить число 20, ведь отец старше на
двадцать лет. В итоге получается совсем другое

уравнение  . Решим это уравнение


1150

Как видно ответы к задаче не поменялись. Сыну по прежнему

40 лет. Дочери по прежнему лет, а отцу 40 + 20 лет.

Другими словами, задача может решаться различными


методами. Поэтому не следует отчаиваться, что не
получается решить ту или иную задачу. Но нужно иметь
ввиду, что существует наиболее простые пути решения
задачи. К центру города можно доехать различными
маршрутами, но всегда существует наиболее удобный,
быстрый и безопасный маршрут.

Примеры решения задач

Задача 1. В двух пачках всего 30 тетрадей. Если бы из


первой пачки переложили во вторую 2 тетради, то в первой
пачке стало бы вдвое больше тетрадей, чем во второй.
Сколько тетрадей было в каждой пачке?

Решение

Обозначим через x количество тетрадей, которое было в


первой пачке. Если всего тетрадей было 30, а переменная x
1151

это количество тетрадей из первой пачке, то количество


тетрадей во второй пачке будет обозначаться через
выражение 30 − x. То есть от общего количества тетрадей
вычитаем количество тетрадей из первой пачки и тем самым
получаем количество тетрадей из второй пачки.

Далее сказано, что если переложить 2 тетради из первой


пачки во вторую, то в первой пачке окажется вдвое больше
тетрадей. Итак, снимем с первой пачки две тетради

и добавим эти две тетради во вторую пачку

Выражения из которых мы будем составлять уравнение


теперь принимают следующий вид:

Попробуем составить уравнение из имеющихся выражений.


Положим на весы обе пачки тетрадей
1152

Левая чаша тяжелее правой. Это потому, что в условии


задачи сказано, что после того как из первой пачки взяли две
тетради и положили их во вторую, количество тетрадей в
первой пачке стало вдвое больше, чем во второй.

Чтобы выровнять весы и получить уравнение, увеличим


правую часть вдвое. Для этого умножим её на 2

Получается уравнение  . Решим данное


уравнение:
1153

Первую пачку мы обозначали через переменную x. Теперь


мы нашли её значение. Переменная x равна 22. Значит в
первой пачке было 22 тетради.

А вторую пачку мы обозначали через выражение 30 − x и


поскольку значение переменой x теперь известно, то можно
вычислить количество тетрадей во второй пачке. Оно равно
30 − 22, то есть 8 шт.

Задача 2. Два человека чистили картофель. Один очищал в


минуту две картофелины, а второй — три картофелины.
Вместе они очистили 400 шт. Сколько времени работал
каждый, если второй проработал на 25 минут больше
первого?

Решение

Обозначим через x время работы первого человека.


Поскольку второй человек проработал на 25 минут больше
первого, то его время будет обозначаться через выражение
1154

Первый рабочий в минуту очищал 2 картофелины, и


поскольку он работал x минут, то всего он очистил 2x
картофелин.

Второй человек в минуту очищал три картофелины, и


поскольку он работал минут, то всего он очистил
картофелин.

Вместе они очистили 400 картофелин

Из имеющихся компонентов составим и решим уравнение. В


левой части уравнения будут картофелины, очищенные
каждым человеком, а в правой части их сумма:

В начале решения данной задачи через переменную x мы


обозначили время работы первого человека. Теперь мы
нашли значение этой переменной. Первый человек работал
65 минут.

А второй человек работал   минут, и поскольку значение


переменной x теперь известно, то можно вычислить время
1155

работы второго человека — оно равно 65 + 25, то есть 90


мин.

Задача из Учебника по алгебре Андрея Петровича


Киселева. Из сортов чая составлена смесь в 32 кг.
Килограмм первого сорта стоит 8 руб., а второго сорта 6 руб.
50 коп. Сколько килограммов взято того и другого сорта, если
килограмм смеси стоит (без прибыли и убытка) 7 руб. 10
коп.?

Решение

Обозначим через x массу чая первого сорта. Тогда масса чая


второго сорта будет обозначаться через выражение 32 − x

Килограмм чая первого сорта стоит 8 руб. Если эти восемь


рублей умножить на количество килограмм чая первого
сорта, то можно будет узнать во сколько рублей обошлись x
кг чая первого сорта.

А килограмм чая второго сорта стоит 6 руб. 50 коп. Если


эти 6 руб. 50 коп. умножить на 32 − x, то можно узнать во
сколько рублей обошлись 32 − x кг чая второго сорта.

В условии сказано, что килограмм смеси стоит 7 руб. 10 коп.


Всего же было приготовлено 32 кг смеси. Умножим 7 руб. 10
коп. на 32 мы сможем узнать сколько стоит 32 кг смеси.
1156

Выражения из которых мы будем составлять уравнение


теперь принимают следующий вид:

Попробуем составить уравнение из имеющихся выражений.


Положим на левую чашу весов стоимость смесей чая первого
и второго сорта, а на правую чашу положим стоимость 32 кг
смеси, то есть общую стоимость смеси, в составе которой
оба сорта чая:

Получили уравнение  . Решим его:


1157

В начале решения данной задачи через переменную x мы


обозначили массу чая первого сорта. Теперь мы нашли
значение этой переменной. Переменная x равна 12,8. Значит
для приготовления смеси было взято 12,8 кг чая первого
сорта.

А через выражение 32 − x мы обозначили массу чая второго


сорта и поскольку значение переменой x теперь известно, то
можно вычислить массу чая второго сорта. Оно равно
32 − 12,8 то есть 19,2. Значит для приготовления смеси было
взято 19,2 кг чая второго сорта.

Задача 3. Велосипедист проехал некоторое расстояние со


скоростью 8 км/ч. Возвратиться он должен был другой
дорогой, которая была на 3 км длиннее первой, и, хотя
возвращаясь, ехал со скоростью 9 км/ч, он употребил

времени на  минут более. Как длинны были дороги?

Решение

Некоторые задачи могут затрагивать темы, которые человек


возможно не изучал. Данная задача относится к такому кругу
задач. В ней затрагиваются понятия расстояния, скорости и
1158

времени. Соответственно, чтобы решить подобную задачу,


нужно иметь представление о тех вещах, о которых
говорится в задаче. В нашем случае, надо знать что
представляет собой расстояние, скорость и время.

В задаче нужно найти расстояния двух дорог. Мы должны


составить уравнение, которое позволит вычислить эти
расстояния.

Вспомним, как взаимосвязаны расстояние, скорость и время.


Каждая из этих величин может быть описана с помощью
буквенного уравнения:

Правую часть одного из этих уравнений мы будем


использовать для составления своего уравнения. Чтобы
узнать какую именно, нужно вернуться к тексту задачи и
обратить внимание на следующий момент:

Следует обратить внимание на момент, где велосипедист на

обратном пути употребил времени на минут более. Эта


подсказка указывает нам, что можно воспользоваться
1159

уравнением   , а именно его правой частью. Это позволит


нам составить уравнение, которое содержит переменную S.

Итак, обозначим длину первой дороги через S. Этот путь


велосипедист проехал со скоростью 8 км/ч. Время за которое

он преодолел этот путь будет обозначаться выражением ,


поскольку время это отношение пройденного расстояния к
скорости

Обратная дорога для велосипедиста была длиннее на 3 км.


Поэтому её расстояние будет обозначаться через выражение
S + 3. Эту дорогу велосипедист проехал со скоростью 9 км/ч.
А значит время за которое он преодолел этот путь будет

обозначаться выражением .

Теперь составим уравнение из имеющихся выражений


1160

Правая чаша тяжелее левой. Это потому, что в задаче


сказано, что на обратную дорогу велосипедист затратил

времени на больше.

Чтобы уравнять весы прибавим к левой части эти самые  


минут. Но сначала переведем минуты в часы, поскольку в
задаче скорость измеряется в километрах в час, а не в
метрах в минуту.

Чтобы   минут перевести в часы, нужно разделить их на 60

 минут составляют   часа. Прибавляем эти   часа к левой


части уравнения:
1161

Получается уравнение   . Решим данное


уравнение. Чтобы избавиться от дробей, обе части части
можно умножить на 72. Далее пользуясь известными
тождественными преобразованиями, найдем значение
переменной S

Через переменную S мы обозначали расстояние первой


дороги. Теперь мы нашли значение этой переменной.
Переменная S равна 15. Значит расстояние первой дороги
составляет 15 км.
1162

А расстояние второй дороги мы обозначили через


выражение S + 3, и поскольку значение переменной S теперь
известно, то можно вычислить расстояние второй дороги.
Это расстояние равно сумме 15 + 3, то есть 18 км.

Задача 4. По шоссе идут две машины с одной и той же


скоростью. Если первая увеличит скорость на 10 км/ч, а
вторая уменьшит скорость на 10 км/ч, то первая за 2 ч
пройдет столько же, сколько вторая за 3 ч. С какой скоростью
идут автомашины?

Решение

Обозначим через v скорость каждой машины. Далее в задаче


приводятся подсказки: скорость первой машины увеличить
на 10 км/ч, а скорость второй — уменьшить на 10 км/ч.
Воспользуемся этой подсказкой

Далее говорится, что при таких скоростях (увеличенных и


уменьшенных на 10 км/ч) первая машина пройдет за 2 часа
столько же расстояния сколько вторая за 3 часа. Фразу
«столько же» можно понимать как «расстояние,
пройденное первой машиной, будет равно расстоянию,
пройденному второй машиной».

Расстояние как мы помним, определяется по формуле .


Нас интересует правая часть этого буквенного уравнения —
она позволит нам составить уравнение, содержащее
переменную v.
1163

Итак, при скорости v + 10 км/ч первая машина пройдет


2(v+10) км, а вторая пройдет 3(v − 10) км. При таком условии
машины пройдут одинаковые расстояния, поэтому для
получения уравнения достаточно соединить эти два
выражения знаком равенства. Тогда получим
уравнение  . Решим его:

В условии задачи было сказано, что машины идут с


одинаковой скоростью. Мы обозначили эту скорость через
переменную v. Теперь мы нашли значение этой переменной.
Переменная v равна 50. Значит скорость обеих машин
составляла 50 км/ч.

Задача 5. За 9 ч по течению реки теплоход проходит тот же


путь, что за 11 ч против течения. Найдите собственную
скорость теплохода, если скорость течения реки 2 км/ч.

Решение

Обозначим через v собственную скорость теплохода.


Скорость течения реки равна 2 км/ч. По течению реки
скорость теплохода будет составлять v + 2 км/ч, а против
течения — (v − 2) км/ч.

В условии задачи сказано, что за 9 ч по течению реки


теплоход проходит тот же путь, что за 11 ч против течения.
1164

Фразу «тот же путь» можно понимать как «расстояние,


пройденное теплоходом по течению реки за 9 часов, равно
расстоянию, пройденному теплоходом против течения
реки за 11 часов». То есть расстояния будут одинаковыми.

Расстояние определяется по формуле . Воспользуемся


правой частью этого буквенного уравнения для составления
своего уравнения.

Итак, за 9 часов по течению реки теплоход


пройдет 9(v + 2) км, а за 11 часов против течения —
11(v − 2) км. Поскольку оба выражения описывают одно и то
же расстояние, приравняем первое выражение ко второму. В
результате получим уравнение  . Решим его:

Значит собственная скорость теплохода составляет 20 км/ч.

При решении задач полезной привычкой является заранее


определить на каком множестве ищется для неё решение.

Допустим, что в задаче требовалось найти время, за которое


пешеход преодолеет указанный путь. Мы обозначили время
через переменную t, далее составили уравнение,
содержащее эту переменную и нашли её значение.
1165

Из практики мы знаем, что время движения объекта может


принимать как целые значения, так и дробные, например 2 ч,
1,5 ч, 0,5 ч. Тогда можно сказать, что решение данной задачи
ищется на множестве рациональных чисел Q, поскольку
каждое из значений 2 ч, 1,5 ч, 0,5 ч может быть представлено
в виде дроби.

Поэтому после того, как неизвестную величину обозначили


через переменную, полезно указать к какому множеству эта
величина принадлежит. В нашем примере время t
принадлежит множеству рациональных чисел Q

t∈Q

Ещё можно ввести ограничение для переменной t, указав что


она может принимать только положительные значения.
Действительно, если объект затратил на путь определенное
время, то это время не может быть отрицательным. Поэтому
рядом с выражением t ∈ Q укажем, что её значение должно
быть больше нуля:

t ∈ R, t > 0

Если решив уравнение, мы получим отрицательное значение


для переменной t, то можно будет сделать вывод, что задача
решена неправильно, поскольку это решение не будет
удовлетворять условию t ∈ Q, t > 0.

Ещё пример. Если бы мы решали задачу в которой


требовалось найти количество человек для выполнения той
или иной работы, то это количество мы обозначили бы через
переменную x. В такой задаче решение искалось бы на
множестве натуральных чисел
1166

x ∈ N

Действительно, количество человек является целым числом,


например 2 человека, 3 человека, 5 человек. Но никак не 1,5
(один целый человек и половина человека) или 2,3 (два
целых человека и еще три десятых человека).

Здесь можно было бы указать, что количество человек


должно быть больше нуля, но числа входящие во множество
натуральных чисел N сами по себе являются
положительными и большими нуля. В этом множестве нет
отрицательных чисел и числа 0. Поэтому выражение x > 0
можно не писать.

Задача 6. Для ремонта школы прибыла бригада в которой


было в 2,5 раза больше маляров, чем плотников. Вскоре
прораб включил в бригаду еще четырех маляров, а двух
плотников перевел на другой объект. В результате маляров в
бригаде оказалось в 4 раза больше чем плотников. Сколько
маляров и сколько плотников было в бригаде первоначально

Решение

Обозначим через x плотников, прибывших на ремонт


первоначально.

Количество плотников является целым числом, большим


нуля. Поэтому укажем, что x принадлежит множество
натуральных чисел

x ∈ N
1167

Маляров было в 2,5 раза больше, чем плотников. Поэтому


количество маляров будет обозначаться как 2,5x.

Далее говорится, что прораб включил в бригаду еще четырех


маляров, а двух плотников перевел на другой объект.
Сделаем для своих выражений тоже самое. Уменьшим
количество плотников на 2

А количество маляров увеличим на 4

Теперь количество плотников и маляров будут обозначаться


через следующие выражения:

Попробуем составить уравнение из имеющихся выражений:


1168

Правая чаша больше, поскольку после включения в бригаду


ещё четырёх маляров, и перемещения двух плотников на
другой объект, количество маляров в бригаде оказалось в 4
раза больше чем плотников. Чтобы уравнять весы, нужно
левую чашу увеличить в 4 раза:

Получили уравнение  . Решим его:


1169

Через переменную x было обозначено первоначальное


количество плотников. Теперь мы нашли значение этой
переменной.  Переменная x равна 8. Значит 8 плотников
было в бригаде первоначально.

А количество маляров было обозначено через


выражение 2,5x и поскольку значение переменной x теперь
известно, то можно вычислить количество маляров — оно
равно 2,5 × 8, то есть 20.

Возвращаемся к началу задачи и удостоверяемся, что


соблюдается условие x ∈ N. Переменная x равна 8, а
элементы множества натуральных чисел N это все числа,
начинающиеся с 1, 2, 3 и так далее до бесконечности. В это
же множество входит число 8, которое мы нашли.

8 ∈ N

Тоже самое можно сказать о количестве маляров. Число 20


принадлежит множеству натуральных чисел:

20 ∈ N

Для понимания сути задачи и правильного составления


уравнения, вовсе необязательно использовать модель весов
1170

с чашами. Можно использовать и другие модели: отрезки,


таблицы, схемы. Можно придумать свою модель, которая
хорошо описывала бы суть задачи.

Задача 9. Из бидона отлили 30% молока. В результате в нем


осталось 14 л. Сколько литров молока было в бидоне
первоначально?

Решение

Искомое значение это первоначальное число литров в


бидоне. Изобразим число литров в виде линии и подпишем
эту линию как X

Сказано, что из бидона отлили 30% молока. Выделим на


рисунке приблизительно 30%

Процент по определению есть одна сотая часть чего-то. Если


30% молока отлили, то остальные 70% остались в бидоне.
На эти 70% приходятся 14 литров, указанные в задаче.
Выделим на рисунке оставшиеся 70%
1171

Теперь можно составить уравнение. Вспомним, как находить


процент от числа. Для этого общее количество чего-то делят
на 100 и полученный результат умножают на искомое
количество процентов. Замечаем, что 14 литров,
составляющих 70% можно получить таким же образом:
первоначальное число литров X разделить на 100 и
полученный результат умножить на 70. Всё это приравнять к
числу 14

Или получить более простое уравнение: 70% записать как


0,70, затем умножить на X и приравнять это выражение к 14

Значит первоначально в бидоне было 20 литров молока.

Задача 9. Взяли два сплава золота и серебра. В одном


количество этих металлов находится в отношении 1 : 9, а в
другом 2 : 3. Сколько нужно взять каждого сплава, чтобы
получить 15 кг нового сплава, в котором золото и серебро
относилось бы как 1 : 4?

Решение

Попробуем сначала узнать сколько золота и серебра будет


содержáться в 15 кг нового сплава. В задаче сказано, что
содержание этих металлов должно быть в отношении 1 : 4,
то есть на одну часть сплава должно приходиться золото, а
1172

на четыре части — серебро. Тогда всего частей в сплаве


будет 1 + 4 = 5, а масса одной части будет 15 : 5 = 3 кг.

Определим сколько золота будет содержáться в 15 кг сплава.


Для этого 3 кг умножим на количество частей золота:

3 кг × 1 = 3 кг

Определим сколько серебра будет содержáться в 15 кг


сплава:

3 кг × 4 = 12 кг

Значит сплав массой 15 кг будет содержать 3 кг золота и 12


кг серебра. Теперь вернёмся к исходным сплавам.
Использовать нужно каждый из них. Обозначим через x
массу первого сплава, а массу второго сплава можно
обозначить через 15 − x

Выразим в процентах все отношения, которые даны в задаче


и заполним ими следующую таблицу:
1173

В первом сплаве золото и серебро находятся в отношении


1 : 9. Тогда всего частей будет 1 + 9 = 10. Из них золота

будет , а серебра  .

Перенесём эти данные в таблицу. 10% занесём в первую


строку в графу «процент золота в сплаве», 90% также
занесём в первую строку графу «процент серебра в
сплаве», а в последнюю графу «масса сплава» занесём
переменную x, поскольку так мы обозначили массу первого
сплава:

Аналогично поступаем со вторым сплавом. Золото и серебро


в нём находятся в отношении 2 : 3. Тогда всего частей будет

2 + 3 = 5. Из них золота будет , а серебра

Перенесём эти данные в таблицу. 40% занесем во вторую


строку в графу «процент золота в сплаве», 60% также
занесём во вторую строку графу «процент серебра в
сплаве», а в последнюю графу «масса сплава» занесём
выражение 15 − x, поскольку так мы обозначили массу
второго сплава:
1174

Заполним последнюю строку. Полученный сплав массой 15 кг

будет содержать 3 кг золота, что составляет

сплава, а серебра будет  сплава. В последнюю


графу записываем массу полученного сплава 15

Теперь по данной таблице можно составить уравнения.


Вспоминаем задачи на концентрацию, сплавы и смеси. Если
мы отдельно сложим золото обоих сплавов и приравняем эту
сумму к массе золота полученного сплава, то сможем узнать
чему равно значение x.

Далее для удобства проценты будем выражать в десятичной


дроби.

В первом сплаве золота было 0,10x, а во втором сплаве


золота было 0,40(15 − x). Тогда в полученном сплаве масса
золота будет суммой масс золота первого и второго сплавов
1175

и эта масса составляет 20% от нового сплава. А 20% от 


нового сплава это 3 кг золота, вычисленные нами ранее. В
результате получаем уравнение 0,10x + 0.40(15 − x) = 3.
Решим это уравнение:

Изначально через x мы обозначили массу первого сплава.


Теперь мы нашли значение этой переменной. Переменная x
равна 10. А массу второго сплава мы обозначили через
15 − x, и поскольку значение переменной x теперь известно,
то можно вычислить массу второго сплава, она равна
15 − 10 = 5 кг.

Значит для получения нового сплава массой 15 кг в котором


золото и серебро относились бы как 1 : 4, нужно взять 10 кг
первого и 5 кг второго сплава.

Уравнение можно было составить, воспользовавшись и


вторым столбцом получившейся таблицы. Тогда мы
получили бы уравнение 0,90x + 0.60(15 − x) = 12. Корень
этого уравнения тоже равен 10
1176

Задача 10. Имеется руда из двух пластов с содержанием


меди в 6% и 11%. Сколько надо взять бедной руды, чтобы
получить при смешивании с богатой 20 тонн с содержанием
меди 8%?

Решение

Обозначим через x массу бедной руды. Поскольку нужно


получить 20 тонн руды, то богатой руды будет взято 20 − x.
Поскольку содержание меди в бедной руде составляет 6%,
то в x тоннах руды будет содержáться 0,06x тонн меди. В
богатой руде содержание меди составляет 11%, а в
20 − x тоннах богатой руды будет содержáться 0,11(20 − x)
тонн меди.

В получившихся 20 тоннах руды содержание меди должно


составлять 8%. Значит в 20 тоннах руды меди будет
содержáться 20 × 0,08 = 1,6 тонн.

Сложим выражения 0,06x и 0,11(20 − x) и приравняем эту


сумму к 1,6. Получим уравнение  0,06x + 0,11(20 − x) = 1,6
1177

Решим данное уравнение:

Значит для получения 20 тонн руды с содержанием меди 8%,


нужно взять 12 тонн бедной руды. Богатой же будет взято
20 − 12 = 8 тонн.

Задача 11. Увеличив среднюю скорость с 250 до 300 м/мин


спортсменка стала пробегать дистанцию на 1 мин быстрее.
Какова длина дистанции?

Решение

Длину дистанции (или расстояние дистанции) можно описать


следующим буквенным уравнением:

Воспользуемся правой частью этого уравнения для


составления своего уравнения. Изначально спортсменка
пробегала дистанцию со скоростью 250 метров в минуту. При
такой скорости длина дистанции будет описываться
выражением 250t
1178

Затем спортсменка увеличила свою скорость до 300 метров в


минуту. При такой скорости длина дистанции будет
описываться выражением 300t

Заметим, что длина дистанции это величина постоянная. От


того, что спортсменка увеличит скорость или уменьшит её,
длина дистанции останется неизменной.

Это позволяет нам приравнять выражение 250t к


выражению 300t, поскольку оба выражения описывают длину
одной и той же дистанции

250t = 300t

Но в задаче сказано, что при скорости 300 метров в минуту


спортсменка стала пробегать дистанцию на 1 минуту
быстрее. Другими словами, при скорости 300 метров в
минуту, общее время движения уменьшится на единицу.
Поэтому в уравнении 250t = 300t в правой части время нужно
уменьшить на единицу:

Получилось простейшее уравнение. Решим его:


1179

При скорости 250 метров в минуту спортсменка пробегает


дистанцию за 6 минут. Зная скорость и время, можно
определить длину дистанции:

S = 250 × 6 = 1500 м

А при скорости 300 метров в минуту спортсменка пробегает


дистанцию за t − 1, то есть за 5 минут. Как было сказано
ранее длина дистанции не меняется:

S = 300 × 5 = 1500 м

Задача 12. Всадник догоняет пешехода, находящегося


впереди него на 15 км. Через сколько часов всадник догонит
пешехода, если каждый час первый проезжает по 10 км, а
второй проходит только по 4 км?

Решение

Данная задача является задачей на движение. Её можно


решить, определив скорость сближения и разделив
изначальное расстояние между всадником и пешеходом на
эту скорость.

Скорость сближения определяется вычитанием меньшей


скорости из большей:

10 км/ч − 4 км/ч = 6 км/ч (скорость сближения)

С каждым часом расстояние в 15 километров будут


сокращаться на 6 км. Чтобы узнать, когда оно сократится
полностью (когда всадник догонит пешехода), нужно 15
разделить на 6
1180

15 : 6 = 2,5 ч

2,5 ч это два целых часа и половина часа. А половина часа


это 30 минут. Значит всадник догонит пешехода через 2 часа
30 минут.

Решим эту задачу с помощью уравнения.

Будем считать, что пешеход и всадник вышли в путь из


одного и того же места. Пешеход вышел раньше всадника и
успел преодолеть 15 км

После этого вслед за ним в путь вышел всадник со скоростью


10 км/ч. А скорость пешехода составляет только 4 км/ч. Это
значит, что всадник через некоторое время догонит
пешехода. Это время нам нужно найти.

Когда всадник догонит пешехода это будет означать, что они


вместе прошли одинаковое расстояние. Расстояние,
1181

пройденное всадником и пешеходом описывается


следующим уравнением:

Воспользуемся правой частью этого уравнения для


составления своего уравнения.

Расстояние, пройденное всадником, будет описываться


выражением 10t. Поскольку пешеход вышел в путь раньше
всадника и успел преодолеть 15 км, то расстояние
пройденное им будет описываться выражением 4t + 15.

На момент, когда всадник догонит пешехода, оба они


пройдут одинаковое расстояние. Это позволяет нам
приравнять расстояния, пройденные всадником и
пешеходом:

Получилось простейшее уравнение. Решим его:

Задачи для самостоятельного решения


Задача 1. Из одного города в другой пассажирский поезд
приезжает на 45 мин быстрее товарного. Вычисли
1182

расстояние между городами, если скорость пассажирского


поезда 48 км/ч, а товарного 36 км/ч.
Показать решение
Задача 2. Из двух городов, расстояние между которыми 150
км, одновременно навстречу друг другу выехали два
автомобиля. Скорость одного автомобиля 65 км/ч, а второго
60 км/ч. Через сколько часов они встретились?
Показать решение
Задача 3. В трех цехах завода всего 685 рабочих. Во втором
цехе рабочих в три раза больше, чем в первом, а в третьем
— на 15 рабочих меньше, чем во втором цехе. Сколько
рабочих в каждом цехе?
Показать решение
Задача 4. Две ремонтные мастерские в течение недели
должны отремонтировать по плану 18 моторов. Первая
мастерская выполнила план на 120%, а вторая — на 125%,
поэтому в течение недели отремонтировали 22 мотора.
Какой план по ремонту моторов на неделю имела каждая
мастерская?
Показать решение
Задача 5. Цена товара повысилась на 30% и составляет
теперь 91 руб. Сколько стоил товар до повышения цены?
Показать решение
Задача 6. Число увеличили на 25%. На сколько процентов
надо уменьшить новое число, чтобы получилось исходное?
Показать решение
1183

Задача 7. При увеличении числа на 20% получилось 144.


Найти первоначальное значение числа.
Показать решение
Задача 8. При уменьшении числа на 10% получилось 45.
Найти первоначальное значение числа.
Показать решение
Задача 9. Цена альбома была снижена сначала на 15%,
потом еще на 15 руб. Новая цена альбома после двух
снижений 19 руб. Определить его первоначальную цену.
Показать решение
Задача 10. Трава при сушке теряет 80% своей массы.
Сколько тонн травы нужно накосить, чтобы получить 4 т
сена?
Показать решение
Задача 11. Сколько килограммов 20%-го раствора соли
нужно добавить к 1 кг 10%-го раствора, чтобы получить 12%-
й раствор соли?
Показать решение
Задача 12. Даны два раствора соли в воде, концентрации
которых равны 20% и 30%. Сколько килограммов каждого
раствора нужно смешать в одном сосуде, чтобы получить 25
кг 25,2%-го раствора?
Показать решение
1184

Решение задач с помощью пропорции

Решение задачи с помощью пропорции сводится к тому,


чтобы сделать неизвестное значение x членом этой
пропорции. Затем используя основное свойство пропорции
получить линейное уравнение и решить его.

Предварительные навыки

 Соотношения
 Пропорция
 Прямая и обратная пропорциональность

Содержание урока

 Как решить задачу с помощью пропорции


 Отношение одноименных величин
 Задачи на прямую пропорциональность
 Задачи на обратную пропорциональность
 Масштаб
 Задачи для самостоятельного решения

Как решить задачу с помощью пропорции

Рассмотрим простейший пример. Трем группам нужно


выплатить стипендию по 1600 рублей каждому. В первой
группе 20 студентов. Значит первой группе будет выплачено
1600 × 20, то есть 32 тыс. рублей.

Во второй группе 17 человек. Значит второй группе будет


выплачено 1600 × 17, то есть 27,200 тыс. руб.
1185

Ну и выплатим стипендию третьей группе. В ней 15 человек.


На них нужно затратить 1600 × 15, то есть 24 тыс. руб.

В результате имеем следующее решение:

Для подобных задач решение можно записывать с помощью


пропорции.

Пропорция по определению есть равенство двух отношений.

К примеру, равенство является пропорцией. Эту


пропорцию можно прочесть следующим образом:

a так относится к b, как c относится d

Аналогично можно соотнести стипендию и студентов, так


чтобы каждому досталось по 1600 рублей.

Итак, запишем первое отношение, а именно отношение


тысячи шестисот рублей на одного человека:

Мы выяснили, что для выплаты 20 студентам по 1600


рублей, нам потребуется 32 тыс. рублей. Значит второе
отношение будет отношением тридцати двух тысяч к
двадцати студентам:

Теперь соединим полученные отношения знаком равенства:


1186

Мы получили пропорцию. Её можно прочесть следующим


образом:

Тысяча шестьсот рублей так относятся к одному


студенту, как тридцать две тысячи рублей
относятся к двадцати студентам.

То есть по 1600 рублей каждому. Если выполнить деление в

обеих частях равенства , то обнаружим, что


одному студенту, как и двадцати студентам достанется по
1600 рублей.

Теперь представим, что сумма денег, необходимых для


выплаты стипендии двадцати студентам, была бы
неизвестной. Скажем, если бы вопрос стоял так: в группе 20
студентов и каждому нужно выплатить по 1600 рублей.
Сколько всего рублей требуется для выплаты стипендии?

В таком случае пропорция   приняла бы

вид  . То есть сумма денег, необходимая для выплаты


стипендии, стала неизвестным членом пропорции. Эту
пропорцию можно прочесть так:

Тысяча шестьсот рублей так относятся к одному


студенту, как неизвестное число рублей относится к
двадцати студентам
1187

Теперь воспользуемся основным свойством пропорции. Оно


гласит, что произведение крайних членов пропорции равно
произведению средних:

Перемножив члены пропорции «крест-накрест», получим


равенство 1600 × 20 = 1 × x. Вычислив обе части равенства,
получим 32000 = x или x = 32000. Иными словами, мы
найдём значение неизвестной величины, которое искали.

Аналогично можно было определить общую сумму и для


остального количества студентов — для 17 и 15. Эти

пропорции выглядели как    и   .


Воспользовавшись основным свойством пропорции, можно
найти значение x

Задача 2. Расстояние равное 100 км автобус проехал за 2


часа. Сколько времени потребуется автобусу, чтобы
проехать 300 км, если будет ехать с той же скоростью?

Можно сначала определить расстояние, которое автобус


проезжает за один час. Затем определить сколько раз это
расстояние содержится в 300 километрах:
1188

100 : 2 = 50 км на каждый час движения

300 км : 50 = 6 часов

Либо можно составить пропорцию «сто километров так


относятся к двум часам, как триста километров к
неизвестному числу часов»:

Отношение одноименных величин

Если крайние или средние члены пропорции поменять


местами, то пропорция не нарушится.

Так, в пропорции   можно поменять местами

крайние члены. Тогда получится пропорция  .

Пропорция также не нарушится, если её перевернуть, то есть


использовать обратные отношения в обеих частях.

Перевернем пропорцию  . Тогда получим

пропорцию  . Взаимосвязь при этом не нарушается.


Отношение между студентами равно отношению между
суммами денег, предназначенных для этих студентов. Такую
1189

пропорцию часто составляют в школе, когда для решения


задачи составляются таблицы

Этот способ записи очень удобен, поскольку позволяет


перевести условие задачи в более понятный вид. Решим
задачу в которой требовалось определить сколько рублей
нужно для выплаты стипендии двадцати студентам.

Условие задачи запишем следующим образом:

Составим таблицу на основе этого условия:

Составим пропорцию, используя данные таблицы:

Используя основное свойство пропорции, получим линейное


уравнение и найдем его корень:
1190

Изначально, мы имели дело с пропорцией  ,


которая составлена из отношений величин разной природы.
В числителях отношений располагались суммы денег, а в
знаменателях количество студентов:

Поменяв местами крайние члены, мы получили

пропорцию  . Эта пропорция составлена из


отношений величин одной природы. В первом отношении
содержатся количества студентов, а во втором — суммы
денег:

Если отношение составлено из величин одной природы, то


мы будем называть его отношением одноименных
величин. Например, отношения между фруктами, деньгами,
физическими величинами, явлениями, действиями.

Отношение может быть составлено, как из одноименных


величин, так и из величин разной природы. Примерами
последних являются отношение расстояния ко времени,
1191

отношения стоимости товара к его количеству, отношение


общей суммы стипендии к количеству студентов.

Пример 2. В школьном саду посажены сосны и березы,


причём на каждую сосну приходится 2 березы. Сколько
посадили сосен в саду, если берез посадили 240?

Определим сколько сосен было посажено в саду. Для этого


составим пропорцию. В условии сказано, что на каждую
сосну приходится 2 березы. Напишем отношение,
показывающее что на одну сосну приходится две березы:

Теперь напишем второе отношение, показывающее что на x


сосен приходится 240 берез

Соединим эти отношения знаком равенства, получим


следующую пропорцию:

«2 березы так относятся к одной сосне,


как 240 берез относятся к x соснам»
1192

Используя основное свойство пропорции, находим значение


x

Либо пропорцию можно составить, предварительно записав


условие, как в прошлом примере:

Получится та же пропорция, но в этот раз она будет


составлена из отношений одноименных величин:

Значит в саду посадили 120 сосен.

Пример 3. Из 225 кг руды получили 34,2 кг меди. Каково


процентное содержание меди в руде?

Можно разделить 34,2 на 225 и полученный результат


выразить в процентах:
1193

Либо составить пропорцию 225 килограммам руды так


приходятся на 100%, как 34,2 кг меди приходятся на
неизвестное число процентов:

Либо составить пропорцию в которой отношения составлены


из одноименных величин:

Задачи на прямую пропорциональность

Понимание отношений одноименных величин приводит к


пониманию решения задач на прямую и обратную
пропорциональность. Начнем с задач на прямую
пропорциональность.
1194

Для начала вспомним, что такое прямая


пропорциональность. Это взаимосвязь между двумя
величинами при которой увеличение одной из них влечет за
собой увеличение другой во столько же раз.

Если расстояние в 50 км автобус прошел за 1 час, то для


прохождения расстояния в 100 км (при той же скорости)
автобусу потребуется 2 часа. Во сколько раз увеличилось
расстояние, во столько же раз увеличилось время движения.
Как показать это с помощью пропорции?

Одно из предназначений отношения заключается в том,


чтобы показать во сколько раз первая величина больше
второй. А значит и мы c помощью пропорции можем
показать, что расстояние и время увеличились в два раза.
Для этого воспользуемся отношением одноименных величин.

Покажем, что расстояние увеличилось в два раза:

Аналогично покажем, что время увеличилось во столько же


раз

Соединим эти отношения знаком равенства, получим


пропорцию:

«100 километров так относятся к 50 километрам, как


2 часа относятся к 1 часу»
1195

Если выполнить деление в обеих частях равенства  , то


обнаружим что расстояние и время были увеличены в
одинаковое число раз.

2=2

Задача 2. За 3 ч на мельнице смололи 27 т пшеничной муки.


Сколько тонн пшеничной муки можно смолоть за 9 ч, если
темп работы не изменится?

Решение

Время работы мельницы и масса перемолотой муки — прямо


пропорциональные величины. При увеличении времени
работы в несколько раз, количество перемолотой муки
увеличится во столько же раз. Покажем это с помощью
пропорции.

В задаче дано 3 ч. Эти 3 ч увеличились до 9 ч. Запишем


отношение 9 ч к 3 ч. Это отношение будет показывать во
сколько раз увеличилось время работы мельницы:

Теперь запишем второе отношение. Это будет отношение x


тонн пшеничной муки к 27 тоннам. Данное отношение будет
показывать, что количество перемолотой муки увеличилось
во столько же раз, сколько и время работы мельницы
1196

Соединим эти отношения знаком равенства, получим

пропорцию  .

Воспользуемся основным свойством пропорции и найдем x

Значит за 9 ч можно смолоть 81 т пшеничной муки.

Вообще, если взять две прямо пропорциональные величины


и увеличить их в одинаковое число раз, то отношение нового
значения к старому значению первой величины будет равно
отношению нового значения к старому значению второй
величины.

Так и в предыдущей задаче старые значения были 3 ч и 27 т.


Эти значения были увеличены в одинаковое число раз (в три
раза). Новыми значениями стали 9 ч и 81 ч. Тогда отношение
нового значения времени работы мельницы к старому

значению     равно отношению нового значения массы

перемолотой муки к старому значению  


1197

Если выполнить деление в обеих частях равенства, то


обнаружим, что время работы мельницы и количество
смолотой муки увеличилось в одинаковое число раз:

3=3

Пропорцию, которую составляют к задачам на прямую


пропорциональность, можно описать с помощью выражения:

 − новое значение первой величины


 − старое значение первой величины
Где
 − новое значение второй величины
 − старое значение второй величины

Применительно к нашей задаче значения переменных     


   будут следующими:

Где   впоследствии стало равно 81.

Задача 2. Для 8 коров в зимнее время доярка ежедневно


заготовляет 80 кг сена, 96 кг корнеплодов, 120 кг силоса и 12
кг концентратов. Определить ежедневный расход этих
кормов для 18 коров.
1198

Решение

Количество коров и масса каждого из кормов — прямо


пропорциональные величины. При увеличении количества
коров в несколько раз, масса каждого из кормов увеличится
во столько же раз.

Составим несколько пропорций, вычисляющих массу каждого


из кормов для 18 коров.

Начнем с сена. Ежедневно для 8 коров его заготовляют 80 кг.


Тогда для 18 коров будет заготовлено x кг сена.

Запишем отношение, показывающее во сколько раз


увеличилось количество коров:

Теперь запишем отношение, показывающее во сколько раз


увеличилась масса сена:

Соединим эти отношения знаком равенства, получим


пропорцию:

Отсюда находим x
1199

Значит для 18 коров нужно заготовить 180 кг сена.


Аналогично определяем массу корнеплодов, силоса и
концентратов.

Для 8 коров ежедневно заготовляют 96 кг корнеплодов. Тогда


для 18 коров будет заготовлено x кг корнеплодов. Составим

пропорцию из отношений     и    , затем вычислим значение


x

Определим сколько силоса и концентратов нужно заготовить


для 18 коров:
1200

Значит для 18 коров ежедневно нужно


заготавливать 180 кг сена, 216 кг корнеплодов, 270 кг силоса
и 27 кг концентратов.

Задача 3. Хозяйка варит вишнёвое варенье, причём на 3


стакана вишни кладёт 2 стакана сахара. Сколько сахара
нужно положить на 12 стаканов вишни? на 10 стаканов

вишни? на   стакана вишни?

Решение

Количество стаканов вишни и количество стаканов сахарного


песка — прямо пропорциональные величины. При
увеличении количества стаканов вишни в несколько раз,
количество стаканов сахара увеличится во столько же раз.

Запишем отношение, показывающее во сколько раз


увеличилось количество стаканов вишни:

Теперь запишем отношение, показывающее во сколько раз


увеличилось количество стаканов сахара:

Соединим эти отношения знаком равенства, получим


пропорцию и найдем значение x
1201

Значит на 12 стаканов вишни нужно положить 8 стаканов


сахара.

Определим количество стаканов сахара для 10

стаканов вишни и   стакана вишни

Задачи на обратную пропорциональность

Для решения задач на обратную пропорциональность опять


же можно использовать пропорцию, составленную из
отношений одноименных величин.

В отличие от прямой пропорциональности, где величины


увеличиваются или уменьшаются в одну и ту же сторону, в
обратной пропорциональности величины изменяются
обратно друг другу.
1202

Если одна величина увеличивается в несколько раз, то


другая уменьшается во столько же раз. И наоборот, если
одна величина уменьшается в несколько раз, то другая
увеличивается во столько же раз.

Допустим, что нужно покрасить забор, состоящий из 8 листов

Один маляр будет красить все 8 листов сам

Если маляров будет 2, то каждый покрасит по 4 листа.


1203

Это конечно же при условии, что маляры будут честными


между собой и справедливо разделят эту работу поровну на
двоих.

Если маляров будет 4, то каждый покрасит по 2 листа

Замечаем, что при увеличении количества маляров в


несколько раз, количество листов которые приходятся на
одного маляра уменьшаются во столько же раз.

Итак, мы увеличили количество маляров с 1 до 4. Другими


словами, увеличили количество маляров в четыре раза.
Запишем это с помощью отношения:

В результате количество листов забора, которые приходятся


на одного маляра уменьшилось в четыре раза. Запишем это
с помощью отношения:

Соединим эти отношения знаком равенства,


получим пропорцию
1204

«4 маляра так относятся к 1 маляру, как 8 листов


относятся к 2 листам»

Задача 2. 15 рабочих закончили отделку квартир в новом


доме за 24 дня. За сколько дней выполнили бы эту работу 18
рабочих?

Решение

Количество рабочих и количество дней, затраченных на


работу — обратно пропорциональные величины. При
увеличении количества рабочих в несколько раз, количество
дней, необходимых для выполнения этой работы,
уменьшится во столько же раз.

Запишем отношение 18 рабочих к 15 рабочим. Это


отношение будет показывать во сколько раз увеличилось
количество рабочих

Теперь запишем второе отношение, показывающее во


сколько раз уменьшилось количество дней. Поскольку
количество дней уменьшится с 24 дней до x дней, то второе
отношение будет отношением старого количества дней (24
дня) к новому количеству дней (x дней)
1205

Соединим полученные отношения знаком равенства,


получим пропорцию:

Отсюда находим x

Значит 18 рабочих выполнят необходимую работу за 20


дней.

Вообще, если взять две обратно пропорциональные


величины и увеличить одну из них в определенное число раз,
то другая уменьшится во столько же раз. Тогда отношение
нового значения к старому значению первой величины будет
равно отношению старого значения к новому значению
второй величины.

Так и в предыдущей задаче старые значения были 15


рабочих и 24 дня. Количество рабочих было увеличено с 15

до 18 (т.е. было увеличено в   раза). В результате


количество дней, необходимых для выполнения работы,
уменьшилось во столько же раз. Новыми значениями стали
18 рабочих и 20 дней. Тогда отношение нового количества
1206

рабочих к старому количеству    равно отношению старого

количества дней к новому количеству

Для составления пропорции к задачам на обратную


пропорциональность можно пользоваться формулой:

 − новое значение первой величины


 − старое значение первой величины
Где
 − старое значение второй величины
− новое значение второй величины

Применительно к нашей задаче значения переменных    


   будут следующими:

Где   впоследствии стало равно 20.

Задача 2. Скорость парохода относится к скорости течения


реки, как 36 : 5. Пароход двигался вниз по течению 5 ч 10
мин. Сколько времени потребуется ему, чтобы вернуться
обратно?
1207

Решение

Собственная скорость парохода составляет 36 км/ч.


Скорость течения реки реки 5 км/ч. Поскольку пароход
двигался по течению руки, то скорость его движения
составила 36 + 5 = 41 км/ч. Время пути составила 5 ч 10 мин.
Для удобства выразим время в минутах:

5 ч 10 мин = 300 мин + 10 мин = 310 мин

Поскольку на обратном пути пароход двигался против


течения реки, то его скорость составила 36 − 5 = 31 км/ч.

Скорость парохода и время его движения — обратно


пропорциональные величины. При уменьшении скорости в
несколько раз, время его движения увеличится во столько же
раз.

Запишем отношение, показывающее во сколько раз


уменьшилась скорость движения:

Теперь запишем второе отношение, показывающее во


сколько раз увеличилось время движения. Поскольку новое
время x будет больше старого времени, в числителе
отношения запишем время x, а в знаменателе старое время,
равное трёхсот десяти минутам

Соединим полученные отношения знаком равенства,

получим пропорцию  . Отсюда найдём значение x


1208

410 минут это 6 часов и 50 минут. Значит пароходу


потребуется 6 часов и 50 минут, чтобы вернуться обратно.

Задача 3. На ремонте дороги работало 15 человек, и они


должны были закончить работу за 12 дней. На пятый день
утром подошли еще несколько рабочих, и оставшаяся работа
была выполнена за 6 дней. Сколько рабочих прибыло
дополнительно?

Решение

Вычтем из 12 дней 4 отработанных дня. Так мы определим


сколько ещё дней осталось работать пятнадцати рабочим

12 дней − 4 дня = 8 дней

На пятый день дополнительно прибыло x рабочих. Тогда


всего рабочих стало 15 + x.

Количество рабочих и количество дней, необходимых для


выполнения работы — обратно пропорциональные
величины. При увеличении количества рабочих в несколько
раз, количество дней уменьшится во столько же раз.

Запишем отношение, показывающее во сколько раз


увеличилось количество рабочих:
1209

Теперь запишем во сколько раз уменьшилось количество


дней, необходимых для выполнения работы:

Соединим эти отношения знаком равенства, получим

пропорцию  . Отсюда можно вычислить значение x

Значит 5 рабочих прибыло дополнительно.

Масштаб

Масштабом называют отношение длины отрезка на


изображении к длине соответствующего отрезка на
местности.

Допустим, что расстояние от дома до школы составляет 8 км.


Попробуем нарисовать план местности, где будут указаны
дом, школа и расстояние между ними. Но изобразить на
бумаге расстояние, равное 8 км мы не можем, поскольку оно
1210

довольно велико. Но зато мы можем уменьшить это


расстояние в несколько раз так, чтобы оно уместилось на
бумаге.

Пусть километры на местности на нашем плане будут


выражаться в сантиметрах. Переведем 8 километров в
сантиметры, получим 800 000 сантиметров.

Уменьшим 800 000 см в сто тысяч раз:

800 000 см : 100 000 см = 8 см

8 см это расстояние от дома до школы, уменьшенное в сто


тысяч раз. Теперь без труда можно нарисовать на бумаге
дом и школу, расстояние между которыми будет 8 см.

Эти 8 см относятся к реальным 800 000 см. Так и запишем с


помощью отношения:

8 : 800 000
1211

Одно из свойств отношения гласит, что отношение не


меняется если его члены умножить или разделить на одно и
то же число.

В целях упрощения отношения 8 : 800 000 оба его члена


можно разделить на 8. Тогда получим отношение 1 : 100 000.
Это отношение и назовём масштабом. Данное отношение
показывает, что один сантиметр на плане относится (или
соответствует) ста тысячам сантиметров на местности.

Поэтому на нашем рисунке необходимо указать, что план


составлен в масштабе 1 : 100 000

Примеры:

1 см на плане относится к 100 000 см на местности;


2 см на плане относится к 200000 см на местности;
3 см на плане относится к 300000 на местности и т.д.
1212

К любой карте или плану указывается в каком масштабе они


сделаны. Этот масштаб позволяет определять реальное
расстояние между объектами.

Так, наш план составлен в масштабе 1 : 100 000. На этом


плане расстояние между домом и школой составляет 8 см.
Чтобы вычислить реальное расстояние между домом и
школой, нужно 8 см увеличить в 100 000 раз. Иными
словами, умножить 8 см на 100 000

8 см × 100 000 = 800 000 см

Получаем 800 000 см или 8 км, если перевести сантиметры в


километры.

Допустим, что между домом и школой располагается дерево.


На плане расстояние между школой и этим деревом
составляет 4 см.
1213

Тогда реальное расстояние между домом и деревом будет


4 см × 100 000 = 400 000 см или 4 км.

Расстояние на местности можно определять с помощью


пропорции. В нашем примере расстояние между домом и
школой будет вычисляться с помощью следующей
пропорции:

Эту пропорцию можно прочитать так:

1 см на плане так относится к 100000 см на


местности, как 8 см на плане относятся к x см на
местности.

Из этой пропорции узнаём, что значение x равно 800000 см.

Пример 2. На карте расстояние между двумя городами


составляет 8,5 см. Определить реальное расстояние между
городами, если карта составлена в масштабе 1 : 1 000 000.

Решение

Масштаб 1 : 1 000 000 указывает, что 1 см на карте


соответствует 1 000 000 см на местности. Тогда 8,5 см будут
соответствовать x см на местности. Составим пропорцию 1 к
1000000 как 8,5 к x
1214

В 1 км содержится 100000 см. Тогда в 8 500 000 см

будет 

Либо можно рассуждать так. Расстояние на карте и


расстояние на местности — прямо пропорциональные
величины. При увеличении расстояния на карте в несколько
раз, расстояние на местности увеличится во столько же раз.
Тогда пропорция примет следующий вид. Первое отношение
будет показывать во сколько раз расстояние на местности
больше расстояния на карте:

Второе отношение покажет, что расстояние на местности во


столько же раз больше, чем 8,5 см на карте:

Отсюда x равен 8 500 000 см или 85 км.

Задача 3. Длина реки Невы 74 км. Чему равняется ее длина


на карте, масштаб которой 1 : 2 000 000

Решение

Масштаб 1 : 2000000 говорит о том, что 1 см на карте


соответствует 2 000 000 см на местности.
1215

А 74 км на это 74 × 100 000 = 7 400 000 см на местности.


Уменьшив 7 400 000 в 2 000 000, мы определим длину реки
Невы на карте

7 400 000 : 2 000 000 = 3,7 см

Значит на карте, масштаб которой 1 : 2 000 000 длина реки


Невы составляет 3,7 см.

Запишем решение с помощью пропорции. Первое отношение


будет показывать сколько раз длина на карте меньше длины
на местности:

Второе отношение будет показывать, что 74 км (7 400 000


см) уменьшились во столько же раз:

Отсюда находим x равный 3,7 см

Задачи для самостоятельного решения


Задача 1. Из 21 кг хлопкового семени получили 5,1 кг масла.
Сколько масла получится из 7 кг хлопкового семени?
Показать решение
1216

Задача 2. На некотором участке железнодорожного пути


старые рельсы длиной в 8 м заменили новыми длиной в 12
м. Сколько потребуется новых двенадцатиметровых рельсов,
если сняли 360 старых рельсов?
Показать решение
Задача 3. 60% учеников класса пошли в кино, а остальные 12
человек – на выставку. Сколько учащихся в классе?
Показать решение
Задача 4. Расстояние на карте между городами 18 см. Какое
действительное расстояние между городами, если масштаб
карты 1 : 500 000?
Показать решение
Задача 5. Пешеход затратил на путь 2,5 ч, двигаясь со
скоростью 3,6 км/ч. Сколько времени затратит пешеход на
тот же путь, если его скорость будет 4,5 км/ч
Показать решение
Задача 6. Перевыполнив план на 15%, завод выпустил за
месяц 230 станков. Сколько станков должен был выпустить
за месяц завод по плану?
Показать решение
1217

Системы линейных уравнений

Предварительные навыки

 Общие сведения об уравнениях


 Решение задач с помощью уравнений

Содержание урока

 Линейные уравнения с двумя переменными


 Система двух линейных уравнений с двумя
переменными
 Метод подстановки
 Метод сложения
 Система линейных уравнений с тремя переменными
 Задачи на составление систем линейных уравнений
 Задания для самостоятельного решения

Линейные уравнения с двумя переменными

У школьника имеется 200 рублей, чтобы пообедать в школе.


Пирожное стоит 25 рублей, а чашка кофе 10 рублей. Сколько
пирожных и чашек кофе можно накупить на 200 рублей?

Обозначим количество пирожных через x, а количество


чашек кофе через y. Тогда стоимость пирожных будет
обозначаться через выражение 25x, а стоимость чашек кофе
через 10y.

25x — стоимость x пирожных


10y — стоимость y чашек кофе
1218

Итоговая сумма должна равняться 200 рублей. Тогда


получится уравнение с двумя переменными x и y

25x + 10y = 200

Сколько корней имеет данное уравнение?

Всё зависит от аппетита школьника. Если он купит 6


пирожных и 5 чашек кофе, то корнями уравнения будут числа
6 и 5.

Говорят, что пара значений 6 и 5 являются корнями


уравнения 25x + 10y = 200. Записывается как (6; 5), при этом
первое число является значением переменной x, а второе —
значением переменной y.

6 и 5 не единственные корни, которые обращают уравнение


25x + 10y = 200 в тождество. При желании на те же 200
рублей школьник может купить 4 пирожных и 10 чашек кофе:

В этом случае корнями уравнения 25x + 10y = 200 является


пара значений (4; 10).

Более того, школьник может вообще не покупать кофе, а


купить пирожные на все 200 рублей. Тогда корнями
уравнения 25x + 10y = 200 будут значения 8 и 0
1219

Или наоборот, не покупать пирожные, а купить кофе на все


200 рублей. Тогда корнями уравнения 25x + 10y = 200 будут
значения 0 и 20

Попробуем перечислить все возможные корни уравнения


25x + 10y = 200. Условимся, что значения x и y принадлежат
множеству целых чисел. И пусть эти значения будут
бóльшими или равными нулю:

x ∈ Z, y ∈ Z;
x ≥ 0, y ≥ 0

Так будет удобно и самому школьнику. Пирожные удобнее


покупать целыми, чем к примеру несколько целых пирожных
и половину пирожного. Кофе также удобнее брать целыми
чашками, чем к примеру несколько целых чашек и половину
чашки.

Заметим, что при нечетном x невозможно достичь равенства


ни при каком y. Тогда значениями x будут следующие числа
0, 2, 4, 6, 8. А зная x можно без труда определить y
1220

Таким образом, мы получили следующие пары значений


(0; 20), (2; 15), (4; 10), (6; 5), (8; 0). Эти пары являются
решениями или корнями уравнения 25x + 10y = 200. Они
обращают данное уравнение в тождество.

Уравнение вида ax + by = c называют линейным


уравнением с двумя переменными. Решением или
1221

корнями этого уравнения называют пару значений (x; y),


которая обращает его в тождество.

Отметим также, что если линейное уравнение с двумя


переменными записано в виде ax + by = c, то говорят, что
оно записано в каноническом (нормальном) виде.

Некоторые линейные уравнения с двумя переменными могут


быть приведены к каноническому виду.

Например, уравнение 2(16x + 3y − 4) = 2(12 + 8x − y) можно


привести к виду ax + by = c. Раскроем скобки в обеих частях
этого уравнения, получим 32x + 6y − 8 = 24 + 16x − 2y.
Слагаемые, содержащие неизвестные сгруппируем в левой
части уравнения, а слагаемые свободные от неизвестных —
в правой. Тогда получим 32x − 16x + 6y + 2y = 24 + 8.
Приведём подобные слагаемые в обеих частях, получим
уравнение 16x + 8y = 32. Это уравнение приведено к
виду ax + by = c и является каноническим.

Рассмотренное ранее уравнение 25x + 10y = 200 также


является линейным уравнением с двумя переменными в
каноническом виде. В этом уравнении параметры a, b и c
равны значениям 25, 10 и 200 соответственно.

На самом деле уравнение ax + by = c имеет бесчисленное


множество решений. Решая уравнение 25x + 10y = 200, мы
искали его корни только на множестве целых чисел. В
результате получили несколько пар значений, которые
обращали данное уравнение в тождество. Но на множестве
рациональных чисел уравнение 25x + 10y = 200 будет иметь
бесчисленное множество решений.
1222

Для получения новых пар значений, нужно взять


произвольное значение для x, затем выразить y. К примеру,
возьмем для переменной x значение 7. Тогда получим
уравнение с одной переменной 25 × 7 + 10y = 200 в котором
можно выразить y

Пусть x = 15. Тогда уравнение 25x + 10y = 200 примет


вид 25 × 15 + 10y = 200. Отсюда находим, что y = −17,5

Пусть x = −3. Тогда уравнение 25x + 10y = 200 примет


вид 25 × (−3) + 10y = 200. Отсюда находим, что y = −27,5

Система двух линейных уравнений с двумя переменными

Для уравнения ax + by = c можно сколько угодно раз брать


произвольные значение для x и находить значения для y.
Отдельно взятое такое уравнение будет иметь бесчисленное
множество решений.
1223

Но бывает и так, что переменные x и y связаны не одним, а


двумя уравнениями. В этом случае они образуют так
называемую систему линейных уравнений с двумя
переменными. Такая система уравнений может иметь одну
пару значений (или по-другому: «одно решение»).

Может случиться и так, что система вовсе не имеет решений.


Бесчисленное множество решений система линейных
уравнений может иметь в редких и в исключительных
случаях.

Два линейных уравнения образуют систему тогда, когда


значения x и y входят в каждое из этих уравнений.

Вернемся к самому первому уравнению 25x + 10y = 200.


Одной из пар значений для этого уравнения была пара (6; 5).
Это случай, когда на 200 рублей можно можно было купить 6
пирожных и 5 чашек кофе.

Составим задачу так, чтобы пара (6; 5) стала единственным


решением для уравнения 25x + 10y = 200. Для этого
составим ещё одно уравнение, которое связывало бы те же x
пирожных и y чашечек кофе.

Поставим текст задачи следующим образом:

«Школьник купил на 200 рублей несколько пирожных и


несколько чашек кофе. Пирожное стоит 25 рублей, а чашка
кофе 10 рублей. Сколько пирожных и чашек кофе купил
школьник, если известно что количество пирожных на одну
единицу больше количества чашек кофе?»

Первое уравнение у нас уже есть. Это уравнение


25x + 10y = 200. Теперь составим уравнение к условию
1224

«количество пирожных на одну единицу больше количества


чашек кофе».

Количество пирожных это x, а количество чашек кофе это y.


Можно записать эту фразу с помощью уравнения x − y = 1.
Это уравнение будет означать, что разница между
пирожными и кофе составляет 1.

Либо второе уравнение можно записать как x = y + 1. Это


уравнение означает, что количество пирожных на единицу
больше, чем количество чашек кофе. Поэтому для получения
равенства, к количеству чашек кофе прибавлена единица.
Это легко можно понять, если воспользоваться моделью
весов, которые мы рассматривали при изучении простейших
задач:

Получили два уравнения: 25x + 10y = 200 и


x = y + 1. Поскольку значения x и y, а именно 6 и 5 входят в
каждое из этих уравнений, то вместе они образуют систему.
Запишем эту систему. Если уравнения образуют систему, то
они обрамляются знаком системы. Знак системы это
фигурная скобка:
1225

Давайте решим данную систему. Это позволит увидеть, как


мы придём к значениям 6 и 5. Существует много методов
решения таких систем. Рассмотрим наиболее популярные из
них.

Метод подстановки

Название этого метода говорит само за себя. Суть его


заключается в том, чтобы одно уравнение подставить в
другое, предварительно выразив одну из переменных.

В нашей системе ничего выражать не нужно. Во втором


уравнении x = y + 1 переменная x уже выражена. Эта
переменная равна выражению y + 1. Тогда можно подставить
это выражение в первое уравнение вместо переменной x

После подстановки выражения y + 1 в первое уравнение


вместо x, получим уравнение 25(y + 1) + 10y = 200. Это
линейное уравнение с одной переменной. Такое уравнение
решить довольно просто:
1226

Мы нашли значение переменной y. Теперь подставим это


значение в одно из уравнений и найдём значение x. Для
этого удобно использовать второе уравнение x = y + 1. В него
и подставим значение y

Значит пара (6; 5) является решением системы уравнений,


как мы и задумывали. Выполняем проверку и убеждаемся,
что пара (6; 5) удовлетворяет системе:

Пример 2. Решить методом подстановки следующую


систему уравнений:
1227

Подставим первое уравнение x = 2 + y во второе уравнение


3x − 2y = 9. В первом уравнении переменная x равна
выражению 2 + y. Это выражение и подставим во второе
уравнение вместо x

Теперь найдём значение x. Для этого подставим значение y в


первое уравнение x = 2 + y

Значит решением системы   является пара значение


(5; 3)

Пример 3. Решить методом подстановки следующую


систему уравнений:

Здесь в отличие от предыдущих примеров, одна из


переменных не выражена явно.

Чтобы подставить одно уравнение в другое, сначала


нужно выразить одну из переменных.

Выражать желательно ту переменную, которая имеет


коэффициент единицу. Коэффициент единицу имеет
1228

переменная x, которая содержится в первом уравнении


x + 2y = 11. Эту переменную и выразим.

После выражения переменной x, наша система примет


следующий вид:

Теперь подставим первое уравнение во второе и найдем


значение y

Подставим y в первое уравнение и найдём x

Значит решением системы является пара значений


(3; 4)

Конечно, выражать можно и переменную y. Корни от этого не


изменятся. Но если выразить y, получится не очень-то и
простое уравнение, на решение которого уйдет больше
времени. Выглядеть это будет следующим образом:
1229

Видим, что в данном примере выражать x намного удобнее,


чем выражать y.

Пример 4. Решить методом подстановки следующую


систему уравнений:

Выразим в первом уравнении x. Тогда система примет вид:

Подставим первое уравнение во второе и найдём y


1230

Подставим y в первое уравнение и найдём x. Можно


воспользоваться изначальным уравнением 7x + 9y = 8, либо

воспользоваться уравнением  , в котором выражена


переменная x. Этим уравнением и воспользуемся, поскольку
это удобно:

Значит решением системы   является пара


значений (5; −3)

Метод сложения

Метод сложения заключается в том, чтобы почленно сложить


уравнения, входящие в систему. Это сложение приводит к
тому, что образуется новое уравнение с одной переменной.
А решить такое уравнение довольно просто.

Решим следующую систему уравнений:


1231

Сложим левую часть первого уравнения с левой частью


второго уравнения. А правую часть первого уравнения с
правой частью второго уравнения. Получим следующее
равенство:

Приведем подобные слагаемые:

В результате получили простейшее уравнение 3x = 27


корень которого равен 9. Зная значение x можно найти
значение y. Подставим значение x во второе уравнение
x − y = 3. Получим 9 − y = 3. Отсюда y = 6.

Значит решением системы   является пара


значений (9; 6)

Пример 2. Решить следующую систему уравнений методом


сложения:

Сложим левую часть первого уравнения с левой частью


второго уравнения. А правую часть первого уравнения с
правой частью второго уравнения. В получившемся
равенстве приведем подобные слагаемые:
1232

В результате получили простейшее уравнение


5x = 20, корень которого равен 4. Зная значение x можно
найти значение y. Подставим значение x в первое уравнение
2x + y = 11. Получим 8 + y = 11. Отсюда y = 3.

Значит решением системы   является пара значений


(4;3)

Процесс сложения подробно не расписывают. Его нужно


выполнять в уме. При сложении оба уравнения должны быть
приведены к каноническому виду. То есть к виду ax + by = c.

Из рассмотренных примеров видно, что основная цель


сложения уравнений это избавление от одной из
переменных. Но не всегда удаётся сразу решить систему
уравнений методом сложения. Чаще всего систему
предварительно приводят к виду, при котором можно
сложить уравнения, входящие в эту систему.

Например, систему   можно сразу решить методом


сложения. При сложении обоих уравнений, слагаемые y и −y
исчезнут, поскольку их сумма равна нулю. В результате
образуется простейшее уравнение 11x = 22, корень которого
равен 2. Затем можно будет определить y равный 5.

А систему уравнений   методом сложения сразу


решить нельзя, поскольку это не приведёт к исчезновению
1233

одной из переменных. Сложение приведет к тому, что


образуется уравнение 8x + y = 28, имеющее бесчисленное
множество решений.

Если обе части уравнения умножить или разделить на одно и


то же число, не равное нулю, то получится уравнение
равносильное данному. Это правило справедливо и для
системы линейных уравнений с двумя переменными. Одно из
уравнений (или оба уравнения) можно умножить на какое-
нибудь число. В результате получится равносильная
система, корни которой будут совпадать с предыдущей.

Вернемся к самой первой системе  , которая


описывала сколько пирожных и чашек кофе купил школьник.
Решением этой системы являлась пара значений (6; 5).

Умножим оба уравнения, входящие в эту систему на какие-


нибудь числа. Скажем первое уравнение умножим на 2, а
второе на 3

В результате получили систему 


Решением этой системы по-прежнему является пара
значений (6; 5)
1234

Это значит, что уравнения входящие в систему можно


привести к виду, пригодному для применения метода
сложения.

Вернемся к системе  , которую мы не смогли


решить методом сложения.

Умножим первое уравнение на 6, а второе на −2

Тогда получим следующую систему:

Сложим уравнения, входящие в эту систему. Сложение


компонентов 12x и −12x даст в результате 0, сложение 18y и
4y даст 22y, а сложение 108 и −20 даст 88. Тогда получится
уравнение 22y = 88, отсюда y = 4.
1235

Если первое время тяжело складывать уравнения в уме, то


можно записывать как складывается левая часть первого
уравнения с левой частью второго уравнения, а правая часть
первого уравнения с правой частью второго уравнения:

Зная, что значение переменной y равно 4, можно найти


значение x. Подставим y в одно из уравнений, например в
первое уравнение 2x + 3y = 18. Тогда получим уравнение с
одной переменной 2x + 12 = 18. Перенесем 12 в правую
часть, изменив знак, получим 2x = 6, отсюда x = 3.

Пример 4. Решить следующую систему уравнений методом


сложения:

Умножим второе уравнение на −1. Тогда система примет


следующий вид:

Сложим оба уравнения. Сложение компонентов x и −x даст в


результате 0, сложение 5y и 3y даст 8y, а сложение 7 и 1
даст 8. В результате получится уравнение 8y = 8, корень
которого равен 1. Зная, что значение y равно 1, можно найти
значение x.
1236

Подставим y в первое уравнение, получим x + 5 = 7, отсюда


x = 2

Пример 5. Решить следующую систему уравнений методом


сложения:

Желательно, чтобы слагаемые содержащие одинаковые


переменные, располагались друг под другом. Поэтому во
втором уравнении слагаемые 5y и −2x поменяем местами. В
результате система примет вид:

Умножим второе уравнение на 3. Тогда система примет вид:

Теперь сложим оба уравнения. В результате сложения


получим уравнение 8y = 16, корень которого равен 2.

Подставим y в первое уравнение, получим 6x − 14 = 40.


Перенесем слагаемое −14 в правую часть, изменив знак,
получим 6x = 54. Отсюда x = 9.

Пример 6. Решить следующую систему уравнений методом


сложения:
1237

Избавимся от дробей. Умножим первое уравнение на 36, а


второе на 12

В получившейся системе    первое уравнение


можно умножить на −5, а второе на 8
1238

Сложим уравнения в получившейся системе. Тогда получим


простейшее уравнение −13y = −156. Отсюда y = 12.
Подставим y в первое уравнение и найдем x

Пример 7. Решить следующую систему уравнений методом


сложения:

Приведем оба уравнения к нормальному виду. Здесь удобно


применить правило пропорции в обоих уравнениях. Если в

первом уравнении правую часть представить как    , а правую

часть второго уравнения как  , то система примет вид:


1239

У нас получилась пропорция. Перемножим её крайние и


средние члены. Тогда система примет вид:

Первое уравнение умножим на −3, а во втором раскроем


скобки:

Теперь сложим оба уравнения. В результате сложения этих


уравнений, мы получим равенство, в обеих частях которого
будет ноль:

Получается, что система  имеет бесчисленное


множество решений.

Но мы не можем просто так взять с неба произвольные


значения для x и y. Мы можем указать одно из значений, а
другое определится в зависимости от значения, указанного
нами. Например, пусть x = 2. Подставим это значение в
систему:
1240

В результате решения одного из уравнений, определится


значение для y, которое будет удовлетворять обоим
уравнениям:

Получившаяся пара значений (2; −2) будет удовлетворять


системе:
1241

Найдём еще одну пару значений. Пусть x = 4. Подставим это


значение в систему:

На глаз можно определить, что значение y равно нулю. Тогда


получим пару значений (4; 0), которая удовлетворяет нашей
системе:

Пример 8. Решить следующую систему уравнений методом


сложения:

Умножим первое уравнение на 6, а второе на 12


1242

Перепишем то, что осталось:

Раскроем скобки в обоих уравнениях и приведём подобные


слагаемые:

Первое уравнение умножим на −1. Тогда система примет


вид:

Теперь сложим оба уравнения. В результате сложения


образуется уравнение 6b = 48, корень которого равен 8.
Подставим b в первое уравнение и найдём a
1243

Система линейных уравнений с тремя переменными

В линейное уравнение с тремя переменными входит три


переменные с коэффициентами, а также свободный член. В
каноническом виде его можно записать следующим образом:

ax + by + cz = d

Данное уравнение имеет бесчисленное множество решений.


Придавая двум переменным различные значения, можно
найти третье значение. Решением в этом случае является
тройка значений (x; y; z) которая обращает уравнение в
тождество.

Если переменные x, y, z связаны между собой тремя


уравнениями, то образуется система трех линейных
уравнений с тремя переменными. Для решения такой
системы можно применять те же методы, которые
применяются к линейным уравнениям с двумя переменными:
метод подстановки и метод сложения.

Пример 1. Решить следующую систему уравнений методом


подстановки:
1244

Выразим в третьем уравнении x. Тогда система примет вид:

Теперь выполним подстановку. Переменная x равна


выражению 3 − 2y − 2z. Подставим это выражение в первое и
второе уравнение:

Раскроем скобки в обоих уравнениях и приведём подобные


слагаемые:

Мы пришли к системе линейных уравнений с двумя


переменными. В данном случае удобно применить метод
сложения. В результате переменная y исчезнет, и мы
сможем найти значение переменной z
1245

Теперь найдём значение y. Для этого удобно


воспользоваться уравнением −y + z = 4. Подставим в него
значение z

Теперь найдём значение x. Для этого удобно


воспользоваться уравнением x = 3 − 2y − 2z. Подставим в
него значения y и z

Таким образом, тройка значений (3; −2; 2) является


решением нашей системы. Проверкой убеждаемся, что эти
значения удовлетворяют системе:
1246

Пример 2. Решить систему методом сложения

Сложим первое уравнение со вторым, умноженным на −2.

Если второе уравнение умножить на −2, то оно примет


вид −6x + 6y − 4z = −4. Теперь сложим его с первым
уравнением:
1247

Видим, что в результате элементарных преобразований,


определилось значение переменной x. Оно равно единице.

Вернемся к главной системе. Сложим второе уравнение с


третьим, умноженным на −1. Если третье уравнение
умножить на −1, то оно примет вид −4x + 5y − 2z = −1. Теперь
сложим его со вторым уравнением:

Получили уравнение x − 2y = −1. Подставим в него значение


x, которое мы находили ранее. Тогда мы сможем определить
значение y

Теперь нам известны значения x и y. Это позволяет


определить значение z. Воспользуемся одним из уравнений,
входящим в систему:

Таким образом, тройка значений (1; 1; 1) является решением


нашей системы. Проверкой убеждаемся, что эти значения
удовлетворяют системе:
1248

Задачи на составление систем линейных уравнений

Задача на составление систем уравнений решается путем


ввода нескольких переменных. Далее составляются
уравнения на основании условий задачи. Из составленных
уравнений образуют систему и решают её. Решив систему,
необходимо выполнить проверку на то, удовлетворяет ли её
решение условиям задачи.

Задача 1. Из города в колхоз выехала машина «Волга».


Обратно она возвращалась по другой дороге, которая была
на 5 км короче первой. Всего в оба конца машина проехала
35 км. Сколько километров составляет длина каждой дороги?

Решение

Пусть x — длина первой дороги, y — длина второй. Если в


оба конца машина проехала 35 км, то первое уравнение
можно записать как x + y = 35. Это уравнение описывает
сумму длин обеих дорог.
1249

Сказано, что обратно машина возвращалась по дороге


которая была короче первой на 5 км. Тогда второе уравнение
можно записать как x − y = 5. Это уравнение показывает, что
разница между длинами дорог составляет 5 км.

Либо второе уравнение можно записать как x = y + 5. Этим


уравнением и воспользуемся.

Поскольку переменные x и y в обоих уравнениях обозначают


одно и то же число, то мы можем образовать из них систему:

Решим эту систему каким-нибудь из изученных ранее


методов. В данном случае удобно воспользоваться методом
подстановки, поскольку во втором уравнении переменная x
уже выражена.

Подставим второе уравнение в первое и найдём y

Подставим найденное значение y в во второе уравнение


x = y + 5 и найдём x
1250

Длина первой дороги была обозначена через переменную x.


Теперь мы нашли её значение. Переменная x равна 20.
Значит длина первой дороги составляет 20 км.

А длина второй дороги была обозначена через y. Значение


этой переменной равно 15. Значит длина второй дороги
составляет 15 км.

Выполним проверку. Для начала убедимся, что система


решена правильно:

Теперь проверим удовлетворяет ли решение (20; 15)


условиям задачи.

Было сказано, что всего в оба конца машина проехала 35 км.


Складываем длины обеих дорог и убеждаемся, что
решение (20; 15) удовлетворяет данному условию:
20 км + 15 км = 35 км

Следующее условие: обратно машина возвращалась по


другой дороге, которая была на 5 км короче первой. Видим,
что решение (20; 15) удовлетворяет и этому условию,
поскольку 15 км короче, чем 20 км на 5 км:
20 км − 15 км = 5 км
1251

При составлении системы важно, чтобы переменные


обозначали одни и те же числа во всех уравнениях,
входящих в эту систему.

Так наша система   содержит два уравнения. Эти


уравнения в свою очередь содержат переменные x и y,
которые обозначают одни и те же числа в обоих уравнениях,
а именно длины дорог, равных 20 км и 15 км.

Задача 2. На платформу были погружены дубовые и


сосновые шпалы, всего 300 шпал. Известно, что все дубовые
шпалы весили на 1 т меньше, чем все сосновые.
Определить, сколько было дубовых и сосновых шпал
отдельно, если каждая дубовая шпала весила 46 кг, а каждая
сосновая 28 кг.

Решение

Пусть x дубовых и y сосновых шпал было погружено на


платформу. Если всего шпал было 300, то первое уравнение
можно записать как x + y = 300.

Все дубовые шпалы весили 46x кг, а сосновые весили 28y кг.
Поскольку дубовые шпалы весили на 1 т меньше, чем
сосновые, то второе уравнение можно записать, как
28y − 46x = 1000. Это уравнение показывает, что разница
масс между дубовыми и сосновыми шпалами, составляет
1000 кг.

Тонны были переведены в килограммы, поскольку масса


дубовых и сосновых шпал измерена в килограммах.
1252

В результате получаем два уравнения, которые образуют


систему

Решим данную систему. Выразим в первом уравнении x.


Тогда система примет вид:

Подставим первое уравнение во второе и найдём y

Подставим y в уравнение x = 300 − y и узнаем чему равно x

Значит на платформу было погружено 100 дубовых и 200


сосновых шпал.

Проверим удовлетворяет ли решение (100; 200) условиям


задачи. Для начала убедимся, что система решена
правильно:
1253

Было сказано, что всего было 300 шпал. Складываем


количество дубовых и сосновых шпал и убеждаемся, что
решение (100; 200) удовлетворяет данному условию:
100 + 200 = 300.

Следующее условие: все дубовые шпалы весили на 1 т


меньше, чем все сосновые. Видим, что решение (100; 200)
удовлетворяет и этому условию, поскольку 46 × 100 кг
дубовых шпал легче, чем 28 × 200 кг сосновых шпал:
5600 кг − 4600 кг = 1000 кг.

Задача 3. Взяли три куска сплава меди с никелем в


отношениях 2 : 1, 3 : 1 и 5 : 1 по массе. Из них сплавлен кусок
массой 12 кг с отношением содержания меди и никеля 4 : 1.
Найдите массу каждого исходного куска, если масса первого
из них вдвое больше массы второго.

Решение

Пусть x — масса первого куска, y — масса второго куска, z —


масса третьего куска. Если из этих кусков сплавлен кусок
1254

массой 12 кг, то первое уравнение можно записать как


x + y + z = 12.

Масса первого куска вдвое больше массы второго куска.


Тогда второе уравнение можно записать как x = 2y.

Полученных двух уравнений недостаточно для решения


данной задачи. Если второе уравнение подставить в первое,
то мы получим уравнение 2y + y + z = 12, откуда 3y + z = 12.
Это уравнение имеет бесчисленное множество решений.

Составим ещё одно уравнение. Пусть это уравнение будет


описывать количество меди, взятого с каждого сплава и
сколько меди оказалось в получившемся сплаве.

Если первый сплав имеет массу x, а медь и никель


находится нём в отношении 2 : 1, то можно записать, что в

новом сплаве содержится   меди от первого куска.

Если второй сплав имеет массу y, а медь и никель находится


в нём в отношении 3 : 1, то можно записать, что в новом

сплаве содержится  меди от второго куска.

Если третий сплав имеет массу z, а медь и никель находится


в отношении 5 : 1, то можно записать, что в новом сплаве

содержится  меди от третьего куска.

Полученный сплав имеет имеет массу 12 кг, а медь и никель


находится в нём в отношении 4 : 1. Тогда можно записать,

что в полученном сплаве содержится меди.


1255

Сложим   , , и приравняем эту сумму к 9,6. Это и будет


нашим третьим уравнением:

Попробуем решить данную систему.

Для начала упростим третье уравнение. Подставим в него


второе уравнение и посмотрим, что из этого выйдет:

Теперь в главной системе вместо уравнения   


запишем уравнение, которое мы сейчас получили, а именно
уравнение 25y + 10z = 115,2
1256

Подставим второе уравнение в первое:

Умножим первое уравнение на −10. Тогда система примет


вид:

Сложим оба уравнения. Тогда получим простейшее


уравнение −5y = −4,8 откуда найдём y равный 0,96. Значит
масса второго сплава составляет 0,96 кг.

Теперь найдём x. Для этого удобно воспользоваться


уравнением x = 2y. Значение y уже известно. Осталось
только подставить его:

Значит масса первого сплава составляет 1,92 кг.

Теперь найдём z. Для этого удобно воспользоваться


уравнением x + y + z = 12. Значения x и y уже известны.
Подставим их куда нужно:
1257

Значит масса третьего сплава составляет 9,12 кг.

Задания для самостоятельного решения


Задание 1. Приведите следующее уравнение к
каноническому (нормальному) виду:

Показать решение
Задание 2. Приведите следующее уравнение к
каноническому (нормальному) виду:

Показать решение
Задание 3. Приведите следующее уравнение к
каноническому (нормальному) виду:

Показать решение
Задание 4. Приведите следующее уравнение к
каноническому (нормальному) виду:

Показать решение
Задание 5. Решите следующую систему уравнений методом
подстановки:

Показать решение
1258

Задание 6. Решите следующую систему уравнений методом


подстановки:

Показать решение
Задание 7. Решите следующую систему уравнений методом
подстановки:

Показать решение
Задание 8. Решите следующую систему уравнений методом
подстановки:

Показать решение
Задание 9. Решите следующую систему уравнений методом
подстановки:

Показать решение
Задание 10. Решите следующую систему уравнений методом
подстановки:

Показать решение
1259

Задание 11. Решите следующую систему уравнений методом


подстановки:

Показать решение
Задание 12. Решите следующую систему уравнений методом
подстановки:

Показать решение
Задание 13. Решите следующую систему уравнений методом
подстановки:

Показать решение
Задание 14. Решите следующую систему уравнений методом
подстановки:

Показать решение
Задание 15. Решите следующую систему уравнений методом
сложения:

Показать решение
1260

Задание 16. Решите следующую систему уравнений методом


сложения:

Показать решение
Задание 17. Решите следующую систему уравнений методом
сложения:

Показать решение
Задание 18. Решите следующую систему уравнений методом
сложения:

Показать решение
Задание 19. Решите следующую систему уравнений методом
сложения:

Показать решение
Задание 20. Решите следующую систему уравнений методом
сложения:

Показать решение
1261

Задание 21. Решите следующую систему уравнений методом


сложения:

Показать решение
Задание 22. Решите следующую систему уравнений методом
сложения:

Показать решение
Задание 23. Решите следующую систему уравнений методом
подстановки:

Показать решение
Задача 24. На прокормление 8 лошадей и 15 коров отпускали
ежедневно 162 кг сéна. Сколько сéна ежедневно выдавали
каждой лошади и каждой корове, если известно, что 5
лошадей получали сéна на 3 кг больше, чем 7 коров?
Показать решение
Задача 25. Для отправки груза было подано несколько
вагонов. Если грузить по 15,5 т в вагон, то 4 т груза останутся
непогруженными; если же грузить по 16,5 т в вагон, то для
1262

полной загрузки вагонов не хватит 8 т груза. Сколько было


подано вагонов и сколько тонн было груза?
Показать решение
Задача 26. В школьном зале поставлены скамейки. Если на
каждую скамью посадить по 5 учеников, то не хватит 8
скамеек; если же на каждую скамью посадить по 6 учеников,
то 2 скамьи останутся свободными. Сколько скамеек было
поставлено в зале и сколько было учеников?
Показать решение
Задача 27. Несколько человек отправляются на экскурсию.
Если при этом каждый внесёт на расходы по 12 руб. 50 коп.,
то для оплаты расходов не хватит 100 руб.; если же каждый
внесёт по 16 руб., то останется излишек 12 руб. Сколько
человек участвует в экскурсии?
Показать решение
1263

Общие сведения о неравенствах

Данный материал может показаться сложным для


понимания. Рекомендуется изучать его маленькими частями.

Предварительные навыки

 Отрицательные числа
 Что такое множество?
 Общие сведения об уравнениях

Содержание урока

 Определение и свойства
 Строгие и нестрогие неравенства
 Двойное неравенство
 Неравенство с переменной
 Как решать неравенства
 Числовые промежутки
o Числовой луч
o Открытый числовой луч
o Отрезок
o Интервал
o Полуинтервал
 Изображение числовых промежутков на координатной
прямой
 Примеры решения неравенств
 Когда решений нет
 Когда решений бесконечно много
 Задания для самостоятельного решения
1264

Определения и свойства

Неравенством мы будем называть два числовых или


буквенных выражения, соединенных знаками >, <, ≥, ≤ или ≠.

Пример: 5 > 3

Данное неравенство говорит о том, что число 5 больше, чем


число 3. Острый угол знака неравенства должен быть
направлен в сторону меньшего числа. Это неравенство
является верным, поскольку 5 больше, чем 3.

Если на левую чашу весов положить арбуз массой 5 кг, а на


правую — арбуз массой 3 кг, то левая чаша перевесит
правую, и экран весов покажет, что левая чаша тяжелее
правой:

Если 5 > 3, то 3 < 5. То есть левую и правую часть


неравенства можно поменять местами, изменив знак
неравенства на противоположный. В ситуации с весами:
большой арбуз можно положить на правую чашу, а
1265

маленький арбуз на левую. Тогда правая чаша перевесит


левую, и экран покажет знак <

Если в неравенстве 5 > 3, не трогая левую и правую часть,


поменять знак на <, то получится неравенство 5 < 3. Это
неравенство не является верным, поскольку число 3 не
может быть больше числа 5.

Числа, которые располагаются в левой и правой части


неравенства, будем называть членами этого неравенства.
Например, в неравенстве 5 > 3 членами являются числа 5 и
3.

Рассмотрим некоторые важные свойства для


неравенства 5 > 3.
В будущем эти свойства будут работать и для других
неравенств.

Свойство 1.
1266

Если к левой и правой части неравенства 5 > 3


прибавить или вычесть одно и то же число, то знак
неравенства не изменится.

Например, прибавим к обеим частям неравенства число 4.


Тогда получим:

Видим, что левая часть по-прежнему больше правой.

Теперь попробуем вычесть из обеих частей


неравенства 5 > 3 какое-нибудь число, скажем число 2

Видим, что левая часть по-прежнему больше правой.

Из данного свойства следует, что любой член неравенства


можно перенести из одной части в другую часть, изменив
знак этого члена. Знак неравенства при этом не изменится.

Например, перенесём в неравенстве 5 > 3, член 5 из левой


части в правую часть, изменив знак этого члена. После
переноса члена 5 в правую часть, в левой части ничего не
останется, поэтому запишем там 0

0 > 3 − 5

0 > −2

Видим, что левая часть по-прежнему больше правой.


1267

Свойство 2.

Если обе части неравенства умножить или разделить на


одно и то же положительное число, то знак неравенства
не изменится.

Например, умножим обе части неравенства 5 > 3 на какое-


нибудь положительное число, скажем на число 2. Тогда
получим:

Видим, что левая часть по-прежнему больше правой.

Теперь попробуем разделить обе части неравенства 5 > 3


на какое-нибудь число. Разделим их на 2

Видим, что левая часть по-прежнему больше правой.

Свойство 3.

Если обе части неравенства умножить или разделить на


одно и то же отрицательное число, то знак неравенства
изменится на противоположный.

Например, умножим обе части неравенства 5 > 3 на какое-


нибудь отрицательное число, скажем на число −2. Тогда
получим:
1268

Видим, что левая часть стала меньше правой. То есть знак


неравенства изменился на противоположный.

Теперь попробуем разделить обе части неравенства 5 > 3


на какое-нибудь отрицательное число. Давайте разделим их
на −1

Видим, что левая часть стала меньше правой. То есть знак


неравенства изменился на противоположный.

Само по себе неравенство можно понимать, как некоторое


условие. Если условие выполняется, то неравенство
является верным. И наоборот, если условие не выполняется,
то неравенство не верно.

Например, чтобы ответить на вопрос является ли верным


неравенство 7 > 3, нужно проверить выполняется ли условие
«больше ли 7, чем 3». Мы знаем, что число 7 больше, чем
число 3. То есть условие выполнено, а значит и неравенство
7 > 3 верно.

Неравенство 8 < 6 не является верным, поскольку не


выполняется условие «8 меньше, чем 6».

Другим способом определения верности неравенства


является составление разности из левой и правой части
1269

данного неравенства. Если разность положительна, то левая


часть больше правой части. И наоборот, если разность
отрицательна, то левая часть меньше правой части. Более
точно это правило выглядит следующим образом:

Число a больше числа b, если разность a − b


положительна. Число a меньше числа b, если разность
a − b отрицательна.

Например, мы выяснили, что неравенство 7 > 3 является


верным, поскольку число 7 больше, чем число 3. Докажем
это с помощью правила, приведённого выше.

Составим разность из членов 7 и 3. Тогда получим 7 − 3 = 4.


Согласно правилу, число 7 будет больше числа 3, если
разность 7 − 3 окажется положительной. У нас она равна 4,
то есть разность положительна. А значит число 7 больше
числа 3.

Проверим с помощью разности верно ли неравенство


3 < 4. Составим разность, получим 3 − 4 = −1. Согласно
правилу, число 3 будет меньше числа 4, если разность 3 − 4
окажется отрицательной. У нас она равна −1, то есть
разность отрицательна. А значит число 3 меньше числа 4.

Проверим верно ли неравенство 5 > 8. Составим разность,


получим 5 − 8 = −3. Согласно правилу, число 5 будет больше
числа 8, если разность 5 − 8 окажется положительной. У нас
разность равна −3, то есть она не является положительной.
А значит число 5 не больше числа 8. Иными словами,
неравенство 5 > 8 не является верным.
1270

Строгие и нестрогие неравенства

Неравенства, содержащие знаки >, < называют строгими. А


неравенства, содержащие знаки ≥, ≤  называют нестрогими.

Примеры строгих неравенства мы рассматривали ранее.


Таковыми являются неравенства 5 > 3, 7 < 9.

Нестрогим, например, является неравенство 2 ≤ 5. Данное


неравенство читают следующим образом: «2 меньше или
равно 5».

Запись 2 ≤ 5 является неполной. Полная запись этого


неравенства выглядит следующим образом:

2 < 5 или 2 = 5

Тогда становится очевидным, что неравенство 2 ≤ 5 состоит


из двух условий: «два меньше пять» и «два равно пять».

Нестрогое неравенство верно в том случае, если


выполняется хотя бы одно из его условий. В нашем примере
верным является условие «2 меньше 5». Значит и само
неравенство 2 ≤ 5 верно.

Пример 2. Неравенство 2 ≤ 2 является верным, поскольку


выполняется одно из его условий, а именно 2 = 2.

Пример 3. Неравенство 5 ≤ 2 не является верным, поскольку


не выполняется ни одно из его условий: ни 5 < 2 ни 5 = 2.
1271

Двойное неравенство

Число 3 больше, чем число 2 и меньше, чем число 4. В виде


неравенства это высказывание можно записать так: 2 < 3 < 4.
Такое неравенство называют двойным.

Двойное неравенство может содержать знаки нестрогих


неравенств. К примеру, если число 5 больше или равно, чем
число 2, и меньше или равно, чем число 7, то можно
записать, что 2 ≤ 5 ≤ 7

Чтобы правильно записать двойное неравенство, сначала


записывают член находящийся в середине, затем член
находящийся слева, затем член находящийся справа.

Например, запишем, что число 6 больше, чем число 4, и


меньше, чем число 9.

Сначала записываем 6

Слева записываем, что это число больше, чем число 4

Справа записываем, что число 6 меньше, чем число 9

Неравенство с переменной

Неравенство, как и равенство может содержать переменную.

Например, неравенство x > 2 содержит переменную x.


Обычно такое неравенство нужно решить, то есть выяснить
1272

при каких значениях x данное неравенство становится


верным.

Решить неравенство означает найти такие значения


переменной x, при которых данное неравенство
становится верным.

Значение переменной, при котором неравенство становится


верным, называется решением неравенства.

Неравенство x > 2 становится верным при x = 3, x = 4, x = 5,


x = 6 и так далее до бесконечности. Видим, что это
неравенство имеет не одно решение, а множество решений.

Другими словами, решением неравенства x > 2 является


множество всех чисел, бóльших 2. При этих числах
неравенство будет верным. Примеры:

3 > 2

4 > 2

5 > 2

Число 2, располагающееся в правой части неравенства x > 2,


будем называть границей данного неравенства. В
зависимости от знака неравенства, граница может
принадлежать множеству решений неравенства либо не
принадлежать ему.

В нашем примере граница неравенства не принадлежит


множеству решений, поскольку при подстановке числа 2 в
неравенство x > 2 получается не верное неравенство 2 > 2.
Число 2 не может быть больше самого себя, поскольку оно
равно самому себе (2 = 2).
1273

Неравенство x > 2 является строгим. Его можно прочитать


так: «x строго больше 2″. То есть все значения,
принимаемые переменной x должны быть строго больше 2. В
противном случае, неравенство верным не будет.

Если бы нам было дано нестрогое неравенство x ≥ 2, то


решениями данного неравенства были бы все числа,
которые больше 2, в том числе и само число 2. В этом
неравенстве граница 2 принадлежит множеству решений
неравенства, поскольку при подстановке числа 2 в
неравенство x ≥ 2 получается верное неравенство 2 ≥ 2.
Ранее было сказано, что нестрогое неравенство является
верным, если выполняется хотя бы одно из его условий. В
неравенстве 2 ≥ 2 выполняется условие 2 = 2, поэтому и
само неравенство 2 ≥ 2 верно.

Как решать неравенства

Процесс решения неравенств во многом схож с процессом


решения уравнений. При решении неравенств мы будем
применять свойства, которые изучили вначале данного
урока, такие как: перенос слагаемых из одной части
неравенства в другую часть, меняя знак; умножение (или
деление) обеих частей неравенства на одно и то же число.

Эти свойства позволяют получить неравенство, которое


равносильно исходному. Равносильными называют
неравенства, решения которых совпадают.

Решая уравнения мы выполняли тождественные


преобразования до тех пор, пока в левой части уравнения не
оставалась переменная, а в правой части значение этой
1274

переменной (например: x = 2, x = 5). Иными словами,


заменяли исходное уравнение на равносильное ему
уравнение до тех пор, пока не получалось уравнение вида
x = a, где a значение переменной x. В зависимости от
уравнения, корней могло быть один, два, бесконечное
множество, либо не быть совсем.

А при решении неравенств мы будем заменять исходное


неравенство на равносильное ему неравенство до тех пор,
пока в левой части не останется переменная этого
неравенства, а в правой части его граница.

Пример 1. Решить неравенство 2x > 6

Итак, нужно найти такие значения x, при подстановке


которых в 2x > 6 получится верное неравенство.

Вначале данного урока было сказано, что если обе части


неравенства разделить на какое-нибудь положительное
число, то знак неравенства не изменится. Если применить
это свойство к неравенству, содержащему переменную, то
получится неравенство равносильное исходному.

В нашем случае, если мы разделим обе части неравенства


2x > 6 на какое-нибудь положительное число, то получится
неравенство, которое равносильно исходному неравенству
2x > 6. 

Итак, разделим обе части неравенства на 2.


1275

В левой части осталась переменная x, а правая часть стала


равна 3. Получилось равносильное неравенство x > 3. На
этом решение завершается, поскольку в левой части
осталась переменная, а в правой части граница неравенства.

Теперь можно сделать вывод, что решениями


неравенства x > 3 являются все числа, которые больше 3.
Это числа 4, 5, 6, 7 и так далее до бесконечности. При этих
значениях неравенство x > 3 будет верным.

4 > 3

5 > 3

6 > 3

7 > 3

Отметим, что неравенство x > 3 является строгим.


«Переменная x строго больше трёх».

А поскольку неравенство x > 3 равносильно исходному


неравенству 2x > 6, то их решения будут совпадать. Иначе
говоря, значения, которые подходят неравенству x > 3, будут
подходить и неравенству 2x > 6. Покажем это.

Возьмём, например, число 5 и подставим его сначала в


полученное нами равносильное неравенство x > 3, а потом в
исходное 2x > 6.
1276

Видим, что в обоих случаях получается верное неравенство.

После того, как неравенство решено, ответ нужно записать в


виде так называемого числового промежутка следующим
образом:

В этом выражении говорится, что значения, принимаемые


переменной x, принадлежат числовому промежутку от трёх
до плюс бесконечности.

Иначе говоря, все числа, начиная от трёх до плюс


бесконечности являются решениями неравенства x > 3.
Знак ∞ в математике означает бесконечность.

Учитывая, что понятие числового промежутка очень важно,


остановимся на нём подробнее.

Числовые промежутки

Числовым промежутком называют множество чисел на


координатной прямой, которое может быть описано с
помощью неравенства.
1277

Допустим, мы хотим изобразить на координатной прямой


множество чисел от 2 до 8. Для этого сначала на
координатной прямой отмечаем точки с координатами 2 и 8,
а затем выделяем штрихами ту область, которая
располагается между координатами 2 и 8. Эти штрихи будут
играть роль чисел, располагающихся между числами 2 и 8

Числа 2 и 8 назовём границами числового


промежутка. Рисуя числовой промежуток, точки для его
границ изображают не в виде точек как таковых, а в виде
кружков, которые можно разглядеть.

Границы могут принадлежать числовому промежутку либо не


принадлежать ему.

Если границы не принадлежат числовому промежутку, то


они изображаются на координатной прямой в виде пустых
кружков.

Если границы принадлежат числовому промежутку, то


кружки необходимо закрасить.

На нашем рисунке кружки были оставлены пустыми. Это


означало, что границы 2 и 8 не принадлежат числовому
промежутку. Значит в наш числовой промежуток будут
входить все числа от 2 до 8, кроме чисел 2 и 8.

Если мы хотим включить границы 2 и 8 в числовой


промежуток, то кружки необходимо закрасить:
1278

В данном случае в числовой промежуток будут входить все


числа от 2 до 8, включая числа 2 и 8.

На письме числовой промежуток обозначается указанием его


границ с помощью круглых или квадратных скобок.

Если границы не принадлежат числовому промежутку, то


границы обрамляются круглыми скобками.

Если границы принадлежат числовому промежутку, то


границы обрамляются квадратными скобками.

На рисунке представлено два числовых промежутка от 2 до 8


с соответствующими обозначениями:

На первом рисунке числовой промежуток обозначен с


помощью круглых скобок, поскольку границы 2 и 8 не
принадлежат этому числовому промежутку.

На втором рисунке числовой промежуток обозначен с


помощью квадратных скобок, поскольку границы 2 и
8 принадлежат этому числовому промежутку.

С помощью числовых промежутков можно записывать ответы


к неравенствам. Например, ответ к двойному
неравенству 2 ≤ x ≤ 8 записывается так:

x ∈ [ 2 ; 8 ]

То есть сначала записывают переменную, входящую в


неравенство, затем с помощью знака принадлежности ∈
1279

указывают к какому числовому промежутку принадлежат


значения этой переменной. В данном случае выражение x ∈ [
2 ; 8 ] указывает на то, что переменная x, входящая в
неравенство 2 ≤ x ≤ 8, принимает все значения в промежутке
от 2 до 8 включительно. При этих значениях неравенство
будет верным.

Обратим внимание на то, что ответ записан с помощью


квадратных скобок, поскольку границы неравенства 2 ≤ x ≤ 8,
а именно числа 2 и 8 принадлежат множеству решений этого
неравенства.

Множество решений неравенства 2 ≤ x ≤ 8 также можно


изобразить с помощью координатной прямой:

Здесь границы числового промежутка 2 и 8 соответствуют


границам неравенства 2 ≤ x ≤ 8, а выделенная штрихами
область соответствует множеству значений x, которые
являются решениями неравенства 2 ≤ x ≤ 8.

В некоторых источниках границы, которые не принадлежат


числовому промежутку, называют открытыми.

Открытыми их называют по той причине, что числовой


промежуток остаётся открытым из-за того, что его границы не
принадлежат этому числовому промежутку. Пустой кружок на
координатной прямой математики называют выколотой
точкой. Выколоть точку значит исключить её из числового
промежутка или из множества решений неравенства.
1280

А в случае, когда границы принадлежат числовому


промежутку, их называют закрытыми (или замкнутыми),
поскольку такие границы закрывают (замыкают) собой
числовой промежуток. Закрашенный кружок на координатной
прямой также говорит о закрытости границ.

Существуют разновидности числовых промежутков.


Рассмотрим каждый из них.

Числовой луч

Числовым лучом называют числовой промежуток, который


задаётся неравенством x ≥ a, где a — граница данного
неравенства, x — решение неравенства.

Пусть a = 3. Тогда неравенство x ≥ a примет вид x ≥ 3.


Решениями данного неравенства являются все числа,
которые больше 3, включая само число 3.

Изобразим числовой луч, заданный неравенством x ≥ 3, на


координатной прямой. Для этого отметим на ней точку с
координатой 3, а всю оставшуюся справа от неё область
выделим штрихами. Выделяется именно правая часть,
поскольку решениями неравенства x ≥ 3 являются числа,
бóльшие 3. А бóльшие числа на координатной прямой
располагаются правее

Здесь точка 3 соответствует границе неравенства x ≥ 3, а


выделенная штрихами область соответствует множеству
значений x, которые являются решениями неравенства x ≥ 3.
1281

Точка 3, являющаяся границей числового луча, изображена в


виде закрашенного кружка, поскольку граница неравенства
x ≥ 3 принадлежит множеству его решений.

На письме числовой луч, заданный


неравенством x ≥ a, обозначается следующим образом:

[ a ; +∞ )

Видно, что с одной стороны граница обрамлена квадратной


скобкой, а с другой круглой. Это связано с тем, что одна
граница числового луча принадлежит ему, а другая нет,
поскольку бесконечность сама по себе границ не имеет и
подразумевается, что по ту сторону нет числа, замыкающего
этот числовой луч.

Учитывая то, что одна из границ числового луча закрыта,


данный промежуток часто называют закрытым числовым
лучом.

Запишем ответ к неравенству x ≥ 3 с помощью обозначения


числового луча. У нас переменная a равна 3

x ∈  [ 3 ; +∞ )

В этом выражении говорится, что переменная x, входящая в


неравенство x ≥ 3, принимает все значения от 3 до плюс
бесконечности.

Иначе говоря, все числа от 3 до плюс бесконечности,


являются решениями неравенства x ≥ 3. Граница 3
принадлежит множеству решений, поскольку
неравенство x ≥ 3 является нестрогим.
1282

Закрытым числовым лучом также называют числовой


промежуток, который задаётся неравенством x ≤ a.
Решениями неравенства x ≤ a являются все числа, которые
меньше a, включая само число a. 

К примеру, если a = 2, то неравенство примет вид x ≤ 2. На


координатной прямой граница 2 будет изображаться
закрашенным кружком, а вся область, находящаяся слева,
будет выделена штрихами. В этот раз выделяется левая
часть, поскольку решениями неравенства x ≤ 2 являются
числа, меньшие 2. А меньшие числа на координатной прямой
располагаются левее

Здесь точка 2 соответствует границе неравенства x ≤ 2, а


выделенная штрихами область соответствует множеству
значений x, которые являются решениями неравенства x ≤ 2.

Точка 2, являющаяся границей числового луча, изображена в


виде закрашенного кружка, поскольку граница неравенства
x ≤ 2 принадлежит множеству его решений.

Запишем ответ к неравенству x ≤ 2 с помощью обозначения


числового луча:

x ∈  ( −∞ ; 2 ]

В этом выражении говорится, что все числа от минус


бесконечности до 2, являются решениями
неравенства x ≤ 2. Граница 2 принадлежит множеству
решений, поскольку неравенство x ≤ 2 является нестрогим.
1283

Открытый числовой луч

Открытым числовым лучом называют числовой


промежуток, который задаётся неравенством x > a, где a —
граница данного неравенства, x — решение неравенства.

Открытый числовой луч во многом похож на закрытый


числовой луч. Различие в том, что граница a не принадлежит
промежутку, как и граница неравенства x > a не принадлежит
множеству его решений.

Пусть a = 3. Тогда неравенство примет вид x > 3. Решениями


данного неравенства являются все числа, которые больше 3,
за исключением числа 3

На координатной прямой граница открытого числового луча,


заданного неравенством x > 3, будет изображаться в виде
пустого кружка. Вся область, находящаяся справа, будет
выделена штрихами:

Здесь точка 3 соответствует границе неравенства x > 3, а


выделенная штрихами область соответствует множеству
значений x, которые являются решениями неравенства x
> 3. Точка 3, являющаяся границей открытого числового
луча, изображена в виде пустого кружка, поскольку граница
неравенства x > 3 не принадлежит множеству его решений.

На письме открытый числовой луч, заданный


неравенством x > a, обозначается следующим образом:

( a ; +∞ )
1284

Круглые скобки указывают на то, что границы открытого


числового луча не принадлежат ему.

Запишем ответ к неравенству x > 3 с помощью обозначения


открытого числового луча:

x ∈  ( 3 ; +∞ )

В этом выражении говорится, что все числа от 3 до плюс


бесконечности, являются решениями неравенства x > 3.
Граница 3 не принадлежит множеству решений, поскольку
неравенство x > 3 является строгим.

Открытым числовым лучом также называют числовой


промежуток, который задаётся неравенством x < a, где a —
граница данного неравенства, x — решение неравенства.
Решениями неравенства x < a являются все числа, которые
меньше a, исключая число a. 

К примеру, если a = 2, то неравенство примет вид x < 2. На


координатной прямой граница 2 будет изображаться пустым
кружком, а вся область, находящаяся слева, будет выделена
штрихами:

Здесь точка 2 соответствует границе неравенства x < 2, а


выделенная штрихами область соответствует множеству
значений x, которые являются решениями неравенства
x < 2. Точка 2, являющаяся границей открытого числового
луча, изображена в виде пустого кружка, поскольку граница
неравенства x < 2 не принадлежит множеству его решений.
1285

На письме открытый числовой луч, заданный неравенством


x < a, обозначается следующим образом:

( −∞ ; a )

Запишем ответ к неравенству x < 2 с помощью обозначения


открытого числового луча:

x ∈  ( −∞ ; 2 )

В этом выражении говорится, что все числа от минус


бесконечности до 2, являются решениями неравенства x < 2.
Граница 2 не принадлежит множеству решений, поскольку
неравенство x < 2 является строгим.

Отрезок

Отрезком называют числовой промежуток, который задаётся


двойным неравенством a ≤ x ≤ b, где a и b — границы
данного неравенства, x — решение неравенства.

Пусть a = 2, b = 8. Тогда неравенство a ≤ x ≤ b примет вид


2 ≤ x ≤ 8. Решениями неравенства 2 ≤ x ≤ 8 являются все
числа, которые больше 2 и меньше 8. При этом границы
неравенства 2 и 8 принадлежат множеству его решений,
поскольку неравенство 2 ≤ x ≤ 8 является нестрогим.

Изобразим отрезок, заданный двойным


неравенством 2 ≤ x ≤ 8 на координатной прямой. Для этого
отметим на ней точки с координатами 2 и 8, а
располагающуюся между ними область выделим штрихами:
1286

Здесь точки 2 и 8 соответствуют границам неравенства


2 ≤ x ≤ 8, а выделенная штрихами область соответствует
множеству значений x, которые являются решениями
неравенства 2 ≤ x ≤ 8. Точки 2 и 8, являющиеся границами
отрезка, изображены в виде закрашенных кружков, поскольку
границы неравенства 2 ≤ x ≤ 8 принадлежат множеству его
решений.

На письме отрезок, заданный неравенством


a ≤ x ≤ b обозначается следующим образом:

[a;b]

Квадратные скобки с обеих сторон указывают на то, что


границы отрезка принадлежат ему. Запишем ответ к
неравенству 2 ≤ x ≤ 8 с помощью этого обозначения:

x ∈  [ 2 ; 8 ]

В этом выражении говорится, что все числа от 2 до 8


включительно, являются решениями неравенства 2 ≤ x ≤ 8.

Интервал

Интервалом называют числовой промежуток, который


задаётся двойным неравенством a < x < b, где a и b —
границы данного неравенства, x — решение неравенства.

Пусть a = 2, b = 8. Тогда неравенство a < x < b примет вид


2 < x < 8. Решениями этого двойного неравенства являются
все числа, которые больше 2 и меньше 8, исключая числа 2 и
8.

Изобразим интервал на координатной прямой:


1287

Здесь точки 2 и 8 соответствуют границам неравенства


2 < x < 8, а выделенная штрихами область соответствует
множеству значений x, которые являются решениями
неравенства 2 < x < 8. Точки 2 и 8, являющиеся границами
интервала, изображены в виде пустых кружков, поскольку
границы неравенства 2 < x < 8 не принадлежат множеству
его решений.

На письме интервал, заданный неравенством a < x < b,


обозначается следующим образом:

(a;b)

Круглые скобки с обеих сторон указывают на то, что границы


интервала не принадлежат ему. Запишем ответ к
неравенству 2 < x < 8 с помощью этого обозначения:

x ∈  ( 2 ; 8 )

В этом выражении говорится, что все числа от 2 до 8,


исключая числа 2 и 8, являются решениями неравенства
2 < x < 8.

Полуинтервал

Полуинтервалом называют числовой промежуток, который


задаётся неравенством a ≤ x < b, где a и b — границы
данного неравенства, x — решение неравенства.

Полуинтервалом также называют числовой промежуток,


который задаётся неравенством a < x ≤ b.
1288

Одна из границ полуинтервала принадлежит ему. Отсюда и


название этого числового промежутка.

В ситуации с полуинтервалом a ≤ x < b ему (полуинтервалу)


принадлежит левая граница.

А в ситуации с полуинтервалом a < x ≤ b ему принадлежит


правая граница.

Пусть a = 2, b = 8. Тогда неравенство a ≤ x < b примет вид


2 ≤ x < 8. Решениями этого двойного неравенства являются
все числа, которые больше 2 и меньше 8, включая число 2,
но исключая число 8.

Изобразим полуинтервал 2 ≤ x < 8 на координатной прямой:

Здесь точки 2 и 8 соответствуют границам неравенства


2 ≤ x < 8, а выделенная штрихами область соответствует
множеству значений x, которые являются решениями
неравенства 2 ≤ x < 8.

Точка 2, являющаяся левой границей полуинтервала,


изображена в виде закрашенного кружка, поскольку левая
граница неравенства 2 ≤ x < 8 принадлежит множеству его
решений.

А точка 8, являющаяся правой границей полуинтервала,


изображена в виде пустого кружка, поскольку правая граница
неравенства 2 ≤ x < 8 не принадлежит множеству его
решений.
1289

На письме полуинтервал, заданный неравенством a ≤ x < b,


обозначается следующим образом:

[ a ; b )

Видно, что с одной стороны граница обрамлена квадратной


скобкой, а с другой круглой. Это связано с тем, что одна
граница полуинтервала принадлежит ему, а другая нет.
Запишем ответ к неравенству 2 ≤ x < 8 с помощью этого
обозначения:

x ∈  [ 2 ; 8 )

В этом выражении говорится, что все числа от 2 до 8,


включая число 2, но исключая число 8, являются решениями
неравенства 2 ≤ x < 8.

Аналогично на координатной прямой можно


изобразить полуинтервал, заданный
неравенством a < x ≤ b. Пусть a = 2, b = 8. Тогда
неравенство a < x ≤ b примет вид 2 < x ≤ 8. Решениями этого
двойного неравенства являются все числа, которые больше
2 и меньше 8, исключая число 2, но включая число 8.

Изобразим полуинтервал 2 < x ≤ 8 на координатной прямой:

Здесь точки 2 и 8 соответствуют границам неравенства


2 < x ≤ 8, а выделенная штрихами область соответствует
множеству значений x, которые являются решениями
неравенства 2 < x ≤ 8.
1290

Точка 2, являющаяся левой границей полуинтервала,


изображена в виде пустого кружка, поскольку левая граница
неравенства 2 < x ≤ 8 не принадлежит множеству его
решений.

А точка 8, являющаяся правой границей полуинтервала,


изображена в виде закрашенного кружка, поскольку правая
граница неравенства 2 < x ≤ 8 принадлежит множеству его
решений.

На письме полуинтервал, заданный неравенством a < x ≤ b,


обозначается так: ( a ; b ]. Запишем ответ к
неравенству 2 < x ≤ 8 с помощью этого обозначения:

x ∈  ( 2 ; 8 ]

В этом выражении говорится, что все числа от 2 до 8,


исключая число 2, но включая число 8, являются решениями
неравенства 2 < x ≤ 8.

Изображение числовых промежутков на координатной


прямой

Числовой промежуток может быть задан с помощью


неравенства или с помощью обозначения (круглых или
квадратных скобок). В обоих случаях нужно суметь
изобразить этот числовой промежуток на координатной
прямой. Рассмотрим несколько примеров.

Пример 1. Изобразить числовой промежуток, заданный


неравенством x > 5
1291

Вспоминаем, что неравенством вида x > a задаётся


открытый числовой луч. В данном случае переменная a
равна 5. Неравенство x > 5 строгое, поэтому граница 5 будет
изображаться в виде пустого кружкá. Нас интересуют все
значения x, которые больше 5, поэтому вся область справа
будет выделена штрихами:

Пример 2. Изобразить числовой промежуток (5; +∞) на


координатной прямой

Это тот же числовой промежуток, который мы изобразили в


предыдущем примере. Но в этот раз он задан не с помощью
неравенства, а с помощью обозначения числового
промежутка.

Граница 5 обрамлена круглой скобкой, значит она не


принадлежит промежутку. Соответственно, кружок остаётся
пустым.

Символ +∞ указывает, что нас интересуют все числа,


которые больше 5. Соответственно, вся область справа от
границы 5 выделяется штрихами:

Пример 3. Изобразить числовой промежуток (−5; 1) на


координатной прямой.
1292

Круглыми скобками с обеих сторон обозначаются интервалы.


Границы интервала не принадлежат ему, поэтому границы −5
и 1 будут изображаться на координатной прямой в виде
пустых кружков. Вся область между ними будет выделена
штрихами:

Пример 4. Изобразить числовой промежуток, заданный


неравенством −5 < x < 1

Это тот же числовой промежуток, который мы изобразили в


предыдущем примере. Но в этот раз он задан не с помощью
обозначения промежутка, а с помощью двойного
неравенства.

Неравенством вида a < x < b, задаётся интервал. В данном


случае переменная a равна −5, а переменная b равна
единице. Неравенство −5 < x < 1 строгое, поэтому границы
−5 и 1 будут изображаться в виде пустых кружка. Нас
интересуют все значения x, которые больше −5, но меньше
единицы, поэтому вся область между точками −5 и 1 будет
выделена штрихами:

Пример 5. Изобразить на координатной прямой числовые


промежутки [-1; 2] и [2; 5]
1293

В этот раз изобразим на координатной прямой сразу два


промежутка.

Квадратными скобками с обеих сторон обозначаются


отрезки. Границы отрезка принадлежат ему, поэтому границы
отрезков [-1; 2] и [2; 5] будут изображаться на координатной
прямой в виде закрашенных кружков. Вся область между
ними будет выделена штрихами.

Чтобы хорошо увидеть промежутки [−1; 2] и [2; 5], первый


можно изобразить на верхней области, а второй на нижней.
Так и поступим:

Пример 6. Изобразить на координатной прямой числовые


промежутки [-1; 2) и (2; 5]

Квадратной скобкой с одной стороны и круглой с другой


обозначаются полуинтервалы. Одна из границ
полуинтервала принадлежат ему, а другая нет.

В случае с полуинтервалом [-1; 2) левая граница будет


принадлежать ему, а правая нет. Значит левая граница будет
изображаться в виде закрашенного кружка. Правая же
граница будет изображаться в виде пустого кружка.
1294

А в случае с полуинтервалом (2; 5] ему будет принадлежать


только правая граница, а левая нет. Значит левая граница
будет изображаться в виде пустого кружка. Правая же
граница будет изображаться в виде закрашенного кружка.

Изобразим промежуток [-1; 2) на верхней области


координатной прямой, а промежуток (2; 5] — на нижней:

Примеры решения неравенств

Неравенство, которое путём тождественных преобразований


можно привести к виду ax > b (или к виду ax < b), будем
называть линейным неравенством с одной переменной.

В линейном неравенстве ax > b, x — это переменная,


значения которой нужно найти, а — коэффициент этой
переменной, b — граница неравенства, которая в
зависимости от знака неравенства может принадлежать
множеству его решений либо не принадлежать ему.

Например, неравенство 2x > 4 является неравенством вида


ax > b. В нём роль переменной a играет число 2, роль
переменной b (границы неравенства) играет число 4.

Неравенство 2x > 4 можно сделать ещё проще. Если мы


разделим обе его части на 2, то получим неравенство x > 2
1295

Получившееся неравенство x > 2 также является


неравенством вида ax > b, то есть линейным неравенством с
одной переменной. В этом неравенстве роль переменной a
играет единица. Ранее мы говорили, что коэффициент 1 не
записывают. Роль переменной b играет число 2.

Отталкиваясь от этих сведений, попробуем решить


несколько простых неравенств. В ходе решения мы будем
выполнять элементарные тождественные преобразования с
целью получить неравенство вида ax > b

Пример 1. Решить неравенство x − 7 < 0

Прибавим к обеим частям неравенства число 7

x − 7 + 7 < 0 + 7

В левой части останется x, а правая часть станет равна 7

x < 7

Путём элементарных преобразований мы привели


неравенство x − 7 < 0 к равносильному неравенству x < 7.
Решениями неравенства x < 7 являются все числа, которые
меньше 7. Граница 7 не принадлежит множеству решений,
поскольку неравенство строгое.

Когда неравенство приведено к виду x < a (или x > a), его


можно считать уже решённым. Наше неравенство x − 7 < 0
тоже приведено к такому виду, а именно к виду x < 7. Но в
большинстве школ требуют, чтобы ответ был записан с
помощью числового промежутка и проиллюстрирован на
координатной прямой.
1296

Запишем ответ с помощью числового промежутка. В данном


случае ответом будет открытый числовой луч (вспоминаем,
что числовой луч задаётся неравенством x < a и
обозначается как ( −∞ ; a)

x ∈  ( −∞ ; 7 )

На координатной прямой граница 7 будет изображаться в


виде пустого кружка, а вся область, находящаяся слева от
границы, будет выделена штрихами:

Для проверки возьмём любое число из промежутка ( −∞ ; 7 ) и


подставим его в неравенство x < 7 вместо переменной x.
Возьмём, например, число 2

2 < 7

Получилось верное числовое неравенство, значит и решение


верное. Возьмём ещё какое-нибудь число, например, число 4

4 < 7

Получилось верное числовое неравенство. Значит решение


верное.

А поскольку неравенство x < 7 равносильно исходному


неравенству x − 7 < 0, то решения неравенства x < 7 будут
совпадать с решениями неравенства x − 7 < 0. Подставим те
же тестовые значения 2 и 4 в неравенство x − 7 < 0

2 − 7 < 0

−5 < 0 — Верное неравенство


1297

4 − 7 < 0

−3 < 0 Верное неравенство

Пример 2. Решить неравенство −4x < −16

Разделим обе части неравенства на −4. Не забываем, что


при делении обеих частей неравенства на отрицательное
число, знак неравенства меняется на противоположный:

Мы привели неравенство −4x < −16 к равносильному


неравенству x > 4. Решениями неравенства x > 4 будут все
числа, которые больше 4. Граница 4 не принадлежит
множеству решений, поскольку неравенство строгое.

Изобразим множество решений неравенства x > 4 на


координатной прямой и запишем ответ в виде числового
промежутка:

Пример 3. Решить неравенство 3y + 1 > 1 + 6y


1298

Перенесём 6y из правой части в левую часть, изменив знак.


А 1 из левой части перенесем в правую часть, опять же
изменив знак:

3y − 6y> 1 − 1

Приведём подобные слагаемые:

−3y > 0

Разделим обе части на −3. Не забываем, что при делении


обеих частей неравенства на отрицательное число, знак
неравенства меняется на противоположный:

Решениями неравенства y < 0 являются все числа, меньшие


нуля. Изобразим множество решений неравенства y < 0 на
координатной прямой и запишем ответ в виде числового
промежутка:

Пример 4. Решить неравенство 5(x − 1) + 7 ≤ 1 − 3(x + 2)

Раскроем скобки в обеих частях неравенства:


1299

Перенесем −3x из правой части в левую часть, изменив знак.


Члены −5 и 7 из левой части перенесем в правую часть,
опять же изменив знаки:

Приведем подобные слагаемые:

Разделим обе части получившегося неравенства на 8

Решениями неравенства   являются все числа, которые

меньше . Граница принадлежит множеству решений,

поскольку неравенство  является нестрогим.

Изобразим множество решений неравенства  на


координатной прямой и запишем ответ в виде числового
промежутка:

 
1300

Пример 5. Решить неравенство 

Умножим обе части неравенства на 2. Это позволит


избавиться от дроби в левой части:

Теперь перенесем 5 из левой части в правую часть, изменив


знак:

После приведения подобных слагаемых, получим


неравенство 6x > 1. Разделим обе части этого неравенства
на 6. Тогда получим:

Решениями неравенства   являются все числа, которые

больше  . Граница   не принадлежит множеству решений,

поскольку неравенство  является строгим.

Изобразим множество решений неравенства   на


координатной прямой и запишем ответ в виде числового
промежутка:
1301

Пример 6. Решить неравенство 

Умножим обе части на 6

После приведения подобных слагаемых, получим


неравенство 5x < 30. Разделим обе части этого неравенства
на 5

Решениями неравенства x < 6 являются все числа, которые


меньше 6. Граница 6 не принадлежит множеству решений,
поскольку неравенство является x < 6 строгим.

Изобразим множество решений неравенства x < 6 на


координатной прямой и запишем ответ в виде числового
промежутка:
1302

Пример 7. Решить неравенство 

Умножим обе части неравенства на 10

В получившемся неравенстве раскроем скобки в левой части:

Перенесем члены без x в правую часть

Приведем подобные слагаемые в обеих частях:

Разделим обе части получившегося неравенства на 10


1303

Решениями неравенства x ≤ 3,5 являются все числа, которые


меньше 3,5. Граница 3,5 принадлежит множеству решений,
поскольку неравенство является x ≤ 3,5 нестрогим.

Изобразим множество решений неравенства x ≤ 3,5 на


координатной прямой и запишем ответ в виде числового
промежутка:

Пример 8. Решить неравенство 4 < 4x < 20

Чтобы решить такое неравенство, нужно переменную x


освободить от коэффициента 4. Тогда мы сможем сказать в
каком промежутке находится решение данного неравенства.

Чтобы освободить переменную x от коэффициента, можно


разделить член 4x на 4. Но правило в неравенствах таково,
что если мы делим член неравенства на какое-нибудь число,
то тоже самое надо сделать и с остальными членами,
входящими в данное неравенство. В нашем случае на 4
нужно разделить все три члена неравенства 4 < 4x < 20

Решениями неравенства 1 < x < 5 являются все числа,


которые больше 1 и меньше 5. Границы 1 и 5 не
принадлежат множеству решений, поскольку
неравенство 1 < x < 5 является строгим.
1304

Изобразим множество решений неравенства 1 < x < 5 на


координатной прямой и запишем ответ в виде числового
промежутка:

Пример 9. Решить неравенство −1 ≤ −2x ≤ 0

Разделим все члены неравенства на −2

Получили неравенство 0,5 ≥ x ≥ 0. Двойное неравенство


желательно записывать так, чтобы меньший член
располагался слева, а больший справа. Поэтому перепишем
наше неравенство следующим образом:

0 ≤ x ≤ 0,5

Решениями неравенства 0 ≤ x ≤ 0,5 являются все числа,


которые больше 0 и меньше 0,5. Границы 0 и 0,5
принадлежат множеству решений, поскольку
неравенство 0 ≤ x ≤ 0,5 является нестрогим.

Изобразим множество решений неравенства 0 ≤ x ≤ 0,5 на


координатной прямой и запишем ответ в виде числового
промежутка:
1305

Пример 10. Решить неравенство 

Умножим обе неравенства на 12

Раскроем скобки в получившемся неравенстве и приведем


подобные слагаемые:

Разделим обе части получившегося неравенства на 2

Решениями неравенства x ≤ −0,5 являются все числа,


которые меньше −0,5. Граница −0,5 принадлежит множеству
решений, поскольку неравенство x ≤ −0,5 является
нестрогим.

Изобразим множество решений неравенства x ≤ −0,5 на


координатной прямой и запишем ответ в виде числового
промежутка:
1306

Пример 11. Решить неравенство 

Умножим все части неравенства на 3

Теперь из каждой части получившегося неравенства вычтем


6

Каждую часть получившегося неравенства разделим на −1.


Не забываем, что при делении всех частей неравенства на
отрицательное число, знак неравенства меняется на
противоположный:

Решениями неравенства 3 ≤ a ≤ 9 являются все числа,


которые больше 3 и меньше 9. Границы 3 и 9 принадлежат
множеству решений, поскольку неравенство 3 ≤ a ≤ 9
является нестрогим.
1307

Изобразим множество решений неравенства 3 ≤ a ≤ 9 на


координатной прямой и запишем ответ в виде числового
промежутка:

Когда решений нет

Существуют неравенства, которые не имеют решений.


Таковым, например, является неравенство 6x > 2(3x + 1). В
процессе решения этого неравенства мы придём к тому, что
знак неравенства > не оправдает своего местоположения.
Давайте посмотрим, как это выглядит.

Раскроем скобки в правой части данного неравенство,


получим 6x > 6x + 2. Перенесем 6x из правой части в левую
часть, изменив знак, получим 6x − 6x > 2. Приводим
подобные слагаемые и получаем неравенство 0 > 2, которое
не является верным.

Для наилучшего понимания, перепишем приведение


подобных слагаемых в левой части следующим образом:

Получили неравенство 0x > 2. В левой части располагается


произведение, которое будет равно нулю при любом x. А
ноль не может быть больше, чем число 2. Значит
неравенство 0x > 2 не имеет решений.
1308

А если не имеет решений приведённое равносильное


неравенство 0x > 2, то не имеет решений и исходное
неравенство 6x > 2(3x + 1).

Пример 2. Решить неравенство 

Умножим обе части неравенства на 3

В получившемся неравенстве перенесем член 12x из правой


части в левую часть, изменив знак. Затем приведём
подобные слагаемые:

Правая часть получившегося неравенства при любом x будет


равна нулю. А ноль не меньше, чем −8. Значит неравенство
0x < −8 не имеет решений.

А если не имеет решений приведённое равносильное


неравенство 0x < −8, то не имеет решений и исходное

неравенство  .

Ответ: решений нет.


1309

Когда решений бесконечно много

Существуют неравенства, имеющие бесчисленное


множество решений. Такие неравенства становятся верными
при любом x.

Пример 1. Решить неравенство 5(3x − 9) < 15x

Раскроем скобки в правой части неравенства:

Перенесём 15x из правой части в левую часть, изменив знак:

Приведем подобные слагаемые в левой части:

Получили неравенство 0x < 45. В левой части располагается


произведение, которое будет равно нулю при любом x. А
ноль меньше, чем 45. Значит решением неравенства 0x
< 45 является любое число.

А если приведённое равносильное неравенство 0x


< 45 имеет бесчисленное множество решений, то и исходное
неравенство 5(3x − 9) < 15x имеет те же решения.

Ответ можно записать в виде числового промежутка:

x ∈ ( −∞; +∞ )

В этом выражении говорится, что решениями


неравенства 5(3x − 9) < 15x являются все числа от минус
бесконечности до плюс бесконечности.
1310

Пример 2. Решить неравенство: 31(2x + 1) − 12x > 50x

Раскроем скобки в левой части неравенства:

Перенесём 50x из правой части в левую часть, изменив знак.


А член 31 из левой части перенесём в правую часть, опять
же изменив знак:

Приведём подобные слагаемые:

Получили неравенство 0x > −31. В левой части


располагается произведение, которое будет равно нулю при
любом x. А ноль больше, чем −31. Значит решением
неравенства 0x < −31 является любое число.

А если приведённое равносильное неравенство 0x


> −31 имеет бесчисленное множество решений, то и
исходное неравенство 31(2x + 1) − 12x > 50x имеет те же
решения.

Запишем ответ в виде числового промежутка:

x ∈ ( −∞; +∞ )

Задания для самостоятельного решения


Задание 1. Решите неравенство:
1311

Показать решение
Задание 2. Решите неравенство:

Показать решение
Задание 3. Решите неравенство:

Показать решение
Задание 4. Решите неравенство:

Показать решение
Задание 5. Решите неравенство:

Показать решение
Задание 6. Решите неравенство:

Показать решение
Задание 7. Решите неравенство:

Показать решение
Задание 8. Решите неравенство:

Показать решение
Задание 9. Решите неравенство:
1312

Показать решение
Задание 10. Решите неравенство:

Показать решение
Задание 11. Решите неравенство:

Показать решение
Задание 12. Решите неравенство:

Показать решение
1313

Системы линейных неравенств с одной переменной

Предварительные навыки

 Общие сведения о неравенствах

Содержание урока


o Примеры решения систем линейных неравенств с
одной переменной
o Когда решений нет
o Задания для самостоятельного решения

Примеры решения систем линейных неравенств с одной


переменной

Несколько линейных неравенств, удовлетворяющих одним и


тем же решениям, образуют систему.

Рассмотрим простейший пример. Система  состоит из


двух неравенств, которые уже решены.

Решениями первого неравенства являются все числа,


которые больше 4. Решениями второго неравенства
являются все числа, которые меньше 9.

Изобразим множество решений каждого неравенства на


координатной прямой и запишем ответы к ним в виде
числовых промежутков:
1314

Но дело в том, что неравенства x > 4 и x < 9 соединены


знаком системы, а значит зависимы друг от друга. Им не
дозволяется раскидываться решениями, как захочется. Наша
задача указать решения, которые одновременно будут
удовлетворять и первому неравенству и второму.

Говоря по-простому, нужно указать числа, которые больше 4,


но меньше 9. Очевидно, что речь идет о числах,
находящихся в промежутке от 4 до 9.

Значит решениями системы   являются числа от 4 до 9.


Границы 4 и 9 не включаются во множество решений
системы, поскольку неравенства x > 4 и x < 9 строгие. Ответ
можно записать в виде числового промежутка:

x ∈ ( 4 ; 9 )

Также, нужно изобразить множество решений системы на


координатной прямой.

Для системы линейных неравенств решение на


координатной прямой изображают так:

Сначала указывают границы обоих неравенств:

На верхней области отмечают множество решений первого


неравенства x > 4
1315

На нижней области отмечают множество решений второго


неравенства x < 9

Нас интересует область, которая отмечена штрихами с обеих


сторон. В этой области и располагаются решения

системы  . Видно, что эта область располагается в


промежутке от 4 до 9. Для наглядности выделим эту область
красным цветом:

Для проверки можно взять любое число из этого промежутка

и подставить его в исходную систему  . Возьмем,


например, число 6

Видим, что решение 6 удовлетворяет обоим неравенствам.


Возьмём ещё какое-нибудь число из промежутка (4; 9),
например, число 8

Видим, что решение 8 удовлетворяет обоим неравенствам.


1316

Исходя из рассмотренного примера, можно сформировать


правило для решения системы линейных неравенств:

Чтобы решить систему линейных неравенств, нужно по


отдельности решить каждое неравенство, и указать в
виде числового промежутка множество решений,
удовлетворяющих каждому неравенству.

Пример 2. Решить систему неравенств 

Решениями первого неравенства являются все числа,


которые больше 17. Решениями второго неравенства
являются все числа, которые больше 12.

Решениями же обоих неравенств являются все числа,


которые больше 17.

Изобразим множество решений системы   на


координатной прямой и запишем ответ в виде числового
промежутка.

Для начала отметим на координатной прямой границы обоих


неравенств:

На верхней области отметим множество решений первого


неравенства x > 17
1317

На нижней области отметим множество решений второго


неравенства x > 12

Нас интересует область, которая отмечена штрихами с обеих


сторон. В этой области и располагаются решения системы

. Видно, что эта область располагается в промежутке


от 17 до плюс бесконечности. Запишем ответ в виде
числового промежутка:

x ∈ ( 17 ; +∞ )

Пример 3. Решить систему неравенств 

Решим каждое неравенство по отдельности. Делать это


можно внутри системы. Если испытываете затруднения при
решении каждого неравенства, обязательно изучите
предыдущий урок
1318

Получили систему  . На этом решение завершается.


Осталось изобразить множество решений системы на
координатной прямой и записать ответ в виде числового
промежутка.

Как и в прошлом примере, сначала нужно отметить границы


обоих неравенств, затем отметить множество решений
каждого неравенства (x > 6 и x > 3). Область координатной
прямой, отмеченная с обеих сторон, будет промежутком, в

котором располагается множество решений системы 

x ∈ ( 6 ; + ∞ )
1319

Пример 4. Решить систему неравенств 

Решим каждое неравенство по отдельности:

Изобразим множество решений системы   на


координатной прямой и запишем ответ в виде числового
промежутка:

Пример 5. Решить неравенство 

Решим каждое неравенство по отдельности:


1320

Изобразим множество решений системы   на


координатной прямой и запишем ответ в виде числового
промежутка:

Когда решений нет

Если неравенства, входящие в систему, не имеют общих


решений, то говорят, что система не имеет решений.

Пример 1. Решить неравенство 


1321

Решим каждое неравенство по отдельности:

Решениями первого неравенства являются все числа,


которые больше 7, включая число 7. Решениями второго
неравенства являются все числа, которые меньше −3,
включая число −3.

Видим, что у данных неравенств нет общих решений.


Увидеть это наглядно позволит координатная прямая.
Отметим на ней множество решений каждого неравенства:

На координатной прямой нет областей, которые отмечены


штрихами с обеих сторон. Это говорит о том, что
неравенства y ≥ 7 и y ≤ −3 не имеют общих решений. Значит

не имеет решений система 


1322

А если не имеет решений приведённая равносильная

система  , то не имеет решений и исходная

система 

Ответ: решений нет.

Пример 2. Решить систему неравенств 

Решим каждое неравенство по отдельности:

Изобразим множество решений неравенств x ≤ −3 и x ≥ 9 на


координатной прямой:
1323

Видим, что на координатной прямой нет областей, которые


отмечены штрихами с обеих сторон. Значит неравенства
x ≤ −3 и x ≥ 9 не имеют общих решений. А значит не имеет

решений система 

А если не имеет решений приведённая равносильная

система  , то не имеет решений и исходная система

Ответ: решений нет.

Пример 3.  Решить систему неравенств 

Решим каждое неравенство по отдельности:


1324

Получили неравенства 0 < −0,2 и a > 5. Первое неравенство


не является верным и не имеет решений. Решением второго
неравенство a > 5 являются все числа, которые больше 5. Но
поскольку первое неравенство не будет верным ни при каком
a, то можно сделать вывод, что у неравенств нет общих
решений. А значит не имеет решений исходная

система 

Ответ: решений нет.

Задания для самостоятельного решения


Задание 1. Решите неравенство:

Показать решение
Задание 2. Решите неравенство:
1325

Показать решение
Задание 3. Решите неравенство:

Показать решение
Задание 4. Решите неравенство:

Показать решение
Задание 5. Решите неравенство:

Показать решение
Задание 6. Решите неравенство:

Показать решение
Задание 7. Решите неравенство:

Показать решение
Задание 8. Решите неравенство:
1326

Показать решение
1327

Операции над множествами

Предварительные навыки

 Отрицательные числа
 Что такое множество?
 Общие сведения о неравенствах
 Системы линейных неравенств с одной переменной

Содержание урока

 Пересечение множеств
 Объединение множеств
 Решение неравенств, содержащих знак ≠
 Решение совокупностей неравенств
 Задания для самостоятельного решения

Пересечение множеств

Рассмотрим два множества: множество друзей Джона и


множество друзей Майкла.

Том,
Друзья Джона =  Фред,
{ Макс,
Джорж }
Лео,
Друзья Майкла = Том,
{ Фред,
Эван }

Видим, что Том и Фред одновременно являются друзьями


Джона и Майкла.
1328

Говоря на языке множеств, элементы Том и Фред


принадлежат как множеству друзей Джона, так и множеству
друзей Майкла.

Зададим новое множество с названием «Общие друзья


Джона и Майкла» и в качестве элементов добавим в него
Тома и Фреда:

Общие друзья = { Том,


Джона и Майкла Фред }

В данном случае множество «Общие друзья Джона и


Майкла» является пересечением множеств друзей Джона и
Майкла.

Пересечением двух (или нескольких) исходных


множеств называется множество, которое состоит из
элементов, принадлежащих каждому из исходных
множеств.

В нашем случае элементы Том и Фред принадлежат каждому


из исходных множеств, а именно: множеству друзей Джона и
множеству друзей Майкла.

Обозначим множество друзей Джона через букву A,


множество друзей Майкла — через букву B, а множество
общих друзей Джона и Майкла обозначим через букву C:

A = { Том, Фред, Макс, Джордж }

B = { Лео, Том, Фред, Эван }

C = { Том, Фред }
1329

Тогда пересечением множеств A и B будет множество C и


записываться следующим образом:

A ∩ B = C

Символ ∩ означает пересечение.

Говоря о множестве, обычно подразумевают элементы,


принадлежащие этому множеству. Символ пересечения ∩
читается, как союз И. Тогда выражение A ∩ B = C можно
прочитать следующим образом:

«Элементы, принадлежащие множеству A И множеству B,


есть элементы, принадлежащие множеству C».

Или еще проще:

«Друзья, одновременно принадлежащие Джону И Майклу,


есть общие друзья Джона и Майкла».

Теперь представим, что у Джона и Майкла нет общих друзей.


Для удобства, как и прежде обозначим множество друзей
Джона через букву A, а множество друзей Майкла через
букву B

A = { Макс, Джордж }

B = { Лео, Эван }

В этом случае говорят, что исходные множества не имеют


общих элементов и пересечением таких множеств является
пустое множество. Пустое множество обозначается
символом ∅

A ∩ B = ∅
1330

Пример 2. Рассмотрим два множества: множество A,


состоящее из чисел 1, 2, 3, 5, 7 и множество B, состоящее из
чисел 1, 2, 3, 4, 6, 12, 18

A = { 1, 2, 3, 5, 7 }

B = { 1, 2, 3, 4, 6, 12, 18 }

Зададим новое множество C и добавим в него элементы,


которые одновременно принадлежат множеству A и
множеству B

C = { 1, 2, 3 }

Множество С является пересечением множеств A и B,


поскольку элементы множества C одновременно
принадлежат множеству A и множеству B

Пример 3. Рассмотрим два множества: множество A,


состоящее из чисел 1, 5, 7, 9 и множество B, состоящее из
чисел 1, 4, 5, 7

A = { 1, 5, 7, 9 }

B = { 1, 4, 5, 7 }

Зададим новое множество C и добавим в него элементы,


которые одновременно принадлежат множеству A и
множеству B

C = { 1, 5, 7 }
1331

Множество С является пересечением множеств A и B,


поскольку элементы множества C одновременно
принадлежат множеству A и множеству B.

Пример 4. Найти пересечение следующих множеств:

A = { 1, 2, 3, 7, 9 }

B = { 1, 3, 5, 7, 9}

С = { 3, 4, 5, 8,  9}

Пересечением множеств A, B и C будет множество,


состоящее из элементов, принадлежащих каждому из
множеств A, B и C. Этими элементами являются числа 3 и 9.

Зададим новое множество D и добавим в него элементы 3 и


9. Затем с помощью символа пересечения ∩ запишем, что
пересечением множеств A, B и C является множество D

D = { 3, 9}

A ∩ B ∩ C = D

Чтобы найти пересечение, вовсе необязательно задавать


множества с помощью букв. Если элементов мало, то
множество можно задать прямым перечислением элементов.

К примеру, пусть первое множество состоит из элементов 1,


3, 5, а второе из элементов 2, 3, 5. Пересечением в данном
случае является множество, состоящее из элементов 3 и 5.
Чтобы записать пересечение, можно воспользоваться
прямым перечислением:
1332

{ 1, 3, 5 } ∩ { 2, 3, 5 } = { 3, 5 }

Числовые промежутки, которые мы рассмотрели в


предыдущих уроках, тоже являются множествами.
Элементами таких множеств являются числа, входящие в
числовой промежуток.

Например, отрезок [2; 6] можно понимать, как множество всех


чисел от 2 до 6. Для наглядности можно перечислить все
целые числа, принадлежащие данному отрезку:

2, 3, 4, 5, 6 ∈ [2; 6]

Следует иметь ввиду, что мы перечислили только целые


числа. Отрезку [2; 6] также принадлежат и другие числа, не
являющиеся целыми, например, десятичные дроби.
Десятичные дроби располагаются между целыми числами,
но их количество настолько велико, что перечислить их не
представляется возможным.

Еще пример. Интервал (2; 6) можно понимать, как множество


всех чисел от 2 до 6, кроме чисел 2 и 6. Ранее мы говорили,
что интервал это такой числовой промежуток, границы
которого не принадлежат ему. Для наглядности можно
перечислить все целые числа, принадлежащие интервалу
(2; 6):

3, 4, 5 ∈ (2; 6)

Поскольку числовые промежутки являются множествами, то


мы можем находить пересечения между различными
числовыми промежутками. Рассмотрим несколько примеров.
1333

Пример 5. Даны два числовых промежутка: [2; 6] и [4; 8].


Найти их пересечение.

Оба промежутка обрамлены квадратными скобками, значит


их границы принадлежат им.

Для наглядности перечислим все целые числа,


принадлежащие промежуткам [2; 6] и [4; 8]:

2, 3, 4, 5, 6 ∈ [2; 6]

4, 5, 6, 7, 8 ∈ [4; 8]

Видно, что числа 4, 5, 6 принадлежат как первому


промежутку [2; 6], так и второму [4; 8].

Тогда пересечением числовых промежутков [2; 6] и [4; 8]


будет числовой промежуток [4; 6]

[2; 6] ∩ [4; 8] = [4; 6]

Изобразим промежутки [2; 6] и [4; 8] на координатной прямой.


На верхней области отметим числовой промежуток [2; 6], на
нижней — промежуток [4; 8]

Видно, что числа, принадлежащие промежутку [4; 6],


принадлежат как промежутку [2; 6], так и промежутку [4; 8].
Можно также заметить, что штрихи, входящие в промежутки
[2; 6] и [4; 8] пересекаются в промежутке [4; 6]. В такой
ситуации, когда перед глазами есть координатная прямая,
понятие пересечения множеств можно понимать в прямом
смысле, что очень удобно.
1334

Пример 6. Найти пересечение числовых промежутков [−2; 3]


и [4; 7]

Оба промежутка обрамлены квадратными скобками, значит


их границы принадлежат им.

Для наглядности перечислим все целые числа,


принадлежащие промежуткам [−2; 3] и [4; 7]:

−2, −1, 0, 1, 2, 3 ∈ [−2; 3]

4, 5, 6, 7 ∈ [4; 7]

Видно, что числовые промежутки [−2; 3] и [4; 7] не имеют


общих чисел. Поэтому их пересечением будет пустое
множество:

[−2; 3] ∩ [4; 7] = Ø

Если изобразить числовые промежутки [−2; 3] и [4; 7] на


координатной прямой, то можно увидеть, что они нигде не
пересекаются:

Пример 7. Дано множество из одного элемента { 2 }. Найти


его пересечение с промежутком (−3; 4)

Множество, состоящее из одного элемента { 2 }, на


координатной прямой изображается в виде закрашенного
кружка, а числовой промежуток (−3; 4) это интервал, границы
1335

которого не принадлежат ему. Значит границы −3 и 4 будут


изображаться в виде пустых кружков:

Пересечением множества { 2 } и числового


промежутка (−3; 4) будет множество, состоящее из одного
элемента { 2 }, поскольку элемент 2 принадлежит как
множеству { 2 }, так и числовому промежутку (−3; 4)

{ 2 } ∩ (−3; 4) = { 2 }

На самом деле мы уже занимались пересечением числовых


промежутков, когда решали системы линейных неравенств.
Вспомните, как мы решали их. Сначала находили множество
решений первого неравенства, затем множество решений
второго. Затем находили множество решений, которые
удовлетворяют обоим неравенствам.

По сути, множество решений, удовлетворяющих обоим


неравенствам, является пересечением множеств решений
первого и второго неравенства. Роль этих множеств берут на
себя числовые промежутки.

Например, чтобы решить систему неравенств   , мы


должны сначала найти множества решений каждого
неравенства, затем найти пересечение этих множеств.

В данном примере решением первого неравенства x ≥ 3


является множество всех чисел, которые больше 3 (включая
само число 3). Иначе говоря, решением неравенства
является числовой промежуток [3; +∞)
1336

Решением второго неравенства x ≤ 6 является множество


всех чисел, которые меньше 6 (включая само число 6). Иначе
говоря, решением неравенства является числовой
промежуток (−∞; 6]

А общим решением системы будет пересечение множеств


решений первого и второго неравенства, то есть
пересечение числовых промежутков [3; +∞) и (−∞; 6]

Если мы изобразим множество решений системы  на


координатной прямой, то увидим, что эти решения
принадлежат промежутку [3; 6], который в свою очередь
является пересечением промежутков [3; +∞) и (−∞; 6]

[3; +∞) ∩ (−∞; 6] = [3; 6]

Поэтому в качестве ответа мы указывали, что значения


переменной x принадлежат числовому промежутку [3; 6], то
есть пересечению множеств решений первого и второго
неравенства

x ∈ [3; 6]

Пример 2. Решить неравенство 


1337

Все неравенства, входящие в систему уже решены. Нужно


только указать те решения, которые являются общими для
всех неравенств.

Решением первого неравенства является числовой


промежуток (−∞; −1).

Решением второго неравенства является числовой


промежуток (−∞; −5).

Решением третьего неравенства является числовой


промежуток (−∞; 4).

Решением системы  будет пересечение числовых


промежутков (−∞; −1), (−∞; −5) и (−∞; 4). В данном случае
этим пересечением является промежуток (−∞; −5).

(−∞; −1) ∩ (−∞; −5) ∩ (−∞; 4) = (−∞; −5)

На рисунке представлены числовые промежутки и


неравенства, которыми эти числовые промежутки заданы.
1338

Видно, что числа, принадлежащие промежутку (−∞; −5),


одновременно принадлежат всем исходным промежуткам.

Запишем ответ к системе   с помощью числового


промежутка:

x ∈ (−∞; −5)

Пример 3. Решить неравенство 

Решением первого неравенства y > 7 является числовой


промежуток (7; +∞).

Решением второго неравенства y < 4 является числовой


промежуток (−∞; 4).

Решением системы   будет пересечение числовых


промежутков (7; +∞) и (−∞; 4).

В данном случае пересечением числовых


промежутков (7; +∞) и (−∞; 4) является пустое множество,
поскольку эти числовые промежутки не имеют общих
элементов:

(7; +∞) ∩ (−∞; 4) = ∅

Если изобразить числовые промежутки (7; +∞) и (−∞; 4) на


координатной прямой, то можно увидеть, что они нигде не
пересекаются:
1339

Объединение множеств

Объединением двух (или нескольких) исходных


множеств называют множество, которое состоит из
элементов, принадлежащих хотя бы одному из исходных
множеств.

На практике объединение множеств состоит из всех


элементов, принадлежащих исходным множествам. Поэтому
и говорят, что элементы такого множества принадлежат хотя
бы одному из исходных множеств.

Рассмотрим множество A с элементами 1, 2, 3 и множество B


с элементами 4, 5, 6.

A = { 1, 2, 3 }

B = { 4, 5, 6 }

Зададим новое множество C и добавим в него все элементы


множества A и все элементы множества B

C = { 1, 2, 3, 4, 5, 6 }

В данном случае объединением множеств A и B является


множество C и обозначается следующим образом:

A ∪ B = C

Символ ∪ означает объединение и заменяет собой союз


ИЛИ. Тогда выражение A ∪ B = C можно прочитать так:
1340

Элементы, принадлежащие множеству A ИЛИ множеству


B, есть элементы, принадлежащие множеству C.

В определении объединения сказано, что элементы такого


множества принадлежат хотя бы одному из исходных
множеств. Данную фразу можно понимать в прямом смысле.

Вернёмся к созданному нами множеству C, куда входят все


элементы множеств A и B. Возьмём для примера из этого
множества элемент 5. Что можно про него сказать?

Если 5 является элементом множества C, а множество С


является объединением множеств A и B, то можно с
уверенностью заявить, что элемент 5 принадлежит хотя бы
одному из множеств A и B. Так оно и есть:

A = { 1, 2, 3 }

B = { 4, 5, 6 }

C = { 1, 2, 3, 4, 5, 6 }

Возьмем ещё один элемент из множества С, например,


элемент 2. Что можно про него сказать?

Если 2 является элементом множества C, а множество С


является объединением множеств A и B, то можно с
уверенностью заявить, что элемент 2 принадлежит хотя бы
одному из множеств A и B. Так оно и есть:

A = {1, 2, 3}

B = {4, 5, 6}

C = { 1, 2, 3, 4, 5, 6 }
1341

Если мы захотим объединить два или более множества и


вдруг обнаружим, что один или несколько элементов
принадлежат каждому из этих множеств, то в объединение
повторяющиеся элементы будут входить только один раз.

Например, рассмотрим множество A с элементами 1, 2, 3, 4 и


множество B с элементами 2, 4, 5, 6.

A = {1, 2, 3, 4}

B = {2, 4, 5, 6}

Видим, что элементы 2 и 4 одновременно принадлежат и


множеству A, и множеству B. Если мы захотим объединить
множества A и B, то новое множество C будет содержать
элементы 2 и 4 только один раз. Выглядеть это будет так:

C = { 1, 2, 3, 4, 5, 6 }

Чтобы при объединении не допустить ошибок, обычно


поступают так: сначала в новое множество добавляют все
элементы первого множества, затем добавляют элементы
второго множества, которые не принадлежат первому
множеству. Попробуем сделать такое объединение с
множествами A и B.

Итак, у нас имеются следующие исходные множества:

A = { 1, 2, 3, 4 }

B = { 2, 4, 5, 6 }

Зададим новое множество С и добавим в него все элементы


множества A
1342

C = { 1, 2, 3, 4,

Теперь добавим элементы из множества B, которые не


принадлежат множеству A. Множеству A не принадлежат
элементы 5 и 6. Их и добавим во множество C

C = { 1, 2, 3, 4, 5, 6 }

Пример 2. Друзьями Джона являются Том, Фред, Макс и


Джордж. А друзьями Майкла являются Лео, Том, Фред и
Эван. Найти объединение множеств друзей Джона и Майкла.

Для начала зададим два множества: множество друзей


Джона и множество друзей Майкла.

Том,
Друзья Джона =  Фред,
{ Макс,
Джорж }
Лео,
Друзья Майкла = Том,
{ Фред,
Эван }

Зададим новое множество с названием «Все друзья Джона и


Майкла» и добавим в него всех друзей Джона и Майкла.

Заметим, что Том и Фред одновременно являются друзьями


Джона и Майкла, поэтому мы добавим их в новое множество
только один раз, поскольку сразу двух Томов и двух Фредов
не бывает.

Все друзья = { Том, Фред, Макс, Джордж,


1343

Джона и Майкла Лео, Эван }

В данном случае множество всех друзей Джона и Майкла


является объединением множеств друзей Джона и Майкла.

Друзья Джона ∪ Друзья Майкла = Все друзья Джона и Майкла

Пример 3. Даны два числовых промежутка: [−7; 0] и [−3; 5].


Найти их объединение.

Оба промежутка обрамлены квадратными скобками, значит


их границы принадлежат им.

Для наглядности перечислим все целые числа,


принадлежащие этим промежуткам:

−7, −6, −5, −4, −3,−2, −1, 0  ∈ [−7; 0]

−3,−2, −1, 0, 1, 2, 3, 4, 5 ∈ [−3; 5]

Объединением числовых промежутков [−7; 0] и [−3; 5] будет


числовой промежуток [−7; 5], который содержит все числа
промежутка [−7; 0] и [−3; 5] без повторов некоторых из чисел

−7, −6, −5, −4, −3,−2, −1, 0, 1, 2, 3, 4, 5 ∈ [−7; 5]

Обратите внимание, что числа −3,−2, −1 принадлежали и


первому промежутку и второму. Но поскольку в объединение
допускается включать такие элементы только один раз, мы
включили их единоразово.

Значит объединением числовых промежутков [−7; 0] и [−3; 5]


будет числовой промежуток [−7; 5]
1344

[−7; 0] ∪ [−3; 5] = [−7; 5]

Изобразим на координатной прямой промежутки [−7; 0] и


[−3; 5]. На верхней области отметим числовой промежуток
[−7; 0], на нижней — промежуток [−3; 5]

Ранее мы выяснили, что промежуток [−7; 5] является


объединением промежутков [−7; 0] и [−3; 5]. Здесь полезно
вспомнить про определение объединения множеств, которое
было приведено в самом начале. Объединение трактуется,
как множество, состоящее из всех элементов,
принадлежащих хотя бы одному из исходных множеств.

Действительно, если взять любое число из


промежутка [−7; 5], то окажется, что оно принадлежит хотя
бы одному из промежутков: либо промежутку [−7; 0] либо
промежутку [−3; 5].

Возьмём из промежутка [−7; 5] любое число, например число


2. Поскольку промежуток [−7; 5] является объединением
промежутков [−7; 0] и [−3; 5], то число 2 будет принадлежать
хотя бы одному из этих промежутков. В данном случае число
2 принадлежит промежутку [−3; 5]

Возьмём ещё какое-нибудь число. Например, число −4. Это


число будет принадлежать хотя бы одному из промежутков:
1345

[−7; 0] или [−3; 5]. В данном случае оно принадлежит


промежутку [−7; 0]

Возьмём ещё какое-нибудь число. Например, число −2. Оно


принадлежит как промежутку [−7; 0], так и промежутку [−3; 5].
Но на координатной прямой оно указывается только один
раз, поскольку в одной точке сразу два числа −2 не бывает.

Не каждое объединение числовых промежутков является


числовым промежутком. Например, попробуем найти
объединение числовых промежутков [−2; −1] и [4; 7].

Идея остаётся та же самая — объединением числовых


промежутков [−2;−1] и [4; 7] будет множество, состоящее из
элементов, принадлежащих хотя бы одному из промежутков:
[−2; −1] или [4; 7]. Но это множество не будет являться
числовым промежутком. Для наглядности перечислим все
целые числа, принадлежащие этому объединению:

[−2; −1] ∪ [4; 7] = { −2, −1, 4, 5, 6, 7 }

Получили множество { −2, −1, 4, 5, 6, 7 }. Это множество не


является числовым промежутком по причине того, что числа,
располагающиеся между −1 и 4, не вошли в полученное
множество
1346

Числовой промежуток должен содержать все числа от левой


границы до правой. Если одно из чисел отсутствует, то
числовой промежуток теряет смысл. Допустим, имеется
линейка длиной 15 см

Эта линейка является числовым промежутком [0; 15],


поскольку содержит все числа в промежутке от 0 до 15
включительно. Теперь представим, что на линейке после
числа 9 сразу следует число 12.

Эта линейка не является линейкой в 15 см, и её


нежелательно использовать для измерения. Также, её
нельзя назвать числовым промежутком [0; 15], поскольку она
не содержит все числа, которые должна была содержать.

Решение неравенств, содержащих знак ≠

Некоторые неравенства содержат знак ≠ (не равно).


Например, 2x ≠ 8. Чтобы решить такое неравенство, нужно
найти множество значений переменной x, при которых левая
часть не равна правой части.

Решим неравенство 2x ≠ 8. Разделим обе части данного


неравенства на 2, тогда получим:
1347

Получили равносильное неравенство x ≠ 4. Решением этого


неравенства является множество всех чисел, не равных 4.
То есть если мы подставим в неравенство x ≠ 4 любое число,
которое не равно 4, то получим верное неравенство.

Подставим, например, число 5

5 ≠ 4 — верное неравенство, поскольку 5 не равно 4

Подставим 7

7 ≠ 4 — верное неравенство, поскольку 7 не равно 4

И поскольку неравенство x ≠ 4 равносильно исходному


неравенству 2x ≠ 8, то решения неравенства x ≠ 4 будут
подходить и к неравенству 2x ≠ 8. Подставим те же тестовые
значения 5 и 7 в неравенство 2x ≠ 8.

2×5≠8

2×7≠8

Изобразим множество решений неравенства x ≠ 4 на


координатной прямой. Для этого выколем точку 4 на
координатной прямой, а всю оставшуюся область с обеих
сторон выделим штрихами:

Теперь запишем ответ в виде числового промежутка. Для


этого воспользуемся объединением множеств. Любое число,
1348

являющееся решением неравенства 2x ≠ 8 будет


принадлежать либо промежутку (−∞; 4) либо промежутку
(4; +∞). Так и записываем, что значения переменной x
принадлежат (−∞; 4) или (4; +∞). Напомним, что для слова
«или» используется символ ∪

x ∈ (−∞; 4) ∪ (4; +∞)

В этом выражении говорится, что значения, принимаемые


переменной x, принадлежат промежутку (−∞; 4) или
промежутку (4; +∞).

Неравенства, содержащие знак ≠, также можно решать, как


обычные уравнения. Для этого знак ≠ заменяют на знак =.
Тогда получится обычное уравнение. В конце решения
найденное значение переменной x нужно исключить из
множества решений.

Решим предыдущее неравенство 2x ≠ 8, как обычное


уравнение. Заменим знак ≠ на знак равенства =, получим
уравнение 2x = 8. Разделим обе части данного уравнения на
2, получим x = 4.

Видим, что при x, равном 4, уравнение обращается в верное


числовое равенство. При других значениях равенства
соблюдаться не будет. Эти другие значения нас и
интересуют. А для этого достаточно исключить найденную
четвёрку из множества решений.

Пример 2. Решить неравенство 3x − 5 ≠ 1 − 2x


1349

Перенесем −2x из правой части в левую часть, изменив знак,


а −5 из левой части перенесём в правую часть, опять же
изменив знак:

Приведем подобные слагаемые в обеих частях:

Разделим обе части получившегося неравенства на 5

Решением неравенства x ≠ 1,2 является множество всех


чисел, не равных 1,2.

Изобразим множество решений неравенства x ≠ 1,2 на


координатной прямой и запишем ответ в виде числового
промежутка:

x ∈ (−∞; 1,2) ∪ (1,2; +∞)

В этом выражении говорится, что значения, принимаемые


переменной x принадлежат промежутку (−∞; 1,2) или
промежутку (1,2; +∞)
1350

Решение совокупностей неравенств

Рассмотрим ещё один вид неравенств, который называется


совокупностью неравенств. Такой тип неравенств,
возможно, вы будете решать редко, но для общего развития
полезно изучить и их.

Совокупность неравенств очень похожа на систему


неравенств. Различие в том, что в системе неравенств нужно
найти множество решений, удовлетворяющих каждому
неравенству, образующему эту систему.

А в случае с совокупностью неравенств, нужно найти


множество решений, удовлетворяющих хотя бы одному
неравенству, образующему эту совокупность.

Совокупность неравенств обозначается квадратной скобкой.


Например, следующая запись из двух неравенств является
совокупностью:

Решим данную совокупность. Сначала нужно решить каждое


неравенство по отдельности.

Решением первого неравенства x ≥ 3 является числовой


промежуток [3; +∞). Решением второго неравенства x ≤ 6
является числовой промежуток (−∞; 6].

Множество значений x, при которых верно хотя бы одно из


неравенств, будет принадлежать промежутку [3; +∞) или
промежутку (−∞; 6]. Так и записываем:

x ∈ [3; +∞) ∪ (−∞; 6]


1351

В этом выражении говорится, что переменная x, входящая в

совокупность  принимает все значения, принадлежащие


промежутку [3; +∞) или промежутку (−∞; 6]. А это то, что нам
нужно. Ведь решить совокупность означает найти множество
решений, удовлетворяющих хотя бы одному неравенству,
образующему эту совокупность. А любое число из
промежутка [3; +∞) или промежутка (−∞; 6] будет
удовлетворять хотя бы одному неравенству.

Например, число 9 из промежутка [3; +∞) удовлетворяет


первому неравенству x ≥ 3. А число −7 из промежутка (−∞; 6]
удовлетворяет второму неравенству x ≤ 6.

Посмотрите внимательно на выражение x ∈ [3; +∞) ∪ (−∞; 6],


а именно на его правую часть. Ведь
выражение [3; +∞) ∪ (−∞; 6] представляет собой объединение
числовых промежутков [3; +∞) и (−∞; 6]. Точнее, объединение
множеств решений первого и второго неравенства.

Стало быть, решением совокупности неравенств


является объединение множеств решений первого и
второго неравенства.

Иначе говоря, решением совокупности  будет


объединение числовых промежутков [3; +∞) и (−∞; 6]

Объединением числовых промежутков [3; +∞) и (−∞; 6]


является промежуток (−∞; +∞). Точнее, объединением
1352

числовых промежутков [3; +∞) и (−∞; 6] является вся


координатная прямая. А вся координатная прямая это все
числа, которые только могут быть

[3; +∞) ∪ (−∞; 6] = (−∞; +∞)

Ответ можно оставить таким, каким мы его записали ранее:

x ∈ [3; +∞) ∪ (−∞; 6]

либо заменить на более короткий:

x ∈ (−∞; +∞)

Возьмём любое число из полученного объединения, и


проверим удовлетворяет ли оно хотя бы одному
неравенству.

Возьмем для примера число 8. Оно удовлетворяет первому


неравенству x ≥ 3.

8 ≥ 3

Возьмем еще какое-нибудь число, например, число 1. Оно


удовлетворяет второму неравенству x ≤ 6

1 ≤ 6

Возьмем еще какое-нибудь число, например, число 5. Оно


удовлетворяет и первому неравенству x ≥ 3 и второму x ≤ 6
1353

Пример 2. Решить совокупность неравенств 

Чтобы решить эту совокупность, нужно найти множество


решений, которые удовлетворяют хотя бы одному
неравенству, образующему эту совокупность.

Для начала найдём множество решений первого неравенства


x < −0,25. Этим множеством является числовой промежуток
(−∞; −0,25).

Множеством решений второго неравенства x ≥ −7 является


числовой промежуток [−7; +∞).

Решением совокупности неравенств   будет


объединение множеств решений первого и второго
неравенства.

x ∈ (−∞; −0,25) ∪ [−7; +∞)

Иначе говоря, решением совокупности  будет


объединение числовых промежутков (−∞; −0,25) и [−7; +∞)

Объединением числовых промежутков (−∞; −0,25) и [−7; +∞)


является является вся координатная прямая. А вся
координатная прямая это все числа, которые только могут
быть

(−∞; −0,25) ∪ [−7; +∞) = (−∞; +∞)


1354

Ответ можно оставить таким, каким мы его записали ранее:

x ∈ (−∞; −0,25) ∪ [−7; +∞)

либо заменить на более короткий:

x ∈ (−∞; +∞)

Пример 3. Решить совокупность неравенств 

Решим каждое неравенство по отдельности:

Множеством решений первого неравенства x < −3 является


числовой промежуток (−∞; −3).

Множеством решений второго неравенства x ≤ 0 является


числовой промежуток (−∞; 0].
1355

Решением совокупности неравенств  будет


объединение множеств решений первого и второго
неравенства.

x ∈ (−∞; −3) ∪ (−∞; 0]

Иначе говоря, решением совокупности  будет


объединение числовых промежутков (−∞; −3) и (−∞; 0]

Объединением числовых промежутков (−∞; −3) и (−∞; 0]


является числовой промежуток (−∞; 0]

(−∞; −3) ∪ (−∞; 0] = (−∞; 0]

Ответ можно оставить таким, каким мы его записали ранее:

x ∈ (−∞; −3) ∪ (−∞; 0]

либо заменить на более короткий:

x ∈ (−∞; 0]

Задания для самостоятельного решения


Задание 1. Найдите пересечение и объединение следующих
множеств:
А = { 1, 2, 5 }
B = { 3, 4, 5 }
Показать решение
1356

Задание 2. Найдите пересечение и объединение следующих


множеств:
А = { −3, −2, −1, 0, 1, 2 }
B = { 1, 2, 3, 4, 5 }
Показать решение
Задание 3. Найдите пересечение и объединение следующих
множеств:
А = { 1, 2, 3 }
B = { 3, 4 }
Показать решение
Задание 4. Найдите пересечение и объединение следующих
числовых промежутков:
[−2; 7) и (0; 10]
Показать решение
Задание 5. Найдите пересечение и объединение следующих
числовых промежутков:
(−∞; 3] и [−2; 1)
Показать решение
Задание 6. Найдите пересечение и объединение следующих
числовых промежутков:
(3; +∞) и [2; +∞)
Показать решение
Задание 7. Найдите пересечение и объединение следующих
числовых промежутков:
[−3; −1] и (−2; 4]
1357

Показать решение
Задание 8. Решите неравенство:

Показать решение
Задание 9. Решите неравенство:

Показать решение
Задание 10. Решите совокупность неравенств:

Показать решение
Задание 11. Решите совокупность неравенств:

Показать решение
Задание 12. Решите совокупность неравенств:

Показать решение
1358

Степень с натуральным показателем

Предварительные навыки


o Умножение
o Порядок действий
o Умножение и деление целых чисел

Содержание урока

 Что такое степень?


 Возведение в степень числа 10
 Представление чисел 10, 100, 1000 в виде степени с
основанием 10
 Возведение в степень отрицательного числа
 Нахождение значений выражений
 Тождественные преобразования степеней
o Умножение степеней
o Возведение в степень произведения
o Возведение степени в степень
o Деление степеней
o Возведение в степень обыкновенных дробей
o Возведение в степень десятичных дробей
 Задания для самостоятельного решения

Что такое степень?

Степенью называют произведение из нескольких


одинаковых множителей. Например:

2 × 2 × 2
1359

Значение данного выражения равно 8

2 × 2 × 2 = 8

Левую часть этого равенства можно сделать короче –


сначала записать повторяющийся множитель и указать над
ним сколько раз он повторяется. Повторяющийся множитель
в данном случае это 2. Повторяется он три раза. Поэтому
над двойкой записываем тройку:

23 = 8

Это выражение читается так: «два в третьей степени


равно восемь» или «третья степень числа 2 равна 8».

Короткую форму записи перемножения одинаковых


множителей используют чаще. Поэтому надо помнить, что
если над каким-то числом надписано другое число, то это
есть перемножение нескольких одинаковых множителей.

Например, если дано выражение 53, то следует иметь ввиду,


что это выражение равносильно записи 5 × 5 × 5.

Число, которое повторяется называют основанием степени.


В выражении 53 основанием степени является число 5.

А число, которое надписано над числом 5 называют


показателем степени. В выражении 53 показателем степени
является число 3. Показатель степени показывает сколько
раз повторяется основание степени. В нашем случае
основание 5 повторяется три раза
1360

Саму операцию перемножения одинаковых множителей


называют возведением в степень.

Например, если нужно найти произведение из четырёх


одинаковых множителей, каждый из которых равен 2, то
говорят, что число 2 возводится в четвёртую степень:

Видим, что число 2 в четвёртой степени есть число 16.

Отметим, что в данном уроке мы рассматриваем степени с


натуральным показателем. Это вид степени, показателем
которой является натуральное число. Напомним, что
натуральными называют целые числа, которые больше нуля.
Например, 1, 2, 3 и так далее.

Вообще, определение степени с натуральным показателем


выглядит следующим образом:

Степень числа a с натуральным показателем n — это


выражение вида an, которое равно произведению n
множителей, каждый из которых равен a

Примеры:
1361

Следует быть внимательным при возведении числа в


степень. Часто по невнимательности человек умножает
основание степени на показатель.

Например, число 5 во второй степени есть произведение


двух множителей каждый из которых равен 5. Это
произведение равно 25

Теперь представим, что мы по невнимательности умножили


основание 5 на показатель 2

Получилась ошибка, поскольку число 5 во второй степени не


равно 10.

Дополнительно следует упомянуть, что степень числа с


показателем 1, есть само это число:
1362

Например, число 5 в первой степени есть само число 5

Соответственно, если у числа отсутствует показатель, то


надо считать, что показатель равен единице.

Например, числа 1, 2, 3 даны без показателя, поэтому их


показатели будут равны единице. Каждое из этих чисел
можно записать с показателем 1

А если возвести 0 в какую-нибудь степень, то получится 0.


Действительно, сколько бы раз ничего не умножалось на
само себя получится ничего. Примеры:

А выражение 00  не имеет смысла. Но в некоторых разделах


математики, в частности анализе и теории множеств,
выражение 00 может иметь смысл.

Для тренировки решим несколько примеров на возведение


чисел в степени.

Пример 1. Возвести число 3 во вторую степень.

Число 3 во второй степени это произведение двух


множителей, каждый из которых равен 3
1363

32 = 3 × 3 = 9

Пример 2. Возвести число 2 в четвертую степень.

Число 2 в четвертой степени это произведение четырёх


множителей, каждый из которых равен 2

24 =2 × 2 × 2 × 2 = 16

Пример 3. Возвести число 2 в третью степень.

Число 2 в третьей степени это произведение трёх


множителей, каждый из которых равен 2

23 =2 × 2 × 2 = 8

Возведение в степень числа 10

Чтобы возвести в степень число 10, достаточно дописать


после единицы количество нулей, равное показателю
степени.

Например, возведем число 10 во вторую степень. Сначала


запишем само число 10 и в качестве показателя укажем
число 2

102

Теперь ставим знак равенства, записываем единицу и после


этой единицы записываем два нуля, поскольку количество
нулей должно быть равно показателю степени
1364

102 = 100

Значит, число 10 во второй степени это число 100. Связано


это с тем, что число 10 во второй степени это произведение
двух множителей, каждый из которых равен 10

102 = 10 × 10 = 100

Пример 2. Возведём число 10 в третью степень.

В данном случае после единицы будут стоять три нуля:

103 = 1000

Пример 3. Возведем число 10 в четвёртую степень.

В данном случае после единицы будут стоять четыре нуля:

104 = 10000

Пример 4. Возведем число 10 в первую степень.

В данном случае после единицы будет стоять один нуль:

101 = 10

Представление чисел 10, 100, 1000 в виде степени с


основанием 10

Чтобы представить числа 10, 100, 1000 и 10000 в виде


степени с основанием 10, нужно записать основание 10, и в
1365

качестве показателя указать число, равное количеству нулей


исходного числа.

Представим число 10 в виде степени с основанием 10.


Видим, что в нём один нуль. Значит, число 10 в виде степени
с основанием 10 будет представлено как 101

10 = 101

Пример 2. Представим число 100 в виде степени


основанием 10. Видим, что число 100 содержит два нуля.
Значит, число 100 в виде степени с основанием 10 будет
представлено как 102

100 = 102

Пример 3. Представим число 1 000 в виде степени с


основанием 10.

1 000 = 103

Пример 4. Представим число 10 000 в виде степени с


основанием 10.

10 000 = 104

Возведение в степень отрицательного числа

При возведении в степень отрицательного числа, его


обязательно нужно заключить в скобки.
1366

Например, возведём отрицательное число −2 во вторую


степень. Число −2 во второй степени это произведение двух
множителей, каждый из которых равен (−2)

(−2)2 = (−2) × (−2) = 4

Если бы мы не заключили в скобки число −2, то получилось


бы что мы вычисляем выражение −22, которое не равно 4.
Выражение −2² будет равно −4. Чтобы понять почему,
коснёмся некоторых моментов.

Когда мы ставим перед положительным числом минус, мы


тем самым выполняем операцию взятия
противоположного значения.

Допустим, дано число 2, и нужно найти его противоположное


число. Мы знаем, что противоположное числу 2 это число −2.
Иными словами, чтобы найти противоположное число для 2,
достаточно поставить минус перед этим числом. Вставка
минуса перед числом уже считается в математике
полноценной операцией. Эту операцию, как было указано
выше, называют операцией взятия противоположного
значения.

В случае с выражением −22 происходит две операции:


операция взятия противоположного значения и возведение в
степень. Возведение в степень является более приоритетной
операцией, чем взятие противоположного значения.

Поэтому выражение −22 вычисляется в два этапа. Сначала


выполняется операция возведения в степень. В данном
случае во вторую степень было возведено положительное
число 2
1367

Затем выполнилось взятие противоположного значения. Это


противоположное значение было найдено для значения 4. А
противоположное значение для 4 это −4

−22  = −4

Скобки же имеют самый высокий приоритет выполнения.


Поэтому в случае вычисления выражения (−2)2 сначала
выполняется взятие противоположного значения, а затем во
вторую степень возводится отрицательное число −2. В
результате получается положительный ответ 4, поскольку
произведение отрицательных чисел есть положительное
число.

Пример 2. Возвести число −2 в третью степень.

Число −2 в третьей степени это произведение трёх


множителей, каждый из которых равен (−2)

(−2)3 = (−2) × (−2) × (−2) = −8

Пример 3. Возвести число −2 в четвёртую степень.

Число −2 в четвёртой степени это произведение четырёх


множителей, каждый из которых равен (−2)

(−2)4 = (−2) × (−2) × (−2) × (−2) = 16

Легко заметить, что при возведении в степень


отрицательного числа может получиться либо
положительный ответ либо отрицательный. Знак ответа
зависит от показателя исходной степени.
1368

Если показатель степени чётный, то ответ будет


положительным. Если показатель степени нечётный, ответ
будет отрицательным. Покажем это на примере числа −3

В первом и в третьем случае показатель был нечётным


числом, поэтому ответ стал отрицательным.

Во втором и в четвёртом случае показатель был чётным


числом, поэтому ответ стал положительным.

Пример 7. Возвести число −5 в третью степень.

Число −5 в третьей степени это произведение трёх


множителей каждый из которых равен −5. Показатель 3
является нечётным числом, поэтому мы заранее можем
сказать, что ответ будет отрицательным:

(−5)3 = (−5) × (−5) × (−5) = −125

Пример 8. Возвести число −4 в четвёртую степень.

Число −4 в четвёртой степени это произведение четырёх


множителей, каждый из которых равен −4. При этом
показатель 4 является чётным, поэтому мы заранее можем
сказать, что ответ будет положительным:

(−4)4 = (−4) × (−4) × (−4) × (−4) = 256


1369

Нахождение значений выражений

При нахождении значений выражений, не содержащих


скобки, возведение в степень будет выполняться в первую
очередь, далее умножение и деление в порядке их
следования, а затем сложение и вычитание в порядке их
следования.

Пример 1. Найти значение выражения 2 + 52

Сначала выполняется возведение в степень. В данном


случае во вторую степень возводится число 5 — получается
25. Затем этот результат складывается с числом 2

2 + 52 = 2 + 25 = 27

Пример 10. Найти значение выражения −62 × (−12)

Сначала выполняется возведение в степень. Заметим, что


число −6 не взято в скобки, поэтому во вторую степень будет
возведено число 6, затем перед результатом будет
поставлен минус:

−62 × (−12) = −36 × (−12)

Завершаем пример, умножив −36 на (−12)

−62 × (−12) = −36 × (−12) = 432

Пример 11. Найти значение выражения −3 × 22


1370

Сначала выполняется возведение в степень. Затем


полученный результат перемножается с числом −3

−3 × 22 = −3 × 4 = −12

Если выражение содержит скобки, то сначала нужно


выполнить действия в этих скобках, далее возведение в
степень, затем умножение и деление, а затем сложение и
вычитание.

Пример 12. Найти значение выражения (32 + 1 × 3) − 15 + 5

Сначала выполняем действия в скобках. Внутри скобок


применяем ранее изученные правила, а именно сначала
возводим во вторую степень число 3, затем выполняем
умножение 1 × 3, затем складываем результаты возведения
в степень числа 3 и умножения 1 × 3. Далее выполняется
вычитание и сложение в порядке их следования. Расставим
такой порядок выполнения действия над исходным
выражением:

(32 + 1 × 3) − 15 + 5 = 12 − 15 + 5 = 2

Пример 13. Найти значение выражения 2 × 53 + 5 × 23

Сначала возведем числа в степени, затем выполним


умножение и сложим полученные результаты:

2 × 53 + 5 × 23 = 2 × 125 + 5 × 8 = 250 + 40 = 290


1371

Тождественные преобразования степеней

Над степенями можно выполнять различные тождественные


преобразования, тем самым упрощая их.

Допустим, потребовалось вычислить выражение (23)2. В


данном примере два в третьей степени возводится во вторую
степень. Иными словами, степень возводится в другую
степень.

(23)2 это произведение двух степеней, каждая из которых


равна 23

При этом каждая из этих степеней является произведением


трёх множителей, каждый из которых равен 2

Получили произведение 2 × 2 × 2 × 2 × 2 × 2, которое равно


64. Значит значение выражения (23)2 или равно 64

Этот пример можно значительно упростить. Для этого


показатели выражения (23)2 можно перемножить и записать
это произведение над основанием 2
1372

Получили 26. Два в шестой степени это произведение шести


множителей, каждый из которых равен 2. Это произведение
равно 64

Данное свойство работает по причине того, что 23 это


произведение 2 × 2 × 2, которое в свою очередь повторяется
два раза. Тогда получается, что основание 2 повторяется
шесть раз. Отсюда можно записать, что 2 × 2 × 2 × 2 × 2 × 2
это 26

Вообще, для любого основания a с показателями m и n,


выполняется следующее равенство:

(an)m = an  ×  m

Это тождественное преобразование называют возведением


степени в степень. Его можно прочитать так: «При
возведении степени в степень основание оставляют без
изменений, а показатели перемножают».

После перемножения показателей, получится другая


степень, значение которой можно найти.

Пример 2. Найти значение выражения (32)2

В данном примере основанием является 3, а числа 2 и 2


являются показателями. Воспользуемся правилом
возведения степени в степень. Основание оставим без
изменений, а показатели перемножим:

Получили 34. А число 3 в четвёртой степени есть 81


1373

Рассмотрим остальные преобразования.

Умножение степеней

Чтобы перемножить степени, нужно по отдельности


вычислить каждую степень, и полученные результаты
перемножить.

Например, умножим 22 на 33.

22 это число 4, а 33 это число 27. Перемножаем числа 4 и 27,


получаем 108

22 × 33 = 4 × 27 = 108

В этом примере основания степеней были разными. В


случае, если основания будут одинаковыми, то можно
записать одно основание, а в качестве показателя записать
сумму показателей исходных степеней.

Например, умножим 22 на 23

В данном примере основания у степеней одинаковые. В этом


случае можно записать одно основание 2 и в качестве
показателя записать сумму показателей степеней 22 и 23.
Иными словами, основание оставить без изменений, а
показатели исходных степеней сложить. Выглядеть это будет
так:

Получили 25. Число 2 в пятой степени есть 32


1374

Данное свойство работает по причине того, что 22 это


произведение 2 × 2, а 23 это произведение 2 × 2 × 2. Тогда
получается произведение из пяти одинаковых множителей,
каждый из которых равен 2. Это произведение представимо в
виде 25

Вообще, для любого a и показателей m и n выполняется


следующее равенство:

Это тождественное преобразование носит название


основного свойства степени. Его можно прочитать так:
«При перемножении степеней с одинаковыми основаниями,
основание оставляют без изменений, а показатели
складывают».

Отметим, что данное преобразование можно применять при


любом количестве степеней. Главное, чтобы основание было
одинаковым.

Например, найдем значение выражения 21 × 22 × 23.


Основание 2 оставим без изменений, а показатели сложим:

В некоторых задачах достаточным бывает выполнить


соответствующее преобразование, не вычисляя итоговую
степень. Это конечно же очень удобно, поскольку вычислять
большие степени не так-то просто.

Пример 1. Представить в виде степени выражение 58 × 25


1375

В данной задаче нужно сделать так, чтобы вместо


выражения 58 × 25 получилась одна степень.

Число 25 можно представить в виде 52. Тогда получим


следующее выражение:

В этом выражении можно применить основное свойство


степени — основание 5 оставить без изменений, а
показатели 8 и 2 сложить:

Задачу можно считать решённой, поскольку мы представили


выражение 58 × 25 в виде одной степени, а именно в виде
степени 510.

Запишем решение покороче:

Пример 2. Представить в виде степени выражение 29 × 32

Число 32 можно представить в виде 25. Тогда получим


выражение 29 × 25. Далее можно применить основание
свойство степени — основание 2 оставить без изменений, а
показатели 9 и 5 сложить. В результате получится
следующее решение:

Пример 3. Вычислите произведение 3 × 3, используя


основное свойство степени.
1376

Все хорошо знают, что три умножить на три равно девять, но


задача требует в ходе решения воспользоваться основным
свойством степени. Как это сделать?

Вспоминаем, что если число дано без показателя, то


показатель нужно считать равным единице. Стало быть
сомножители 3 и 3 можно записать в виде 31 и 31

31 × 31

Теперь воспользуемся основным свойством степени.


Основание 3 оставляем без изменений, а показатели 1 и 1
складываем:

31 × 31 = 32

Далее вычисляем значение выражения. Число 3 во второй


степени равно числу 9

31 × 31 = 32 = 9

Пример 4. Вычислите произведение 2 × 2 × 32 × 33,


используя основное свойство степени.

Произведение 2 × 2 заменим на 21 × 21, затем на 21 + 1, а затем


на 22. Произведение 32 × 33 заменим на 32 + 3, а затем на 35

Далее вычисляем значение каждой степени и находим


произведение:
1377

Пример 5. Выполнить умножение x × x

Это два одинаковых буквенных сомножителя с показателями


1. Для наглядности запишем эти показатели. Далее
основание x оставим без изменений, а показатели сложим:

Находясь у доски, не следует записывать перемножение


степеней с одинаковыми основаниями так подробно, как это
сделано здесь. Такие вычисления нужно выполнять в уме.
Подробная запись скорее всего будет раздражать учителя и
он снизит за это оценку. Здесь же подробная запись дана,
чтобы материал был максимально доступным для
понимания.

Решение данного примера желательно записать так:

Пример 6. Выполнить умножение x2 × x

Показатель второго сомножителя равен единице. Для


наглядности запишем его. Далее основание оставим без
изменений, а показатели сложим:

Пример 7. Выполнить умножение y3y2y

Показатель третьего сомножителя равен единице. Для


наглядности запишем его. Далее основание оставим без
изменений, а показатели сложим:
1378

Пример 8. Выполнить умножение aa3a2a5

Показатель первого сомножителя равен единице. Для


наглядности запишем его. Далее основание оставим без
изменений, а показатели сложим:

Пример 9. Представить степень 38 в виде произведения


степеней с одинаковыми основаниями.

В данной задаче нужно составить произведение степеней,


основания которых будут равны 3, и сумма показателей
которых будет равна 8. Можно использовать любые
показатели. Представим степень 38 в виде произведения
степеней 35 и 33

В данном примере мы опять же опирались на основное


свойство степени. Ведь выражение 35 × 33 можно записать
как 35 + 3, откуда 38.

Конечно можно было представить степень 38 в виде


произведения других степеней. Например, в виде 37 × 31,
поскольку это произведение тоже равно 38
1379

Представление степени в виде произведения степеней с


одинаковыми основаниями это по большей части творческая
работа. Поэтому не нужно бояться экспериментировать.

Пример 10. Представить степень x12 в виде различных


произведений степеней с основаниями x.

Воспользуемся основным свойство степени. Представим x12 в


виде произведений с основаниями x, и сумма показателей
которых равна 12

Конструкции с суммами показателей были записаны для


наглядности. Чаще всего их можно пропустить. Тогда
получится компактное решение:
1380

Возведение в степень произведения

Чтобы возвести в степень произведение, нужно возвести в


указанную степень каждый множитель этого произведения и
перемножить полученные результаты.

Например, возведём во вторую степень произведение 2 × 3.


Возьмём в скобки данное произведение и в качестве
показателя укажем 2

Теперь возведём во вторую степень каждый множитель


произведения 2 × 3 и перемножим полученные результаты:

Принцип работы данного правила основан на определении


степени, которое было дано в самом начале.

Возвести произведение 2 × 3 во вторую степень означает


повторить данное произведение два раза. А если повторить
его два раза, то можно получить следующее:

2 × 3 × 2 × 3

От перестановки мест сомножителей произведение не


меняется. Это позволяет сгруппировать одинаковые
множители:

2 × 2 × 3 × 3

Повторяющиеся множители можно заменить на короткие


записи — основания с показателями. Произведение 2 × 2
можно заменить на 22, а произведение 3 × 3 можно заменить
1381

на 32. Тогда выражение 2 × 2 × 3 × 3 обращается в


выражение 22 × 32.

Пусть ab исходное произведение. Чтобы возвести данное


произведение в степень n, нужно по отдельности возвести
множители a и b в указанную степень n

Данное свойство справедливо для любого количества


множителей. Следующие выражения также справедливы:

Пример 2. Найти значение выражения (2 × 3 × 4)2

В данном примере нужно возвести во вторую степень


произведение 2 × 3 × 4. Чтобы сделать это, нужно возвести
во вторую степень каждый множитель этого произведения и
перемножить полученные результаты:

Пример 3. Возвести в третью степень произведение


a × b × c

Заключим в скобки данное произведение, и в качестве


показателя укажем число 3
1382

Далее возводим в третью степень каждый множитель


данного произведения:

Пример 4. Возвести в третью степень произведение 3xyz

Заключим в скобки данное произведение, и в качестве


показателя укажем 3

(3xyz)3

Возведём в третью степень каждый множитель данного


произведения:

(3xyz)3 = 33x3y3z3

Число 3 в третьей степени равно числу 27. Остальное


оставим без изменений:

(3xyz)3 = 33x3y3z3 = 27x3y3z3

В некоторых примерах умножение степеней с одинаковыми


показателями можно заменять на произведение оснований с
одним показателем.

Например, вычислим значение выражения 52 × 32. Возведем


каждое число во вторую степень и перемножим полученные
результаты:

52 × 32 = 25 × 9 = 225

Но можно не вычислять по отдельности каждую степень.


Вместо этого, данное произведение степеней можно
заменить на произведение с одним показателем (5 × 3)2.
1383

Далее вычислить значение в скобках и возвести полученный


результат во вторую степень:

52 × 32 = (5 × 3)2 = (15)2 = 225

В данном случае опять же было использовано правило


возведения в степень произведения. Ведь, если (a × b)n = an
× bn, то an × bn  = (a × b)n. То есть левая и правая часть
равенства поменялись местами.

Возведение степени в степень

Это преобразование мы рассматривали в качестве примера,


когда пытались понять суть тождественных преобразований
степеней.

При возведении степени в степень основание оставляют без


изменений, а показатели перемножают:

(an)m = an  ×  m

К примеру, выражение (23)2 является возведением степени в


степень — два в третьей степени возводится во вторую
степень. Чтобы найти значение этого выражения, основание
можно оставить без изменений, а показатели перемножить:

(23)2 = 23 × 2 = 26

Далее вычислить степень 26, которая равна 64

(23)2 = 23 × 2 = 26 = 64
1384

Данное правило основано на предыдущих правилах:


возведении в степень произведения и основного свойства
степени.

Вернёмся к выражению (23)2. Выражение в скобках 23


представляет собой произведение из трёх одинаковых
множителей, каждый из которых равен 2. Тогда в выражении
(23)2 степень, находящуюся внутри скобок можно заменить на
произведение 2 × 2 × 2.

(2 × 2 × 2)2

А это есть возведение в степень произведения, которое мы


изучили ранее. Напомним, что для возведения в степень
произведения, нужно возвести в указанную степень каждый
множитель данного произведения и полученные результаты
перемножить:

(2 × 2 × 2)2 = 22 × 22 × 22

Теперь имеем дело с основным свойством степени.


Основание оставляем без изменений, а показатели
складываем:

(2 × 2 × 2)2 = 22 × 22 × 22 = 22 + 2 + 2 = 26

Как и раньше получили 26. Значение этой степени равно 64

(2 × 2 × 2)2 = 22 × 22 × 22 = 22 + 2 + 2 = 26 = 64

В степень также может возводиться произведение,


сомножители которого тоже являются степенями.

Например, найдём значение выражения (22 × 32)3. Здесь


показатели каждого множителя нужно умножить на общий
1385

показатель 3. Далее найти значение каждой степени и


вычислить произведение:

(22 × 32)3 = 22×3  × 32×3 = 26 × 36 = 64 × 729 = 46656

Примерно тоже самое происходит при возведении в степени


произведения. Мы говорили, что при возведении в степень
произведения, в указанную степень возводится каждый
множитель этого произведения.

Например, чтобы возвести произведение 2 × 4 в третью


степень, нужно записать следующее выражение:

Но ранее было сказано, что если число дано без показателя,


то показатель надо считать равным единице. Получается,
что множители произведения 2 × 4 изначально имеют
показатели равные 1. Значит в третью степень возводилось
выражение 21 × 41. А это есть возведение степени в степень.

Перепишем решение с помощью правила возведения


степени в степень. У нас должен получиться тот же
результат:

Пример 2. Найти значение выражения (33)2

Основание оставляем без изменений, а показатели


перемножаем:

Получили 36. Число 3 в шестой степени есть число 729


1386

Пример 3. Выполнить возведение в степень в выражении


(xy)³

Возведём в третью степень каждый множитель


произведения:

Пример 4. Выполнить возведение в степень в выражении


(abc)⁵

Возведём в пятую степень каждый множитель произведения:

Пример 5. Выполнить возведение в степень в выражении


(−2ax)3

Возведём в третью степень каждый множитель


произведения:

Поскольку в третью степень возводилось отрицательное


число −2, оно было взято в скобки.

Далее нужно вычислить то, что вычисляется. В данном


случае можно вычислить (−2)3 — получится −8. Буквенная
часть останется без изменений:
1387

Пример 6. Выполнить возведение в степень в выражении


(10xy)2

Пример 7. Выполнить возведение в степень в выражении


(−5x)3

Пример 8. Выполнить возведение в степень в выражении


(−3y)4

Пример 9. Выполнить возведение в степень в выражении


(−2abx)⁴

Пример 10. Упростите выражение x5 × (x2)3 

Степень x5 пока оставим без изменений, а в выражении


(x2)3 выполним возведение степени в степени:

x5 × (x2)3 = x5 × x2 × 3 = x5 × x6

Теперь выполним умножение x5× x6. Для этого


воспользуемся основным свойством степени — основание
x оставим без изменений, а показатели сложим:
1388

x5 × (x2)3 = x5 × x2× 3 = x5 × x6 = x5 + 6 = x11

Пример 9. Найти значение выражения 43 × 22, используя


основное свойство степени.

Основное свойство степени можно использовать в случае,


если основания  исходных степеней одинаковы. В данном
примере основания разные, поэтому для начала исходное
выражение нужно немного видоизменить, а именно сделать
так, чтобы основания степеней стали одинаковыми.

Посмотрим внимательно на степень 43. Основание у этой


степени есть число 4, которое можно представить в виде 22.
Тогда исходное выражение примет вид (22)3 × 22. Выполнив
возведение степени в степень в выражении (22)3, мы получим
26. Тогда исходное выражение примет вид 26 × 22, вычислить
которое можно, используя основное свойство степени.

Запишем решение данного примера:

Деление степеней

Чтобы выполнить деление степеней, нужно найти значение


каждой степени, затем выполнить деление обыкновенных
чисел.

Например, разделим 43 на 22.

Вычислим 43, получим 64. Вычислим 22, получим 4. Теперь


разделим 64 на 4, получим 16
1389

Если при делении степеней основания окажутся


одинаковыми, то основание можно оставить без изменений, а
из показателя степени делимого вычесть показатель степени
делителя.

Например, найдем значение выражения 23 : 22

Основание 2 оставим без изменений, а из показателя


степени делимого вычтем показатель степени делителя:

Значит, значение выражения 23 : 22 равно 2.

Данное свойство основано на умножении степеней с


одинаковыми основаниями, или как мы привыкли говорить на
основном свойстве степени.

Вернемся к предыдущему примеру 23 : 22. Здесь делимое это


23, а делитель 22.

Разделить одно число на другое означает найти такое число,


которое при умножении на делитель даст в результате
делимое.

В нашем случае, разделить 23 на 22 означает найти такую


степень, которая при умножении на делитель 22 даст в
результате 23. А какую степень можно умножить на 22, чтобы
1390

получить 23 ? Очевидно, что только степень 21. Из основного


свойства степени имеем:

Убедиться, что значение выражения 23 : 22 равно 21 можно


непосредственно вычислив само выражение 23 : 22. Для этого
сначала найдём значение степени 23, получим 8. Затем
найдём значение степени 22, получим 4. Разделим 8 на 4,
получим 2 или 21, поскольку 2 = 21.

23 : 22 = 8 : 4 = 2

Таким образом, при делении степеней с одинаковыми


основаниями выполняется следующее равенство:

Может случиться и так, что одинаковыми могут оказаться не


только основания, но и показатели. В этом случае в ответе
получится единица.

Например, найдём значение выражения 22 : 22. Вычислим


значение каждой степени и выполним деление получившихся
чисел:

При решении примера 22 : 22 также можно применить


правило деления степеней с одинаковыми основаниями. В
результате получается число в нулевой степени, поскольку
разность показателей степеней 22 и 22 равна нулю:
1391

В математике принято считать, что любое число в нулевой


степени есть единица:

Почему число 2 в нулевой степени равно единице мы


выяснили выше. Если вычислить 22 : 22 обычным методом, не
используя правило деления степеней, получится единица.

Пример 2. Найти значение выражения 412 : 410

Воспользуемся правилом деления степеней. Основание 4


оставим без изменений, а из показателя степени делимого
вычтем показатель степени делителя:

412 : 410 = 412 − 10 = 42 = 16

Пример 3. Представить частное x3 : x в виде степени с


основанием x

Воспользуемся правилом деления степеней. Основание x


оставим без изменений, а из показателя степени делимого
вычтем показатель степени делителя. Показатель делителя
равен единице. Для наглядности запишем его:

Пример 4. Представить частное x3 : x2 в виде степени с


основанием x
1392

Воспользуемся правилом деления степеней. Основание x


оставим без изменений, а из показателя степени делимого
вычтем показатель степени делителя:

Деление степеней можно записывать в виде дроби. Так,


предыдущий пример можно записать следующим образом:

Числитель и знаменатель дроби   разрешается записывать


в развёрнутом виде, а именно в виде произведений
одинаковых множителей. Степень x3 можно записать как
x × x × x, а степень x2 как x × x. Тогда конструкцию x3 − 2
можно будет пропустить и воспользоваться сокращением
дроби. В числителе и в знаменателе можно будет сократить
по два множителя x. В результате останется один множитель
x

Или ещё короче:

Также, полезно уметь быстро сокращать дроби, состоящие

из степеней. Например, дробь   можно сократить на x2.


1393

Чтобы сократить дробь   на x2 нужно числитель и

знаменатель дроби   разделить на x2

Деление степеней подробно можно не расписывать.


Приведённое сокращение можно выполнить короче:

Или ещё короче:

Пример 5. Выполнить деление x12 : x3

Воспользуемся правилом деления степеней. Основание x


оставим без изменений, а из показателя степени делимого
вычтем показатель степени делителя:

Запишем решение при помощи сокращения дроби. Деление

степеней x12 : x3 запишем в виде   . Далее сократим данную


дробь на x3.
1394

Пример 6. Найти значение выражения 

В числителе выполним умножение степеней с одинаковыми


основаниями:

Теперь применяем правило деления степеней с


одинаковыми основаниями. Основание 7 оставляем без
изменений, а из показателя степени делимого вычтем
показатель степени делителя:

Завершаем пример, вычислив степень 72

Пример 7. Найти значение выражения 

Выполним в числителе возведение степени в степень.


Сделать это нужно с выражением (23)4

Теперь выполним в числителе умножение степеней с


одинаковыми основаниями:
1395

Теперь применяем правило деления степеней с


одинаковыми основаниями:

Значит, значение выражения   равно 16

В некоторых примерах можно сокращать одинаковые


множители в ходе решения. Это позволяет упростить
выражение и само вычисление в целом.

Например, найдём значение выражения  . Степень 43


запишем в виде возведения степени в степень (22)3. Тогда
получим следующее выражение:

В числителе выполним возведение степени в степень.


Сделать это нужно с выражением (22)3

В числителе и в знаменателе получившегося выражения


содержится степень 26, которую можно сократить на 26

Видим, что в результате осталась единственная степень 32,


значение которой равно 9.
1396

Пример 8. Найти значение выражения 

В знаменателе содержится произведение степеней с


одинаковыми показателями. Согласно правилу возведения в
степень произведения, конструкцию 75 × 45 можно
представить в виде степени с одним показателем (7 × 4)5.
Далее перемножим выражение в скобках, получим 285. В
результате исходное выражение примет следующий вид:

Теперь можно применить правило деления степеней:

Значит, значение выражения  равно 28. Запишем


решение полностью:

Возведение в степень обыкновенных дробей

Чтобы возвести в степень обыкновенную дробь, нужно


возвести в указанную степень числитель и знаменатель этой
дроби.

Например, возведём обыкновенную дробь   во вторую


степень. Возьмём в скобки данную дробь и в качестве
показателя укажем 2
1397

Если не брать в скобки всю дробь, то это равносильно


возведению в степень только числителя данной дроби.
Иными словами, если мы хотим возвести во вторую степень

дробь  , мы не должны записывать это как  .

Итак, чтобы вычислить значение выражения  , нужно


возвести во вторую степень числитель и знаменатель данной
дроби:

Получили дробь в числителе и в знаменателе которой


содержатся степени. Вычислим каждую степень по
отдельности

Значит обыкновенная дробь   во второй степени равна

дроби  .

Приведённое правило работает следующим образом.

Дробь   во второй степень это произведение двух дробей,

каждая из которых равна 


1398

Мы помним, что для перемножения дробей необходимо


перемножить их числители и знаменатели:

А поскольку в числителе и в знаменателе происходит


перемножение одинаковых множителей, то выражения 2 × 2
и 3 × 3 можно заменить на 22 и 32 соответственно:

Откуда и получится ответ  .

Вообще, для любого a и b ≠ 0 выполняется следующее


равенство:

Это тождественное преобразование называют возведением


в степень обыкновенной дроби.

Пример 2. Возвести дробь   в третью степень

Заключим данную дробь в скобки и в качестве показателя


укажем число 3. Далее возведём числитель и знаменатель
данной дроби в третью степень и вычислим получившуюся
дробь:
1399

Отрицательная дробь возводится в степень таким же


образом, но перед вычислениями надо определиться какой
знак будет иметь ответ. Если показатель четный, то ответ
будет положительным. Если показатель нечетный, то ответ
будет отрицательным.

Например, возведём дробь   во вторую степень:

Показатель является чётным числом. Значит ответ будет


положительным. Далее применяем правило возведения в
степень дроби и вычисляем получившуюся дробь:

Ответ положителен по причине того, что выражение   


представляет собой произведение двух сомножителей,

каждый из которых равен дроби 

А произведение отрицательных чисел (в том числе и


рациональных) есть положительное число:
1400

Если возводить дробь   в третью степень, то ответ будет


отрицательным, поскольку в данном случае показатель будет
нечётным числом. Правило возведения в степень остаётся
тем же, но перед выполнением этого возведения, нужно
будет поставить минус:

Здесь ответ отрицателем по причине того, что

выражение   представляет собой произведение трёх

множителей, каждый из которых равен дроби 

Сначала перемножили   и  , получили  , но затем

умножив   на   мы получим отрицательный ответ 

Пример 3. Найти значение выражения 

Выполним возведение в степень обыкновенной дроби:


1401

Далее вычислим значение получившегося выражения:

Возведение в степень десятичных дробей

При возведении в степень десятичной дроби её необходимо


заключить в скобки. Например, возведём во вторую степень
десятичную дробь 1,5

Допускается переводить десятичную дробь в обыкновенную


и возводить в степень эту обыкновенную дробь. Решим
предыдущий пример, переведя десятичную дробь в
обыкновенную:

Пример 2. Найти значение степени (−1,5)3

Показатель степени является нечётным числом. Значит


ответ будет отрицательным

Пример 3. Найти значение степени (−2,4)2


1402

Показатель степени является чётным числом. Значит ответ


будет положительным:

Задания для самостоятельного решения


Задание 1. Найдите значение выражения:

Показать решение
Задание 2. Найдите значение выражения:

Показать решение
Задание 3. Найдите значение выражения:

Показать решение
Задание 4. Найдите значение выражения:

Показать решение
Задание 5. Найдите значение выражения:

Показать решение
Задание 6. Найдите значение выражения:

Показать решение
Задание 7. Представьте в виде степени произведение:
1403

Показать решение
Задание 8. Представьте в виде степени произведение:

Показать решение
Задание 9. Представьте в виде степени произведение:

Показать решение
Задание 10. Представьте в виде степени произведение:

Показать решение
Задание 11. Представьте в виде степени произведение:

Показать решение
Задание 12. Представьте в виде степени произведение:

Показать решение
Задание 13. Представьте в виде степени частное:

Показать решение
Задание 14. Представьте в виде степени частное:

Показать решение
Задание 15. Представьте в виде степени частное:

Показать решение
1404

Задание 16. Представьте в виде степени частное:

Показать решение
Задание 17. Представьте в виде степени частное:

Показать решение

Задание 18. Представьте в виде степени частное и


найдите значение получившейся степени при x = 3 и n = 2
Показать решение
Задание 19. Представьте в виде степени частное:

Показать решение

Задание 20. Сократите дробь на c¹


Показать решение
Задание 21. Представьте в виде степени следующее
произведение:

Показать решение
Задание 22. Представьте в виде степени следующее
произведение:

Показать решение
Задание 23. Представьте в виде степени следующее
произведение:
1405

Показать решение
Задание 24. Представьте в виде степени следующее
произведение:

Показать решение
Задание 25. Представьте в виде степени следующее
произведение:

Показать решение
Задание 26. Представьте следующую степень в виде
произведения степеней:

Показать решение
Задание 27. Представьте следующую степень в виде
произведения степеней:

Показать решение
Задание 28. Представьте следующую степень в виде
произведения степеней:

Показать решение
Задание 29. Пользуясь тождественными преобразованиями
степеней, найдите значение следующего выражения:
1406

Показать решение
Задание 30. Пользуясь тождественными преобразованиями
степеней, найдите значение следующего выражения:

Показать решение
Задание 31. Пользуясь тождественными преобразованиями
степеней, найдите значение следующего выражения:

Показать решение
Задание 32. Представьте в виде степени следующее
выражение:

Показать решение
Задание 33. Представьте в виде степени следующее
выражение:

Показать решение
Задание 34. Представьте в виде степени следующее
выражение:

Показать решение
Задание 35. Представьте в виде степени следующее
выражение:
1407

Показать решение
Задание 36. Представьте в виде степени следующее
выражение:

Показать решение
Задание 37. Представьте в виде степени следующее
выражение:

Показать решение
Задание 38. Найдите значение следующего выражения:

Показать решение
Задание 39. Найдите значение следующего выражения:

Показать решение
Задание 40. Найдите значение следующего выражения:

Показать решение
Задание 41. Найдите значение следующего выражения:

Показать решение
1408

Задание 42. Найдите значение следующего выражения:

Показать решение
Задание 43. Найдите значение следующего выражения:

Показать решение
Задание 44. Найдите значение следующего выражения:

Показать решение
1409

Степень с целым показателем

Степень с целым показателем — это степень, показателем


которой является любое целое число.

В прошлом уроке мы изучили степень с натуральным


показателем. Этот вид степени тоже является степенью с
целым показателем, поскольку натуральные числа относятся
к целым числам.

Также, мы рассмотрели степень, показателем которой


является 0. Этот вид степени тоже является степенью с
целым показателем, поскольку 0 относится к целым числам.

Рассмотрим ещё один вид степени с целым показателем, а


именно показателем которой является целое отрицательное
число. Выглядят эти степени так:

2−2, 10−7, a−8

В дальнейшем любую степень с натуральным, нулевым или


целым отрицательным показателем, мы будем называть
степенью с целым показателем.

Предварительные навыки

 Действия с дробями
 Буквенные выражения
 Степень с натуральным показателем

Содержание урока

 Правило вычисления
 Тождественные преобразования
1410

 Поднятие степени из знаменателя в числитель и


наоборот
 Возведение числа 10 в целую отрицательную степень
 Представление чисел 0,1, 0,01, 0,001 в виде степени с
основанием 10
 Стандартный вид числа
 Задания для самостоятельного решения

Правило вычисления

Рассмотрим следующую последовательность степеней:

20, 21, 22, 23, 24, 25

Первая степень в этой последовательности это степень 20.


Предыдущая степень с целым показателем будет уже с
отрицательным показателем и выглядеть как 2−1.

2−1, 20, 21, 22, 23, 24, 25

А предыдущая степень с целым показателем, которая


располагается до 2−1, будет степень 2−2

2−2, 2−1, 20, 21, 22, 23, 24, 25

Продолжим эту последовательность в сторону степеней с


целыми отрицательными показателями:

2−5, 2−4, 2−3, 2−2, 2−1, 20, 21, 22, 23, 24, 25

Теперь попробуем вычислить эти степени. Степени с


натуральными показателями и степень, показателем которой
является 0, вычисляются легко:
1411

А как вычислить степени с отрицательными показателями?


Для начала немного отойдём от темы и затронем несколько
закономерностей.

В отрицательную степень число возводится немного иначе.


Следует понимать, что если при возведении в
положительную степень число увеличивается, то при
возведении в отрицательную степень это число наоборот
уменьшается.

Если мы возьмём какое-нибудь число n, и начнём


последовательно увеличивать его степень, то получим
последовательность чисел, в которой каждое число меньше
следующего в n раз.

Например, возьмём число 2. Начиная с нуля будем


последовательно увеличивать его показатель:

20, 21, 22, 23, 24, 25

Вычислим эти степени:

1, 2, 4, 8, 16, 32

Получили последовательность чисел, в которой каждое


число меньше следующего числа в 2 раза. Тогда логично
предположить, что число, располагающееся до единицы,
будет в два раза меньше единицы. Его можно получить, если
1 разделить на 2
1412

Вернёмся к нашей исходной последовательности, где мы


вычисляли степени. Получается, что степень 2−1 мы

вычислили. Она равна рациональному числу 

Предыдущее за числом должно быть в два раза меньше,

чем . Чтобы его получить разделим   на 2

Получили  . Это значение степени 2−2

Продолжая деление на 2 можно получить значения


остальных степеней с целыми отрицательными
показателями:
1413

Заметим, что в данной последовательности значения


степеней с отрицательными показателями являются
обратными числами к значениям степеней с натуральными
показателями:

К примеру, значение степени в 22 есть число 4. А значение

степени 2−2 есть число  . Числа 4 и  являются обратными


друг другу. А степени 22 и 2−2 отличаются только тем, что у
них противоположные показатели.

Можно сделать вывод, что для вычисления степени с


отрицательным показателем, нужно записать дробь, в
числителе которой единица, а в знаменателе та же самая
степень, но с противоположным показателем. Покажем это
на примере степени 2−2
1414

Вычислим степень, находящуюся в знаменателе:

Таким образом, чтобы вычислить степень вида a−n можно


воспользоваться следующим правилом:

Данное правило можно доказать, используя правило деления


степеней с одинаковыми основаниями. Допустим,
потребовалось вычислить выражение 23 : 25. Запишем это
деление в виде дроби

Воспользуемся правилом деления степеней с одинаковыми


основаниями:

Получили степень с отрицательным показателем 2−2. Ранее

мы выяснили, что её значение равно  . Чтобы убедиться в

этом, попробуем вычислить выражение   как обычно, не


используя правило деления степеней:
1415

Получили рациональное число  . Сократим его на 8. Тогда

получим 

Пример 2. Найти значение выражения 9−2

Воспользуемся правилом вычисления степени с целым


отрицательным показателем:

Пример 3. Найти значение выражения 3−3

Следует упомянуть, что правило  работает только


тогда, когда a ≠ 0.

Действительно, если a будет равным нулю, то в знаменателе


получим 0, а на нуль делить нельзя.

Пример 4. Найти значение выражения 


1416

Пример 5. Найти значение выражения 

При возведении обыкновенных дробей в отрицательную

степень, можно пользоваться формулой  . Решим


предыдущие два примера с помощью этой формулы:

Желательно уметь возводить обыкновенную дробь в


отрицательную степень как с помощью формулы, так и без
неё.

Тождественные преобразования

Все тождественные преобразования, которые мы


рассматривали при изучении степени с натуральным
показателем, сохраняются и для степеней с целыми
отрицательными показателями.
1417

Например, чтобы представить выражение 2−1 × 2−3 в виде


степени, можно воспользоваться основным свойством
степени:

2−1 × 2−3 = 2−1 + (−3) = 2−4

Пример 2. Найти значение выражения 5−15 × 516

Воспользуемся основным свойством степени:

5−15 × 516 = 5−15 + 16 = 51 = 5

или:

Видим, что первый вариант решения намного проще и


удобнее.

Пример 3. Найти значение выражения (10−4)−1

Воспользуемся правилом возведения степени в степень:

(10−4)−1 = 10−4 × (−1) = 104 = 10000

Пример 4. Найти значение выражения 

Представим число основание 10 в виде произведения 2 × 5.


Тогда числитель примет вид (2 × 5)−6
1418

В числителе применим правило возведения в степень


произведения:

Сократим получившуюся дробь на 5−6

Вычислим степень 2−6

Поднятие степени из знаменателя в числитель и наоборот

Если знаменатель дробного выражения содержит степень, то


данную степень можно поднять в числитель, изменив знак
показателя этой степени на противоположный. Значение
выражения при этом не меняется. Данное преобразование
иногда используется при упрощении выражений.

Рассмотрим следующее равенство:

Данное равенство является верным, поскольку выражение   


равно 20, а любое число в нулевой степени есть единица.
1419

Попробуем поднять степень 22 из знаменателя в числитель,


изменив знак показателя этой степени на противоположный.
При этом, поднятую степень и ту степень, которая
располагалась в числителе, соединим знаком умножения:

Получили выражение 22 × 2−2. Чтобы его вычислить,


воспользуемся основным свойством степени:

22 × 2−2 = 22 + (−2) = 20 = 1

Получился тот же результат, что и раньше. Значит значение


выражения не изменилось. Как это работает?

Если в равенстве   поменять местами левую и правую

часть, то получим равенство  . Это позволяет заменять

в выражениях дробь вида   на тождественно равное ей


выражение a−n.

Теперь представим выражение  в виде произведения  .


То есть заменим деление умножением. Напомним, что при
замене деления умножением, делимое умножают на число,
обратное делителю. А обратное делителю число в данном

случае это дробь 


1420

Теперь воспользуемся правилом  . В

произведении   заменим дробь   на тождественно


равное ей выражение 2−2

Далее, как и раньше применяем основное свойство степени:

Получился тот же результат 1.

Таким же образом можно опустить степень из числителя в


знаменатель, изменив знак показателя этой степени на
противоположный.

Рассмотрим выражение  . Чтобы найти его значение,


воспользуемся правилом деления степеней с одинаковыми

основаниями. В результате получим

Теперь попробуем решить этот пример, опустив степень 2−2


из числителя в знаменатель, изменив знак показателя этой
степени на противоположный. При этом, опущенную
степень 2−2 и ту степень, которая располагалась в
1421

знаменателе, соединим знаком умножения. А в числителе


останется единица:

Дальнейшее вычисление не составит особого труда:

Как и в прошлом примере выражение   представимо в виде

произведения 

Этим и объясняется появление единицы в числителе, после


того как степень 2−2 была опущена в знаменатель.

Переносимых в знаменатель либо в числитель степеней

может быть несколько. Например, знаменатель дроби   


содержит степени 32, a3, b4. Перенесём эти степени в
числитель, изменив знаки их показателей на
противоположные. В результате получим
выражение 3−2a−3b−4.

Пример 2. Поднять степени из знаменателя дроби   в


числитель
1422

Пример 3. Поднять степени из знаменателя дроби   в


числитель

Пример 4. Поднять степень из знаменателя дроби   в


числитель

Пример 5. Опустить степень из числителя дроби   в


знаменатель

Пример 6. Степень из числителя дроби   опустить в


знаменатель, а степень из знаменателя поднять в числитель
1423

Представлять дробь   в виде произведения   вовсе


не обязательно. Если пропустить эту запись, то данный
пример можно решить короче:

Пример 7. В дроби  перенести из знаменателя в


числитель только те степени, которые имеют отрицательные
показатели:

Пример 8. Представить произведение 3x−5 в виде дроби, не


содержащей степени с отрицательным показателем.

Перепишем произведение 3x−5 с помощью знака умножения:

3 × x−5

Сомножитель 3 оставим без изменений, а сомножитель x−5

заменим на тождественно равную ему дробь 


1424

Теперь согласно правилу умножения целого числа на дробь,

умножим множитель 3 на числитель дроби  . В результате

образуется дробь 

Пример 9. Представить произведение 3(x + y)−4 в виде


дроби, не содержащей степени с отрицательным
показателем.

Выражение состоит из сомножителей 3 и (x + y)−4.


Сомножитель 3 оставим без изменений, а сомножитель

(x + y)−4 заменим на тождественно равную ему дробь 

Теперь умножим множитель 3 на числитель дроби  .В

результате образуется дробь 

Пример 10. Представить дробь   в виде произведения.


1425

Чтобы решить этот пример, достаточно поднять степень x2 в


числитель, изменив знак показателя этой степени на
противоположный:

Как и в прошлых примерах дробь   можно было представить

в виде произведения  . Затем воспользовавшись

правилом , заменить сомножитель   на тождественно


равный ему сомножитель x−2.

Пример 11. Представить дробь   в виде произведения.

Пример 12. Найти значение выражения 

Поднимем степень 2−3 из знаменателя в числитель, а степень


10−2 из числителя опустим в знаменатель:
1426

Вычислим значения степеней, содержащихся в числителе и в


знаменателе:

Сократим полученную дробь на 25. Тогда останется дробь  ,


значение которой равно 2.

А если бы мы не подняли степень 2−3 в числитель, и степень


10−2 не опустили в знаменатель, а стали вычислять каждую
степень по отдельности, то получили бы не очень компактное
решение:

Возведение числа 10 в целую отрицательную степень

Число 10 в отрицательную степень возводится таким же


образом, как и другие числа. Например:
1427

Замечаем, что количество нулей, которые получаются в


ответе равны модулю показателя исходной степени.
Например, в степени 10−2 модуль показателя равен 2. Это
значит, что в ответе будет содержаться два нуля. Так оно и
есть:

Чтобы возвести число 10 в отрицательную степень,


нужно перед единицей записать количество нулей,
равное модулю показателя исходной степени.

При этом после первого нуля следует поставить запятую.


Примеры:

Представление чисел 0,1, 0,01, 0,001 в виде степени с


основанием 10

Чтобы представить числа 0,1, 0,01, 0,001 в виде степени с


основанием 10, нужно записать основание 10, и в качестве
показателя указать отрицательный показатель, модуль
которого равен количеству нулей исходного числа.

Представим число 0,1 в виде степени с основанием 10.


Видим, что в числе 0,1 один нуль. Значит, число 0,1 в виде
степени с основанием 10 будет представлено как 10−1.
Показатель степени 10−1 равен −1. Модуль этого показателя
равен количеству нулей в числе 0,1
1428

0,1 = 10−1

Число 0,1 это результат деления  , а эта дробь есть


значение степени 10−1.

Пример 2. Представить число 0,01 в виде степени с


основанием 10.

В числе 0,01 два нуля. Значит, число 0,01 в виде степени с


основанием 10 будет представлено как 10−2. Показатель
степени 10−2 равен −2. Модуль этого показателя равен
количеству нулей в числе 0,01

0,01 = 10−2

Число 0,01 это результат деления  , то есть , а эта дробь


есть значение степени 10−2.

Пример 3. Представить число 0,001 в виде степени с


основанием 10.

0,001 = 10−3

Пример 4. Представить число 0,0001 в виде степени с


основанием 10.

0,0001 = 10−4
1429

Пример 5. Представить число 0,00001 в виде степени с


основанием 10.

0,00001 = 10−5

Стандартный вид числа

Запишем число 2 000 000 в виде произведения числа 2 и


1 000 000

2 × 1 000 000

Сомножитель 1 000 000 можно заменить на степень 106

2 × 106

Такой вид записи называют стандартным видом числа.


Стандартный вид числа позволяет записывать в компактном
виде как большие, так и маленькие числа.

Например, маленькое число 0,005 можно записать в виде


произведения числа 5 и десятичной дроби 0,001.

5 × 0,001

Десятичную дробь 0,001 можно заменить на степень с 10−3

5 × 10−3

Значит, число 0,005 в стандартном виде будет выглядеть


как 5 × 10−3

0,005 = 5 × 10−3

По стандартному виду числа можно вычислить изначальное


число. Так, при записи числа 2 000 000 в стандартном виде,
1430

мы получили произведение 2 × 106. Если вычислить это


произведение, то снова получим 2 000 000

2 × 106 = 2 × 1 000 000 = 2 000 000

А при записи числа 0,005 в стандартном виде мы получили


произведение 5 × 10−3. Если вычислить это произведение, то
получим 0,005

То есть записывая число в стандартном виде нужно


записывать его так, чтобы сохранить его изначальное
значение.

Стандартным видом числа называют запись вида


a × 10n, где 1 ≤ a < 10 и n — целое число.

Число а это исходное число, которое надо записать в


стандартном виде. Оно должно удовлетворять неравенству
1 ≤ a < 10. Чаще всего исходное число надо приводить к
виду, при котором неравенство 1 ≤ a < 10 становится
верным.

Например, представим число 12 в стандартном виде. Для


начала проверим становится ли верным
неравенство 1 ≤ a < 10 при подстановке числа 12 вместо а

1 ≤ 12 < 10

Неравенство верным не становится. Чтобы сделать


неравенство верным, приведём число 12 к виду, при котором
оно удовлетворяло бы данному неравенству. Для этого
передвинем в числе 12 запятую влево на одну цифру:
1431

1,2

Число 12 обратилось в число 1,2. Это число будет


удовлетворять неравенству 1 ≤ a < 10

1 ≤ 1,2 < 10

Теперь наша задача состоит в том, чтобы записать


произведение a × 10n. С числом а мы разобрались — этим
числом у нас будет 1,2. А как подобрать степень с
основанием 10?

После переноса запятой на одну цифру влево, число 12


утратило своё изначальное значение. Запятая на одну цифру
влево двигается тогда, когда число делят на 10. А чтобы
восстановить изначальное значение числа запятую нужно
передвинуть обратно в правую сторону на одну цифру, то
есть умножить число 1,2 на 10.

Значит, чтобы записать число 12 в стандартном виде, нужно


представить его в виде произведения 1,2 × 10¹

12 = 1,2 × 10¹

Пример 2. Записать число 0,5 в стандартном виде.

Число 0,5 не удовлетворяет неравенству 1 ≤ a< 10, поэтому


передвинем запятую в этом числе на одну цифру вправо. В
результате получим число 5, которое удовлетворяет
неравенству 1 ≤ a< 10.

Теперь запишем произведение вида a × 10n. Число a в


данном случае это 5. А степень с основанием 10 надо
1432

выбрать так, чтобы произведение a × 10n стало равным


числу 0,5. Число 0,5 получится если умножить число 5 на
множитель 0,1, который представим в виде степени 10−1. В
результате получим следующую запись:

0,5 = 5 × 10−1

Пример 3. Записать число 652 000 в стандартном виде.

Число 652 000 не удовлетворяет неравенству 1 ≤ a< 10,


поэтому передвинем запятую в этом числе на пять цифр
влево. В результате получим число 6,52000 которое
удовлетворяет неравенству 1 ≤ a< 10.

Теперь запишем произведение вида a × 10n. Число a в


данном случае это 6,52000. А степень с основанием 10 надо
выбрать так, чтобы произведение a × 10n стало равным
числу 652 000. Число 652 000 получится если число 6,52000
умножить на 100 000, а это есть степень 105. В результате
получим следующую запись:

652 000 = 6,52000 × 105

Нули в конце десятичной дроби 6,52000 можно отбросить.


Тогда получим более компактную запись:

652 000 = 6,52 × 105

Пример 5. Записать число 1 024 000 в стандартном виде.

Число 1 024 000 не удовлетворяет неравенству 1 ≤ a< 10,


поэтому передвинем запятую в этом числе на шесть цифр
1433

влево. В результате получим число 1,024000 которое


удовлетворяет неравенству 1 ≤ a< 10.

Теперь запишем произведение вида a × 10n. Число a в


данном случае это 1,024000 . А степень с основанием 10
надо выбрать так, чтобы произведение a × 10n было равно
изначальному числу 1 024 000. Число 1 024 000 получится
если число 1,024000 умножить на 1 000 000, а это есть
степень 106. В результате получим следующую запись:

1 024 000 = 1,024000 × 106

Нули в конце десятичной дроби 1,024000 можно отбросить:

1 024 000 = 1,024 × 106

Отбрасывать можно только те нули, которые располагаются


в конце, и после которых нет других цифр, бóльших нуля. В
приведённом примере были отброшены только три нуля, а
нуль располагавшийся между запятой и цифрой 2 был
сохранен, несмотря на то, что он тоже располагался после
запятой.

Пример 6. Записать число 0,000325 в стандартном виде.

Передвинем в данном числе запятую так, чтобы оно


удовлетворяло неравенству 1 ≤ a< 10. В результате получим
число 3,25

Теперь запишем произведение вида a × 10n. Число a в


данном случае это 3,25. А степень с основанием 10 надо
выбрать так, чтобы произведение a × 10n было равно
изначальному числу 0,000325. Число 0,000325 получится
1434

если число 3,25 умножить на множитель 0,0001 который


представим в виде степени 10−4. В результате получим
следующую запись:

0,000325 = 3,25 × 10−4

Задания для самостоятельного решения


Задание 1. Вычислите степень 3−2
Показать решение
Задание 2. Вычислите степень (−3)−2
Показать решение
Задание 3. Вычислите степень −3−2
Показать решение
Задание 4. Вычислите степень (−1)−9
Показать решение

Задание 5. Вычислите степень


Показать решение

Задание 6. Вычислите степень


Показать решение
Задание 7. Вычислите степень −(−2)−3
Показать решение

Задание 8. Вычислите степень


1435

Показать решение
Задание 9. Найдите значение выражения 8 × 4−3
Показать решение
Задание 10. Найдите значение выражения 18 × (−9)−1
Показать решение
Задание 11. Найдите значение выражения 2−3 − (−2)−4
Показать решение

Задание 12. Найдите значение выражения


Показать решение
Задание 13. Представить произведение a−4b в виде дроби, не
содержащей степени с отрицательным показателем.
Показать решение
Задание 14. Представить произведение 7xy−3 в виде дроби,
не содержащей степени с отрицательным показателем.
Показать решение
Задание 15. Представить произведение 6(xy)−6 в виде дроби,
не содержащей степени с отрицательным показателем.
Показать решение
Задание 16. Представить произведение x−1y−2 в виде дроби,
не содержащей степени с отрицательным показателем.
Показать решение
Задание 17. Представить произведение 9a−1(a − b)−2 в виде
дроби, не содержащей степени с отрицательным
показателем.
1436

Показать решение

Задание 18. Представьте дробь  в виде произведения.


Показать решение

Задание 19. Представьте дробь  в виде произведения.


Показать решение

Задание 20. Представьте дробь  в виде произведения.


Показать решение

Задание 21. Представьте дробь  в виде произведения.


Показать решение

Задание 22. Представьте дробь  в виде произведения.


Показать решение

Задание 23. Представьте дробь  в виде произведения.


Показать решение

Задание 24. Представьте дробь  в виде произведения.


Показать решение

Задание 25. Представьте дробь  в виде произведения.


Показать решение
1437

Задание 26. Представьте дробь  в виде произведения.


Показать решение
Задание 27. Представьте число 3 000 000 в стандартном
виде.
Показать решение
Задание 28. Представьте число 0,35 в стандартном виде.
Показать решение
Задание 29. Представьте число 21,56 в стандартном виде.
Показать решение
Задание 30. Представьте число 0,000008 в стандартном
виде.
Показать решение
Задание 31. Представьте число 0,000335 в стандартном
виде.
Показать решение

Задание 32. Найдите значение выражения .


Показать решение

Задание 33. Найдите значение выражения .


Показать решение

Задание 34. Найдите значение выражения .


Показать решение
1438

Задание 35. Представьте в виде степени выражение .


Показать решение

Задание 36. Представьте в виде степени выражение .


Показать решение

Задание 37. Представьте в виде степени выражение .


Показать решение

Задание 38. Представьте в виде степени выражение .


Показать решение
1439

Периметр, площадь и объём

Данный материал содержит геометрические фигуры с


измерениями. Приведённые измерения являются
приблизительными и могут не совпадать с измерениями в
реальной жизни.
Содержание урока

 Периметр геометрической фигуры


 Площадь геометрической фигуры
 Площадь прямоугольника
 Площадь квадрата
 Обозначения
 Перевод единиц измерения площади
 Единицы измерения площади земельных участков
 Прямоугольный параллелепипед и куб
 Объём геометрической фигуры
 Перевод единиц измерения объёма
 Таблица квадратов
 Таблица кубов
 Задания для самостоятельного решения

Периметр геометрической фигуры

Периметр геометрической фигуры — это сумма всех её


сторон. Чтобы вычислить периметр, нужно измерить каждую
сторону и сложить результаты измерений.

Вычислим периметр следующей фигуры:


1440

Это прямоугольник. Детальнее мы поговорим об этой фигуре


позже. Сейчас просто вычислим периметр этого
прямоугольника. Длина его равна 9 см, а ширина 4 см.

У прямоугольника противоположные стороны равны. Это


видно на рисунке. Если длина равна 9 см, а ширина равна 4
см, то противоположные стороны будут равны 9 см и 4 см
соответственно:

Найдём периметр. Для этого сложим все стороны.


Складывать их можно в любом порядке, поскольку от
перестановки мест слагаемых сумма не меняется. Периметр
часто обозначается заглавной латинской буквой P (англ.
perimeters). Тогда получим:

P = 9 см + 4 см + 9 см + 4 см = 26 см.

Поскольку у прямоугольника противоположные стороны


равны, нахождение периметра записывают короче —
1441

складывают длину и ширину, и умножают её на 2, что будет


означать «повторить длину и ширину два раза»

P = 2 × (9 + 4) = 18 + 8 = 26 см.

Квадрат это тот же прямоугольник, но у которого все стороны


равны. Например, найдём периметр квадрата со стороной 5
см. Фразу «со стороной 5 см» нужно понимать как «длина
каждой стороны квадрата равна 5 см»

Чтобы вычислить периметр, сложим все стороны:

P = 5 см + 5 см + 5 см + 5 см = 20 см

Но поскольку все стороны равны, вычисление периметра


можно записать в виде произведения. Сторона квадрата
равна 5 см, и таких сторон 4. Тогда эту сторону, равную 5 см
нужно повторить 4 раза

P = 5 см × 4 = 20 см

Площадь геометрической фигуры

Площадь геометрической фигуры — это число, которое


характеризует размер данной фигуры.
1442

Следует уточнить, что речь в данном случае идёт о площади


на плоскости. Плоскостью в геометрии называют любую
плоскую поверхность, например: лист бумаги, земельный
участок, поверхность стола.

Площадь измеряется в квадратных единицах. Под


квадратными единицами подразумевают квадраты, стороны
которых равны единице. Например, 1 квадратный сантиметр,
1 квадратный метр или 1 квадратный километр.

Измерить площадь какой-нибудь фигуры означает выяснить


сколько квадратных единиц содержится в данной фигуре.

Например, площадь следующего прямоугольника равна трём


квадратным сантиметрам:

Это потому что в данном прямоугольнике содержится три


квадрата, каждый из которых имеет сторону, равную одному
сантиметру:

Справа представлен квадрат со стороной 1 см (он в данном


случае является квадратной единицей). Если посмотреть
сколько раз этот квадрат входит в прямоугольник,
представленный слева, то обнаружим, что он входит в него
три раза.
1443

Следующий прямоугольник имеет площадь, равную шести


квадратным сантиметрам:

Это потому что в данном прямоугольнике содержится шесть


квадратов, каждый из которых имеет сторону, равную одному
сантиметру:

Допустим, потребовалось измерить площадь следующей


комнаты:
1444

Определимся в каких квадратах будем измерять площадь. В


данном случае площадь удобно измерить в квадратных
метрах:

Итак, наша задача состоит в том, чтобы определить сколько


таких квадратов со стороной 1 м содержится в исходной
комнате. Заполним этим квадратом всю комнату:

Видим, что квадратный метр содержится в комнате 12 раз.


Значит, площадь комнаты составляет 12 квадратных метров.

Площадь прямоугольника

В предыдущем примере мы вычислили площадь комнаты,


последовательно проверив сколько раз в ней содержится
1445

квадрат, сторона которого равна одному метру. Площадь


составила 12 квадратных метров.

Комната представляла собой прямоугольник. Площадь


прямоугольника можно вычислить перемножив его длину и
ширину.

Чтобы вычислить площадь прямоугольника, нужно


перемножить его длину и ширину.

Вернёмся к предыдущему примеру. Допустим, мы измерили


длину комнаты рулеткой и оказалось, что длина составила 4
метра:

Теперь измерим ширину. Пусть она составила 3 метра:


1446

Умножим длину (4 м) на ширину (3 м).

4 × 3 = 12

Как и в прошлый раз получаем двенадцать квадратных


метров. Это объясняется тем, что измерив длину, мы тем
самым узнаём сколько раз можно уложить в эту длину
квадрат со стороной, равной одному метру. Уложим четыре
квадрата в эту длину:
1447

Затем мы определяем сколько раз можно повторить эту


длину с уложенными квадратами. Это мы узнаём, измерив
ширину прямоугольника:
1448

Площадь квадрата

Квадрат это тот же прямоугольник, но у которого все стороны


равны. Например, на следующем рисунке представлен
квадрат со стороной 3 см. Фраза «квадрат со стороной 3
см» означает, что все стороны равны 3 см
1449

Площадь квадрата вычисляется таким же образом, как и


площадь прямоугольника — длину умножают на ширину.

Вычислим площадь квадрата со стороной 3 см. Умножим


длину 3 см на ширину 3 см

3×3=9

В данном случае требовалось узнать сколько квадратов со


стороной 1 см содержится в исходном квадрате. В исходном
квадрате содержится девять квадратов со стороной 1 см.
Действительно, так оно и есть. Квадрат со стороной 1 см,
входит в исходный квадрат девять раз:

Умножив длину на ширину, мы получили выражение 3 × 3, а


это есть произведение двух одинаковых множителей, каждый
из которых равен 3. Иными словами выражение 3 × 3
представляет собой вторую степень числа 3. А значит
1450

процесс вычисления площади квадрата можно записать в


виде степени 32.

Поэтому вторую степень числа называют квадратом числа.


При вычислении второй степени числа a, человек тем самым
находит площадь квадрата со стороной a. Операцию
возведения числа во вторую степень по другому называют
возведением в квадрат.

Обозначения

Площадь обозначается заглавной латинской буквой S (англ.


Square — квадрат). Тогда площадь квадрата со стороной a
см будет вычисляться по следующему правилу

S = a2

где a — длина стороны квадрата. Вторая степень указывает


на то, что происходит перемножение двух одинаковых
сомножителей, а именно длины и ширины. Ранее было
сказано, что у квадрата все стороны равны, а значит равны
длина и ширина квадрата, выраженные через букву a.

Если задача состоит в том, чтобы определить сколько


квадратов стороной 1 см содержится в исходном квадрате, то
в качестве единиц измерения площади нужно указывать см2.
Это обозначение заменяет словосочетание «квадратный
сантиметр».

Например, вычислим площадь квадрат со стороной 2 см.


1451

Значит, квадрат со стороной 2 см, имеет площадь, равную


четырём квадратным сантиметрам:

Если задача состоит в том, чтобы определить сколько


квадратов со стороной 1 м содержится в исходном квадрате,
то в качестве единиц измерения нужно указывать м2. Это
обозначение заменяет словосочетание «квадратный метр».

Вычислим площадь квадрата со стороной 3 метра


1452

Значит, квадрат со стороной 3 м, имеет площадь равную


девяти квадратным метрам:

Аналогичные обозначения используются при вычислении


площади прямоугольника. Но длина и ширина
прямоугольника могут быть разными, поэтому они
обозначаются через разные буквы, например a и b. Тогда
площадь прямоугольника, длиной a и шириной b
вычисляется по следующему правилу:

S=a×b

Как и в случае с квадратом, единицами измерения площади


прямоугольника могут быть см2, м2, км2. Эти обозначения
заменяют словосочетания «квадратный сантиметр»,
«квадратный метр», «квадратный километр»
соответственно.

Например, вычислим площадь прямоугольника, длиной 6 см


и шириной 3 см
1453

Значит, прямоугольник длиной 6 см и шириной 3 см имеет


площадь, равную восемнадцати квадратным сантиметрам:

В качестве единицы измерения допускается использовать


словосочетание «квадратных единиц». Например, запись
S = 3 кв.ед означает, что площадь квадрата или
прямоугольника равна трём квадратам, каждый из которых
имеет единичную сторону (1 см, 1 м или 1 км).

Перевод единиц измерения площади

Единицы измерения площади можно переводить из одной


единицы измерения в другую. Рассмотрим несколько
примеров:

Пример 1. Выразить 1 квадратный метр в квадратных


сантиметрах.
1454

1 квадратный метр это квадрат со стороной 1 м. То есть все


четыре стороны имеют длину, равную одному метру.

Но 1 м = 100 см. Тогда все четыре стороны тоже имеют


длину, равную 100 см

Вычислим новую площадь этого квадрата. Умножим длину


100 см на ширину 100 см или возведём в квадрат число 100

S = 1002 = 10 000 см2


1455

Получается, что на один квадратный метр приходится десять


тысяч квадратных сантиметров.

1 м2  = 10 000 см2

Это позволяет в будущем умножить любое количество


квадратных метров на 10 000 и получить площадь,
выраженную в квадратных сантиметрах.

Чтобы перевести квадратные метры в квадратные


сантиметры, нужно количество квадратных метров
умножить на 10 000.

А чтобы перевести квадратные сантиметры в квадратные


метры, нужно наоборот количество квадратных сантиметров
разделить на 10 000.

Например, переведём 100 000 см2 в квадратные метры.


Рассуждать в этом случае можно так: «если 10 000 см2 это
один квадратный метр, то сколько раз 100 000 см2 будут
содержать по 10 000 см2»

100 000 см2 : 10 000 см2 = 10 м2

Другие единицы измерения можно переводить таким же


образом. Например, переведём 2 км2 в квадратные метры.

Один квадратный километр это квадрат со стороной 1 км. То


есть все четыре стороны имеют длину, равную одному
километру. Но 1 км = 1000 м. Значит, все четыре стороны
квадрата также равны 1000 м. Найдём новую площадь
квадрата, выраженную в квадратных метрах. Для этого
умножим длину 1000 м на ширину 1000 м или возведём в
квадрат число 1000
1456

S = 10002 = 1 000 000 м2

Получается, что на один квадратный километр приходится


один миллион квадратных метров:

1 км2  = 1 000 000 м2

Это позволяет в будущем умножить любое количество


квадратных километров на 1 000 000 и получить площадь,
выраженную в квадратных метрах.

Чтобы перевести квадратные километры в квадратные


метры, нужно количество квадратных километров
умножить на 1 000 000.

Итак, вернёмся к нашей задаче. Требовалось перевести 2 км2


в квадратные метры. Умножим 2 км2 на 1 000 000

2 км2 × 1 000 000 = 2 000 000 м2

А чтобы перевести квадратные метры в квадратные


километры, нужно наоборот количество квадратных метров
разделить на 1 000 000.

Например, переведём 3 500 000 м2 в квадратные километры.


Рассуждать в этом случае можно так: «если 1 000 000 м2 это
один квадратный километр, то сколько раз 3 500 000 м2
будут содержать по 1 000 000 м2»

3 500 000 м2 : 1 000 000 м2 = 3,5 км2

Пример 2. Выразить 7 м2 в квадратных сантиметрах.

Умножим 7 м2 на 10 000
1457

7 м2 = 7 м2 × 10 000 = 70 000 см2

Пример 3. Выразить 5 м2 13 см2 в квадратных сантиметрах.

5 м2 13 см2 = 5 м2 × 10 000 + 13 см2 = 50 013 см2

Пример 4. Выразить 550 000 см2 в квадратных метрах.

Узнаем сколько раз 550 000 см2 содержит по 10 000 см2. Для


этого разделим 550 000 см2 на 10 000 см2

550 000 см2 : 10 000 см2 = 55 м2

Пример 5. Выразить 7 км2 в квадратных метрах.

Умножим 7 км2 на 1 000 000

7 км2 × 1 000 000 = 7 000 000 м2

Пример 6. Выразить 8 500 000 м2 в квадратных километрах.

Узнаем сколько раз 8 500 000 м2 содержит по 1 000 000 м2.


Для этого разделим 8 500 000 м2 на 1 000 000 м2

8 500 000 м2 × 1 000 000 м2 = 8,5 км2

Единицы измерения площади земельных участков

Площади небольших земельных участков удобно измерять в


квадратных метрах.
1458

Площади более крупных земельных участков измеряются в


арах и гектарах.

Ар (сокращённо: a) — это площадь равная ста квадратным


метрам (100 м2). В виду частого распространения такой
площади (100 м2) она стала использоваться, как отдельная
единица измерения.

Например, если сказано что площадь какого-нибудь поля


составляет 3 а, то нужно понимать, что это три квадрата
площадью 100 м2 каждый, то есть:

3 а = 100 м2 × 3 = 300 м2

В народе ар часто называют соткой, поскольку ар равен


квадрату, площадью 100 м2. Примеры:

1 сотка = 100 м2

2 сотки = 200 м2

10 соток = 1000 м2

Гектар (сокращенно: га) — это площадь, равная 10 000 м2.


Например, если сказано что площадь какого-нибудь леса
составляет 20 гектаров, то нужно понимать, что это двадцать
квадратов площадью 10 000 м2 каждый, то есть:

20 га = 10 000 м2 × 20 = 200 000 м2


1459

Прямоугольный параллелепипед и куб

Прямоугольный параллелепипед — это геометрическая


фигура, состоящая из грáней, рёбер и вершин. На рисунке
показан прямоугольный параллелепипед:

Желтым цветом показаны грáни параллелепипеда, чёрным


цветом — рёбра, красным — вершины.

Прямоугольный параллелепипед обладает длиной, шириной


и высотой. На рисунке показано где длина, ширина и высота:

Параллелепипед, у которого длина, ширина и высота равны


между собой, называется кубом. На рисунке показан куб:
1460

Объём геометрической фигуры

Объём геометрической фигуры — это число, которое


характеризует вместимость данной фигуры.

Объём измеряется в кубических единицах. Под кубическими


единицами подразумевают кубы длиной 1, шириной 1 и
высотой 1. Например, 1 кубический сантиметр или 1
кубический метр.

Измерить объём какой-нибудь фигуры означает выяснить


сколько  кубических единиц вмещается в данную фигуру.

Например, объём следующего прямоугольного


параллелепипеда равен двенадцати кубическим
сантиметрам:
1461

Это потому что в данный параллелепипед вмещается


двенадцать кубов длиной 1 см, шириной 1 см и высотой 1 см:

Объём обозначается заглавной латинской буквой V. Одна из


единиц измерения объема это кубический сантиметр (см3).
Тогда объём V рассмотренного нами параллелепипеда равен
12 см3

V = 12 см3

Объём любого параллелепипеда вычисляют следующим


образом: перемножают его длину, ширину и высоту .

Объём прямоугольного параллелепипеда равен


произведению его длины, ширины и высоты.

V = abc

где, a — длина, b — ширина, c — высота

Так, в предыдущем примере мы визуально определили, что


объём параллелепипеда равен 12 см3. Но можно измерить
длину, ширину и высоту данного параллелепипеда и
перемножить результаты измерений. Мы получим тот же
результат
1462

Объём куба вычисляется таким же образом, как и объём


прямоугольного параллелепипеда — перемножают длину,
ширину и высоту.

Например, вычислим объём куба, длина которого 3 см. У


куба длина, ширина и высота равны между собой. Если
длина равна 3 см, то равны этим же трём сантиметрам
ширина и высота куба:

Перемножаем длину, ширину, высоту и получаем объём,


равный двадцати семи кубическим сантиметрам:

V = 3 × 3 × 3 = 27 см³


1463

Действительно, в исходный куб вмещается 27 кубиков


длиной 1 см

При вычислении объёма данного куба мы перемножили


длину, ширину и высоту. Получилось произведение 3 × 3 × 3.
Это есть произведение трёх сомножителей, каждый из
которых равен 3. Иными словами, произведение 3 × 3 × 3
является третьей степенью числа 3 и может быть записано в
виде 33.

V = 33 = 27 см3

Поэтому третью степень числа называют кубом числа. При


вычислении третьей степени числа a, человек тем самым
находит объём куба, длиной a. Операцию возведения числа
в третью степень по другому называют возведением в куб.

Таким образом, объём куба вычисляется по следующему


правилу:

V = a3

Где a — длина куба.


1464

Кубический дециметр. Кубический метр

Не все объекты нашего мира удобно измерять в кубических


сантиметрах. Например, объём комнаты или дома удобнее
измерять в кубических метрах (м3). А объём бака, аквариума
или холодильника удобнее измерять в кубических
дециметрах (дм3).

Другое название одного кубического дециметра – один литр.

1 дм3 = 1 литр

Перевод единиц измерения объёма

Единицы измерения объёма можно переводить из одной


единицы измерения в другую. Рассмотрим несколько
примеров:

Пример 1. Выразить 1 кубический метр в кубических


сантиметрах.

Один кубический метр это куб со стороной 1 м. Длина,


ширина и высота этого куба равны одному метру.
1465

Но 1 м = 100 см. Значит, длина, ширина и высота тоже равны


100 см

Вычислим новый объём куба, выраженный в кубических


сантиметрах. Для этого перемножим его длину, ширину и
высоту. Либо возведём число 100 в куб:

V = 1003 = 1 000 000 см3

Получается, что на один кубический метр приходится один


миллион кубических сантиметров:

1 м3  = 1 000 000 см3

Это позволяет в будущем умножить любое количество


кубических метров на 1 000 000 и получить объём,
выраженный в кубических сантиметрах.

Чтобы перевести кубические метры в кубические


сантиметры, нужно количество кубических метров
умножить на 1 000 000.
1466

А чтобы перевести кубические сантиметры в кубические


метры, нужно наоборот количество кубических сантиметров
разделить на 1 000 000.

Например, переведём 300 000 000 см3 в кубические метры.


Рассуждать в этом случае можно так: «если 1 000 000 см3
это один кубический метр, то сколько раз 300 000 000
см3 будут содержать по 1 000 000 см3»

300 000 000 см3 : 1 000 000 см3 = 300 м3

Пример 2. Выразить 3 м3 в кубических сантиметрах.

Умножим 3 м3 на 1 000 000

3 м3 × 1 000 000 = 3 000 000 см3

Пример 3. Выразить 60 000 000 см3 в кубических метрах.

Узнаем сколько раз 60 000 000 см3 содержит по 1 000 000


см3. Для этого разделим 60 000 000 см3 на 1 000 000 см3

60 000 000 см3 : 1 000 000 см3 = 60 м3

Вместимость бака, банки или канистры измеряют в литрах.


Литр это тоже единица измерения объема. Один литр равен
одному кубическому дециметру.

1 литр = 1 дм3

Например, если вместимость банки составляет 1 литр, это


значит что объём этой банки составляет 1 дм3. При решении
1467

некоторых задач может быть полезным умение переводить


литры в кубические дециметры и наоборот. Рассмотрим
несколько примеров.

Пример 1. Перевести 5 литров в кубические дециметры.

Чтобы перевести 5 литров в кубические дециметры,


достаточно умножить 5 на 1

5 л × 1 = 5 дм3

Пример 2. Перевести 6000 литров в кубические метры.

Шесть тысяч литров это шесть тысяч кубических дециметров:

6000 л × 1 = 6000 дм3

Теперь переведём эти 6000 дм3 в кубические метры.

Длина, ширина и высота одного кубического метра равны 10


дм

Если вычислить объём этого куба в дециметрах, то получим


1000 дм3
1468

V = 103= 1000 дм3

Получается, что одна тысяча кубических дециметров


соответствует одному кубическому метру. А чтобы
определить сколько кубических метров соответствуют шести
тысячамл кубических дециметров, нужно узнать сколько раз
6 000 дм3 содержит по 1 000 дм3

6 000 дм3 : 1 000 дм3 = 6 м3

Значит, 6000 л = 6 м3.

Таблица квадратов

В жизни часто приходиться находить площади различных


квадратов. Для этого каждый раз требуется возводить
исходное число во вторую степень.

Квадраты первых 99 натуральных чисел уже вычислены и


занесены в специальную таблицу, называемую таблицей
квадратов.
1469

Первая строка данной таблицы (цифры от 0 до 9) это


единицы исходного числа, а первый столбец (цифры от 1 до
9) это десятки исходного числа.

Например, найдём квадрат числа 24 по данной таблице.


Число 24 состоит из цифр 2 и 4. Точнее, число 24 состоит из
двух десятков и четырёх единиц.

Итак, выбираем цифру 2 в первом столбце таблицы (столбце


десятков), а цифру 4 выбираем в первой строке (строке
единиц). Затем, двигаясь вправо от цифры 2 и вниз от цифры
4, найдём точку пересечения. В результате окажемся на
позиции, где располагается число 576. Значит, квадрат числа
24 есть число 576

242 = 576
1470

Таблица кубов

Как и в ситуации с квадратами, кубы первых 99 натуральных


чисел уже вычислены и занесены в таблицу, называемую
таблицей кубов.

Куб числа по таблице определяется таким же образом, как и


квадрат числа. Например, найдём куб числа 35. Это число
состоит из цифр 3 и 5. Выбираем цифру 3 в первом столбце
таблицы (столбце десятков), а цифру 5 выбираем в первой
строке (строке единиц). Двигаясь вправо от цифры 3 и вниз
от цифры 5, найдём точку пересечения. В результате
окажемся на позиции, где располагается число 42875.
Значит, куб числа 35 есть число 42875.
1471

353 = 42875

Задания для самостоятельного решения


Задача 1. Длина прямоугольника составляет 6 см, а ширина
2 см. Найдите периметр.
Показать решение
Задача 2. Длина прямоугольника составляет 6 см, а ширина
2 см. Найдите площадь.
Показать решение
Задача 3. Площадь прямоугольника составляет 12 см2.
Длина составляет 6 см. Найдите ширину прямоугольника.
Показать решение
Задача 4. Вычислите площадь квадрата со стороной 8 см
1472

Показать решение
Задача 5. Вычислите объем прямоугольного
параллелепипеда, длина которого 6 см, ширина 4 см, высота
3 см.
Показать решение
Задача 6. Объем прямоугольного параллелепипеда
составляет 200 см3. Найдите высоту параллелепипеда, если
его длина равна 10 см, а ширина 5 см
Показать решение
Задача 7. Площади земельного участка, засеянные
пшеницей и льном, пропорциональны числам 4 и 5. На какой
площади засеяна пшеница, если под льном засеяно 15 га
Показать решение

Задача 8. Длина зернохранилища 42 м, ширина составляет


длины, а высота – 0,1 длины. Определите сколько тонн зерна
вмещает зернохранилище, если 1 м3 его весит 740 кг.
Показать решение
Задача 9. 12. Бассейн имеет форму прямоугольного
параллелепипеда, длина которого равна 5,8 м, а ширина –
3,5 м. Две трубы наполняют его водой в течение 13 ч 32 мин.,
причём через одну из них вливается 25 л/мин, а через
вторую – 0,75 этого количества. Определите высоту (глубину)
бассейна.
Показать решение
1473

Одночлены

Предварительные навыки

 Буквенные выражения
 Степень с натуральным показателем

Содержание урока

 Определения и примеры
 Приведение одночлена к стандартному виду
 Сложение и вычитание одночленов
 Умножение одночленов
 Деление одночленов
 Возведение одночлена в степень
 Разложение одночлена на множители
 Задания для самостоятельного решения

Определения и примеры

Одночлен — это произведение чисел, переменных и


степеней. Например, выражения 5a, 3ab2 и −62aa2b3 являются
одночленами.

Приведём ещё примеры одночленов:

Одночленом также является любое отдельное число, любая


переменная или любая степень. Например, число 9 является
1474

одночленом, переменная x является одночленом, степень 52


является одночленом.

Приведение одночлена к стандартному виду

Рассмотрим следующий одночлен:

Этот одночлен выглядит не очень аккуратно. Чтобы сделать


его проще, нужно привести его к так называемому
стандартному виду.

Приведение одночлена к стандартному виду заключается в


перемножении однотипных сомножителей, входящих в этот
одночлен. То есть числа нужно перемножать с числами,
переменные с переменными, степени со степенями. В
результате этих действий получается упрощённый одночлен,
который тождественно равен предыдущему.

Ещё один нюанс заключается в том, что в одночлене степени


можно перемножать только в том случае, если они имеют
одинаковые основания.

Итак, приведём одночлен 3a25a3b2 к стандартному виду. В


этом одночлене содержатся числа 3 и 5. Перемножим их,
получим число 15. Записываем его:

15

Далее в одночлене 3a25a3b2 содержатся степени a2 и a3,


которые имеют одинаковое основание a. Из тождественных
преобразований со степенями известно, что при
перемножении степеней с одинаковыми основаниями,
1475

основание оставляют без изменений, а показатели


складывают. Тогда перемножение степеней a2 и a3 даст в
результате a5. Записываем a5 рядом с числом 15

15a5

Далее в одночлене 3a25a3b2 содержится степень b2. Её не с


чем перемножать, поэтому она остаётся без изменений.
Записываем её как есть к новому одночлену:

15a5b2

Мы привели одночлен 3a25a3b2 к стандартному виду. В


результате получили одночлен 15a5b2

3a25a3b2 = 15a5b2

Числовой сомножитель 15 называют коэффициентом


одночлена. Приводя одночлен к стандартному виду,
коэффициент нужно записывать в первую очередь, и только
потом переменные и степени.

Если коэффициент в одночлене отсутствует, то говорят, что


коэффициент равен единице. Так, коэффициентом
одночлена abc является 1, поскольку abc это произведение
единицы и abc

abc = 1 × abc

А коэффициентом одночлена −abc будет −1, поскольку −abc


это произведение минус единицы и abc

−abc = −1 × abc
1476

Степенью одночлена называют сумму показателей всех


переменных входящих в этот одночлен.

Например, степенью одночлена 15a5b2 является 7. Это


потому что переменная a имеет показатель 5, а переменная
b имеет показатель 2. Отсюда 5 + 2 = 7. Показатель
числового сомножителя 15 считать не нужно, поскольку нас
интересуют только показатели переменных.

Ещё пример. Степенью одночлена 7ab2 является 3. Здесь


переменная a имеет показатель 1, а переменная b имеет
показатель 2. Отсюда 1 + 2 = 3.

Если одночлен не содержит переменных или степеней, а


состоит из числа, то говорят, что степень такого одночлена
равна нулю. Например, степень одночлена 11 равна нулю.

Не следует путать степень одночлена и степень числа.


Степень числа это произведение из нескольких одинаковых
множителей, тогда как степень одночлена это сумма
показателей всех переменных входящих в этот одночлен. В
одночлене 11 нет переменных, поэтому его степень равна
нулю.

Пример 1. Привести одночлен 5xx3ya2 к стандартному виду

Перемножим числа 5 и 3, получим 15. Это будет


коэффициент одночлена:

15

Далее в одночлене 5xx3ya2 содержатся переменные x и x.


Перемножим их, получим x2.
1477

15x2

Далее в одночлене 5xx3ya2 содержится переменная y,


которую не с чем перемножать. Записываем её без
изменений:

15x2y

Далее в одночлене 5xx3ya2 содержится степень a2, которую


тоже не с чем перемножать. Её также оставляем без
изменений:

15x2ya2

Получили одночлен 15x2ya2, который приведён к


стандартному виду. Буквенные сомножители принято
записывать в алфавитном порядке. Тогда одночлен
15x2ya2 примет вид 15a2x2y.

Поэтому, 5xx3ya2 = 15a2x2y.

Пример 2. Привести одночлен 2m3n × 0,4mn к стандартному


виду

Перемножим числа, переменные и степени по отдельности.

2m3n × 0,4mn = 2 × 0,4 × m3 × m × n × n = 0,8m4n2

Числа, переменные и степени при перемножении


разрешается заключать в скобки. Делается это для удобства.
Так, в данном примере перемножение чисел 2 и 0,4 можно
заключить в скобки. Также в скобки можно заключить
перемножение m3 × m и n × n
1478

2m3n × 0,4mn = (2 × 0,4) × (m3 × m) × (n × n) = 0,8m4n2

Но желательно выполнять все элементарные действия в


уме. Так, решение можно записать значительно короче:

2m3n × 0,4mn = 0,8m4n2

Но чтобы в уме приводить одночлен к стандартному


виду, тема умножения целых чисел и умножения степеней
должна быть изучена на хорошем уровне.

Сложение и вычитание одночленов

Одночлены можно складывать и вычитать. Чтобы это было


возможно, они должны иметь одинаковую буквенную часть.
Коэффициенты могут быть любыми. Сложение и вычитание
одночленов это по сути приведение подобных слагаемых,
которое мы рассматривали при изучении буквенных
выражений.

Чтобы сложить (вычесть) одночлены, нужно сложить


(вычесть) их коэффициенты, а буквенную часть оставить без
изменений.

Пример 1. Сложить одночлены 6a2b и 2a2b

6a2b + 2a2b

Сложим коэффициенты 6 и 2, а буквенную часть


6a2b оставим без изменений

6a2b + 2a2b = 8a2b


1479

Пример 2. Вычесть из одночлена 5a2b3 одночлен 2a2b3

5a2b3 − 2a2b3

Можно заменить вычитание сложением, и сложить


коэффициенты одночленов, оставив буквенную часть без
изменения:

5a2b3 − 2a2b3 = 5a2b3 + (−2a2b3) = 3a2b3

Либо сразу из коэффициента первого одночлена вычесть


коэффициент второго одночлена, а буквенную часть
оставить без изменения:

5a2b3 − 2a2b3 = 3a2b3

Умножение одночленов

Одночлены можно перемножать. Чтобы перемножить


одночлены, нужно перемножить их числовые и буквенные
части.

Пример 1. Перемножить одночлены 5x и 8y

Перемножим числовые и буквенные части по отдельности.


Для удобства перемножаемые сомножители будем
заключать в скобки:

5x × 8y = (5 × 8) × (x × y) = 40xy

Пример 2. Перемножить одночлены 5x2y3 и 7x3y2c

Перемножим числовые и буквенные части по отдельности. В


процессе умножения будем применять правило
1480

перемножения степеней с одинаковыми основаниями.


Перемножаемые сомножители будем заключать в скобки:

5x2y3 × 7x3y2c = (5 × 7) × (x2x3) × (y3y2) × c = 35x5y5c

Пример 3. Перемножить одночлены −5a2bc и 2a2b4

−5a2bc × 2a2b4 = (−5 × 2) × (a2a2) × (bb4) × c = −10a4b5c

Пример 4. Перемножить одночлены x2y5 и (−6xy2)

x2y5 × (−6xy2) = −6 × (x2x) × (y5y2) = −6x3y7

Пример 5. Найти значение выражения 

Деление одночленов

Одночлен можно разделить на другой одночлен. Для этого


нужно коэффициент первого одночлена разделить на
коэффициент второго одночлена, а буквенную часть первого
одночлена разделить на буквенную часть второго одночлена.
При этом используется правило деления степеней.

Например, разделим одночлен 8a2b2 на одночлен 4ab.


Запишем это деление в виде дроби:
1481

Первый одночлен 8a2b2 будем называть делимым, а второй


4ab — делителем. А одночлен, который получится в
результате, назовём частным.

Разделим коэффициент делимого на коэффициент


делителя, получим 8 : 4 = 2. В исходном выражении ставим
знак равенства и записываем этот коэффициент частного:

Теперь делим буквенную часть. В делимом содержится a2, в


делителе — просто a. Делим a2 на a, получаем a,
поскольку a2 : a = a2 − 1 = a. Записываем в частном a после 2

Далее в делимом содержится b2, в делителе — просто b.


Делим b2 на b, получаем b, поскольку b2 : b = b2 − 1 = b.
Записываем в частном b после a

Значит, при делении одночлена 8a2b2 на одночлен 4ab


получается одночлен 2ab.

Сразу можно выполнить проверку. При умножении частного


на делитель должно получаться делимое. В нашем случае,
если 2ab умножить на 4ab, должно получиться 8a2b2

2ab × 4ab = (2 × 4) × (aa) × (bb) = 8a2b2


1482

Не всегда можно первый одночлен разделить на второй


одночлен. Например, если в делителе окажется переменная,
которой нет в делимом, то говорят, что деление невозможно.

К примеру, одночлен 6xy2 нельзя разделить на одночлен


3xyz. В делителе 3xyz содержится переменная z, которая не
содержится в делимом 6xy2.

Проще говоря, мы не сможем найти частное, которое при


умножении на делитель 3xyz дало бы делимое 6xy2,
поскольку такое умножение обязательно будет содержать
переменную z, которой нет в 6xy2.

Но если в делимом содержится переменная, которая не


содержится в делителе, то деление будет возможным. В
этом случае переменная, которая отсутствовала в делителе,
будет перенесена в частное без изменений.

Например, при делении одночлена 4x2y2z на 2xy, получается


2xyz. Сначала разделили 4 на 2 получили 2, затем x2
разделили на x, получили x, затем y2 разделили на y,
получили y. Затем приступили к делению переменной z на
такую же переменную в делителе, но обнаружили, что такой
переменной в делителе нет. Поэтому перенесли переменную
z в частное без изменений:

Для проверки умножим частное 2xyz на делитель 2xy. В


результате должен получиться одночлен 4x2y2z

2xyz × 2xy = (2 × 2) × (xx) × (yy) × z = 4x2y2z


1483

Но в некоторых дробях, если невозможно выполнить


деление, бывает возможным выполнить сокращение.
Делается это с целью упростить выражение.

Так, в предыдущем примере нельзя было разделить


одночлен 6xy2 на одночлен 3xyz. Но можно сократить эту
дробь на одночлен 3xy. Напомним, что сокращение дроби
это деление числителя и знаменателя на одно и то же число
(в нашем случае на одночлен 3xy). В результате сокращения
дробь становится проще, но её значение не меняется:

В числителе и знаменателе мы пришли к делению


одночленов, которое можно выполнить:

Процесс деления обычно выполняется в уме, записывая над


числителем и знаменателем получившийся результат:

Пример 2. Разделить одночлен 12a2b3c3 на одночлен 4a2bc

Пример 3. Разделить одночлен x2y3z на одночлен xy2


1484

Дополнительно упомянем, что деление одночлена на


одночлен также невозможно, если одна из степеней,
входящая в делимое, имеет показатель меньший, чем
показатель той же степени из делителя.

Например, разделить одночлен 2x на одночлен x2 нельзя,


поскольку степень x, входящая в делимое, имеет показатель
1, тогда как степень x2, входящая в делитель, имеет
показатель 2. Мы не сможем найти частное, которое при
перемножении с делителем x2 даст в результате делимое 2x.

Конечно, мы можем выполнить деление x на x2,


воспользовавшись свойством степени с целым показателем:

и такое частное при перемножении с делителем x2 будет


давать в результате делимое 2x

Но нас пока интересуют только те частные, которые


являются так называемыми целыми выражениями. Целые
выражения это те выражения, которые не являются дробями,
в знаменателе которых содержится буквенное выражение. А

частное целым выражением не является. Это дробное


выражение, в знаменателе которого содержится буквенное
выражение.
1485

Возведение одночлена в степень

Одночлен можно возвести в степень. Для этого используют


правило возведения степени в степень.

Пример 1. Возвести одночлен xy во вторую степень.

Чтобы возвести одночлен xy во вторую степень, нужно


возвести во вторую степень каждый сомножитель этого
одночлена

(xy)2 = x2y2

Пример 2. Возвести одночлен −5a3b во вторую степень.

(−5a3b)2 = (−5)2 × (a3)2 × b2 = 25a6b2

Пример 3. Возвести одночлен −a2bc3 в пятую степень.

В данном примере коэффициентом одночлена является −1.


Этот коэффициент тоже нужно возвести в пятую степень:

(−a2bc3)5 = (−1)5 × (a2)5 × b5 × (c3)5 = −1a10b5c15 = −a10b5c15

Когда коэффициент равен −1, то саму единицу не


записывают. Записывают только минус и потом остальные
сомножители одночлена. В приведенном примере сначала
получился одночлен −1a10b5c15, затем он был заменён на
тождественно равный ему одночлен −a10b5c15.
1486

Пример 4. Представить одночлен 4x2 в виде одночлена,


возведённого в квадрат.

В данном примере нужно найти произведение, которое во


второй степени будет равно выражению 4x2. Очевидно, что
это произведение 2x. Если это произведение возвести во
вторую степень (в квадрат), то получится 4x2

(2x)2 = 22x2 = 4x2

Значит, 4x2 = (2x)2. Выражение (2x)2 это и есть одночлен,


возведённый в квадрат.

Пример 5. Представить одночлен 121a6 в виде одночлена,


возведённого в квадрат.

Попробуем найти произведение, которое во второй степени


будет равно выражению 121a6.

Прежде всего заметим, что число 121 получается, если число


11 возвести в квадрат. То есть первый сомножитель
будущего произведения мы нашли. А степень a6 получается в
том случае, если возвести в квадрат степень a3. Значит
вторым сомножителем будущего произведения будет a3.

Таким образом, если произведение 11a3 возвести во вторую


степень, то получится  121a6

(11a3)2 = 112 × (a3)2 = 121a6

Значит, 121a6 = (11a3)2. Выражение (11a3)2 это и есть


одночлен, возведённый в квадрат.
1487

Разложение одночлена на множители

Поскольку одночлен является произведением чисел,


переменных и степеней, то он может быть разложен на
множители, из которых состоит.

Пример 1. Разложить одночлен 3a3b2 на множители

Данный одночлен можно разложить на множители 3, a, a, a,


b, b

3a3b2 = 3aaabb

Либо степень b2 можно не раскладывать на множители b и b

3a3b2 = 3aaab2

Либо степень b2 разложить на множители b и b, а степень a3


оставить без изменений

3a3b2 = 3a3bb

В каком виде представлять одночлен зависит от решаемой


задачи. Главное, чтобы разложение было тождественно
равно исходному одночлену.

Пример 2. Разложить одночлен 10a2b3c4 на множители.

Разложим коэффициент 10 на множители 2 и 5, степень a2


разложим на множители aa, степень b3 — на множители bbb,
степень c4 — на множители cccc

10a2b3c4  = 2 × 5 × aabbbcccc
1488

Задания для самостоятельного решения


Задание 1. Приведите одночлен −2aba к стандартному виду.
Показать решение
Задание 2. Приведите одночлен 0,5m × 2n к стандартному
виду.
Показать решение
Задание 3. Приведите одночлен −8ab(−2,5)b2 к стандартному
виду.
Показать решение
Задание 4. Приведите одночлен 0,15pq × 4pq2 к
стандартному виду.
Показать решение
Задание 5. Приведите одночлен −2x3 × 0,5xy2 к стандартному
виду.
Показать решение
Задание 6. Приведите одночлен 2m3n × 0,4mn к
стандартному виду.
Показать решение

Задание 7. Приведите одночлен  к стандартному


виду.
Показать решение

Задание 8. Приведите одночлен  к


стандартному виду.
Показать решение
1489

Задание 9. Перемножьте одночлены 2x и 2y


Показать решение
Задание 10. Перемножьте одночлены 6x, 5x и y
Показать решение
Задание 11. Перемножьте одночлены 2x2, 2x3 и y2
Показать решение
Задание 12. Перемножьте одночлены −8x и 5x3
Показать решение
Задание 13. Перемножьте одночлены x2y5 и (−6xy2)
Показать решение
Задание 14. Выполните умножение:

Показать решение
Задание 15. Выполните умножение:

Показать решение
Задание 16. Возведите одночлен x2y2z2 в третью степень
Показать решение
Задание 17. Возведите одночлен xy2z3 в пятую степень.
Показать решение
Задание 18. Возведите одночлен 4x во вторую степень.
Показать решение
Задание 19. Возведите одночлен 2y3 в третью степень.
1490

Показать решение
Задание 20. Возведите одночлен −0,6x3y2 в третью степень.
Показать решение
Задание 21. Возведите одночлен −x2yz3 в пятую степень.
Показать решение
Задание 22. Возведите одночлен −x3y2z во вторую степень.
Показать решение
Задание 23. Представьте одночлен −27x6y9 в виде одночлена,
возведённого в куб.
Показать решение
Задание 24. Представьте одночлен −a3b6 в виде одночлена,
возведённого в куб.
Показать решение

Задание 25. Выполните деление


Показать решение

Задание 26. Выполните деление


Показать решение

Задание 27. Выполните деление


Показать решение

Задание 28. Выполните деление


Показать решение
1491

Задание 29. Выполните деление


Показать решение

Задание 30. Выполните деление


Показать решение
1492

Многочлены

Предварительные навыки

 Степень с натуральным показателем


 Периметр, площадь и объём
 Одночлены

Содержание урока


o

 Определения и примеры
 Сложение многочленов
 Вычитание многочленов
 Представление многочлен в виде суммы
или разности
 Многочлен и его стандартный вид
 Изменение порядка следования членов
 Умножение одночлена на многочлен
 Умножение многочлена на многочлен
 Вынесение общего множителя за скобки
 Проверка на тождественность
o Задания для самостоятельного решения

Определения и примеры

Многочлен — это сумма одночленов.

Например, выражение 2x + 4xy2 + x + 2xy2 является
многочленом. Проще говоря, многочлен это несколько
одночленов, соединенных знаком «плюс».
1493

В некоторых многочленах одночлены могут соединяться


знаком «минус». Например, 3x − 5y − 2x. Следует иметь
ввиду, что это по-прежнему сумма одночленов. Многочлен
3x − 5y − 2x это сумма одночленов 3x, −5y и − 2x, то
есть 3x + (−5y) + (−2x). После раскрытия скобок образуется
многочлен  3x − 5y − 2x.

3x + (−5y) + (−2x) = 3x − 5y − 2x

Соответственно, рассматривая по отдельности каждый


одночлен многочлена, его нужно рассматривать вместе со
знаком, который перед ним располагается. Так, в многочлене
3x − 5y − 2x минус перед одночленом 5y относится к
коэффициенту 5, а минус перед одночленом 2x относится к
коэффициенту 2. Чтобы не противоречить определению
многочлена, вычитание можно заменять сложением:

3x − 5y − 2x = 3x + (−5y) + (−2x)

Но это действие нагромождает многочлен скобками, поэтому


вычитание на сложение не заменяют, учитывая в будущем,
что каждый одночлен многочлена будет рассматриваться
вместе со знаком, который перед ним располагается.

Одночлены, из которых состоит многочлен, называют


членами многочлена.

Если многочлен состоит из двух членов, то такой многочлен


называют двучленом. Например, многочлен x + y является
двучленом.

Если многочлен состоит из трёх членов, то такой многочлен


называют трехчленом. Например, многочлен x + y + z
является трехчленом.
1494

Если какой-нибудь многочлен содержит обычное число, то


это число называют свободным членом многочлена.
Например, в многочлене 3x + 5y + z + 7 член 7 является
свободным членом. Свободный член многочлена не
содержит буквенной части.

Многочленом также является любое числовое выражение.


Так, следующие выражения являются многочленами:

2+3

5+3+2

5 − 4 + 9

Сложение многочленов

К многочлену можно прибавить другой многочлен. Например,


прибавим к многочлену 2x + y многочлен 3x + y.

Заключим в скобки каждый многочлен и соединим их знаком


«плюс», указывая тем самым, что мы складываем
многочлены:

(2x + y) + (3x + y)

Теперь раскрываем скобки:

2x + y + 3x + y

Далее приведём подобные слагаемые:

2x + y + 3x + y = 5x + 2y
1495

Таким образом, при сложении многочленов 2x + y и 3x + y


получается многочлен 5x + 2y.

Разрешается также сложение многочленов в столбик. Для


этого их следует записать так, чтобы подобные слагаемые
располагались друг под другом, затем выполнить самó
сложение. Решим предыдущий пример в столбик:

Если в одном из многочленов окажется слагаемое, которое


не имеет подобного слагаемого в другом многочлене, оно
переносится к результату без изменений. Как говорят при
сложении обычных чисел — «сносится».

Например, сложим в столбик многочлены 2x2 + y3 + z + 2 и


5x2 + 2y3. Для начала запишем их так, чтобы подобные
слагаемые располагались друг под другом, затем выполним
их сложение. Обнаруживаем, что во втором многочлене не
содержатся слагаемые, которые можно было бы сложить со
слагаемыми z и 2 из первого многочлена. Поэтому
слагаемые z и 2 переносятся к результату без изменений
(вместе со своими знаками)

Решим этот же пример с помощью скобок:


1496

(2x2 + y3 + z + 2) + (5x2 + 2y3) = 2x2 + y3 + z + 2 + 5x2 + 2y3 =


(2x2 + 5x2) + (y3 + 2y3) + z + 2 = 7x2 + 3y3 + z + 2

Пример 3. Сложить многочлены 7x3 + y + z2 и x3 − z2

Решим этот пример в столбик. Запишем второй многочлен


под первым так, чтобы подобные слагаемые располагались
друг под другом:

Во втором многочлене не было слагаемого, которого можно


было бы сложить со слагаемым y из первого многочлена,
поэтому это слагаемое было перенесёно к результату без
изменений. А сложение подобных слагаемых z2 и −z2 дало в
результате 0. Ноль по традиции не записываем. Поэтому
окончательный ответ это 8x3 + y.

Решим этот же пример с помощью скобок:

(7x3 + y + z2) + (x3 − z2) = 7x3 + y + z2 + x3 − z2 = (7x3 + x3) +


(z2 − z2) + y = 8x3 + y

Вычитание многочленов

Из многочлена можно вычесть другой многочлен. Например,


вычтем из многочлена 2x + y многочлен 3x + y.

Заключим в скобки каждый многочлен и соединим их знаком


«минус», указывая тем самым, что мы выполняем вычитание:
1497

(2x + y) − (3x + y)

Теперь раскроем скобки:

2x + y − 3x − y

Приведём подобные слагаемые. Слагаемые y и −y являются


противоположными. Сумма противоположных слагаемых
равна нулю

y + (−y) = 0

Приводя подобные слагаемые, мы обычно складываем их.


Но в качестве знака операции можно использовать знак
одночлена. Так, приводя подобные слагаемые y и −y мы
сложили их по правилу приведения подобных слагаемых. Но
можно не складывая, записать их друг за другом

y−y

Получится тот же результат, поскольку выражения y + (−y)


и y − y одинаково равны нулю:

y−y=0

Возвращаемся к нашему примеру. Вычеркнем члены y и −y:

А сложение подобных слагаемых 2x и −3x, даст в результате


−x

2x + (−3x) = −x

Или без сложения, записав члены друг за другом:

2x − 3x = −x
1498

Значит, при вычитании из многочлена (2x + y) многочлена


(3x + y) получится одночлен −x.

Решим этот же пример в столбик:

Пример 2. Вычесть из многочлена 13x − 11y + 10z многочлен


−15x + 10y − 15z

Решим этот пример с помощью скобок, а затем в столбик:

(13x − 11y + 10z) − (−15x + 10y − 15z) = 13x − 11y + 10z + 15x


− 10y + 15z = (13x + 15x) + (−11y − 10y) + (10z + 15z) =
28x + (−21y) + 25z = 28x − 21y + 25z

Следует быть внимательным при вычитании в столбик. Если


не следить за знаками, вероятность допустить ошибку очень
высокá. Нужно учитывать не только знак операции
вычитания, но и знак располагающийся перед слагаемым.

Так, в данном примере из слагаемого 10z вычиталось


слагаемое −15z

10z − (−15z)

Результат вычисления этого выражения должен быть


положительным, поскольку 10z − (−15z) = 10z + 15z.
1499

Складывая или вычитая многочлены при помощи скобок,


первый многочлен в скобки можно не заключать. Так, в
данном примере из многочлена 13x − 11y + 10z требовалось
вычесть многочлен −15x + 10y − 15z

Вычитание было записано так:

(13x − 11y + 10z) − (−15x + 10y − 15z)

Но первый многочлен можно не заключать в скобки:

13x − 11y + 10z − (−15x + 10y − 15z)

Заключение первого многочлена в скобки на первых порах


позволяет начинающим наглядно увидеть, что второй
многочлен полностью вычитается из первого, а не из
определенной его части.

Представление многочлена в виде суммы или разности

Многочлен можно представить в виде суммы или разности


многочленов. По сути это обратное действие раскрытию
скобок, поскольку идея подразумевает, что имеется некий
многочлен, и из него можно образовать сумму или разность
многочленов, заключив в скобки некоторые из членов
исходного многочлена.

Пусть имеется многочлен 3x + 5y + z + 7. Представим его в


виде суммы двух многочленов.

Итак, из членов исходного многочлена нужно образовать два


многочлена, сложенные между собой. Давайте заключим в
скобки члены 3x и 5y, а также члены z и 7. Далее объединим
их с помощью знака «плюс»
1500

(3x + 5y) + (z + 7)

Значение исходного многочлена при этом не меняется. Если


раскрыть скобки в получившемся выражении (3x + 5y) +
(z + 7), то снова получим многочлен 3x + 5y + z + 7.

(3x + 5y) + (z + 7) = 3x + 5y + z + 7

В скобки также можно было бы заключить члены 3x, 5y, z и


прибавить это выражение в скобках к члену 7

(3x + 5y + z) + 7

Представляя многочлен в виде разности многочленов, нужно


придерживаться следующего правила. Если члены
заключаются в скобки после знака минуса, то этим членам
внутри скобок нужно поменять знаки на противоположные.

Вернемся к многочлену 3x + 5y + z + 7. Представим его в


виде разности двух многочленов. Давайте заключим в скобки
многочлен 3x и 5y, а также z и 7, затем объединим их знаком
«минус»

(3x + 5y) − (z + 7)

Но мы видим, что после знака минуса следует заключение


членов z и 7 в скобки. Поэтому этим членам нужно поменять
знаки на противоположные. Делать это нужно внутри скобок:

(3x + 5y) − (−z − 7)

Заключая члены в скобки, нужно следить за тем, чтобы


значение нового выражения тождественно было равно
предыдущему выражению. Этим и объясняется замена
знаков членов внутри скобок. Если в выражении (3x + 5y) −
1501

(−z − 7) раскрыть скобки, то получим изначальный многочлен


3x + 5y + z + 7.

(3x + 5y) − (−z − 7) = 3x + 5y + z + 7

Вообще, представляя многочлен в виде суммы или разности,


можно придерживаться следующих правил:

Если перед скобками ставится знак «плюс», то все члены


внутри скобок записываются со своими же знаками.

Если перед скобками ставится знак «минус», то все


члены внутри скобок записываются с
противоположными знаками.

Пример 1. Представить многочлен 3x4 + 2x3 + 5x2 − 4 в виде


суммы каких-нибудь двучленов:

(3x4 + 2x3) + (5x2 − 4)

Пример 2. Представить многочлен 3x4 + 2x3 + 5x2 − 4 в виде


разности каких-нибудь двучленов:

(3x4 + 2x3) − (−5x2 + 4)

Перед вторыми скобками располагался минус, поэтому


члены 5x2 и −4 были записаны с противоположными знаками.

Многочлен и его стандартный вид

Многочлен, как и одночлен, можно привести к стандартному


виду. В результате получается упрощенный многочлен, с
которым удобно работать.
1502

Чтобы привести многочлен к стандартному виду, нужно


привести подобные слагаемые в этом многочлене. Подобные
слагаемые в многочлене называют подобными членами
многочлена, а приведение подобных слагаемых в
многочлене — приведением его подобных членов.

Подобные члены многочлена это члены, имеющие


одинаковую буквенную часть.

Приведём многочлен 2x + 4xy2 + x − xy2 к стандартному виду.


Для этого приведём его подобные члены. Подобными
членами в этом многочлене являются 2x и x, а также 4xy2
и −xy2.

В результате получили многочлен 3x + 3xy2, который не


имеет подобных членов. Такой вид многочлена называют
многочленом стандартного вида.

Как и у одночлена, у многочлена имеется степень. Чтобы


определить степень многочлена, сначала его нужно привести
к стандартному виду, затем выбрать тот одночлен, степень
которого является наибольшей из всех.

В предыдущем примере мы привели многочлен


2x + 4xy2 + x − xy2 к стандартному виду. В результате
получили многочлен 3x + 3xy2. Он состоит из двух
одночленов. Степенью первого одночлена является 1, а
степенью второго одночлена является 3. Наибольшая из
этих степеней является 3. Значит, многочлен 3x + 3xy2
является многочленом третьей степени.
1503

А поскольку многочлен 3x + 3xy2 тождественно равен


предыдущему многочлену 2x + 4xy2 + x − xy2, то и этот
предыдущий многочлен является многочленом третьей
степени.

Степенью многочлена стандартного вида называют


наибольшую из степеней, входящих в него одночленов.

В некоторых многочленах прежде всего требуется привести к


стандартному виду одночлены, входящие в него, и только
потом приводить сам многочлен к стандартному виду.

Например, приведем многочлен 3xx4 + 3xx3 − 5x2x3 − 5x2x к


стандартному виду. Этот многочлен состоит из одночленов,
которые не приведены к стандартному виду. Сначала
приведём их к стандартному виду:

3xx4 + 3xx3 − 5x2x3 − 5x2x = 3x5 + 3x4 − 5x5 − 5x3

Теперь получившийся многочлен 3x5 + 3x4 − 5x5 − 5x3 можно


привести к стандартному виду. Для этого приведем его
подобные члены. Подобными являются члены 3x5 и −5x5.
Больше подобных членов нет. Члены 3x4 и −5x3 будут
переписаны без изменений:

3xx4 + 3xx3 − 5x2x3 − 5x2x = 3x5 + 3x4 − 5x5 − 5x3 = −2x5 + 3x4 − 5x3

Пример 2. Привести многочлен 3ab + 4cc + ab + 3c2 к


стандартному виду.

Сначала приведем одночлен 4cc, входящий в исходный


многочлен, к стандартному виду, получим 4с2
1504

3ab + 4cc + ab + 3c2 = 3ab + 4с2 + ab + 3c2

Далее приведём подобные члены:

3ab + 4cc + ab + 3c2 = 3ab + 4с2 + ab + 3c2 = 4ab + 7c2

Пример 3. Привести многочлен 4x2 − 4y − x2 + 17y − y к


стандартному виду.

Подобными членами в данном многочлене являются 4x2 и


−x2, а также −4y, 17y и −y. Приведем их:

4x2 − 4y − x2 + 17y − y = 3x2 + 12y

Приводя подобные члены, можно использовать скобки. Для


этого подобные члены следует заключить в скобки, затем
объединить выражения в скобках с помощью знака «плюс».

Решим предыдущий пример с помощью скобок. Подобными


членами в нём были 4x2 и −x2, а также −4y, 17y и −y.
Заключим их в скобки и объединим с помощью знака «плюс»

4x2 − 4y − x2 + 17y − y = (4x2 − x2) + (−4y + 17y − y)

Теперь в скобках выполним приведение подобных членов:

4x2 − 4y − x2 + 17y − y = (4x2 − x2) + (−4y + 17y − y) =


(3x2) + (12y)

В получившемся выражении (3x2) + (12y) раскроем скобки:

4x2 − 4y − x2 + 17y − y = (4x2 − x2) + (−4y + 17y − y) =


(3x2) + (12y) = 3x2 + 12y
1505

Конечно, такой подход нагромождает выражение, но зато


позволяет свести к минимуму допущение ошибок.

Пример 4. Привести многочлен 12x2 − 9y − 9x2 + 6y + y к


стандартному виду.

Заключим в скобки подобные слагаемые и объединим их с


помощью знака «плюс»

12x2 − 9y − 9x2 + 6y + y = (12x2 − 9x2) + (−9y + 6y + y)

Далее вычисляем содержимое скобок:

12x2 − 9y − 9x2 + 6y + y = (12x2 − 9x2) + (−9y + 6y + y) =


(3x2) + (−2y)

Избавляемся от скобок при помощи раскрытия:

12x2 − 9y − 9x2 + 6y + y = (12x2 − 9x2) + (−9y + 6y + y) =


(3x2) + (−2y) = 3x2 − 2y

Изменение порядка следования членов

Рассмотрим двучлен x − y. Как сделать так, чтобы член −y


располагался первым, а член x вторым?

Многочлен это сумма одночленов. То есть исходный двучлен


двучлен x − y является суммой x и −y

x + (−y)

От перестановки мест слагаемых сумма не меняется. Тогда x


и −y можно поменять местами
1506

−y + x

Пример 2. В двучлене −y − x поменять местами члены.

Двучлен −y − x это сумма членов −y и −x

−y + (−x)

Тогда согласно переместительному закону сложения


получим (−x) + (−y). Избавим выражение от скобок:

−x − y

Таким образом, решение можно записать покороче:

−y − x = −x − y

Пример 3. Упорядочить члены многочлена x + xy3 − x2 в


порядке убывания степеней.

Наибольшую степень в данном многочлене имеет член xy3,


далее −x2, а затем x. Запишем их в этом порядке:

x + xy3 − x2 = xy3 − x2 + x

Умножение одночлена на многочлен

Одночлен можно умножить на многочлен. Чтобы умножить


одночлен на многочлен, нужно этот одночлен умножить на
каждый член многочлена и полученные произведения
сложить.
1507

Например, умножим одночлен 3x2 на многочлен 2x + y + 5.


При умножении одночлена на многочлен, последний нужно
заключать в скобки:

3x2(2x + y + 5)

Теперь умножим одночлен 3x2 на каждый член многочлена


2x + y + 5. Получающиеся произведения будем складывать:

3x2(2x + y + 5) = 3x2 × 2x + 3x2 × y + 3x2 × 5

Вычислим получившиеся произведения:

3x2(2x + y + 5) = 3x2 × 2x + 3x2 × y + 3x2 × 5 = 6x3 + 3x2y + 15x2

Таким образом, при умножении одночлена 3x2 на многочлен


2x + y + 5 получается многочлен 6x3 + 3x2y + 15x2.

Умножение желательно выполнять в уме. Так решение


получается короче:

3x2(2x + y + 5) = 6x3 + 3x2y + 15x2

В некоторых примерах одночлен располагается после


многочлена. В этом случае опять же каждый член
многочлена нужно перемножить с одночленом и полученные
произведения сложить.

Например, предыдущий пример мог быть дан в следующем


виде:

(2x + y + 5) × 3x2

В этом случае мы умножили бы каждый член многочлен


(2x + y + 5) на одночлен 3x2 и сложили бы полученные
результаты:
1508

(2x + y + 5) × 3x2 = 2x × 3x2 + y × 3x2 + 5 × 3x2 = 6x3 + 3x2y +


15x2

Умножение одночлена на многочлен (или умножение


многочлена на одночлен) основано на распределительном
законе умножения.

a(b + c) = ab + ac

То есть чтобы умножить число a на сумму b + c, нужно число


a умножить на каждое слагаемое суммы b + c, и полученные
произведения сложить.

Вообще, умножение одночлена на многочлен, да и


распределительный закон умножения имеют геометрический
смысл.

Допустим, имеется прямоугольник со сторонами a и b

Увеличим сторону b на c
1509

Достроим отсутствующую сторону и закрасим для


наглядности получившийся прямоугольник:

Теперь вычислим площадь получившегося большого


прямоугольника. Он включает в себя желтый и серый
прямоугольники.

Чтобы вычислить площадь получившегося большого


прямоугольника, можно по отдельности вычислить площади
желтого и серого прямоугольников и сложить полученные
результаты. Площадь желтого прямоугольника будет равна
ab, а площадь серого ac

ab + ac

А это всё равно что длину большого прямоугольника


умножить на его ширину. Длина в данном случае это b + c, а
ширина это a
1510

(b + c) × a

или ширину умножить на длину, чтобы расположить буквы a,


b и c в алфавитном порядке:

a × (b + c)

Таким образом, выражения a × (b + c) и ab + ac равны


одному и тому же значению (одной и той же площади)

a × (b + c) = ab + ac

К примеру, пусть у нас имеется прямоугольник длиной 4 см, и


шириной 2 см, и мы увеличили длину на 2 см

Тогда площадь данного прямоугольника будет равна


2 × (4 + 2) или сумме площадей желтого и серого
прямоугольников: 2 × 4 + 2 × 2. Выражения 2 × (4 + 2) и
2 × 4 + 2 × 2 равны одному и тому же значению 12

2 × (4 + 2) = 12

2 × 4 + 2 × 2 = 12

Поэтому,

2 × (4 + 2) = 2 × 4 + 2 × 2 = 12.


1511

Действительно, в получившемся большом прямоугольнике


содержится двенадцать квадратных сантиметров:

Пример 2. Умножить одночлен 2a на многочлен a2 − 7a − 3

Умножим одночлен 2a на каждый член многочлена a2 − 7a − 3


и сложим полученные произведения:

2a(a2 − 7a − 3) = 2a × a2 + 2a × (−7a) + 2a × (−3) = 2a3 +


(−14a2) + (−6a) = 2a3 − 14a2 − 6a

Или покороче:

2a(a2 − 7a − 3) = 2a3 − 14a2 − 6a

Пример 3. Умножить одночлен −a2b2 на многочлен


a2b2 − a2 − b2

Умножим одночлен −a2b2 на каждый член многочлена


a2b2 − a2 − b2 и сложим полученные произведения:

Или покороче:
1512

Пример 4. Выполнить умножение −1,4x2y6(5x3y − 1,5xy2 − 2y3)

Умножим одночлен −1,4x2y6 на каждый член многочлена


5x3y − 1,5xy2 − 2y3 и сложим полученные произведения:

Или покороче:

Пример 5. Выполнить умножение 

Умножим одночлен на каждый член многочлена

и сложим полученные произведения:

Или покороче:
1513

Выполняя короткие решения, результаты записывают сразу


друг за другом вместе со знаком полученного члена.
Рассмотрим поэтапно, как было выполнено короткое
решение данного примера.

Сначала одночлен  нужно умножить на первый член

многочлена , то есть на  . Умножение

выполняется в уме. Получается результат . В исходном


выражении ставим знак равенства и записываем первый
результат:

После этого в исходном выражении никаких знаков ставить


нельзя. Нужно сразу приступать к следующему умножению.

Следующим шагом будет умножение одночлена   на

второй член многочлена  , то есть на  .

Получается результат . Этот результат является

положительным, то есть со знаком плюс . В исходном


выражении этот результат записывается вместе с этим

плюсом сразу после члена


1514

После этого в исходном выражении никаких знаков ставить


нельзя. Нужно сразу приступать к следующему умножению.

Следующим шагом будет умножение одночлена   на

третий член многочлена  , то есть на  .

Получается результат . Этот результат является


отрицательным, то есть со знаком минус. В исходном
выражении этот результат записывается вместе со своим

минусом сразу после члена

Иногда встречаются выражения, в которых сначала нужно


выполнить умножение одночлена на многочлен, затем опять
на одночлен. Например:

2(a + b)c

В этом примере сначала член 2 умножается на многочлен


(a + b), затем результат умножается на c. Для начала
выполним умножение 2 на (a + b) и заключим полученный
результат в скобки

2(a + b)c = (2a + 2b)с
1515

Скобки говорят о том, что результат умножения 2 на (a + b)


полностью умножается на c. Если бы мы не заключили
скобки 2a + 2b, то получилось бы выражение 2a + 2b × с, в
котором на с умножается только 2b. Это привело бы к
изменению значения изначального выражения, а это
недопустимо.

Итак, получили (2a + 2b)с. Теперь умножаем многочлен


(2a + 2b) на одночлен c и получаем окончательный
результат:

2(a + b)c = (2a + 2b)с = 2ac + 2bc

Умножение также можно было бы выполнить сначала


умножив (a + b) на с и полученный результат перемножить с
членом 2

2(a + b)c = 2(ac + bc) = 2ac + 2bc

В данном случае срабатывает сочетательный закон


умножения, который говорит о том, что если выражение
состоит из нескольких сомножителей, то произведение не
будет зависеть от порядка действий:

a × b × с = (a × b) × с = a × (b × с)

То есть умножение можно выполнять в любом порядке. Это


не приведёт к изменению значения изначального выражения.

Умножение многочлена на многочлен

Чтобы умножить многочлен на многочлен, нужно каждый


член первого многочлена умножить на каждый член второго
многочлена и полученные произведения сложить.
1516

Например, умножим многочлен x + 3 на y + 4

Заключим в скобки каждый многочлен и объединим их


знаком умножения ×

(x + 3) × (y + 4)

Либо запишем их друг за другом без знака ×. Это тоже будет


означать умножение:

(x + 3)(y + 4)

Теперь умножим каждый член первого многочлена (x + 3) на


каждый член второго многочлена (y + 4). Здесь опять же
будет применяться распределительный закон умножения:

(a + b)c= ac + bc

Отличие в том, что у нас вместо переменной c имеется


многочлен (y + 4), состоящий из членов y и 4. Наша задача
умножить (x + 3) сначала на y, затем на 4. Чтобы не
допустить ошибку, можно представить, что члена 4 пока не
существует вовсе. Для этого его можно закрыть рукой:

Получаем привычное для нас умножение многочлена на


одночлен. А именно, умножение многочлена (x + 3) на
одночлен y. Выполним это умножение:
1517

(x + 3)(y + 4) = xy + 3y

Мы умножили (x + 3) на y. Теперь осталось умножить (x + 3)


на 4. Для этого умножаем каждый член многочлена (x + 3) на
одночлен 4. На этот раз в исходном выражении (x + 3)(y + 4)
рукой закроем y, поскольку на него мы уже умножали
многочлен (x + 3)

Получаем умножение многочлена (x + 3) на одночлен 4.


Выполним это умножение. Умножение необходимо
продолжать в исходном примере (x + 3)(y + 4) = xy + 3y

(x + 3)(y + 4) = xy + 3y + 4x + 12

Таким образом, при умножении многочлена (x + 3) на


многочлен (y + 4) получается многочлен xy + 3y + 4x + 12.

По другому умножение многочлена на многочлен можно


выполнить ещё так: каждый член первого многочлена
умножить на второй многочлен целиком и полученные
произведения сложить.

Решим предыдущий пример, воспользовавшись этим


способом. Умножим каждый член многочлена x + 3 на весь
многочлен y + 4 целиком и сложим полученные
произведения:
1518

(x + 3)(y + 4) = x(y + 4) + 3(y + 4)

В результате приходим к умножению одночлена на


многочлен, которое мы изучили ранее. Выполним это
умножение:

(x + 3)(y + 4) = x(y + 4) + 3(y + 4) = xy + 4x + 3y + 12

Получится тот же результат что и раньше, но члены


полученного многочлена будут располагаться немного по
другому.

Умножение многочлена на многочлен имеет геометрический


смысл. Допустим, имеется прямоугольник, длина которого a и
ширина b

Площадь этого прямоугольника будет равна a × b.

Увеличим длину данного прямоугольника на x, а ширину на y


1519

Достроим отсутствующие стороны и закрасим для


наглядности получившиеся прямоугольники:

Теперь вычислим площадь получившегося большого


прямоугольника. Для этого вычислим по отдельности
площадь каждого прямоугольника, входящего в этот большой
прямоугольник и сложим полученные результаты. Площадь
жёлтого прямоугольника будет равна ab, площадь серого xb,
площадь фиолетового ay, площадь розового xy

ab + xb + ay + xy
1520

А это всё равно что умножить длину получившегося


большого прямоугольника на его ширину. Длина в данном
случае это a + x, а ширина b + y

(a + x)(b + y)

То есть выражения (a + x)(b + y) и ab + xb + ay + xy


тождественно равны

(a + x)(b + y) = ab + xb + ay + xy

Представим, что у нас имелся прямоугольник, длиной 6 см и


шириной 3 см, и мы увеличили его длину на 2 см, а ширину
на 1 см

Достроим отсутствующие стороны и закрасим для


наглядности получившиеся прямоугольники:
1521

Площадь получившегося большого прямоугольника будет


равна (6 + 2)(3 + 1) или сумме площадей прямоугольников,
входящих в большой прямоугольник:
6 × 3 + 2 × 3 + 6 × 1 + 2 × 1. В обоих случаях получим один и
тот же результат 32

(6 + 2)(3 + 1) = 32

6 × 3 + 2 × 3 + 6 × 1 + 2 × 1 = 32

Поэтому,

(6 + 2)(3 + 1) = 6 × 3 + 2 × 3 + 6 × 1 + 2 × 1 = 18 + 6 + 6 + 2 = 32

Действительно, в получившемся большом прямоугольнике


содержится тридцать два квадратных сантиметра:
1522

Пример 2. Умножить многочлен a + b на c + d

Заключим исходные многочлены в скобки и запишем их друг


за другом:

(a + b)(c + d)

Теперь умножим каждый член первого многочлена (a + b) на


каждый член второго многочлена (c + d)

(a + b)(c + d) = ac + bc + ad + bd

Пример 4. Выполнить умножение (−x − 2y)(x + 2y2)

Умножим каждый член многочлена (−x − 2y) на каждый член


многочлена (x + 2y2)

(−x − 2y)(x + 2y2) = −x2 − 2xy − 2xy2 − 4y3

Результат перемножения членов нужно записывать вместе


со знаками этих членов. Рассмотрим поэтапно, как был
решён данный пример.
1523

Итак, требуется умножить многочлен (−x − 2y) на многочлен


(x + 2y2). Сначала надо умножить многочлен (−x − 2y) на
первый член многочлена (x + 2y2), то есть на x.

Умножаем −x на x, получаем −x2. В исходном


выражении (−x − 2y)(x + 2y2) ставим знак равенства и
записываем −x2

(−x − 2y)(x + 2y2) = −x2

После этого в исходном выражении никаких знаков ставить


нельзя. Нужно сразу приступать к следующему умножению. А
именно умножению −2y на x . Получится −2xy. Этот результат
является отрицательным, то есть со знаком минус. В
исходном выражении записываем результат −2xy сразу
после члена −x2

(−x − 2y)(x + 2y2) = −x2 − 2xy

Теперь умножаем многочлен (−x − 2y) на второй член


многочлена (x + 2y2), то есть на 2y2

Умножаем −x на 2y2, получаем −2xy2. В исходном выражении


записываем этот результат сразу после члена −2xy

(−x − 2y)(x + 2y2) = −x2 − 2xy − 2xy2

Приступаем к следующему умножению. А именно умножению


−2y на 2y2. Получаем −4y3. В исходном выражении этот
результат записываем вместе со своим минусом сразу после
члена −2xy2

(−x − 2y)(x + 2y2) = −x2 − 2xy − 2xy2 − 4y3
1524

Пример 5. Выполнить умножение (4a2 + 2ab − b2)(2a − b)

Умножим каждый член многочлена (4a2 + 2ab − b2) на каждый


член многочлена (2a − b)

В получившемся выражении можно привести подобные


слагаемые:

Пример 6. Выполнить умножение −(a + b)(с − d)

В этот раз перед скобками располагается минус. Этот минус


является коэффициентом −1. То есть исходное выражение
является произведением трёх сомножителей: −1,
многочлена (a + b) и многочлена (с − d).

−1(a + b)(с − d)

Согласно сочетательному закону умножения, если


выражение состоит из нескольких сомножителей, то его
можно вычислять в любом порядке.

Поэтому сначала можно перемножить многочлены (a + b) и


(с − d) и полученный в результате многочлен умножить на −1.
Перемножение многочленов (a + b) и (с − d) нужно
выполнять в скобках

−1(a + b)(с − d) = −1(ac + bc − ad − bd)


1525

Теперь перемножаем −1 и многочлен (ac + bc − ad − bd). В


результате все члены многочлена (ac + bc − ad − bd)
поменяют свои знаки на противоположные:

−1(a + b)(с − d) = −1(ac + bc − ad − bd) = −ac − bc + ad + bd

Либо можно было перемножить −1 с первым


многочленом (a + b) и результат перемножить с
многочленом (с − d)

−1(a + b)(с − d) = (−a − b)(с − d) = −ac − bc + ad + bd

Пример 7. Выполнить умножение x2(x + 5)(x − 3)

Сначала перемножим многочлены (x + 5) и (x − 3), затем


полученный в результате многочлен перемножим с x2

Пример 8. Выполнить умножение (a + 1)(a + 2)(a + 3)

Сначала перемножим многочлены (a + 1) и (a + 2), затем


полученный многочлен перемножим с многочленом (a + 3)

Итак, перемножим (a + 1) и (a + 2)

Полученный многочлен (a2 + a + 2a + 2) перемножим с (a + 3)


1526

Если быстрое перемножение многочленов на первых порах


даётся тяжело, можно воспользоваться подробным
решением, суть которого заключается в том, чтобы записать,
как каждый член первого многочлена умножается на весь
второй многочлен целиком. Такая запись хоть и занимает
место, но позволяет свести к минимуму допущение ошибок.

Например, выполним умножение (a + b)(c + d)

Запишем как каждый член многочлена a + b умножается на


весь многочлен c + d целиком. В результате придём к
умножению одночлена на многочлен, выполнять которое
проще:

(a + b)(c + d) = a(с + d) + b(с + d) = aс + ad + bс + bd

Такая запись удобна при умножении двучлена на какой-


нибудь многочлен, в котором содержится больше двух
членов. Например:

(x + y)(x2 + 2xy + y2) = x(x2 + 2xy + y2) + y(x2 + 2xy + y2) =


x3 + 2x2y + xy2 + x2y + 2xy2 + y3 = x3 + 3x2y + 3xy2 + y3

Или при перемножении многочленов, содержащих больше


двух членов. Например, умножим многочлен x2 + 2x − 5 на
многочлен x3 − x + 2

(x2 + 2x − 5)(x3 − x + 2)

Запишем перемножение исходных многочленов в виде


умножения каждого члена многочлена x2 + 2x − 5 на
многочлен x3 − x + 2.
1527

Получили привычное для нас умножения одночленов на


многочлены. Выполним эти умножения:

В получившемся многочлене приведём подобные члены:

Одночлены, входящие в получившийся многочлен,


расположим в порядке убывания степеней. Делать это
необязательно. Но такая запись будет красивее:

Вынесение общего множителя за скобки

Мы уже учились выносить общий множитель за скобки в


простых буквенных выражениях. Теперь мы немного
углубимся в эту тему, и научимся выносить общий
множитель за скобки в многочлене. Принцип вынесения
будет таким же, как и в простом буквенном
выражении. Небольшие трудности могут возникнуть лишь с
многочленами, состоящими из степеней.

Рассмотрим простой двучлен 6xy + 3xz. Вынесем в нём


общий множитель за скобки. В данном случае за скобки
можно вынести общий множитель 3x. Напомним, что при
1528

вынесении общего множителя за скобки, каждое слагаемое


исходного выражения надо разделить на этот общий
множитель:

Или покороче:

В результате получили 3x(2y + z). При этом в скобках


образовался другой более простой многочлен (2y + z).
Выносимый за скобки общий множитель выбирают так, чтобы
в скобках остались члены, которые не содержат общего
буквенного множителя, а модули коэффициентов этих
членов не имели общего делителя, кроме единицы.

Поэтому в приведенном примере за скобки был вынесен


общий множитель 3x. В скобках образовался многочлен
2y + z, модули коэффициентов которого не имеют общего
делителя кроме единицы. Это требование можно выполнить,
если найти наибольший общий делитель (НОД) модулей
коэффициентов исходных членов. Этот НОД станóвится
коэффициентом общего множителя, выносимого за скобки. В
нашем случае исходный многочлен был 6xy + 3xz.
Коэффициенты исходных членов это числа 6 и 3, а их НОД
равен 3.

А буквенную часть общего множителя выбирают так, чтобы


члены в скобках не имели общих буквенных множителей. В
данном случае это требование выполнилось легко. Общий
буквенный множитель был виден невооруженным глазом —
это был множитель x.
1529

Пример 2. Вынести общий множитель за скобки в


многочлене x2 + x + xy

Все члены данного многочлены имеют коэффициент


единицу. Наибольший общий делитель модулей из этих
единиц есть единица. Поэтому числовая часть выносимого за
скобки множителя будет единицей. Но единицу в качестве
коэффициента не записывают.

Далее выбираем буквенную часть общего множителя.


Прежде всего надо понимать, что любой член, входящий в
многочлен, является одночленом. А одночлен это
произведение чисел, переменных и степеней. Даже если
членом многочлена является обычное число, его всегда
можно представить в виде произведения единицы и самого
этого числа. Например, если в многочлене содержится число
5, его можно представить в виде 1 × 5. Если в многочлене
содержится число 8, то его можно представить в виде
произведения множителей 2 × 2 × 2 (или как 2 × 4)

С переменными такая же ситуация. Если в многочлене


содержится член, являющийся переменной или степенью, их
всегда можно представить в виде произведения. К примеру,
если многочлен содержит одночлен x, его можно представить
в виде произведения 1 × x. Если же многочлен содержит
одночлен x3, его можно представить в виде произведения
xxx.

Одночлены, из которых состоит многочлен x2 + x + xy, можно


разложить на множители так, чтобы мы смогли увидеть
1530

буквенный сомножитель, который является общим для всех


членов.

Итак, первый член многочлена x2 + x + xy, а именно x2 можно


представить в виде произведения x × x. Второй член x можно
представить в виде 1 × x. А третий член xy оставим без
изменения, или для наглядности перепишем его с помощью
знака умножения x × y

Каждый член многочлена представлен в виде произведения


множителей, из которых состоят эти члены. Легко заметить,
что во всех трёх произведениях общим сомножителем
является x. Выделим его:

Этот множитель x и вынесем за скобки. Опять же при


вынесении общего множителя за скобки каждое слагаемое
исходного выражения делим на этот общий множитель. В
нашем случае каждый член многочлена x × x + 1 × x + x × y
нужно разделить на общий множитель x

Значит, при вынесении общего множителя за скобки в


многочлене x2 + x + xy, получается x(x + 1 + y)
1531

Или покороче:

В результате в скобках остаются члены, которые не имеют


общих буквенных сомножителей, а модули коэффициентов
этих членов не имеют общих делителей, кроме 1.

Пример 2. Вынести общий множитель за скобки в


многочлене 15x2y3 + 12xy2 + 3xy2

Определим коэффициент общего множителя, выносимого за


скобки. Наибольший общий делитель модулей
коэффициентов 15, 12 и 3 это число 3. Значит, число 3 будет
коэффициентом общего множителя, выносимого за скобки.

Теперь определим буквенную часть общего множителя,


выносимого за скобки. Её нужно выбирать так, чтобы в
скобках остались члены, которые не содержат общего
буквенного множителя.

Перепишем буквенные части исходного многочлена


15x2y3 + 12xy2 + 3xy2 в виде разложения на множители. Это
позволит хорошо увидеть, что именно можно вынести за
скобки:

Видим, что среди буквенных частей общим множителем


является xyy, то есть xy2. Если вынести этот множитель за
скобки, в скобках останется многочлен, не имеющий общего
буквенного множителя.
1532

В итоге общим множителем, выносимым за скобки, будет


множитель 3xy2

Или покороче:

Для проверки можно выполнить умножение 3xy2(5xy + 4 + 1).


В результате должен получиться многочлен
15x2y3 + 12xy2 + 3xy2

3xy2(5xy + 4 + 1) = 3xy2 × 5xy + 3xy2 × 4 + 3xy2 × 1


= 15x2y3 + 12xy2 + 3xy2

Пример 3. Вынести общий множитель за скобки в выражении


x2 + x

В данном случае за скобки можно вынести x

Это потому что первый член x2 можно представить как xx. А


второй член x представить как 1 × x

x2 + x = xx + 1 × x 

Не следует на письме подробно расписывать содержимое


каждого члена, разлагая его на множители. Это легко
делается в уме.
1533

Пример 4. Вынести общий множитель за скобки в


многочлене 5y2 − 15y

В данном случае за скобки можно вынести 5y. Наибольший


общий делитель модулей коэффициентов 5 и 15 это число 5.
Среди буквенных множителей общим является y

Пример 5. Вынести общий множитель за скобки в


многочлене 5y2 − 15y3

В данном примере за скобки можно вынести 5y2. Наибольший


общий делитель модулей коэффициентов 5 и 15 это число 5.
Среди буквенных множителей общим является y2

Пример 6. Вынести общий множитель за скобки в


многочлене 20x4 − 25x2y2 − 10x3

В данном примере за скобки можно вынести 5x2. Наибольший


общий делитель модулей коэффициентов 20, −25 и −10 это
число 5. Среди буквенных множителей общим является x2
1534

Пример 7. Вынести общий множитель за скобки в


многочлене am  + am  + 1

Второй член am  + 1 представляет собой произведение из am и


a, поскольку am  × a = am  + 1

Заменим в исходном примере член am  + 1 на тождественно


равное ему произведение am  × a. Так проще будет увидеть
общий множитель:

Теперь можно увидеть, что общим множителем является am.


Его и вынесем за скобки:

Проверка на тождественность

Решение задачи с многочленами порой растягивается на


несколько строк. Каждое следующее преобразование должно
быть тождественно равно предыдущему. Если возникают
сомнения в правильности своих действий, то можно
подставить произвольные значения переменных в исходное
и полученное выражение. Если исходное и полученное
выражение будут равны одному и тому же значению, то
можно быть уверенным, что задача была решена правильно.

Допустим, нам нужно вынести общий множитель за скобки в


следующем многочлене:

2x + 4x2
1535

В данном случае за скобки можно вынести общий множитель


2x

2x + 4x2 = 2x(1 + 2x)

Представим, что мы не уверены в таком решении. В этом


случае нужно взять любое значение переменной x и
подставить его сначала в исходное выражение 2x + 4x2,
затем в полученное 2x(1 + 2x). Если в обоих случаях
результат будет одинаковым, то это будет означать, что
задача решена правильно.

Возьмём произвольное значение x и подставим его в


исходное выражение 2x + 4x2. Пусть x = 2. Тогда получим:

2x + 4x2 = 2 × 2 + 4 × 22 = 4 + 16 = 20

Теперь подставим значение 2 в преобразованное выражение


2x(1 + 2x)

2x(1 + 2x) = 2 × 2 × (1 + 2 × 2) = 4 × 5 = 20

То есть при x = 2 выражения 2x + 4x2 и 2x(1 + 2x) равны


одному и тому же значению. Это значит, что задача была
решена правильно. Тоже самое будет происходить и при
других значениях переменных x. Например, проверим наше
решение при x = 1

2x + 4x2 = 2 × 1 + 4 × 12 = 2 + 4 = 6


2x(1 + 2x) = 2 × 1 × (1 + 2 × 1) = 2 × 3 = 6
1536

Пример 2. Вычесть из многочлена 5x2 − 3x + 4 многочлен


4x2 − x и проверить полученный результат, подставив вместо
переменной x произвольное значение.

Выполним вычитание:

Мы выполнили два преобразования: cначала раскрыли


скобки, а затем привели подобные члены. Теперь проверим
эти два преобразования на тождественность. Пусть x = 2.
Подставим это значение сначала в исходное выражение, а
затем в преобразованные:

Видим, что при каждом преобразовании значение выражения


при x = 2 не менялось. Это значит, что задача была решена
правильно.

Задания для самостоятельного решения


Задание 1. Сложить многочлены 8x + 11 и 7x + 5
Показать решение
Задание 2. Вычесть из многочлена 8x + 11 многочлен 7x + 5
Показать решение
Задание 3. Выполнить сложение
8a + (3b + 5a)
Показать решение
1537

Задание 4. Выполнить сложение

Показать решение
Задание 5. Выполнить сложение

Показать решение
Задание 6. Выполнить сложение

Показать решение
Задание 7. Приведите следующий многочлен к стандартному
виду:

Показать решение
Задание 8. Приведите следующий многочлен к стандартному
виду:

Показать решение
Задание 9. Упростите следующее выражение:

Показать решение
Задание 10. Упростите следующее выражение:

Показать решение
Задание 11. Упростите следующее выражение:
1538

Показать решение
Задание 12. Представьте многочлен 5a2 − 2a − 3ab + b2 в
виде суммы двух слагаемых, одно из которых 5a² − 2a
Показать решение
Задание 13. В многочлене 2x3 + 5x2y − 4xy2 − y3 заключить
крайние члены в скобки со знаком плюс (+) перед ними, а
средние члены заключить в скобки со знаком минус (−) перед
ними.
Показать решение
Задание 14. Не изменяя значения выражения
2a3 − 3a2b + 3ab2 − b3, заключите его в скобки, поставив
перед скобками знак (−)
Показать решение
Задание 15. Представьте трёхчлен 2a − b + 4 в виде
разности двух выражений с уменьшаемым 2a
Показать решение
Задание 16. Привести подобные слагаемые в следующем
многочлене:

Показать решение
Задание 17. Выполните умножение одночлена на многочлен:

Показать решение
Задание 18. Выполните умножение одночлена на многочлен:
1539

Показать решение
Задание 19. Выполните умножение одночлена на многочлен:

Показать решение
Задание 20. Выполните умножение одночлена на многочлен:

Показать решение
Задание 21. Выполните умножение одночлена на многочлен:

Показать решение
Задание 22. Выполните умножение одночлена на многочлен:

Показать решение
Задание 23. Выполните умножение одночлена на многочлен:

Показать решение
Задание 24. Выполните умножение одночлена на многочлен:

Показать решение
Задание 25. Выполните умножение одночлена на многочлен:

Показать решение
1540

Задание 26. Выполните умножение многочлена на


многочлен:

Показать решение
Задание 27. Выполните умножение многочлена на
многочлен:

Показать решение
Задание 28. Выполните умножение многочлена на
многочлен:

Показать решение
Задание 29. Выполните умножение многочлена на
многочлен:

Показать решение
Задание 30. Выполните умножение многочлена на
многочлен:

Показать решение
Задание 31. Выполните умножение многочлена на
многочлен:

Показать решение
Задание 32. Выполните умножение многочлена на
многочлен:
1541

Показать решение
Задание 33. Выполните умножение многочлена на
многочлен:

Показать решение
Задание 34. Выполните умножение многочлена на
многочлен:

Показать решение
Задание 35. Выполните умножение многочлена на
многочлен:

Показать решение
Задание 36. В многочлене 6a + 12 вынесите общий
множитель за скобки
Показать решение
Задание 37. В многочлене 5mn − 5m вынесите общий
множитель за скобки
Показать решение
Задание 38. В многочлене x3 − x2 вынесите общий множитель
за скобки
Показать решение
Задание 39. В многочлене 3x2 − 6x3 вынесите общий
множитель за скобки
1542

Показать решение
Задание 40. В многочлене x4 − x2 вынесите общий множитель
за скобки
Показать решение
Задание 41. В многочлене x2y − xy2 вынесите общий
множитель за скобки
Показать решение
Задание 42. В многочлене a3b2 + a2b3 вынесите общий
множитель за скобки
Показать решение
Задание 43. В многочлене a8b2 + ab4 вынесите общий
множитель за скобки
Показать решение
Задание 44. Вынесите общий множитель за скобки в
следующем многочлене:

Показать решение
Задание 45. Вынесите общий множитель за скобки в
следующем многочлене:

Показать решение
Задание 46. Вынесите общий множитель за скобки в
следующем многочлене:

Показать решение
1543

Задание 47. Вынесите общий множитель за скобки в


следующем многочлене:

Показать решение
Задание 48. Вынесите общий множитель за скобки в
следующем многочлене:

Показать решение
Задание 49. Вынесите общий множитель за скобки в
следующем многочлене:

Показать решение
Задание 50. Вынесите общий множитель за скобки в
следующем многочлене:

Показать решение
Задание 51. Вынесите общий множитель за скобки в
следующем многочлене:

Показать решение
Задание 52. Вынесите общий множитель за скобки в
следующем многочлене:

Показать решение
Задание 53. Вынесите общий множитель за скобки в
следующем многочлене:
1544

Показать решение
Задание 54. Вынесите общий множитель за скобки в
следующем многочлене:

Показать решение
1545

Формулы сокращённого умножения

Продолжаем изучать многочлены. В данном уроке мы


научимся перемножать многочлены с помощью формул
сокращённого умножения.

Предварительные навыки

 Степень с натуральным показателем


 Периметр, площадь и объём
 Многочлены

Содержание урока

 Квадрат суммы двух выражений


 Квадрат разности двух выражений
 Куб суммы и куб разности
 Умножение разности двух выражений на их сумму
 Умножение разности двух выражений на неполный
квадрат их суммы
 Умножение суммы двух выражений на неполный квадрат
их разности
 Задания для самостоятельного решения

Квадрат суммы двух выражений

Существует ряд случаев, когда умножение многочлена на


многочлен можно значительно упростить. Таковым к примеру
является случай (2x + 3y)2.

Выражение (2x + 3y)2 это перемножение двух многочленов,


каждый из которых равен (2x + 3y)
1546

(2x + 3y)2 = (2x + 3y)(2x + 3y)

Получили умножение многочлена на многочлен. Выполним


его:

(2x + 3y)2 = (2x + 3y)(2x + 3y) = 4x2 + 6xy + 6xy + 9y2 =


4x2 + 12xy + 9y2

То есть выражение (2x + 3y)2 равно 4x2 + 12xy + 9y2

(2x + 3y)2 = 4x2 + 12xy + 9y2

Решим аналогичный пример, который попроще:

(a + b)2

Выражение (a + b)2 это перемножение двух многочленов,


каждый из которых равен (a + b)

(a + b)2 = (a + b)(a + b)

Выполним это умножение:

(a + b)2 = (a + b)(a + b) = a2 + ab + ab + b2 = a2 + 2ab + b2

То есть выражение (a + b)2 равно a2 + 2ab + b2

(a + b)2 = a2 + 2ab + b2

Оказывается, что случай (a + b)2 можно распространить для


любых a и b. Первый пример, который мы решили, а именно
(2x + 3y)2 можно решить с помощью тождества (a + b)2 =
a2 + 2ab + b2. Для этого нужно подставить вместо переменных
a и b соответствующие члены из выражение (2x + 3y)2. В
данном случае переменной a соответствует член 2x, а
переменной b соответствует член 3y
1547

a = 2x

b = 3y

И далее можно воспользоваться тождеством (a + b)2 =


a2 + 2ab + b2, но вместо переменных a и b нужно подставлять
выражения 2x и 3y соответственно:

(2x + 3y)2 = (2x)2 + 2 × 2x × 3y + (3y)2 = 4x2 + 12xy + 9y2

Как и в прошлый раз получили многочлен 4x2 + 12xy + 9y2.


Решение обычно записывают покороче, выполняя в уме все
элементарные преобразования:

(2x + 3y)2 = 4x2 + 12xy + 9y2

Тождество (a + b)2 = a2 + 2ab + b2 называют формулой


квадрата суммы двух выражений. Эту формулу можно
прочитать так:

Квадрат суммы двух выражений равен квадрату первого


выражения плюс удвоенное произведение первого
выражения на второе плюс квадрат второго выражения.

Рассмотрим выражение (2 + 3)2. Его можно вычислить двумя


способами: выполнить сложение в скобках и возвести
полученный результат в квадрат, либо воспользоваться
формулой квадрата суммы двух выражений.

Первый способ:

(2 + 3)2 = 52 = 25

Второй способ:

(2 + 3)2 = 22 + 2 × 2 × 3 + 32 = 4 + 12 + 9 = 25


1548

Пример 2. Преобразовать выражение (5a + 3)2 в многочлен.

Воспользуемся формулой квадрата суммы двух выражений:

(a + b)2 = a2 + 2ab + b2

(5a + 3)2 = (5a)2 + 2 × 5a × 3 + 32 = 25a2 + 30a + 9

Значит, (5a + 3)2 = 25a2 + 30a + 9.

Попробуем решить данный пример, не пользуясь формулой


квадрата суммы. У нас должен получиться тот же результат:

(5a + 3)2 = (5a + 3)(5a + 3) = 25a2 + 15a + 15a + 9 = 25a2 + 30a


+9

Формула квадрата суммы двух выражений имеет


геометрический смысл. Мы помним, что для вычисления
площади квадрата нужно возвести во вторую степень его
сторону.

Например, площадь квадрата со стороной a будет равна a2.


Если увеличить сторону квадрата на b, то площадь будет
равна (a + b)2

Рассмотрим следующий рисунок:


1549

Представим, что сторону квадрата, изображённого на данном


рисунке увеличили на b. У квадрата все стороны равны. Если
его сторону увеличить на b, то остальные стороны тоже
увеличатся на b

Получился новый квадрат, который больше предыдущего.


Чтобы хорошо увидеть его, достроим отсутствующие
стороны:

Чтобы вычислить площадь этого квадрата, можно по


отдельности вычислить квадраты и прямоугольники,
входящие в него, затем сложить полученные результаты.
1550

Сначала можно вычислить квадрат со стороной a — его


площадь будет равна a2. Затем можно вычислить
прямоугольники со сторонами a и b — они будут равны ab.
Затем можно вычислить квадрат со стороной b

В результате получается следующая сумма площадей:

a2 + ab + ab + b2

Сумму площадей одинаковых прямоугольников можно


заменить на умножение 2ab, которое буквально будет
означать «повторить два раза площадь прямоугольника
ab». Алгебраически это получается путём приведения
подобных слагаемых ab и ab. В результате получается
выражение a2 + 2ab + b2, которое является правой частью
формулы квадрата суммы двух выражений:

(a + b)2 = a2 + 2ab + b2

Квадрат разности двух выражений

Формула квадрата разности двух выражений выглядит


следующим образом:
1551

(a − b)2 = a2 − 2ab + b2

Эту формулу можно прочитать так:

Квадрат разности двух выражений равен квадрату


первого выражения минус удвоенное произведение
первого выражения на второе плюс квадрат второго
выражения.

Формула квадрата разности двух выражений выводится


таким же образом, как и формула квадрата суммы двух
выражений. Выражение (a − b)2 представляет собой
произведение двух многочленов, каждый из которых равен (a
− b)

(a − b)2 = (a − b)(a − b)

Если выполнить это умножение, то получится многочлен a2


− 2ab + b2

(a − b)2 = (a − b)(a − b) = a2 − ab − ab + b2 = a2 − 2ab + b2

Пример 1. Преобразовать выражение (7x − 5)2 в многочлен.

Воспользуемся формулой квадрата разности двух


выражений:

(a − b)2 = a2 − 2ab + b2

(7x − 5)2 = (7x)2 − 2 × 7x × 5 + 52 = 49x2 − 70x + 25

Значит, (7x − 5)2 = 49x2 − 70x + 25.

Попробуем решить данный пример, не пользуясь формулой


квадрата разности. У нас должен получиться тот же
результат:
1552

(7x − 5)2 = (7x − 5)(7x − 5) = 49x2 − 35x − 35x + 25 = 49x2 −


70x + 25.

Формула квадрата разности двух выражений тоже имеет


геометрический смысл. Если площадь квадрата со стороной
a равна a2, то площадь квадрата, сторона которого
уменьшена на b, будет равна (a − b)2

Рассмотрим следующий рисунок:

Представим, что сторону квадрата, изображённого на данном


рисунке уменьшили на b. У квадрата все стороны равны.
Если одну сторону уменьшить на b, то остальные стороны
тоже уменьшатся на b
1553

Получился новый квадрат, который меньше предыдущего. На


рисунке он выделен жёлтым. Сторона его равна a − b,
поскольку старая сторона a уменьшилась на b. Чтобы
вычислить площадь этого квадрата, можно из
первоначальной площади квадрата a2 вычесть площади
прямоугольников, которые получились в процессе
уменьшения сторон старого квадрата. Покажем эти
прямоугольники:

Тогда можно написать следующее выражение: старая


площадь a2 минус площадь ab минус площадь (a − b)b
1554

a2 − ab − (a − b)b

Раскроем скобки в выражении (a − b)b

a2 − ab − ab + b2

Приведем подобные слагаемые:

a2 − 2ab + b2

В результате получается выражение a2 − 2ab + b2, которое


является правой частью формулы квадрата разности двух
выражений:

(a − b)2 = a2 − 2ab + b2

Формулы квадрата суммы и квадрата разности в общем


называют формулами сокращённого умножения. Эти
формулы позволяют значительно упростить и ускорить
процесс перемножения многочленов.

Ранее мы говорили, что рассматривая член многочлена по


отдельности, его нужно рассматривать вместе со знаком,
который перед ним располагается.

Но применяя формулы сокращённого умножения, знак


исходного многочлена не следует рассматривать в качестве
знака самого этого члена.

Например, если дано выражение (5x − 2y)2, и мы хотим


воспользоваться формулой (a − b)2 = a2 − 2ab + b2, то вместо
b нужно подставлять 2y, а не −2y. Это особенность работы с
формулами, которую не следует забывать.
1555

(5x − 2y)2
a = 5x
b = 2y
(5x − 2y)2 = (5x)2 − 2 × 5x × 2y + (2y)2 = 25x2 − 20xy + 4y2

Если подставлять −2y, то это будет означать, что разность в


скобках исходного выражения была заменена на сумму:

(5x − 2y)2 = (5x + (−2y))2

и в таком случае нужно применять не формулу квадрата


разности, а формулу квадрата суммы:

(5x + (−2y)2
a = 5x
b = −2y
(5x + (−2y))2 = (5x)2 + 2 × 5x × (−2y) + (−2y)2 = 25x2 − 20xy + 4y2

Исключением могут быть выражения вида (x − (−y))2. В


данном случае, применяя формулу (a − b)2 = a2 − 2ab + b2
вместо b следует подставить (−y)

(x − (−y))2 = x2 − 2 × x × (−y) + (−y)2 = x2 + 2xy + y2

Но возводя в квадрат выражения вида x − (−y), удобнее


будет заменять вычитание на сложение x + y. Тогда
первоначальное выражение примет вид (x + y)2 и можно
будет воспользоваться формулой квадрата суммы, а не
разности:

(x + y)2 = x2 + 2xy + y2
1556

Куб суммы и куб разности

Формулы куба суммы двух выражений и куба разности двух


выражений выглядят следующим образом:

(a + b)3 = a3 + 3a2b + 3ab2 + b3

(a − b)3 = a3 − 3a2b + 3ab2 − b3

Формулу куба суммы двух выражений можно прочитать так:

Куб суммы двух выражений равен кубу первого


выражения плюс утроенное произведение квадрата
первого выражения на второе плюс утроенное
произведение первого выражения на квадрат второго
плюс куб второго выражения. 

А формулу куба разности двух выражений можно прочитать


так:

Куб разности двух выражений равен кубу первого


выражения минус утроенное произведение квадрата
первого выражения на второе плюс утроенное
произведение первого выражения на квадрат второго
минус куб второго выражения. 

При решении задач желательно знать эти формулы наизусть.


Если не запомнили — не беда! Их можно выводить
самостоятельно. Мы это уже умеем.

Выведем формулу куба суммы самостоятельно:

(a + b)3
1557

Выражение (a + b)3 представляет собой произведение из


трёх многочленов, каждый из которых равен (a + b)

(a + b)3 = (a + b)(a + b)(a + b)

Но выражение (a + b)3 также может быть записано как (a + b)


(a + b)2

(a + b)3 = (a + b)(a + b)2

При этом сомножитель (a + b)2 является квадратом суммы


двух выражений. Этот квадрат суммы равен выражению
a2 + 2ab + b2.

Тогда (a + b)3 можно записать как (a + b)(a2 + 2ab + b2).

(a + b)3 = (a + b)(a2 + 2ab + b2)

А это есть умножение многочлена на многочлен. Выполним


его:

(a + b)3 = (a + b)(a2 + 2ab + b2) = a3 + 2a2b + ab2 + a2b + 2ab2 +


b3 = a3 + 3a2b + 3ab2 + b3

Аналогично можно вывести формулу куба разности двух


выражений:

(a − b)3 = (a − b)(a2 − 2ab + b2) = a3 − 2a2b + ab2 − a2b + 2ab2 −


b3 = a3 − 3a2b + 3ab2 − b3

Пример 1. Преобразуйте выражение (x + 1)3 в многочлен.

Воспользуемся формулой куба суммы двух выражений:

(a + b)3 = a3 + 3a2b + 3ab2 + b3
1558

(x + 1)3 = x3 + 3 × x2 × 1 + 3 × x × 12 + 13 = x3 + 3x2 + 3x + 1

Попробуем решить данный пример, не используя формулу


куба суммы двух выражений. У нас получится тот же
результат, но решение станет длиннее:

(x + 1)3 = (x + 1)(x + 1)(x + 1) = (x + 1)(x2 + 2x + 1) = x3 +


2x2 + x + x2 + 2x + 1 = x3 + 3x2 + 3x + 1

Пример 2. Преобразовать выражение (6a2 + 3b3)3 в


многочлен.

Воспользуемся формулой куба суммы двух выражений:

(a + b)3 = a3 + 3a2b + 3ab2 + b3

(6a2 + 3b3)3 = (6a2)3 + 3 × (6a2)2 × 3b3 + 3 × 6a2 × (3b3)2 + (3b3)3 =


216a6 + 3 × 36a4 × 3b3 + 3 × 6a2 × 9b6 + 27b9

Пример 3. Преобразовать выражение (n2 − 3)3 в многочлен.

Воспользуемся формулой куба разности двух выражений:

(a − b) = a3 − 3a2b + 3ab2 − b3

(n2 − 3)3 = (n2)3 − 3 × (n2)2 × 3 + 3 × n2 × 32 − 33 = n6 − 9n4  + 27n2


− 27

Пример 4. Преобразовать выражение (2x2 − x3)3 в многочлен.

Воспользуемся формулой куба разности двух выражений:

(a − b) = a3 − 3a2b + 3ab2 − b3
1559

(2x2 − x3)3 = (2x2)3 − 3 × (2x2)2 × x3 + 3 × 2x2 × (x3)2 − (x3)3 =
8x6 − 3 × 4x4 × x3 + 3 × 2x2 × x6 − x9 =
8x6 − 12x7 + 6x8 − x9

Умножение разности двух выражений на их сумму

Встречаются задачи, в которых требуется умножить разность


двух выражений на их сумму. Например:

(a − b)(a + b)

В этом выражении разность двух выражений a и b умножена


на сумму этих же двух выражений. Выполним данное
умножение:

(a − b)(a + b) = a2 + ab − ab − b2 = a2 − b2

То есть выражение (a − b)(a + b) равно a2 − b2

(a − b)(a + b) = a2 − b2

Видим, что при умножении разности двух выражений на их


сумму, получается разность квадратов этих выражений.

Произведение разности двух выражений и их суммы


равно разности квадратов этих выражений.

Случай (a − b)(a + b) можно распространить для любых a и


b. Проще говоря, если при решении задачи потребуется
умножить разность двух выражений на их сумму, то это
умножение можно заменить на разность квадратов этих
выражений.

Пример 1. Выполнить умножение (2x − 5)(2x + 5)


1560

В этом примере разность выражений 2x и 5 умножена на


сумму этих же выражений. Тогда согласно формуле (a − b)
(a + b) = a2 − b2 имеем:

(2x − 5)(2x + 5) = (2x)2 − 52

Вычислим правую часть, получим 4x2 − 25

(2x − 5)(2x + 5) = (2x)2 − 52 = 4x2 − 25

Попробуем решить данный пример, не пользуясь формулой


(a − b)(a + b) = a2 − b2. У нас получится тот же результат
4x2 − 25

(2x − 5)(2x + 5) = 4x2 − 10x + 10x − 25 = 4x2 − 25

Пример 2. Выполнить умножение (4x − 5y)(4x + 5y)

Воспользуемся формулой умножения разности двух


выражений на их сумму:

(a − b)(a + b) = a2 − b2

(4x − 5y)(4x + 5y) = (4x)2 − (5y)2 = 16x2 − 25y2

Пример 3. Выполнить умножение (2a + 3b)(2a − 3b)

Воспользуемся формулой умножения разности двух


выражений на их сумму:

(a − b)(a + b) = a2 − b2

(2a + 3b)(2a − 3b) = (2a)2 − (3b)2 = 4a2 − 9b2


1561

В данном примере сумма членов 2a и 3b располагалась


раньше, чем разность этих членов. А в формуле (a − b)(a +
b) = a2 − b2 разность располагается раньше.

Нет никакой разницы как располагаются сомножители (a − b)


в (a + b) в формуле. Они могут быть быть записаны как
(a − b)(a + b), так и (a + b)(a − b). Результат по прежнему
будет равен a2 − b2, поскольку от перестановки сомножителей
произведение не меняется.

Так и в данном примере сомножители (2a + 3b) и (2a − 3b)


можно записать как (2a + 3b)(2a − 3b), так и (2a − 3b)(2a +
3b). Результат всё так же будет равен 4a2 − 9b2.

Пример 3. Выполнить умножение (7 + 3x)(3x − 7)

Воспользуемся формулой умножения разности двух


выражений на их сумму:

(a − b)(a + b) = a2 − b2

(7 + 3x)(3x − 7) = (3x)2 − 72 = 9x2 − 49

Пример 4. Выполнить умножение (x2 − y3)(x2 + y3)

(a − b)(a + b) = a2 − b2

(x2 − y3)(x2 + y3) = (x2)2 − (y3)2 = x4 − y6

Пример 5. Выполнить умножение (−5x − 3y)(5x − 3y)

В выражении (−5x − 3y) вынесем за скобки −1, тогда


исходное выражение примет следующий вид:
1562

(−5x − 3y)(5x − 3y) = −1(5x + 3y)(5x − 3y)

Произведение (5x + 3y)(5x − 3y) заменим на разность


квадратов:

(−5x − 3y)(5x − 3y) = −1(5x + 3y)(5x − 3y) = −1((5x)2 − (3y)2)

Разность квадратов была заключена в скобки. Если этого не


сделать, то получится, что −1 умножается только на (5x)2. А
это приведет к ошибке и изменению значения исходного
выражения.

Далее вычисляем выражение в скобках:

(−5x − 3y)(5x − 3y) = −1(5x + 3y)(5x − 3y) = −1((5x)2 − (3y)2) =


−1(25x2 − 9y2)

Теперь умножим −1 на выражение в скобках и получим


окончательный результат:

(−5x − 3y)(5x − 3y) = −1(5x + 3y)(5x − 3y) = −1((5x)2 − (3y)2) =


−1(25x2 − 9y2) = −25x2 + 9y2

Умножение разности двух выражений на неполный квадрат


их суммы

Встречаются задачи, в которых требуется умножить разность


двух выражений на неполный квадрат их суммы. Выглядит
это произведение следующим образом:

(a − b)(a2 + ab + b2)
1563

Первый многочлен (a − b) является разностью двух


выражений, а второй многочлен (a2 + ab + b2) является
неполным квадратом суммы этих двух выражений.

Неполный квадрат суммы это многочлен вида a2 + ab + b2. Он


похож на обычный квадрат суммы a2 + 2ab + b2 за
исключением того, что в нём произведение первого и второго
выражений не удваивается.

Например, выражение 4x2 + 6xy + 9y2 является неполным


квадратом суммы выражений 2x и 3y.

Действительно, первый член выражения 4x2 + 6xy + 9y2, а


именно 4x2 является квадратом выражения 2x, поскольку
(2x)2 = 4x2. Третий член выражения 4x2 + 6xy + 9y2, а именно
9y2 является квадратом выражения 3y, поскольку (3y)2 = 9y2.
Член находящийся в середине 6xy, является произведением
выражений 2x и 3y.

Итак, умножим разность (a − b) на неполный квадрат суммы


a2 + ab + b2

(a − b)(a2 + ab + b2) = a(a2 + ab + b2) − b(a2 + ab + b2) =


a3 + a2b + ab2 − a2b − ab2 − b3 = a3 − b3

То есть выражение (a − b)(a2 + ab + b2) равно a3 − b3

(a − b)(a2 + ab + b2) = a3 − b3

Это тождество называют формулой умножения разности


двух выражений на неполный квадрат их суммы. Эту
формулу можно прочитать так:
1564

Произведение разности двух выражений и неполного


квадрата их суммы равно разности кубов этих
выражений.

Пример 1. Выполнить умножение (2x − 3y)(4x2 + 6xy + 9y2)

Первый многочлен (2x − 3y) это разность двух выражений 2x


и 3y. Второй многочлен 4x2 + 6xy + 9y2 это неполный квадрат
суммы двух выражений 2x и 3y. Это позволяет не приводя
длинных вычислений, воспользоваться формулой (a − b)(a2 +
ab + b2) = a3 − b3. В нашем случае умножение (2x − 3y)
(4x2 + 6xy + 9y2) можно заменить на разность кубов 2x и 3y

(2x − 3y)(4x2 + 6xy + 9y2) = (2x)3 − (3y)3 = 8x3 − 27y3

Попробуем решить этот же пример, не пользуясь формулой


(a − b)(a2 + ab + b2) = a3 − b3. У нас получится тот же
результат, но решение станет длиннее:

(2x − 3y)(4x2 + 6xy + 9y2) = 2x(4x2 + 6xy + 9y2) − 3y(4x2 + 6xy +


9y2) =
8x3 + 12x2y + 18xy2 − 12x2y − 18xy2 − 27y3 = 8x3 − 27y3

Пример 2. Выполнить умножение (3 − x)(9 + 3x + x2)

Первый многочлен (3 − x) является разностью двух


выражений, а второй многочлен является неполным
квадратом суммы этих двух выражений. Это позволяет
воспользоваться формулой (a − b)(a2 + ab + b2) = a3 − b3

(3 − x)(9 + 3x + x2) = 33 − x3 = 27 − x3


1565

Умножение суммы двух выражений на неполный квадрат их


разности

Встречаются задачи, в которых требуется умножить сумму


двух выражений на неполный квадрат их разности. Выглядит
это произведение следующим образом:

(a + b)(a2 − ab + b2)

Первый многочлен (a + b) является суммой двух выражений,


а второй многочлен (a2 − ab + b2) является неполным
квадратом разности этих двух выражений.

Неполный квадрат разности это многочлен вида a2 − ab + b2.


Он похож на обычный квадрат разности a2 − 2ab + b2 за
исключением того, что в нём произведение первого и второго
выражений не удваивается.

Например, выражение 4x2 − 6xy + 9y2 является неполным


квадратом разности выражений 2x и 3y. 

(2x)2 − 2x × 3y + (3y)2 = 4x2 − 6xy + 9y2

Вернёмся к изначальному примеру. Умножим сумму a + b на


неполный квадрат разности a2 − ab + b2

(a + b)(a2 − ab + b2) = a(a2 − ab + b2) + b(a2 − ab + b2) =


a3 − a2b + ab2 + a2b − ab2 + b3 = a3 + b3

То есть выражение (a + b)(a2 − ab + b2) равно a3 + b3

(a + b)(a2 − ab + b2) = a3 + b3
1566

Это тождество называют формулой умножения суммы двух


выражений на неполный квадрат их разности. Эту формулу
можно прочитать так:

Произведение суммы двух выражений и неполного


квадрата их разности равно сумме кубов этих
выражений.

Пример 1. Выполнить умножение (2x + 3y)(4x2 − 6xy + 9y2)

Первый многочлен (2x + 3y) это сумма двух выражений 2x и


3y, а второй многочлен 4x2 − 6xy + 9y2 это неполный квадрат
разности этих выражений. Это позволяет не приводя
длинных вычислений, воспользоваться формулой (a + b)(a2 −
ab + b2) = a3 + b3. В нашем случае умножение (2x + 3y)(4x2
− 6xy + 9y2) можно заменить на сумму кубов 2x и 3y

(2x + 3y)(4x2 − 6xy + 9y2) = (2x)3 + (3y)3 = 8x3 + 27y3

Попробуем решить этот же пример, не пользуясь формулой


(a + b)(a2 − ab + b2) = a3 + b3. У нас получится тот же
результат, но решение станет длиннее:

(2x + 3y)(4x2 − 6xy + 9y2) = 2x(4x2 − 6xy + 9y2) + 3y(4x2 − 6xy +


9y2) =
8x3 − 12x2y + 18xy2 + 12x2y − 18xy2 + 27y3 = 8x3 + 27y3

Пример 2. Выполнить умножение (2x + y)(4x2 − 2xy + y2)

Первый многочлен (2x + y) является суммой двух


выражений, а второй многочлен (4x2 − 2xy + y2) является
неполным квадратом разности этих выражений. Это
1567

позволяет воспользоваться формулой (a + b)


(a2 − ab + b2) = a3 + b3

(2x + y)(4x2 − 2xy + y2) = (2x)3 + y3 = 8x3 + y3

Попробуем решить этот же пример, не пользуясь формулой


(a + b)(a2 − ab + b2) = a3 + b3. У нас получится тот же
результат, но решение станет длиннее:

(2x + y)(4x2 − 2xy + y2) = 2x(4x2 − 2xy + y2) + y(4x2 − 2xy + y2) = 


8x3 − 4x2y + 2xy2 + 4x2y − 2xy2 + y3 = 8x3 + y3

Задания для самостоятельного решения


Задание 1. Преобразуйте выражение (m + n)2 в многочлен.
Показать решение
Задание 2. Преобразуйте выражение (x + 8)2 в многочлен.
Показать решение
Задание 3. Преобразуйте выражение (2x2 + 3x3)2 в многочлен.
Показать решение
Задание 4. Преобразуйте выражение (5a + 5)2 в многочлен.
Показать решение
Задание 5. Преобразуйте выражение (9 − x)2 в многочлен.
Показать решение
Задание 6. Преобразуйте выражение (x − 25)2 в многочлен.
Показать решение
Задание 7. Преобразуйте выражение (3x2 − y3)2 в многочлен.
Показать решение
1568

Задание 8. Выполните умножение (x − y)(x + y)


Показать решение
Задание 9. Выполните умножение (2x − y)(2x + y)
Показать решение
Задание 10. Выполните умножение (7 + 3y)(3y − 7)
Показать решение
Задание 11. Выполните умножение (x2 − 5)(x2 + 5)
Показать решение
Задание 12. Выполните умножение (a3 − b2)(a3 + b2)
Показать решение
Задание 13. Выполните умножение (5a2 + 2b3)(5a2 − 2b3)
Показать решение
Задание 14. Выполните умножение (9x − y2)(y2 + 9x)
Показать решение
Задание 15. Выполните умножение (2 − x)(4 + 2x + x2)
Показать решение
Задание 16. Выполните умножение (3 − 2)(9 + 6 + 4)
Показать решение
Задание 17. Выполните умножение (4x + 1)(16x2 − 4x + 1)
Показать решение
1569

Разложение многочлена на множители

Разложить многочлен на множители означает представить


его в виде произведения двух или нескольких многочленов.

Примером разложения многочлена на множители является


вынесение общего множителя за скобки, поскольку исходный
многочлен обращается в произведение двух сомножителей,
один из которых является одночленом, а другой
многочленом.

Предварительные навыки


o Степень с натуральным показателем
o Периметр, площадь и объём
o Многочлены
o Формулы сокращённого умножения

Содержание урока


o Разложение способом вынесения общего множителя
за скобки
o Разложение способом группировки
o Разложение по формуле квадрата суммы двух
выражений
o Разложение по формуле квадрата разности двух
выражений
o Разложение по формуле куба суммы двух
выражений
1570

o Разложение по формуле куба разности двух


выражений
o Разложение по формуле разности квадратов двух
выражений
o Разложение по формуле сумме кубов двух
выражений
o Разложение по формуле разности кубов двух
выражений
o Разложение различными способами
o Задания для самостоятельного решения

Разложение многочлена на множители способом вынесения


общего множителя за скобки

При вынесении общего множителя за скобки образуется


произведение из двух сомножителей, один из которых
является одночленом, а другой многочленом. Например:

6x + 3xy = 3x(2 + y)

В рамках изучения многочленов, одночлен принято считать


многочленом, состоящим из одного члена. Поэтому, когда в
многочлене выносится за скобки общий множитель, то
говорят что исходный многочлен представлен в виде
произведения многочленов.

В нашем примере многочлен 6x + 3xy был представлен в


виде произведения многочленов 3x и (2 + y). По-другому
говорят, что многочлен 6x + 3xy разложен на множители 3x
и (2 + y)

Существуют также многочлены, в которых можно вынести за


скобки такой общий множитель, который является
1571

двучленом. Например, рассмотрим многочлен


5a(x + y) + 7a(x + y). В этом многочлене общим множителем
является двучлен (x + y). Вынесем его за скобки:

Разложение многочлена на множители способом группировки

Некоторые многочлены содержат группу членов, имеющих


общий множитель. Такие группы можно заключать в скобки и
далее выносить общий множитель за эти скобки. В
результате получается разложение исходного многочлена на
множители, которое называют разложением на множители
способом группировки.

Рассмотрим следующий многочлен:

ax + ay + 3x + 3y

Члены ax и ay имеют общий множитель a. Выпишем эти


члены и заключим их в скобки:

(ax + ay)

Далее в многочлене ax + ay + 3x + 3y члены 3x и 3y имеют


общий множитель 3. Выпишем эти члены и тоже заключим их
в скобки:

(3x + 3y)

Теперь соединим выражения (ax + ay) и (3x + 3y) знаком


«плюс»

(ax + ay) + (3x + 3y)


1572

В многочлене (ax + ay) вынесем за скобки общий множитель


a, а в многочлене (3x + 3y) вынесем за скобки общий
множитель 3. Делать это нужно в исходном выражении:

Далее замечаем, что двучлен (x + y) является общим


множителем. Вынесем его за скобки. Продолжаем решение в
исходном примере. В результате получим:

Запишем решение покороче, не расписывая подробно, как


каждый член был разделен на общий множитель. Тогда
решение получится более компактным:

Чтобы проверить правильно ли мы разложили многочлен на


множители, выполним умножение (x + y)(a + 3). Если мы всё
сделали правильно, то получим многочлен ax + ay + 3x + 3y

(x + y)(a + 3) = ax + ay + 3x + 3y

Пример 2. Разложить многочлен 9x + ax − 9y − ay на


множители способом группировки.
1573

Члены 9x и −9y имеют общий множитель 9. А члены ax и −ay


имеют общий множитель a. Сгруппируем их с помощью
скобок, и объединим с помощью знака «плюс»

(9x − 9y) + (ax − ay)

В первой группе (9x  − 9y) вынесем за скобки общий


множитель 9. Во второй группе (ax − ay) вынесем за скобки
за скобки общий множитель a

(9x − 9y) + (ax − ay) = 9(x − y) + a(x − y)

Далее вынесем за скобки двучлен (x − y)

(9x − 9y) + (ax − ay) = 9(x − y) + a(x − y) = (x − y)(9 + a)

Пример 3. Разложить многочлен ab − 3b + b2 − 3a на


множители способом группировки.

Сгруппируем первый член ab с четвёртым членом −3a. А


второй член −3b сгруппируем с третьим членом b2. Не
забываем, что объединять группы нужно с помощью знака
«плюс»

(ab − 3a) + (−3b + b2)

В первой группе вынесем за скобки общий множитель a, во


второй группе — общий множитель b

(ab − 3a) + (−3b + b2) = a(b − 3) + b(−3 + b)

Во втором произведении b(−3 + b) в сомножителе (−3 + b)


изменим порядок следования членов. Тогда получим b(b − 3)

(ab − 3a) + (−3b + b2) = a(b − 3) + b(b − 3)


1574

Теперь вынесем за скобки общий множитель (b − 3)

(ab − 3a) + (−3b + b2) = a(b − 3) + b(b − 3) = (b − 3)(a + b)

Пример 4. Разложить многочлен


x2y + x + xy2 + y + 2xy + 2 на множители способом
группировки.

Сгруппируем первый член многочлена со вторым, третий с


четвёртым, пятый с шестым:

В первой группе вынесем за скобки общий множитель x, во


второй группе — общий множитель y, в третьей группе —
общий множитель 2

Далее замечаем, что многочлен (xy + 1) является общим


множителем. Вынесем его за скобки:

Разложение многочлена на множители по формуле квадрата


суммы двух выражений

Формулы сокращённого умножения, которые мы


рассматривали в прошлом уроке, можно применять для
разложения многочленов на множители.
1575

Вспомним, как выглядит формула квадрата суммы двух


выражений:

(a + b)2 = a2 + 2ab + b2

Поменяем местами левую и правую часть, получим:

a2 + 2ab + b2 = (a + b)2

Левая часть этого равенства является многочленом, а


правая часть — произведением многочленов, поскольку
выражение (a + b)2 представляет собой перемножение двух
сомножителей, каждый из которых равен многочлену (a + b).

Стало быть, если нам встретится выражение вида


a2 + 2ab + b2, то мы можем представить его в виде
произведения (a + b)(a + b). Иными словами, разложить на
множители (a + b) и (a + b).

a2 + 2ab + b2 = (a + b)(a + b)

Пример 1. Разложить на множители многочлен 4x2 + 12xy +


9y2

Чтобы воспользоваться формулой a2 + 2ab + b2 = (a + b)2,


нужно узнать чему в данном случае равна переменная a и
чему равна переменная b.

Первый член многочлена 4x2 + 12xy + 9y2 является


результатом возведения в квадрат одночлена 2x, поскольку
(2x)2 = 4x2. Третий член 9y2 является результатом возведения
в квадрат одночлена 3y, поскольку (3y)2 = 9y2, а член 12xy это
есть удвоенное произведение членов 2x и 3y, то есть
2 × 2x × 3y = 12xy.
1576

Очевидно, что переменная a в данном случае равна 2x, а


переменная b равна 3y

a = 2x
b = 3y

Тогда можно сделать вывод, что когда-то выражение


4x2 + 12xy + 9y2 выглядело в виде квадрата суммы (2x + 3y)2,
но в результате применения формулы квадрата суммы оно
обратилось в многочлен 4x2 + 12xy + 9y2. Наша задача —
вернуть ему былую форму, то есть представить в виде
(2x + 3y)2

4x2 + 12xy + 9y2 = (2x + 3y)2

А поскольку (2x + 3y)2 это произведение двух сомножителей,


каждый из которых равен многочлену (2x + 3y), то исходный
многочлен 4x2 + 12xy + 9y2 можно представить в виде
разложения на множители (2x + 3y) и (2x + 3y)

4x2 + 12xy + 9y2 = (2x + 3y)(2x + 3y)

Полностью решение можно записать так:

4x2 + 12xy + 9y2 = (2x)2 + 2 × 2x × 3y + (3y)2 = (2x + 3y)2 =


(2x + 3y)(2x + 3y)

Пример 2. Разложить на множители многочлен x2 + 12x + 36

Первый член данного многочлена является результатом


возведения в квадрат одночлена x, поскольку x2 = x2, третий
член — результатом возведения в квадрат числа 6,
1577

поскольку 62 = 36, а член 12x это удвоенное произведение


членов x и 6, поскольку 2 × x × 6 = 12x.

Воспользуемся формулой a2 + 2ab + b2 = (a + b)2. Роль


переменной a играет одночлен x, а роль переменной b играет
одночлен 6. Отсюда:

x2 + 12x + 36 = (x + 6)2

А поскольку (x + 6)2 это произведение двух сомножителей,


каждый из которых равен многочлену (x + 6), то исходный
многочлен x2 + 12x + 36 можно представить в виде
разложения на множители (x + 6) и (x + 6)

x2 + 12x + 36 = (x + 6)(x + 6)

Разложение многочлена на множители по формуле квадрата


разности двух выражений

Как и по формуле квадрата суммы двух выражений,


многочлен можно разложить на множители по формуле
квадрата разности двух выражений.

Формула квадрата разности двух выражений выглядит так:

(a − b)2 = a2 − 2ab + b2

Если в этой формуле поменять местами левую и правую


часть, то получим:

a2 − 2ab + b2 = (a − b)2

Поскольку правая часть это произведение двух


сомножителей, каждый из которых равен (a − b), то
1578

многочлен вида a2 − 2ab + b2 можно разложить на множители


(a − b) и (a − b).

a2 − 2ab + b2 = (a − b)(a − b)

Пример 1. Разложить на множители многочлен 9x2 − 12xy +


4y2

Чтобы воспользоваться формулой a2 − 2ab + b2 = (a − b)2,


нужно узнать чему в данном случае равна переменная a и
чему равна переменная b.

Первый член данного многочлена является результатом


возведения в квадрат одночлена 3x, поскольку (3x)2 = 9x2.
Третий член 4y2 является результатом возведения в квадрат
одночлена 2y, поскольку (2y)2 = 4y2, а член 12xy это
удвоенное произведение членов 3x и 2y, то есть 2 × 3x × 2y =
12xy.

Очевидно, что переменная a в данном случае равна 3x, а


переменная b равна 2y

a = 3x
b = 2y

Тогда можно сделать вывод, что когда-то выражение


9x2 − 12xy + 4y2 выглядело в виде квадрата разности (3x −
2y)2, но в результате применения формулы квадрата
разности оно обратилось в многочлен 9x2 − 12xy + 4y2. Наша
задача — вернуть ему былую форму, то есть представить в
виде (3x − 2y)2

9x2 − 12xy + 4y2 = (3x − 2y)2


1579

А поскольку (3x − 2y)2 это произведение двух сомножителей,


каждый из которых равен многочлену (3x − 2y), то исходный
многочлен 9x2 − 12xy + 4y2 можно представить в виде
разложения на множители (3x − 2y) и (3x − 2y)

9x2 − 12xy + 4y2 = (3x − 2y)(3x − 2y)

Полностью решение можно записать так:

9x2 − 12xy + 4y2 = (3x)2 − 2 × 3x × 2y + (2y)2 = (3x − 2y)2 = (3x −


2y)(3x − 2y)

Пример 2. Разложить на множители многочлен x2 − 4x + 4

Воспользуемся формулой квадрата разности двух


выражений:

x2 − 4x + 4 = x2 − 2 × x × 2 + 22 = (x − 2)2 = (x − 2)(x − 2)

Разложение многочлена на множители по формуле куба


суммы двух выражений

Вспомним, как выглядит формула куба суммы двух


выражений:

(a + b)3 = a3 + 3a2b + 3ab2 + b3

Поменяем местами левую и правую часть, получим:

a3 + 3a2b + 3ab2 + b3 = (a + b)3

Левая часть этого равенства является многочленом, а


правая часть — произведением многочленов, поскольку
1580

выражение (a + b)3 представляет собой перемножение трёх


сомножителей, каждый из которых равен многочлену (a + b).

Стало быть, если нам встретится выражение вида


a3 + 3a2b +3ab2 + b3, то мы можем представить его в виде
произведения (a + b)(a + b)(a + b). Иными словами,
разложить на множители (a + b), (a + b) и (a + b).

a3 + 3a2b + 3ab2 + b3 = (a + b)(a + b)(a + b)

Пример 1. Разложить на множители многочлен m3 + 6m2n +


12mn2 + 8n3

Прежде чем применять формулу куба суммы, следует


проанализировать данный многочлен. А именно, убедиться
что перед нами действительно куб суммы двух выражений.

Чтобы убедиться, что исходное выражение является кубом


суммы двух выражений, следует узнать чему в данном
случае равна переменная a и чему равна переменная b.

Первый член данного многочлена является результатом


возведения в куб одночлена m

m3 = m3

Последний член 8n3 является результатом возведения в куб


одночлена 2n

(2n)3 = 8n3

Второй член 6m2n является утроенным произведением


квадрата первого выражения m и последнего 2n

3 × m2 × 2n = 6m2n


1581

Третий член 12mn2 является утроенным произведением


первого выражения m и квадрата последнего выражения 2n

3 × m × (2n)2 = 3 × m × 4n2 = 12mn2

То есть исходный многочлен m3 + 6m2n + 12mn2 + 8n3 по всем


параметрам соответствует кубу суммы двух выражений.
Переменной a в данном многочлене соответствует m, а
переменной b соответствует 2n

a=m
b = 2n

Тогда можно сделать вывод, что когда-то выражение


m3 + 6m2n + 12mn2 + 8n3 выглядело в виде куба суммы
(m + 2n)3, но в результате применения формулы куба суммы
оно обратилось в многочлен m3 + 6m2n + 12mn2 + 8n3. Наша
задача — вернуть ему былую форму, то есть представить в
виде (m + 2n)3

m3 + 6m2n + 12mn2 + 8n3 = (m + 2n)3

А поскольку (m + 2n)3 это произведение трёх сомножителей,


каждый из которых равен многочлену (m + 2n), то исходный
многочлен m3 + 6m2n + 12mn2 + 8n3 можно представить в виде
разложения на множители (m + 2n), (m + 2n) и (m + 2n)

m3 + 6m2n + 12mn2 + 8n3 = (m + 2n)(m + 2n)(m + 2n)

Пример 2. Разложить на множители многочлен 125x3 +


75x2 + 15x + 1
1582

Первый член данного многочлена является результатом


возведения в куб одночлена 5x

(5x)3 = 125x3

Последний член 1 является результатом возведения в куб


одночлена 1

13 = 1

Второй член 75x2 является утроенным произведением


квадрата первого выражения 5x и последнего 1

3 × (5x)2 × 1 = 3 × 25x2 = 75x2

Третий член 15x является утроенным произведением


первого выражения 5x и квадрата второго выражения 1

3 × 5x × 12 = 15x

Воспользуемся формулой a3 + 3a2b + 3ab2 + b3 = (a + b)3. Роль


переменной a играет одночлен 5x, а роль переменной b
играет одночлен 1

a = 5x
b=1

Поэтому,

125x3 + 75x2 + 15x + 1 = (5x + 1)3

А поскольку (5x + 1)3 это произведение трёх сомножителей,


каждый из которых равен многочлену (5x + 1), то исходный
многочлен 125x3 + 75x2 + 15x + 1 можно представить в виде
разложения на множители (5x + 1), (5x + 1) и (5x + 1)
1583

125x3 + 75x2 + 15x + 1 = (5x + 1)(5x + 1)(5x + 1)

Разложение многочлена на множители по формуле куба


разности двух выражений

Как и по формуле куба суммы двух выражений, многочлен


можно разложить на множители по формуле куба разности
двух выражений.

Вспомним, как выглядит формула куба разности двух


выражений:

(a − b)3 = a3 − 3a2b + 3ab2 − b3

Если в этой формуле поменять местами левую и правую


часть, то получим:

a3 − 3a2b + 3ab2 − b3 = (a − b)3

Поскольку правая часть это произведение трёх


сомножителей, каждый из которых равен (a − b), то
многочлен вида a3 − 3a2b + 3ab2 − b3 можно разложить на
множители (a − b), (a − b) и (a − b).

a3 − 3a2b + 3ab2 − b3 = (a − b)(a − b)(a − b)

Пример 1. Разложить на множители многочлен 64 − 96x +


48x2 − 8x3

Прежде чем применять формулу куба разности, следует


проанализировать данный многочлен. А именно, убедиться
что перед нами действительно куб разности двух выражений.
1584

Чтобы убедиться, что исходное выражение является кубом


разности двух выражений, следует узнать чему в данном
случае равна переменная a и чему равна переменная b.

Первый член данного многочлена является результатом


возведения в куб одночлена 4

43 = 64

Последний член 8x3 является результатом возведения в куб


одночлена 2x

(2x)3 = 8x3

Второй член 96x является утроенным произведением


квадрата первого выражения 4 и последнего 2x

3 × 42 × 2x = 3 × 16 × 2x = 96x

Третий член 48x2 является утроенным произведением


первого выражения 4 и квадрата второго выражения 2x

3 × 4 × (2x)2 = 3 × 4 × 4x2 = 48x2

Видим, что исходный многочлен 64 − 96x + 48x2 − 8x3 по всем


параметрам соответствует кубу разности двух выражений.
Переменной a в данном многочлене соответствует 4, а
переменной b соответствует 2x

a=4
b = 2x

Тогда можно сделать вывод, что когда-то выражение


64 − 96x + 48x2 − 8x3 выглядело в виде куба разности
(4 − 2x)3, но в результате применения формулы куба
1585

разности оно обратилось в многочлен 64 − 96x + 48x2 − 8x3.


Наша задача — вернуть ему былую форму, то есть
представить в виде (4 − 2x)3

64 − 96x + 48x2 − 8x3 = (4 − 2x)3

А поскольку (4 − 2x)3 это произведение трёх сомножителей,


каждый из которых равен (4 − 2x), то исходный многочлен
64 − 96x + 48x2 − 8x3 можно представить в виде разложения
на множители (4 − 2x), (4 − 2x) и (4 − 2x)

64 − 96x + 48x2 − 8x3 = (4 − 2x)(4 − 2x)(4 − 2x)

Пример 2. Разложить на множители многочлен 27 − 135x +


225x2 − 125x3

Первый член данного многочлена является результатом


возведения в куб одночлена 3

33 = 27

Последний член 125 является результатом возведения в куб


одночлена 5x

(5x)3 = 125x3

Второй член 135x является утроенным произведением


квадрата первого выражения 3 и последнего 5x

3 × 32 × 5x = 3 × 9 × 5x = 135x

Третий член 225x2 является утроенным произведением


первого выражения 3 и квадрата второго выражения 5x

3 × 3 × (5x)2 = 3 × 3 × 25x2 = 225x2


1586

Воспользуемся формулой a3 − 3a2b + 3ab2 − b3 = (a − b)3. Роль


переменной a играет одночлен 3, а роль переменной b
играет одночлен 5x

a=3
b = 5x

Поэтому,

27 − 135x + 225x2 − 125x3 = (3 − 5x)3

А поскольку (3 − 5x)3 это произведение трёх сомножителей,


каждый из которых равен многочлену (3 − 5x), то исходный
многочлен 27 − 135x + 225x2 − 125x3 можно представить в
виде разложения на множители (3 − 5x), (3 − 5x) и (3 − 5x)

125x3 + 75x2 + 15x + 1 = (3 − 5x)(3 − 5x)(3 − 5x)

Разложение многочлена на множители по формуле разности


квадратов двух выражений

Вспомним, как выглядит формула умножения разности двух


выражений на их сумму:

(a − b)(a + b) = a2 − b2

Если в этой формуле поменять местами левую и правую


часть, то получим:

a2 − b2 = (a − b)(a + b)

Эту формулу называют разностью квадратов. Она


позволяет разложить выражение вида a2 − b2 на множители
(a − b) и (a + b).
1587

Пример 1. Разложить на множители многочлен 16x2 − 25y2

Чтобы воспользоваться формулой a2 − b2 = (a − b)(a + b),


следует узнать чему в данном случае равна переменная a и
чему равна переменная b.

Первый член 16x2 является результатом возведения в


квадрат одночлена 4x

(4x)2 = 16x2

Второй член 25y2 является результатом возведения в


квадрат одночлена 5y

(5y)2 = 25y2

То есть в данном случае переменной a соответствует


одночлен 4x, а переменной b соответствует одночлен 5y

a = 4x
b = 5y

Теперь можно воспользоваться формулой a2 − b2 = (a − b)


(a + b). Подставим в неё наши значения a и b

(4x)2 − (5y)2 = (4x − 5y)(4x + 5y)

Полностью решение можно записать так:

16x2 − 25y2 = (4x)2 − (5y)2 = (4x − 5y)(4x + 5y)

Для проверки можно выполнить умножение (4x − 5y)(4x + 5y).


Если мы всё сделали правильно, то должны получить 16x2 −
25y2

(4x − 5y)(4x + 5y) = 16x2 − 20xy + 20xy − 25y2 = 16x2 − 25y2


1588

Пример 2. Разложить на множители многочлен x2 − y2

В данном случае переменной a соответствует x, а


переменной b соответствует y. Тогда по формуле квадрата
разности имеем:

x2 − y2 = (x − y)(x + y)

Случай как в данном примере является наиболее простым,


поскольку здесь сразу видно чему равно a и чему равно b.

Чаще всего члены, из которых состоит исходная разность,


являются результатами возведения во вторую степень каких-
нибудь одночленов. Чтобы узнать чему в таком случае равны
a и b, нужно как в первом примере представить члены
исходной разности в виде одночленов возведённых в
квадрат.

Например, чтобы разложить многочлен 4x4 − 9y6 на


множители, нужно исходные члены представить в виде
одночленов возведённых в квадрат. Первый член в виде
одночлена, возведенного в квадрат, можно записать как
(2x2)2, поскольку вычисление этого выражение даёт в
результате 4x4

(2x2)2 = 4x4

А член 9y6 в виде одночлена, возведенного в квадрат, можно


записать как (3y3)2, поскольку вычисление этого выражение
даёт в результате 9y6

(3y3)2 = 9y6
1589

Теперь мы знаем, чему равны a и b. Они равны 2x2 и 3y3


соответственно. Подставим их в формулу a2 − b2 = (a − b)(a +
b)

(2x2)2 − (3y3)2 = (2x2 − 3y3)(2x2 + 3y3)

Полностью решение можно записать так:

4x4 − 9y6 = (2x2)2 − (3y3)2 = (2x2 − 3y3)(2x2 + 3y3)

Несмотря на простоту разложения по формуле разности


квадратов, частые ошибки приходятся именно на эти задачи.
Чтобы убедиться, что задача решена правильно, не мешает
выполнить умножение в получившемся разложении. Если
задача решена правильно, то должен получиться
изначальный многочлен.

Проверим умножением данный пример. У нас должен


получиться многочлен 4x4 − 9y6

(2x2 − 3y3)(2x2 + 3y3) = 2x2(2x2 + 3y3) − 3y3(2x2 + 3y3)


= 4x4 + 6x2y3 − 6x2y3 − 9y6 = 4x4 − 9y6

Пример 4. Разложить на множители многочлен 81 − 64

Представим члены исходной разности в виде одночленов


возведенных в квадрат. Далее воспользуемся формулой
разности квадратов:

81 − 64 = 92 − 82 = (9 − 8)(9 + 8)


1590

Разложение многочлена на множители по формуле сумме


кубов двух выражений

Мы помним, что произведение суммы двух выражений и


неполного квадрата их разности равно сумме кубов этих
выражений:

(a + b)(a2 − ab + b2) = a3 + b3

Если в этой формуле поменять местами левую и правую


часть, то получим формулу, называемую суммой кубов
двух выражений:

a3 + b3 = (a + b)(a2 − ab + b2)

Эта формула позволяет разложить выражение вида a3 + b3


на множители (a + b) и (a2 − ab + b2).

Пример 1. Разложить на множители многочлен 27x3 + 64y3

Представим члены 27x3 и 64y3 в виде одночленов,


возведённых в куб

27x3 + 64y3 = (3x)3 + (4y)3

Теперь воспользуемся формулой суммы кубов. Переменная


a в данном случае равна 3x, переменная b равна 4y

27x3 + 64y3 = (3x)3 + (4y)3 = (3x + 4y)((3x)2 − 3x × 4y + (4y)2) =


(3x + 4y)(9x2 − 12xy + 16y2)

Пример 2. Разложить на множители многочлен 125 + 8

Представим члены 125 и 8 в виде одночленов, возведённых


в куб:
1591

125 + 8 = 53 + 23

Далее воспользуемся формулой суммы кубов:

125 + 8 = 53 + 23 = (5 + 2)(25 − 10 + 4)

Разложение многочлена на множители по формуле разности


кубов двух выражений

Произведение разности двух выражений и неполного


квадрата их суммы равно разности кубов этих выражений:

(a − b)(a2 + ab + b2) = a3 − b3

Если в этой формуле поменять местами левую и правую


часть, то получим формулу, называемую разностью кубов
двух выражений:

a3 − b3 = (a − b)(a2 + ab + b2)

Эта формула позволяет разложить выражение вида a3 − b3


на множители (a − b) и (a2 + ab + b2).

Пример 1. Разложить на множители многочлен 64x3 − 27y3

Представим члены 64x3 и 27y3 в виде одночленов,


возведённых в куб:

64x3 − 27y3 = (4x)3 − (3y)3

Теперь воспользуемся формулой разности кубов.


Переменная a в данном случае равна 4x, переменная b
равна 3y
1592

64x3 − 27y3 = (4x)3 − (3y)3 = (4x − 3y)((4x)2 + 4x × 3y + (3y)2) =


(4x − 3y)(16x2 + 12xy + 9y2)

Пример 2. Разложить на множители многочлен 64 − 27

Представим члены 64 и 27 в виде одночленов, возведённых


в куб:

64 − 27 = 43 − 33 = (4 − 3)(16 + 12 + 9)

Пример 3. Разложить на множители многочлен 125x3 − 1

Представим члены 125x3 и 1 в виде одночленов,


возведённых в куб:

125x3 − 1 = (5x)3 − 13

Теперь воспользуемся формулой разности кубов.


Переменная a в данном случае равна 5x, переменная b
равна 1

125x3 − 1 = (5x)3 − 13 = (5x − 1)((5x)2 + 5x × 1 + 12) =


(5x − 1)(25x2 + 5x + 1)

Разложение многочлена на множители различными


способами

К некоторым многочленам можно применять различные


способы разложения на множители. Например, к одному
многочлену можно применить способ вынесения общего за
скобки, а затем воспользоваться одной из формул
сокращённого умножения.
1593

Пример 1. Разложить на множители многочлен ax2 − ay2 

В данном многочлене содержится общий множитель a.


Вынесем его за скобки:

ax2 − ay2 = a(x2 − y2)

При этом в скобках образовался многочлен, который


является разностью квадратов. Применив формулу разности
квадратов. Тогда получим:

ax2 − ay2 = a(x2 − y2) = a(x − y)(x + y)

Пример 2. Разложить на множители многочлен 3x2 + 6xy +


3y2

Вынесем за скобки общий множитель 3

3x2 + 6xy + 3y2 = 3(x2 + 2xy + y2)

В скобках образовался многочлен, который является


квадратом суммы двух выражений, а именно выражений x и
y. Тогда этот квадрат суммы можно представить как (x + y)2 и
далее записать в виде двух сомножителей, каждый из
которых равен (x + y)

3x2 + 6xy + 3y2 = 3(x2 + 2xy + y2) = 3(x + y)2 = 3(x + y)(x + y)

Задания для самостоятельного решения


Задание 1. Следующий многочлен разложите на множители
способом группировки:
1594

Показать решение
Задание 2. Следующий многочлен разложите на множители
способом группировки:

Показать решение
Задание 3. Следующий многочлен разложите на множители
способом группировки:

Показать решение
Задание 4. Следующий многочлен разложите на множители
способом группировки:

Показать решение
Задание 5. Следующий многочлен разложите на множители
способом группировки:

Показать решение
Задание 6. Следующий многочлен разложите на множители
способом группировки:

Показать решение
Задание 7. Разложите на множители многочлен:
x2 + 12x + 36
Показать решение
Задание 8. Разложите на множители многочлен:
8xy + y2 + 16x2
1595

Показать решение
Задание 9. Разложите на множители многочлен:

Показать решение
Задание 10. Разложите на множители многочлен:

Показать решение
Задание 11. Разложите на множители многочлен:

Показать решение
Задание 12. Разложите на множители многочлен:

Показать решение
Задание 13. Разложите на множители многочлен:

Показать решение
Задание 14. Разложите на множители многочлен:

Показать решение
Задание 15. Разложите на множители многочлен:

Показать решение
Задание 16. Разложите на множители многочлен:

Показать решение
1596

Задание 17. Разложите на множители многочлен:

Показать решение
Задание 18. Разложите на множители многочлен:

Показать решение
Задание 19. Разложите на множители многочлен:

Показать решение
Задание 20. Разложите на множители многочлен:

Показать решение
Задание 21. Разложите на множители многочлен:

Показать решение
Задание 22. Разложите на множители многочлен:

Показать решение
Задание 23. Разложите на множители многочлен:

Показать решение
Задание 24. Разложите на множители многочлен:

Показать решение
1597

Задание 25. Разложите на множители многочлен:

Показать решение
Задание 26. Разложите на множители многочлен:

Показать решение
Задание 27. Разложите на множители многочлен:

Показать решение
Задание 28. Разложите на множители многочлен:

Показать решение
Задание 29. Разложите на множители многочлен:

Показать решение
Задание 30. Разложите на множители многочлен:

Показать решение
Задание 31. Разложите на множители многочлен:

Показать решение
Задание 32. Разложите на множители многочлен:

Показать решение
1598

Задание 33. Разложите на множители многочлен:

Показать решение
Задание 34. Разложите на множители многочлен:

Показать решение
Задание 35. Разложите на множители многочлен:

Показать решение
Задание 36. Разложите на множители многочлен:

Показать решение
Задание 37. Разложите на множители многочлен:

Показать решение
Задание 38. Разложите на множители многочлен:

Показать решение
Задание 39. Разложите на множители многочлен:

Показать решение
Задание 40. Разложите на множители многочлен:

Показать решение
1599

Задание 41. Разложите на множители многочлен:

Показать решение
Задание 42. Разложите на множители многочлен:

Показать решение
Задание 43. Разложите на множители многочлен:

Показать решение
Задание 44. Разложите на множители многочлен:

Показать решение
Задание 45. Разложите на множители многочлен:

Показать решение
Задание 46. Разложите на множители многочлен:

Показать решение
Задание 47. Разложите на множители многочлен:

Показать решение
Задание 48. Разложите на множители многочлен:

Показать решение
1600

Задание 49. Разложите на множители многочлен:

Показать решение
Задание 50. Разложите на множители многочлен:

Показать решение
Задание 51. В следующем выражении вынесите за скобки
общий множитель 2a, затем выражение в скобках разложите
на множители:

Показать решение
Задание 52. В следующем выражении вынесите за скобки
общий множитель 4, затем выражение в скобках разложите
на множители:

Показать решение
Задание 53. В следующем выражении вынесите за скобки
общий множитель 2x2y2, затем выражение в скобках
разложите на множители:

Показать решение
Задание 54. В следующем выражении вынесите за скобки
общий множитель 4x3y3, затем выражение в скобках
разложите на множители:

Показать решение
1601
1602

Деление многочленов

Продолжаем изучать многочлены. В данном уроке мы


научимся их делить.

Предварительные навыки


o
 Деление
 Степень с натуральным показателем
 Периметр, площадь и объём
 Многочлены

Содержание урока


o Деление многочлена на одночлен
o Деление одночлена на многочлен
o Деление многочлена на многочлен
o Деление многочлена на многочлен с остатком
o Когда деление многочленов невозможно
o Задания для самостоятельного решения

Деление многочлена на одночлен

Чтобы разделить многочлен на одночлен, нужно разделить


на этот одночлен каждый член многочлена, затем сложить
полученные частные.

Например, разделим многочлен 15x2y3 + 10xy2 + 5xy3 на


одночлен xy. Запишем это деление в виде дроби:
1603

Теперь делим каждый член многочлена 15x2y3 + 10xy2 + 5xy3


на одночлен xy. Получающиеся частные будем складывать:

Получили привычное для нас деление одночленов.


Выполним это деление:

Таким
образом, при делении многочлена 15x2y3 + 10xy2 + 5xy3 на
одночлен xy получается многочлен 15xy2 + 10y + 5y2.

При делении одного числа на другое, частное должно быть


таким, чтобы при его перемножении с делителем,
получалось делимое. Это правило сохраняется и при
делении многочлена на одночлен.

В нашем примере произведение полученного многочлена


15xy2 + 10y + 5y2 и делителя xy должно быть равно
многочлену 15x2y3 + 10xy2 + 5xy3, то есть исходному
делимому. Проверим так ли это:

(15xy2 + 10y + 5y2)xy = 15x2y3 + 10xy2 + 5xy3

Деление многочлена на одночлен очень похоже на сложение


дробей с одинаковыми знаменателями. Мы помним, что для
сложения дробей с одинаковыми знаменателями, нужно
1604

сложить их числители, а знаменатель оставить без


изменений.

Например, чтобы сложить дроби , и нужно записать


следующее выражение:

Если мы вычислим выражение , то получим дробь ,


значение которой равно 1,5.

При этом выражение мы можем вернуть в исходное

состояние , и вычислить по отдельности каждую


дробь, затем сложить полученные частные. Результат по
прежнему будет равен 1,5

Тоже самое происходит при делении многочлена на


одночлен. Одночлен берёт на себя роль общего знаменателя
для всех членов многочлена. Например, при делении
многочлена ax + bx + cx на многочлен x, образуется три
дроби с общим знаменателем x

Вычисление каждой дроби даст в результате многочлен


a + b + c
1605

Пример 2. Разделить многочлен 8m3n + 24m2n2 на одночлен


8m2n

Пример 3. Разделить многочлен 4c2d − 12c4d3 на одночлен


−4c2d

Деление одночлена на многочлен

Не существует тождественного преобразования,


позволяющего разделить одночлен на многочлен.

Допустим, мы захотели разделить одночлен 2xy на


многочлен 5x + 3y + 5.

Результатом этого деления должен быть многочлен,


перемножение которого с многочленом 5x + 3y + 5 даёт
одночлен 2xy. Но не существует многочлена, перемножение
которого с многочленом 5x + 3y + 5 давало бы в результате
одночлен 2xy, поскольку перемножение многочленов даёт в
результате многочлен, а не одночлен.
1606

Но в учебниках можно встретить задания на нахождение


значения выражения при заданных значениях переменных. В
исходных выражениях таких заданий бывает выполнено
деление одночлена на многочлен. В этом случае никаких
преобразований выполнять не нужно. Достаточно подставить
значения переменных в исходное выражение и вычислить
получившееся числовое выражение.

Например, найдём значение выражения при x = 2.

Выражение представляет собой деление одночлена


на многочлен. В данном случае мы не сможем выполнить
какие-либо преобразования. Единственное, что мы сможем
сделать — это подставить число 2 в исходное выражение
вместо переменной x и найти значение выражения:

Деление многочлена на многочлен

Если первый многочлен умножить на второй многочлен,


получается третий многочлен. Например, если умножить
многочлен x + 5 на многочлен x + 3, получается многочлен
x2 + 8x + 15

(x + 5)(x + 3) = x2 + 5x + 3x + 15 = x2 + 8x + 15

(x + 5)(x + 3) = x2 + 8x + 15
1607

Если произведение разделить на множитель, то получится


множимое. Это правило распространяется не только для
чисел, но и для многочленов.

Тогда согласно этому правилу, деление полученного нами


многочлена x2 + 8x + 15 на многочлен x + 3 должно давать в
результате многочлен x + 5.

Деление многочлена на многочлен выполняется уголком.


Отличие будет в том, что при делении многочленов не нужно
определять первое неполное делимое, как в случае деления
обычных чисел.

Выполним уголком деление многочлена x2 + 8x + 15 на


многочлен x + 3. Так мы поэтапно увидим, как получается
многочлен x + 5.

В данном случае результат нам известен заранее. Это будет


многочлен x + 5. Но чаще всего результат бывает
неизвестным. Поэтому решение будем комментировать так,
будто результат нам неизвестен.

Результатом деления должен быть новый многочлен. Члены


этого многочлена будут появляться один за другим в
процессе деления.

Сейчас наша задача найти первый член нового многочлена.


Как это сделать?
1608

Когда мы изначально перемножали многочлены x + 5 и x + 3,


мы сначала умножили первый член первого многочлена на
первый член второго многочлена. Тем самым мы получили
первый член третьего многочлена:

Если мы обратно разделим первый член третьего


многочлена на первый член второго многочлена, то получим
первый член первого многочлена. А это то, что нам нужно.
Ведь мы должны прийти к многочлену x + 5.

Этот же принцип нахождения первого члена будет


выполняться и при решении других задач на деление
многочленов.

Итак, чтобы найти первый член нового многочлена, нужно


первый член делимого разделить на первый член делителя.

Если первый член делимого (в нашем случае это x2)


разделить на первый член делителя (это x), получится x. То
есть первым членом нового многочлена является x.
Записываем его под правым углом:

Теперь, как и при делении обычных чисел, умножаем x на


делитель x + 3. На этом этапе нужно суметь умножить
одночлен на многочлен. При умножении x на x + 3,
получается x2 + 3x. Записываем этот многочлен под делимым
x2+ 8x+ 15 так, чтобы подобные члены располагались друг
под другом:
1609

Теперь из делимого x2 + 8x + 15 вычитаем x2 + 3x. Подобные


члены вычитаем из подобных им членов. Если из x2 вычесть
x2, получится 0. Ноль не записываем. Далее если из 8x
вычесть 3x, получится 5x. Записываем 5x так, чтобы этот
член оказался под членами 3x и 8x

Теперь, как и при делении обычных чисел, сносим


следующий член делимого. Следующий член это 15. Сносить
его нужно вместе со своим знаком:

Теперь делим многочлен 5x + 15 на x + 3. Для этого нужно


найти второй член нового многочлена. Чтобы его найти,
нужно первый член делимого (сейчас это член 5x) разделить
на первый член делителя (это член x). Если 5x разделить на
x, получится 5. То есть вторым членом нового многочлена
является 5. Записываем его под правым углом, вместе со
своим знаком (член 5 в данном случае положителен)
1610

Теперь умножаем 5 на делитель x + 3. При умножении 5 на


x + 3, получается 5x + 15. Записываем этот многочлен под
делимым 5x + 15

Теперь из делимого 5x + 15 вычитаем 5x + 15. Если из


5x + 15 вычесть 5x + 15 получится 0.

На этом деление завершено.

После выполнения деления можно выполнить проверку,


умножив частное на делитель. В нашем случае, если частное
x + 5 умножить на делитель x + 3, должен получаться
многочлен x2 + 8x + 15

(x + 5)(x + 3) = x2 + 5x + 3x + 15 = x2 + 8x + 15

Пример 2. Разделить многочлен x2 − 8x + 7 на многочлен


x − 7

Записываем уголком данное деление:


1611

Находим первый член частного. Разделим первый член


делимого на первый член делителя, получим x. Записываем
x под правым углом:

Умножаем x на x − 7, получаем x2 − 7x. Записываем этот


многочлен под делимым x2 − 8x + 7 так, чтобы подобные
члены располагались друг под другом:

Вычитаем из x2 − 8x + 7 многочлен x2 − 7x. При вычитании x2


из x2 получается 0. Ноль не записываем. А при вычитании
−7x из −8x получается −x, поскольку
−8x − (−7x) = −8x + 7x = −x. Записываем −x под членами −7x
и −8x. Далее сносим следующий член 7

Следует быть внимательным при вычитании отрицательных


членов. Часто на этом этапе допускаются ошибки. Если на
первых порах вычитание в столбик даётся тяжело, то можно
использовать обычное вычитание многочленов в строку,
которое мы изучили ранее. Для этого нужно отдельно
выписать делимое и вычесть из него многочлен, который под
1612

ним располагается. Преимущество этого метода заключается


в том, что следующие члены делимого сносить не нужно —
они автоматически перейдут в новое делимое. Давайте
воспользуемся этим методом:

Вернёмся к нашей задаче. Разделим многочлен −x + 7 на


x − 7. Для этого нужно найти второй член частного. Чтобы его
найти, нужно первый член делимого (сейчас это член −x)
разделить на первый член делителя (это член x). Если −x
разделить на x, получится −1. Записываем −1 под правым
углом вместе со своим знаком:

Умножаем −1 на x − 7, получаем −x + 7. Записываем этот


многочлен под делимым −x + 7

Теперь из −x + 7 вычитаем −x + 7. Если из −x + 7 вычесть


−x + 7 получится 0
1613

Деление завершено. Таким образом, частное от деления


многочлена x2 − 8x + 7 на многочлен x − 7 равно x − 1

Выполним проверку. Умножим частное x − 1 на делитель


x − 7. У нас должен получиться многочлен x2 − 8x + 7

(x − 1)(x − 7) = x2 − x − 7x + 7 = x2 − 8x + 7

Пример 3. Разделить многочлен x6 + 2x4 + x7 + 2x5 на


многочлен x2 + x3

Найдём первый член частного. Разделим первый член


делимого на первый член делителя, получим x4

Умножаем x4 на делитель x2 + x3 и полученный результат


записываем под делимым. Если x4 умножить на x2 + x3
1614

получится x6 + x7. Члены этого многочлена записываем под


делимым так, чтобы подобные члены располагались друг под
другом:

Теперь из делимого вычитаем многочлен x6 + x7. Вычитание


x6 из x6 даст в результате 0. Вычитание x7 из x7 тоже даст в
результате 0. Оставшиеся члены 2x4 и 2x5 снесём:

Получилось новое делимое 2x4 + 2x5. Это же делимое можно


было получить, выписав отдельно многочлен x6 + 2x4 + x7 + 2x5
и вычтя из него многочлен x6 + x7

Разделим многочлен 2x4 + 2x5 на делитель x2 + x3. Как и


раньше сначала делим первый член делимого на первый
член делителя, получим 2x2. Записываем этот член в
частном:
1615

Умножаем 2x2 на делитель x2 + x3 и полученный результат


записываем под делимым. Если 2x2 умножить на x2 + x3
получится 2x4 + 2x5. Записываем члены этого многочлена под
делимым так, чтобы подобные члены располагались друг под
другом. Затем выполним вычитание:

Вычитание многочлена 2x4 + 2x5 из многочлена 2x4 + 2x5 дало


в результате 0, поэтому деление успешно завершилось.

В промежуточных вычислениях члены нового делимого


располагались друг от друга, образуя большие расстояния.
Это было по причине того, что при умножении частного на
делитель, результаты были записаны так, чтобы подобные
члены располагались друг под другом.

Эти расстояния между членами нового делимого образуются


тогда, когда члены исходных многочленов расположены
беспорядочно. Поэтому перед делением желательно
упорядочить члены исходных многочленов в порядке
убывания степеней. Тогда решение примет более
аккуратный и понятный вид.

Решим предыдущий пример, упорядочив члены исходных


многочленов в порядке убывания степеней. Если члены
многочлена x6 + 2x4 + x7 + 2x5 упорядочить в порядке убывания
1616

степеней, то получим многочлен x7 + x6 + 2x5 + 2x4. А если


члены многочлена x2 + x3 упорядочить в порядке убывания
степеней, то получим многочлен x3 + x2

Тогда деление уголком многочлена x6 + 2x4 + x7 + 2x5 на


многочлен x2 + x3 примет следующий вид:

Деление завершено. Таким образом, частное от деления


многочлена x6 + 2x4 + x7 + 2x5 на многочлен x2 + x3 равно x4 +
2x2

Выполним проверку. Умножим частное x4 + 2x2 на делитель


x2 + x3. У нас должен получиться многочлен x6 + 2x4 + x7 + 2x5

(x4 + 2x2)(x2 + x3) = x4 (x2 + x3) + 2x2(x2 + x3) = x6 + 2x4 + x7 + 2x5

При перемножении многочленов члены исходных


многочленов тоже желательно упорядочивать в порядке
убывания степеней. Тогда члены полученного многочлена
тоже будут упорядочены в порядке убывания степеней.

Перепишем умножение (x4 + 2x2)(x2 + x3) упорядочив члены


многочленов в порядке убывания степеней.

(x4 + 2x2)(x3 + x2) = x4(x3 + x2) + 2x2(x3 + x2) = x7 + x6 + 2x5 + 2x4


1617

Пример 4. Разделить многочлен 17x2 − 6x4 + 5x3 − 23x + 7 на


многочлен 7 − 3x2 − 2x

Упорядочим члены исходных многочленов в порядке


убывания степеней и выполним уголком данное деление:

Значит,

Пример 5. Разделить многочлен 4a4 − 14a3b − 24a2b2 − 54b4 на


многочлен a2 − 3ab − 9b2

Найдем первый член частного. Разделим первый член


делимого на первый член делителя, получим 4a2.
Записываем 4a2 в частном:
1618

Умножим 4a2 на делитель a2 − 3ab − 9b2 и полученный


результат запишем под делимым:

Вычтем из делимого полученный многочлен


4a4 − 12a3b − 36a2b2

Теперь делим −2a3b + 12a2b2 − 54b4 на делитель


a2 − 3ab − 9b2. Разделим первый член делимого на первый
член делителя, получим −2ab. Записываем −2ab в частном:

Умножим −2ab на делитель a2 − 3ab − 9b2 и полученный


результат запишем под делимым −2a3b + 12a2b2 − 54b4

Вычтем из многочлена −2a3b + 12a2b2 − 54b4 многочлен


−2a3b + 12a2b2 − 18ab3. При вычитании подобных членов
1619

обнаруживаем, что члены −54b4 и 18ab3 не являются


подобными, а значит их вычитание не даст никакого
преобразования. В этом случае выполняем вычитание там
где это можно, а именно вычтем −2a3b из −2a3b и 6a2b2 из
12a2b2, а вычитание 18ab3 из −54b4 запишем в виде разности
−54b4 − (+18ab3) или −54b4 − 18ab3

Этот же результат можно получить, если выполнить


вычитание многочленов в строку с помощью скобок:

Вернёмся к нашей задаче. Разделим 6a2b2 − 54b4 − 18ab3 на


делитель a2 − 3ab − 9b2. Делим первый член делимого на
первый член делителя, получим 6b2. Записываем 6b2 в
частном:

Умножим 6b2 на делитель a2 − 3ab − 9b2 и полученный


результат запишем под делимым 6a2b2 − 54b4 − 18ab3. Сразу
1620

вычтем этот полученный результат из делимого


6a2b2 − 54b4 − 18ab3

Деление завершено. Таким образом, частное от деления


многочлена 4a4 − 14a3b − 24a2b2 − 54b4 на многочлен
a2 − 3ab − 9b2 равно 4a2 − 2ab + 6b2.

Выполним проверку. Умножим частное 4a2 − 2ab + 6b2 на


делитель a2 − 3ab − 9b2. У нас должен получиться многочлен
4a4 − 14a3b − 24a2b2 − 54b4

Деление многочлена на многочлен с остатком

Как и при делении обычных чисел, при делении многочлена


на многочлен может образоваться остаток от деления.
1621

Для начала вспомним деление обычных чисел с остатком.


Например, разделим уголком 15 на 2. С остатком это
деление будет выполнено так:

То есть при делении 15 на 2 получается 7 целых и 1 в


остатке. Ответ записывается следующим образом:

Рациональное число читается как семь целых плюс одна


вторая. Знак «плюс» по традиции не записывают. Но если
при делении многочлена на многочлен образуется остаток,
то этот плюс записывать нужно.

Например, если при делении многочлена a на многочлен b


получится частное c, да еще останется остаток q, то ответ
будет записан так:

Например, разделим многочлен 2x3 − x2 − 5x + 4 на многочлен


x − 3
1622

Найдем первый член частного. Разделим первый член


делимого на первый член делителя, получим 2x2.
Записываем 2x2 в частном:

Умножим 2x2 на делитель x − 3 и полученный результат


запишем под делимым:

Вычтем из делимого полученный многочлен 2x3 − 6x2

Теперь делим 5x2 − 5x + 4 на делитель x − 3. Разделим


первый член делимого на первый член делителя, получим
5x. Записываем 5x в частном:

Умножим 5x на делитель x − 3 и полученный результат


запишем под делимым 5x2 − 5x + 4
1623

Вычтем из многочлена 5x2 − 5x + 4 многочлен 5x2 − 15x

Теперь делим 10x + 4 на делитель x − 3. Разделим первый


член делимого на первый член делителя, получим 10.
Записываем 10 в частном:

Умножим 10 на делитель x − 3 и полученный результат


запишем под делимым 10x + 4. Сразу вычтем этот
полученный результат из делимого 10x + 4
1624

Число 34, полученное в результате вычитания многочлена


10x − 30 из многочлена 10x + 4, является остатком. Мы не
сможем найти следующий член частного, который при
умножении с делителем x − 3 дал бы нам в результате 34.

Поэтому при делении многочлена 2x3 − 2x2 − 5x + 4 на


многочлен x − 3 получается 2x2 + 5x + 10 и 34 в остатке.
Ответ записывается таким же образом, как и при делении
обычных чисел. Сначала записывается целая часть (она
располагается под правым углом) плюс остаток,
разделенный на делитель:

Когда деление многочленов невозможно

Деление многочлена на многочлен невозможно в случае,


если степень делимого окажется меньше степени делителя.

Например, нельзя разделить многочлен x3 + x на многочлен


x4 + x2, поскольку делимое является многочленом третьей
степени, а делитель — многочленом четвёртой степени.
1625

Вопреки этому запрету можно попробовать разделить


многочлен x3 + x на многочлен x4 + x2, и даже получить
частное x−1, которое при перемножении с делителем будет
давать делимое:

Но при делении многочлена на многочлен должен


получаться именно многочлен, а частное x−1 многочленом не
является. Ведь многочлен состоит из одночленов, а
одночлен в свою очередь это произведение чисел,

переменных и степеней. Выражение x−1 это дробь , которая


не является произведением.

Пусть имеется прямоугольник со сторонами 4 и 2

Площадь этого прямоугольника будет равна 4 × 2 = 8 кв.ед.

Увеличим длину и ширину этого прямоугольника на x


1626

Достроим отсутствующие стороны:

Теперь прямоугольник имеет длину x + 4 и ширину x + 2.


Площадь этого прямоугольника будет равна произведению
(x + 4)(x + 2) и выражаться многочленом x2 + 6x + 8

(x + 4)(x + 2) = x2 + 4x + 2x + 8 = x2 + 6x + 8

При этом мы можем выполнить обратную операцию, а


именно разделить площадь x2 + 6x + 8 на ширину x + 2 и
получить длину x + 4.
1627

Степень многочлена x2 + 6x + 8 равна сумме степеней


многочленов-сомножителей x + 4 и x + 2, а значит ни одна из
степеней многочленов-сомножителей не может превосходить
степень многочлена-произведения. Следовательно, чтобы
обратное деление было возможным, степень делителя
должна быть меньше степени делимого.

Задания для самостоятельного решения


Задание 1. Выполните деление:

Показать решение
Задание 2. Выполните деление:

Показать решение
Задание 3. Выполните деление:

Показать решение
Задание 4. Выполните деление:
1628

Показать решение
Задание 5. Выполните деление:

Показать решение
Задание 6. Выполните деление:

Показать решение
Задание 7. Выполните деление:

Показать решение
Задание 8. Выполните деление:

Показать решение
Задание 9. Выполните деление:

Показать решение
Задание 10. Выполните деление:
1629

Показать решение
Задание 11. Выполните деление:

Показать решение
Задание 12. Выполните деление:

Показать решение
Задание 13. Выполните деление:

Показать решение
Задание 14. Выполните деление:

Показать решение
Задание 15. Выполните деление:

Показать решение
1630

Тождественные преобразования многочленов

Предварительные навыки


o
 Степень с натуральным показателем
 Многочлены
 Разложение многочлена на множители

Содержание урока


o Возведение двучлена в степень
o Возведение трёхчлена в степень
o Выделение полного квадрата из квадратного
трёхчлена
o Задания для самостоятельного решения

Возведение двучлена в степень

Двучлен — это многочлен, состоящий из двух членов. В


прошлых уроках мы возводили двучлен во вторую и третью
степень, тем самым получили формулы сокращенного
умножения:

(a + b)2 = a2 + 2ab + b2

(a + b)3 = a3 + 3a2b + 3ab2 + b3

Но двучлен можно возводить не только во вторую и третью


степень, но и в четвёртую, пятую или более высокую
степень.
1631

К примеру, возведём двучлен a + b в четвертую степень:

(a + b)4

Представим это выражение в виде произведения двучлена


a + b и куба этого же двучлена

(a + b)(a + b)3

Сомножитель (a + b)3 можно заменить на правую часть


формулы куба суммы двух выражений. Тогда получим:

(a + b)(a3 + 3a2b + 3ab2 + b3)

А это обычное перемножение многочленов. Выполним его:

То есть при возведении двучлена a + b в четвертую степень


получается многочлен a4 + 4a3b + 6a2b2 + 4ab3 + b4

(a + b)4 = a4 + 4a3b + 6a2b2 + 4ab3 + b4

Возведение двучлена a + b в четвертую степень можно


выполнить ещё и так: представить выражение (a + b)4 в виде
произведения степеней (a + b)2(a + b)2

(a + b)2(a + b)2

Но выражение (a + b)2 равно a2 + 2ab + b2. Заменим в


выражении (a + b)2(a + b)2 квадраты суммы на многочлен
a2 + 2ab + b2
1632

(a2 + 2ab + b2)(a2 + 2ab + b2)

А это опять же обычное перемножение многочленов.


Выполним его. У нас получится тот же результат, что и
раньше:

Возведение трёхчлена в степень

Трёхчлен — это многочлен, состоящий из трёх членов.


Например, выражение a + b + c является трёхчленом.

Иногда может возникнуть задача возвести трёхчлен в


степень. Например, возведём в квадрат трехчлен a + b + c

(a + b + c)2

Два члена внутри скобок можно заключить в скобки. К


примеру, заключим сумму a + b в скобки:

((a + b) + c)2

В этом случае сумма a + b будет рассматриваться как один


член. Тогда получается, что в квадрат мы возводим не
трёхчлен, а двучлен. Сумма a + b будет первым членом, а
член c — вторым членом. А как возводить в квадрат двучлен
мы уже знаем. Для этого можно воспользоваться формулой
квадрата суммы двух выражений:
1633

(a + b)2 = a2 + 2ab + b2

Применим эту формулу к нашему примеру:

Таким же способом можно возвести в квадрат многочлен,


состоящий из четырёх и более членов. Например, возведем
в квадрат многочлен a + b + c + d

(a + b + c + d)2

Представим многочлен в виде суммы двух выражений: a + b


и c + d. Для этого заключим их в скобки:

((a + b) + (c + d))2

Теперь воспользуемся формулой квадрата суммы двух


выражений:

Выделение полного квадрата из квадратного трёхчлена

Ещё одно тождественное преобразование, которое может


пригодиться при решении задач это выделение полного
квадрата из квадратного трёхчлена.

Квадратным трехчленом называют трёхчлен второй степени.


Например, следующие трехчлены являются квадратными:
1634

Идея выделения полного квадрата из таких трехчленов


заключается в том, чтобы представить исходный квадратный
трехчлен в виде выражения (a + b)2 + c, где (a + b)2 полный
квадрат, а c — некоторое числовое или буквенное
выражение.

Например, выделим полный квадрат из трёхчлена


4x2 + 16x + 19.

Для начала нужно построить выражение вида a2 + 2ab + b2.


Строить мы его будем из трехчлена 4x2 + 16x + 19. Для
начала определимся какие члены будут играть роли
переменных a и b

Роль переменной a будет играть член 2x, поскольку первый


член трехчлена 4x2 + 16x + 19, а именно 4x2 получается если
2x возвести в квадрат:

(2x)2 = 4x2

Итак, переменная a равна 2x

a = 2x

Теперь возвращаемся к исходному трёхчлену и сразу


обращаем внимание на выражение 16x. Это выражение
является удвоенным произведением первого выражения a (в
нашем случае это 2x) и второго пока неизвестного нам
выражения b. Временно поставим на его место
вопросительный знак:
1635

2 × 2x × ? = 16x

Если внимательно посмотреть на выражение 2 × 2x × ? = 16x,


то интуитивно станет понятно, что членом b в данной
ситуации является число 4, поскольку выражение 2 × 2x
равно 4x, и чтобы получить 16x нужно домножить 4x на 4.

2 × 2x × 4 = 16x

Отсюда делаем вывод, что переменная b равна 4

b=4

Значит, нашим полным квадратом будет выражение (2x)2 + 2


× 2x × 4 + 42

Теперь у нас всё готово для выделения полного квадрата из


трёхчлена 4x2 + 16x + 19.

Итак, возвратимся к исходному трехчлену 4x2 + 16x + 19 и


попробуем аккуратно внедрить в него полученный нами
полный квадрат (2x)2 + 2 × 2x × 4 + 42

4x2 + 16x + 19 =

Вместо 4x2 записываем (2x)2

4x2 + 16x + 19 = (2x)2

Далее вместо 16x записываем удвоенное произведение, а


именно 2 × 2x × 4

4x2 + 16x + 19 = (2x)2 + 2 × 2x × 4

Далее прибавляем квадрат второго выражения:

4x2 + 16x + 19 = (2x)2 + 2 × 2x × 4 + 42
1636

А член 19 пока переписываем как есть:

4x2 + 16x + 19 = (2x)2 + 2 × 2x × 4 + 42 + 19

Теперь обратим внимание на то, что полученный нами


многочлен (2x)2 + 2 × 2x × 4 + 42 + 19 не тождественен
изначальному трёхчлену 4x2 + 16x + 19. Убедиться в этом
можно приведя многочлен (2x)2 + 2 × 2x × 4 + 42 + 19 к
стандартному виду:

(2x)2 + 2 × 2x × 4 + 42 + 19 = 4x2 + 16x + 42 + 19

Видим, что получается многочлен 4x2 + 16x + 42 + 19, а


должен был получиться 4x2 + 16x + 19. Это по причине того,
что член 42 был искусственно внедрён в изначальный
трёхчлен с целью организовать полный квадрата из
трёхчлена 4x2 + 16x + 19.

Чтобы сохранить значение исходного многочлена, нужно


после прибавления члена 42 сразу же вычесть его

4x2 + 16x + 19 = (2x)2 + 2 × 2x × 4 + 42 − 42 + 19

Теперь выражение (2x)2 + 2 × 2x × 4 + 42 можно свернуть, то


есть записать в виде (a + b)2. В нашем случае получится
выражение (2x + 4)2

4x2 + 16x + 19 = (2x)2 + 2 × 2x × 4 + 42 − 42 + 19 = (2x + 4)2 − 42


+ 19
1637

Оставшиеся члены −42 и 19 можно сложить. −42 это −16,


отсюда −16 + 19 = 3

4x2 + 16x + 19 = (2x)2 + 2 × 2x × 4 + 42 − 42 + 19 = (2x + 4)2 − 42


+ 19 = (2x + 4)2 + 3

Значит, 4x2 + 16x + 19 = (2x + 4)2 + 3

Пример 2. Выделить полный квадрат из квадратного


трёхчлена x2 + 2x + 2

Сначала построим выражение вида a2 +2ab + b2. Роль


переменной a в данном случае играет x, поскольку x2 = x2.

Следующий член исходного трёхчлена 2x перепишем в виде


удвоенного произведение первого выражения (это у нас x) и
второго выражения b (это будет 1).

2 × x × 1 = 2x

Если b = 1, то полным квадратом будет выражение


x2 + 2x + 12.

Теперь вернёмся к исходному квадратному трёхчлену и


внедрим в него полный квадрата x2 + 2x + 12

x2 + 2x + 2 = x2 + 2x + 12 − 12 + 2 = (x + 1)2 + 1

Как и в прошлом примере член b (в данном примере это 1)


после прибавления сразу был вычтен с целью сохранения
значения исходного трёхчлена.

Рассмотрим следующее числовое выражение:

9+6+2
1638

Значение этого выражения равно 17

9 + 6 + 2 = 17

Попробуем выделить в этом числовом выражении полный


квадрат. Для этого сначала построим выражение вида
a2 + 2ab + b2. Роль переменной a в данном случае играет
число 3, поскольку первый член выражения 9 + 6 + 2, а
именно 9 можно представить как 32.

Второй член 6 представим в виде удвоенного произведения


первого члена 3 и второго 1

2×3×1=6

То есть переменная b будет равна единице. Тогда полным


квадратом будет выражение 32 + 2 × 3 × 1 + 12. Внедрим его в
исходное выражение:

32 + 6 + 2 = 32 + 2 × 3 × 1 + 12 − 12 + 2

Свернем полный квадрат, а члены −12 и 2 слóжим:

32 + 6 + 2 = 32 + 2 × 3 × 1 + 12 − 12 + 2 = (3 + 1)2 + 1

Получилось выражение (3 + 1)2 + 1, которое по прежнему


равно 17

(3 + 1)2+1 = 42 + 1 = 17

Допустим, у нас имеются квадрат и два прямоугольника.


Квадрат со стороной 3 см, прямоугольник со сторонами 2 см
и 3 см, а также прямоугольник со сторонами 1 см и 2 см
1639

Вычислим площадь каждой фигуры. Площадь квадрата будет


составлять 32 = 9 см2, площадь розового прямоугольника —
2 × 3 = 6 см2, площадь сиреневого — 1 × 2 = 2 см2

Запишем сумму площадей этих прямоугольников:

9+6+2

Это выражение можно понимать как объединение квадрата и


двух прямоугольников в единую фигуру:
1640

Тогда получается фигура, площадь которой 17 см2.


Действительно, в представленной фигуре содержится 17
квадратов со стороной 1 см.

Попробуем из имеющейся фигуры образовать квадрат.


Причем максимально большой квадрат. Для этого будем
использовать части от розового и сиреневого
прямоугольника.

Чтобы образовать максимально большой квадрат из


имеющейся фигуры, можно желтый квадрат оставить без
изменений, а половину от розового прямоугольника
прикрепить к нижней части желтого квадрата:

Видим, что до образования полного квадрата не хватает еще


одного квадратного сантиметра. Его мы можем взять от
1641

сиреневого прямоугольника. Итак, возьмем один квадрат от


сиреневого прямоугольника и прикрепим его к образуемому
большому квадрату:

Теперь внимательно посмотрим к чему мы пришли. А именно


на желтую часть фигуры и розовую часть, которая по сути
увеличила прежний жёлтый квадрат. Не означает ли это то,
что была сторона квадрата равная 3 см, и эта сторона была
увеличена на 1 см, что привело в итоге к увеличению
площади?

(3 + 1)2
1642

Выражение (3 + 1)2 равно 16, поскольку 3 + 1 = 4, а 42 = 16.


Этот же результат можно получить, если воспользоваться
формулой квадрата суммы двух выражений:

(3 + 1)2 = 32 + 6 + 1 = 9 + 6 + 1 = 16

Действительно, в образовавшемся квадрате содержится 16


квадратов.

Оставшийся один квадратик от сиреневого прямоугольника


можно прикрепить к образовавшемуся большому квадрату.
Ведь речь изначально шла о единой фигуре:

(3 + 1)2 + 1

Прикрепление маленького квадратика к имеющемуся


большому квадрату описывается выражением (3 + 1)2 + 1. А
это есть выделение полного квадрата из выражения 9 + 6 + 2

9 + 6 + 2 = 32 + 6 + 2 = 32 + 2 × 3 × 1 + 12 − 12 + 2 = (3 + 1)2 + 1


1643

Выражение (3 + 1)2 + 1, как и выражение 9 + 6 + 2 равно 17.


Действительно, площадь образовавшейся фигуры равна 17
см2.

Пример 4. Выполним выделение полного квадрата из


квадратного трёхчлена x2 + 6x + 8

x2 + 6x + 8 = x2 + 2 × x × 3 + 32 − 32 + 8 = (x + 3)2 − 1

В некоторых примерах при построении выражения


a2 + 2ab + b2 не бывает возможным сразу определить
значения переменных a и b.

Например, выполним выделение полного квадрата из


квадратного трёхчлена x2 + 3x + 2

Переменной a соответствует x. Второй член 3x нельзя


представить в виде удвоенного произведения первого
выражения и второго. В этом случае второй член следует
умножить на 2, и чтобы значение исходного многочлена не
1644

изменилось, сразу же выполнить деление на 2. Выглядеть


это будет так:

Получившаяся дробь и содержит значения переменных a


и b. Наша задача суметь правильно их распознать.
Перепишем эту дробь в виде произведения множителя 2,

дроби и  переменной x

Теперь второй член представлен в виде удвоенного


произведения первого выражения и второго. Переменная a,
как было сказано ранее, равна x. А переменная b равна

дроби

Возвращаемся к нашему примеру и прибавляем квадрат


второго выражения, и чтобы значение выражения не
изменилось, сразу же вычитаем его:

Прибавляем оставшийся член 2


1645

Свернём полный квадрат:

Оставшийся квадрат второго выражения и число 2 можно


сложить. В итоге получим:

Пример 6. Выполним выделение полного квадрата из


квадратного трёхчлена 9x2 + 18x + 7

Пример 7. Выполним выделение полного квадрата из


квадратного трёхчлена x2 − 10x + 1

В данном трёхчлене первые два члена связаны знаком


«минус». В этом случае как и раньше нужно выделить
полный квадрат, но это будет квадрат разности. Проще
говоря, нужно построить выражение вида a2 − 2ab + b2.
1646

Пример 8. Выполним выделение полного квадрата из


квадратного трёхчлена 16x2 + 4x + 1

Пример 9. Разложить многочлен x2 + 6x + 8 на множители


при помощи выделения полного квадрата.

Сначала выделим полный квадрат:

Получившийся многочлена (x + 3)2 − 1 является разностью


квадратов, поскольку единица может быть представлена в
виде 12. Воспользуемся формулой разности квадратов и
разложим многочлен (x + 3)2 − 1 на множители:

Задания для самостоятельного решения


Задание 1. Выполните возведение многочлена в степень:

Показать решение
Задание 2. Выполните возведение многочлена в степень:

Показать решение
Задание 3. Выполните возведение многочлена в степень:
1647

Показать решение
Задание 4. Выделите полный квадрат из квадратного
трёхчлена:

Показать решение
Задание 5. Выделите полный квадрат из квадратного
трёхчлена:

Показать решение
Задание 6. В следующем выражении выделите полный
квадрат:

Показать решение
Задание 7. В следующем выражении выделите полный
квадрат:

Показать решение
Задание 8. В следующем выражении выделите полный
квадрат:

Показать решение
Задание 9. В следующем выражении выделите полный
квадрат:

Показать решение
1648

Задание 10. В следующем выражении выделите полный


квадрат:

Показать решение
1649

Квадратный корень

Предварительные навыки

 Степень с натуральным показателем


 Периметр, площадь и объём

Содержание урока


o Основные сведения
o Определения
o Примеры извлечения квадратных корней

 Приближённое значение квадратного корня


 Приближённое значение квадратного корня с
недостатком и избытком
 Границы, в пределах которых располагаются корни
 Тождественные преобразования с квадратными корнями
o Квадратный корень из произведения
o Квадратный корень из дроби
o Вынесение множителя из-под знака корня
o Внесение множителя под знак корня
 Задания для самостоятельного решения

Основные сведения

Чтобы найти площадь квадрата, нужно длину его стороны


возвести во вторую степень.

Найдём площадь квадрата, длина стороны которого 3 см


1650

S = 32 = 9 см2

Теперь решим обратную задачу. А именно, зная площадь


квадрата определим длину его стороны. Для этого
воспользуемся таким инструментом как кóрень. Корень
бывает квадратный, кубический, а также n-й степени.

Сейчас наш интерес вызывает квадратный корень. По


другому его называют кóрнем второй степени.

Для нахождения длины стороны нашего квадрата, нужно


найти число, вторая степень которого равна 9. Таковым
является число 3. Это число и является кóрнем.

Введём для работы с корнями новые обозначения.

Символ кóрня выглядит как . Это по причине того, что слово


корень в математике употребляется как радикал. А слово
радикал происходит от латинского radix (что в переводе
означает корень). Первая буква слова radix это r
впоследствии преобразилась в символ корня  .
1651

Под корнем располагáют подкореннóе выражение. В нашем


случае подкоренным выражением будет число 9 (площадь
квадрата)

Нас интересовал квадратный корень (он же корень второй


степени), поэтому слева над корнем указываем число 2. Это
число называют показателем корня (или степенью корня)

Получили выражение, которое читается так: «квадратный


корень из числа 9». С этого момента возникает новая задача
по поиску самогó корня.

Если число 3 возвести во вторую степень, то получится


число 9. Поэтому число 3 и будет ответом:

Значит квадрат площадью 9 см2 имеет сторону, длина


которой 3 см. Приведённое действие называют
извлечéнием квадрáтного кóрня.

Нетрудно догадаться, что квадратным корнем из числа 9


также является отрицательное число −3. При его возведении
во вторую степень тоже получается число 9

Получается, что выражение   имеет два значения: 3 и −3.


Но длина стороны квадрата не может быть отрицательным
1652

числом, поэтому для нашей задачи ответ будет только один,


а именно 3.

Вообще, квадратный корень имеет два противоположных


значения: положительное и отрицательное.

Например, извлечём квадратный корень из числа 4

Это выражение имеет два значения: 2 и −2, поскольку при


возведении этих чисел во вторую степень, получится один и
тот же результат 4

Поэтому ответ к выражению вида  записывают с плюсом и


минусом. Плюс с минусом означает, что квадратный корень
имеет два противоположных значения.

Запишем ответ к выражению  с плюсом и минусом:

Определения

Дадим определение квадратному корню.

Квадратным корнем из числа a называют такое число b,


вторая степень которого равна a.
1653

То есть число b должно быть таким, чтобы выполнялось


равенство b2 = a. Число b (оно же корень) обозначается через

радикал   так, что  . На практике левая и правая


часть поменяны местами и мы видим привычное

выражение 

Например, квадратным корнем из числá 16 есть число 4,


поскольку число 4 во второй степени равно 16

42 = 16

Корень 4 можно обозначить через радикал   так,

что  .

Также квадратным корнем из числá 16 есть число −4,


поскольку число −4 во второй степени равно 16

(−4)2 = 16

Если при решении задачи интересует только положительное


значение, то корень называют не просто квадратным, а
арифметическим квадратным.

Арифметический квадратный корень из числá a — это


неотрицательное число b (b ≥ 0), при котором выполняется
равенство b2 = a.

В нашем примере квадратными корнями из числá 16


являются корни 4 и −4, но арифметическим из них является
только корень 4.
1654

В разговорном языке можно использовать сокращение. К

примеру, выражение   полностью читается так:


«квадратный корень из числá шестнадцать», а в
сокращённом варианте можно прочитать так: «корень из
шестнадцати».

Не следует путать понятия корень и квадрат. Квадрат это


число, которое получилось в результате возведения какого-
нибудь числá во вторую степень. Например, числа 25, 36, 49
являются квадратами, потому что они получились в
результате возведения во вторую степень чисел 5, 6 и 7
соответственно.

Корнями же являются числа 5, 6 и 7. Они являются теми


числами, которые во второй степени равны 25, 36 и 49
соответственно.

Чаще всего в квадратных корнях показатель кóрня вообще не

указывается. Так, вместо записи можно использовать

запись . Если в учебнике по математике встретится корень


без показателя, то нужно понимать, что это квадратный
корень.

Квадратный корень из единицы равен единице. То есть


справедливо следующее равенство:

Это по причине того, что единица во второй степени равна


единице:
1655

12 = 1

и квадрат, состоящий из одной квадратной единицы, имеет


сторону, равную единице:

Квадратный корень из нуля равен нулю. То есть

справедливо равенство , поскольку 02 = 0.

Выражение вида   смысла не имеет. Например, не имеет

смысла выражение  , поскольку вторая степень любого


числа есть число положительное. Невозможно найти число,
вторая степень которого будет равна −4.

Если выражение вида   возвести во вторую степень, то

есть если записать  , то это выражение будет равно


подкореннóму выражению a

Например, выражение   равно 4


1656

Это потому что выражение  равно значению 2. Но это


значение сразу возвóдится во вторую степень и получается
результат 4.

Еще примеры:

Корень из квадрата числá равен модулю этого числá:

Например, корень из числá 5, возведённого во вторую


степень, равен модулю числá 5

Если во вторую степень возвóдится отрицательное число,


ответ опять же будет положительным. Например, корень из
числá −5, возведённого во вторую степень, равен модулю
числá −5. А модуль числа −5 равен 5

Действительно, если не пользуясь правилом 

, вычислять выражение  обычным методом — сначала


возвести число −5 во вторую степень, затем извлечь
полученный результат, то полýчим ответ 5
1657

Не следует путать правило   с правилом  .

Правило   верно при любом a, тогда как

правило  верно в том случае, если выражение  


имеет смысл.

В некоторых учебниках знак корня может выглядеть без


верхней линии. Выглядит это так:

Примеры: √4, √9, √16.

Мéньшему числу соответствует мéньший корень, а


бóльшему числу соответствует бóльший корень.

Например, рассмотрим числа 49 и 64. Число 49 меньше, чем


число 64.

49 < 64

Если извлечь квадратные корни из этих чисел, то числу 49


будет соответствовать меньший корень, а числу 64 —
бóльший. Действительно, √49 = 7, а √64 = 8,

√49 < √64

Отсюда:

7<8
1658

Примеры извлечения квадратных корней

Рассмотрим несколько простых примеров на извлечение


квадратных корней.

Пример 1. Извлечь квадратный корень √36

Данный квадратный корень равен числу, квадрат которого


равен 36. Таковым является число 6, поскольку 62 = 36

√36 = 6

Пример 2. Извлечь квадратный корень √49

Данный квадратный корень равен числу, квадрат которого


равен 49. Таковым является число 7, поскольку 72 = 49

√49 = 7

В таких простых примерах достаточно знать таблицу


умножения. Так, мы помним, что число 49 входит в таблицу
умножения на семь. То есть:

7 × 7 = 49

Но 7 × 7 это 72

72 = 49

Отсюда, √49 = 7.

Пример 3. Извлечь квадратный корень √100


1659

Данный квадратный корень равен числу, квадрат которого


равен 100. Таковым является число 10, поскольку 102 = 100

√100 = 10

Число 100 это последнее число, корень которого можно


извлечь с помощью таблицы умножения. Для чисел, бóльших
100, квадратные корни можно находить с помощью таблицы
квадратов.

Пример 3. Извлечь квадратный корень √256

Данный квадратный корень равен числу, квадрат которого


равен 256. Чтобы найти это число, воспользуемся таблицей
квадратов.

Нахóдим в таблице квадратов число 256 и двигаясь от него


влево и вверх определяем цифры, которые образуют число,
квадрат которого равен 256.
1660

Видим, что это число 16. Значит √256 = 16.

Пример 4. Найти значение выражения 2√16

В данном примере число 2 умножается на выражение с


корнем. Сначала вычислим корень √16, затем перемнóжим
его с числом 2

Пример 7. Решить уравнение 

В данном примере нужно найти значение переменной x, при


котором левая часть будет равна 4.

Значение переменной x равно 16, поскольку  . Значит


корень уравнения равен 16.

Примечание. Не следует путать корень уравнения и


квадратный корень. Корень уравнения это значение
переменной, при котором уравнение обращается в
верное числовое равенство. А квадратный корень это
число, вторая степень которого равна выражению,
находящемуся под радикалом  .

Подобные примеры решают, пользуясь определением


квадратного корня. Давайте и мы поступим так же.
1661

Из определения мы знаем, что квадратный корень  равен


числу b, при котором выполняется равенство b2 = a.

Применим равенство b2 = a к нашему примеру  . Роль


переменной b у нас играет число 4, а роль переменной a —

выражение, находящееся под корнем , а именно


переменная x

В выражении 42 = x вычислим левую часть, полýчим 16 = x.


Поменяем левую и правую часть местами, полýчим x = 16. В
результате приходим к тому, что нашлось значение
переменной x.

Пример 8. Решить уравнение 

Перенесем −8 в правую часть, изменив знак:

Возведем правую часть во вторую степень и приравняем её к


переменной x

Вычислим правую часть, полýчим 64 = x. Поменяем левую и


правую часть местами, полýчим x = 64. Значит корень

уравнения   равен 64
1662

Пример 9. Решить уравнение 

Воспользуемся определением квадратного корня:

Роль переменной b играет число 7, а роль переменной a —


подкореннóе выражение 3 + 5x. Возведем число 7 во вторую
степень и приравняем его к 3 + 5x

В выражении 72 = 3 + 5x вычислим левую часть полýчим


49 = 3 + 5x. Получилось обычное линейное уравнение.
Решим его:

Корень уравнения   равен  . Выполним проверку,


подставив его в исходное уравнение:
1663

Пример 10. Найти значение выражения 

В этом выражении число 2 умножается на квадратный корень


из числа 49.

Сначала нужно извлечь квадратный корень и перемножить


его с числом 2

Приближённое значение квадратного корня

Не каждый квадратный корень можно извлечь. Извлечь


квадратный корень можно только в том случае, если удаётся
найти число, вторая степень которого равна подкореннóму
выражению.

Например, извлечь квадратный корень  можно, потому


что удаётся найти число, вторая степень которого равна
подкореннóму выражению. Таковым является число 8,

поскольку 82 = 64. То есть


1664

А извлечь квадратный корень  нельзя, потому что


невозможно найти число, вторая степень которого равна 3. В
таком случае говорят, что квадратный корень из числа 3 не
извлекается.

Зато можно извлечь квадратный корень из числа 3


приближённо. Извлечь квадратный корень приближённо
означает найти значение, которое при возведении во вторую
степень будет максимально близко к подкореннóму
выражению.

Приближённое значение ищут с определенной точностью: с


точностью до целых, с точностью до десятых, с
точностью до сотых и так далее.

Найдём значение корня  приближённо с точностью до


десятых. Словосочетание «с точностью до десятых»

говорит о том, что приближённое значение корня  будет


представлять собой десятичную дробь, у которой после
запятой одна цифра.

Для начала найдём ближайшее меньшее число, корень


которого можно извлечь. Таковым является число 1. Корень
из этого числа равен самому этому числу:

√1 = 1

Аналогично находим ближайшее бóльшее число, корень


которого можно извлечь. Таковым является число 4. Корень
из этого числа равен 2

√4 = 2
1665

√1 меньше, чем √4

√1 < √4

А √3 больше, чем √1 но меньше, чем √4. Запишем это в виде


двойного неравенства:

√1 < √3 < √4

Точные значения корней √1 и √4 известны. Это числа 1 и 2

1 < √3 < 2

Тогда очевидно, что значение корня √3 будет представлять


собой десятичную дробь, потому что между числами 1 и 2
нет целых чисел.

Для нахождения приближённого значения квадратного корня


√3 будем проверять десятичные дроби, располагающиеся в
интервале от 1 до 2, возводя их в квадрат. Делать это будем
до тех пор пока не полýчим значение, максимально близкое к
3. Проверим к примеру дробь 1,1

1,12 = 1,21

Получился результат 1,21, который не очень близок к


подкореннóму выражению 3. Значит 1,1 не годится в
качестве приближённого значения квадратного корня √3,
потому что оно малó.

Проверим тогда дробь 1,8

1,82 = 3,24

Получился результат 3,24, который близок к подкореннóму


выражению, но превосходит его на 0,24. Значит 1,8 не
1666

годится в качестве приближенного значения корня √3, потому


что оно великó.

Проверим тогда дробь 1,7

1,72 = 2,89

Получился результат 2,89, который уже близок к


подкореннóму выражению. Значит 1,7 и будет приближённым
значением квадратного корня √3. Напомним, что знак
приближенного значения выглядит как ≈

√3 ≈ 1,7

Значение 1,6 проверять не нужно, потому что в результате


получится число 2,56, которое дальше от трёх, чем значение
2,89. А значение 1,8, как было показано ранее, является уже
большим.

В данном случае мы нашли приближенное значение корня √3


с точностью до десятых. Значение можно получить ещё
более точно. Для этого его следует находить с точностью до
сотых.

Чтобы найти значение с точностью до сотых проверим


десятичные дроби в интервале от 1,7 до 1,8

1,7 < √3 < 1,8

Проверим дробь 1,74

1,742 = 3,0276
1667

Получился результат 3,0276, который близок к подкореннóму


выражению, но превосходит его на 0,0276. Значит значение
1,74 великó для корня √3.

Проверим тогда дробь 1,73

1,732 = 2,9929

Получился результат 2,9929, который близок к подкореннóму


выражению √3. Значит 1,73 будет приближённым значением
квадратного корня √3 с точностью до сотых.

Процесс нахождения приближённого значения квадратного


корня продолжается бесконечно. Так, корень √3 можно
находить с точностью до тысячных, десятитысячных и так
далее:

√3 = 1,732 (вычислено с точностью до тысячных)

√3 = 1,7320 (вычислено с точностью до десятитысячных)

√3 = 1,73205 (вычислено с точностью до ста тысячных).

Ещё квадратный корень можно извлечь с точностью до


целых. Приближённое значение квадратного корня √3 с
точностью до целых равно единице:

√3 ≈ 1

Значение 2 будет слишком большим, поскольку при


возведении этого числа во вторую степень получается число
4, которое больше подкоренного выражения. Нас же
интересуют значения, которые при возведении во вторую
степень равны подкореннóму выражению или максимально
близки к нему, но не превосходят его.
1668

В зависимости от решаемой задачи допускается находить


значение, вторая степень которого больше подкоренного
выражения. Это значение называют приближённым
значением квадратного корня с избытком. Поговорим об этом
подробнее.

Приближенное значение квадратного корня с недостатком


или избытком

Иногда можно встретить задание, в котором требуется найти


приближённое значение корня с недостатком или избытком.

В предыдущей теме мы нашли приближённое значение корня


√3 с точностью до десятых с недостатком. Недостаток
понимается в том смысле, что до значения 3 нам
недоставало ещё некоторых частей. Так, найдя
приближённое значение √3 с точностью до десятых, мы
получили 1,7. Это значение является значением с
недостатком, поскольку при возведении этого числа во
вторую степень полýчим результат 2,89. Этому результату
недостаёт ещё 0,11 чтобы получить число 3. То есть,
2,89 + 0,11 = 3.

С избытком же называют приближённые значения, которые


при возведении во вторую степень дают результат, который
превосходит подкореннóе выражение. Так, вычисляя корень
√3 приближённо, мы проверили значение 1,8. Это значение
является приближённым значением корня √3 с точностью до
десятых с избытком, поскольку при возведении 1,8 во вторую
степень, получаем число 3,24. Этот результат превосходит
подкореннóе выражение на 0,24. То есть 3,24 − 3 = 0,24.
1669

Приближённое значение квадратного корня √3 с точностью


до целых тоже был найден с недостатком:

√3 ≈ 1

Это потому что при возведении единицы в квадрат получаем


единицу. То есть до числа 3 недостаёт ещё 2.

Приближённое значение квадратного корня √3 с точностью


до целых можно найти и с избытком. Тогда этот корень
приближённо будет равен 2

√3 ≈ 2

Это потому что при возведении числа 2 в квадрат получаем


4. Число 4 превосходит подкореннóе выражение 3 на
единицу. Извлекая приближённо квадратный корень с
избытком желательно уточнять, что корень извлечен именно
с избытком:

√3 ≈ 2 (с избытком)

Потому что приближённое значение чаще всего ищется с


недостатком, чем с избытком.

Дополнительно следует упомянуть, что в некоторых


учебниках словосочетания «с точностью до целых», «с
точностью до десятых», с «точностью до сотых»,
заменяют на словосочетания «с точностью до 1», «с
точностью до 0,1», «с точностью до 0,01» соответственно.

Так, если в задании сказано извлечь квадратный корень из


числа 5 с точностью до 0,01, то это значит что корень
следует извлекать приближённо с точностью до сотых:
1670

√5 ≈ 2,23

Пример 2. Извлечь квадратный корень из числа 51 с


точностью до 1

√51 ≈ 7

Пример 3. Извлечь квадратный корень из числа 51 с


точностью до 0,1

√51 ≈ 7,1

Пример 4. Извлечь квадратный корень из числа 51 с


точностью до 0,01

√51 ≈ 7,14

Границы, в пределах которых располагаются корни

Если исходное число принадлежит промежутку [1; 100],


то квадратный корень из этого исходного числа будет
принадлежать промежутку [1; 10].

Например, пусть исходным числом будет 64. Данное число


принадлежит промежутку [1; 100]. Сразу делаем вывод, что
квадратный корень из числа 64 будет принадлежать
промежутку [1; 10]. Теперь вспоминаем таблицу умножения.
Какое перемножение двух одинаковых сомножителей даёт в
результате 64? Ясно, что перемножение 8 × 8, а это есть
82 = 64. Значит квадратный корень из числа 64 есть 8
1671

Пример 2. Извлечь квадратный корень из числа 49

Число 49 принадлежит промежутку [1; 100]. Значит


квадратный корень будет принадлежать промежутку [1; 10].
Этим корнем будет число 7, поскольку 72 = 49

√49 = 7

Пример 2. Извлечь квадратный корень из числа 1

Число 1 принадлежит промежутку [1; 100]. Значит


квадратный корень будет принадлежать промежутку [1; 10].
Этим корнем будет число 1, поскольку 12 = 1

√1 = 1

Пример 3. Извлечь квадратный корень из числа 100

Число 100 принадлежит промежутку [1; 100]. Значит


квадратный корень будет принадлежать промежутку [1; 10].
Этим корнем будет число 10, поскольку 102 = 100

√100 = 10

Понятно, что промежуток [1; 100] содержит ещё и числа,


квадратные корни из которых не извлекаются. Для таких
чисел корень нужно извлекать приближённо. Тем не менее,
приближённый корень тоже будет располагаться в пределах
промежутка [1; 10].
1672

Например, извлечём квадратный корень из числа 37. Нет


целого числа, вторая степень которого была бы равна 37.
Поэтому извлекать квадратный корень следует приближённо.
Извлечём его к примеру с точностью до сотых:

√37 ≈ 6,08

Для облегчения можно находить ближайшее меньшее число,


корень из которого извлекается. Таковым в данном примере
было число 36. Квадратный корень из него равен 6. И далее
отталкиваясь от числа 6, можно находить приближённое
значение корня √37, проверяя различные десятичные дроби,
целая часть которых равна 6.

Квадраты чисел от 1 до 10 обязательно нужно знать


наизусть. Ниже представлены эти квадраты:

12 = 1
22 = 4
32 = 9
42 = 16
52 = 25
62 = 36
72 = 49
82 = 64
92 = 81
102 = 100

И обратно, следует знать значения квадратных корней этих


квадратов:
1673

Если к любому числу от 1 до 10 в конце дописать ноль (или


несколько нулей), и затем возвести это число во вторую
степень, то в полученном числе будет в два раза больше
нулей.

Например, 62 = 36. Допишем к числу 6 один ноль, полýчим 60.


Возведём число 60 во вторую степень, полýчим 3600

602 = 3600

А если к числу 6 дописать два нуля, и возвести это число во


вторую степень, то полýчим число, в котором четыре нуля.
То есть в два раза больше нулей:

6002 = 360000

Тогда можно сделать следующий вывод:

Если исходное число содержит знакомый нам квадрат и


чётное количество нулей, то можно извлечь квадратный
корень из этого числа. Для этого следует извлечь корень
1674

из знакомого нам квадрата и затем записать половину


количества нулей из исходного числа.

Например, извлечём квадратный корень из числа 900.


Видим, что в данном числе есть знакомый нам квадрат 9.
Извлекаем из него корень, получаем 3

Теперь из исходного числа записываем половину от


количества нулей. В исходном числе 900 содержится два
нуля. Половина этого количества нулей есть один ноль.
Записываем его в ответе после цифры 3

Пример 2. Извлечём квадратный корень из числа 90000

Здесь опять же имеется знакомый нам квадрат 9 и чётное


количество нулей. Извлекаем корень из числа 9 и
записываем половину от количества нулей. В исходном
числе содержится четыре нуля. Половиной же этого
количества нулей будет два нуля:

Пример 3. Извлечем квадратный корень из числа 36000000

Здесь имеется знакомый нам квадрат 36 и чётное количество


нулей. Извлекаем корень из числа 36 и записываем половину
от количества нулей. В исходном числе шесть нулей.
Половиной же будет три нуля:
1675

Пример 4. Извлечем квадратный корень из числа 2500

Здесь имеется знакомый нам квадрат 25 и чётное количество


нулей. Извлекаем корень из числа 25 и записываем половину
от количества нулей. В исходном числе два нуля. Половиной
же будет один ноль:

Если подкореннóе число увеличить (или уменьшить) в


100, 10000 то корень увеличится (или уменьшится) в 10,
100 раз соответственно.

Например, . Если увеличим подкореннóе число в 100


раз, то квадратный корень увеличится в 10 раз:

И наоборот, если в равенстве  уменьшим подкореннóе


число в 100 раз, то квадратный корень уменьшится в 10 раз:

Пример 2. Увеличим в равенстве  подкореннóе


число в 10000, тогда квадратный корень 70 увеличиться в
100 раз
1676

Пример 3. Уменьшим в равенстве  подкореннóе


число в 100 раз, тогда квадратный корень 70 уменьшится в
10 раз

Эта закономерность позволяет извлечь квадратный корень


из десятичной дроби, если в данной дроби после запятой
содéржатся две цифры, и эти две цифры образуют знакомый
нам квадрат. В таких случаях данную десятичную дробь
следует умножить на 100. Затем извлечь квадратный корень
из получившегося числа и уменьшить подкореннóе число в
сто раз.

Например, извлечём квадратный корень из числа 0,25. В


данной десятичной дроби после запятой содержатся две
цифры и эти две цифры образуют знакомый нам квадрат 25.

Умнóжим десятичную дробь 0,25 на 100, полýчим 25. А из


числа 25 квадратный корень извлекается легко:

Но нам изначально нужно было извлечь корень из 0,25, а не


из 25. Чтобы исправить ситуацию, вернём нашу десятичную

дробь. Если в равенстве  подкореннóе число


уменьшить в 100 раз, то полýчим под корнем 0,25 и
соответственно ответ уменьшится в 10 раз:
1677

Обычно в таких случаях достаточно уметь передвигáть


запятую. Потому что сдвинуть в числе запятую вправо на две
цифры это всё равно что умножить это число на 100.

В предыдущем примере в подкоренном числе 0,25 можно


было сдвинуть запятую вправо на две цифры, а в
полученном ответе сдвинуть её влево на одну цифру.

Например, извлечем корень из числа 0,81. Мысленно


передвинем запятую вправо на две цифры, полýчим 81.
Теперь извлечём квадратный корень из числа 81, полýчим
ответ 9. В ответе 9 передвинем запятую влево на одну

цифру, полýчим 0,9. Значит, .

Это правило работает и в ситуации, когда после запятой


содержатся четыре цифры и эти цифры образуют знакомый
нам квадрат.

Например, десятичная дробь 0,1225 содержит после запятой


четыре цифры. Эти четыре цифры образуют число 1225,
квадратный корень из которого равен 35.

Тогда можно извлечь квадратный корень и из 0,1225.


Умнóжим данную десятичную дробь на 10000, полýчим 1225.
Из числа 1225 квадратный корень можно извлечь с помощью
таблицы квадратов:
1678

Но нам изначально нужно было извлечь корень из 0,1225, а

не из 1225. Чтобы исправить ситуацию, в равенстве  


подкореннóе число уменьшим в 10000 раз. В результате под
корнем образуется десятичная дробь 0,1225, а правая часть
уменьшится в 100 раз

Эта же закономерность будет работать и при извлечении


корней из дробей вида 12,25. Если цифры из которых состоит
десятичная дробь образуют знакомый нам квадрат, при этом
после запятой содержится чётное количество цифр, то
можно извлечь корень из этой десятичной дроби.

Умнóжим десятичную дробь 12,25 на 100, полýчим 1225.


Извлечём корень из числа 1225
1679

Теперь в равенстве уменьшим подкореннóе число в


100 раз. В результате под корнем образуется число 12,25, и
соответственно ответ уменьшится в 10 раз

Если исходное число принадлежит промежутку [100;


10000], то квадратный корень из этого исходного числа
будет принадлежать промежутку [10; 100].

В этом случае применяется таблица квадратов:

Например, пусть исходным числом будет 576. Данное число


принадлежит промежутку [100; 10000]. Сразу делаем вывод,
что квадратный корень из числа 576 будет принадлежать
промежутку [10; 100]. Теперь открываем таблицу квадратов и
смотрим какое число во второй степени равно 576
1680

Видим, что это число 24. Значит .

Пример 2. Извлечь квадратный корень из числа 432.

Число 432 принадлежит промежутку [100; 10000]. Значит


квадратный корень следует искать в промежутке [10; 100].
Открываем таблицу квадратов и смотрим какое число во
второй степени равно 432. Обнаруживаем, что число 432 в
таблице квадратов отсутствует. В этом случае квадратный
корень следует искать приближённо.

Извлечем квадратный корень из числа 432 с точностью до


десятых.

В таблице квадратов ближайшее меньшее число к 432 это


число 400. Квадратный корень из него равен 20.
Отталкиваясь от числа 20, будем проверять различные
десятичные дроби, целая часть которых равна 20.

Проверим, например, число 20,8. Для этого возведём его в


квадрат:
1681

20,82 = 432,64

Получилось число 432,64 которое превосходит исходное


число 432 на 0,64. Видим, что значение 20,8 великó для
корня √432. Проверим тогда значение 20,7

20,72 = 428,49

Значение 20,7 годится в качестве корня, поскольку в


результате возведения этого числа в квадрат получается
число 428,49, которое меньше исходного числа 432, но
близко к нему. Значит √432 ≈ 20,7.

Необязательно запоминать промежутки чтобы узнать в каких


границах располагается корень. Можно воспользоваться
методом нахождения ближайших квадратов с чётным
количеством нулей на конце.

Например, извлечём корень из числа 4225. Нам известен


ближайший меньший квадрат 3600, и ближайший больший
квадрат 4900

3600 < 4225 < 4900

Извлечём квадратные корни из чисел 3600 и 4900. Это числа


60 и 70 соответственно:

Тогда можно понять, что квадратный корень из числа 4225


располагается между числами 60 и 70. Можно даже найти его
методом подбора. Корни 60 и 70 исключаем сразу, поскольку
это корни чисел 3600 и 4900. Затем можно проверить,
1682

например, корень 64. Возведём его в квадрат (или умнóжим


данное число само на себя)

Корень 64 не годится. Проверим корень 65

Получается 4225. Значит 65 является корнем числа 4225

Тождественные преобразования с квадратными корнями

Над квадратными корнями можно выполнять различные


тождественные преобразования, тем самым облегчая их
вычисление. Рассмотрим некоторые из этих преобразований.

Квадратный корень из произведения

Квадратный корень из произведения это выражение вида

, где a и b некоторые числа.


1683

Например, выражение  является квадратным корнем из


произведения чисел 4 и 9.

Чтобы извлечь такой квадратный корень, нужно по


отдельности извлечь квадратные корни из множителей 4 и 9,

представив выражение  в виде произведения

корней  . Вычислив по отдельности эти корни полýчим


произведение 2 × 3, которое равно 6

Конечно, можно не прибегать к таким манипуляциям, а


вычислить сначала подкореннóе выражение 4 × 9, которое
равно 36. Затем извлечь квадратный корень из числа 36

Но при извлечении квадратных корней из больших чисел это


правило может оказаться весьма полезным.

Допустим, потребовалось извлечь квадратный корень из


числа 144. Этот корень легко определяется с помощью
таблицы квадратов — он равен 12

Но предстáвим, что таблицы квадратов под рукой не


оказалось. В этом случае число 144 можно разложить на
простые множители. Затем из этих простых множителей
составить числа, квадратные корни из которых извлекаются.

Итак, разлóжим число 144 на простые множители:


1684

Получили следующее разложение:

В разложéнии содержатся четыре двойки и две тройки. При


этом все числа, входящие в разложение, перемнóжены. Это
позволяет предстáвить произведения одинаковых
сомножителей в виде степени с показателем 2.

Тогда четыре двойки можно заменить на запись 22 × 22, а две


тройки заменить на 32

В результате будем иметь следующее разложение:

Теперь можно извлекáть квадратный корень из разложения


числа 144
1685

Применим правило извлечения квадратного корня из


произведения:

Ранее было сказано, что если подкореннóе выражение


возведенó во вторую степень, то такой квадратный корень
равен модулю из подкореннóго выражения.

Тогда получится произведение 2 × 2 × 3, которое равно 12

Простые множители представляют в виде степени для


удобства и короткой записи. Допускается также записывать
их под кóрнем как есть, чтобы впоследствии перемнóжив их,
получить новые сомножители.

Так, разложив число 144 на простые множители, мы


получили разложение 2 × 2 × 2 × 2 × 3 × 3. Это разложение
можно записать под кóрнем как есть:

затем перемнóжить некоторые сомножители так, чтобы


получились числа, квадратные корни из которых
извлекаются. В данном случае можно дважды перемнóжить
две двойки и один раз перемнóжить две тройки:

Затем применить правило извлечения квадратного корня из


произведения и получить окончательный ответ:
1686

С помощью правила извлечения квадратного корня из


произведения можно извлекать корень и из других больших
чисел. В том числе, из тех чисел, которых нет в таблице
квадратов.

Например, извлечём квадратный корень из числа 13456.


Этого числа нет в таблице квадратов, поэтому
воспользуемся правилом извлечения квадратного корня из
произведения, предварительно разложив число 13456 на
простые множители.

Итак, разложим число 13456 на простые множители:

В разложении имеются четыре двойки и два числа 29.


Двойки дважды предстáвим как 22. А два числа 29
предстáвим как 292. В результате полýчим следующее
разложение числа 13456
1687

Теперь будем извлекать квадратный корень из разложения


числа 13456

Итак, если a ≥ 0 и b ≥ 0, то  . То есть корень из


произведения неотрицательных множителей равен
произведению корней из этих множителей.

Докажем равенство  . Для этого воспользуемся


определением квадратного корня.

Согласно определению, квадратным корня из числа a есть


число b, при котором выполняется равенство b2 = a.

В нашем случае нужно удостовериться, что правая часть

равенства  при возведении во вторую степень


даст в результате подкореннóе выражение левой части, то
есть выражение ab.

Итак, выпишем правую часть равенства  и


возведём ее во вторую степень:

Теперь воспользуемся правилом возведения в степень


произведения. Согласно этому правилу, каждый множитель
данного произведения нужно возвести в указанную степень:
1688

Ранее было сказано, что если выражение вида  возвести


во вторую степень, то получится подкореннóе выражение.
Применим это правило. Тогда полýчим ab. А это есть

подкореннóе выражение квадратного корня

Значит равенство  справедливо, поскольку при


возведéнии правой части во вторую степень, получается
подкореннóе выражение левой части.

Правило извлечения квадратного корня из произведения


работает и в случае, если под кóрнем располагается более
двух множителей. То есть справедливым будет следующее
равенство:

, при a ≥ 0 и b ≥ 0, c ≥ 0.

Пример 1. Найти значение квадратного корня 

Запишем корень в виде произведения корней,


извлечём их, затем найдём значение полученного
произведения:

Пример 2. Найти значение квадратного корня 


1689

Предстáвим число 250 в виде произведения чисел 25 и 10.


Делать это будем под знáком корня:

Теперь под кóрнем образовалось два одинаковых множителя


10 и 10. Перемнóжим их, полýчим 100

Далее применяем правило извлечения квадратного кóрня из


произведения и получáем окончательный ответ:

Пример 3. Найти значение квадратного корня 

Воспользуемся правилом возведения степени в степень.


Степень 114 предстáвим как (112)2.

Теперь воспользуемся правилом извлечения квадратного


кóрня из квадрата числа:

В нашем случае квадратный корень из числа (112)2 будет


равен 112. Говоря простым языком, внешний показатель
степени 2 исчезнет, а внутренний останется:
1690

Далее возводим число 11 во вторую степень и получаем


окончательный ответ:

Этот пример также можно решить, воспользовавшись


правилом извлечения квадратного корня из произведения.
Для этого подкореннóе выражение 114 нужно записать в виде
произведения 112 × 112. Затем извлечь квадратный корень из
этого произведения:

Пример 4. Найти значение квадратного корня

Перепишем степень 34 в виде (32)2, а степень 56 в виде (53)2

Далее используем правило извлечения квадратного кóрня из


произведения:

Далее используем правило извлечения квадратного кóрня из


квадрата числа:

Вычислим произведение получившихся степеней и полýчим


окончательный ответ:
1691

Сомножители, находящиеся под корнем, могут быть


десятичными дробями. Например, извлечём квадратный

корень из произведения

Запишем корень  в виде произведения корней,


извлечём их, затем найдём значение полученного
произведения:

Пример 6. Найти значение квадратного корня

Пример 7. Найти значение квадратного корня

Если первый сомножитель умножить на число n, а второй


сомножитель разделить на это число n, то произведение не
изменится.

Например, произведение 8 × 4 равно 32

8 × 4 = 32
1692

Умнóжим сомножитель 8 скажем на число 2, а сомножитель


4 раздéлим на это же число 2. Тогда получится
произведение 16 × 2, которое тоже равно 32.

(8 × 2) × (4 : 2) = 32

Это свойство полезно при решении некоторых задач на


извлечение квадратных корней. Сомножители подкореннóго
выражения можно умнóжить и разделить так, чтобы корни из
них извлекались.

Например, извлечём квадратный корень из произведения

. Если сразу воспользоваться правилом извлечения


квадратного корня из произведения, то не полýчится извлечь
корни √1,6 и √90, потому что они не извлекаются.

Проанализировав подкореннóе выражение 1,6 × 90, можно


заметить, что если первый сомножитель 1,6 умножить на 10,
а второй сомножитель 90 разделить на 10, то полýчится
произведение 16 × 9. Из такого произведения квадратный
корень можно извлечь, пользуясь правилом извлечения
квадратного корня из произведения.

Запишем полное решение данного примера:

Процесс умножения и деления можно выполнять в уме.


Также можно пропустить подробную запись извлечения
квадратного корня из каждого сомножителя. Тогда решение
станóвится короче:
1693

Пример 9. Найти значение квадратного корня

Умнóжим первый сомножитель на 10, а второй раздéлим на


10. Тогда под кóрнем образуется произведение 36 × 0,04,
квадратный корень из которого извлекается:

Если в равенстве поменять местами левую и

правую часть, то полýчим равенство  . Это


преобразовáние позволяет упрощáть вычисление некоторых
корней.

Например, узнáем чему равно значение выражения .

Квадратные корни из чисел 10 и 40 не извлекаются.

Воспользуемся правилом  , то есть заменим

выражение из двух корней  на выражение с одним


корнем, под которым будет произведение из чисел 10 и 40

Теперь найдём значение произведения, находящегося под


корнем:

А квадратный корень из числа 400 извлекается. Он равен 20


1694

Сомножители, располагáющиеся под корнем, можно


расклáдывать на множители, группировáть, представлять в
виде степени, а также перемножáть для получения новых
сомножителей, корни из которых извлекаются.

Например, найдём значение выражения .

Воспользуемся правилом

Сомножитель 32 это 25. Предстáвим этот сомножитель как


2 × 24

Перемнóжим сомножители 2 и 2, полýчим 4. А сомножитель


24 предстáвим в виде степени с показателем 2

Теперь воспóльзуемся правилом и вычислим


окончательный ответ:

Пример 12. Найти значение выражения

Воспользуемся правилом
1695

Сомножитель 8 это 2 × 2 × 2, а сомножитель 98 это 2 × 7 × 7

Теперь под кóрнем имеются четыре двойки и две семёрки.


Четыре двойки можно записать как 22 × 22, а две семёрки как
72

Теперь воспользуемся правилом и вычислим


окончательный ответ:

Квадратный корень из дроби

Квадратный корень вида  равен дроби, в числителе


которой квадратный корень из числа a, а в знаменателе —
квадратный корень из числа b

Например, квадратный корень из дроби   равен дроби, в


числителе которой квадратный корень из числа 4, а в
знаменателе — квадратный корень из числа 9
1696

Вычислим квадратные корни в числителе и знаменателе:

Значит, квадратный корень из дроби равен .

Докáжем, что равенство является верным.

Возведём правую часть во вторую степень. Если в

результате полýчим дробь , то это будет означать, что

равенство верно:

Пример 1. Извлечь квадратный корень 

Воспользуемся правилом извлечения квадратного корня из


дроби:
1697

Пример 2. Извлечь квадратный корень 

Переведём подкореннóе выражение в неправильную дробь,


затем воспользуемся правилом извлечения квадратного
корня из дроби:

Пример 3. Извлечь квадратный корень

Квадратным корнем из числа 0,09 является 0,3. Но можно


извлечь этот корень, воспользовавшись правилом
извлечения квадратного корня из дроби.

Предстáвим подкоренное выражение в виде обыкновенной


дроби. 0,09 это девять сотых:

Теперь можно воспользоваться правилом извлечения


квадратного корня из дроби:

Пример 4. Найти значение выражения 

Извлечём корни из 0,09 и 0,25, затем сложим полученные


результаты:
1698

Также можно воспользоваться правилом извлечения


квадратного корня из дроби:

В данном примере первый способ оказался проще и удобнее.

Пример 5. Найти значение выражения 

Сначала вычислим квадратный корень, затем перемнóжим


его с 10. Получившийся результат вычтем из 4

Пример 6. Найти значение выражения 

Сначала найдём значение квадратного корня . Он равен


0,6 поскольку 0,62 = 0,36

Теперь вычислим получившееся выражение. Согласно


порядку действий, сначала надо выполнить умножение,
затем сложение:
1699

Вынесение множителя из-под знака корня

В некоторых задачах может быть полезным вынесение


множителя из-под знака корня.

Рассмотрим квадратный корень из произведения  .


Согласно правилу извлечения квадратного корня из
произведения, нужно извлечь квадратный корень из каждого
множителя данного произведения:

В нашем примере квадратный корень извлекается только из

множителя 4. Его мы извлечём, а выражение  оставим без


изменений:

Это и есть вынесение множителя из-под знака корня.

На практике подкореннóе выражение чаще всего требуется


разложить на множители.

Пример 2. Вынести множитель из-под знака корня в

выражении

Разлóжим подкореннóе выражение на множители 9 и 2.


Тогда полýчим:
1700

Теперь воспользуемся правило извлечения квадратного


корня из произведения. Извлечь можно только корень из
множителя 9. Множитель 2 остáвим под кóрнем:

Пример 3. Вынести множитель из-под знака корня в

выражении

Разлóжим подкореннóе выражение на множители 121 и 3.


Тогда полýчим:

Теперь воспользуемся правилом извлечения квадратного


корня из произведения. Извлечь можно только корень из
множителя 121. Выражение √3 остáвим под корнем:

Пример 4. Вынести множитель из-под знака корня в

выражении

Воспользуемся правилом извлечения квадратного корня из


произведения:

Квадратный корень извлекается только из числа 121.


Извлечём его, а выражение √15 оставим без изменений:
1701

Получается, что множитель 11 вынесен из-под знака корня.


Вынесенный множитель принято записывать до выражения с
корнем. Поменяем выражения √15 и 11 местами:

Пример 5. Вынести множитель из-под знака корня в

выражении

Разлóжим подкореннóе выражение на множители 4 и 3

Воспользуемся правилом извлечения квадратного корня из


произведения:

Извлечём корень из числа 4, а выражение √3 остáвим без


изменений:

Пример 6. Упростить выражение 

Предстáвим второе слагаемое в виде .А

третье слагаемое предстáвим в виде


1702

Теперь в выражениях и вынесем множитель из-


под знака корня:

Во втором слагаемом перемнóжим числа −4 и 4.


Остальное перепишем без изменений:

Замечáем, что получившемся выражении квадратный корень


√3 является общим множителем. Вынесем его за скобки:

Вычислим содержимое скобок, полýчим −1

Если множителем является −1, то записывают только минус.


Единица опускается. Тогда полýчим окончательный ответ
−√3

Внесение множителя под знак корня

Рассмотрим следующее выражение:


1703

В этом выражении число 5 умнóжено на квадратный корень


из числа 9. Найдём значение этого выражения.

Сначала извлечём квадратный корень, затем перемнóжим


его с числом 5.

Квадратный корень из 9 равен 3. Перемнóжим его с числом


5. Тогда полýчим 15

Число 5 в данном случае было множителем. Внесём этот


множитель под знак корня. Но сделать это нужно таким
образом, чтобы в результате наших действий значение
исходного выражения не изменилось. Проще говоря, после
внесения множителя 5 под знак корня, получившееся
выражение по-прежнему должно быть равно 15.

Значение выражения не изменится, если число 5 возвести во


вторую степень и только тогда внести его под корень:

Итак, если данó выражение , и нужно внести множитель a


под знак корня, то надо возвести во вторую степень
множитель a и внести его под корень:

Пример 1. Внести множитель под знак корня в выражении


1704

Возведём число 7 во вторую степень и внесём его под знак


корня:

Пример 2. Внести множитель под знак корня в

выражении 

Возведём число 10 во вторую степень и внесем его под знак


корня:

Пример 3. Внести множитель под знак корня в

выражении 

Вносить под знак корня можно только положительный

множитель. Ранее было сказано, что выражение вида  не


имеет смысла.

Однако, если перед знаком кóрня располагается


отрицательный множитель, то минус можно оставить за
знáком корня, а самó число внести под знак корня.

Пример 4. Внести множитель по знак корня в

выражении 
1705

В этом примере под знак корня внóсится только 3. Минус


остаётся за знáком корня:

Пример 5. Выполнить возведéние в степень в следующем


выражении:

Воспользуемся формулой квадрата суммы двух выражений:

(a + b)2 = a2 + 2ab + b2

Роль переменной a в данном случае играет выражение √3,


роль переменной b — выражение √2. Тогда полýчим:

Теперь необходимо упростить получившееся выражение.

Для выражений и  применим правило .

Ранее мы говорили, что если выражение вида  возвести


во вторую степень, то это выражение будет равно
подкореннóму выражению a.

А в выражении для множителей и применим

правило . То есть заменим произведение


корней на один общий корень:
1706

Приведём подобные слагаемые. В данном случае можно

сложить слагаемые 3 и 2. А в слагаемом вычислить


произведение, которое под кóрнем:

Задания для самостоятельного решения


Задание 1. Найдите значение квадратного корня:

Показать решение
Задание 2. Найдите значение квадратного корня:

Показать решение
Задание 3. Найдите значение квадратного корня:

Показать решение
Задание 4. Найдите значение выражения:

Показать решение
Задание 5. Найдите значение квадратного корня:

Показать решение
1707

Задание 6. Найдите значение квадратного корня:

Показать решение
Задание 7. Найдите значение квадратного корня:

Показать решение
Задание 8. Найдите значения следующих выражений:

Показать решение
Задание 9. Извлеките квадратный корень из числа 4624
Показать решение
Задание 10. Извлеките квадратный корень из числа 11025
1708

Показать решение
Задание 11. Найдите значение квадратного корня:

Показать решение
Задание 12. Найдите значение квадратного корня:

Показать решение
Задание 13. Найдите значение квадратного корня:

Показать решение
Задание 14. Найдите значение квадратного корня:

Показать решение
Задание 15. Найдите значение квадратного корня:

Показать решение
Задание 16. Найдите значение выражения:

Показать решение
Задание 17. Найдите значение выражения:

Показать решение
Задание 18. Найдите значение выражения:
1709

Показать решение
Задание 19. Найдите значение выражения:

Показать решение
Задание 20. Найдите значение выражения:

Показать решение
Задание 21. Найдите значение выражения:

Показать решение
Задание 22. Найдите значение выражения:

Показать решение
Задание 23. Найдите значение выражения:

Показать решение
Задание 24. Найдите значение выражения:

Показать решение
Задание 25. Найдите значение выражения:

Показать решение
Задание 26. Найдите значение выражения:
1710

Показать решение
Задание 27. Найдите значение выражения:

Показать решение
Задание 28. Найдите значение выражения:

Показать решение
Задание 29. Найдите значение выражения:

Показать решение
Задание 30. Найдите значение выражения:

Показать решение
Задание 31. Найдите значение выражения:

Показать решение
Задание 32. Найдите значение выражения:

Показать решение
Задание 33. Найдите значение выражения:

Показать решение
1711

Задание 34. Вынести множитель из-под знака корня:

Показать решение
Задание 35. Вынести множитель из-под знака корня:

Показать решение
Задание 36. Вынести множитель из-под знака корня:

Показать решение
Задание 37. Вынести множитель из-под знака корня:

Показать решение
Задание 38. Вынести множитель из-под знака корня:

Показать решение
Задание 39. Вынести множитель из-под знака корня:

Показать решение
Задание 40. Вынести множитель из-под знака корня:

Показать решение
Задание 41. Вынести множитель из-под знака корня:
1712

Показать решение
Задание 42. Вынести множитель из-под знака корня:

Показать решение
Задание 43. Вынести множитель из-под знака корня:

Показать решение
Задание 44. Вынести множитель из-под знака корня в
следующих выражениях:

Показать решение
Задание 45. Внести множитель под знак корня:

Показать решение
Задание 46. Внести множитель под знак корня:

Показать решение
1713

Задание 47. Внести множитель под знак корня:

Показать решение
Задание 48. Внести множитель под знак корня:

Показать решение
Задание 49. Внести множитель под знак корня:

Показать решение
Задание 50. Внести множитель под знак корня в следующих
выражениях:

Показать решение
1714

Задание 51. Упростить выражение:

Показать решение
Задание 52. Упростить выражение:

Показать решение
Задание 53. Упростить выражение:

Показать решение
Задание 54. Упростить выражение:

Показать решение
Задание 55. Упростить выражение:

Показать решение
Задание 56. Упростить выражение:

Показать решение
Задание 57. Упростить выражение:

Показать решение
Задание 58. Упростить выражение:

Показать решение
1715

Задание 59. Упростить выражение:

Показать решение
Задание 60. Упростить выражение:

Показать решение
1716

Алгоритм извлечения квадратного корня

Квадратный корень легко извлекается с помощью


калькулятора. Для этого достаточно набрать на нём
исходное число и нажать клавишу корня

Если калькулятора под рукой нет, то квадратный корень


извлекают пользуясь алгоритмом извлечения квадратного
корня.

Применение алгоритма может оказаться весьма полезным на


контрольных и экзаменах. Ведь чаще всего на таких
мероприятиях использовать калькулятор запрещено.

Предварительные навыки

 Разряды для начинающих


 Умножение
 Формулы сокращённого умножения

 Квадратный корень

Содержание урока

 Как пользоваться алгоритмом


 Как работает алгоритм
 Задания для самостоятельного решения

Как пользоваться алгоритмом

Рассмотрим применение алгоритма извлечения квадратного


корня на конкретных примерах. О том, почему алгоритм
следует применять именно так, поговорим позже.
1717

Пример 1. Извлечём квадратный корень из числа 4096 с


помощью алгоритма извлечения квадратного корня.

Прежде всего сгруппируем число 4096 по две цифры.


Двигаясь с конца влево сделаем небольшую мéтку:

Сгруппированные цифры исходного числа называют


грáнями, а саму группировку по две цифры разделением на
грáни. Количество грáней позволяет предположить сколько
цифр будет содержаться в извлечённом корне. В нашем
примере извлечённый корень будет содержать две цифры,
поскольку исходное число содержит две грани.

Теперь нужно извлечь квадратный корень из числа 40 с


точностью до целых, получаем 6. Записываем 6 после знака
равенства:

Далее возвóдим число 6 в квадрат и полученный результат


записываем под числом 40

Далее вычитаем из числа 40 число 36, получаем 4.


Записываем это число под 36
1718

Снóсим оставшиеся цифры из под корня, а именно 96.


Получаем остаток 496

Теперь нужно найти следующую цифру корня. Её находят


так. Первую найденную цифру корня, а именно 6 умножаем
на 2, получаем 12. К числу 12 в конце нужно дописáть ещё
одну цифру (эта цифра впоследствии и станет следующей
цифрой корня) и умножить образовавшееся число на ту же
самую дописанную цифру. Полученное произведение должно
быть равно остатку 496 или хотя бы максимально близким к
нему, но не превосходящим его.

Итак, проверим например цифру 5. Допишем её к числу 12 и


умножим образовавшееся число 125 на 5

Получилось число 625, которое больше остатка 496. Значит


цифра 5 не годится в качестве следующей цифры корня.
Проверим тогда цифру 4. Допишем ее к числу 12 и умножим
образовавшееся число 124 на 4
1719

Получилось число 496, которое в точности является нашим


остатком. Значит дописанная к числу 12 цифра 4 является
следующей цифрой корня. Возвращаемся к исходному
примеру и записываем цифру 4 в ответе после цифры 6

А число 496, которое получилось в результате умножения


124 на 4 записываем под остатком 496

Выполняем вычитание 496 − 496 = 0. Ноль в остатке говорит


о том, что решение окончено:

Для удобства поиска второй цифры, слева от остатка


проводят вертикáльную линию и уже за этой линией
записывают умножение. В нашем случае умножение 124 на
4. Результат умножение сразу записывают под остатком:
1720

Итак, квадратный корень из числа 4096 равен 64

Пример 2. Извлечём квадрáтный корень из числа 441 с


помощью алгоритма извлечения квадратного корня.

Прежде всего сгруппируем число 441 по две цифры.


Двигаясь с конца влево сделаем небольшую мéтку. В данном
случае в числе 441 только три цифры. Поэтому группируем
цифры 4 и 1. Крайняя четвёрка слева будет сама по себе:

Теперь нужно извлечь квадратный корень из числа 4 с


точностью до целых, получаем 2. Записываем 2 после знака
равенства:

Далее возвóдим число 2 в квадрат и полученный результат


записываем под числом 4
1721

Вычитаем из числа 4 число 4, получаем 0. Ноль принято не


записывать. Снóсим оставшиеся цифры корня, а именно 41

Теперь нахóдим следующую цифру корня. Первую


найденную цифру корня, а именно 2 умножаем на 2,
получаем 4. К числу 4 в конце нужно дописáть ещё одну
цифру (эта цифра впоследствии и станет следующей цифрой
корня) и умножить получившееся число на ту же самую
дописанную цифру. Полученное произведение должно быть
равно остатку 41 или хотя бы максимально близким ему, но
не превосходящим его.

Итак, проверим например цифру 2. Допишем её к числу 4 и


умножим получившееся число 42 на ту же самую дописанную
цифру 2. Результат умножения будем записывать сразу под
остатком 41

Получилось число 84, которое больше остатка 41. Значит


цифра 2 не годится в качестве следующей цифры корня.
Проверим тогда цифру 1. Допишем ее к числу 4 и умножим
получившееся число 41 на на ту же самую дописанную
цифру 1
1722

Получилось число 41, которое в точности является нашим


остатком. Значит дописанная к числу 4 цифра 1 является
следующей цифрой корня. Записываем цифру 1 после
цифры 2

А число 41, которое получилось в результате умножения 41


на 1, записываем под остатком 41

Выполняем вычитание 41 − 41 = 0. Ноль в остатке говорит о


том, что решение окончено:
1723

Пример 3. Извлечём квадратный корень из числа 101761 с


помощью алгоритма извлечения квадратного корня.

Разбиваем число 101761 на грани:

Получилось три грани. Значит корень будет состоять из трёх


цифр.

Извлекáем квадратный корень из первой грани (из числа 10)


с точностью до целых, получаем 3. Записываем 3 после
знака равенства:

Далее возвóдим число 3 в квадрат и полученный результат


записываем под первой гранью (под числом 10)

Вычитаем из числа 10 число 9, получаем 1. Снóсим


следующую грань, а именно число 17. Получаем остаток 117
1724

Теперь нахóдим вторую цифру корня. Первую найденную


цифру корня, а именно 3 умножаем на 2, получаем 6. К числу
6 в конце нужно дописать ещё одну цифру (эта цифра
впоследствии и станет второй цифрой корня) и умножить
образовавшееся число на ту же самую дописанную цифру.
Полученное произведение должно быть равно остатку 117
или хотя бы максимально близким к нему, но не
превосходящим его.

Итак, проверим например цифру 2. Допишем её к числу 6 и


умножим образовавшееся число 62 на ту же самую
дописанную цифру 2. Результат умножения будем
записывать сразу под остатком 117

Получилось число 124, которое больше остатка 117. Значит


цифра 2 не годится в качестве второй цифры корня.
Проверим тогда цифру 1. Допишем ее к числу 6 и умножим
образовавшееся число 61 на на ту же самую дописанную
цифру 1
1725

Получилось число 61, которое не превосходит остатка 117.


Значит дописанная к числу 6 цифра 1 является второй
цифрой корня. Записываем её в ответе после цифры 3

Теперь выполняем вычитание 117 − 61 = 56.

Снóсим следующую грань, а именно число 61. Получаем


новый остаток 5661
1726

Теперь нахóдим третью цифру корня. Первые две найденные


цифры корня, а именно число 31 умножаем на 2, получаем
62. К числу 62 в конце нужно дописать ещё одну цифру (эта
цифра впоследствии и станет третьей цифрой корня) и
умножить образовавшееся число на ту же самую дописанную
цифру. Полученное произведение должно быть равно
остатку 5661 или хотя бы максимально близким к нему, но не
превосходящим его.

Итак, проверим например цифру 9. Допишем её к числу 62 и


умножим образовавшееся число 629 на ту же самую
дописанную цифру 9. Результат умножения будем
записывать сразу под остатком 5661

Получилось число 5661, которое в точности является нашим


остатком. Значит дописанная к числу 62 цифра 9 является
1727

третьей цифрой корня. Записываем цифру 9 в ответе после


цифры 1

Выполняем вычитание 5661 − 5661 = 0. Ноль в остатке


говорит о том, что решение окончено:

Пример 4. Извлечём квадратный корень из числа 30,25 с


помощью алгоритма извлечения квадратного корня.

Данное число является десятичной дробью. В данном случае


на грани следует разбить целую и дробную часть. Целую
часть на грани следует разбить, двигаясь влево от запятой. А
дробную — двигаясь вправо от запятой:
1728

Получилось по одной грани в каждой части. Это значит, что


корень будет состоять из двух цифр: одна цифра будет в
целой части корня и одна цифра в дробной.

Извлечём квадратный корень из первой грани (из числа 30) с


точностью до целых, получаем 5. Записываем 5 после знака
равенства:

Далее возвóдим число 5 в квадрат и полученный результат


записываем под первой гранью (под числом 30)

Вычитаем из числа 30 число 25, получаем 5.

Извлечение корня из целой части подкоренного выражения


завершено. На данный момент мы извлекли корень из числа
30,25 с точностью до целых, получили ответ 5. Последний
остаток 5 показывает, что целая часть 30 превосходит
квадрат 52 на 5 квадратных единиц.

Чтобы дальше извлечь корень (с точностью до десятых),


снесём следующую грань, а именно число 25, получим
1729

остаток 525. А в ответе после числа 5 следует поставить


запятую, поскольку сейчас мы будем искать дробную часть
корня.

Затем снóсим следующую грань, а именно число 25.


Получаем остаток 525

Далее работаем по тому же принципу, что и раньше.


Нахóдим следующую цифру корня. Для этого уже найденный
корень, а именно число 5 умножим на 2 получим 10. К числу
10 в конце нужно дописать ещё одну цифру (эта цифра
впоследствии и станет следующей цифрой корня) и
умножить образовавшееся число на ту же самую дописанную
цифру. Полученное произведение должно быть равно
остатку 525 или хотя бы максимально близким к нему, но не
превосходящим его.

Итак, проверим например цифру 5. Допишем её к числу 10 и


умножим получившееся число 105 на ту же самую
дописанную цифру 5
1730

Получилось число 525, которое в точности является нашим


остатком. Значит дописанная к числу 10 цифра 5 является
следующей цифрой корня. Возвращаемся к исходному
примеру и записываем цифру 5 после в ответе после
запятой:

Выполняем вычитание 525 − 525 = 0. Ноль в остатке говорит


о том, что решение окончено:

В подкоренном выражении можно было использовать


следующий прием: умножить подкоренное число на 100 и
получить под корнем число 3025. Далее извлечь из него
квадратный корень, как из обычного целого числа. Тогда
получился бы ответ 55
1731

Затем можно обратно разделить 3025 на 100 (или сдвинуть


запятую влево на две цифры). В результате под корнем
полýчится прежнее число 30,25, а правая часть уменьшится в
десять раз и полýчится квадратный корень из числа 30,25.

Пример 5. Извлечём квадратный корень из числа 632,5225 с


помощью алгоритма извлечения квадратного корня.

Данное число является десятичной дробью. Разбиваем


число на грани. На грани следует разбить целую и дробную
часть. Целую часть на грани следует разбить, двигаясь влево
от запятой. А дробную — двигаясь вправо от запятой:

Получилось четыре грани. При этом две грани в целой части,


и две грани в дробной. Это значит, что корень будет состоять
из четырёх цифр: две цифры будет в целой части корня, и
две цифры после запятой.

Извлечём квадратный корень из первой грани (из числа 6) с


точностью до целых, получаем 2. Записываем 2 после знака
равенства:
1732

Далее возвóдим число 2 в квадрат и полученный результат


записываем под первой гранью (под числом 6)

Вычитаем из числа 6 число 4, получаем 2. Затем снóсим


следующую грань, а именно число 32. Получаем остаток 232

Теперь нахóдим вторую цифру корня. Первую уже найденную


цифру корня, а именно 2 умножаем на 2, получаем 4. К числу
4 в конце нужно дописáть ещё одну цифру (эта цифра
впоследствии и станет второй цифрой корня) и умножить
получившееся число на ту же самую дописанную цифру.
Полученное произведение должно быть равно остатку 232
или хотя бы максимально близким к нему, но не
превосходящим его.

Итак, проверим например цифру 6. Допишем её к числу 4 и


умножим получившееся число 46 на ту же самую дописанную
цифру 6. Результат умножения будем записывать сразу под
остатком 232
1733

Получилось число 276, которое больше остатка 232. Значит


цифра 6 не годится в качестве второй цифры корня.
Проверим тогда цифру 5. Допишем ее к числу 4 и умножим
получившееся число 45 на на ту же самую дописанную
цифру 5

Получилось число 225, которое не превосходит остатка 232.


Значит дописанная к числу 4 цифра 5 является второй
цифрой корня. Записываем её в ответе после цифры 2

Теперь выполняем вычитание 232 − 225 = 7.


1734

Извлечение корня из целой части подкоренного выражения


завершено. На данный момент мы извлекли корень из числа
632,5225 с точностью до целых, получили ответ 25.
Последний остаток 7 показывает, что целая часть 632
превосходит квадрат 252 на 7 квадратных единиц.

Чтобы дальше извлечь корень (с точностью до десятых и


сотых), снесём следующую грань, а именно число 52,
получим остаток 752. А в ответе после числа 25 поставим
запятую, поскольку сейчас мы будем искать дробные части
корня:

Далее работаем по тому же принципу, что и раньше.


Нахóдим первую цифру корня после запятой. Для этого уже
найденные цифры, а именно 25 умножим на 2 получим 50. К
числу 50 в конце нужно дописáть ещё одну цифру (эта цифра
впоследствии и станет первой цифрой корня после запятой)
и умножить образовавшееся число на ту же самую
1735

дописанную цифру. Полученное произведение должно быть


равно остатку 752 или хотя бы максимально близким к нему,
но не превосходящим его.

Итак, проверим например цифру 2. Допишем её к числу 50 и


умножим получившееся число 502 на ту же самую
дописанную цифру 2. Можно интуитивно понять, что цифра 2
великá, поскольку 502 × 2 = 1004. А число 1004 больше
остатка 752. Тогда очевидно, что первой цифрой после
запятой будет цифра 1

Теперь выполняем вычитание 752 − 501 = 251. Сразу снóсим


следующую грань 25. Полýчим остаток 25125
1736

Теперь нахóдим вторую цифру корня после запятой. Не


обращая внимания на запятую, найденные цифры корня
умнóжим на 2. Полýчим 502.

К числу 502 в конце нужно дописáть ещё одну цифру (эта


цифра впоследствии и станет второй цифрой корня после
запятой) и умножить образовавшееся число на ту же самую
дописанную цифру. Полученное произведение должно быть
равно остатку 25125 или хотя бы максимально близким к
нему, но не превосходящим его.

Итак, проверим например цифру 6. Допишем её к числу 502 и


умнóжим образовавшееся число 5026 на ту же самую
дописанную цифру 6. Результат умножения будем
записывать сразу под остатком 25125

Получилось число 30156, которое больше остатка 25125.


Значит цифра 6 не годится в качестве второй цифры корня
после запятой. Проверим тогда цифру 5. Допишем ее к числу
502 и умножим получившееся число 5025 на на ту же самую
дописанную цифру 5
1737

Получилось число 25125, которое в точности является


нашим остатком. Значит дописанная к числу 502 цифра 5
является второй цифрой корня после запятой. Записываем
цифру 5 в ответе после цифры 1

Теперь выполняем вычитание 25125 − 25125 = 0. Ноль в


остатке говорит о том, что решение окончено:
1738

В этом примере можно было воспользоваться методом


умножения подкоренного выражения на 10000. Тогда
подкоренное число приняло бы вид 6325225. Его можно
разделить на грани, двигаясь справа налево. В результате
получился бы корень 2515
1739

Затем подкоренное число 6325225 делят на 10000, чтобы


вернуться к изначальному числу 632,5225. В результате
этого деления ответ умéньшится в 100 раз и обратится в
число 25,15.

Пример 4. Используя алгоритм извлечения квадратного


корня, извлечь квадратный корень из числа 11 с точностью
до тысячных:

В данном числе только одна грань 11. Извлечём из неё


корень с точностью до целых, получим 3

Теперь возвóдим число 3 в квадрат и полученный результат


записываем под первой гранью (под числом 11)

Выполним вычитание 11 − 9 = 2

Извлечение корня из целой части подкоренного выражения


завершено. На данный момент мы извлекли корень из числа
11 с точностью до целых, получили ответ 3. Последний
остаток 2 показывает, что целая часть 11 превосходит
квадрат 32 на две квадратные единицы.
1740

Наша задача была извлечь корень из числа 11 с точностью


до тысячных. Значит нужно снести следующую грань, но её в
данном случае нет.

Если после целого числа поставить запятую и написать


сколько угодно нулей, то значение этого числа не измéнится.
Так, после 11 можно поставить запятую и написать несколько
нулей (несколько граней), которые в последствии можно
будет снóсить к остаткам.

Если корень извлекáется с точностью до тысячных, то в


ответе после запятой должно быть три цифры. Поэтому в
подкоренном выражении поставим запятую и запишем три
грани, состоящие из нулей:

Теперь можно снести следующую грань, а именно два нуля.


Получим остаток 200. А в ответе после числа 3 поставим
запятую, поскольку сейчас мы будем искать дробные части
корня:

Теперь нахóдим первую цифру после запятой в ответе.


Первую найденную цифру корня, а именно число 3 умножаем
на 2, получаем 6. К числу 6 нужно дописáть ещё одну цифру
(эта цифра впоследствии и станет первой цифрой после
1741

запятой) и умножить образовавшееся число на ту же самую


дописанную цифру. Полученное произведение должно быть
равно остатку 200 или хотя бы максимально близким к нему,
но не превосходящим его.

В данном случае подойдёт цифра 3

Выполним вычитание 200 − 189 и снесём следующую грань


00

Нахóдим вторую цифру корня после запятой. Не обращая


внимания на запятую, найденные цифры корня умнóжим на
2. Полýчим 66.

К числу 66 в конце нужно дописáть ещё одну цифру (эта


цифра впоследствии и станет второй цифрой корня после
запятой) и умножить образовавшееся число на ту же самую
дописанную цифру. Полученное произведение должно быть
равно остатку 1100 или хотя бы максимально близким к нему,
но не превосходящим его.
1742

В данном случае подойдёт цифра 1

Выполним вычитание 1100−661 и снесём следующую грань


00

Нахóдим третью цифру корня после запятой. Не обращая


внимания на запятую, найденные цифры корня умножим на
2. Получим 662.

К числу 662 нужно дописáть ещё одну цифру (эта цифра


впоследствии и станет третьей цифрой корня после запятой)
и умножить образовавшееся число на ту же самую
дописанную цифру. Полученное произведение должно быть
1743

равно остатку 43900 или хотя бы максимально близким к


нему, но не превосходящим его.

Проверим цифру 7

Получилось число 46389, которое больше остатка 43900.


Значит цифра 7 не годится в качестве третьей цифры корня
после запятой. Проверим тогда цифру 6. Допишем ее к числу
662 и умножим получившееся число 6626 на на ту же самую
дописанную цифру 6
1744

Получилось число 39756, которое не превосходит остатка


43900. Значит дописанная к числу 662 цифра 6 является
третьей цифрой корня после запятой. Записываем цифру 6 в
ответе после цифры 1

Выполним вычитание 43900 − 39756 = 4144


1745

Дальнейшее вычисление не требуется, поскольку корень


нужно было извлечь с точностью до тысячных.

Но в таких примерах как этот, цифры после запятой можно


находить бесконечно. Например, так можно продолжить
данный пример, найдя значение корня с точностью до
десятитысячных:
1746

Как работает алгоритм

Алгоритм извлечения квадратного корня основан на формуле


квадрата суммы двух выражений:

(a + b)2 = a2 + 2ab + b2

Геометрически эту формулу можно представить так:


1747

То есть сторона a увеличивается на b. Это приводит к


увеличению изначального квадрата. Чтобы вычислить
площадь такого квадрата, нужно по отдельности вычислить
площади квадратов и прямоугольников, входящих в этот
квадрат и сложить полученные результаты. Нужно хорошо
понимать данный рисунок. Без его понимания невозможно
понять как работает алгоритм извлечения квадратного корня.

Отметим, что формула квадрата суммы двух выражений


позволяет возвести в квадрат любое число. Используя
разряды, исходное число представляют в виде суммы чисел
и далее эту сумму возвóдят в квадрат.

Например, так можно возвести число 21 в квадрат:


представить данное число в виде суммы двух десятков и
одной единицы, и далее эту сумму возвести в квадрат :

212 = (20 + 1)2 = 202 + 2 × 20 × 1 + 12 = 400 + 40 + 1 = 441

Геометрически это будет выглядеть так: сторона квадрата


равная 21 разбивается на две составляющие: 20 и 1.
1748

Затем по отдельности вычисляются площади квадратов и


прямоугольников, входящих в большой квадрат. А именно:
один квадрат со стороной 20 (получается площадь, равная
400), два прямоугольника со сторонами 20 и 1 (получается
две площади по 20), один квадрат со стороной 1 (получается
площадь, равная 1). Результаты вычисления площадей
складываются и получается итоговое значение 441.

Заметим также, что при возведéнии десятков в квадрат


получились сотни. В данном случае при возведéнии числа 20
в квадрат получилось число 400. Это позволяет
предположить, что если корень является двузначным
числом, то десятки этого корня следует искать в сотнях

подкоренного числа. Действительно, . Десятки корня


это цифра 2, является корнем числа 4, которое отвечает за
сотни числа 441.

А при возведéнии сóтен в квадрат получаются десятки тысяч.


Например, возведём в квадрат число 123, используя
формулу квадрата суммы двух выражений. Число 123 это
одна сотня, два десятка и три единицы:
1749

1232 = (100 + 20 + 3)2

При изучении многочленов мы выяснили, что если многочлен


содержит более двух членов и возникла необходимость
применить формулу квадрата суммы, то некоторые из членов
можно взять в скобки, чтобы получилось выражение вида
(a + b)2

Рассмотрим подробное извлечение квадратного корня из


числа 4096. Заодно пройдёмся по основным этапам
алгоритма извлечения квадратного корня, рассмотренного в
предыдущей теме.

Допустим, что число 4096 это площадь следующего


квадрата:
1750

Извлечь корень из числа 4096 означает найти длину стороны


данного квадрата:

Для начала узнáем из скольких цифр будет состоять корень.


Ближáйшие от 4096 известные нам квадраты это 3600 и
4900. Между ними располагается квадрат 4096. Запишем это
в виде неравенства:

Запишем каждое число под знáком корня:

Квадратные корни из чисел 3600 и 4900 нам известны. Это


корни 60 и 70 соответственно:

Корни 60 и 70 являются двузначными числами. Если


квадратный корень из числа 4096 располагается между
числами 60 и 70, то этот корень тоже будет двузначным
числом.
1751

Двузначное число состоит из десятков и единиц. Это значит,


что квадратный корень из числа 4096 можно представить в
виде суммы a + b, где a — десятки корня, b — единицы
корня. Сумма a + b во второй степени будет равна 4096

(a + b)2 = 4096

Тогда сторона квадрата будет разбита на две составляющие:


aиb

Перепишем в равенстве (a + b)2 = 4096 левую часть в виде


a2 + 2ab + b2

a2 + 2ab + b2 = 4096

Тогда рисунок, иллюстрирующий квадрат площадью 4096,


можно представить так:
1752

Если мы узнáем значения переменных a и b, то узнáем длину


стороны данного квадрата. Проще говоря, узнáем сам
корень.

Вернёмся к извлечению корня. Мы выяснили, что корнем


будет двузначное число. Двузначное число состоит из
десятков и единиц. При возведéнии десятков в квадрат,
получаются сотни. Тогда десятки искомого корня следует
искать в сотнях подкоренного числа. В подкоренном числе 40
сотен. Отделим их небольшой помéткой:

Извлечём корень из числа 40. Из числа 40 корень не


извлекается. Поэтому извлечение следует выполнить
приближённо с точностью до целых.

Ближáйший мéньший квадрат к числу 40 это 36. Извлечём


корень из этого квадрата, получим 6. Тем сáмым полýчим
первую цифру корня:
1753

На самом деле корень извлечён не из числа 40, а из сорокá


сотен. Метка, которая постáвлена после числа 40, отделяет
разряды числа, находящегося под знáком корня. Нужно
понимать, что в данном случае 40 это 4000.

Из 4000 как и 40 корень не извлекается, поэтому его тоже


следует извлекать приближённо. Для этого следует найти
ближáйший мéньший квадрат к числу 4000. Но нужно
принимать во внимание следующий момент. Десятки это
числа с одним нулем на конце. Примеры:

10 — один десяток

30 — три десятка

120 — двенадцать десятков

При возведéнии таких чисел в квадрат, получаются числа с


двумя нулями на конце:

102 = 100

302 = 900

1202 = 14400

Мы ищем десятки корня в сотнях числá 4096, то есть в числе


4000. Но нет такого числá с нулем на конце, вторая степень
которого равна 4000. Поэтому мы ищем ближáйший мéньший
квадрат, но опять же с двумя нулями на конце. Таковым
является квадрат 3600. Корень следует извлекать из этого
квадрата.
1754

Вернемся к нашему рисунку. Большой квадрат со стороной a


и площадью a2 это тот самый квадрат 3600. Укажем вместо
a2 значение 3600

Теперь извлечём квадратный корень из квадрата 3600. Ранее


мы говорили, что если число содержит уже знакомый нам
квадрат и чётное количество нулей, то можно извлечь корень
из этого числа. Для этого сначала следует извлечь корень из
знакомого нам квадрата, а затем записать половину от
количества нулей исходного числа:

Итак, мы нашли сторону квадрата, площадь которого 3600.


Подпишем сторону a как 60
1755

Но ранее в ответе мы написали не 60, а 6. Это является


сокращённым вариантом. Число 6 в данном случае означает
шесть десятков:

Итак, десятки корня найдены. Их шесть. Теперь нужно найти


единицы корня. Единицы корня это длина оставшейся
маленькой стороны квадрата, то есть значение переменной
b.

Чтобы найти b, нужно из общего квадрата, площадь которого


4096 вычесть квадрат, площадь которого 3600. В результате
останется фигура, площадь которой 4096 − 3600 = 496
1756

На рисунке видно как из квадрата, площадь которого 4096


отделился квадрат, площадь которого 3600. Осталась
фигура, площадь которой 496.

Именно поэтому в процессе применения алгоритма первая


найденная цифра корня возводится в квадрат, чтобы
результат возведения вычесть из сотен подкоренного
выражения.

Так, из 40 сотен вычитаются 36 сотен, остаётся 4 сотни плюс


сносятся девяносто шесть единиц. Эти четыре сотни и
девяносто шесть единиц вместе образуют 496 единиц:

Оставшаяся фигура есть ни что иное как удвоенное


произведение первого выражение a плюс квадрат второго
выражения b
1757

Сумма площадей 2ab + b2 должна вмещаться в число 496.


Запишем это в виде следующего равенства:

2ab + b2 = 496

Значение a уже известно. Оно равно 60. Тогда равенство


примет вид:

2 × 60 × b + b2 = 496

120b + b2 = 496

Теперь наша задача найти такое значение b, при котором


левая часть станет равна 496 или хотя близкой к этому
числу. Поскольку b является единицами искомого корня, то
значение b является однозначным числом. То есть значение
b это число от 1 до 9. Это число можно найти методом
подбора. В данном случае очевидно, что числом b является 4

120 × 4 + 42 = 496

480 + 16 = 496

496 = 496
1758

Но для удобства поиска этой цифры, переменную b выносят


за скобки. Вернёмся к выражению 120b + b2 = 496 и вынесем
b за скобки:

 b(120 + b) = 496

Теперь правую часть можно понимать так: к 120 следует


прибавить некоторое число b, которое при умножении с тем
же сáмым b даст в результате 496.

Именно поэтому при использовании алгоритма, уже


найденную цифру умножают на 2. Так, 6 мы умножили на 2
получили 12 и уже к 12 дописывали цифру и умножáли
образовавшееся число на ту же дописанную цифру, пытаясь
получить остаток 496.

Но это опять же упрощённый вариант. На самом деле на 2


умножается не просто 6, а найденные десятки (в нашем
случае число 60), получается число 120. Затем следует
нахождение числá вида b(120 + b). То есть к 120
прибавляется число b, которое при перемножении с b даёт
остаток 496.

Итак, b = 4. Тогда:

4(120 + 4) = 496
1759

4 × 124 = 496

496 = 496

При подстановке числá 4 вместо b получается остаток 496.


Это значит, что единицы корня найдены. Квадрат, площадь
которого 4096, имеет сторону равную 60 + 4, то есть 64.

Если из общей площади вычесть 3600, затем 496, полýчим 0.


Остаток, равный нулю, говорит о том, что решение
завершено:

4096 − 3600 − 496 = 0

Пример 2. Извлечь квадратный корень из числа 54756

Пусть число 54756 это площадь следующего квадрата:


1760

Извлечь корень из числа 54756 означает найти длину


стороны данного квадрата:

Пока неизвестно является ли квадратный корень из числа


54756 целым либо дробным числом. Узнáем для начала из
скольких цифр будет состоять целый корень.

Число 54756 больше числá 10000, но меньше числá 90000

10000 < 54756 < 90000

Корни из 10000 и 90000 являются трёхзначными числами.


1761

Тогда корень из 54756 тоже будет трёхзначным числом. А


трёхзначное число состоит из сотен, десятков и единиц.

Квадратный корень из числа 54756 можно представить в


виде суммы a + b + с, где a — сотни корня, b — десятки
корня, с — единицы корня. Сумма a + b + с во второй
степени будет равна 54756

(a + b + c)2 = 54756

Тогда сторона квадрата будет разбита на три составляющие:


a, b и c

Выполним в левой части равенства (a + b + c)2 = 54756


возведéние в квадрат:

Тогда рисунок иллюстрирующий квадрат, площадью 54756


можно представить так:
1762

Два прямоугольника площадью ab в приведённом ранее


равенстве заменены на 2ab, а два прямоугольника
площадью (a + b)c заменены на 2ac + 2bc, поскольку
(a + b)c = ac + bc. Если повторить выражение ac + bc
дважды, то полýчится 2ac + 2bc

2(ac + bc) = 2ac + 2bc

Если мы узнáем значения переменных a, b и c, то узнáем


длину стороны данного квадрата. Проще говоря, узнáем сам
корень.

Вернёмся к извлечению корня. Мы выяснили, что корнем


будет трёхзначное число. Трёхзначное число состоит из
сотен, десятков и единиц.

При возведéнии сотен в квадрат, получаются десятки тысяч.


Тогда сотни искомого корня следует искать в десятках тысяч
подкоренного числа. В подкоренном числе 5 десятков тысяч.
Отделим их мéткой:
1763

Извлечём корень из числа 5. Из числа 5 корень не


извлекается. Поэтому извлечение следует выполнить
приближённо с точностью до целых Ближáйший мéньший
квадрат к 5 это 4. Извлечём корень из этого квадрата,
получим 2. Тем самым полýчим первую цифру корня:

На самом деле корень извлечён не из числа 5, а из пяти


десятков тысяч. Метка, которая поставлена после числá 5,
отделяет разряды числá, находящегося под знáком корня.
Нужно понимать, что в данном случае 5 это 50000.

Из 50000 как и 5 корень не извлекается, поэтому его тоже


следует извлекать приближённо. Для этого следует найти
ближáйший мéньший квадрат к числу 50000. Но нужно
принимать во внимание, что сотни это числа с двумя нулями
на конце. Примеры:

100 — одна сотня

500 — пять сотен

900 — девять сотен

При возведéнии таких чисел в квадрат, получаются числа, у


которых четыре нуля на конце:

1002 = 10000

5002 = 250000

9002 = 810000
1764

Мы ищем сотни корня в десятках тысяч числа 54756, то есть


в числе 50000. Но нет такого числá с двумя нулями на конце,
вторая степень которого равна 50000. Поэтому мы ищем
ближáйший мéньший квадрат, но опять же с четырьмя
нулями на конце. Таковым является квадрат 40000.

Вернёмся к нашему рисунку. Большой квадрат со стороной a


и площадью a2 это тот самый квадрат 40000. Укажем вместо
a2 значение 40000

Теперь извлечём корень из квадрата 40000

Итак, мы нашли сторону квадрата, площадь которого 40000.


Подпишем сторону a как 200
1765

Но ранее в ответе мы написали не 200, а 2. Это является


сокращённым вариантом. Число 2 в данном случае означает
две сотни:

Теперь вытаскиваем остаток. Из пяти десятков тысяч корень


извлечён только из четырёх десятков тысяч. Значит в остатке
остался один десяток тысяч. Вытащим его:

Опять же надо понимать, что 4 это 40000, а 1 это 10000. С


помощью рисунка это можно пояснить так: квадрат, площадь
которого 40000, вычитается от общего квадрата, площадь
которого 54756. Остаётся фигура, площадь которой
54756 − 40000 = 14756
1766

Теперь нужно найти десятки корня. Рассмотрим на рисунке


сумму площадей ab + ab + b2 (или 2ab + b2). В эту сумму
будет входить один десяток тысяч, который остался в
результате нахождения сóтен корня, удвоенное
произведение сотен и десятков корня 2ab, а также десятки
корня в квадрате b2.

Десятки в квадрате составляют сотни. Поэтому десятки корня


следует искать в сотнях подкоренного числа. Под корнем
сейчас 47 сотен. Снесём их к остатку 1, предварительно
отделив их под корнем мéткой:

Один десяток тысяч это сто сотен, плюс снесено 47 сотен.


Итого 100 + 47 = 147 сотен. В эти 147 сотен должна входить
сумма 2ab + b2
1767

2ab + b2 = 14700

Переменная a уже известна, она равна 200. Подставим это


значение в данное равенство:

2 × 200 × b + b2 = 14700
 400b + b2 = 14700

Теперь наша задача найти такое значение b, при котором


левая часть станет равна 14700 или хотя близкой к этому
числу, но не превосходящей его. Поскольку b является
десятками искомого корня, то значение b является
двузначным числом с одним нулём на конце. Такое число
можно найти методом подбора. Для удобства вынесем в
левой части за скобки b

b(400 + b) = 14700

Теперь левую часть можно понимать так: к 400 следует


прибавить некоторое число b, которое при умножении с тем
же самым b даст в результате 14700 или близкое к 14700
число, не превосходящее его. Подставим например 40

40(400 + 40) = 14700

17600 ≠ 14700

Получается 17600, которое превосходит число 14700. Значит


число 40 не годится в качестве десятков корня. Проверим
тогда число 30

30(400 + 30) = 14700

12900 ≤ 14700
1768

Получилось число 12900, которое не превосходит 14700.


Значит число 30 подходит в качестве десятков корня. Числа,
расположенные между 30 до 40 проверять не нужно,
поскольку сейчас нас интересуют только двузначные числа с
одним нулем на конце:

Вернемся к нашему рисунку. Сторона b это десятки корня.


Укажем вместо b найденные десятки 30. А квадрат, площадь
которого b2 это найденные десятки во второй степени, то
есть число 900. Также укажем площади прямоугольников ab.
Они равны произведению сотен корня на десятки корня, то
есть 200 × 30 = 6000
1769

Ранее в ответе мы написали не 30, а 3. Это является


сокращённым вариантом. Число 3 в данном случае означают
три десятка.

Теперь вытаскиваем остаток. В 147 сотен вместилось только


129 сотен. Значит в остатке осталось 147 − 129 = 18 сотен
плюс сносим число 56 из подкоренного выражения. В
результате образýется новый остаток 1856

С помощью рисунка это можно пояснить так: от фигуры,


площадь которой 14756, вычитается площадь 12900.
Остаётся фигура, площадь которой 14756 − 12900 = 1856
1770

Теперь нужно найти единицы корня. Рассмотрим на рисунке


сумму площадей 2(a + b)c + c2. В эту сумму и должен входить
последний остаток 1856

2(a + b)c + c2 = 1856

Переменные a и b уже известны, они равны 200 и 30


соответственно. Подставим эти значения в данное
равенство:

2(200 + 30)c + c2 = 1856

 2 × 230c + c2 = 1856

460c + c2 = 1856

Теперь наша задача найти такое значение c, при котором


левая часть станет равна 1856 или хотя близкой к этому
числу, но не превосходящей его. Поскольку c является
1771

единицами искомого корня, то значение с является


однозначным числом. То есть значение с это число от 1 до 9.
Это число можно найти методом подбора. Для удобства
вынесем в левой части за скобки с

с(460 + c) = 1856

Теперь левую часть можно понимать так: к 460 следует


прибавить нéкоторое число с, которое при умножении с тем
же сáмым с даст в результате 1856 или близкое к 1856 число,
не превосходящее его. Подставим, например, число 4

4(460 + 4) = 1856

4 × 464 = 1856

1856 = 1856

Именно поэтому при использовании алгоритма первые


найденные цифры умножают на 2. Так, 23 мы умнóжили на 2,
получили 46 и уже к 46 дописывали цифру и умножáли
образовавшееся число на ту же самую дописанную цифру,
пытаясь получить остаток 1856
1772

Итак, с = 4. При подстановке вместо с числá 4 получается


остаток 1856. Это значит, что единицы корня найдены.

Квадрат, площадь которого 54756, имеет сторону равную


200 + 30 + 4, то есть 234.

Если из общей площади 54756 вычесть 40000, 6000, 6000,


900, 920, 920 и 16, то получим 0. Остаток равный нулю
говорит о том, что решение завершено:

54756 − 40000 − 6000 − 6000 − 900 − 920 − 920 − 16 = 0

Пример 3. Извлечь квадратный корень из числа 3

Квадратный корень из числа 3 не извлекается. Ранее мы


говорили, что квадратные корни из таких чисел можно
извлекать только приближённо с определенной точностью.

Пусть 3 это площадь следующего квадрата:


1773

Извлечь корень из числа 3 значит найти длину стороны


данного квадрата:

Корень из 3 больше корня из 1, но меньше корня из 4

√1 < √3 < √4

Корни из 1 и 4 являются целыми числами.

√1 < √3 < √4

1 < √3 < 2

Между числами 1 и 2 нет целых чисел. Значит корень из


числа 3 будет десятичной дробью. Найдём этот корень с
точностью до десятых.

Квадратный корень из числа 3 можно представить в виде


суммы a + b, где a — целая часть корня, b — дробная часть.
1774

Тогда сторону квадрата можно разбить на две


составляющие: a и b

Сумма a + b во второй степени должна приближённо


равняться 3.

(a + b)2 ≈ 3

Выполним в левой части данного равенства возведéние в


квадрат:

a2 + 2ab + b2 ≈ 3

Тогда рисунок, иллюстрирующий квадрат площадью 3, можно


представить так:

Найдём a. Извлечём корень из числа 3 с точностью до


целых, получим 1
1775

Если a2 это 1, а площадь всего квадрата равна 3, то в остатке


останется 2. В этот остаток должна вмещаться площадь
оставшейся фигуры:

Найдём b. Для этого рассмотрим сумму площадей 2ab + b2.


Эта сумма должна приближённо равняться остатку 2, но не
превосходить его

2ab + b2 ≈ 2

Значение a уже известно, оно равно единице:

2b + b2 ≈ 2

Вынесем за скобки b

b(2 + b) ≈ 2
1776

Теперь в левой части к 2 следует прибавить нéкоторое число


b, которое при умножении с тем же b будет приближённо
равняться 2.

Значение b является дробным числом, а именно десятой


частью. Оно равно какому-нибудь числу из промежутка [0,1;
0,9]. Возьмём любое число из этого промежутка и подставим
его в равенство. Подставим к примеру 0,8

0,8(2 + 0,8) ≈ 2

2,24 ≈ 2

Получилось 2,24 которое превосходит 2. Значит 0,8 не


годится в качестве значения b. Проверим тогда 0,7

0,7(2 + 0,7) ≈ 2

1,89 ≈ 2

Получилось 1,89 которое приближённо равно 2 и не


превосходит его. Значит 0,7 является значением b

Значит квадратный корень из 3 с точностью до десятых


приближённо равен 1 + 0,7
1777

К сожалению, понять механизм алгоритма извлечения


квадратного корня намного сложнее, чем использовать сам
алгоритм. Решите несколько примеров на применение
алгоритма, и понимание механизма его работы будет
даваться вам значительно проще.

Задания для самостоятельного решения


Задание 1. Извлечь квадратный корень из числа 169,
используя алгоритм извлечения квадратного корня
Показать решение
Задание 2. Извлечь квадратный корень из числа 289,
используя алгоритм извлечения квадратного корня
Показать решение
Задание 3. Извлечь квадратный корень из числа 1089,
используя алгоритм извлечения квадратного корня
Показать решение
Задание 4. Извлечь квадратный корень из числа 1764,
используя алгоритм извлечения квадратного корня
Показать решение
Задание 5. Извлечь квадратный корень из числа 4761,
используя алгоритм извлечения квадратного корня
Показать решение
Задание 6. Извлечь квадратный корень из числа 132496,
используя алгоритм извлечения квадратного корня
Показать решение
1778

Задание 7. Извлечь квадратный корень из числа 157 с


точностью до сотых, используя алгоритм извлечения
квадратного корня
Показать решение
Задание 8. Извлечь квадратный корень из числа 240,25
используя алгоритм извлечения квадратного корня
Показать решение
1779

Квадратное уравнение

Предварительные навыки

 Буквенные выражения
 Общие сведения об уравнениях
 Решение задач с помощью уравнений
 Квадратный корень

Содержание урока

 Что такое квадратное уравнение и как его решать?


 Формулы корней квадратного уравнения
 Примеры решения квадратных уравнений
 Примеры решения задач
 Задания для самостоятельного решения

Что такое квадратное уравнение и как его решать?

Мы помним, что уравнение это равенство, содержащее в


себе переменную, значение которой нужно найти.

Если переменная, входящая в уравнение, возведенá во


вторую степень (в квадрат), то такое уравнение называют
уравнением второй степени или квадратным
уравнением.

Например, следующие уравнения являются квадратными:


1780

Решим первое из этих уравнений, а именно x2 − 4 = 0.

Все тождественные преобразования, которые мы применяли


при решении обычных линейных уравнений, можно
применять и при решении квадратных.

Итак,  в уравнении x2 − 4 = 0 перенесем член −4 из левой


части в правую часть, изменив знак:

Получили уравнение x2 = 4. Ранее мы говорили, что


уравнение считается решённым, если в одной части
переменная записана в первой степени и её коэффициент
равен единице, а другая часть равна какому-нибудь числу. То
есть чтобы решить уравнение, его следует привести к виду
x = a, где a — корень уравнения.

У нас переменная x всё ещё во второй степени, поэтому


решение необходимо продолжить.

Чтобы решить уравнение x2 = 4, нужно ответить на вопрос


при каком значении x левая часть станет равна 4. Очевидно,
что при значениях 2 и −2. Чтобы вывести эти значения
воспользуемся определением квадратного корня.

Число b называется квадратным корнем из числа a, если

b2 = a и обозначается как

У нас сейчас похожая ситуация. Ведь, что такое x2 = 4?


Переменная x в данном случае это квадратный корень из
числа 4, поскольку вторая степень x прирáвнена к 4.
1781

Тогда можно записать, что . Вычисление правой части


позвóлит узнать чему равно x. Квадратный корень имеет два
значения: положительное и отрицательное. Тогда получаем
x = 2 и x = −2.

Обычно записывают так: перед квадратным корнем ставят


знак «плюс-минус», затем находят арифметическое значение
квадратного корня. В нашем случае на этапе когда записано

выражение , перед следует поставить знак ±

Затем найти арифметическое значение квадратного корня

Выражение x = ± 2 означает, что x = 2 и x = −2. То


есть корнями уравнения x2 − 4 = 0 являются числа 2 и −2.
Запишем полностью решение данного уравнения:

Выполним проверку. Подставим корни 2 и −2 в исходное


уравнение и выполним соответствующие вычисления. Если
при значениях 2 и −2 левая часть равна нулю, то это будет
означать, что уравнение решено верно:
1782

В обоих случаях левая часть равна нулю. Значит уравнение


решено верно.

Решим ещё одно уравнение. Пусть требуется решить


квадратное уравнение (x + 2)2 = 25

Для начала проанализируем данное уравнение. Левая часть


возведенá в квадрат и она равна 25. Какое число в квадрате
равно 25? Очевидно, что числа 5 и −5

То есть наша задача найти x, при которых выражение x + 2


будет равно числам 5 и −5. Запишем эти два уравнения:

Решим оба уравнения. Это обычные линейные уравнения,


которые решаются легко:

Значит корнями уравнения (x + 2)2 = 25 являются числа 3 и


−7.
1783

В данном примере как и в прошлом можно использовать


определение квадратного корня. Так, в уравнения
(x + 2)2 = 25 выражение (x + 2) представляет собой
квадратный корень из числа 25. Поэтому можно cначала

записать, что .

Тогда правая часть станет равна ±5. Полýчится два


уравнения: x + 2 = 5 и x + 2 = −5. Решив по отдельности
каждое из этих уравнений мы придём к корням 3 и −7.

Запишем полностью решение уравнения (x + 2)2 = 25

Из рассмотренных примеров видно, что квадратное


уравнение имеет два корня. Чтобы не забыть о найденных
корнях, переменную x можно подписывать нижними
индексами. Так, корень 3 можно обозначить через x1, а
корень −7 через x2

В предыдущем примере тоже можно было сделать так.


Уравнение x2 − 4 = 0 имело корни 2 и −2. Эти корни можно
было обозначить как x1 = 2 и x2 = −2. 

Бывает и так, что квадратное уравнение имеет только один


корень или вовсе не имеет корней. Такие уравнения мы
рассмотрим позже.
1784

Сделаем проверку для уравнения (x + 2)2 = 25. Подставим в


него корни 3 и −7. Если при значениях 3 и −7 левая часть
равна 25, то это будет означать, что уравнение решено
верно:

В обоих случаях левая часть равна 25. Значит уравнение


решено верно.

Квадратное уравнение бывает дано в разном виде. Наиболее


его распространенная форма выглядит так:

ax2 + bx + c = 0,
где a, b, c — некоторые числа, x — неизвестное.

Это так называемый общий вид квадратного уравнения. В


таком уравнении все члены собраны в общем месте (в одной
части), а другая часть равна нулю. По другому такой вид
уравнения называют нормальным видом квадратного
уравнения.

Пусть дано уравнение 3x2 + 2x = 16. В нём переменная x


возведенá во вторую степень, значит уравнение является
квадратным. Приведём данное уравнение к общему виду.

Итак, нам нужно получить уравнение, которое будет похоже


на уравнение ax2 + bx + c = 0. Для этого в уравнении
3x2 + 2x = 16 перенесем 16 из правой части в левую часть,
изменив знак:
1785

3x2 + 2x − 16 = 0

Получили уравнение 3x2 + 2x − 16 = 0. В этом уравнении


a = 3, b = 2, c = −16.

В квадратном уравнении вида ax2 + bx + c = 0 числа a, b и c


имеют собственные названия. Так, число a называют
первым или старшим коэффициентом; число b
называют вторым коэффициентом; число
c называют свободным членом.

В нашем случае для уравнения 3x2 + 2x − 16 = 0 первым или


старшим коэффициентом является 3; вторым
коэффициентом является число 2;  свободным членом
является число −16. Есть ещё другое общее название для
чисел a, b и c — параметры.

Так, в уравнении 3x2 + 2x − 16 = 0 параметрами являются


числа 3, 2 и −16.

В квадратном уравнении желательно упорядочивать члены


так, чтобы они располагались в таком же порядке как у
нормального вида квадратного уравнения.

Например, если дано уравнение −5 + 4x2 + x = 0, то его


желательно записать в нормальном виде, то есть в виде
ax2+ bx + c = 0.

В уравнении −5 + 4x2 + x = 0 видно, что свободным членом


является −5, он должен располагаться в конце левой части.
Член 4x2 содержит старший коэффициент, он должен
располагаться первым. Член x соответственно будет
располагаться вторым:
1786

Квадратное уравнение в зависимости от случая может


принимать различный вид. Всё зависит от того, чему равны
значения a, b и с.

Если коэффициенты a, b и c не равны нулю, то квадратное


уравнение называют полным. Например, полным является
квадратное уравнение 2x2 + 6x − 8 = 0.

Если какой-то из коэффициентов равен нулю (то есть


отсутствует), то уравнение значительно уменьшается и
принимает более простой вид. Такое квадратное уравнение
называют неполным. Например, неполным является
квадратное уравнение 2x2 + 6x = 0, в нём имеются
коэффициенты a и b (числа 2 и 6), но отсутствует свободный
член c.

Рассмотрим каждый из этих видов уравнений, и для каждого


из этих видов определим свой способ решения.

Пусть дано квадратное уравнение 2x2 + 6x − 8 = 0. В этом


уравнении a = 2, b = 6, c = −8. Если b сделать равным нулю,
то уравнение примет вид:

Получилось уравнение 2x2 − 8 = 0. Чтобы его решить


перенесем −8 в правую часть, изменив знак:

2x2 = 8
1787

Для дальнейшего упрощения уравнения воспользуемся


ранее изученными тождественными преобразованиями. В
данном случае можно разделить обе части на 2

У нас получилось уравнение, которое мы решали в начале


данного урока. Чтобы решить уравнение x2 = 4, следует
воспользоваться определением квадратного корня. Если

x2 = 4, то . Отсюда x = 2 и x = −2.

Значит корнями уравнения 2x2 − 8 = 0 являются числа 2 и −2.


Запишем полностью решение данного уравнения:

Выполним проверку. Подставим корни 2 и −2 в исходное


уравнение и выполним соответствующие вычисления. Если
при значениях 2 и −2 левая часть равна нулю, то это будет
означать, что уравнение решено верно:
1788

В обоих случаях левая часть равна нулю, значит уравнение


решено верно.

Уравнение, которое мы сейчас решили, является неполным


квадратным уравнением. Название говорит само за себя.
Если полное квадратное уравнение выглядит как
ax2 + bx + c = 0, то сделав коэффициент b нулём получится
неполное квадратное уравнение ax2 + c = 0.

У нас тоже сначала было полное квадратное уравнение


2x2 + 6x − 4 = 0. Но мы сделали коэффициент b нулем, то
есть вместо числа 6 поставили 0. В результате уравнение
обратилось в неполное квадратное уравнение 2x2 − 4 = 0.

В начале данного урока мы решили квадратное уравнение


x2 − 4 = 0. Оно тоже является уравнением вида ax2 + c = 0, то
есть неполным. В нем a = 1, b = 0, с = −4.

Также, неполным будет квадратное уравнение, если


коэффициент c равен нулю.

Рассмотрим полное квадратное уравнение 2x2 + 6x − 4 = 0.


Сделаем коэффициент c нулём. То есть вместо числа 4
поставим 0
1789

Получили квадратное уравнение 2x2 + 6x=0, которое является


неполным. Чтобы решить такое уравнение, переменную x
выносят за скобки:

Получилось уравнение x(2x + 6) = 0 в котором нужно найти x,


при котором левая часть станет равна нулю. Заметим, что в
этом уравнении выражения x и (2x + 6) являются
сомножителями. Одно из свойств умножения говорит, что
произведение равно нулю, если хотя бы один из
сомножителей равен нулю (или первый сомножитель или
второй).

В нашем случае равенство будет достигаться, если x будет


равно нулю или (2x + 6) будет равно нулю. Так и запишем
для начала:

Получилось два уравнения: x = 0 и 2x + 6 = 0. Первое


уравнение решать не нужно — оно уже решено. То есть
первый корень равен нулю.

Чтобы найти второй корень, решим уравнение 2x + 6 = 0. Это


обычное линейное уравнение, которое решается легко:

Видим, что второй корень равен −3.


1790

Значит корнями уравнения 2x2 + 6x = 0 являются числа 0 и


−3. Запишем полностью решение данного уравнения:

Выполним проверку. Подставим корни 0 и −3 в исходное


уравнение и выполним соответствующие вычисления. Если
при значениях 0 и −3 левая часть равна нулю, то это будет
означать, что уравнение решено верно:

Следующий случай это когда числа b и с равны нулю.


Рассмотрим полное квадратное уравнение 2x2 + 6x − 4 = 0.
Сделаем коэффициенты b и c нулями. Тогда уравнение
примет вид:

Получили уравнение 2x2 = 0. Левая часть является


произведением, а правая часть равна нулю. Произведение
равно нулю, если хотя бы один из сомножителей равен нулю.
Очевидно, что x = 0. Действительно, 2 × 02 = 0. Отсюда, 0 = 0.
При других значениях x равенства достигаться не будет.
1791

Проще говоря, если в квадратном уравнении вида


ax2 + bx + c = 0 числа b и с равны нулю, то корень такого
уравнения равен нулю.

Отметим, что когда употребляются словосочетания «b равно


нулю» или «с равно нулю«, то подразумевается, что
параметры b или c вовсе отсутствуют в уравнении.

Например, если дано уравнение 2x2 − 32 = 0, то мы говорим,


что b = 0. Потому что если сравнить с полным уравнением
ax2 + bx + c = 0, то можно заметить, что в уравнении
2x2 − 32 = 0 присутствует старший коэффициент a, равный 2;
присутствует свободный член −32; но отсутствует
коэффициент b.

Наконец, рассмотрим полное квадратное уравнение


ax2 + bx + c = 0. В качестве примера решим квадратное
уравнение x2 − 2x + 1 = 0.

Итак, требуется найти x, при котором левая часть станет


равна нулю. Воспользуемся изученными ранее
тождественными преобразованиями.

Прежде всего заметим, что левая часть уравнения


представляет собой квадрат разности двух выражений. Если
мы вспомним как раскладывать многочлен на множители, то
получим в левой части (x − 1)2.

Рассуждаем дальше. Левая часть возведенá в квадрат и она


равна нулю. Какое число в квадрате равно нулю? Очевидно,
что только 0. Поэтому наша задача найти x, при котором
1792

выражение x − 1 равно нулю. Решив простейшее уравнение


x − 1 = 0, можно узнать чему равно x

Этот же результат можно получить, если воспользоваться


квадратным корнем. В уравнении (x − 1)2 = 0 выражение
(x − 1) представляет собой квадратный корень из нуля. Тогда

можно записать, что . В этом примере записывать


перед корнем знак ± не нужно, поскольку корень из нуля
имеет только одно значение — ноль. Тогда получается
x − 1 = 0. Отсюда x = 1.

Значит корнем уравнения x2 − 2x + 1 = 0 является единица.


Других корней у данного уравнения нет. В данном случае мы
решили квадратное уравнение, имеющее только один
корень. Такое тоже бывает.

Не всегда бывают даны простые уравнения. Рассмотрим


например уравнение x2 + 2x − 3 = 0.

В данном случае левая часть уже не является квадратом


суммы или разности. Поэтому нужно искать другие пути
решения.

Заметим, что левая часть уравнения представляет собой


квадратный трехчлен. Тогда можно попробовать выделить
полный квадрат из этого трёхчлена и посмотреть что это нам
даст.

Выделим полный квадрат из квадратного трёхчлена,


располагающего в левой части уравнения:
1793

В получившемся уравнении перенесем −4 в правую часть,


изменив знак:

Теперь воспользуемся квадратным корнем. В уравнении


(x + 1)2 = 4 выражение (x + 1) представляет собой
квадратный корень из числа 4. Тогда можно записать, что

. Вычисление правой части даст выражение


x + 1 = ±2. Отсюда полýчится два уравнения: x + 1 = 2 и
x + 1 = −2, корнями которых являются числа 1 и −3

Значит корнями уравнения x2 + 2x − 3 = 0 являются числа 1 и


−3.

Выполним проверку:
1794

Пример 3. Решить уравнение x2 − 6x + 9 = 0, выделив полный


квадрат.

Выделим полный квадрат из левой части:

Далее воспользуемся квадратным корнем и узнáем чему


равно x

Значит корнем уравнения x2 − 6x + 9 = 0 является 3.


Выполним проверку:
1795

Пример 4. Решить квадратное уравнение 4x2 + 28x − 72 = 0,


выделив полный квадрат:

Выделим полный квадрат из левой части:

Перенесём −121 из левой части в правую часть, изменив


знак:

Воспользуемся квадратным корнем:

Получили два простых уравнения: 2x + 7 = 11 и 2x + 7 = −11.


Решим их:

Пример 5. Решить уравнение 2x2 + 3x − 27 = 0


1796

Это уравнение немного посложнее. Когда мы выделяем


полный квадрат, первый член квадратного трёхчлена мы
представляем в виде квадрата какого-нибудь выражения.

Так, в прошлом примере первым членом уравнения был 4x2.


Его можно было представить в виде квадрата выражения 2x,
то есть (2x)2 = 22x2 = 4x2. Чтобы убедиться что это правильно,
можно извлечь квадратный корень из выражения 4x2. Это
квадратный корень из произведения — он равен
произведению корней:

В уравнении 2x2 + 3x − 27 = 0 первый член это 2x2. Его нельзя


представить в виде квадрата какого-нибудь выражения.
Потому что нет числá, квадрат которого равен 2. Если бы
такое число было, то этим числом был бы квадратный корень
из числа 2. Но квадратный корень из числа 2 извлекается
только приближённо. А приближённое значение не годится
для представления числá 2 в виде квадрата.

Если обе части исходного уравнения умножить или


разделить на одно и то же число, то полýчится уравнение
равносильное исходному. Это правило сохраняется и для
квадратного уравнения.

Тогда можно разделить обе части нашего уравнения на 2.


Это позвóлит избавиться от двойки перед x2 что
впоследствии даст нам возможность выделить полный
квадрат:
1797

Перепишем левую часть в виде трёх дробей со


знаменателем 2

Сократим первую дробь на 2. Остальные члены левой части


перепишем без изменений. Правая часть по-прежнему станет
равна нулю:

Выделим полный квадрат.

При представлении члена в виде удвоенного


произведения, появление множителя 2 привело бы к тому,

что этот множитель и знаменатель дроби сократились бы.


Чтобы этого не произошло, удвоенное произведение было

домножено на . При выделении полного квадрата всегда


нужно стараться сделать так, чтобы значение изначального
выражения не изменилось.
1798

Свернём полученный полный квадрат:

Приведём подобные члены:

Перенесём дробь в правую часть, изменив знак:

Воспользуемся квадратным корнем. Выражение

представляет собой квадратный корень из числа

Для вычисления правой части воспользуемся правилом


извлечения квадратного корня из дроби:
1799

Тогда наше уравнение примет вид:

Полýчим два уравнения:

Решим их:

Значит корнями уравнения 2x2 + 3x − 27 = 0 являются числа 3

и .

Корень удобнее оставить в таком виде, не выполняя


деления числителя на знаменатель. Так проще будет
выполнять проверку.

Выполним проверку. Подставим найденные корни в исходное


уравнение:
1800

В обоих случаях левая часть равна нулю, значит уравнение


2x2 + 3x − 27 = 0 решено верно.

Решая уравнение 2x2 + 3x − 27 = 0, в самом начале мы


разделили обе его части на 2. В результате получили
квадратное уравнение, в котором коэффициент перед x2
равен единице:

Такой вид квадратного уравнения называют приведённым


квадратным уравнением.

Любое квадратное уравнение вида ax2 + bx + c = 0 можно


сделать приведённым. Для этого нужно разделить обе его
части на коэффициент, который располагается перед x². В
данном случае обе части уравнения ax2 + bx + c = 0 нужно
разделить на a
1801

Пример 6. Решить квадратное уравнение 2x2 + x + 2 = 0

Сделаем данное уравнение приведённым:

Выделим полный квадрат:


1802

Получили уравнение , в котором квадрат

выражения равен отрицательному числу . Такого


быть не может, поскольку квадрат любого числа или
выражения всегда положителен.

Следовательно, нет такого значения x, при котором левая

часть стала бы равна . Значит уравнение не


имеет корней.
1803

А поскольку уравнение равносильно исходному


уравнению 2x2 + x + 2 = 0, то и оно (исходное уравнение) не
имеет корней.

Формулы корней квадратного уравнения

Выделять полный квадрат для каждого решаемого


квадратного уравнения не очень удобно.

Можно ли создать универсальные формулы для решения


квадратных уравнений? Оказывается можно. Сейчас мы этим
и займёмся.

Взяв за основу буквенное уравнение ax2 + bx + c = 0, и


выполнив некоторые тождественные преобразования, мы
сможем получить формулы для вывода корней квадратного
уравнения ax2 + bx + c = 0. В эти формулы можно будет
подставлять коэффициенты a, b, с и получать готовые
решения.

Итак, выделим полный квадрат из левой части уравнения


ax2 + bx + c = 0. Сначала сделаем данное уравнение
приведённым. Разделим обе его части на a

Теперь в получившемся уравнении выделим полный квадрат:


1804

Перенесем члены и в правую часть, изменив знак:

Приведём правую часть к общему знаменателю. Дроби,


состоящие из букв, привóдят к общему знаменателю
методом «крест-нáкрест». То есть знаменатель первой дроби
станóвится дополнительным множителем второй дроби, а
знаменатель второй дроби станóвится дополнительным
множителем первой дроби:

В числителе правой части вынесем за скобки a


1805

Сократим правую часть на a

Поскольку все преобразования были тождественными, то

получившееся уравнение имеет те же корни,


что и исходное уравнение ax2 + bx + c = 0.

Уравнение будет иметь корни только тогда,


если правая часть больше нуля или равна нулю. Это потому
что в левой части выполнено возведéние в квадрат, а
квадрат любого числа положителен или равен нулю (если в
этот квадрат возвóдится ноль). А чему будет равна правая
часть зависит от того, что будет подставлено вместо
переменных a, b и c.

Поскольку при любом a не рáвным нулю, знаменатель

правой части уравнения всегда будет

положительным, то знак дроби будет зависеть от знака


её числителя, то есть от выражения b2 − 4ac.
1806

Выражение b2 − 4ac называют дискриминантом


квадратного уравнения. Дискриминант это латинское
слово, означающее различитель. Дискриминант квадратного
уравнения обозначается через букву D

D = b2 − 4ac

Дискриминант позволяет заранее узнать имеет ли уравнение


корни или нет. Так, в предыдущем задании мы долго решали
уравнение 2x2 + x + 2 = 0 и оказалось, что оно не имеет
корней. Дискриминант же позволил бы нам заранее узнать,
что корней нет. В уравнении 2x2 + x + 2 = 0 коэффициенты a,
b и c равны 2, 1 и 2 соответственно. Подставим их в формулу
D = b2−4ac

D = b2 − 4ac = 12 − 4 × 2 × 2 = 1 − 16 = −15.

Видим, что D (оно же b2 − 4ac) является отрицательным


числом. Тогда нет смысла решать уравнение 2x2 + x + 2 = 0,
выделяя в нём полный квадрат, потому что когда мы дойдем

до уравнения вида , окажется что правая


часть станет меньше нуля (из-за отрицательного
дискриминанта). А квадрат числа не может быть
отрицательным. Следовательно, корней у данного уравнения
не будет.

Станóвится понятно почему древние люди считали


выражение b2 − 4ac различителем. Это выражение подобно
индикатору позволяет различить уравнение имеющего корни
от уравнения, не имеющего корней.
1807

Итак, D равно b2 − 4ac. Подставим в уравнении

вместо выражения b2 − 4ac букву D

Если дискриминант исходного уравнения окажется меньше


нуля (D < 0), то уравнение примет вид:

В этом случае говорят, что у исходного уравнения корней


нет, поскольку квадрат любого числа не должен быть
отрицательным.

Если дискриминант исходного уравнения окажется больше


нуля (D > 0), то уравнение примет вид:

В этом случае уравнение будет иметь два корня. Для их


вывода воспользуемся квадратным корнем:
1808

Получили уравнение . Из него полýчится два

уравнения: и . Выразим x в каждом из


уравнений:

Получившиеся два равенства это и есть универсальные


формулы для решения квадратного уравнения
ax2 + bx + c = 0. Их называют формулами корней
квадратного уравнения.

Чаще всего эти формулы обозначаются как x1 и x2. То есть


для вычисления первого корня используется формула c
индексом 1; для вывода второго корня — формула с
индексом 2. Обозначим свои формулы так же:

Очерёдность применения формул не важнá.

Решим например квадратное уравнение x2 + 2x − 8 = 0 с


помощью формул корней квадратного уравнения.
Коэффициенты данного квадратного уравнения это числа 1,
2 и −8. То есть, a = 1, b = 2, c = −8.
1809

Прежде чем использовать формулы корней квадратного


уравнения, нужно найти дискриминант этого уравнения.

Найдём дискриминант квадратного уравнения. Для этого


воспользуемся формулой D = b2 − 4ac. Вместо переменных a,
b и c у нас будут коэффициенты уравнения x2 + 2x − 8 = 0

D = b2 − 4ac = 22− 4 × 1 × (−8) = 4 + 32 = 36

Дискриминант больше нуля. Значит уравнение имеет два


корня. Теперь можно воспользоваться формулами корней
квадратного уравнения:

Значит корнями уравнения x2 + 2x − 8 = 0 являются числа 2 и


−4. Проверкой убеждаемся, что корни найдены верно:

Наконец, рассмотрим случай когда дискриминант


квадратного уравнения равен нулю. Вернёмся к уравнению
1810

. Если дискриминант равен нулю, то правая


часть уравнения примет вид:

И в этом случае квадратное уравнение будет иметь только


один корень. Воспользуемся квадратным корнем:

Далее выражаем x

Это ещё одна формула для вывода корня квадратного корня.


Рассмотрим её применение. Ранее мы решили уравнение
x2 − 6x + 9 = 0, имеющее один корень 3. Решили мы его
методом выделения полного квадрата. Теперь попробуем
решить с помощью формул.

Найдём дискриминант квадратного уравнения. В этом


уравнении a = 1, b = −6, c = 9. Тогда по формуле
дискриминанта имеем:

D = b2 − 4ac = (−6)2 − 4 × 1 × 9 = 36 − 36 = 0


1811

Дискриминант равен нулю (D = 0). Это означает, что


уравнение имеет только один корень, и вычисляется он по

формуле

Значит корнем уравнения x2 − 6x + 9 = 0 является число 3.

Для квадратного уравнения, имеющего один корень также

применимы формулы и . Но применение


каждой из них будет давать один и тот же результат.

Применим эти две формулы для предыдущего уравнения. В


обоих случаях получим один и тот же ответ 3

Если квадратное уравнение имеет только один корень, то

желательно применять формулу , а не формулы

и . Это позволяет сэкономить время и


место.

Пример 3. Решить уравнение 5x2 − 6x + 1 = 0

Найдём дискриминант квадратного уравнения:


1812

Дискриминант больше нуля. Значит уравнение имеет два


корня. Воспользуемся формулами корней квадратного
уравнения:

Значит корнями уравнения 5x2 − 6x + 1 = 0 являются числа 1

и .

Ответ: 1; .

Пример 4. Решить уравнение x2 + 4x + 4 = 0

Найдём дискриминант квадратного уравнения:

Дискриминант равен нулю. Значит уравнение имеет только

один корень. Он вычисляется по формуле

Значит корнем уравнения x2 + 4x + 4 = 0 является число −2.


1813

Ответ: −2.

Пример 5. Решить уравнение 3x2 + 2x + 4 = 0

Найдём дискриминант квадратного уравнения:

Дискриминант меньше нуля. Значит корней у данного


уравнения нет.

Ответ: корней нет.

Пример 6. Решить уравнение (x + 4)2 = 3x + 40

Приведём данное уравнение к нормальному виду. В левой


части располагается квадрата суммы двух выражений.
Раскрóем его:

Перенесём все члены из правой части в левую часть,


изменив их знаки. В правой части останется ноль:

Приведём подобные члены в левой части:

В получившемся уравнении найдём дискриминант:


1814

Дискриминант больше нуля. Значит уравнение имеет два


корня. Воспользуемся формулами корней квадратного
уравнения:

Значит корнями уравнения (x + 4)2 = 3x + 40 являются числа


3 и −8.

Ответ: 3; −8.

Пример 7. Решить уравнение

Умнóжим обе части данного уравнения на 2. Это позвóлит


нам избавиться от дроби в левой части:

В получившемся уравнении перенесём 22 из правой части в


левую часть, изменив знак. В правой части останется 0

Приведём подобные члены в левой части:


1815

В получившемся уравнении найдём дискриминант:

Дискриминант больше нуля. Значит уравнение имеет два


корня. Воспользуемся формулами корней квадратного
уравнения:

Значит корнями уравнения являются числа 23 и


−1.

Ответ: 23; −1.

Пример 8. Решить уравнение

Умнóжим обе части на наименьшее общее кратное


знаменателей обеих дробей. Это позвóлит избавиться от
дробей в обеих частях. Наименьшее общее кратное чисел 2
и 3 это число 6. Тогда получим:
1816

В получившемся уравнении раскроем скобки в обеих частях:

Теперь перенесём все члены из правой части в левую часть,


изменив у них знаки. В правой части останется 0

Приведём подобные члены в левой части:

В получившемся уравнении найдём дискриминант:

Дискриминант больше нуля. Значит уравнение имеет два


корня. Воспользуемся формулами корней квадратного
уравнения:

Значит корнями уравнения являются числа и 2.

Примеры решения квадратных уравнений

Пример 1. Решить уравнение x2 = 81


1817

Это простейшее квадратное уравнение, в котором надо


определить число, квадрат которого равен 81. Таковыми
являются числа 9 и −9. Воспользуемся квадратным корнем
для их вывода:

Ответ: 9, −9.

Пример 2. Решить уравнение x2 − 9 = 0

Это неполное квадратное уравнение. Для его решения нужно


перенести член −9 в правую часть, изменив знак. Тогда
получим:

Ответ: 3, −3.

Пример 3. Решить уравнение x2 − 9x = 0

Это неполное квадратное уравнение. Для его решения


сначала нужно вынести x за скобки:
1818

Левая часть уравнения является произведением.


Произведение равно нулю, если хотя один из сомножителей
равен нулю.

Левая часть станет равна нулю, если отдельно x равно нулю,


или если выражение x − 9 равно нулю. Получится два
уравнения, одно из которых уже решено:

Ответ: 0, 9.

Пример 4. Решить уравнение x2 + 4x − 5 = 0

Это полное квадратное уравнение. Его можно решить


методом выделения полного квадрата или с помощью
формул корней квадратного уравнения.

Решим данное уравнение с помощью формул. Сначала


найдём дискриминант:

D = b2 − 4ac = 42 − 4 × 1 × (−5) = 16 + 20 = 36

Дискриминант больше нуля. Значит уравнение имеет два


корня. Вычислим их:
1819

Ответ: 1, −5.

Пример 5. Решить уравнение

Умнóжим обе части на наименьшее общее кратное чисел 5, 3


и 6. Это позвóлит избавиться от дробей в обеих частях:

В получившемся уравнении перенесём все члены из правой


части в левую часть, изменив знак. В правой части останется
ноль:

Приведём подобные члены:

Решим получившееся уравнение с помощью формул:


1820

Ответ: 5, .

Пример 6. Решить уравнение x2 = 6

В данном примере как и в первом нужно воспользоваться


квадратным корнем:

Однако, квадратный корень из числа 6 не извлекается. Он


извлекается только приближённо. Корень можно извлечь с
определённой точностью. Извлечём его с точностью до
сотых:

Но чаще всего корень оставляют в виде радикала:

Ответ:

Пример 7. Решить уравнение (2x + 3)2 + (x − 2)2 = 13

Раскроем скобки в левой части уравнения:


1821

В получившемся уравнении перенесём 13 из правой части в


левую часть, изменив знак. Затем приведём подобные
члены:

Получили неполное квадратное уравнение. Решим его:

Ответ: 0, −1,6.

Пример 8. Решить уравнение (5 + 7x)(4 − 3x) = 0

Данное уравнение можно решить двумя способами.


Рассмотрим каждый из них.

Первый способ. Раскрыть скобки и получить нормальный


вид квадратного уравнения.

Раскроем скобки:

Приведём подобные члены:


1822

Перепишем получившееся уравнение так, чтобы член со


старшим коэффициентом располагался первым, член со
вторым коэффициентом — вторым, а свободный член
располагался третьим:

Чтобы старший член стал положительным, умнóжим обе


части уравнения на −1. Тогда все члены уравнения поменяют
свои знаки на противоположные:

Решим получившееся уравнение с помощью формул корней


квадратного уравнения:

Второй способ. Найти значения x, при которых сомножители


левой части уравнения равны нулю. Этот способ удобнее и
намного короче.

Произведение равно нулю, если хотя бы один из


сомножителей равен нулю. В данном случае равенство в
уравнении (5 + 7x)(4 − 3x) = 0 будет достигаться, если
выражение (5 + 7x) равно нулю, или же выражение (4 − 3x)
1823

равно нулю. Наша задача выяснить при каких x это


происходит:

Примеры решения задач

Предстáвим, что возникла необходимость построить


небольшую комнату, площадь которой 8 м2. При этом длина
комнаты должна быть в два раза больше её ширины. Как
определить длину и ширину такой комнаты?

Сделаем примерный рисунок этой комнаты, который


иллюстрирует вид сверху:

Обозначим ширину комнаты через x. А длину комнаты через


2x, потому что по условию задачи длина должна быть в два
раза больше ширины. Множитель 2 и выполнит это
требование:
1824

Поверхность комнаты (её пол) является прямоугольником.


Для вычисления площади прямоугольника, нужно длину
данного прямоугольника умножить на его ширину. Сделаем
это:

2x × x

По условию задачи площадь должна быть 8 м2. Значит


выражение 2x × x следует приравнять к 8

2x × x = 8

Получилось уравнение. Если решить его, то можно найти


длину и ширину комнаты.

Первое что можно сделать это выполнить умножение в левой


части уравнения:

2x2 = 8

В результате этого преобразования переменная x перешла


во вторую степень. А мы говорили, что если переменная,
входящая в уравнение, возведенá во вторую степень (в
1825

квадрат), то такое уравнение является уравнением второй


степени или квадратным уравнением.

Для решения нашего квадратного уравнения воспользуемся


изученными ранее тождественными преобразованиями. В
данном случае можно разделить обе части на 2

Теперь воспользуемся квадратным корнем. Если x2 = 4, то

. Отсюда x = 2 и x = −2.

Через x была обозначена ширина комнаты. Ширина не


должна быть отрицательной, поэтому в расчёт берём только
значение 2. Такое часто бывает при решении задачи, в
которых применяется квадратное уравнение. В ответе
получаются два корня, но условию задачи удовлетворяет
только один из них.

А длина была обозначена через 2x. Значение x теперь


известно, подставим его в выражение 2x и вычислим длину:

2x = 2 × 2 = 4

Значит длина равна 4 м, а ширина 2 м. Это решение


удовлетворяет условию задачи, поскольку площадь комнаты
равна 8 м2

4 × 2 = 8 м2

Ответ: длина комнаты составляет 4 м, а ширина 2 м.


1826

Пример 2. Огородный участок, имеющий форму


прямоугольника, одна сторона которого на 10 м больше
другой, требуется обнести изгородью. Определить длину
изгороди, если известно, что площадь участка равна 1200 м2

Решение

Длина прямоугольника, как правило, больше его ширины.


Пусть ширина участка x метров, а длина (x + 10) метров.
Площадь участка составляет 1200 м2. Умножим длину
участка на его ширину и приравняем к 1200, получим
уравнение:

x(x + 10) = 1200

Решим данное уравнение. Для начала раскроем скобки в


левой части:

Перенесём 1200 из правой части в левую часть, изменив


знак. В правой части останется 0

Решим получившееся уравнение с помощью формул:


1827

Несмотря на то, что квадратное уравнение имеет два корня,


в расчёт берём только значение 30. Потому что ширина не
может выражаться отрицательным числом.

Итак, через x была обозначена ширина участка. Она равна


тридцати метрам. А длина была обозначена через
выражение x + 10. Подставим в него найденное значение x и
вычислим длину:

x + 10 = 30 + 10 = 40 м

Значит длина участка составляет сорок метров, а ширина


тридцать метров. Эти значения удовлетворяют условию
задачи, поскольку если перемножить длину и ширину (числа
40 и 30) получится 1200 м2

40 × 30 = 1200 м2

Теперь ответим на вопрос задачи. Какова длина изгороди?


Чтобы её вычислить нужно найти периметр участка.

Периметр прямоугольника это сумма всех его сторон. Тогда:

P = 2(a + b) = 2 × (40 + 30) = 2 × 70 = 140 м.

Ответ: длина изгороди огородного участка составляет 140 м.

Задания для самостоятельного решения


Задание 1. Решить уравнение:

Показать решение
Задание 2. Решить уравнение:
1828

Показать решение
Задание 3. Решить уравнение:

Показать решение
Задание 4. Решить уравнение, используя выделение полного
квадрата:

Показать решение
Задание 5. Решить уравнение, используя выделение полного
квадрата:

Показать решение
Задание 6. Решить уравнение, используя выделение полного
квадрата:

Показать решение
Задание 7. Решить уравнение:

Показать решение
Задание 8. Решить уравнение:

Показать решение
Задание 9. Решить уравнение:

Показать решение
1829

Задание 10. Решить уравнение:

Показать решение
Задание 11. Решить уравнение:

Показать решение
Задание 12. Решить уравнение:

Показать решение
Задание 13. Решить уравнение:

Показать решение
Задание 14. Решить уравнение:

Показать решение
Задание 15. Решить уравнение:

Показать решение
Задание 16. Решить уравнение:

Показать решение
Задание 17. Решить уравнение:

Показать решение
1830

Задание 18. Решить уравнение:

Показать решение
1831

Квадратное уравнение с чётным вторым коэффициентом

Если в квадратном уравнении ax2 + bx + c = 0 второй


коэффициент b является чётным, то решение этого
уравнения можно немного упростить. Дискриминант для
такого уравнения можно вычислить по формуле D1 = k2 − ac, а

корни по формулам и .

Содержание урока

 Примеры
 Вывод формул
 Задания для самостоятельного решения

Примеры

Решим квадратное уравнение x2 + 6x − 16 = 0. В нём второй


коэффициент является чётным. Чтобы воспользоваться
формулами для чётного коэффициента, нужно сначала
узнать чему равна переменная k.

Любое четное число n можно представить в виде


произведения числа 2 и числа k, то есть 2k.

n = 2k

Например, число 10 можно представить как 2 × 5.

10 = 2 × 5

В этом произведении k = 5.


1832

Число 12 можно представить как 2 × 6.

12 = 2 × 6

В этом произведении k = 6.

Число −14 можно представить как 2 × (−7)

В этом произведении k = −7.

Как видим, сомножитель 2 не меняется. Меняется только


сомножитель k.

В уравнении x2 + 6x − 16 = 0 вторым коэффициентом


является число 6. Это число можно представить как 2 × 3. В
этом произведении k = 3. Теперь можно воспользоваться
формулами для чётного коэффициента.

Найдем дискриминант по формуле D1 = k2 − ac

D1 = k2 − ac = 32 − 1 × (−16) = 9 + 16 = 25

Теперь вычислим корни по формулам: и

Значит корнями уравнения x2 + 6x − 16 = 0 являются числа 2


и −8.
1833

В отличие от стандартной формулы для вычисления


дискриминанта (D=b2 − 4ac), в формуле D1 = k2 − ac не нужно
выполнять умножение числа 4 на ac.

И в отличие от формул и формулы

и не содержат в знаменателе
множитель 2 что опять же освобождает нас от
дополнительных вычислений.

Пример 2. Решить квадратное уравнение 5x2 − 6x + 1=0

Второй коэффициент является чётным числом. Его можно


представить в виде 2 × (−3). То есть k = −3. Найдём
дискриминант по формуле D1 = k2 − ac

D1 = k2 − ac = (−3)2 − 5 × 1 = 9 − 5 = 4

Дискриминант больше нуля. Значит уравнение имеет два


корня. Для их вычисления воспользуемся формулами

Пример 3. Решить квадратное уравнение x2 − 10x − 24 = 0


1834

Второй коэффициент является чётным числом. Его можно


представить в виде 2 × (−5). То есть k = −5. Найдём
дискриминант по формуле D1 = k2 − ac

D1 = k2 − ac = (−5)2 − 1 × (−24) = 25 + 24 = 49

Дискриминант больше нуля. Значит уравнение имеет два


корня. Для их вычисления воспользуемся формулами

Обычно для определения числа k поступают так: делят


второй коэффициент на 2.

Действительно, если второй коэффициент b является


чётным числом, то его можно представить как b = 2k. Чтобы
из этого равенства выразить сомножитель k, нужно
произведение b разделить на сомножитель 2

Например, в предыдущем примере для определения числа k


можно было просто разделить второй коэффициент −10 на 2
1835

Пример 5. Решить квадратное уравнение

Коэффициент b равен . Это выражение состоит из

множителя 2 и выражения . То есть оно уже представлено


в виде 2k. Получается, что

Найдём дискриминант по формуле D1 = k2 − ac

Дискриминант больше нуля. Значит уравнение имеет два


корня. Для их вычисления воспользуемся формулами

При вычислении корня уравнения получилась дробь, в


которой содержится квадратный корень из числа 2.
Квадратный корень из числа 2 извлекается только
приближённо. Если выполнить это приближённое
извлечение, а затем сложить результат с 2, и затем
разделить числитель на знаменатель, то получится не очень
красивый ответ.

В таких случаях ответ записывают, не выполняя


приближённых вычислений. В нашем случае первый корень

уравнения будет равен .

Вычислим второй корень уравнения:


1836

Вывод формул

Давайте наглядно увидим, как появились формулы для


вычисления корней квадратного уравнения с чётным вторым
коэффициентом.

Рассмотрим квадратное уравнение ax2 + bx + c = 0.


Допустим, что коэффициент b является чётным числом.
Тогда его можно обозначить как 2k

b = 2k

Заменим в уравнении ax2 + bx + c = 0 коэффициент b на


выражение 2k

ax2 + 2kx + c = 0

Теперь вычислим дискриминант по ранее известной


формуле:

D = b2 − 4ac = (2k)2 − 4ac = 4k2 − 4ac

Вынесем в получившемся выражении за скобки общий


множитель 4

D = b2 − 4ac = (2k)2 − 4ac = 4k2 − 4ac = 4(k2 − ac)

Что можно сказать о получившемся дискриминанте? При


чётном втором коэффициенте он состоит из множителя 4 и
выражения k2 − ac.
1837

В выражении 4(k2 − ac) множитель 4 постоянен. Значит знак


дискриминанта зависит от выражения k2 − ac. Если это
выражение меньше нуля, то и D будет меньше нуля. Если
это выражение больше нуля, то и D будет больше нуля. Если
это выражение равно нулю, то и D будет равно нулю.

То есть выражение k2 − ac это различитель — дискриминант.


Такой дискриминант принято обозначать буквой D1

D1 = k2 − ac

Теперь посмотрим как выводятся формулы и

В нашем уравнении ax2 + bx + c = 0 коэффициент b заменён


на выражение 2k. Воспользуемся стандартными формулами

для вычисления корней. То есть формулами и

. Только вместо b будем подставлять 2k. Также на


забываем, что D у нас равно выражению 4(k2 − ac)

Но ранее было сказано, что выражение k2 − ac обозначается


через D1. Тогда в наших преобразованиях следует сделать и
эту замену:
1838

Теперь вычислим квадратный корень, расположенный в


числителе. Это квадратный корень из произведения — он
равен произведению корней. Остальное перепишем без
изменений:

Теперь в получившемся выражении вынесем за скобки


общий множитель 2

Сократим получившуюся дробь на 2

Аналогично вывóдится формула для вычисления второго


корня:
1839

Задания для самостоятельного решения


Задание 1. Решить уравнение:

Показать решение
Задание 2. Решить уравнение:

Показать решение
Задание 3. Решить уравнение:

Показать решение
Задание 4. Решить уравнение:

Показать решение
Задание 5. Решить уравнение:

Показать решение
Задание 6. Решить уравнение:

Показать решение
Задание 7. Решить уравнение:

Показать решение
1840

Теорема Виета

Предварительные навыки

 Общие сведения об уравнениях


 Решение задач с помощью уравнений
 Квадратный корень

Содержание урока


o Что называют теоремой?
o Теорема Виета
o Доказательство теоремы Виета
o Теорема, обратная теореме Виета
o Примеры решения уравнений по теореме, обратной
теореме Виета
o Когда квадратное уравнение неприведённое
o Задания для самостоятельного решения

Что называют теоремой?

Если человек обнаружил в математике какую-нибудь


закономерность, позволяющую быстро решить ту или иную
задачу, то ему не следует говорить о том, что он сделал
открытие. Потому что может случиться так, что эта
закономерность работает только для определённых случаев,
а для других не работает или вовсе решает задачу
неправильно.

Чтобы поделиться своим открытием с другими людьми,


найденную закономерность следует сформулировать в виде
1841

утверждения, а затем доказать это утверждение, приводя


неоспоримые факты.

Сформулированное утверждение называют теоремой. А


доказательство теоремы состоит из фактов, логических
рассуждений и вычислений, которые не оспариваются.

Например, теоремой можно назвать следующее


утверждение:

«Если числитель и знаменатель обыкновенной дроби


умнóжить на какое-нибудь число, то значение данной
дроби не измéнится».

А затем привести такое доказательство:

Пусть, имеется дробь . Умнóжим числитель и знаменатель

этой дроби на число с. Тогда полýчится дробь . Докáжем,

что дроби  и равны. То есть докажем, что равенство


является верным.

Для доказательства этого равенства воспользуемся


основным свойством пропорции:

От перестановки мест сомножителей произведение не


меняется. Поэтому в получившемся равенстве можно
упорядочить правую часть по алфавиту:
1842

Поскольку равенство является пропорцией, а пропорция

это равенство двух отношений, то дроби и равны.


Теорема доказана.

Теорема Виета

Французский математик Франсуа Виет выявил интересную


взаимосвязь между коэффициентами приведённого
квадратного уравнения и корнями этого же уравнения. Эта
взаимосвязь представлена в виде теоремы и формулируется
так:

Сумма корней приведённого квадратного уравнения


x2 + bx + c = 0 равна второму коэффициенту, взятому с
противоположным знáком, а произведение корней равно
свободному члену.

То есть, если имеется приведённое квадратное уравнение


x2 + bx + c = 0, а его корнями являются числа x1 и x2, то
справедливы следующие два равенства:

Знак системы (фигурная скобка) говорит о том, что значения


x1 и x2 удовлетворяют обоим равенствам.

Покажем теорему Виета на примере приведённого


квадратного уравнения x2 + 4x + 3 = 0.
1843

Мы пока не знаем какие корни имеет уравнение


x2 + 4x + 3 = 0. Но по теореме Виета можно записать, что
сумма этих корней равна второму коэффициенту 4, взятому с
противоположным знáком. Если коэффициент 4 взять с
противоположным знáком, то получим −4. Тогда:

А произведение корней по теореме Виета будет равно


свободному члену. В уравнении x2 + 4x + 3 = 0 свободным
членом является 3. Тогда:

Теперь проверим действительно ли сумма корней равна −4,


и равно ли произведение 3. Для этого найдём корни
уравнения x2 + 4x + 3 = 0. А для удобства воспользуемся
формулами для чётного второго коэффициента:

Корнями уравнения являются числа −1 и −3. По теореме


Виета их сумма должна была равняться второму
коэффициенту уравнения x2 + 4x + 3 = 0, взятому с
противоположным знаком. Действительно, так оно и есть.
Вторым коэффициентов в уравнении x2 + 4x + 3 = 0 является
1844

4. Если взять его с противоположным знаком и приравнять


сумму корней x1 + x2 к этому коэффициенту, то получается
верное равенство:

А произведение корней −1 и −3 по теореме Виета должно


было равняться свободному члену уравнения x2 + 4x + 3 = 0,
то есть числу 3. Видим, что это условие тоже выполняется:

Значит выражение  является справедливым.

Рассмотрим квадратное уравнение x2 − 8x + 15 = 0. По


теореме Виета сумма корней этого уравнения равна второму
коэффициенту, взятому с противоположным знаком. Второй
коэффициент равен −8. Если взять его с противоположным
знаком, то получим 8. Тогда:

А произведение корней равно свободному члену. В


уравнении x2 − 8x + 15 = 0 свободным членом является 15.
Тогда:
1845

Теперь проверим действительно ли сумма корней равна 8, и


равно ли произведение 15. Для этого найдём корни данного
уравнения. А для удобства воспользуемся формулами для
чётного второго коэффициента. В этот раз пропустим
нéкоторые подробные записи:

Видим, что корнями уравнения x2 − 8x + 15 = 0 являются


числа 5 и 3. Их сумма равна 8. То есть сумма корней равна
второму коэффициенту уравнения x2 − 8x + 15 = 0, взятому с
противоположным знаком.

А произведение чисел 5 и 3 равно 15. То есть равно


свободному члену уравнения x2 − 8x + 15 = 0.

Значит выражение является справедливым.

Замечание. Чтобы теорема Виета выполнялась, квадратное


уравнение обязательно должно быть приведённым и иметь
корни.

Например, рассмотрим квадратное уравнение x2 − 2x + 4 = 0.


Напишем сумму и произведение корней этого уравнения:
1846

Но уравнение x2 − 2x + 4 = 0 не имеет корней, сумма которых


равна 2, а произведение которых равно 4. Убедиться в этом
можно, вычислив дискриминант:

D1 = k2 − ac = (−1)2 − 1 × 4 = −3

А значит записывать выражение не имеет смысла.

Теорема Виета полезна тем, что позволяет до начала


решения узнать знаки корней уравнения.

Например, запишем для уравнения x2 − 5x + 6 = 0 сумму и


произведение его корней. Сумма корней равна второму
коэффициенту, взятому с противоположным знаком, а
произведение корней равно свободному члену:

Посмотрев на эти два равенства можно сразу понять, что оба


корня должны быть положительными. Потому что
произведение x1 × x2 = 6 будет выполняться только в двух
случаях: если значения x1 и x2 положительны либо они оба
отрицательны. Если эти значения будут отрицательными, то
не будет выполняться равенство x1 + x2 = 5, поскольку его
правая часть равна положительному числу. А значения x1 и
x2 должны удовлетворять как равенству x1 + x2 = 5, так и
равенству x1 × x2 = 6.

Ещё одна польза от теоремы Виета в том, что корни можно


найти методом подбора. В данном примере корни должны
быть такими, чтобы они удовлетворяли как равенству x1 + x2 =
1847

5 так и равенству x1 × x2 = 6. Очевидно, что таковыми


являются корни 3 и 2

Значит, x1 = 3, x2 = 2

Доказательство теоремы Виета

Пусть дано приведённое квадратное уравнение


x2 + bx + c = 0. Если его дискриминант больше нуля, то оно
имеет два корня, сумма которых равна второму
коэффициенту, взятому с противоположным знаком, а
произведение корней равно свободному члену:

Докажем, что равенства x1 + x2 = −b и x1 × x2 = c имеют место


быть.

Вспомним формулы корней квадратного уравнения:

Найдём сумму корней x1 и x2. Для этого подставим в


выражение x1 + x2 вместо x1 и x2 соответствующие выражения
1848

из правой части формул корней квадратного уравнения. Не


забываем, что в приведённом квадратном уравнении
x2 + bx + c = 0 старший коэффициент a равен единице. Тогда
в процессе подстановки знаменатель станет равен просто 2

Запишем правую часть в виде дроби с одним знаменателем:

Раскроем скобки в числителе и приведём подобные члены:

Сократим дробь на 2, тогда получим −b

Значит x1 + x2 действительно равно −b

x1 + x2 = −b

Теперь аналогично докажем, что произведение x1 × x2 равно


свободному члену c.
1849

Подставим вместо x1 и x2 соответствующие выражения из


формул корней квадратного уравнения. Не забываем, что
коэффициент a всё ещё равен единице:

Чтобы перемнóжить дроби, нужно перемнóжить их числители


и знаменатели:

В числителе теперь содержится произведение суммы двух


выражений и разности этих же выражений. Воспользуемся
тождеством (a + b)(a − b) = a2 − b2. Тогда в числителе
полýчится А знаменатель будет равен 4

Теперь в числителе выражение (−b)2 станет равно b2, а


выражение станет равно просто D

Но D равно b2 − 4ac. Подстáвим это выражение вместо D, не


забывая что a = 1. То есть вместо b2 − 4ac надо подставить
b2 − 4c
1850

В получившемся выражении раскроем скобки в числителе и


приведём подобные члены:

Сократим получившуюся дробь на 4

Значит x1 × x2 действительно равно c.

x1 × x2 = c

Таким образом, сумма корней приведённого квадратного


уравнения x2 + bx + c = 0 равна второму коэффициенту,
взятому с противоположным знáком (x1 + x2 = −b), а
произведение корней равно свободному члену (x1 × x2 = c).
Теорема доказана.
1851

Теорема, обратная теореме Виета

Когда записана сумма и произведение корней приведённого


квадратного уравнения, обычно начинается подбор
подходящих корней к этому уравнению. В этот момент в
работу включается так называемая теорема, обратная
теореме Виета. Она формулируется так:

Если числа x1 и x2 таковы, что их сумма равна второму


коэффициенту уравнения x2 + bx + c = 0, взятому с
противоположным знáком, а произведение чисел x1 и x2
равно свободному члену уравнения x2 + bx + c = 0, то
числа x1 и x2 являются корнями уравнения x2 + bx + c = 0.

Обратные теоремы бывают поставлены так, что их


утверждением является заключение первой теоремы.

Так, доказывая теорему Виета мы пришли к заключению, что


сумма x1 и x2 равна −b, а произведение x1 и x2 равно c. В
обратной же теореме это заключение служит утверждением.

Ранее мы решили уравнение x2 − 5x + 6 = 0 и написали для


него такую сумму и произведение корней:

А затем подобрали корни 3 и 2. По сути мы применили


теорему, обратную теореме Виета. Числа 3 и 2 таковы, что
их сумма равна второму коэффициенту уравнения
x2 − 5x + 6 = 0, взятому с противоположным знаком (числу 5),
а произведение чисел 3 и 2 равно свободному члену (числу
6). Значит числа 3 и 2 являются корнями уравнения
x2 − 5x + 6 = 0.
1852

Пример 2. Решить квадратное уравнение x2 − 6x + 8 = 0 по


теореме, обратной теореме Виета.

В данном уравнении a = 1. Значит квадратное уравнение


является приведённым. Его можно решить по теореме,
обратной теореме Виета.

Сначала запишем сумму и произведение корней уравнения.


Сумма корней будет равна 6, поскольку второй коэффициент
исходного уравнения равен −6. А произведение корней будет
равно 8

Теперь имея эти два равенства можно подобрать


подходящие корни. Они должны удовлетворять как равенству
x1 + x2 = 6, так и равенству x1 × x2 = 8

Подбор корней удобнее выполнять с помощью их


произведения. Используя равенство x1 × x2 = 8 нужно найти
такие x1 и x2, произведение которых равно 8.

Число 8 можно получить если перемножить числа 4 и 2 либо


1 и 8.

4×2=8
1×8=8

Но значения x1 и x2 надо подбирать так, чтобы они


удовлетворяли не только равенству x1 × x2 = 8, но и равенству
x1 + x2 = 6.
1853

Сразу делаем вывод, что значения 1 и 8 не годятся,


поскольку они хоть и удовлетворяют равенству x1 × x2 = 8, но
не удовлетворяют равенству x1 + x2 = 6.

Зато значения 4 и 2 подходят как равенству x1 × x2 = 8, так и


равенству x1 + x2 = 6, поскольку эти значения удовлетворяют
обоим равенствам:

Значит корнями уравнения x2 − 6x + 8 = 0 являются числа 4 и


2.

Обратная теорема, как и любая теорема нуждается в


доказательстве. Докажем теорему, обратную теореме Виета.
Для удобства корни x1 и x2 обозначим как m и n. Тогда
утверждение теоремы, обратной теореме Виета примет
следующий вид:

Если числа m и n таковы, что их сумма равна второму


коэффициенту уравнения x2 + bx + c = 0, взятому с
противоположным знáком, а произведение чисел m и n
равно свободному члену уравнения x2 + bx + c = 0, то
числа m и n являются корнями уравнения x2 + bx + c = 0

Для начала запишем, что сумма m и n равна −b, а


произведение mn равно c
1854

Чтобы доказать, что числа m и n являются корнями


уравнения x2 + bx + c = 0, нужно поочередно подстáвить
буквы m и n в это уравнение вместо x, затем выполнить
возможные тождественные преобразования. Если в
результате преобразований левая часть станет равна нулю,
то это будет означать, что числа m и n являются корнями
уравнения x2 + bx + c = 0.

Помимо букв m и n нам нужно знать чему равен параметр b.


Выразим его из равенства m + n = −b. Легче всего это
сделать, умножив обе части этого равенства на −1

Теперь всё готово для подстановок. Подстáвим m в


уравнение x2 + bx + c = 0 вместо x, а выражение −m − n
подставим вместо b

Видим, что при x = m получается верное равенство. Значит


число m является корнем уравнения x2 + bx + c = 0.

Аналогично докажем, что число n является корнем уравнения


x2 + bx + c = 0. Подставим вместо x букву n, а вместо c
подставим mn, поскольку c = mn.
1855

Видим, что при x = n тоже получается верное равенство.


Значит число n является корнем уравнения.

Следовательно, числа m и n являются корнями уравнения


x2 + bx + c = 0.

Примеры решения уравнений по теореме, обратной теореме


Виета

Пример 1. Решить квадратное уравнение x2 − 4x + 4 = 0 по


теореме, обратной теореме Виета.

Запишем сумму корней x1 и x2 и приравняем её к второму


коэффициенту, взятому с противоположным знаком. Также
запишем произведение корней x1 и x2 и приравняем его к
свободному члену:

В данном примере очевидно, что корнями являются числа 2 и


2. Потому что их сумма равна 4 и произведение равно 4

Значение x1 совпадает с x2. Это тот случай, когда квадратное


уравнение имеет только один корень. Если мы попробуем
решить данное уравнение с помощью формул корней
1856

квадратного уравнения, то обнаружим что дискриминант

равен нулю, и корень вычисляется по формуле

Данный пример показывает, что теорема обратная теореме


Виета, работает и для уравнений, имеющих только один
корень. Признаком того, что квадратное уравнение имеет
только один корень является то, что значения x1 и x2
совпадают.

Пример 2. Решить уравнение x2 + 3x + 2 = 0 по теореме,


обратной теореме Виета.

Запишем сумму и произведение корней данного уравнения:

Теперь подберём значения x1 и x2. Здесь начинается самое


интересное. Произведение корней равно 2. Число 2 можно
получить перемножив 1 и 2. Но сумма корней x1 + x2 равна
отрицательному числу −3. Значит значения 1 и 2 не
подходят.

Сумма бывает отрицательной если оба слагаемых


отрицательны либо отрицательным является одно
слагаемое, модуль которого больше.
1857

Если подберём корни с разными знаками, то не будет


выполняться равенство x1 × x2 = 2.

Если подберем положительные корни, то будет выполняться


равенство x1 × x2 = 2, но не будет выполняться равенство
x1 + x2 = −3.

Очевидно, что корнями являются два отрицательных числа.


Произведение отрицательных чисел есть положительное
число. А сумма отрицательных чисел есть отрицательное
число.

Тогда равенствам будут удовлетворять числа −1 и −2.

Итак, корнями являются числа −1 и −2

Пример 3. Решить уравнение x2 + 16x + 15 = 0 по теореме,


обратной теореме Виета.

Запишем сумму и произведение корней данного уравнения:

Как и в прошлом примере сумма корней равна


отрицательному числу, а произведение корней —
положительному числу.
1858

Произведение бывает положительным если оба


сомножителя положительны либо оба сомножителя
отрицательны. Первый вариант отпадает сразу, поскольку
сумма корней равна отрицательному числу. Тогда
получается, что оба корня будут отрицательными.
Попробуем подобрать их.

Число 15 можно получить, если перемножить числа −1 и −15


или (−3) и (−5). В данном случае подходит первый вариант,
поскольку сумма чисел −1 и −15 равна −16, а их
произведение равно 15. Значит корнями уравнения
x2 + 16x + 15 = 0 являются числа −1 и −15

Пример 4. Решить уравнение x2 − 10x − 39 = 0 по теореме,


обратной теореме Виета.

Запишем сумму и произведение корней данного уравнения:

Произведение корней равно отрицательному числу. Значит


один из корней является отрицательным. Число −39 можно
получить если перемножить числа −3 и 13 либо −13 и 3. Из
этих комбинаций больше годится комбинация −3 и 13,
поскольку при перемножении этих чисел получается −39, а
при сложении 10
1859

Значит корнями уравнения x2 − 10x − 39 = 0 являются числа


−3 и 13

Пример 5. Первый корень уравнения x2 + bx + 45 = 0 равен


15. Найти второй корень этого уравнения, а также значение
коэффициента b.

По теореме Виета произведение корней приведённого


квадратного уравнения равно свободному члену. В данном
случае это произведение равно 45

x1 × x2 = 45

При этом один из корней уже известен — это корень 15.

15 × x2 = 45

Тогда второй корень будет равен 3, потому что число 45


получается, если 15 умножить на 3

15 × 3 = 45

Значит x2 = 3

Этот второй корень также можно было бы получить, выразив


из равенства 15 × x2 = 45 переменную x2
1860

Теперь определим значение коэффициента b. Для этого


напишем сумму корней уравнения:

15 + 3 = 18

По теореме Виета сумма корней приведенного квадратного


уравнения равна второму коэффициенту, взятому с
противоположным знаком. Если сумма корней равна 18, а 18
это положительное число, то в самóм уравнении этот
коэффициент будет отрицательным:

x2 − 18x + 45 = 0

Значит b = −18.

Обычно решение к такой задаче записывают так. Сначала


записывают основную теорему Виета в виде суммы и
произведения корней:

Затем в это выражение подставляют имеющиеся известные


значения. В нашем случае известно, что первый корень
равен 15, а свободный член уравнения x2 + bx + 45 = 0 равен
45

Из этой системы следует найти x2 и b. Выразим эти


параметры:
1861

Из этой системы мы видим, что x2 равно 3. Подставим его в


первое равенство:

Теперь из первого равенства мы видим, что −b равно 18

Но нас интересует b, а не −b. Следует помнить, что −b это


−1b. Чтобы найти b нужно 18 разделить на −1. Тогда b станет
равно −18

Этот же результат можно получить если в выражении


умножить первое равенство на −1
1862

Теперь возвращаемся к исходному уравнению x2 + bx + 45 =


0 и подставляем найденное значение b

Выполним умножение −18 на x. Получим −18x

Раскроем скобки:

Пример 6. Используя теорему Виета, написать приведённое


квадратное уравнение, корнями которых являются числа 2 и
8.

В этом задании корни уже известны. То есть x1 = 2, x2 = 8. По


ним надо составить квадратное уравнение вида
x2 + bx + c = 0.

Запишем сумму и произведение корней:

По теореме Виета сумма корней приведённого квадратного


уравнения равна второму коэффициенту, взятому с
противоположным знаком. Если сумма корней 2 и 8 равна 10,
то в самóм уравнении число 10 должно быть с
противоположным знаком. Значит b = −10.

Произведение корней по теореме Виета равно свободному


члену. У нас это произведение равно 16.
1863

Значит b = −10, c = 16. Отсюда:

x2 − 10x + 16 = 0

Пример 7. Используя теорему Виета, написать приведённое


квадратное уравнение, корнями которых являются числа
и .

Запишем сумму и произведение корней:

Сумма корней равна 2. Тогда в уравнении второй


коэффициент будет равен −2. А произведение корней равно
−1. Значит свободный член будет равен −1. Тогда:

x2 − 2x − 1 = 0

Когда квадратное уравнение неприведённое

Теорема Виета выполняется только тогда, когда квадратное


уравнение является приведённым.

Если квадратное уравнение не является приведённым, но


всё равно возникла необходимость применить теорему
Виета, то обе части неприведённого квадратного уравнения
следует разделить на коэффициент, который располагается
перед x2.

Если к примеру в квадратном уравнении ax2 + bx + c = 0


коэффициент a не равен единице, то данное уравнение
1864

является неприведённым. Чтобы сделать его приведённым,


надо разделить обе его части на коэффициент, который
располагается перед x2, то есть на a

Получилось уравнение , которое является

приведённым. В нём второй коэффициент равен , а

свободный член равен . Тогда сумма и произведение


корней будут выглядеть так:

Например, решим квадратное уравнение 4x2 + 5x + 1 = 0. Это


уравнение не является приведённым. Приведённым оно
станет, если разделить обе его части на коэффициент,
который располагается перед x2, то есть на 4
1865

Получили приведённое квадратное уравнение. В нём второй

коэффициент равен , а свободный член . Тогда по теореме


Виета имеем:

Отсюда методом подбора находим корни −1 и

Возможно этот метод вы редко будете использовать при


решении квадратных уравнений. Но знать о нём не
помешает.

Пример 2. Решить квадратное уравнение 3x2 − 7x + 2 = 0

Данное уравнение не является приведённым, а значит его


пока нельзя решить по теореме, обратной теореме Виета.

Сделаем данное уравнение приведенным. Разделим обе


части на коэффициент, который располагается перед x2
1866

Получили уравнение . Запишем сумму и


произведение корней этого уравнения:

Отсюда методом подбора находим корни 2 и

Пример 3. Решить квадратное уравнение 2x2 − 3x − 2 = 0

Это неприведённое квадратное уравнение. Чтобы сделать


его приведённым, нужно разделить обе его части на 2.
Сделать это можно в уме. Если 2x2 разделить на 2, то
полýчится x2
1867

Далее если −3x разделить на 2, то полýчится . Чтобы


видеть где коэффициент, а где переменная, такое

выражение записывают в виде

Далее если −2 разделить на 2, то полýчится −1

Прирáвниваем получившееся выражение к нулю:

Теперь применяем теорему Виета. Сумма корней будет


равна второму коэффициенту, взятому с противоположным
знáком, а произведение корней свободному члену:

Отсюда методом подбора находим корни 2 и

Задания для самостоятельного решения


Задание 1. Написать сумму и произведение корней для
квадратного уравнения:

Показать решение
1868

Задание 2. Написать сумму и произведение корней для


квадратного уравнения:

Показать решение
Задание 3. Написать сумму и произведение корней для
квадратного уравнения:

Показать решение
Задание 4. Решить квадратное уравнение по теореме,
обратной теореме Виета:

Показать решение
Задание 5. Решить квадратное уравнение по теореме,
обратной теореме Виета:

Показать решение
Задание 6. Решить квадратное уравнение по теореме,
обратной теореме Виета:

Показать решение
Задание 7. Решить квадратное уравнение по теореме,
обратной теореме Виета:

Показать решение
Задание 8. Решить квадратное уравнение по теореме,
обратной теореме Виета:
1869

Показать решение
Задание 9. Решить квадратное уравнение по теореме,
обратной теореме Виета:

Показать решение
1870

Разложение квадратного трёхчлена на множители

Предварительные навыки

 Разложение многочлена на множители

 Квадратное уравнение

 Квадратное уравнение с чётным вторым коэффициентом

 Теорема Виета

Содержание урока

 Как разложить квадратный трёхчлен на множители


 Как это работает
 Примеры разложений
 Задания для самостоятельного решения

Как разложить на множители квадратный трёхчлен

Квадратный трёхчлен — это многочлен вида ax2 + bx + c.

В прошлых уроках мы решали квадратные уравнения. Общий


вид таких уравнений выглядел так:

ax2 + bx + c = 0

Левая часть этого уравнения является квадратным


трёхчленом.

Одним из полезных преобразований при решении задач


является разложение квадратного трёхчлена на множители.
Для этого исходный квадратный трёхчлен приравнивают к
нулю и решают квадратное уравнение. В этом случае
1871

говорят, что выполняется поиск корней квадратного


трёхчлена.

Полученные корни x1 и x2 следует подстáвить в следующее


выражение, которое и станет разложением:

a(x − x1)(x − x2)

Таким образом, чтобы разложить квадратный трёхчлен на


множители при помощи решения квадратного уравнения,
нужно воспользоваться следующей готовой формулой:

ax2 + bx + c = a(x − x1)(x − x2)

Где левая часть — исходный квадратный трёхчлен.

Пример 1. Разложить на множители следующий квадратный


трёхчлен:

x2 − 8x + 12

Найдём корни квадратного трёхчлена. Для этого приравняем


данный квадратный трёхчлен к нулю и решим квадратное
уравнение:

x2 − 8x + 12 = 0

В данном случае коэффициент b является чётным. Поэтому


можно воспользоваться формулами для чётного второго
коэффициента. Чтобы сэкономить время, некоторые
подробные вычисления можно пропустить:
1872

Итак, x1 = 6, x2 = 2. Теперь воспользуемся формулой


ax2 + bx + c = a(x − x1)(x − x2). В левой части вместо
выражения ax2 + bx + c напишем свой квадратный трёхчлен
x2 − 8x + 12. А в правой части подставим имеющиеся у нас
значения. В данном случае a = 1, x1 = 6, x2 = 2

x2 − 8x + 12 = 1(x − 6)(x − 2) = (x − 6)(x − 2)

Если a равно единице (как в данном примере), то решение


можно записать покороче:

x2 − 8x + 12 = (x − 6)(x − 2)

Чтобы проверить правильно ли разложен квадратный


трёхчлен на множители, нужно раскрыть скобки у правой
части получившегося равенства.

Раскроем скобки у правой части равенства, то есть в


выражении (x − 6)(x − 2). Если мы всё сделали правильно, то
должен получиться квадратный трёхчлен x2 − 8x + 12

(x − 6)(x − 2) = x2 − 6x − 2x + 12 = x2 − 8x + 12

Пример 2. Разложить на множители следующий квадратный


трёхчлен:

2x2 − 14x + 24
1873

Приравняем данный квадратный трёхчлен к нулю и решим


уравнение:

2x2 − 14x + 24 = 0

Как и в прошлом примере коэффициент b является чётным.


Поэтому можно воспользоваться формулами для чётного
второго коэффициента:

Итак, x1 = 4, x2 = 3. Приравняем квадратный трехчлен


2x2 − 14x + 24 к выражению a(x − x1)(x − x2), где вместо
переменных a, x1 и x2 подстáвим соответствующие значения.
В данном случае a = 2

2x2 − 14x + 24 = 2(x − 4)(x − 3)

Выполним проверку. Для этого раскроем скобки у правой


части получившегося равенства. Если мы всё сделали
правильно, то должен получиться квадратный трёхчлен
2x2 − 14x + 24

2(x − 4)(x − 3) = 2(x2 − 4x −3x + 12) = 2(x2 − 7x + 12) =


2x2 − 14x + 24
1874

Как это работает

Разложение квадратного трёхчлена на множители


происходит, если вместо коэффициентов квадратного
трёхчлена подстáвить теорему Виета и выполнить
тождественные преобразования.

Для начала рассмотрим случай, когда коэффициент a


квадратного трёхчлена равен единице:

x2 + bx + c

Вспоминаем, что если квадратное уравнение является


приведённым, то теорема Виета имеет вид:

Тогда приведённый квадратный трехчлен x2 + bx + c можно


разложить на множители следующим образом. Сначала
выразим b из уравнения x1 + x2 = −b. Для этого можно
умножить обе его части на −1

Переменную c из теоремы Виета выражать не нужно — она


уже выражена. Достаточно поменять местами левую и
правую часть:
1875

Теперь подставим выраженные переменные b и c в


квадратный трёхчлен x2 + bx + c

Раскроем скобки там где это можно:

В получившемся выражении выполним разложение


многочлена на множители способом группировки. В данном
случае удобно сгруппировать первый член со вторым, а
третий с четвёртым:

Из первых скобок вынесем общий множитель x, из вторых


скобок — общий множитель −x2

Далее замечаем, что выражение (x − x1) является общим


множителем. Вынесем его за скобки:

Мы пришли к тому, что выражение x2 + bx + c стало равно


(x − x1)(x − x2)

x2 + bx + c = (x − x1)(x − x2)
1876

Но это был случай, когда исходный квадратный трёхчлен


является приведённым. В нём коэффициент a равен
единице. И соответственно, в формуле разложения такого
квадратного трехчлена коэффициент a можно опустить.

Теперь рассмотрим случай, когда коэффициент a


квадратного трёхчлена не равен единице. Это как раз тот
случай, когда в формуле разложения присутствует перед
скобками коэффициент a

ax2 + bx + c = a(x − x1)(x − x2)

Вспоминаем, что если квадратное уравнение не является


приведённым, то есть имеет вид ax2 + bx + c = 0, то теорема
Виета принимает следующий вид:

Это потому что теорема Виета работает только для


приведённых квадратных уравнений. А чтобы уравнение
ax2 + bx + c = 0 стало приведённым, нужно разделить обе его
части на a
1877

Далее чтобы квадратный трёхчлен вида ax2 + bx + c


разложить на множители, нужно вместо b и c подставить
соответствующие выражения из теоремы Виета. Но в этот

раз нам следует использовать равенства и

Для начала выразим b и c. В первом равенстве умножим обе


части на a. Затем обе части получившегося равенства
умножим на −1

Теперь из второго равенства выразим c. Для этого умножим


обе его части на a

Теперь подставим выраженные переменные b и с в


квадратный трёхчлен ax2 + bx + c. Для наглядности каждое
преобразование будем выполнять на новой строчке:

Здесь вместо переменных b и c были подставлены


выражения −ax1 − ax2 и ax1x2, которые мы ранее выразили из
теоремы Виета. Теперь раскроем скобки там где это можно:
1878

В получившемся выражении выполним разложение


многочлена на множители способом группировки. В данном
случае удобно сгруппировать первый член со вторым, а
третий с четвёртым:

Теперь из первых скобок вынесем общий множитель ax, а из


вторых — общий множитель −ax2

Далее замечаем, что выражение x − x1 тоже является общим


множителем. Вынесем его за скобки:

Вторые скобки содержат общий множитель a. Вынесем его за


скобки. Его можно расположить в самом начале выражения:

Мы пришли к тому, что выражение ax2 + bx + c стало равно


a(x − x1)(x − x2)

ax2 + bx + c = a(x − x1)(x − x2)

Отметим, что если квадратный трехчлен не имеет корней, то


его нельзя разложить на множители. Действительно, если не
найдены корни квадратного трёхчлена, то нéчего будет
подставлять в выражение a(x − x1)(x − x2) вместо переменных
x1 и x2.
1879

Если квадратный трёхчлен имеет только один корень, то этот


корень одновременно подставляется в x1 и x2. Например,
квадратный трёхчлен x2 + 4x + 4 имеет только один корень −2

Тогда значение −2 в процессе разложения на множители


будет подставлено вместо x1 и x2. А значение a в данном
случае равно единице. Её можно не записывать, поскольку
это ничего не даст:

Скобки внутри скобок можно раскрыть. Тогда получим


следующее:

При этом если нужно получить короткий ответ, последнее


выражение можно записать в виде (x + 2)2 поскольку
выражение (x + 2)(x + 2) это перемножение двух
сомножителей, каждый из которых равен (x + 2)

Примеры разложений

Пример 1. Разложить на множители следующий квадратный


трёхчлен:

3x2 − 2x − 1
1880

Найдём корни квадратного трёхчлена:

Воспользуемся формулой разложения. В левой части


напишем квадратный трёхчлен 3x2 − 2x − 1, а в правой части
— его разложение в виде a(x − x1)(x − x2), где вместо a, x1 и x2
подстáвим соответствующие значения:

Во вторых скобках можно заменить вычитание сложением:

Пример 2. Разложить на множители следующий квадратный


трёхчлен:

3 − 11x + 6x2

Упорядочим члены так, чтобы старший коэффициент


располагался первым, средний — вторым, свободный член
— третьим:

6x2 − 11x + 3
1881

Найдём корни квадратного трёхчлена:

Воспользуемся формулой разложения:

Упростим получившееся разложение. Вынесем за первые


скобки общий множитель 3

Теперь воспользуемся сочетательным законом умножения.


Напомним, что он позволяет перемножать сомножители в
любом порядке. Умножим 3 на вторые скобки. Это позвóлит
избавиться от дроби в этих скобках:

Пример 3. Разложить на множители следующий квадратный


трёхчлен:
1882

3x2 + 7x − 6

Найдём корни квадратного трёхчлена:

Воспользуемся формулой разложения:

Пример 4. Найдите значение k, при котором разложение на


множители трёхчлена 3x2 − 8x + k содержит множитель
(x − 2)

Если разложение содержит множитель (x − 2), то один из


корней квадратного трёхчлена равен 2. Пусть корень 2 это
значение переменной x1

Чтобы найти значение k, нужно знать чему равен второй


корень. Для его определения воспользуемся теоремой
Виета.
1883

В данном случае квадратный трёхчлен не является


приведённым, поэтому сумма его корней будет равна дроби

, а произведение корней — дроби 

Выразим из первого равенства переменную x2 и сразу


подстáвим найденное значение во второе равенство вместо
x2

Теперь из второго равенства выразим k. Так мы найдём его


значение.

Пример 5. Разложить на множители следующий квадратный


трёхчлен:
1884

Перепишем данный трёхчлен в удобный для нас вид. Если в


первом члене заменить деление умножением, то получим

. Если поменять местами сомножители, то получится

. То есть коэффициент a станет равным

Коэффициент b можно перевести в обыкновенную дробь. Так


проще будет искать дискриминант:

Найдём корни квадратного трёхчлена:

Воспользуемся формулой разложения:

Задания для самостоятельного решения


Задание 1. Разложить на множители квадратный трёхчлен:

Показать решение
1885

Задание 2. Разложить на множители квадратный трёхчлен:

Показать решение
Задание 3. Разложить на множители квадратный трёхчлен:

Показать решение
Задание 4. Разложить на множители квадратный трёхчлен:

Показать решение
Задание 5. Разложить на множители квадратный трёхчлен:

Показать решение
Задание 6. Разложить на множители квадратный трёхчлен:

Показать решение
Задание 7. Разложить на множители квадратный трёхчлен:

Показать решение
Задание 8. Разложить на множители квадратный трёхчлен:

Показать решение
Задание 9. Разложить на множители квадратный трёхчлен:

Показать решение
Задание 10. Разложить на множители квадратный трёхчлен:
1886

Показать решение
Задание 11. Разложить на множители квадратный трёхчлен:

Показать решение
Задание 12. Разложить на множители квадратный трёхчлен:

Показать решение
1887

Обобщённое понятие модуля числа

В данном уроке мы рассмотрим понятие модуля числа более


подробно.

Предварительные навыки

 Отрицательные числа

 Модуль числа

 Общие сведения о неравенствах

Содержание урока

 Что такое модуль?


 Раскрытие модуля
 Преобразование выражений с модулями
 Задания для самостоятельного решения

Что такое модуль?

Модуль — это расстояние от начала координат до какого-


нибудь числа на координатной прямой. Поскольку
расстояние не бывает отрицательным, то и модуль всегда
неотрицателен. Так, модуль числа 3 равен 3, как и модуль
числа −3 равен 3

| 3 |= 3

|−3|= 3

Предстáвим, что на координатной прямой расстояние между


целыми числами равно одному шагу. Теперь если отметить
1888

числа −3 и 3, то расстояние до них от начала координат


будет одинаково равно трём шагам:

Модуль это не только расстояние от начала координат до


какого-нибудь числа. Модуль это также расстояние между
любыми двумя числами на координатной прямой. Такое
расстояние выражается в виде разности между этими
числами, заключенной под знак модуля:

|x1 − x2|

Где x1 и x2 — числа на координатной прямой.

Например, отметим на координатной прямой числа 2 и 5.

Расстояние между числами 2 и 5 можно записать с помощью


модуля. Для этого запишем разность из чисел 2 и 5 и
заключим эту разность под знак модуля:

|2 − 5| = |−3| = 3

Видим, что расстояние от числа 2 до числа 5 равно трём


шагам:
1889

Если расстояние от 2 до 5 равно 3, то и расстояние от 5 до 2


тоже равно 3

То есть, если в выражении |5 − 2| поменять числа местами,


то результат не изменится:

|5 − 2| = | 3 | = 3

Тогда можно записать, что |2 − 5| = |5 − 2|. Вообще,


справедливо следующее равенство:

|x1 − x2| = |x2 − x1|

Это равенство можно прочитать так: Расстояние от x1 до x2


равно расстоянию от x2 до x1.

Раскрытие модуля

Когда мы говорим, что |3|= 3 или |−3|= 3 мы выполняем


действие называемое раскрытием модуля.

Правило раскрытия модуля выглядит так:


1890

Такую запись мы ранее не использовали. Дело в том, что


равенство можно задавать несколькими формулами.
Фигурная скобка указывает, что возможны два случая в
зависимости от условия. В данном случае условиями
являются записи «если x ≥ 0» и «если x < 0».

В зависимости от того что будет подставлено вместо x,


выражение |x| будет равно x, если подставленное число
больше или равно нулю. А если вместо x подставлено число
меньшее нуля, то выражение |x| будет равно −x.

Второй случай на первый взгляд может показаться


противоречивым, поскольку запись |x| = −x выглядит будто
модуль стал равен отрицательному числу. Следует иметь
ввиду, что когда x < 0, то под знáком модуля располагается
отрицательное число. После знака равенства нужно
подстáвить данное отрицательное число вместо x и раскрыть
скобки.

Например, найдём модуль числа −7, используя правило


раскрытия модуля:

Итак, x = −7

|−7|

В данном случае выполняется второе условие x < 0, ведь


−7 < 0
1891

Поэтому используем вторую формулу. А именно |x| = −x.


Подстáвим вместо x число −7

Отсюда:

Поэтому |−7| = 7.

Пример 2. Пусть x = 5. То есть мы рассматриваем модуль


числа 5

|5|

В данном случае выполняется первое условие x ≥ 0, ведь


5 ≥ 0

Поэтому используем первую формулу. А именно | x | = x.


Получаем | 5 | = 5.

Пример 3. Пусть x = √4 − 6. То есть мы рассматриваем


модуль выражения √4 − 6,

|√4 − 6|

Корень из числа 4 равен 2. Тогда модуль примет вид

|√4 − 6| = |2 − 6| = |−4|


1892

x который был равен √4−6 теперь стал равен −4. В данном


случае выполняется второе условие x < 0, ведь −4 < 0

Следовательно, используем вторую формулу |x| = −x.


Продолжаем решение в исходном примере:

|√4 − 6| = |2 − 6| = |−4| = −(−4) = 4

На практике обычно рассуждают так:

«Модуль раскрывается со знаком плюс, если подмодульное


выражение больше или равно нулю; модуль раскрывается
со знаком минус, если подмодульное выражение меньше
нуля».

Примеры:

|2| = 2 — модуль раскрылся со знаком плюс, поскольку 2 ≥ 0

|−4| = −(−4) = 4 — модуль раскрылся со знаком минус,


поскольку −4 < 0

В некоторых учебниках можно встретить следующую запись


правила раскрытия модуля:

В этой записи первое условие которое ранее выглядело как


x ≥ 0 расписано подробнее, а именно сказано что если x > 0,
1893

то выражение |x| будет равно x, а если x=0, то выражение |x|


будет равно нулю.

Пример 4. Пусть x = 0. То есть мы рассматриваем модуль


нуля:

|0|

В данном случае выполняется условие x=0, ведь 0 = 0

Отсюда: |0| = 0

Пример 5. Раскрыть модуль в выражении |x|+ 3

Если x ≥ 0, то модуль раскроется со знаком плюс, и тогда


исходное выражение примет вид x + 3.

Если x < 0, то модуль раскроется со знаком минус, и тогда


исходное выражение примет вид −x + 3. Чтобы сделать это
выражение более удобным для восприятия, поменяем
местами его члены, полýчим 3 − x

Теперь запишем решение так:

Проверим это решение при произвольных значениях x.


1894

Допустим, требуется найти значение выражения |x|+ 3 при


x = 5. Поскольку 5 ≥ 0, то модуль, содержащийся в
выражении |x|+ 3 раскрóется со знаком плюс и тогда решение
примет вид:

|x|+ 3 = x + 3 = 5 + 3 = 8

Найдём значение выражения |x|+ 3 при x = −6. Поскольку


−6 < 0, то модуль содержащийся в выражении |x|+ 3
раскроется со знаком минус и тогда решение примет вид:

|x| + 3 = 3 − x = 3 − (−6) = 9

Пример 6. Раскрыть модуль в выражении x +|x + 3|

Если x + 3 ≥ 0, то модуль |x + 3| раскроется со знаком плюс и


тогда исходное выражение примет вид x + x + 3, откуда 2x +
3.

Если x + 3 < 0, то модуль |x + 3| раскроется со знаком минус и


тогда исходное выражение примет вид x − (x + 3), откуда x −
x − 3 = −3.

Запишем решение так:

Заметим, что условия x + 3 ≥ 0 и x + 3 < 0 являются


неравенствами. Их можно привести к более простому виду,
решив их:
1895

Тогда условия из решения можно заменить на равносильные


x ≥ −3 и x < −3

Во втором случае когда x строго меньше −3 выражение x +|x


+ 3| всегда будет равно постоянному числу −3.

Например, найдём значение выражения x +|x + 3| при x = −5.


Поскольку −5 < −3, то согласно нашему решению значение
выражения x +|x + 3| будет равно −3

При x = −5,
x +|x + 3| = x − x − 3 = −5 − (−5) − 3 = −3

Найдём значение выражения x +|x + 3| при x = 4. Поскольку 4


≥ −3, то согласно нашему решению модуль выражения x +|
x + 3| раскрывается со знаком плюс, и тогда исходное
выражение принимает вид 2x+3, откуда подставив 4 получим
11

При x = 4,
x +|x + 3| = 2x+3 = 2 × 4 + 3 = 8 + 3 = 11

Найдём значение выражения x +|x + 3| при x=−3.

Поскольку −3 ≥ −3, то согласно нашему решению модуль


выражения x +|x + 3| раскрывается со знаком плюс, и тогда
исходное выражение принимает вид 2x+3, откуда подставив
−3 получим −3

x +|x + 3| = 2x + 3 = 2 × (−3) + 3 = −6 + 3 = −3


1896

Пример 3. Раскрыть модуль в выражении

Как и прежде используем правило раскрытия модуля:

Но это решение не будет правильным, поскольку в первом


случае написано условие x ≥ 0, которое допускает что при

x = 0 знаменатель выражения обращается в ноль, а на


ноль делить нельзя.

В данном примере удобнее использовать подробную запись


правила раскрытия модуля, где отдельно рассматривается
случай при котором x = 0

Перепишем решение так:

В первом случае написано условие x > 0. Тогда выражение

станет равно 1. Например, если x = 3, то числитель и


знаменатель станут равны 3, откуда полýчится 1
1897

И так будет при любом x, бóльшем нуля.

Во втором случае написано условие x = 0. Тогда решений не


будет, потому что на ноль делить нельзя.

В третьем случае написано условие x < 0. Тогда выражение

станет равно −1. Например, если x = −4, то числитель


станет равен 4, а знаменатель −4, откуда полýчится единица
−1

Пример 4. Раскрыть модуль в выражении 

Если x ≥ 0, то модуль, содержащийся в числителе,


раскроется со знаком плюс, и тогда исходное выражение

примет вид , которое при любом x, бóльшем нуля, будет


равно единице:

Если x < 0, то модуль раскроется со знаком минус, и тогда

исходное выражение примет вид


1898

Но надо учитывать, что при x = − 1 знаменатель выражения

обращается в ноль. Поэтому второе условие x < 0


следует дополнить записью о том, какие значения может
принимать x

Преобразование выражений с модулями

Модуль, входящий в выражение, можно рассматривать как


полноценный множитель. Его можно сокращать и выносить
за скобки. Если модуль входит в многочлен, то его можно
сложить с подобным ему модулем.

Как и у обычного буквенного множителя, у модуля есть свой


коэффициент. Например, коэффициентом модуля |x|
является 1, а коэффициентом модуля −|x| является −1.
Коэффициентом модуля 3|x+1| является 3, а коэффициентом
модуля −3|x+1| является −3.

Пример 1. Упростить выражение |x| + 2|x| − 2x + 5y и


раскрыть модуль в получившемся выражении.

Решение

Выражения|x| и 2|x| являются подобными членами. Слóжим


их. Остальное оставим без изменений:

Раскроем модуль в получившемся выражении. Если |x| ≥ 0,


то получим 3x − 2x + 5y, откуда x + 5y.
1899

Если |x| < 0, то получим −3x − 2x + 5y, откуда −5x + 5y.


Вынесем за скобки множитель −5, получим −5(x − y)

В итоге имеем следующее решение:

Пример 2. Раскрыть модуль в выражении: −|x|

Решение

В данном случае перед знаком модуля стоит минус. Его


можно понимать как минус единицу перед знаком модуля.
Если x ≥ 0, то модуль раскроется со знаком плюс, и тогда
исходное выражение примет вид −x

Если x < 0, то модуль раскроется со знаком минус, и тогда


исходное выражение примет вид −(−x) откуда получим
просто x

Задания для самостоятельного решения


Задание 1. Раскройте модуль:

Показать решение
Задание 2. Раскройте модуль:

Показать решение
Задание 3. Раскройте модуль:
1900

Показать решение
Задание 4. Раскройте модуль:

Показать решение
Задание 5. Раскройте модуль:

Показать решение

Вам также может понравиться